Sandoval - Political Law [Pt.1]

March 26, 2018 | Author: Brenner Bengwayan | Category: Territorial Waters, United States Constitution, Politics, International Politics, International Relations


Comments



Description

LEX REVIEWS AND SEMINARS INC; NATIONAL BAR REVIEW CENTERFOUR HUNDRED AND SEVENTY-NINE (479) QUESTIONS AND ANSWERS IN POLITICAL LAW AND PUBLIC INTERNATIONAL LAW (Culled from Significant Laws and Decisions of the Supreme Court) Attorney EDWIN REY SANDOVAL (As of August 25, 2006) PART I A. POLITICAL LAW That branch of public law which deals with the organization and operations of the governmental organs of the State and defines the relations of the State with the inhabitants of its territory. Scope/Divisions of Political Law: 1) Constitutional Law—the study of the maintenance of the proper balance between authority as represented by the three inherent powers of the state and liberty as guaranteed by the Bill of Rights. 2) Administrative Law-- That branch of public law which fixes the organization, determines the competence of administrative authorities who executes the law, and indicates to the individual remedies for the violation of his right. 3) Law on Municipal Corporations 4) Law of Public Officers 5) Elections Law Basis: 1) 1987 Constitution 2) 1973 and 1935 Constitutions 3) Organic laws made to apply to the Philippines— a) Philippine Bill of 1902 b) Jones Law of 1916 c) Tydings-McDuffie Law of 1934 4) Statutes, executive orders and decrees, and judicial decisions 5) US Constitution Constitution legislation direct from the people states general principles; intended not merely to meet existing conditions; it is the fundamental law of the State Statute legislation from the people‘s representative; provides the details of the subject matter of which it treats; intended primarily to meet existing conditions only; it conforms to the Constitution PHILIPPINE CONSTITUTION Constitution—it is the document, which serves as the fundamental law of the State; that body of rules and maxims in accordance with which the power of sovereignty are habitually exercised. That written instrument enacted by direct action of the people by which the fundamental powers of the government are established, limited and defined, and by which those powers are distributed among the several departments for their safe and useful exercise for the benefit of the body politic. It is the basic and paramount law to which all other laws must conform and to which all persons, including the highest officials of the land, must defer. No act shall be valid, how noble its intention, if it conflicts with the Constitution. The Constitution must ever remain supreme. All must bow to the mandate of this law. Right or wrong, the Constitution must be upheld as long as the sovereign people have not changed it. Classification: 1. Written or unwritten 1 Written one whose precepts are embodied in one document or set of documents -consists of rules which have not been Unwritten integrated into a single, concrete form but are scattered in various sources Examples: a. statutes of fundamentalcharacter; b. judicial decisions; c. commentaries of publicists; d. customs and traditions; e. certain common law principles 2. Enacted (conventional) or Evolved (Cumulative) Enacted (conventional) formally struck off at a definite time and place following a conscious or deliberate effort taken by a constituent body or ruler Evolved (Cumulative) the result of political evolution, not inaugurated at any specific time but changing by accretion rather than by any systematic method 3. Rigid Flexible Rigid one that can be amended only by a formal and usually difficult process Flexible one that can be changed by ordinary legislation - The Philippine Constitution is written, conventional and rigid. It is embodied in one document and can be amended only by a formal and usually difficult process. Interpretation: 1) Verba Legis—whenever possible, the words used in the Constitution must be given their ordinary meaning except where technical terms are employed. 2) When there is Ambiguity—ratio legis et anima--A doubtful provision shall be examined in the light of the history of the times and the conditions and circumstances under which the Constitution was framed. (Civil Liberties Union vs. Executive Secretary, 194 SCRA 317) 3) Ut magis valeat quam pereat—the Constitution has to be interpreted as a whole. (Francisco vs. HR, G.R. No. 160261, November 10, 2003) If the plain meaning of the word is not found to be clear, resort to other aids is available—construe the Constitution from what ―appears upon its face‖. The proper interpretation, therefore, depends more on how it was understood by the people adopting it than in the framers‘ understanding thereof. In case of doubt, the provision should be considered as self-executing; mandatory rather than directory; and prospective rather than retroactive. Self-executing provision—one which is complete in itself and becomes operative without the aid of supplementary or enabling legislation, or that which supplies a sufficient rule by means of which the right it grants may be enjoyed or protected. Essential Qualities of the Written Constitution: 1) Broad; 2) Brief; and 3) Definite. Essential parts of a good written Constitution: a) Constitution of Liberty—sets forth the fundamental civil and political rights of the citizens and imposes limitations on the powers of the government as a means of securing the enjoyment of those rights. e.g. Bill of Rights b) Constitution of Government—outlines the organization of the government, enumerates its powers, lays down certain rules relative to its administration and defines the electorate. e .g. Legislative, Executive and Judicial Departments, Constitutional Commissions c) Constitution of Sovereignty—the provisions pointing out the mode or procedure in accordance with which formal changes in the fundamental law may be brought about. e.g. Art. XVIIAmendments or Revisions Effects of Declaration of Unconstitutionality: 2 Views: a) ORTHODOX VIEW— 2 i. ii. iii. iv. v. vi. an unconstitutional act is not a law; it confers no rights; it imposes no duties; it affords no protection; it creates no office; it is inoperative, as if it had not been passed at all. b) MODERN VIEW—Courts simply refuse to recognize the law and determine the rights of the parties as if the statute had no existence. Certain legal effects of the statute prior to its declaration of unconstitutionality may be recognized. Thus, a public officer who implemented an unconstitutional law prior to the declaration of unconstitutionality cannot be held liable (Ynot vs. IAC). Partial Unconstitutionality Requisites: a) The legislature must be willing to retain the valid portion(s), usually shown by the presence of a separability clause in the law—INTENT OF THE LEGISLATIVE; and b) The valid portion can stand independently as law—INDEPENDENCE OF THE PROVISIONS. Distinguish sovereignty from dominion. Held: Sovereignty is the right to exercise the functions of a State to the exclusion of any other State. It is often referred to as the power of imperium, which is defined as the government authority possessed by the State. On the other hand, dominion, or dominium, is the capacity of the State to own or acquire property such as lands and natural resources. (Separate Opinion, Kapunan, J., in Isagani Cruz v. Secretary of DENR, G.R. No. 135385, Dec. 6, 2000, En Banc, See Footnote 86) What is the Doctrine of Constitutional Supremacy? Held: Under the doctrine of constitutional supremacy, if a law or contract violates any norm of the Constitution, that law or contract, whether promulgated by the legislative or by the executive branch or entered into by private persons for private purposes, is null and void and without any force and effect. Thus, since the Constitution is the fundamental, paramount and supreme law of the nation, it is deemed written in every statute and contract. (Manila Prince Hotel v. GSIS, 267 SCRA 408 [1997] [Bellosillo]) What are self-executing and non-self executing provisions of the Constitution? Held: A provision which lays down a general principle, such as those found in Article II of the 1987 Constitution, is usually not self-executing. But a provision which is complete in itself and becomes operative without the aid of supplementary or enabling legislation, or that which supplies sufficient rule by means of which the right it grants may be enjoyed or protected, is self-executing. Thus a constitutional provision is self-executing if the nature and extent of the right conferred and the liability imposed are fixed by the Constitution itself, so that they can be determined by an examination and construction of its terms, and there is no language indicating that the subject is referred to the legislature for action. (Manila Prince Hotel v. GSIS, 267 SCRA 408 [1997] [Bellosillo]) Are provisions of the Constitution self-executing or non-self executing? Why? Held: Unless it is expressly provided that a legislative act is necessary to enforce a constitutional mandate, the presumption now is that all provisions are self-executing. If the constitutional provisions are treated as requiring legislation instead of self-executing, the legislature would have the power to ignore and practically nullify the mandate of the fundamental law. This can be cataclysmic. (Manila Prince Hotel v. GSIS, 267 SCRA 408 [1997] [Bellosillo]) What is the ―Filipino First‖ Policy enshrined in the Constitution? Ans.: In the grant of rights, privileges, and concessions covering the national economy and patrimony, the State shall give preference to qualified Filipinos. (Sec. 10, 2nd par., Art. XII, 1987 Constitution) Is the ―Filipino First‖ Policy expressed in Section 10, Article XII of the Constitution a selfexecuting provision? 3 Held: Yes. It is a mandatory, positive command which is complete in itself and which needs no further guidelines or implementing laws or rules for its enforcement. From its very words the provision does not require any legislation to put it in operation. It is per se judicially enforceable. When our Constitution mandates that [i]n the grant of rights, privileges, and concessions covering the national economy and patrimony, the State shall give preference to qualified Filipinos, it means just that – qualified Filipinos must be preferred. (Manila Prince Hotel v. GSIS, G.R. No. 118295, May 2, 1997, 267 SCRA 408 [Bellosillo]) Give examples of non-self executing provisions of the Constitution. Held: By its very nature, Article II of the Constitution is a ―declaration of principles and state policies.‖ These principles in Article II are not intended to be self-executing principles ready for enforcement through the courts. They are used by the judiciary as aids or as guides in the exercise of its power of judicial review, and by the legislature in its enactment of laws. As held in the leading case of Kilosbayan, Incorporated v. Morato (246 SCRA 540, 564, July 17, 1995), the principles and state policies enumerated in Article II and some sections of Article XII are not ―self-executing provisions, the disregard of which can give rise to a cause of action in courts. They do not embody judicially enforceable constitutional rights but guidelines for legislation.‖ (Tanada v. Angara, 272 SCRA 18 [1997], En Banc [Panganiban]) When are acts of persons considered ―State action‖ covered by the Constitution? Held: In constitutional jurisprudence, the act of persons distinct from the government are considered ―state action‖ covered by the Constitution (1) when the activity it engages in is a ―public function‖; (2) when the government is so significantly involved with the private actor as to make the government responsible for his action; and (3) when the government has approved or authorized the action. (Manila Prince Hotel v. GSIS, 267 SCRA 408 [1997] [Bellosillo]) PREAMBLE WE, THE SOVEREIGN FILIPINO PEOPLE, IMPLORING THE AID OF ALMIGHTY GOD, IN ORDER TO BUILD A JUST AND HUMANE SOCIETY AND ESTABLISH A GOVERNMENT THAT SHALL EMBODY OUR IDEALS AND ASPIRATIONS, PROMOTE THE COMMON GOOD, CONSERVE AND DEVELOP OUR PATRIMONY, AND SECURE TO OURSELVES AND OUR POSTERITY THE BLESSINGS OF INDEPENDENCE AND DEMOCRACY UNDER THE RULE OF LAW AND A REGIME OF TRUTH, JUSTICE, FREEDOM, LOVE, EQUALITY, AND PEACE, DO ORDAIN AND PROMULGATE THIS CONSTITUTION. The Preamble is not a source of power or right for any department of government. It sets down the origin, scope, and purpose of the Constitution. It bears witness to the fact that the Constitution is the manifestation of the sovereign will of the Filipino people. The identification of the Filipino people as the author of the constitution calls attention to an important principle: that the document is not just the work of representatives of the people but of the people themselves who put their mark approval by ratifying it in a plebiscite. 1) It does not confer rights nor impose duties. 2) Indicates authorship of the Constitution; enumerates the primary aims and aspirations of the framers; and serves as an aid in the construction of the Constitution. ARTICLE I NATIONAL TERRITORY The national territory comprises the Philippine archipelago, with all the islands and waters embraced therein, and all other territories over which the Philippines has sovereignty or jurisdiction, consisting of its terrestrial, fluvial and aerial domains, including its territorial sea, the seabed, the subsoil, the insular shelves, and other submarines areas. The waters around, between and connecting the islands of the archipelago, regardless of their breadth and dimensions, form part of the internal waters of the Philippines.‖ Two (2) Parts of the National Territory: 1) The Philippine archipelago with all the islands and waters embraced therein; and 2) All other territories over which the Philippines has sovereignty or jurisdiction. Do you consider the Spratlys Group of Islands as part of Philippine Archipelago? _Spratlys Group of Islands is not part of the Philippine Archipelago because it is too far away from the three main islands of the Philippines. It is found, geographically, almost in the middle of the South China Sea. It is not part of the Philippine Archipelago. Historically, when we talk about Philippine Archipelago, we refer to those islands and waters that were ceded by the Spain to the United States by virtue of Treaty of Paris in 1898. And that did not include the Spratlys Group of Islands yet. 4 Under the treaty, the islands that were ceded by Spain were identified—the main islands—Luzon, Visayas and Mindanao. Clearly, it did not include the Spratlys Group of Islands. Spratlys Group of Islands was only discovered sometime in the 1950‘s by a Filipino, Tomas Cloma. The latter waived his rights over the islands in favor of the Philippine Government. In effect, the government stepped into the shoes of the discoverer. By then President Marcos, what he did the moment Tomas Cloma waived his rights over the Spratlys Group of Islands, is to have the islands immediately occupied by Philippine troops. He then issued PD 1596, constituting the Spratlys Group of Islands as a regular municipality claiming it the Municipality of Kalayaan placing it under the Province of Palawan. And then he had the elections immediately held in the islands so from that time on until now, we continue to hold elections there. The Philippine exercises not only jurisdiction but also sovereignty over the Spratlys Group of Islands, yet it is not part of the Philippine Archipelago. Geographically, it is too far away from the Philippine Archipelago. On May 20, 1980, the Philippines registered its claim with the UN Secretariat. The Philippine claim to the islands is justified by reason of history, indispensable need, and effective occupation and control. Thus, in accordance with the international law, the Spratlys Group of islands is subject to the sovereignty of the Philippines. Do you consider the Spratlys group of Islands as part of our National Territory? Yes. Article I of the Constitution provides: ―The national territory comprises the Philippine archipelago, x x x, and all other territories over which the Philippines has sovereignty or jurisdiction, x x x.‖ The Spratlys Group of islands falls under the second phrase ―and all other territories over which the Philippines has sovereignty or jurisdiction‖. It is part of our national territory because Philippines exercise sovereignty (through election of public officials) over Spratlys Group of Islands. What was the basis of the Philippines‘ claim over the Spratlys? Through discovery of Tomas Cloma and occupation Modes of acquiring territories: 1) Discovery and Occupation—which are terra nullius (land belonging to no one) Doctrine of Effective Occupation—discovery alone is not enough. Mere discovery gives only an inchoate right to the discoverer. For title to finally vest, discovery must be followed by effective occupation in a reasonable time and attestation of the same. 2) Cession by Treaty. Examples are Treaty of Paris, treaty between France and US ceding Louisiana to the latter and treaty between Russia and US ceding Alaska to the latter; 3) Prescription—which is a concept under the Civil Code. Territory may also be acquired through continuous and uninterrupted possession over a long period of time. However, in international law, there is no rule of thumb as to the length of time for acquisition of territory through prescription. In this connection, consider the Grotius Doctrine of immemorial prescription, which speaks of uninterrupted possession going beyond memory. 4) Conquest or Subjugation (conquistadores)—this is no longer recognized inasmuch as the UN Charter prohibits resort to threat or use of force against the territorial integrity or political independence of any state; and 5) Accretion—another concept in the Civil Code. It is the increase in the land area of the State, either through natural means, or artificially, through human labor. Other territories over which the Philippines has sovereignty or jurisdiction: 1. Batanes—(1935 Constitution); 2. Those contemplated under Article I, 1973 Constitution—belonging to the Philippines by historic right or legal title; PD 1596, June 11, 1978-- constituting the Spratly‘s Group of Islands as a regular municipality claiming it the Municipality of Kalayaan, placing it under the Province of Palawan. ―xxx The waters around, between and connecting the islands of the archipelago, regardless of their breadth and dimensions, form part of the internal waters of the Philippines.‖ This second sentence of Article I is not the Archipelago Doctrine. This is only our restatement/reaffirmation of our adherence to the Archipelago Doctrine simply because we are an archipelago consisting of 7,107 islands. It is essential for our national survival that we adhere to the archipelago principle. Archipelago Doctrine—merely emphasizes the unity of lands and waters. It is a body of waters interconnected with other natural features. Under the United Nation Convention on the Law of Sea (UNCLOS), it consists of drawing imaginary baseline connecting the outermost islands of the archipelago in which all waters, islands is considered as one integrated whole. An archipelago is defined as group of 5 islands, interconnecting waters and other natural features which are so closely interrelated that such islands, waters and natural features form an intrinsic geographical, economical and political entity, or which historically been regarded as such. Correlate this doctrine to right of innocent of passage, right of arrival under stress and UNCLOS requiring the designation of archipelagic seaways so that foreign vessels may pas through an archipelago. Components of National Territory: i. Terrestrial—land mass on which the inhabitants live; ii. Fluvial—maritime; a. Internal or national waters—bodies of water within the land mass, among them are: i. Rivers—which may be: 1. National 2. Boundary—divides the territories of States 3. International—flows thru various States a. Thalweg Doctrine—for boundary rivers, in the absence of an agreement between the riparian states, the boundary line is laid on the middle of the main navigable channel. b. Middle of the Bridge Doctrine—where there is a bridge over a boundary river, the boundary line is the middle or center of the bridge. ii. Bays and gulfs—a bay is a well-marked indentation whose penetration is in such proportion to the width of its mouth as to contain a land-locked waters and constitutes more than a curvature of the coast. Also referred to as juridical bay. The area must be as large as, or larger than, a semicircle whose diameter is a line drawn across the mouth of such indentation, or if the mouth is less than 24 miles wide. e.g. Hudson Bay in Canada, one whose waters are considered internal because of the existence of a historic title. iii. Straits—narrow passageways connecting 2 bodies of water. If the distance between the 2 opposite coast is not more than 6 miles, they are considered internal waters. In international law, when a strait within a country has a width of more than six (6) miles, the center lane in excess of the three (3) miles on both sides is considered international waters. iv. Canals—the most famous is the Suez Canal, which is neutralized, and the Panama Canal, which is open to everyone in times of war or peace. b. Archipelagic waters—are the waters enclosed by the archipelagic baselines, regardless of their depth or distance from the coast. Archipelagic State—a state made up wholly of one or two archipelagos. It may include other islands. Straight Archipelagic Baseline—to determine the archipelagic waters, the state shall draw straight baselines connecting the outermost points of the outermost islands and drying reefs, provided that the ratio of the area of the water to the area of the land, including atolls, is between 1:1 and 9:1. The length of such baselines shall not exceed 100 nautical miles, except up to 3% of the total number of baselines enclosing any archipelago may exceed that length, up to a maximum 125 miles. The baselines drawn should not depart, to any appreciable extent, from the general configuration of the archipelago. All the waters within the baselines shall then be considered internal waters. The breadth of the 12-mile territorial sea, the contiguous zone, the exclusive economic zone and the continental shelf shall then be measured from the archipelagic baselines. Vessels may be allowed innocent passage within the archipelagic waters, but this right may be suspended, after publication, in the interest of international security. The coastal state may also designate archipelagic sea lanes for continuous, unobstructed transit of vessels. c. Territorial Sea—the belt of the sea located between the coast and the internal waters of the coastal state on the other hand, and the high seas on the other, extending up to 12 nautical miles from the low-water mark, or in the case of archipelagic states, from the baselines. Baseline—is a line from which the breadth of the territorial sea, the contiguous zone and the exclusive economic zone is measured in order to determine the maritime boundary of the coastal state. 6 Normal Baseline Method ii. The continental shelf does not form part of the Philippine territory. not territory of any particular State. 6) Separation of powers. Continental shelf—it is the seabed and subsoil of the submarine areas extending beyond the Philippine territorial sea throughout the natural prolongation of the land territory. These are the waters which do not constitute the internal waters. Characteristics of a republican form of government: 1) The people do not govern themselves directly but through their representatives. whichever is the farthest. the mutual interdependence of the three departments of the government. 1. The Philippines has the sovereign rights over the continental shelf for the purpose of exploring it and exploiting its natural resources. Lukban. territorial sea and exclusive economic zones of a state. Freedom of navigation—refers to the right to sail ship on the high sea. It extends up to: i. g. High Seas—treated as res communes.Types of baseline: i. f.thus. 39 Phil 778) 2) Rule of the majority. archipelagic waters. Aerial—this refers to the air space above the land and waters of the State (See Discussions under International Law) ARTICLE II DECLARATION OF PRINCIPLES AND STATE POLICIES Sec. and 2. To punish infringement of the above laws and regulations committed within its territory. To prevent infringement of customs. 2) It is founded upon popular suffrage. Straight Baseline method d. the supreme power resides on the body of people. e. (Plurality in elections) 3) Accountability of public officials 4) Bill of rights 5) Legislature cannot pass irrepealable laws. fiscal immigration or sanitary laws and regulations within its territory or territorial sea. 3) There is the tripartite system of the government. The outer edge of the continental margin. iii. Republicanism What is a republican form of government? It is a government of the people. The coastal state may exercise limited jurisdiction over the contiguous zone: 1. Manifestations: 1) Ours is a government of law and not of men (Villavicencio vs. and for the people. or ii. subject to international law and the laws of the flag of the state. by the people. 7 . They are beyond the jurisdiction and sovereign rights of States. a representative government wherein the powers and duties of government are exercised and discharged for the common good and welfare. Article II The Philippines is a democratic and republican State. (Relate this to Article XI) Essential features: Representation and Renovation. Sovereignty resides in the people and all government authority emanates from them. Exclusive Economic Zone—shall not extend beyond 200 nautical miles from the archipelagic baselines. Contiguous Zone—extends up to 12 nautical miles from the territorial sea. this shall not exceed 24 nautical miles from the archipelagic baselines. thus. A distance of 200 nautical miles from the archipelagic baselines. equality. Inhabitants. the # of which is capable for self-sufficiency and self-defense.STATE—a community of persons. Monte de Piedad. b) Government of Paramount Forces—established and maintained by the military forces who invade and occupy a territory of the enemy in the course of war. Doctrine of Parens Patriae—the government as guardian of the rights of the people may initiate legal actions for and in behalf of particular individual. The first is lodged in the President and the second is vested in Congress while in Parliamentary there is fusion of both executive and legislative powers in Parliament. 8 . more or less numerous. 58 SCRA 94) 4) Sovereignty—It is the right to exercise the functions of a State to the exclusion of any other State. Nation State is a legal or juristic concept while a Nation is an ethnic or racial concept State vs. there is separation of legislative and executive powers. 42 SCRA 23) State vs. Citizens. 3) Government—the agency or instrumentality through which the will of the State is formulated. c) Independent Government—established by the inhabitants of the country who rise in insurrection against the parent State. with all nations. Parliament. expressly or impliedly. which are considered to be automatically part of our own laws. by force or by the voice of the majority. and accountable to. De Jure vs. Electors. as a member of the family of nations. the rightful legal government and maintains itself against the will of the latter. justice. a. Elements of State: 1) People—the inhabitants of the State. Cabañas vs. a) De facto proper—government that gets possession and control of. and possessing a government to which a great body of inhabitants render habitual obedience. freedom. 2) Territory—a fixed portion of the surface of the earth inhabited by the people of the State. (CIR vs. Pilapil. the country is bound by generally accepted principles of international law. By the doctrine of incorporation. cooperation and amity. permanently occupying a definite portion of territory. It embodies interdependence by integration. expressed and realized. b. Government State possesses a government to which a great body of inhabitants render habitual obedience while a Government is merely an instrumentality of the State through which the will of the State is implemented and realized. or usurps. although the actual exercise of the executive powers is vested in a Prime Minister who is chosen by. (Government of the Philippine Islands vs. and adheres to the policy of peace. of both sexes for perpetuity. 2) In Presidential it embodies interdependence by separation and coordination while in Parliamentary. c. it is however subject to restrictions and limitations voluntarily agreed to by the Philippines. Presidential vs. Parliamentary 1) In Presidential. De Facto De Jure has a rightful title but no power or control. Campos Rueda. In its Declaration of Principles and State Policies. 35 SCRA 738. either because the same has been withdrawn from it or because it has not yet actually entered into the exercise thereof while a De Facto Actually exercises the power or control but without legal title. the Constitution adopts the generally accepted principles of international law as part of the law of the land. While sovereignty has traditionally been deemed absolute and all-encompassing on the domestic level. independent of external control. Asuncion. it did not repudiate the Covenant or Declaration in the same way it repudiated the Constitution. maintaining peace and order over it. Characteristics: i. 133-J. L-30389. iii. which may be exercised by the state even if the individual is outside the territory of the State. and defending it against foreign invasion. Secretary of DENR. (Republic vs.―Government of Laws and Not of Men. G. 1982) Effect of Revolutionary Government—it is bound by no constitution. ii. David. No. b) Political—the sum total of all the influences which lie behind the law. L049. vii. As the de jure government. iv. Separate Opinion of Justice Kapunan in Cruz vs. b) Personal—power of the State over its nationals. 135385. G. (Lee Hong Hok vs. vi. c) Extraterritorial—power of the State over persons. 2003) Jurisdiction—is the manifestation of sovereignty. except those of treason. often referred to as Imperium—is the State‘s authority to govern. December 2000) Effect of Belligerent Occupation—there is no change in sovereignty. Principle of Jus Postliminium—at the end of the occupation. Case No. 104768.R. December 27. a) Territorial—power of the State over persons and things within its territory subject to its control and protection. Municipal laws remain in force. Sandiganbayan. Director of Prisons. Kinds of Sovereignty: a) Legal—the power to issue final commands. No. directives and orders issued by government officers did not exceed the authority granted them by the revolutionary government. The directives or orders should not have also violated the Covenant or the Declaration. municipal laws remain in force unless changed by the belligerent occupant. d) External—also known as independence—freedom from external control. No.‖—sovereignty of the people also includes the concept that government officials have only the authority given them by law and defined by law. when the occupant is ousted from the territory. 9 . It is the government authority possessed by the State expressed in the concept of sovereignty.R. and such authority continues only with the consent of the people. things or acts beyond its territorial limits by reason of their effects to its territory. the political laws which have been suspended shall automatically become effective again. the revolutionary government could not escape responsibility for the State‘s good faith compliance with its treaty obligations under international law. 1972. 1945) Effect of Change of Sovereignty—political laws of the former sovereign are abrogated unless they are expressly reenacted by the affirmative act of the new sovereign. During the interregnum when no constitution or Bill of Rights existed. However. July 21. political laws. Dominium—is the capacity of the State to own or acquire property such as lands and natural resources. However. it includes passing laws governing a territory. are suspended. (Peralta vs. Adm. (Macariola vs. c) Internal—the supreme power over everything within its territory. Permanence Exclusiveness Comprehensiveness Absoluteness Indivisibility Inalienability Imprescriptibility Sovereignty. v. May31. No. November 12. However. that the State may not be sued at all? How may consent of the State to be sued given? Held: The rule. they are constantly developing and evolving. is derisively called ―the royal prerogative of dishonesty‖ because it grants the state the prerogative to defeat any legitimate claim against it by simply invoking its non-suability. 227 SCRA 693. not too infrequently. it has been necessary to distinguish them . which could serve as a basis of civil action between the private parties.The Doctrine of State Immunity from Suit Discuss the basis of the doctrine of State immunity from suit. sue the State thereby) or a special law.‖ its clear import then is that the State may at times be sued . the doctrine. the government is deemed to have descended to the level of the other contracting party and to have divested itself of its sovereign immunity. Ruiz (136 SCRA 487). Nov. where the questioned transaction dealt with the improvements on the wharves in the naval installation at Subic Bay. The result is that State immunity now extends only to acts jure imperii. Xxx 10 .‖ reflects nothing less than a recognition of the sovereign character of the State and an express affirmation of the unwritten rule effectively insulating it from the jurisdiction of courts . In this situation. that a continued adherence to the doctrine of non-suability cannot be deplored. In this jurisdiction. The State's consent may be given either expressly or impliedly.e. however. The restrictive application of State immunity is now the rule in the United States. is deemed to have tacitly given its consent to be sued. in any case. This rule is a necessary consequence of the principle of independence and equality of States.. a sovereign is exempt from suit. 3083. NLRC. On the contrary x x x the doctrine only conveys. we held: ―The traditional rule of immunity exempts a State from being sued in the courts of another State without its consent or waiver. We have had occasion to explain in its defense. where the Philippine government ―consents and submits to be sued upon any money claim involving liability arising from contract. 11.between sovereign and governmental acts (jure imperii) and private. NLRC. distinction must still be made between one which is executed in the exercise of its sovereign function and another which is done in its proprietary capacity. for the loss of governmental efficiency and the obstacle to the performance of its multifarious functions would be far greater in severity than the inconvenience that may be caused private parties. which requires that all money claims against the government must first be filed with the Commission on Audit which must act upon it within sixty days. i. 327. As has been aptly observed by Justice Holmes. Rejection of the claim will authorize the claimant to elevate the matter to the Supreme Court on certiorari and. And because the activities of states have multiplied. thus opening itself to a counterclaim or when it enters into a contract. if such fundamental principle is to be abandoned and the availability of judicial remedy is not to be accordingly restricted. 1993 [Vitug]) The rule that when the State enters into a contract with a private individual or entity. Nov. (Department of Agriculture v. the rules of International Law are not petrified. commercial and proprietary acts (jure gestionis).. it is deemed to have descended to the level of that private individual or entity and. is that without any qualification? What is the Restrictive Doctrine of State Immunity from Suit? Held: This rule x x x is not x x x without qualification. Not all contracts entered into by the government operate as a waiver of its non-suability. Held: The basic postulate enshrined in the Constitution that ―[t]he State may not be sued without its consent. 1993 [Vitug]) Is the rule absolute. in effect. It is based on the very essence of sovereignty. 227 SCRA 693. is not really absolute for it does not say that the state may not be sued under any circumstances. therefore.e. Express consent may be made through a general law (i. Commonwealth Act No. as amended by Presidential Decree No. True. (Department of Agriculture v. the United Kingdom and other states in Western Europe. is conceded when the State itself commences litigation. 11. ―the state may not be sued without its consent. 1445 [Sections 49-50]. not because of any formal conception or obsolete theory. but on the logical and practical ground that there can be no legal right as against the authority that makes the law on which the right depends. In United States of America v. on the other hand. express or implied.‖ Implied consent. the general law waiving the immunity of the state from suit is found in Act No. The plaintiffs opposed the motion contending that the government has waived its immunity from suit based on the acts and pronouncements of the President. En Banc [Campos. Lansang v. Public officials are not exempt. should indemnify the heirs and victims of the Mendiola incident? Consequently.]) 1) When the Republic is sued by name. As the demonstration became unruly. farmers and farmworkers marched in Mendiola on their way to Malacanang protesting against the implementation of the Comprehensive Agrarian Reform Program of the government. Held: 1. 1993 [Vitug]) The restrictive application of State immunity is proper only when the proceedings arise out of commercial transactions of the foreign sovereign. this Court has pronounced that an officer cannot shelter himself by the plea that he is a public agent acting under the color of his office when his acts are wholly without authority. In this case the projects are an integral part of the naval base which is devoted to the defense of both the United States and the Philippines. En Banc [Campos. Held: Some instances when a suit against the State is proper are: Republic v. In that rally. Jr. Sandoval. It does not apply where the contracts relate to the exercise of its sovereign functions. they are not utilized for nor dedicated to commercial or business purposes. CA. March 19. 193 SCRA 282 [1991]). Stated differently. 1993. 1993. Hundreds of landless peasants. Has the government waived its immunity from suit in the Mendiola massacre. 23. The doctrine of state immunity from suit applies to complaints filed against public officials for acts done in the performance of their duties. 3) When the suit is on its face against a government officer but the case is such that ultimate liability will belong not to the officer but to the government. Consequently.‖ The next day. Jr. No. 2) When the suit is against an unincorporated government agency. such as appropriation of the amount necessary to pay the damages awarded to the plaintiff. in their personal capacity. this doctrine still found application. in what is now referred to as the infamous ―Mendiola Massacre. 220 SCRA 124. 11. As early as 1954.R. 2 nd Div. therefore. this Court saying that immunity from suit cannot institutionalize irresponsibility and nonaccountability nor grant a privileged status not claimed by any other official of the Republic.]) State instances when a suit against the State is proper. 220 SCRA 124. G. the latter brought an action for damages against the government. [Quisumbing]) 2. (Amado J. 102667. a State may be said to have descended to the level of an individual and can thus be deemed to have tacitly given its consent to be sued only when it enters into business contracts. After investigation. as well as the recommendation of the Task Force to indemnify the victims and/or their heirs. The rule is that the suit must be regarded as one against the State where the satisfaction of the judgment against the public official concerned will require the State itself to perform a positive act. an indignation rally was held where no less than the President herself joined. Neither does it apply where the public official is clearly being sued not in his official capacity but in his personal capacity. Until recently in 1991 (Chavez v. they exceeded their authority. the police and military personnel assigned in the area performed their functions in accordance with law but when later they fired their guns directlty at the demonstrators. she promised to look into the plight of the victims and their heirs and she created a Task Force to investigate the cause of the Mendiola massacre. March 19. Nov. Feb. indisputably a function of the government of the highest order. police and military personnel assigned in the area violently dispersed the rallyists causing deaths and injuries to several demonstrators.(Department of Agriculture v.‖ When is a suit against a public official deemed to be a suit against the State? Discuss. 2000. and. its commercial activities or economic affairs. the Task Force found that although initially. from liability arising from acts committed in bad faith. Sandiganbayan. The Solicitor General filed a motion to dismiss invoking State immunity from suit. 227 SCRA 693. For the government‘s failure to indemnify the victims and their heirs. (Republic v. The rule does not apply where the public official is charged in his official capacity for acts that are unlawful and injurious to the rights of others. the Task Force recommended that the individual police and military officers involved be prosecuted criminally and for the government to indemnify the victims and/or their heirs. NLRC. Sandoval. is the suit filed against the Republic by petitioners in said case really a suit against the State? 11 . although the acts complained of may have been committed while he occupied a public position. then party defendants. this doctrine still found application. the Commission is merely a preliminary venue. this Court saying that immunity from suit cannot institutionalize irresponsibility and nonaccountability nor grant a privileged status not claimed by any other official of the Republic.P. were discharging their official functions when the incident occurred. the State has consented to be sued. the same are not tantamount to the State having waived its immunity from suit. However. Xxx This is not a suit against the State with its consent. Until recently in 1991 (Chavez v. Firstly. days after the incident. Therefore. 880 as there was unnecessary firing by them in dispersing the marchers. This is precisely the essence of it being a fact-finding body. Secondly. It is important to note that A. still it cannot be maintained that such consent was given considering the circumstances obtaining in the instant case. Thirdly. The principle of state immunity from suit does not apply. the case does not qualify as a suit against the State. as in this case. the recommendation made by the Commission regarding indemnification of the heirs of the deceased and the victims of the incident by the government does not in any way mean that liability automatically attaches to the State. petitioners rely on President Aquino‘s speech promising that the government would address the grievances of the rallyists. their functions ceased to be official the moment they exceeded their authority. the ultimate liability does not pertain to the government.‖ In the exercise of its functions.O. does not mean that there was an admission by the State of any liability. such recommendation not having become final and executory. 11 expressly states that the purpose of creating the Commission was to have a body that will conduct an ―investigation of the disorder. Although the military officers and personnel. Although consent to be sued may be given impliedly. and what is relevant to Our discussion reads: ―1. The President‘s act of joining the marchers. this Court has pronounced that an officer cannot shelter himself by the plea that he is a public agent acting under the color of his office when his acts are wholly without authority. it cannot be inferred that the State has admitted any liability. whatever may be the findings of the Commission.‖ In effect. Sandiganbayan.Held: Petitioners x x x advance the argument that the State has impliedly waived its sovereign immunity from suit. In fact to borrow the words of petitioner x x x. While it is true that nothing is better settled than the general rule that a sovereign state and its political subdivisions cannot be sued in the courts except when it has given its consent. By this alone. ―it was an act of solidarity by the government with the people.‖ Moreover. The military and police forces were deployed to ensure that the rally would be peaceful and orderly as well as to guarantee the safety of the very people that they are duty-bound to protect. 11 provides guidelines. Xxx While the Republic in this case is sued by name. Moreover. the same shall only serve as the cause of action in the event that any party decides to litigate his/her claim. much less can it be inferred that it has consented to the suit. Blg. As early as 1954.O. Whatever recommendation it makes cannot in any way bind the State immediately. Based on the Commission findings. and by the heirs and victims to demand indemnification from the government. when the relief demanded by the suit requires no affirmative official action on the part of the State nor the affirmative discharge of any obligation which belongs to the State in its political capacity. it cannot be invoked by both the military officers to release them from any liability. deaths and casualties that took place. A. The Commission is not the end in itself. the members of the police and military crowd dispersal units committed a prohibited act under B. the facts as found by the trial court showed that they fired at the unruly crowd to disperse the latter. It is their considered view that by the recommendation made by the Commission for the government to indemnify the heirs and victims of the Mendiola incident and by the public addresses made by then President Aquino in the aftermath of the killings. there was lack of justification by the government forces in the use of firearms. 193 SCRA 282 [1991]). even though the officers or agents who are made defendants claim to hold or act only by virtue 12 . whatever acts or utterances that then President Aquino may have done or said. Its conclusions regarding the existence of probable cause for the commission of any offense and of the persons probably guilty of the same shall be sufficient compliance with the rules on preliminary investigation and the charges arising therefrom may be filed directly with the proper court. Camarines Norte. Thus. 220 SCRA 124. respondent may not validly invoke the Royal Prerogative of Dishonesty and conveniently hide under the State‘s cloak of invincibility against suit. Justice and equity sternly demand that the State‘s cloak of invincibility against suit be shred in this particular instance. for years. (EPG Construction Co. March 19. that there be fidelity to legal norms on the part of officialdom if the rule of law were to be maintained. Telesforo Ybasco. Respondent‘s argument is misplaced inasmuch as the principle of State immunity finds no application in the case before us.of a title of the state and as its agents and servants. and Theresa Marquez. and allow itself to be an instrument in the perpetration thereof. Court of Appeals (191 SCRA 713 [1990]). [Buena]) Citizenship 1. may be held liable for damages. invoking the constitutional doctrine of Non-suability of the State. is not absolute for it does not say that the state may not be sued under any circumstances. Held: The Philippine law on citizenship adheres to the principle of jus sanguinis. 1934 in Napier Terrace. True enough. therefore. in Amigable v. Telesforo Ybasco. a child follows the nationality or citizenship of the parents regardless of the place of his/her birth. Thereunder. Rosalind Ybasco Lopez was born on May 16. this Court. qualified to run for Governor of her province? Held: Private respondent Rosalind Ybasco Lopez was born on May 16. and give illustrative case. the liability should fall on the named defendants in the lower court. 354 SCRA 566. as opposed to the doctrine of jus soli which determines nationality or citizenship on the basis of place of birth. En Banc [Campos. this Court – as the staunch guardian of the citizens‘ rights and welfare – cannot sanction an injustice so patent on its face. in effect. and that petitioners-contractors be duly compensated – on the basis of quantum meruit – for construction done on the public works housing project. (Citations omitted) Although the Amigable and Ministerio cases generously tackled the issue of the State‘s immunity from suit vis a vis the payment of just compensation for expropriated property. respondent likewise argues that the State may not be sued in the instant case. Aug. This Court has made it quite clear that even a ―high position in the government does not confer a license to persecute or recklessly injure another. an Australian. Western Australia. otherwise known as the Royal Prerogative of Dishonesty. Sandoval. 1934 in Napier Terrace. this Court nonetheless finds the doctrine enunciated in the aforementioned cases applicable to the instant controversy. if not more so.‖ It is just as important. Under these circumstances. Historically. 2001.16. a Filipino citizen and native of Daet. In line with the ruling of this Court in Shauf v. shred the protective shroud which shields the state from suit. considering that the ends of justice would be subverted if we were to uphold. in any case. where both the government and the public have. Vigilar. received and accepted benefits from said housing project and reaped the fruits of petitioners-contractors‘ honest toil and labor. Instead. 1993. and Theresa Marquez. the rule. reiterating our decree in the landmark case of Ministerio v. to the spouses. 9. What citizenship principle do the Philippines adhere to? Explain. Broome. an Australian. considering that this principle yields to certain settled exceptions. COMELEC. in this particular instance. En Banc [Purisima]) 2. Is she a Filipino citizen and. Camarines Norte.]) May the Government validly invoke the doctrine of State immunity from suit if its invocation will serve as an instrument for perpetrating an injustice on a citizen? Held: To our mind. (Valles v. (Republic v.‖ The inescapable conclusion is that the State cannot be held civilly liable for the deaths that followed the incident. v. it would be the apex of injustice and highly inequitable for us to defeat petitioners-contractors‘ right to be duly compensated for actual work performed and services rendered. 2nd Div. Cuenca. To be sure. 2000. Broome. Western Australia. the State‘s immunity from suit. a Filipino citizen and native of Daet. CFI of Cebu that ―the doctrine of governmental immunity from suit cannot serve as an instrument for perpetrating an injustice on a citizen. Jr. to the spouses. this was a year before the 1935 Constitution took into effect and at that time. having been found to have acted beyond the scope of their authority. Mar. what served as the Constitution of the Philippines were the 13 . Incidentally. herein public officials. 337 SCRA 543. then at most. The fact of her being born in Australia is not tantamount to her losing her Philippine citizenship. all inhabitants of the Philippines who were Spanish subjects on April 11. 2000. Article IV. No.‖ Philippine citizenship can never be treated like a commodity that can be claimed when needed and suppressed when convenient. could qualify a person to being a ―natural-born‖ citizen of the Philippines. 1987 Constitution) The date.A. The 1935 Charter only provides that the election should be made ―upon reaching the age of majority. private respondent can also claim Australian citizenship resulting to her possession of dual citizenship. prescribes the procedure that should be followed in order to make a valid election of Philippine citizenship. The span of fourteen (14) years that lapsed from the time that person reached the age of majority until he finally expressed his intention to elect Philippine citizenship is clearly way beyond the contemplation of the requirement of electing ―upon reaching the age of majority. the proper period for electing Philippine citizenship was. (Re: Application for Admission to the Philippine Bar. If Australia follows the principle of jus soli.‖ is defined to include ―those who are citizens of the Philippines from birth without having to perform any act to acquire or perfect their Philippine citizenship.‖ (Section 2. One who is privileged to elect Philippine citizenship has only an inchoate right to such citizenship. Telesforo Ybasco was deemed to be a Philippine citizen. 914. month and year of birth of FPJ appeared to be 20 August 1939 during the regime of the 1935 Constitution. did not last long. four modes of acquiring citizenship – naturalization. So also. 315 [1912]). i. the citizenship of a legitimate child born of a Filipino mother and an alien father followed the citizenship of the father.e. he should avail of the right with fervor. 1879 in Daet. per Roa v. 9. (Valles v. In these decisions. the child elected Philippine citizenship. jus soli and jus sanguinis. These were the Philippine Bill of July 1.principal organic acts by which the United States governed the country. The phrase ―reasonable time‖ has been interpreted to mean that the election should be made within three (3) years from reaching the age of majority. under the Philippine Bill of 1902 and the Jones Law. The signing into law of the 1935 Philippine Constitution has established the principle of jus sanguinis as basis for the acquisition of Philippine citizenship x x x. Section 1(3) of the 1935 Constitution. 625 did not prescribe a time period within which the election of Philippine citizenship should be made. 625 which was enacted pursuant to Section 1(3). In the opinions of the Secretary of Justice on cases involving the validity of election of Philippine citizenship. which were the laws in force at the time of her birth. qualified to run for President? Held: The term ―natural-born citizens. With the adoption of the 1935 Constitution and the reversal of Roa in Tan Chong v. these laws defined who were deemed to be citizens of the Philippine Islands. 1899 and resided therein including their children are deemed to be Philippine citizens. Bar Matter No.A. Secretary of Labor (Supra. herein private respondent Rosalind Ybasco Lopez. based on the pronouncements of the Department of State of the United States Government to the effect that the election should be made within a ―reasonable time‖ after attaining the age of majority. 1902 and the Philippine Autonomy Act of August 29. 337 SCRA 543. Ching. Thus. a fact duly evidenced by a certified true copy of an entry in the Registry of Births. the 1935 Constitution and C. Aug. By virtue of the same laws. having been born to a Filipino father. Collector of Customs (23 Phil. Only two. No. Article IV of the 1935 Constitution. also known as the Jones Law. En Banc [Purisima]) 3. which held that jus soli was never applied in 14 . COMELEC. in turn. Through its history. Is FPJ a natural-born Filipino citizen and. Private respondent‘s father. upon reaching the age of majority. res judicata and jus sanguinis – had been in vogue. was born on January 5. the herein private respondent. Telesforo‘s daughter. jus soli. enthusiasm and promptitude. x Among others. is a Filipino citizen. this dilemma was resolved by basing the time period on the decisions of this Court prior to the effectivity of the 1935 Constitution. Vicente D. Oct. x x Under both organic acts. which confers citizenship by virtue of blood relationship. the principle of jus sanguinis. Rosalind Ybasco Lopez. is likewise a citizen of the Philippines. Does a legitimate child born under the 1935 Constitution of a Filipino mother and an alien father who elected Philippine citizenship fourteen (14) years after attaining the age of majority become a Filipino? Held: Under Article IV. 1. 1999. C.. 1916.‖ The age of majority then commenced upon reaching twenty-one (21) years. Jus soli. therefore. Thus. As such. En Banc [Kapunan]) 4. was subsequently retained under the 1973 and 1987 Constitutions. However. unless. Camarines Norte. Telesforo Ybasco. and 84 years old at the time of his death on 11 September 1954. the birth certificate of FPJ. FPJ was born to them on 20 August 1939. Petitioner. Poe and Bessie Kelley. and a mestiza Espanol mother. Lorenzo Pou. Poe. Xxx 15 . Marta Reyes. in San Carlos. Lorenzo Poe was 84 years old. The parents of FPJ were Allan F. the marriage certificate of Allan F. Xxx Petitioner submits. It would be extremely doubtful if the Records Management and Archives Office would have had complete records of all residents of the Philippines from 1898 to 1902. and the death certificate of Lorenzo Pou are documents of public record in the custody of a public officer. and a Filipino citizen. FPJ evidently being an illegitimate son according the petitioner. Rule 130.the Philippines). unmarried. however. a resident of San Carlos. a Filipino. Poe was Lorenzo Poe. The birth certificate of FPJ. Poe was stated to be twenty-five years old. identified him to be a Filipino. Allan F. Poe and Bessie Kelley. and the birth certificate of FPJ. are prima facie evidence of the facts therein stated. Pangasinan. Poe and Bessie Kelly. Allan F. the death certificate of Lorenzo Pou. twenty-one years old and married. 5. Introduced by petitioner was an ―uncertified‖ copy of a supposed certificate of the alleged marriage of Allan F. The father of Allan F. Pangasinan. and At the time of his death on 11 September 1954. Poe and Bessie Kelley were married to each other on 16 September. of the Rules of Court provides: ―Entries in official records. Pangasinan. 1940. likewise failed to show that Lorenzo Pou was at any other place during the same period. Poe. an American citizen. 2. Poe and Bessie Kelley reflected the date of their marriage to be on 16 September 1940. The certificate of birth of the father of FPJ. however. and an American citizen. or by a person in the performance of a duty specially enjoined by law. at the age of 84 years. Entries in official records made in the performance of his duty by a public officer of the Philippines. Documentary evidence adduced by petitioner would tend to indicate that the earliest established direct ascendant of FPJ was his paternal grandfather Lorenzo Pou. in any case. Allan F. twenty-four years old. and Bessie Kelley to be twenty-two years old. that in establishing filiation (relationship or civil status of the child to the father [or mother]) or paternity (relationship or civil status of the father to the child) of an illegitimate child. Xxx Being public documents. that the place of residence of a person at the time of his death was also his residence before death. In the absence of any evidence to the contrary. Would the above facts be sufficient or insufficient to establish the fact that FPJ is a natural-born citizen? The marriage certificate of Allan F. showed that he was born on 17 May 1915 to an Espanol father. While the records of birth of Lorenzo Pou had not been presented in evidence. would disclose that he was born on 20 August 1939 to Allan F. The documents have been submitted in evidence by both contending parties during the proceedings before the COMELEC. his death certificate. it should be sound to conclude. the mandatory rules under civil law must be used. married to Marta Reyes. the father of Allan F. Poe. It could thus be assumed that Lorenzo Pou was born sometime in the year 1870 when the Philippines was still a colony of Spain. the only conclusions that could be drawn with some degree of certainty from the documents would be that – 1. 3. 4. Poe and Paulita Gomez on 05 July 1936. constitute prima facie proof of their contents. or at least to presume. jus sanguinis or blood relationship would now become the primary basis of citizenship by birth. married to Bessie Kelley. Section 44. unmarried. the residence of Lorenzo Pou was stated to be San Carlos. In his death certificate. The marriage certificate of Allan F. Considering the reservations made by the parties on the veracity of some of entries on the birth certificate of respondent and the marriage certificate of his parents.‖ Xxx The death certificate of Lorenzo Pou would indicate that he died on 11 September 1954. In the same certificate. Petitioner would argue that Lorenzo Pou was not in the Philippines during the crucial period of from 1898 to 1902 considering that there was no existing record about such fact in the Records Management and Archives Office. he could not have transmitted his citizenship to respondent FPJ. however. 771) and Serra v.e. The issue was whether the stepson followed the naturalization of the stepfather. or on 16 September 1940. citizenship is significant in civil relationships found in different parts of the Civil Code x x x. Xxx Petitioner would have it that even if Allan F. which. the matter about pedigree is not necessarily precluded from being applicable by the Civil Code or Family Code provisions. Vivo (20 SCRA 562. to indicate that the legitimate or illegitimate civil status of the individual would also affect his political rights or. recognizing his own paternal relationship with FPJ. the disquisition made by amicus curiae Joaquin G. an adopted child would be considered the child of his adoptive parents and accorded the same rights as their legitimate child but such legal fiction extended only to define his rights under civil law (See Ching Leng v. might be accepted to prove the acts of Allan F.. Birth to unmarried parents would make FPJ an illegitimate child. Such distinction. Bessie Kelley. provisions on ―citizenship‖ could be found in the Civil Code. Republic (91 Phil. his relationship to the State. The case was not about an illegitimate child of a Filipino father. Poe were a Filipino citizen. X x x But the documentary evidence introduced by no less than respondent himself. and the invidious discrimination survived when the Spanish Civil Code became the primary source of our own Civil Code. for instance. such provisions must be taken in the context of private relations. a stepson who was the child of a Chinese mother and a Chinese father. But if the pronouncement was irrelevant to the lis mota. in general. the pronouncement would not be a decision but a mere obiter dictum which did not establish doctrine. sister of Bessie Kelley Poe submitted as Exhibit 20 before the COMELEC. Xxx These distinctions between legitimacy and illegitimacy were codified in the Spanish Civil Code. The ordinary rules on evidence could well and should govern. the pronouncement would be a decision constituting doctrine under the rule of stare decisis. FPJ so followed the citizenship of his mother. SJ. On the above score. although good law. provided distinctions in the rights of legitimate and illegitimate children. While. Xxx Thus. and as one family x x x. the domain of civil law x x x. The provisions are intended merely to govern the private and personal affairs of the family. do not have preclusive effects on matters alien to personal and family relations. remains and should remain only in the sphere of civil law and not unduly impede or impinge on the domain of political law. i. Paa v. It was about a stepson of a Filipino. Galang. This discriminatory attitude may be traced to the Spanish family and property laws. Morano v. 914. de Leon (82 Phil. I therefore invite the Court to look closely into these cases. if any. consisting of a birth certificate of respondent and a marriage certificate of his parents showed that FPJ was born on 20 August 1939 to a Filipino father and an American mother who were married to each other a year later. 16 . 21 SCRA 753). Similarly. In adoption. basing his stand on the ruling of this Court in Morano v. indeed. The Civil Code or Family Code provisions on proof of filiation or paternity. the latter being an illegitimate child. the duly notarized declaration made by Ruby Kelley Mangahas. If the pronouncement of the Court on jus sanguinis was on the lis mota. is most convincing. Civil law provisions point to an obvious bias against illegitimacy. Bernas. The stepson did not have the blood of the naturalized father. unreported). There is little. For instance. The relevance of ―citizenship‖ or ―nationality‖ to Civil Law is best exemplified in Article 15 of the Civil Code x x x that explains the need to incorporate in the code a reiteration of the Constitutional provisions on citizenship. October 1958. Petitioner contended that as an illegitimate child. citing Chiongbian v. L-11931. The proof of filiation or paternity for purposes of determining his citizenship status should be deemed independent from and not inextricably tied up with that prescribed for civil law purposes.It should be apparent that the growing trend to liberalize the acknowledgment or recognition of illegitimate children is an attempt to break away from the traditional idea of keeping well apart legitimate and non-legitimate relationships within the family in favor of the greater interest and welfare of the child. he states – ―We must analyze these cases and ask what the lis mota was in each of them. Chan. while defining proprietary and successional rights of members of the family. Poe. Nothing about jus sanguinis there. Vivo. unreported) and not his political status. an American citizen. living together with Bessie Kelley and his children (including respondent FPJ) in one house. ―First. The case was about the citizenship of Quintin Chan who was the son of Leoncio Chan. Mr. 161434. It was to ensure a Filipino nationality for the illegitimate child of an alien father in line with the assumption that the mother had custody. Where jurisprudence regarded an illegitimate child as taking after the citizenship of its mother. The Court said obiter that even if Leoncio. it would make an illegitimate distinction between a legitimate child and an illegitimate child. ―Third. The fact of the matter – perhaps the most significant consideration – is that the 1935 Constitution. Justice Mendoza (a former member of this Court). But the Supreme Court said that there was no valid proof that Leoncio was in fact the son of a Filipina mother. This case was not about the illegitimate son of a Filipino father.‖ The other amici curiae. Cayat (68 Phil. Republic. I would grant that the distinction between legitimate children and illegitimate children rests on real differences. The issue was whether one who was already a Filipino because of his mother who still needed to be naturalized. unfortunately hinging solely on pure obiter dicta. based on a contrary to fact assumption. If Leoncio was not Filipino. Leoncio. En Banc [Vitug]) 17 . 2004. The thesis of petitioner. But instead it followed with an obiter dictum. (Maria Jeanette Tecson. G. It was about a legitimate son of a father who had become Filipino by election to public office before the 1935 Constitution pursuant to Article IV. Xxx ―Aside from the fact that such a pronouncement would have no textual foundation in the Constitution. No one was illegitimate here. This statement about Quintin. Quintin would not be Filipino because Quintin was illegitimate. Why deprive the child of the fullness of political rights for no fault of his own? To disqualify an illegitimate child from holding an important public office is to punish him for the indiscretion of his parents. were Filipino. simply repeating the obiter dictum in Morano v. et al. de Leon. Real differences may justify distinction for one purpose but not for another purpose. should indeed fail. Quintin Chan claimed that his father.‖ There utterly is no cogent justification to prescribe conditions or distinctions where there clearly are none provided. Chiongbian v. And if there is neither justice nor rationality in the distinction. 12). Serra v. then the distinction transgresses the equal protection clause and must be reprobated. Quintin therefore was not only not a natural-born Filipino but was not even a Filipino. It was to help the child. X x x It was obiter dictum. neither was his son Quintin. It was not the fault of the child that his parents had illicit liaison.R. The Court therefore concluded that Leoncio was not Filipino. X x x But real differences alone do not justify invidious distinction. Quintin therefore argued that he got his citizenship from Leoncio. it did so for the benefit of the child. Professor Ruben Balane and Dean Merlin Magallona. it would also violate the equal protection clause of the Constitution not once but twice. Section 1(2) of the 1935 Constitution. First. There is nothing there about invidious jus sanguinis. ―The Court should have stopped there. would exercise parental authority and had the duty to support her illegitimate child. There is neither justice nor rationality in that. Providing neither conditions nor distinctions. This is a more complicated case. not to prejudice or discriminate against him. v. Chan (21 SCRA 753). Serra was an illegitimate child of a Chinese father and a Filipino mother. the Constitution states that among the citizens of the Philippines are ―those whose fathers are citizens of the Philippines. ―x x x What is the relevance of legitimacy or illegitimacy to elective public service? What possible state interest can there be for disqualifying an illegitimate child from becoming a public officer. month and year of birth of respondent FPJ. March 3. it would make an illegitimate distinction between the illegitimate child of a Filipino father and the illegitimate child of a Filipino mother. The case was not about the illegitimate son of a Filipino father. ―Finally. Paa v. No. Quintin‘s father. at bottom. his father. the fundamental law prevailing on the day. and second. pure and simple. was the illegitimate son of a Chinese father and a Filipino mother. have expressed similar views. Vivo.―Second. was absolutely unnecessary for the case. COMELEC. can never be more explicit than it is. ―The doctrine on constitutionally allowable distinctions was established long ago by People v. were mere acts of assertion of her Australian citizenship before she effectively renounced the same. COMELEC (185 SCRA 703 [1990]) and in the more recent case of Mercado v. the fact that a child of Filipino parent/s was born in another country has not been included as a ground for losing one‘s Philippine citizenship. HRET. the Court ruled that the mere fact that he is an American did not mean that he is no longer a Filipino. 2001.R. May 26. as well. 530. Thus. 9. To be naturalized. Aug. the mere fact that private respondent Rosalind Ybasco Lopez was a holder of an Australian passport and had an alien certificate of registration are not acts constituting an effective renunciation of citizenship and do not militate against her claim of Filipino citizenship. A person who at the time of his birth is a citizen of a particular country. This issue was put to rest in the case of Aznar v. En Banc [Purisima]) 6. How may Philippine citizenship be renounced? Is the application for an alien certificate of registration. Manzano and COMELEC. HRET. (Antonio Bengson III v. what must an applicant prove? When and what are the conditions before the decision granting Philippine citizenship becomes executory? Held: To be naturalized. and by Republic Act No. As held by this Court in the aforecited case of Aznar. What qualifications must be possessed by an applicant for naturalization? 18 . COMELEC. it was held that the fact that respondent Manzano was registered as an American citizen in the Bureau of Immigration and Deportation and was holding an American passport on April 22. The application of the herein private respondent for an alien certificate of registration. May 7. as in the case of Mercado v. (Antonio Bengson III v. Held: There are two ways of acquiring citizenship: (1) by birth.A. Manzano. No. In the case of Aznar. and the naturalized citizen. private respondent had dual citizenship – she was an Australian and a Filipino. Petitioner‘s contention that the application of private respondent for an alien certificate of registration. naturalized citizens are those who have become Filipino citizens through naturalization. were just assertions of his American nationality before the termination of his American citizenship. These ways of acquiring citizenship correspond to the two kinds of citizens: the natural-born citizen. and her Australian passport. an applicant has to prove that he possesses all the qualifications and none of the disqualifications provided by law to become a Filipino citizen. or (4) committed any act prejudicial to the interest of the nation or contrary to any government announced policies (Section 1. 142840. under Commonwealth Act 63. only a year before he filed a certificate of candidacy for vice-mayor of Makati. tantamount to acts of renunciation of Philippine citizenship? Held: In order that citizenship may be lost by renunciation. Thus. such renunciation must be express. is bereft of merit. at the most. 2001. No. natural-born citizens ―are those citizens of the Philippines from birth without having to perform any act to acquire or perfect his Philippine citizenship. otherwise known as the Revised Naturalization Law. and the possession of foreign passport. May 7. As defined in the x x x Constitution. En Banc [Kapunan]) 8. 337 SCRA 543. the same must be express.R. G. Manzano and COMELEC (G. generally under Commonwealth Act No. and that an application for an alien certificate of registration was not tantamount to renunciation of his Philippine citizenship. Since private respondent did not lose or renounce her Philippine citizenship.‖ On the other hand. Moreover. the applicant has (1) not left the Philippines.R. What are the ways of acquiring citizenship? Discuss. 1999). For renunciation to effectively result in the loss of citizenship. which repealed the former Naturalization Law (Act No. The decision granting Philippine citizenship becomes executory only after two (2) years from its promulgation when the court is satisfied that during the intervening period. an application for an alien certificate of registration does not amount to an express renunciation or repudiation of one‘s citizenship. And. petitioner‘s claim that respondent must go through the process of repatriation does not hold water. (Valles v. No. 530). 307 SCRA 630. 1997. and (2) by naturalization. G. and her holding of an Australian passport. (3) has not been convicted of any offense or violation of government promulgated rules. 2000. 135083. En Banc [Kapunan]) 7.5. 473. is a naturalborn citizen thereof. (2) has dedicated himself to a lawful calling or profession. R. 2927). in Mercado v. 142840. Repatriation. provides the following disqualifications: (a) He must not be opposed to organized government or affiliated with any association or group of persons who uphold and teach doctrines opposing all organized governments. Distinguish naturalization from repatriation. and must have conducted himself in a proper and irreproachable manner during the entire period of his residence in the Philippines in his relation with the constituted government as well as with the community in which he is living. (2) service in the armed forces of the allied forces in World War II (Section 1. May 7. in an application for naturalization? Held: Section 4.R. 2001. On the other hand. on the other hand. No. or assassination for the success and predominance of their ideas. En Banc [Kapunan]) 10. naturalization is governed by Commonwealth Act No. 473. (e) He must be able to speak and write English or Spanish and any of the principal languages. G. En Banc [Kapunan]) 9. mingled socially with the Filipinos. government and civic are taught or prescribed as part of the school curriculum. during the period of his residence in the Philippines (or not less than six months before filing his application). (3) service in the Armed Forces of the United States at any other time (Sec. (Antonio Bengson III v. 473. No. 965 [1953]). Act 473. HRET.Held: Section 2. Act 473. 1. This means that a naturalized Filipino who lost his citizenship will be restored to his prior status as a naturalized Filipino 19 . and (f) He must have enrolled his minor children of school age. HRET. profession. 2630 [1960]). repatriation simply consists of the taking of an oath of allegiance to the Republic of the Philippines and registering said oath in the Local Civil Registry of the place where the person concerned resides or last resided. (b) He must have resided in the Philippines for a continuous period of not less than ten years. (Antonio Bengson III v.R. 142840. personal assault. G. (f) He must have. (4) marriage of a Filipino woman to an alien (Sec. Republic Act No. in any of the public schools or private schools recognized by the Bureau of Private Schools of the Philippines where Philippine history. (e) He must not be suffering from mental alienation or incurable contagious diseases. 142840. traditions and ideals of the Filipinos. or who have not evinced a sincere desire to learn and embrace the customs. (d) He must not have been convicted of any crime involving moral turpitude. (b) He must not be defending or teaching the necessity or propriety of violence. What are the disqualifications under Section 4. 2001. or lawful occupation. 8171 [1995]).A. naturalization as a mode for reacquiring Philippine citizenship is governed by Commonwealth Act No. (d) He must own real estate in the Philippines worth not less than five thousand pesos. (h) He must not be a citizen or subject of a foreign country whose laws do not grant Filipinos the right to become naturalized citizens or subjects thereof. As a mode of initially acquiring Philippine citizenship. 63 (An Act Providing for the Ways in Which Philippine Citizenship May Be Lost or Reacquired [1936]). No. Held: Naturalization is a mode for both acquisition and reacquisition of Philippine citizenship. Philippine currency. during the entire period of the residence in the Philippines required of him prior to the hearing of his petition for naturalization as Philippine citizen. May 7.A. a former Filipino citizen who wishes to reacquire Philippine citizenship must possess certain qualifications and none of the disqualifications mentioned in Section 4 of C. (g) He must not be a citizen or subject of a nation with whom the Philippines is at war. As distinguished from the lengthy process of naturalization. repatriation results in the recovery of the original nationality. 63). Act 473 provides the following qualifications: (a) He must be not less than 21 years of age on the day of the hearing of the petition. Under this law. Republic Act No. Republic Act No. or must have some known lucrative trade. as amended. during the period of such war. and discuss the applicable laws in each. 1. C. Xxx Moreover. (c) He must not be a polygamist or believer in the practice of polygamy. and (5) political and economic necessity (Ibid). may be had under various statutes by those who lost their citizenship due to: (1) desertion of the armed forces (Section 4. (c) He must be of good moral character and believes in the principles underlying the Philippine Constitution. an application for repatriation could be filed with the Special Committee on Naturalization chaired by the Solicitor General with the Undersecretary of Foreign Affairs and the Director of the National Intelligence Coordinating Agency as the other members. Pangasinan in accordance with the aforecited provision. G. Having thus taken the required oath of allegiance to the Republic and having registered the same in the Civil Registry of Mangatarem. Petitioner‘s contention that respondent Cruz is no longer a natural-born citizen since he had to perform an act to regain his citizenship is untenable. upon reaching the age of majority. Before what agency should application for repatriation under R. June 28. (Gerardo Angat v. 2001. 2630 x x x. (Antonio Bengson III v. 1987. COMELEC. On the other hand. No. No. he lost his Filipino citizenship when he rendered service in the Armed Forces of the United States. 1995.A. qualified to run for Congressman? Held: Repatriation results in the recovery of the original nationality. if he was originally a natural-born citizen before he lost his Philippine citizenship. May 7. Antonio Bengson III v. May a natural-born Filipino who became an American citizen still be considered a natural-born Filipino upon his reacquisition of Philippine citizenship and.A. En Banc [Panganiban]. 8171? Held: R. Under the 1973 Constitution definition.citizen. dated June 5. 14. he will be restored to his former status as a natural-born Filipino. Two requisites must concur for a person to be considered as such: (1) a person must be a Filipino citizen from birth and (2) he does not have to perform any act to obtain or perfect his Philippine citizenship. or return to.A. 257 SCRA 727. Aquino‘s Memorandum of March 27. respondent Cruz is deemed to have recovered his original status as a natural-born citizen. is an act providing for the repatriation (a) of Filipino women who have lost their Philippine citizenship by marriage to aliens and (b) of natural-born Filipinos who have lost their Philippine citizenship on account of political or economic necessity. not with the Regional Trial Court. Republic. [T]he term ―natural-born citizen‖ was first defined in Article III. Those born of Filipino mothers before the effectivity of the 1973 Constitution were likewise 20 . there were two categories of Filipino citizens which were not considered natural-born: (1) those who were naturalized and (2) those born before January 17. (Gerardo Angat v. Sept. 725. 132244. Although the agency was deactivated by virtue of President Corazon C. 1999 [Vitug]) 14. G. In respondent Cruz‘s case.A. On the other hand. 8171. 1995. However. elected Philippine citizenship. No. therefore. No. Section 4 of the 1973 Constitution as follows: Section 4. however. if he was originally a natural-born citizen before he lost his Philippine citizenship. G. abrogated.R. 1996. Who may validly avail of repatriation under R. 2001. This means that a naturalized Filipino who lost his citizenship will be restored to his prior status as a naturalized Filipino citizen. It bears stressing that the act of repatriation allows him to recover. 63 enumerates the three modes by which Philippine citizenship may be reacquired by a former citizen: (1) by naturalization. No. Commonwealth Act No. a status which he acquired at birth as the son of a Filipino father. (Frivaldo v. his original status before he lost his Philippine citizenship. HRET. Hence. of Filipino mothers who. Those ―naturalized citizens‖ were not considered natural-born obviously because they were not Filipinos at birth and had to perform an act to acquire Philippine citizenship. 1975. No. A natural-born citizen is one who is a citizen of the Philippines from birth without having to perform any act to acquire or perfect his Philippine citizenship.R. En Banc [Kapunan]) 12. he will be restored to his former status as a natural-born Filipino.R. G. amending C. it was not. 142840. which has lapsed into law on October 23. No. 142840. HRET. The Committee was reactivated on June 8. 14. Sept.D.R. May 7. 132244. he subsequently reacquired Philippine citizenship under R. 1973 (the date of effectivity of the 1973 Constitution) . En Banc [Kapunan]) 11. How may Filipino citizens who lost their citizenship reacquire the same? Answer: Filipino citizens who have lost their citizenship may x x x reacquire the same in the manner provided by law. and (3) by direct act of Congress. the application should be filed with said Agency. Republic. No. No. 63.A 8171 be filed? Held: Under Section 1 of P. (2) by repatriation. 1999 [Vitug]) 13. Held: 1) Those born of Filipino fathers and/or mothers in foreign countries which follow the principle of jus soli. Cite instances when a citizen of the Philippines may possess dual citizenship considering the citizenship clause (Article IV) of the Constitution. 135083. the concern of the Constitutional Commission was not with dual citizens per se but with naturalized citizens who maintain their allegiance to their countries of origin even after their naturalization. 1987 Constitution. on citizenship? Consequently. unless by their act or omission they are deemed to have renounced Philippine citizenship. The present Constitution. upon the filing of their certificate of candidacy. Section 1 hereof shall be deemed natural-born citizens. did not have to undergo the process of naturalization to obtain Philippine citizenship. 1999 [Mendoza]) 17. Dual allegiance. While dual citizenship is involuntary. Article IV. Section 2 of Article IV adds a sentence: ―Those who elect Philippine citizenship in accordance with paragraph (3). May 7. a person is simultaneously considered a national by the said states. As respondent Cruz was not required by law to go through naturalization proceedings in order to reacquire his citizenship. necessarily is a natural-born Filipino.‖ Consequently. What is the main concern of Section 5. May 26. 21 . Unlike those with dual allegiance. Held: Dual citizenship arises when. for candidates with dual citizenship. such a situation may arise when a person whose parents are citizens of a state which adheres to the principle of jus sanguinis is born in a state which follows the doctrine of jus soli. 2) Those born in the Philippines of Filipino mothers and alien fathers if by the laws of their father‘s country such children are citizens of that country. 307 SCRA 630. Distinguish Dual Citizenship from Dual Allegiance. is concurrently considered a citizen of both states. G. he possessed all the necessary qualifications to be elected as member of the House of Representatives.A. The reason therefore is clear: as to such persons. such an individual has not effectively renounced his foreign citizenship. Manzano. i. HRET. 142840. Hence.not considered natural-born because they also had to perform an act to perfect their Philippine citizenship. No. (Antonio Bengson III v. he is perforce a natural-born Filipino. after losing Philippine citizenship. 2001. Noteworthy is the absence in the said enumeration of a separate category for persons who. For instance. the phrase ―dual citizenship‖ in R.R. Section 40(d) (Local Government Code) must be understood as referring to ―dual allegiance. are persons with mere dual citizenship disqualified to run for elective local positions under Section 40(d) of the Local Government Code? Held: In including Section 5 in Article IV on citizenship. refers to a situation in which a person simultaneously owes. Manzano. No. it should suffice if. such candidates at the same time forswear allegiance to the other country of which they are also citizens and thereby terminate their status as dual citizens. dual allegiance is the result of an individual‘s volition.‖ Consequently. as a result of the concurrent application of the different laws of two or more states. Such a person. who must x x x be subject to strict process with respect to the termination of their status.. En Banc [Mendoza]) 16. As such. ipso facto and without any voluntary act on his part. 1999. May 26. only naturalized Filipinos are considered not natural-born citizens.R. on the other hand. 3) Those who marry aliens if by the laws of the latter‘s country the former are considered citizens. they elect Philippine citizenship to terminate their status as persons with dual citizenship considering that their condition is the unavoidable consequence of conflicting laws of different states. No. by some positive act. A citizen who is not a naturalized Filipino. loyalty to two or more states. 7160. It may be that. persons with mere dual citizenship do not fall under this disqualification. (Mercado v. En Banc [Kapunan]) 15. (Mercado v. they would either be natural-born or naturalized depending on the reasons for the loss of their citizenship and the mode prescribed by the applicable law for the reacquisition thereof. from the point of view of the foreign state and of its laws. By electing Philippine citizenship. After defining who are natural-born citizens. now considers those born of Filipino mothers before the effectivity of the 1973 Constitution and who elected Philippine citizenship upon reaching the majority age as natural-born. That is of no moment. however. 307 SCRA 630.e. subsequently reacquire it. G. It is apparent from the enumeration of who are citizens under the present Constitution that there are only two classes of citizens: (1) those who are natural-born and (2) those who are naturalized in accordance with law. As such. Under the LOI.e. Hence. 307 SCRA 630. Commissioner of Immigration (41 SCRA 292 [1971]). In this regard. Neither does it amount to an ―insidious incursion‖ of the military in the task of law enforcement in violation of Section 5[4]. No. Manzano. 1999 [Mendoza]) 18. their duty to provide the necessary equipment to the Marines and render logistical support to these soldiers. i. Aug. Such being the case. 9. which sufficiently provides the metes and bounds of the Marines' authority. In fact. The Court ruled in that case that in order that the doctrine of res judicata may be applied in cases of citizenship. Although the general rule was set forth in the case of Moy Ya Lim Yao.(Mercado v. 2) the Solicitor General or his authorized representative took active part in the resolution thereof. it cannot be properly argued that military authority is supreme over civilian authority. it does not matter whether the AFP Chief actually participates in the Task Force Tulungan since he does not exercise any authority or control over the same. Moreover. Petitioner is correct insofar as the general rule is concerned. 135083. The real authority in these operations. G. Republic (51 SCRA 248 [1973]). the resolution or decision thereon is generally not considered res judicata in any subsequent proceeding challenging the same. and 3) the finding on citizenship is affirmed by this Court. the Metro Manila Police Chief is the overall leader of the PNP-Philippine Marines joint visibility patrols. the PNP. Chief of Staff of the AFP. there can be no appointment to a civilian position to speak of. by his alleged involvement in civilian law enforcement. x x x. Since none of the Marines was incorporated or enlisted as members of the PNP. May 26. to make the effort easier or simpler. (Valles v.R. likewise. and not with the military. has been virtually appointed to a civilian post in derogation of the aforecited provision. He insists that the same issue of citizenship may be threshed out anew.. there can be no ―insidious incursion‖ of the military in civilian affairs nor can there be a violation of the civilian supremacy clause in the Constitution. The calling of the marines in this case constitutes permissible use of military assets for civilian law enforcement. It is noteworthy that the local police forces are the ones in charge of the visibility patrols at all times. In view of the foregoing. Article II of the Constitution? Will this not amount to an "insidious incursion" of the military in the task of law enforcement in violation of Section 5(4). the real authority belonging to the PNP. the following must be present: 1) a person‘s citizenship be raised as a material issue in a controversy where said person is a party. Held: Petitioner maintains further that when citizenship is raised as an issue in judicial or administrative proceedings. as stated in the LOI. COMELEC. It elucidated that reliance may somehow be placed on these antecedent official findings. The general rule is that res ―judicata does‖ not apply in cases hinging on the issue of citizenship? What is the exception to this rule? Discuss. though not really binding. En Banc [Purisima]) Civilian Supremacy Clause 19. Will this not violate the civilian supremacy clause under Section 3. it is not correct to say that General Angelo Reyes. Article XVI of the Constitution. It is. the case did not foreclose the weight of prior rulings on citizenship. the deployment of the Marines in the joint visibility patrols does not destroy the civilian character of the PNP. the police forces are tasked to brief or orient the soldiers on police patrol procedures. in the case of Burca v. citing the case of Moy Ya Lim Yao v. is lodged with the head of a civilian institution. 22 . Considering the above circumstances. Article XVI of the Constitution? Held: The deployment of the Marines does not constitute a breach of the civilian supremacy clause. 337 SCRA 543. The limited participation of the Marines is evident in the provisions of the LOI itself. The President issued Letter of Instruction (LOI) ordering the deployment of members of the Philippine Marines in the metropolis to conduct joint visibility patrols with members of the Philippine National Police in various shopping malls. However. the principle of res judicata generally does not apply in cases hinging on the issue of citizenship. the Marines render nothing more than assistance required in conducting the patrols. 2000. the deployment of the Marines to assist the PNP does not unmake the civilian character of the police force. an exception to this general rule was recognized. It is their responsibility to direct and manage the deployment of the Marines. Enforcement of customs laws. Several fishermen apprehended for violating the ordinances in question challenged their constitutionality contending that the ordinances violated their preferential right as subsistence and marginal fishermen to the use of our communal marine resources guaranteed by the Constitution. What we have here is mutual support and cooperation between the military and civilian authorities. 2. 6. Relief and rescue operations during calamities and disasters. Will you sustain the challenge? Held: The ―preferential right‖ of subsistence or marginal fishermen to the use of marine resources is not absolute. Ronaldo B. are: 1. Conduct of nationwide tests for elementary and high school students. and imposing penalties for violations thereof. Such a right belongs to a different category of rights altogether for it concerns nothing less than self-preservation and self-perpetuation. (Oposa v.R. Composite civilian-military law enforcement activities. in order to stop the illegal practice of cyanide fishing which destroys the corals and other marine resources. 8. 17. No. Anti-drug enforcement activities. Peace and order policy formulation in local government units. 11. it is because of the well-founded fear of its framers that unless the rights to a balanced and healthful ecology and to health are mandated as state policies by the Constitution itself x x x the day would not be too far when all else would be lost not only for the present generation. Article II).. 15. not derogation of civilian supremacy. 16. 14. the advancement of which may even be said to predate all governments and constitutions.It is worth mentioning that military assistance to civilian authorities in various forms persists in Philippine jurisdiction. Conduct of licensure examinations. Jr. pursuant to the first paragraph of Section 2. If they are now explicitly mentioned in the fundamental charter. (IBP v. long pursued to the knowledge of Congress and. Conservation of natural resources. their ―exploration. In accordance with the Regalian Doctrine. 224 SCRA 792 [1993][Davide]) 21. but also for those to come – generations which stand to inherit nothing but parched earth incapable of sustaining life. but also for the 23 . What must be borne in mind is the State policy enshrined in the Constitution regarding the duty of the State to protect and advance the right of the people to a balanced and healthful ecology in accord with the rhythm and harmony of nature (Section 16. 10. exemplifying the activities that bring both the civilian and the military together in a relationship of cooperation. The Province of Palawan and the City of Puerto Princesa enacted ordinances prohibiting the catching and/or exportation of live tropical fishes. it does not follow that it is less important than any of the civil and political rights enumerated in the latter. En Banc [Kapunan]) The Right to a Balanced and Healthful Ecology 20. unbroken. The ordinances in question are meant precisely to protect and conserve our marine resources to the end that their enjoyment may be guaranteed not only for the present generation. under Section 7. Zamora. development and utilization x x x shall be under the full control and supervision of the State. 9. Development of the culture and the arts. 15. Amateur sports promotion and development. Administration of the Philippine National Red Cross. Sanitary inspections. Conduct of census work. Factoran. Administration of the Civil Aeronautics Board. 4. Hon. 2000. The Philippine experience reveals that it is not averse to requesting the assistance of the military in the implementation and execution of certain traditionally ―civil‖ functions. Held: While the right to a balanced and healthful ecology is to be found under the Declaration of Principles and State Policies and not under the Bill of Rights. development and conservation x x x imply certain restrictions on whatever right of enjoyment there may be in favor of anyone. Aug. never before questioned. 13. Implementation of the agrarian reform program. Assistance in installation of weather forecasting devices. marine resources belong to the State. 12. yet. Article XIII. Article XII of the Constitution. executive practice. their mandated protection. 3. Elections. Is the right to a balanced and healthful ecology any less important than any of the civil and political rights enumerated in the Bill of Rights? Explain. 141284. x x x [S]ome of the multifarious activities wherein military aid has been rendered. As a matter of fact. 5. these basic rights need not even be written in the Constitution for they are assumed to exist from the inception of humankind. 7.‖ Moreover. This unquestionably constitutes a gloss on executive power resulting from a systematic. G. and. Such rules and regulations are 24 . or students violating ―School Rules on Discipline. It has a wide sphere of autonomy certainly extending to the choice of students. As may be gleaned from the above provision. G. What are the essential freedoms subsumed in the term ―academic freedom‖? Held: 1. 1997) Academic Freedom 22. (Tano v. Salvador P. Define and discuss the academic freedom of institutions of higher learning. Socrates. the Constitution allows merely the regulation and supervision of educational institutions. Thus. Nov. and Who may be admitted to study. and how best to attain them . its aims and objectives. In this connection. Inc. The right of the school to discipline its students is at once apparent in the third freedom.R. Inc. including setting of academic standards. or a laywoman seeking admission to a seminary.‖ We have thus sanctioned its invocation by a school in rejecting students who are academically delinquent. v. 774-775. 230 SCRA 761. (Miriam College Foundation. Said constitutional provision is not to be construed in a niggardly manner or in a grudging fashion. Court of Appeals. Moreover. 288. v. such power to regulate is subject to the requirement of reasonableness. 99327. How should the State‘s power to regulate educational institutions be exercised? Held: Section 4[1]. 110249. this Court recognizes the expertise of educational institutions in the various fields of learning. 15. 21.‖ but likewise ―who may be admitted to study. Held: Academic freedom of educational institutions has been defined as the right of the school or college to decide for itself. Jr. 2000. ―how it shall be taught. (University of San Agustin. 8. March 7. what may be taught (and) how it shall be taught. Court of Appeals. The essential freedoms subsumed in the term ―academic freedom‖ encompass the freedom to determine for itself on academic grounds: (1) (2) (3) (4) Who may teach. Aug. 348 SCRA 265. they are afforded ample discretion to formulate reasonable rules and regulations in the admission of students.generations to come.‖ A school certainly cannot function in an atmosphere of anarchy. i. 1st Div. When a student commits a serious breach of discipline or failed to maintain the required academic standard. this Court cited with approval the formulation made by Justice Felix Frankfurter of the essential freedoms subsumed in the term ―academic freedom‖ encompassing not only ―the freedom to determine x x x on academic grounds who may teach. 1994 [Nocon]) 24. In Ateneo de Manila University v.e.. Perpetual Help College of Rizal. not the deprivation of their rights. Dec.R.. Article XIV of the Constitution recognizes the State‘s power to regulate educational institutions: The State recognizes the complementary roles of public and private institutions in the educational system and shall exercise reasonable supervision and regulation of all educational institutions.‖ (Isabelo. Within the parameters thereof. The right to a balanced and healthful ecology carries with it a correlative duty to refrain from impairing the environment. What may be taught. En Banc [Vitug]) 2. 27 May 1993). the obligation of the school to educate a student would imply a corresponding obligation on the part of the student to study and obey the rules and regulations of the school . Thus. [Kapunan]) 23. 1993. 227 SCRA 595-597. That would be to frustrate its purpose and nullify its intent. they are competent to determine who are entitled to admission and re-admission. v. How it shall be taught. No. No. he forfeits his contractual right. Capulong (G. since a contract creates reciprocal rights and obligations. Inc.free from outside coercion or interference save possibly when the overriding public welfare calls for some restraint. While it is true that an institution of learning has a contractual obligation to afford its students a fair opportunity to complete the course they seek to pursue. Gov. there can be no doubt that the establishment of an educational institution requires rules and regulations necessary for the maintenance of an orderly educational program and the creation of an educational environment conducive to learning. Moreover. foster love of humanity. In Angeles v. develop moral character and personal discipline. It contemplates neither ―economic seclusion‖ nor ―mendicancy in the international community. teach the rights and duties of citizenship. (UP Board of Regents v. 112 SCRA 26. Faculty Admission Committee. By instilling discipline. the Court upheld the expulsion of students found guilty of hazing by petitioner therein. Inc. nowhere in the above formulation is the right to discipline more evident than in ―who may be admitted to study. the SC pointed out that academic freedom of institutions of higher learning is a freedom granted to ―institutions of higher learning‖ which is thus given a ―wide sphere of authority certainly extending to the choice of students. appreciation of the role of national heroes in the historical development of the country. Sison.‖ Finally. Hon. in Ateneo de Manila v.‖ If such institution of higher learning can decide who can and who cannot study in it. 134625. Court of Appeals and Arokiaswamy William Margaret Celine. This freedom of a university does not terminate upon the ―graduation‖ of a student. for it is precisely the ―graduation‖ of such a student that is in question.equally necessary for the protection of the students. and property (Angeles v. 1999. faculty. Dec. the right to discipline the student likewise finds basis in the freedom ―what to teach. Court of Appeals. respect for human rights. 37 [1982]).‖ 25 . the school has an interest in teaching the student discipline. More importantly.R. Article XIV. logic dictates that it also has the right to determine whom to exclude or expel. strengthen ethical and spiritual values. Loyola School of Theology (68 SCRA 277 [1975]). Where it is shown that the conferment of an honor or distinction was obtained through fraud. This we would be most loathe to do. Does the Constitutional policy of a ―self-reliant and independent national economy‖ rule out foreign competition? Held: The constitutional policy of a ―self-reliant and independent national economy‖ does not necessarily rule out the entry of foreign investments. as well as upon whom to impose lesser sanctions such as suspension and the withholding of graduation privileges. (Miriam College Foundation. the school teaches discipline. 2000. the school not only has the right but the duty to develop discipline in its students. a necessary. 348 SCRA 265. G. 1973 and the present 1987 Constitution .‖ If a school has the freedom to determine whom to admit. if not indispensable. holding that: No one can be so myopic as to doubt that the immediate reinstatement of respondent students who have been investigated and found guilty by the Disciplinary Board to have violated petitioner university‘s disciplinary rules and standards will certainly undermine the authority of the administration of the school. v. [All educational institutions] shall inculcate patriotism and nationalism. goods and services. May a university validly revoke a degree or honor it has conferred to a student after the graduation of the latter after finding that such degree or honor was obtained through fraud? Held: In Garcia v. we also said that discipline was a means for the school to carry out its responsibility to help its students ―grow and develop into mature. Constitution). encourage critical and creative thinking. The Constitution no less imposes such duty. 1 st Div. and promote vocational efficiency (Section 3[2]. No. Sison. responsible. broaden scientific and technological knowledge. Thus. value in any field of learning. Accordingly. effective and worthy citizens of the community. [Kapunan]) 25. a university has the right to revoke or withdraw the honor or distinction it has thus conferred. Capulong (222 SCRA 643 [1993]). it will seriously impair petitioner university‘s academic freedom which has been enshrined in the 1935.‖ Incidentally. [Mendoza]) Economic Policy 26. 15. 2 nd Div. 31. Aug. it certainly can also determine on whom it can confer the honor and distinction of being its graduates. D. 20. 11. when the APT [Asset Privatization Trust] drafted the ASBR [Asset Specific Bidding Rules] sometime in 1993. 391 by E. No. Consequently.‖ x x x However. Chapter 5. Blg.P. The express repeal of B. 146. The participation of foreign investors in the governing body of any public utility enterprise shall be limited to their proportionate share in its capital. a joint venture that would engage in the business of operating a public utility. Section 10 of the Constitution? Held: Petitioner asserts that a shipyard is a public utility pursuant to Section 13 (b) of Commonwealth Act No. 391. Consequently. Angara. No. contend that shipyards are no longer public utilities by express provision of Presidential Decree No. x x x.D. a public utility and. Article XIV. declassifying the shipbuilding and ship repair industry as a public utility.Aside from envisioning a trade policy based on ―equality and reciprocity. No franchise. a joint venture is governed by the laws on contracts and on partnership. (JG Summit Holdings. nor shall such franchise. 1977 manifests the intention of the parties to abide by the constitutional mandate on capitalization of public utilities.‖ thereby demonstrating a clear policy against a sheltered domestic trade environment. certificate. of Kobe. the following provision of the Article XII of the Constitution applies: ―Sec. P. the law first repealed shall not be thereby revived unless expressly so provided (Administrative Code of 1987.D. The reason is simple: said decree was already inexistent when the ASBR was issued.D. 272 SCRA 18 [1997]) 27. which provided incentives to the shipbuilding and ship repair industry.‖ The progenitor of this constitutional provision. 666. While it is true that the repeal of a statute does not operate to impair rights that have become vested or accrued while the statute was in force.‖ the fundamental law encourages industries that are ―competitive in both domestic and foreign markets. on the other hand. such as a shipyard. the Investment Incentive Policy Act of 1983. and all the executive and managing officers of such corporation or association shall be citizens of the Philippines. 666. Ltd. Executive Order No.O. Respondents. must observe the proportion of 60%-40% Filipino-foreign capitalization. v. Neither shall any such franchise or right be granted except under the condition that it shall be subject to amendment. Court of Appeals. 666 was expressly repealed by Section 20 of Batas Pambansa Blg. The JVA [Joint Venture Agreement] between NIDC [National Investment and Development Corporation] and Kawasaki [Kawasaki Heavy Industries. Blg. Book I. certificate. No. Enumerate the Constitutional provisions recognizing and protecting the rights and interests of the indigenous peoples. as said executive order did not provide otherwise. No. there are no vested rights of the parties that should be protected in the case at bar. The State shall encourage equity participation in public utilities by the general public. the Omnibus Investments Code of 1987. No. required the same proportion of 60%-40% capitalization. 2000. Nov. 226 did not revive Section 1 of P. 226. Considered more of a partnership. or any other form of authorization for the operation of a public utility shall be granted except to citizens of the Philippines or to corporations or associations organized under the laws of the Philippines at least sixty per centum of whose capital is owned by such citizens. A shipyard such as PHILSECO being a public utility as provided by law.P. alteration. Section 1 of P. Section 5 of the 1973 Constitution. 391. Is PHILSECO (Philippine Shipyard and Engineering Corporation). Section 21). (Tanada v. x x x A joint venture is an association of persons or companies jointly undertaking some commercial enterprise with all of them generally contributing assets and sharing risks. or repeal by the Congress when the common good so requires. 1975 explicitly stated that a ―shipyard‖ was not a ―public utility. 345 SCRA 143. could be operated only by a corporation at least 60% of whose capital is owned by Filipino citizens in accordance with Article XII. 26 . 666 dated March 5. as a shipyard. Subsequently. Inc. but one in favor of the gradual development of robust industries that can compete with the best in the foreign markets. P. 1st Div. or authorization be exclusive in character or for a longer period than fifty years. was issued and Section 85 thereof expressly repealed B. 666 no longer existed in our statute books. Japan] entered into on January 27. The joint venture created between NIDC and Kawasaki falls within the purview of an ―association‖ pursuant to Section 5 of Article XIV of the 1973 Constitution and Section 11 of Article XII of the 1987 Constitution. Indeed. hence. [Ynares-Santiago]) The Rights of Indigenous Cultural Communities/Indigenous Peoples 28. When a law which expressly repeals a prior law is itself repealed. Appropriating Funds Therefor." It is simply known as "The Indigenous Peoples Rights Act of 1997" or the IPRA. 8371). It grants these people the ownership and possession of their ancestral domains and ancestral lands. Xxx Within their ancestral domains and ancestral lands.R. homestead rights of small settlers. 135385. whenever applicable in accordance with law. et al. use and disposition of property and its increments. economic and political inequalities.) Section 17. The Congress may provide for the applicability of customary laws governing property rights and relations in determining the ownership and extent of ancestral domains. J. Held: Republic Act No. ownership. and protect the rights of cultural communities to preserve and develop their cultures." 27 . G. Discuss the Indigenous Peoples Rights Act (R. (Separate Opinion. traditions. 6.Held: The framers of the 1987 Constitution. The State. Science. The State shall recognize. et al. the State shall regulate the acquisition. respect. Creating a National Commission on Indigenous Peoples. The Congress shall give the highest priority to the enactment of measures that protect and enhance the right of all the people to human dignity.. and remove cultural inequalities by equitably diffusing wealth and political power for the common good. Secretary of DENR. entitled State Principles and Policies) Section 5. and for Other Purposes. Dec. (Article II of the Constitution. looking back to the long destitution of our less fortunate brothers. and defines the extent of these lands and domains. 6... social. entitled General Provisions) Puno. The ownership given is the indigenous concept of ownership under customary law which traces its origin to native title. entitled National Economy and Patrimony) Section 1. Establishing Implementing Mechanisms. reduce social. in Isagani Cruz v. No. Arts. Section 12. Protect and Promote the Rights of Indigenous Cultural Communities/Indigenous Peoples. in the disposition and utilization of other natural resources. Technology. the majority of the members of which shall come from such communities. the right to preserve and protect their culture. (Ibid. Culture. and the right to develop their own sciences and technologies (Sections 29 to 37).R. entitled Social Justice and Human Rights) Section 6. En Banc) 30. 8371 is entitled "An Act to Recognize. subject to the provisions of the Constitution and national development policies and programs. to wit: Section 22. The Congress may create a consultative body to advise the President on policies affecting indigenous cultural communities. and cultural well-being. entitled Education. 2000. (Article XII of the Constitution. traditions. and the rights of indigenous communities to their ancestral lands. They incorporated in the fundamental law several provisions recognizing and protecting the rights and interests of the indigenous peoples. the ICCs/IPs are given the right to selfgovernance and empowerment (Sections 13 to 20). It shall consider these rights in the formulation of national plans and policies. including lands of the public domain under lease or concession. and to reach out particularly to the marginalized sectors of society. 135385. Kapunan. social justice and human rights (Sections 21 to 28). Secretary of Environment and Natural Resources.. fittingly saw the historic opportunity to actualize the ideals of people empowerment and social justice. shall protect the rights of indigenous cultural communities to their ancestral lands to ensure their economic. (Article XIII of the Constitution. To this end. The IPRA recognizes the existence of the indigenous cultural communities or indigenous peoples (ICCs/IPs) as a distinct sector in Philippine society. No. and Sports) (Separate Opinion. En Banc) 29.A. including the indigenous peoples. G. (Article XVI of the Constitution. The State shall apply the principles of agrarian reform or stewardship. subject to prior rights. and institutions. No. institutions and community intellectual rights. Dec. The State recognizes and promotes the rights of indigenous peoples within the framework of national unity and development. Define "indigenous peoples/indigenous cultural communities. (Article XIV of the Constitution. 2000. J. in Isagani Cruz v. IPRA now employs the politically-correct conjunctive term ―indigenous peoples/indigenous cultural communities‖ as follows: Section 3.R. Definition of Terms.Held: 1.. Indigenous Cultural Communities or Indigenous Peoples refer to a group of people or homogeneous societies who have continuously lived as an organized community on communally bounded and defined territory. (Separate Opinion. under claims of ownership since time immemorial. the Indigenous Cultural Communities (ICCs) or the Indigenous Peoples (IPs).‖ Do they constitute part of the land of the public domain? Held: Ancestral domains and ancestral lands are the private property of indigenous peoples and do not constitute part of the land of the public domain. who have continuously lived as organized community on communally bounded and defined territory. in Isagani Cruz v. The IPRA is a law dealing with a specific group of people. became historically differentiated from the Filipino majority. et al. Drawing inspiration from both our fundamental law and international law. pasture. possessed and utilized their territories under claim of ownership since time immemorial. These are defined in Section 3(a) and (b) of the Indigenous Peoples Rights Act x x x. agricultural. J. occupied. force majeure or displacement by force. residential. Puno.. cultural and political institutions but who may have been displaced from their traditional territories or who may have resettled outside their ancestral domains. 2000. These lands include but are not limited to residential lots. traditions and other distinctive cultural traits. These groups of people have actually occupied. 135385. 6. who retain some or all of their own social. The IPRA grants to ICCs/IPs a distinct kind of ownership over ancestral domains and ancestral lands. G. Indigenous peoples shall likewise include peoples who are regarded as indigenous on account of their descent from the populations which inhabited the country at the time of conquest or colonization. or. except when interrupted by war. communally or individually since time immemorial. No. but who may have been displaced from their traditional domains or who may have resettled outside their ancestral domains x x x. Define ―ancestral domains‖ and ―ancestral lands. economic. . Secretary of Environment and Natural Resources. forests. the following terms shall mean: (i) Indigenous peoples/Indigenous cultural communities. possessed and utilized such territories. They also include lands which may no longer be exclusively occupied by ICCs/IPs but from which they traditionally had access to for their subsistence and traditional activities. or at the time of inroads of nonindigenous religions and cultures.. (Separate Opinion. particularly the home ranges of ICCs/IPs who are still nomadic and/or shifting cultivators (Section 3[a]. non-indigenous religions and cultures. sharing common bonds of language. who retain some or all of their own social. inland waters. or who have. worship areas. through resistance to political. hunting grounds. Ancestral lands are lands held by the ICCs/IPs under the same conditions as ancestral domains except that these are limited to lands and that these lands are not merely occupied and possessed but are also utilized by the ICCs/IPs under claims of individual or traditional group ownership. customs. They share common bonds of language. burial grounds. social and cultural inroads of colonization. 135385. Secretary of DENR. non-indigenous religions and cultures. cultural and political institutions. The term ―ICCs‖ is used in the 1987 Constitution while that of ―IPs‖ is the contemporary international language in the International Labor Organization (ILO) Convention 169 and the United Nations (UN) Draft Declaration on the Rights of Indigenous Peoples . continuously until the present. and other distinctive cultural traits. traditions.refer to a group of people or homogenous societies identified by self-ascription and ascription by others. social and cultural inroads of colonization. IPRA). private forests.e. became historically differentiated from the majority of Filipinos. and other lands individually owned whether alienable or not. Dec.R. Kapunan. economic.. mineral and other natural resources. bodies of water. . G. i.For purposes of this Act. Dec. in Isagani Cruz v. 2000.. J. occupied or possessed by ICCs/IPs by themselves or through their ancestors. En Banc) 2. swidden farms and 28 . customs. Ancestral lands are not the same as ancestral domains. No. Ancestral domains comprise lands. they. deceit. and natural resources therein and includes ancestral lands. rice terraces or paddies. Ancestral domains are all areas belonging to ICCs/IPs held under a claim of ownership. ICCs/IPs also include descendants of ICCs/IPs who inhabited the country at the time of conquest or colonization. and who have. En Banc) 31. 6. stealth or as a consequence of government projects or any other voluntary dealings with government and/or private individuals or corporations. coastal areas. et al. by their resistance to political. or the establishment of present State boundaries. The land is not acquired from the State. and the true and only source of title. Secretary of DENR. 2000.. Discuss the concept of ―jura regalia‖ and how it evolved in the Philippines. Insular Government (41 Phil. the United States and the Philippine Government. The concept of native title in the IPRA was taken from the 1909 case of Carino v. in Isagani Cruz v. 2000. 2000. express or implied. The belief that the Spanish Crown is the origin of all land titles in the Philippines has persisted because title to land must emanate from some source for it cannot issue forth from nowhere. 135385. in Isagani Cruz v.. which presupposes a transfer of title from the State to a private person. G. and from him all lands were held. 594). however. however. (Separate Opinion. What is the concept of ―native title?‖ What is a Certificate of Ancestral Domain Title (CADT)? Held: Native title refers to ICCs/IPs preconquest rights to lands and domains held under a claim of private ownership as far back as memory reaches. 935 [1909]. Spain or its successor-in-interest. Insular Government (41 Phil. 29 . J. J. Dec. 135385.tree lots (Section 3[b]. Formal recognition. Puno.S. (Separate Opinion. 212 U. In the landmark case of Carino v. The rights of ICCs/IPs to their ancestral domains (which also include ancestral lands) by virtue of native title shall be recognized and respected (Section 11. The theory of the feudal system was that title to all lands was originally held by the King. Held: Ownership by virtue of native title presupposes that the land has been held by its possessor and his predecessor-in-interest in the concept of an owner since time immemorial. There has been no transfer of title from the State as the land has been regarded as private in character as far back as memory goes. in Isagani Cruz v. Dec.R. No. 53 L. from the Spanish Crown or its successors. IPRA). J. G. These lands are deemed never to have been public lands and are indisputably presumed to have been held that way since before the Spanish Conquest. when solicited by ICCs/IPs concerned.R. G. therefore. Dec. it refers to a right which the sovereign has over anything in which a subject has a right of property or propriedad. In contrast. Secretary of DENR. private title to land must be traced to some grant. 212 U.. (Separate Opinion. 135385. En Banc) 33. et al. under the concept of jura regalia. The IPRA categorically declares ancestral lands and domains held by native title as never to have been public land. En Banc) 35. 935. 2000. 6. that is. does not negate native title to lands held in private ownership since time immemorial. These were rights enjoyed during feudal times by the king as the sovereign. Secretary of DENR. or (2) by torrens title under the Public Land Act and the Land Registration Act with respect to ancestral lands only.R. the Philippine Republic. or those rights which the King has by virtue of his prerogatives. Like a torrens title. In Spanish law. the King was regarded as the original proprietor of all lands.R. By fiction of law. the King theoretically retained the title . ownership of land by acquisitive prescription against the State involves a conversion of the character of the property from alienable public land to private land. IPRA)... The Regalian theory. En Banc) 32. in Isagani Cruz v. a CADT is evidence of private ownership of land by native title. How may ICCs/IPs acquire rights to their ancestral domains and ancestral lands? Held: The rights of the ICCs/IPs to their ancestral domains and ancestral lands may be acquired in two modes: (1) by native title over both ancestral lands and domains. Domains and lands held under native title are. and while the use of lands was granted out to others who were permitted to hold them under certain conditions. Distinguish ownership of land under native title and ownership by acquisitive prescription against the State.. et al. Puno. 6. 6. shall be embodied in a Certificate of Ancestral Domain Title (CADT). No. et al.S. is a right of private ownership peculiarly granted to ICCs/IPs over their ancestral lands and domains. In its broad sense. G. Native title. Carino firmly established a concept of private land title that existed irrespective of any royal grant from the State. which shall recognize the title of the concerned ICCs/IPs over the territories identified and delineated . 6. 449. The theory of jura regalia was therefore nothing more than a natural fruit of conquest . the term ―jura regalia‖ refers to royal grants. Ed. Does it negate native title to lands held in private ownership since time immemorial? Held: Generally. the American Colonial government. (Separate Opinion. Dec. Kapunan. J. and thereafter. En Banc) 34. indisputably presumed to have never been public lands and are private. Secretary of DENR. No. No. Puno.. 135385. . By virtue of the communal character of ownership. 8371. x x x (Carino v. or ownership of land by Filipinos by virtue of possession under a claim of ownership since time immemorial and independent of any grant from the Spanish Crown. G. In contrast. the land has been held by individuals under a claim of private ownership. Philippine Bill of 1902). The specification of what areas belong to the ancestral domains is x x x important to ensure that no unnecessary encroachment on private properties outside the ancestral domains will result during the delineation process. The mere fact that Section 3[a] defines ancestral domains to include the natural resources found therein does not ipso facto convert the character of such natural resources as private property of the indigenous peoples. (Separate Opinion. Secretary of DENR.‖ (Section 2. Insular Government. perhaps. in part.‖ The phrase ―private but community property‖ is merely descriptive of the indigenous peoples‘ concept of ownership as distinguished from that provided in the Civil Code. 594 [1909]). Similarly. petroleum. Specifically. wildlife. We applied the Carino doctrine in the 1946 case of Oh Cho v. petitioners and the Solicitor General assail Sections 3[a]. Xxx Carino was decided by the U. Article XII of the Constitution which states. x x x. 5. as an exception to the theory of jura regalia. Kapunan. the property held in common ―cannot be sold. Article XII. Supreme Court in 1909. It might. Its purpose is definitional and not declarative of a right or title. where we stated that ―[a]ll lands that were not acquired from the Government either by purchase or by grant. limit and composition of ancestral domains by setting forth the standards and guidelines in determining whether a particular area is to be considered as part of and within the ancestral domains. at a time when decisions of the U. No. for such possession would justify the presumption that the land had never been part of the public domain or that it had been private property even before the Spanish conquest . Dec. at 941) The above ruling institutionalized the recognition of the existence of native title to land. and describes the extent. coal. that ―[a]ll lands of the public domain. made the following pronouncement: x x x Every presumption is and ought to be taken against the Government in a case like the present. disposed or destroyed‖ because it was meant to benefit the whole indigenous community and not merely the individual member. Does R.A. J. as well as the State‘s full control and supervision over the exploration. belong to the public domain. and other natural resources are owned by the State. 2000. otherwise known as ―the Indigenous People‘s Rights Act‖ infringe upon the State‘s ownership over the natural resources within the ancestral domains? Held: Petitioners posit that IPRA deprives the State of its ownership over mineral lands of the public domain and other natural resources. Ed. Director of Lands (75 Phil. and 7 of IPRA as violative of Section 2. waters. be proper and sufficient to say that when. Court were binding as precedent in our jurisdiction (Section 10. flora and fauna. It does not confer or recognize any right of ownership over the natural resources to the indigenous peoples. That IPRA is not intended to bestow ownership over natural resources to the indigenous peoples is also clear from the deliberations of the bicameral conference committee on Section 7 which recites the rights of indigenous peoples over their ancestral domains x x x. reversing the decision of the pre-war Philippine Supreme Court. Section 3[a] serves only as a yardstick which points out what properties are within the ancestral domains.S. and never to have been public land. fisheries. and other mineral oils. the indigenous peoples‘ concept of ownership emphasizes the importance of communal or group ownership. Xxx Section 3[a] merely defines the coverage of ancestral domains. En Banc) 36. in Isagani Cruz v. In other words. development and utilization of natural resources. all forces of potential energy.449. as far back as testimony or memory goes. forests or timber. supra note 75. 53 L. it will be presumed to have been held in the same way from before the Spanish conquest. 135385. 30 .S. 6. 890 [1946]). the United States Supreme Court. minerals. Section 5 in relation to Section 3[a] cannot be construed as a source of ownership rights of indigenous peoples over the natural resources simply because it recognizes ancestral domains as their ―private but community property. Constitution) They would have the Court declare as unconstitutional Section 3[a] of IPRA because the inclusion of natural resources in the definition of ancestral domains purportedly results in the abdication of State ownership over these resources.R. but [a]n exception to the rule would be any land that should have been in the possession of an occupant and of his predecessors in interest since time immemorial. et al. 6. which enumerates certain rights of the indigenous peoples over the natural resources found within their ancestral domains. et al. because it could only acquire lands which were not yet privately-owned or occupied by 31 . As the owner of natural resources over the Philippines after the latter‘s cession from Spain. 215 US 16.Further. title in these minerals was in all cases sourced from the State. 2000. were considered by Spain as an abundant source of revenue to finance its battle in wars against other nations. 347 SCRA 128. Dec. Secretary of Environment and Natural Resources. Although the United States made a distinction between minerals found in public lands and those found in private lands. 2000. the United States saw it fit to allow both Filipino and American citizens to explore and exploit minerals in public lands. x x x. is the basis of the theory of jura regalia. Section 7 makes no mention of any right of ownership of the indigenous peoples over the natural resources. 2000. En Banc [Per Curiam]) 39. a judicious examination of Reavies reveals that. to prevent foreign control of the country through economic domination. the United States viewed natural resources as a source of wealth for its nationals. NCIP and Flavier. Natural resources. J. bodies of water traditionally and actually occupied by indigenous peoples. Secretary of Environment and Natural Resources. has not been recognized in the Philippines. in Cruz v. it merely upheld the right of the indigenous peoples to claim ownership of minerals under the Philippine Bill of 1902. recognized the separability of title over lands and that over minerals which may be found therein. contain any recognition of ownership vis-à-vis the natural resources. Kapunan. whether in public or private lands. 6. 284-293. invoke the case of Reavies v. However. Article XII of the 1987 Constitution that all lands of the public domain are owned by the State? Consequently. Kapunan. like native title to land. sacred places. contrary to the position of NCIP and Flavier. What is the underlying reason for the State‘s consistent assertion of ownership and control over natural resources from the Spanish regime up to the present? Held: The unique value of natural resources has been acknowledged by the State and is the underlying reason for its consistent assertion of ownership and control over said natural resources from the Spanish regime up to the present. did Spain acquire title over all lands in the Philippines in the 16th century? Held: Dominium was the basis for the early Spanish decrees embracing the theory of jura regalia. unlike native title to land. Rather. Section 7[a] merely recognizes the ―right to claim ownership over lands. The framers of the 1935 Constitution found it necessary to maintain the State‘s ownership over natural resources to insure their conservation for future generations of Filipinos. On the other hand. by asserting its ownership over minerals wherever these may be found. in Cruz v..‖ Neither does Section 7[b].. 1017 [1909]. En Banc [Per Curiam]) 38. including the preservation of the ecological balance therein and the need to ensure that the indigenous peoples will not be unduly displaced when the State-approved activities involving the natural resources located therein are undertaken. in Cruz v. X x x. What is evident is that the IPRA protects the indigenous peoples‘ rights and welfare in relation to the natural resources found within their ancestral domains. In fact. 54 L Ed 72) in support of their thesis that native title to natural resources has been upheld in this jurisdiction. the Court did not recognize native title to natural resources. 284-293. especially minerals. traditional hunting and fishing grounds.. 347 SCRA 128. there was no similar favorable treatment as regards natural resources. Has the concept of native title to natural resources. 347 SCRA 128. been recognized in the Philippines? Held: The concept of native title to natural resources. (Separate Opinion. thereby posing danger to its internal security and independence. then the lands which Spain acquired in the 16th century were limited to non-private lands. and to grant patents to private mineral lands. Article XII of the 1987 Constitution that all lands of the public domain are owned by the State is likewise founded on dominium. En Banc [Per Curiam]) 37. Spain. Secretary of Environment and Natural Resources. Fianza (40 Phil. J. and all improvements made by them at any time within the domains. 6. (Separate Opinion. Hence. not imperium. J. Kapunan. What was the basis for the early Spanish decrees embracing the theory of jura regalia? Is this also the basis of the declaration in Section 2. Dec.. and to avoid situations whereby the Philippines would become a source of international conflicts. While x x x native title to land or private ownership by Filipinos of land by virtue of time immemorial possession in the concept of an owner was acknowledged and recognized as far back during the Spanish colonization of the Philippines. 284-293. The declaration in Section 2. Dec. If dominium. (Separate Opinion. carried out. See Footnote 86) The Right of the State to Recover Properties Unlawfully Acquired by Public Officials or Employees 40. The prosecution‘s motion to withdraw or to dismiss is not the least binding upon the court. Does the right of the State to recover properties unlawfully acquired by public officials or employees which may not be barred by prescription. A sufficient standard is one which defines legislative policy. may lie within the sound discretion of the government prosecutor. 2000. even by the justice secretary. the prosecution of offenses arising from. Article XI of the Constitution apply to criminal cases for the recovery of ill-gotten wealth? Held: Section 15. through the PCGG.‖ From the proceedings of the Constitutional Commission of 1986.]) STRUCTURE OF GOVERNMENT The Doctrine of Separation of Powers 41. based on the evidence proffered. from them or from their nominees as transferees. 299 SCRA 744. public lands.e. including the matter of whom to present as witnesses. On the contrary. In a criminal case. March 19. 1998 [Panganiban]) Delegation of Powers 42. 1987 Constitution provides that ―[T]he right of the State to recover properties unlawfully acquired by public officials or employees.J. i. Discuss the nature of the Party-List system. setting forth therein the policy to be executed. It indicates the circumstances under which the legislative command is to be effected. and not to criminal cases. in Isagani Cruz v. (Presidential Ad Hoc Fact-Finding Committee on Behest Loans. Hon. validly bind itself to cause the dismissal of all cases against the Marcos heirs pending before the Sandiganbayan and other courts in a Compromise Agreement entered into between the former and the latter? Held: This is a direct encroachment on judicial power. maps out its boundaries and specifies the public agency to apply it. 1997) The Legislative Department 43. without any qualification. because granting such motion is equivalent to effecting a disposition of the case itself. Aniano A. however. 270 SCRA 106. for said dismissal is not within its sole power and discretion. or estoppel. No. Kapunan. 1999.R. Thus. J. in what manner it will dispose of the case.the Filipinos. or estoppel under Section 15. (Santiago v. decisional rules require the trial court to make its own evaluation of the merits of the case. relating or incident to. open to all? 32 . Secretary of DENR. It is valid only if the law (a) is complete in itself. C. marks its limits. et al. En Banc [Davide. the PCGG. 130140. but the court decides. 135385. G. laches. Hence. as the government prosecutor of ill-gotten wealth cases. the matter of its dismissal or pursuance lies within the full discretion and control of the judge. G. Spain acquired title only over lands which were unoccupied and unclaimed. Is it. once acquired by the trial court. Desierto. En Banc. to withdraw the information or to dismiss the complaint. or involving ill-gotten wealth contemplated in Section 15. or implemented by the delegate.. Thus. Oct. PCGG. et al. Dec. laches. shall not be barred by prescription.R. Jurisdiction. 25. particularly in regard to criminal jurisdiction.. 6. the manner in which the prosecution is handled. COMELEC. v. (Separate Opinion. (Chavez v. 9. there must be a showing that the delegation itself is valid. Article XI. it was clear that this provision applies only to civil actions for recovery of ill-gotten wealth. is not lost despite a resolution. No. May the Government. cannot guarantee the dismissal of all such criminal cases against the Marcoses pending in the courts.. Well-settled is the doctrine that once a case has been filed before a court of competent jurisdiction. Article XI of the Constitution may be barred by prescription. Dec. What are the tests of a valid delegation of power? Held: [I]n every case of permissible delegation. and (b) fixes a standard – the limits of which are sufficiently determinate and determinable – to which the delegate must conform in the performance of his functions. Section 11 of RA 7941 leaves no doubt as to the participation of political parties in the party-list system. En Banc [Panganiban]) 45. regional. it would create a mirage. 147589. without any qualification.R. that any political party – or any organization or group for that matter – may do so. Crucial to the resolution of this case is the fundamental social justice principle that those who have less in life should have more in law. Is the enumeration of marginalized and underrepresented sectors to be represented under the partylist system in RA 7941 exclusive? Will it be correct to assert that the party-list system is not exclusive to the marginalized and underrepresented sectors. It would gut the substance of the party-list system. That political parties may participate in the party-list elections does not mean. allowing all individuals and groups.Held: 1. June 26. Article IX [C] of the Constitution. G. Instead of generating hope. X x x During the deliberations in the Constitutional Commission. No. June 26. UNIDO. It gives the great masses of our people genuine hope and genuine power. 2001. private respondents cannot be disqualified from the party-list elections. COMELEC. It is a message to the destitute and the prejudiced. Section 2 of RA 7941 also provides for ―a party-list system of registered national. to have the same opportunity to participate in party-list elections would desecrate this lofty objective and mongrelize the social justice mechanism into an atrocious veneer for traditional politics. it would further weaken them and aggravate their marginalization.‖ Section 3 expressly states that a ―party‖ is ―either a political party or a sectoral party or a coalition of parties. It intends to make the marginalized and the underrepresented not merely passive recipients of the State‘s benevolence. and sectoral parties or organizations. merely on the ground that they are political parties. but active participants in the mainstream of representative democracy. empowered to participate directly in the enactment of laws designed to benefit them. The party-list system is one such tool intended to benefit those who have less in life. that change is possible. the Court cannot accept the submissions x x x that the party-list system is. COMELEC. Section 5. COMELEC. Christian S. The requisite character of these parties or organizations must be consistent with the purpose of the party-list system. Clearly. however. the law defines ―political party‖ as ―an organized group of citizens advocating an ideology or platform. regional and sectoral parties or organizations or coalitions thereof. No.‖ More to the point. regularly nominates and supports certain of its leaders and members as candidates for public office. G. Are political parties – even the major ones – prohibited from participating in the party-list elections? Held: Under the Constitution and RA 7941. provides that members of the House of Representatives may ―be elected through a party-list system of registered national.‖ x x x. Furthermore. X x x (Ang Bagong Bayani – OFW Labor Party v. Xxx For its part. but that even the super-rich and overrepresented can validly participate in party-list elections? 33 . as the most immediate means of securing their adoption. and even to those in the underground. open to all. including those which now dominate district elections. therefore. x x x.R. It is an invitation for them to come out of their limbo and seize the opportunity. En Banc [Panganiban]) 44. Magsasaka. (Ang Bagong Bayani – OFW Labor Party v. political parties – even the major ones – may participate in the party-list elections. G. or a regional party in Mindanao. 2001. a national party. principles and policies for the general conduct of government and which. No.R. X x x Indubitably. Thus. (Ang Bagong Bayani – OFW Labor Party v. therefore. but also to enable them to become veritable lawmakers themselves. 147589. June 26. 2001. Comm. a sectoral party. it also prejudices them. as laid down in the Constitution and RA 7941. Such position does not only weaken the electoral chances of the marginalized and underrepresented. Instead of enabling the marginalized. Monsod pointed out that the participants in the party-list system may ―be a regional party. 147589. En Banc [Panganiban]) 2. political parties may be registered under the party-list system. Article VI of the Constitution. under Sections 7 and 8. The party-list system is a social justice tool designed not only to give more law to the great masses of our people who have less in life.‖ Furthermore. Logic shows that the system has been opened to those who have never gotten a foothold within it – those who cannot otherwise win in regular elections and who therefore need the ―simplest scheme possible‖ to do so. COMELEC.Held: While the enumeration of marginalized and underrepresented sectors is not exclusive. G. the party-list system truly empowers the masses and ushers a new hope for genuine change. June 26. No. indeed. to the prejudice of the intended beneficiaries. It was for them that the party-list system was enacted – to give them not only genuine hope. in order to enhance the chance of sectoral groups and organizations to gain representation in the House of Representatives through the simplest scheme possible. to give them opportunity to be elected and to represent the specific concerns of their constituencies.‖ Does the Constitution require all such allocated seats to be filled up all the time and under all circumstances? per centum of the total number of representatives including those under the party-list. Section 5(2). X x x [W]e stress that the party-list system seeks to enable certain Filipino citizens – specifically those belonging to marginalized and underrepresented sectors. but also prejudice the chance of the marginalized and underrepresented. Verily. X x x While the business moguls and the mega-rich are. the urban poor. the x x x position to treat them similarly defies reason and common sense. for the stark reality is that their economic clout engenders political power more awesome than their numerical limitation. political power does not necessarily emanate from the size of one‘s constituency. it demonstrates the clear intent of the law that not all sectors can be represented under the party-list system. as indeed many of them came out and participated during the last elections. the law could not have given the same tool to others. The party-list system is a tool for the benefit of the underprivileged. the purpose of the party-list provision was to open up the system. Traditionally. contrary to the intention of the law to enhance it. allowing the non-marginalized and overrepresented to vie for the remaining seats under the party-list system would not only dilute. The assertion x x x that the party-list system is not exclusive to the marginalized and underrepresented disregards the clear statutory policy. The State cannot now disappoint and frustrate them by disabling the desecrating this social justice vehicle. a tiny minority. Its claim that even the superrich and overrepresented can participate desecrates the spirit of the party-list system. Because the marginalized and underrepresented had not been able to win in the congressional district elections normally dominated by traditional politicians and vested groups. 147589. organizations and parties – to be elected to the House of Representatives. Conversely. The interests of these two sectors are manifestly disparate. even those in the underground movement – to come out and participate. hence. the OSG and the Comelec disregard the fundamental difference between the congressional district elections and the party-list elections. it invites those marginalized and underrepresented in the past – the farm hands. destitution and infirmity. En Banc [Panganiban]) 46. they are neither marginalized nor underrepresented. it is likely to arise more directly from the number and amount of one‘s bank accounts. In its noblest sense. and simply to give them a direct vote in Congress and in the larger affairs of the State. 20 percent of the seats in the House of Representatives were set aside for the party-list system. As earlier noted. therefore. In arguing that even those sectors who normally controlled 80 percent of the seats in the House could participate in the party-list elections for the remaining 20 percent. but genuine power. Indeed. the law crafted to address the peculiar disadvantage of Payatas hovel dwellers cannot be appropriated by the mansion owners of Forbes Park.‖ Xxx Held: The Constitution simply states that ―[t]he party-list representatives shall constitute twenty 34 . numerically speaking. Article VI of the Constitution provides that ―[t]he party-list representatives shall constitute twenty per centum of the total number of representatives including those under the partylist. 2001. the fisher folk. It is ironic.R. it would be illogical to open the system to those who have long been within it – those privileged sectors that have long dominated the congressional district elections. Xxx Verily. (Ang Bagong Bayani – OFW Labor Party v. that the marginalized and underrepresented in our midst are the majority who wallow in poverty. En Banc [Panganiban]) 47.only those garnering a minimum of two percent of the total valid votes cast for the party-list system are "qualified" to have a seat in the House of Representatives.‖ In other words. Article VI of the Constitution.the additional seats which a qualified party is entitled to shall be computed "in proportion to their total number of votes. 2000.R. No. The Constitution explicitly sets down only the percentage of the total membership in the House of Representatives reserved for party-list representatives. the twenty percent allocation . regardless of the number of votes it actually obtained. As said earlier. 2003 Resolution. It however deemed it necessary to require parties. proportional representation . that is. one "qualifying" and two additional seats. Congress declared therein a policy to promote ―proportional representation‖ in the election of party-list representatives in order to enable Filipinos belonging to the marginalized and underrepresented sectors to contribute legislation that would benefit them. (Veterans Federation Party v. the Constitution and Republic Act No. organizations and coalitions participating in the system to obtain at least two percent of the total votes cast for the party-list system in order to be entitled to a party-list seat. The critical question now is this: To determine the ―total votes cast for the party-list system. G. no winning party. These are: First. Fourth. Hence. Third. Comelec. Oct. the two-percent benchmark required by law has now been allegedly attained by movants. 136781 and Companion Cases. the two percent threshold .We rule that a simple reading of Section 5. COMELEC. they now pray for their proclamation as winners in the last party-list elections. To determine the ―total votes cast for the party-list system. the Court declared that ―the ineligibility of a candidate receiving majority votes does not entitle the eligible candidate receiving the next highest number of votes to be declared elected. No. A minority or defeated candidate cannot be deemed elected to the office. there is a need to review related jurisprudence on the matter. It merely provides a ceiling for party-list seats in Congress. Second. easily conveys the equally simple message that Congress was vested with the broad power to define and prescribe the mechanics of the party-list system of representation. the votes cast for an ineligible or disqualified candidate cannot be considered ―stray. does the clause ―total votes cast for the party-list system‖ include only those ballots cast for qualified party-list candidates? Labo v. only those parties garnering a minimum of two percent of the total votes cast for the party-list system are entitled to have a seat in the House of Representatives.R.‖ should the votes tallied for the disqualified candidates be deducted? Held: The instant Motions for proclamation contend that the disqualification of many party-list organizations has reduced the ―total number of votes cast for the party-list elections. the three seat limit . organization or coalition can have more than three seats in the House of Representatives.‖ Because of this reduction." (Veterans Federation Party v. Article VI of the Constitution is not mandatory.‖ 35 . Those garnering more than this percentage could have ―additional seats in proportion to their total number of votes.each qualified party. Xxx Considering the foregoing statutory requirements. Labo and Grego Not Applicable To answer this question.‖ should the votes tallied for the disqualified candidates be deducted? Otherwise stated. which were mentioned in our February 18. is entitled to a maximum of three seats. In the exercise of its constitutional prerogative. En Banc [Panganiban]) 48. Recall that under Section 11(b) of RA 7941 (the Party-List Act). it will be shown x x x that Section 5(2). 136781. 6.‖ Furthermore.the combined number of all party-list congressmen shall not exceed twenty percent of the total membership of the House of Representatives. Comelec and Grego v. Oct. 2000. 6. including those elected under the party list. especially In Labo. COMELEC. What are the inviolable parameters to determine the winners in a Philippine-style party-list election? Held: To determine the winners in a Philippine-style party-list election. G. Congress enacted RA 7941. 7941 mandate at least four inviolable parameters. ‖ In other words. Doing so will further concretize and give flesh to the policy declaration in RA 7941 x x x. June 25. Hence. history. And it must demonstrate that in a conflict of interest. the eligible candidate obtaining the next higher number of votes may be deemed elected. (Ang Bagong Bayani – OFW Labor Party v. Another reason for not applying Labo and Grego is that these cases involve single elective posts. Section 10 of this latter law clearly provides that the votes cast for a party. culled from the law and the Constitution. fully aware in fact and in law of a candidate‘s disqualification so as to bring such awareness within the realm of notoriety. 2003. it must show – through its constitution. would nonetheless cast their votes in favor of the ineligible. it has chosen or is likely to choose the interest of such sectors. that they represent the interests of the marginalized and underrepresented. 147589. organization or coalition must represent the marginalized and underrepresented groups identified in Section 5 of RA 7941. In other words. articles of incorporation. while even major political parties are expressly allowed by RA 7941 and the Constitution to participate in the party-list system. however. not for interpretation. Second.However. but merely for application. third.R. the latter is a qualification of the former ruling and law. platform of government and track record – that it represents and seeks to uplift marginalized and underrepresented sectors. COMELEC. First. there is room. which is allegedly a religious group. a sectoral organization or a coalition ―not entitled to be voted for shall not be counted‖: Xxx The language of the law is clear. the political party. while the present controversy pertains to the acquisition of a number of congressional seats depending on the total election results – such that even those garnering second. the electorate may be said to have waived the validity and efficacy of their votes by notoriously misapplying their franchise or throwing away their votes. Subtracting the votes garnered by these disqualified party-list groups from the total votes cast under the party-list system will reduce the base figure to 6. hence. They were not meant to cover partylist elections. X x x Third. they must comply with the declared statutory policy of enabling ―Filipino citizens belonging to marginalized and underrepresented sectors x x x to be elected to the House of Representatives. the Court notes the express constitutional provision that the religious sector may not be represented in the party-list system. they must show. In such case. fourth or lesser places could be proclaimed winners depending on their compliance with other requirements. in view of the objections directed against the registration of Ang Buhay Hayaang Yumabong. the Court finds it appropriate to lay down the following guidelines. Likewise.‖ In short. ―this rule would be different if the electorate. to assist the Comelec in its work. namely: 1) the one who obtained the highest number of votes is disqualified. and 2) the electorate is fully aware in fact and in law of a candidate‘s disqualification so as to bring such awareness within the realm of notoriety but would nonetheless cast their votes in favor of the ineligible candidate. that the foregoing pronouncements (1) referred to regular elections for local offices and (2) involved the interpretation of Section 6 of RA 6646. State the guidelines for screening Party-List Participants. no recourse to extrinsic aids is warranted when the language of the law is plain and unambiguous. x x x 36 . bylaws. En Banc [Panganiban]) 49. which held that the exception mentioned in Labo v. however. This means that the two-percent threshold can be more easily attained by the qualified marginalized and under-represented groups. the votes cast for a ―notoriously disqualified‖ candidate may be considered ―stray‖ and excluded from the canvass. while they are not disqualified merely on the ground that they are political parties. Verily. disregarding the votes of disqualified party-list participants will increase and broaden the number of representatives from these sectors.185. Comelec ―is predicated on the concurrence of two assumptions. RA 7941 is a special statute governing the election of party-list representatives and is the controlling law in matters pertaining thereto. No. Since Labo and Section 6 of RA 6646 came into being prior to the enactment of RA 7941. On the other hand. G. Held: In this light. in which case. which are specifically governed by RA 7941.523. Grego and other related cases that came after the enactment of RA 7941 should be construed as inapplicable to the latter. The foregoing pronouncement was reiterated in Grego. sector.‖ Note. majority of its membership should belong to the marginalized and underrepresented. by an industrialist.‖ Note should be taken of paragraph 5. 147589. 7) It has ceased to exist for at least one (1) year. but also deleterious to the objective of the law: to enable citizens belonging to marginalized and underrepresented sectors and organization to be elected to the House of Representatives. These laws include Section 2 of RA 7941. the party or organization must be a group of citizens. which disqualifies a party or group for violation of or failure to comply with election laws and regulations. and how should it be construed? Held: The immunity from arrest or detention of Senators and members of the House of Representatives x x x arises from a provision of the Constitution. 2001. By the very nature of the party-list system. organized by citizens and operated by citizens. the nominees must be Filipino citizens ―who belong to marginalized and underrepresented sectors. rules or regulations relating to elections. En Banc [Panganiban]) 50. 5) It violates or fails to comply with laws. To allow otherwise is to betray the State policy to give genuine representation to the marginalized and underrepresented. 6) It declares untruthful statements in its petition. G. The history of the provision shows that the privilege has always been granted in a restrictive sense. No. Fifth. It must be independent of the government. the government. 2 [5]. Article IX [C]) The prohibition was explained by a member of the Constitutional Commission in this wise: ―[T]he prohibition is on any religious organization registering as a political party. The provision granting an exemption as a special privilege cannot be extended beyond the ordinary meaning of its terms. which enumerates the grounds for disqualification as follows: 1) It is a religious sect or denomination. neither can those of the urban poor or the working class. The 1935 Constitution provided in its Article VI on the Legislative Department: 37 . therefore. the party or organization must not be an adjunct of. so also must its nominees. COMELEC. which states that the party-list system seeks to ―enable Filipino citizens belonging to marginalized and underrepresented sectors. or 8) It fails to participate in the last two (2) preceding elections or fails to obtain at least two per centum (2%) of the votes cast under the party-list system in the two (2) preceding elections for the constituency in which it had registered. Discuss the history of the constitutional provision granting immunity from arrest or detention of Members of Congress. the Constitution provides that ―religious denominations and sects shall not be registered. foreign political party. x x x while lacking a well-defined political constituency. x x x Seventh. it is the registration of a religious sect as a political party. or a project organized or an entity funded or assisted by.‖ (Sec. It may not be extended by intendment. organization or association organized for religious purposes. The participation of the government or its officials in the affairs of a party-list candidate is not only illegal and unfair to other parties. That is not prohibited here. implication or equitable considerations.Furthermore. organizations and parties. organizations and parties x x x to become members of the House of Representatives.‖ Fourth. the interests of the youth cannot be fully represented by a retiree. not only the candidate party or organization must represent marginalized and underrepresented sectors. 4) It is receiving support from any foreign government. To repeat. (Ang Bagong Bayani – OFW Labor Party v. foundation. organization. I do not see any prohibition here against a priest running as a candidate. the party must not only comply with the requirements of the law.‖ Surely. Eighth. Sixth. 2) It advocates violence or unlawful means to seek its goal. whether directly or through any of its officers or members or indirectly through third parties for partisan election purposes.R. the nominee must likewise be able to contribute to the formulation and enactment of appropriate legislation that will benefit the nation as a whole.‖ A party or organization. its nominees must likewise do so. that does not comply with this policy must be disqualified. a party or an organization must not be disqualified under Section 6 of RA 7941. under Section 2 of RA 7941. 3) It is a foreign party or organization. June 26. at 773 [1992]). it is enough that Congress is in session. and in going to and returning from the same. to practically overrule the will of the people. giving priority to any right or interest – not even the police power of the State. The members of Congress cannot compel absent members to attend sessions if the reason for the absence is a legitimate one. He remains a Congressman unless expelled by Congress or. by reason of such fault or misconduct. Article VI of the Constitution which states that – (2) A majority of each House shall constitute a quorum to do business. disqualified. Sec. to wit: X x x but the Batasang Pambansa shall surrender the member involved to the custody of the law within twenty four hours after its adjournment for a recess or for its next session. The 1973 Constitution broadened the privilege of immunity as follows: Article VIII. Because of the broad coverage of felony and breach of the peace. To do otherwise would be to deprive the people of their right to elect their officers. Feb. Jalosjos filed a motion before the Court asking that he be allowed to fully discharge the duties of a Congressman. 2000. Moreover. He contended that his reelection being an expression of popular will cannot be rendered inutile by any ruling. the exemption applied only to civil arrests. be privileged from arrest during their attendance at the sessions of Congress. otherwise such privilege shall cease upon its failure to do so. He was subject to the same general laws governing all persons still to be tried or whose convictions were pending appeal. It can be readily seen x x x that the Aguinaldo case involves the administrative removal of a public officer for acts done prior to his present term of office. and under such penalties. The confinement of a Congressman charged with a crime punishable by imprisonment of more than six years is not merely authorized by law. For relatively minor offenses. in all offenses punishable by not more than six years imprisonment. A congressman like the accused-appellant. x x x. the accused-appellant has not given any reason why he should be exempted from the operation of Section 11. En Banc [Ynares-Santiago]) 51. in the same way that preventive suspension is not removal. including attendance at legislative sessions and committee meetings despite his having been convicted in the first instance of a non-bailable offense. The Senators and Members of the House of Representatives shall in all cases except treason. It does not apply to imprisonment arising from the enforcement of criminal law. Resolve. Held: The accused-appellant argues that a member of Congress‘ function to attend sessions is underscored by Section 16(2). felony. When the people have elected a man to office. For offenses punishable by more than six years imprisonment. that – The Court should never remove a public officer for acts done prior to his present term of office. Accused-appellant‘s reliance on the ruling in Aguinaldo v. otherwise. 324 SCRA 689. there was no immunity from arrest. will not extricate him from his predicament. it has constitutional foundations. Accused-appellant Congressman Romeo G. Jalosjos. convicted under Title Eleven of the Revised Penal Code could not claim parliamentary immunity from arrest. A Member of the Batasang Pambansa shall. (People v. 38 . The requirement that he should be attending sessions or committee meetings has also been removed. It is not for the Court. Article VI of the Constitution. Santos (212 SCRA 768. 3. The present Constitution adheres to the same restrictive rule minus the obligation of Congress to surrender the subject Congressman to the custody of the law. as such House may provide.Sec. 15. if he had been guilty of any. However. The restrictive interpretation of immunity and the intent to confine it within carefully defined parameters is illustrated by the concluding portion of the provision. which states. inter alia. and that they disregarded or forgave his fault or misconduct. be privileged from arrest during his attendance at its sessions and in going to and returning from the same. it must be assumed that they did this with the knowledge of his life and character. confinement pending appeal is not removal. but a smaller number may adjourn from day to day and may compel the attendance of absent Members in such manner. 9. and breach of the peace. Hon. although said provision by no means does away with the applicability of the principle in appropriate cases. It is not the injury to the complainant. it also would be a mockery of the purposes of the correction system.. the Regional Trial Court of Makati granted several motions to temporarily leave his cell at the Makati City Jail. Catalino Macaraig. There is no showing that the above privileges are peculiar to him or to a member of Congress. No. 2000. This can not be countenanced because x x x aside from its being contrary to well-defined Constitutional restraint. Gustilo (19 Phil. As stated in United States v. X x x The accused-appellant avers that his constituents in the First District of Zamboanga del Norte want their voices to be heard and that since he is treated as bona fide member of the House of Representatives. he may no longer serve his full term in office.. (People v. 3. A person charged with crime is taken into custody for purposes of the administration of justice.‖ (Section 1. they do so knowing that at any time. It also serves as an example and warning to others. What the accused-appellant seeks is not of an emergency nature. Gonzales. Such an aberrant situation not only elevates accused-appellant‘s status to that of a special class. Jr. It will be recalled that when a warrant for accused-appellant‘s arrest was issued. at the discretion of the authorities or upon court orders. the accused may be denied bail and thus subjected to incarceration if there is risk of his absconding. 463). he fled and evaded capture despite a call from his colleagues in the House of Representatives for him to attend the sessions ands to surrender voluntarily to the authorities. et al. after his transfer at the New Bilibid Prison in Muntinlupa City. To give a more drastic illustration. 19 November 1990. 212). Moreover. it is the injury to the public which State action in criminal law seeks to redress. et al. They did so with the knowledge that he could achieve only such legislative results which he could accomplish within the confines of prison. The accused-appellant states that the plea of the electorate which voted him into office cannot be supplanted by unfounded fears that he might escape eventual punishment if permitted to perform congressional duties outside his regular place of confinement. Accused-appellant argues that on several occasions. G. whether pending appeal or after final conviction. it is now the same body whose call he initially spurned which accused-appellant is invoking to justify his present motion. it would be a mockery of the aims of the State‘s penal system.One rationale behind confinement. He also calls attention to various instances. Xxx When the voters of his district elected the accused-appellant to Congress. if voters elect a person with full knowledge that he is suffering from a terminal illness. as held in a recent case (Neptali A. 208. En Banc [Ynares-Santiago]) 52. is public selfdefense. Allowing accused-appellant to attend congressional sessions and committee meetings for five (5) days or more in a week will virtually make him a free man with all the privileges appurtenant to his position. for official or medical reasons x x x. Feb. v. 87636. when he was likewise allowed/permitted to leave the prison premises x x x. Emergency or compelling temporary leaves from imprisonment are allowed to all prisoners. Article VIII of the 1987 Constitution) 39 . He also claims that the concept of temporary detention does not necessarily curtail his duty to discharge his mandate and that he has always complied with the conditions/restrictions when he is allowed to leave jail. It cannot abdicate that obligation mandated by the 1987 Constitution. After conviction in the Regional Trial Court.R. May the Supreme Court properly inquire into the motives of the lawmakers in conducting legislative investigations? Can it enjoin the Congress or any of its regular and special committees from making inquiries in aid of legislation? Held: The ―allocation of constitutional boundaries‖ is a task that this Court must perform under the Constitution. 324 SCRA 689. The jurisdiction to delimit constitutional boundaries has been given to this Court. Society must protect itself. We remain unpersuaded. Jalosjos. they did so with full awareness of the limitations on his freedom of action. 191 SCRA 452. the latter urges a co-equal branch of government to respect his mandate. ―[t]he political question doctrine neither interposes an obstacle to judicial determination of the rival claims. Ironically. 40 . Jr.‖ On motion of Senator Orlando Mercado. must be material or necessary to the exercise of a power in it vested by the Constitution. Under Sec. v. thereupon. En Banc [Padilla]) 54. 1950. and the Committee voted to pursue and continue its investigation of the matter. In order therefore to ascertain the character or nature of an inquiry. 20. Bengzon. Thereafter. 87 Phil. July 18. likewise refused to testify invoking his constitutional right to due process. Arnault v. the petitioners filed the present petition for prohibition with a prayer for temporary restraining order and/or injunctive relief. The Senate Blue Ribbon Committee. Jr. 1 of the As held in Jean L. the Senate may refer to any committee or committees any speech or resolution filed by any Senator which in its judgment requires an appropriate inquiry in aid of legislation. Leon Nazareno.). 1991. Petitioners and Ricardo Lopa were subpoenaed by the Committee to appear before it and testify on ―what they know‖ regarding the ―sale of the thirtysix (36) corporations belonging to Benjamin ‗Kokoy‘ Romualdez. 3019. the Senate Minority Floor Leader. 29. to be within the jurisdiction of the legislative body making it. it issued a resolution dated 5 June 1989 rejecting the petitioners‘ plea to be excused from testifying.The Court is thus of the considered view that it has jurisdiction over the present controversy for the purpose of determining the scope and extent of the power of the Senate Blue Ribbon Committee to conduct inquires into private affairs in purported aid of legislation. it was so incidental to the legislative function as to be implied. 0035 before the Sandiganbayan. The power of both houses of Congress to conduct inquiries in aid of legislation is not. Nov. L-3820. this Court held that although there was no express provision in the 1935 Constitution giving such power to both houses of Congress. v. En Banc [Padilla]) 53. Thus. such as to legislate or to expel a member. absolute or unlimited. Thus. Article VI provides x x x. speedy and adequate remedy in the ordinary course of law. including the right to due process and the right not to be compelled to testify against one‘s self. Senate Blue Ribbon Committee. the Senate Blue Ribbon Committee started its investigation on the matter.‖ At the hearing held on 23 May 1989.‖ It follows then that the rights of persons under the Bill of Rights must be respected. the matter was referred by the Senate to the Committee on Accountability of Public Officers (Blue Ribbon Committee). therefore. Nazareno. the investigation must be ―in aid of legislation in accordance with its duly published rules of procedure‖ and that ―the rights of persons appearing in or affected by such inquiries shall be respected. Nov. (Bengzon. as provided therein. and averring that the publicity generated by respondent Committee‘s inquiry could adversely affect his rights as well as those of the other petitioners who are his co-defendants in Civil Case No. X x x Claiming that the Senate Blue Ribbon Committee is poised to subpoena and require their attendance and testimony in proceedings before the Committee. Petitioner Jose F. suspended its inquiry and directed the petitioners to file their memorandum on the constitutional issues raised. Senate Rules of Procedure Governing Inquiries in Aid of Legislation. 1991. et al. They may also extend to any and all matters vested by the Constitution in Congress and/or in the Senate alone. Is the power of both houses of Congress to conduct inquiries in aid of legislation absolute or unlimited? Held: The 1987 Constitution expressly recognizes the power of both houses of Congress to conduct inquiries in aid of legislation (In Arnault v. 20. after which. Jr. 0035 before the Sandiganbayan. 29). Senate Blue Ribbon Committee. The power to conduct formal inquiries or investigations is specifically provided for in Sec. Juan Ponce Enrile delivered a speech ―on a matter of personal privilege‖ before the Senate on the alleged ―take-over of SOLOIL Incorporated. Hon.S. Its exercise is circumscribed by the afore-quoted provision of the Constitution. 4 of the aforementioned Rules. 203 SCRA 767. Section 21. 87 Phil. Such inquiries may refer to the implementation or re-examination of any law or in connection with any proposed legislation or the formulation of future legislation. resort must be had to the speech or resolution under which such an inquiry is proposed to be made. the inquiry. Ricardo Lopa declined to testify on the ground that his testimony may ―unduly prejudice‖ the defendants in Civil Case No. particularly with regard to Republic Act No. 203 SCRA 767. in excess of its jurisdiction and legislative rights. and that there is no appeal nor any other plain. On 13 September 1988. the Anti-Graft and Corrupt Practices Act. the flagship on the First Manila Management of Companies (FMMC) by Ricardo Lopa‖ and called upon ―the Senate to look into the possible violation of the law in the case. (No. (Bengzon. 7716 (The Expanded-VAT Law) did not ―originate exclusively‖ in the House of Representatives as required by Art. the issue has been pre-empted by that court. 24 was adopted from the American Federal Constitution. In fine. the Court may take judicial notice that Mr. Nov. for the respondent Committee to probe and inquire into the same justiciable controversy already before the Sandiganbayan. it was held: ―x x x. The power of Congress to conduct inquiries in aid of legislation is inherent in the legislative process. it is not unlimited. because a bill originating in the House may undergo such extensive changes 41 . 3019. or political system for the purpose of enabling Congress to remedy them. There appears to be. petitioners point out that although Art. no intended legislation involved. 11197 and S. the possibility of its influence being made to bear on the ultimate judgment of the Sandiganbayan can not be discounted. Ricardo Lopa died during the pendency of this case. 3019. No. Held: This argument will not bear analysis.Held: A perusal of the speech of Senator Enrile reveals that he (Senator Enrile) made a statement which was published in various newspapers on 2 September 1988 accusing Mr. the speech of Senator Enrile contained no suggestion of contemplated legislation. En Banc [Padilla]) 55. therefore. otherwise known as ―The Anti-Graft and Corrupt Practices Act. economic. v. 7716 must retain the essence of H. Sec. had violated the law in connection with the alleged sale of the 36 or 39 corporations belonging to Benjamin ―Kokoy‖ Romualdez to the Lopa Group. 5 of RA No. 24 of the Constitution. Senate Blue Ribbon Committee. according to them.‖ (italics supplied) It cannot be overlooked that when respondent Committee decided to conduct its investigation of the petitioners. the purpose of the inquiry to be conducted by respondent Blue Ribbon Committee was to find out whether or not the relatives of President Aquino. Watkins v. the ―Anti-Graft and Corrupt Practices Act‖. In short. Nor is the Congress a law enforcement or trial agency. 1630. It comprehends probes into departments of the Federal Government to expose corruption. he merely called upon the Senate to look into a possible violation of Sec. is the alleged sale of the 36 (or 39) corporations belonging to Benjamin ―Kokoy‖ Romualdez. That power is broad. Investigations conducted solely for the personal aggrandizement of the investigators or to ‗punish‘ those investigated are indefensible. particularly Mr. Besides. it must be related to and in furtherance of a legislative task of Congress. Ricardo Lopa. In this connection. Jr. No. a matter that appears more within the province of the courts rather than of the legislature. There is no general authority to expose the private affairs of individuals without justification in terms of the functions of Congress. But broad as is this power of inquiry.‖ X x x Verily. Since the issues in said complaint had long been joined by the filing of petitioners‘ respective answers thereto. since the aim of the investigation is to find out whether or not the relatives of the President or Mr. as defendants therein. A perusal of that complaint shows that one of its principal causes of action against herein petitioners. 20. 203 SCRA 767. (Bengzon. It encompasses inquiries concerning the administration of existing laws as well as proposed or possibly needed statutes. 0035 had already been filed with the Sandiganbayan. but if the Committee‘s judgment were to be reached before that of the Sandiganbayan. United States.which is required by the Constitution to ―originate exclusively‖ in the House of Representatives. This was freely conceded by the Solicitor General in his arguments in this case. Sec. Xxx It appears. Petitioners‘ contention is that Republic Act No. This means. In John T. it is notable in two respects: the verb ―shall originate‖ is qualified in the Philippine Constitution by the word ―exclusively‖ and the phrase ―as on other bills‖ in the American version is omitted. No. No inquiry is an end in itself. the issue sought to be investigated by the respondent Committee is one over which jurisdiction had been acquired by the Sandiganbayan. Republic Act No. therefore. These are functions of the executive and judicial departments of government. To allow the respondent Committee to conduct its own investigation of an issue already before the Sandiganbayan would not only pose the possibility of conflicting judgments between a legislative committee and a judicial tribunal. H. that to be considered as having originated in the House. VI.but the revenue bill .‖ In other words. would be an encroachment into the exclusive domain of judicial jurisdiction that had much earlier set in. it is not the law . Ricardo Lopa had violated Section 5 of RA No. It includes surveys of defects in our social. Ricardo ―Baby‖ Lopa of ―having taken over the FMMC Group of Companies. the complaint in Civil Case No. VI. that the contemplated inquiry by respondent Committee is not really ―in aid of legislation‖ because it is not related to a purpose within the jurisdiction of Congress. To begin with. It is important to emphasize this. 1991. because it is in fact the result of the consolidation of two distinct bills. inefficiency or waste. 11197. are expected to approach the same problems from the national perspective. 661-663. No. No. 11197 was only a substitute of those earlier bills. what is important to note is that. Aug. 253) to amend the VAT law was first filed on July 22. No. 1630. what the Constitution simply means is that the initiative for filing revenue. and. who are elected at large. 11197. on the other hand. private bills and bills of local application must come from the House of Representatives on the theory that. bills authorizing an increase of the public debt. 1630. To insist that a revenue statute . eight months before the House passed H. the members of the House can be expected to be more sensitive to the local needs and problems. For that matter. It is the exercise of a check on the executive power.in the Senate that the result may be a rewriting of the whole. 1129 had been filed in the Senate. then. The possession of a similar power by the U. We are dealing here with the legislative power which under the Constitution is vested not only in any particular chamber but in the Congress of the Philippines. (Tolentino v. Several other bills had been filed in the House before S. 1129 was filed in the Senate. 25.S. The Court cannot. On the other hand. tariff. no justification for comparing the legislative powers of the House and of the Senate on the basis of the possession of a similar nonlegislative power by the Senate. There is really no difference between the Senate preserving H. in order that they may have opportunity of being heard thereon by petition or otherwise if they shall so desire. X x x Given. S. the senators. 1992. as a result of the Senate action. No. of the subjects of legislation that are being considered.and not only the bill which initiated the legislative process culminating in the enactment of the law .must substantially be the same as the House bill would be to deny the Senate's power not only to ―concur with amendments‖ but also to ―propose amendments. 1630 was passed not in substitution of H. 11197 and the submission by the Committee on February 7. if the question were simply the priority in the time of filing of bills. and 3) To fairly apprise the people. and H. No. The possibility of a third version by the conference committee will be discussed later. No. Article VI of the 1987 Constitution are: 1) To prevent hodge-podge or log-rolling legislation. Indeed. so long as action by the Senate as a body is withheld pending receipt of the House bill. There is. The contention that the constitutional design is to limit the Senate's power in respect of revenue bills in order to compensate for the grant to the Senate of the treaty-ratifying power and thereby equalize its powers and those of the House overlooks the fact that the powers being compared are different. No. the fact is that it was in the House that a bill (H. it would seem. 11197 on November 23. a distinct bill may be produced. Senate has never been thought of as giving it more legislative powers than the House of Representatives. Article VI of the 1987 Constitution. 11197 but of another Senate bill (S. 11197) into consideration‖ in enacting S. however. It is insisted. separately presenting a bill of its own on the same subject matter. Both views are thereby made to bear on the enactment of such laws. 11197 up to the enacting clause and then writing its own version following the enacting clause (which. Nor does the Constitution prohibit the filing in the Senate of a substitute bill in anticipation of its receipt of the bill from the House." Held: The objectives of Section 26(1). 1993. No. Secretary of Finance. 235 SCRA 630. and which might therefore be overlooked and carelessly and unintentionally adopted.‖ The exercise of the treaty-ratifying power is not the exercise of legislative power. and if all the parts of the statute are related and germane to the subject matter embodied in the title or so long as the same are 42 . 1129) earlier filed and that what the Senate did was merely to ―take (H. It was only after the Senate had received H. that S. or tax bills. 1994 of S. Section 26(1) of Article VI of the 1987 Constitution is sufficiently complied with where x x x the title is comprehensive enough to embrace the general objective it seeks to achieve.‖ It would be to violate the coequality of legislative power of the two houses of Congress and in fact make the House superior to the Senate. therefore. the Senate can propose its own version even with respect to bills which are required by the Constitution to originate in the House. 1994. No. the power of the Senate to propose amendments. understand the alarm expressed over the fact that on March 1. No. No. 2) To prevent surprise or fraud upon the legislature by means of provisions in bills of which the titles gave no information. In either case the result are two bills on the same subject. No. Discuss the objectives of Section 26(1). that "[e]very bill passed by the Congress shall embrace only one subject which shall be expressed in the title thereof. En Banc [Mendoza]) 56. consisting of ―a Senate and a House of Representatives. petitioners admit is an amendment by substitution). through such publication of legislative proceedings as is usually made. No. No. therefore. 1993 that the process of legislation in respect of it began with the referral to the Senate Committee on Ways and Means of H. elected as they are from the districts. At this point. After all it does not appear that the Senate ever considered it. the signing of H. Section 44 of R. en Banc [Purisima]) 57. is relevant to the subject matter of registration as it seeks to ensure the integrity of the registration process by providing guideline for the COMELEC to follow in the reassignment of election officers. fully index or catalogue. more than mere comity. it bears stressing that the Constitution does not require Congress to employ in the title of an enactment.‘ Consequently. Bill No.A. (Arroyo v. (Agripino A. (Agripino A. What is the Bicameral Conference Committee? Discuss the nature of its function and its jurisdiction. These excursions occur even where the rules impose strict limitations on conference committee jurisdiction. in enacting a law. the courts have no concern. Prado. ‗mere failure to conform to parliamentary usage will not invalidate that action (taken by a deliberative body) when the requisite number of members have agreed to a particular measure.R. in enacting a law. No. courts have declared that ‗the rules adopted by deliberative bodies are subject to revocation. Held: Under the enrolled bill doctrine. Should the challenge be sustained? Held: Section 44 of RA 8189 is not isolated considering that it is related and germane to the subject matter stated in the title of the law. In Osmena v. 227 SCRA 703.R. No. Even where the conference committee is not by rule limited in its jurisdiction. language of such precision as to mirror. 7189 by the Speaker of the House and the President of the Senate and the certification by the secretaries of both Houses of Congress that it was passed on November 21. v. Aug. both here and abroad. et al. it is not limited in its jurisdiction to this question. G. No. Pendatun. PRESCRIBING THE PROCEDURES THEREOF AND AUTHORIZING THE APPROPRIATION OF FUNDS THEREFOR. They may be waived or disregarded by the legislative body. v. 1997 [Mendoza]) 59. De Guzman. Secretary of Finance) that the enrolled bill embodies a conclusive presumption. Courts must accordingly decline the invitation to exercise their power. there is no claim either here or in the decision in the EVAT cases (Tolentino v. 1996 are conclusive of its due enactment. 129118. Its broader function is described thus: A conference committee may deal generally with the subject matter or it may be limited to resolving the precise differences between the two houses. But occasionally a conference committee produces unexpected results. results beyond its mandate. legislative custom severely limits the freedom with which new subject matter can be inserted into the conference bill. Nov. 2000. in varying forms of expression. which provides for the reassignment of election officers. and with their observance. In this regard. 14... 1993. Jr. x x x To be sure.not inconsistent with or foreign to the general subject and title.‘‖ It must be realized that each of the three departments of our government has its separate sphere which the others may not invade without upsetting the delicate balance on which our constitutional order rests. July 19. et al. Discuss the Enrolled Bill Doctrine. The title of RA 8189 is "The Voter's Registration Act of 1996" with a subject matter enunciated in the explanatory note as "AN ACT PROVIDING FOR A GENERAL REGISTRATION OF VOTERS. compels reluctance on the part of the courts to enter upon an inquiry into an alleged violation of the rules of the House. ADOPTING A SYSTEM OF CONTINUING REGISTRATION.‘ And it has been said that ‗Parliamentary rules are merely procedural. 277 SCRA 268. In one case (Astorga v. all the contents and the minute details therein. G. in the absence of showing that there was a violation of a constitutional provision or the right of private individuals. COMELEC. De Guzman. 2000. Jr. modification or waiver at the pleasure of the body adopting them. En Banc [Purisima]) 58. 11. Held: While it is true that a conference committee is the mechanism for compromising differences between the Senate and the House. De Venecia. July 19. Due regard for the working of our system of government. (Philippine Judges Association v. It is not an alien provision but one which is related to the conduct and procedure of continuing registration of voters. En Banc [Cruz]) 60. Villegas) we ―went 43 . all deny to the courts the power to inquire into allegations that. 8189 (The Voter's Registration Act of 1996) which provides for automatic transfer to a new station of any Election Officer who has already served for more than four years in a particular city or municipality was assailed for being violative of Section 26(1) of Article VI of the Constitution allegedly because it has an isolated and different subject from that of RA 8189 and that the same is not expressed in the title of the law. it was held: ―At any rate. This is symptomatic of the authoritarian power of conference committee. a House of Congress failed to comply with its own rules? Held: The cases. COMELEC. a House of Congress failed to comply with its own rules." Section 44. Do courts have the power to inquire into allegations that. 129118. The enrolled bill rule rests on the following considerations: X x x As the President has no authority to approve a bill not passed by Congress. carries.S. and to accept. The enrolled bill doctrine. this Court has refused to determine claims that the three-fourths vote needed to pass a proposed amendment to the Constitution had not been obtained. is in conformity with the Constitution. whether the Act. But. VI. so authenticated. or at some remote period of time. 16[4]). in the absence of evidence to the contrary. at the request of one-fifth of the members present (Id. and having the official attestations of the Speaker of the House of Representatives. 672. The respect due to coequal and independent departments requires the judicial department to act upon that assurance. 1997 [Mendoza]) 63.‖ (Philippine Judges Ass‘n v. 36 L. Clark. Sec. 14. Secretary of Finance) It has refused to look into charges that an amendment was made upon the last reading of a bill in violation of Art. Ed. a solemn assurance by the legislative and executive departments of the government. v. Xxx 44 . and 4) The President‘s objection to a bill he had vetoed (Id.‖ thus: ―They should be public. when the question properly arises. and why? Held: The Journal is regarded as conclusive with respect to matters that are required by the Constitution to be recorded therein. as having passed Congress. 3) The yeas and nays upon repassing a bill over the President‘s veto (Id. is well-established. 143 U. because ―a duly authenticated bill or resolution imports absolute verity and is binding on the courts. charged. It is cited with approval by text writers here and abroad. 298-299.. (Tolentino v. the Journals have also been accorded conclusive effects. Sec. Aug. Prado) In other cases. of the President of the Senate. all bills authenticated in the manner stated. 14.‖ x x x This Court has refused to even look into allegations that the enrolled bill sent to the President contained provisions which had been ―surreptitiously‖ inserted in the conference committee x x x. they should be permanent. 294. because all are required to conform to them. 26[2]). The veto power is not absolute. 303 [1891]) To overrule the doctrine now. What matters are required to be entered on the Journal? Held: 1) The yeas and nays on the third and final reading of a bill (Art. But even as the Constitution grants the power. that it was passed by Congress. that rights acquired today upon the faith of what has been declared to be law shall not be destroyed tomorrow. this Court will respect the certification of the presiding officers of both Houses that a bill has been duly passed. VI. and of the President of the United States.‖ (Arroyo v. respectively. where as here there is no evidence to the contrary. Sec. (Arroyo v. by facts resting only in the memory of individuals. 277 SCRA 268. (Marshall Field & Co.behind‖ an enrolled bill and consulted the Journal to determine whether certain provisions of a statute had been approved by the Senate. 14. Thus.. Pons. Under this rule. De Venecia. Aug. 1997 [Mendoza]) 62. Aug. x x x is to repudiate the massive teaching of our cases and overthrow an established rule of evidence.). De Venecia. 277 SCRA 268. Sec. with the duty of enacting and executing the laws. 26(2) of the Constitution that ―upon the last reading of a bill. it also provides limitations to its exercise. 649. an enrolled Act in the custody of the Secretary of State. 1997 [Mendoza]) 61. 298. in United States v. 27[1]). De Venecia. as a rule of evidence. When should the Legislative Journal be regarded as conclusive upon the courts. leaving the court to determine. (Arroyo v. this Court spoke of the imperatives of public policy for regarding the Journals as ―public memorials of the most permanent character. this Court has denied claims that the tenor of a bill was otherwise than as certified by the presiding officers of both Houses of Congress. on its face. 2) The yeas and nays on any question. no amendment shall be allowed. With respect to other matters. What are the limitations on the veto power of the President? Held: The act of the Executive in vetoing the particular provisions is an exercise of a constitutionally vested power. 277 SCRA 268. in the case of Bengzon v. provisions. A disqualification case was filed against a candidate for Congressman before the election with the COMELEC. 208 SCRA 133. While the proclamation of a winning candidate in an election is ministerial. En Banc [Gutierrez]) 66. 312) declared ―that an item‖ of an appropriation bill obviously means an item which in itself is a specific appropriation of money. An item in a bill refers to the particulars. returns and qualifications of candidates for either the Senate or the House only when the latter become members of either the Senate or the House of Representatives. the Administration needs the money to run the machinery of government and it can not veto the entire bill even if it may contain objectionable features. 1995 elections. He claims that jurisdiction over the petition for disqualification is exclusively lodged with the House of Representatives Electoral Tribunal (HRET). 1992. Drilon. [P]etitioner vigorously contends that after the May 8. Under the above-stated provision. A candidate who has not been proclaimed and who has not taken his oath of office cannot be said to be a member of the House of Representatives subject to Section 17 of Article VI of the Constitution. compelled to approve into law the entire bill.P. The President is. 1992. He or she cannot act like an editor crossing out specific lines. therefore.The OSG is correct when it states that the Executive must veto a bill in its entirety or not at all. However. returns and qualifications of their respective Members. Section 17 of Article VI of the 1987 Constitution reads: The Senate and the House of Representatives shall have an Electoral Tribunal which shall be the sole judge of all contests relating to the election. We disagree. 143-145. Thus. Petitioner conveniently confuses the distinction between an unproclaimed candidate to the House of Representatives and a member of the same.A. when it comes to appropriation. although he was not yet proclaimed because of that pending disqualification case. (Bengzon v. therefore. April 15. Held: The terms item and provision in budgetary legislation and practice are concededly different. Neither may the President set aside or reverse a final and executory judgment of this Court through the exercise of the veto power. 6646 allows suspension of proclamation under circumstances mentioned therein. 143-145. (Bengzon v. Is the COMELEC now ousted of jurisdiction to resolve the pending disqualification case and. (Bengzon v. should dismiss the case. 881 in conjunction with Sec. the COMELEC lost its jurisdiction over the question of petitioner‘s qualifications to run for member of the House of Representatives. which happens to be put into an appropriation bill. 6 of R. 208 SCRA 133. Drilon. It is an indivisible sum of money dedicated to a stated purpose . En Banc [Gutierrez]) 65. Distinguish an ―item‖ from a ―provision‖ in relation to the veto power of the President. En Banc [Gutierrez]) 64. May the President veto a law? May she veto a decision of the SC which has long become final and executory? Held: We need no lengthy justifications or citations of authorities to declare that no President may veto the provisions of a law enacted thirty-five (35) years before his or her term of office. The Constitution provides that only a particular item or items may be vetoed. The power to disapprove any item or items in an appropriate bill does not grant the authority to veto a part of an item and to approve the remaining portion of the same item. Secretary of Justice (299 U. the details. not some general provision of law. It is for this reason that the Constitution has wisely provided the ―item veto power‖ to avoid inexpedient riders being attached to an indispensable appropriation or revenue measure. In the exercise of the veto power. Ed. including its undesirable parts. 410. 57 Ct 252. April 15. revenue or tariff bills. April 15. 414. the electoral tribunal clearly assumes jurisdiction over all contests relative to the election. The United States Supreme Court.. Obtaining the highest number of votes in an election does not automatically vest the position in the winning candidate. The latter failed to resolve that disqualification case before the election and that candidate won. petitioner avers that the COMELEC committed serious error and grave abuse of discretion in directing the suspension of his proclamation as the winning candidate in the Second Congressional District of Makati City.S. 143- 145. 1992. B. 208 SCRA 133. Blg. or paragraphs in a bill that he or she dislikes. Drilon. petitioner‘s contention that ―after the conduct of the election and (petitioner) has been established the 45 . Given the yet-unresolved question of jurisdiction. considering that jurisdiction is now vested with the House of Representatives Electoral Tribunal (HRET)? Held: 1. 81 L. the distinct and severable parts x x x of the bill. it is generally all or nothing. VI. its power of judicial review in exceptional cases. Section 17 of the Constitution. Petitioner further argues that the HRET assumes jurisdiction only if there is a valid proclamation of the winning candidate. 18. J. there is no proclamation at all and the mere assumption of office by the proclaimed candidate does not deprive the COMELEC at all of its power to declare such nullity. COMELEC‘s jurisdiction over election contests relating to his election. J.‖ The Court did recognize. or with grave abuse of discretion or paraphrasing Morrero v. 1995. but only ―in the exercise of this Court‘s so-called extraordinary jurisdiction x x x upon a determination that the Tribunal‘s decision or resolution was rendered without or in excess of its jurisdiction. Under Article VI. 336 SCRA 458. 1995. his subsequent proclamation is void ab initio. En Banc [Kapunan. or upon a demonstration of a very clear unmitigated error. the Court may do so. Sec. upon a clear showing of such arbitrary and improvident use by the Tribunal of its power as constitutes a denial of due process of law. Held: But x x x in an electoral contest where the validity of the proclamation of a winning candidate who has taken his oath of office and assumed his post as congressman is raised. 340-341. En Banc [Kapunan. if any? Held: The Constitution mandates that the House of Representatives Electoral Tribunal and the Senate Electoral Tribunal shall each. be the sole judge of all contests relating to the election. (Guerrero v. the COMELEC‘s decision to discontinue exercising jurisdiction over the case is justifiable. taken his oath. (Aquino v. July 26. 336 SCRA 458.winner of the electoral exercise from the moment of election. 248 SCRA 400. respectively. Bocar (66 Phil. in deference to the HRET‘s own jurisdiction and functions. returns. the COMELEC is automatically divested of authority to pass upon the question of qualification‖ finds no basis in law. returns and qualifications of members of the House of Representatives. Is there an appeal from a decision of the Senate or House of Representatives Electoral Tribunal? What then is the remedy. Where the proclamation is null and void. 6646 to continue to hear and decide questions relating to qualifications of candidates. COMELEC. returns and qualifications of their respective members. The Court has stressed that ―x x x so long as the Constitution grants the HRET the power to be the sole judge of all contests relating to the election. 2000. it is obvious that the HRET at this point has no jurisdiction over the question. 2000. Will the rule be the same if that candidate wins and was proclaimed winner and already assumed office as Congressman? Held: While the COMELEC is vested with the power to declare valid or invalid a certificate of candidacy. returns. COMELEC. its refusal to exercise that power following the proclamation and assumption of the position by Farinas is a recognition of the jurisdictional boundaries separating the COMELEC and the Electoral Tribunal of the House of Representatives (HRET). Thus. the Court has explained that while the judgments of the Tribunal are beyond judicial interference. The power granted to the Electoral Tribunal x x x excludes the exercise of any authority on the part of this Court that would in any wise restrict it or curtail it or even affect the same. En Banc [Quisumbing]) 68. with due regard to the people‘s mandate. for it avoids duplicity of proceedings and a clash of jurisdiction between constitutional bodies. July 26. 17. however. (Romualdez-Marcos v. HRET. and qualifications of members of the House of Representatives.A. He contends that if a candidate fails to satisfy the statutory requirements to qualify him as a candidate. 417-419.]) 67. The reason for this ruling is self-evident. Sept. 248 SCRA 300. Petitioner not being a member of the House of Representatives. that issue is best addressed to the HRET. COMELEC. 18. In Robles v. according to petitioner. 429). and qualifications ends. Sept. En Banc [Quisumbing]) 69. returns and qualifications of members of Congress begins only after a candidate has become a member of the House of Representatives (Art. suffice it to say that HRET‘s jurisdiction as the sole judge of all contests relating to the elections. manifestly constituting such grave abuse of discretion that there has to be a remedy for such abuse. 1987 Constitution). As to the House of Representatives Electoral Tribunal‘s supposed assumption of jurisdiction over the issue of petitioner‘s qualifications after the May 8. the HRET has sole and exclusive jurisdiction over all contests relative to the election. Thus.]) 2. and assumed office as a member of the House of Representatives. as a rule. and the HRET‘s own jurisdiction begins. because even after the elections the COMELEC is empowered by Section 6 (in relation to Section 7) of R. 1995 elections. of course. not be reviewed by this Court. once a winning candidate has been proclaimed.‖ 46 . COMELEC. (Guerrero v. any final action taken by the HRET on a matter within its jurisdiction shall. G. This does not mean. it must be given legal effect. The judiciary has full power to. (2) he emphasized he was leaving the Palace.E. His presidency is now in the past tense. for the sake of peace and in order to begin the healing process of our nation. The press release was petitioner‘s valedictory. 2001 or by the totality of prior. contemporaneous and posterior facts and circumstantial evidence bearing a material relevance on the issue. when his personal or property rights have been invaded. J. and will. Did former President Estrada resign as President or should be considered resigned as of January 20. either. In the 1910 case of Forbes. however humble or of whatever country. Dec. the facts show that petitioner did not write any formal letter of resignation before he evacuated Malacanang Palace in the afternoon of January 20. In granting a writ of prohibition. 283 SCRA 520. 1997 [Vitug]) The Executive Department 70. Without doubt. Nos. (1) he acknowledged the oath-taking of the respondent as President of the Republic albeit with reservation about its legality. that he may. 146710-15. As long as the resignation is clear. sued petitioner W. 22. Harding and C. that the Governor-General. The thing which the judiciary can not do is mulct the Governor-General 47 . this Court. Petitioner‘s reference is to a future challenge after occupying the office of the president which he has given up. whether or not petitioner resigned has to be determined from his acts and omissions before. HRET. we hold that petitioner resigned as President. speaking thru Mr. (4) he assured that he will not shirk from any future challenge that may come ahead on the same service of our country.R. The validity of a resignation is not governed by any formal requirement as to form. It can be written. the respondent Tiaco. respectively. the seat of the presidency. March 2. we hold that the resignation of the petitioner cannot be doubted. On the contrary. held: ―The principle of nonliability x x x does not mean that the judiciary has no authority to touch the acts of the Governor-General. 2001 after the oath-taking of respondent Arroyo. the national spirit of reconciliation and solidarity could not be attained if he did not give up the presidency. etc. Cameron Forbes. It can be express. during and after January 20. Governor-General of the Philippine Islands. Chuoco Tiaco and Crossfield. under cover of his office. his final act of farewell. Using this totality test. and (5) he called on his supporters to join him in the promotion of a constructive national spirit of reconciliation and solidarity. It can be oral. Certainly. a Chinese citizen.R. It was confirmed by his leaving Malacanang. Consequently. 2001 when President Gloria Macapagal Arroyo took her oath as the 14 th President of the Republic? Held: Resignation x x x is a factual question and its elements are beyond quibble: there must be an intent to resign and the intent must be coupled by acts of relinquishment. Such a construction would mean that tyranny. In the cases at bar. under the guise of the execution of the law. En Banc [Puno]) 71. (Libanan v. do what he will. it comes in only when it has to vindicate a denial of due process or correct an abuse of discretion so grave or glaring that no less than the Constitution itself calls for remedial action. It can be implied. like the judges of the courts and the members of the Legislature. may not be personally mulcted in civil damages for the consequences of an act executed in the performance of his official duties. but must submit in silence. declare an act of the GovernorGeneral illegal and void and place as nearly as possible in status quo any person who has been deprived his liberty or his property by such act. 2001. Justice Johnson. He did not say he was leaving the Palace due to any kind of inability and that he was going to re-assume the presidency as soon as the disability disappears . v. Trowbridge.The Court does not x x x venture into the perilous area of correcting perceived errors of independent branches of the Government. that a person injured by the executive authority by an act unjustifiable under the law has no remedy. even by the highest authority of the state. In the press release containing his final statement. he was referring to the past opportunity given him to serve the people as President. it means. when the matter is properly presented to it and the occasion justly warrants it. Discuss our legal history on executive immunity. simply. This remedy is assured to every person. Held: The doctrine of executive immunity in this jurisdiction emerged as a case law. unimpeded and unrestrained. Xxx In sum. for damages for allegedly conspiring to deport him to China. destroying rights of person and of property. could walk defiantly abroad. (3) he expressed his gratitude to the people for the opportunity to serve them. wholly free from interference of courts or legislatures. (Estrada v. Chief of Police and Chief of the Secret Service of the City of Manila. Desierto. disrespect engendered for the person of one of the highest officials of the State and for the office he occupies. The impeachment trial of petitioner Estrada was aborted by 48 . be they government officials or private individuals. now Secretary of Finance. must answer for the consequences of his act. To quote his disquisition: ―In the Philippines though. In such case. Atty. might honestly differ. The move was led by then Member of Parliament. but also when he is without authority. as such. G. x x x (Estrada v. L. Action upon important matters of state delayed. he acts. its framers did not reenact the executive immunity provision of the 1973 Constitution. If he decide wrongly. It can be said that at that point most of us were suffering from AIDS (or absolute immunity defense syndrome). the judicial faculty.‖ Mr.‖ The Opposition in the then Batasang Pambansa sought the repeal of this Marcosian concept of executive immunity in the 1973 Constitution. Desierto. reasonably qualified for that position. who acted upon orders of the President. brightened the modifications effected by this constitutional amendment on the existing law on executive privilege. Pacifico Agabin. When the 1987 Constitution was crafted. provided he actually used discretion and judgment. ―Presidential Immunity And All The King‘s Men: The Law Of Privilege As A Defense To Actions For Damages. in determining whether he had authority to act or not. he is still protected provided the question of his authority was one over which two men. the time and substance of the chief executive spent in wrangling litigation. On the contrary. and. particularly that portion which touched the liability of judges and drew an analogy between such liability and that of the Governor-General.‖ The effort failed. we broadened its coverage so as to include not only the President but also other persons. Neither does this principle of nonliability mean that the chief executive may not be personally sued at all in relation to acts which he claims to perform as such official. Second. any more than it can a member of the Philippine Commission or the Philippine Assembly. Alberto Romulo. Sinco Professorial Chair Lecture entitled. March 2. 146710-15. viz: ―x x x. The 1973 Constitution ceased to exist when President Marcos was ousted from office by the People Power revolution in 1986. 2001.R. Nos. not as GovernorGeneral but as a private individual. it was amended and one of the amendments involved executive immunity.‖ Our 1935 Constitution took effect but it did not contain any specific provision on executive immunity. What is held here is that he will be protected from personal liability for damages not only when he acts within his authority. In 1981. 113 [1987]) petitioner‘s learned counsel. And third. it clearly appears from the discussion heretofore had. Public policy forbids it. we sought to do the American one better by enlarging and fortifying the absolute immunity concept.J. Justice Johnson underscored the consequences if the Chief Executive was not granted immunity from suit.‖ (62 Phil. Article VII stated: ―The President shall be immune from suit during his tenure.‖ In his second Vicente G. but he is not protected if the lack of authority to act is so plain that two such men could not honestly differ over its determination. that is. In other words. He denounced the immunity as a return to the anachronism ―the king can do no wrong. former Dean of the UP College of Law.personally in damages which result from the performance of his official duty. he is entitled to protection in determining the question of his authority. Then came the tumult of the martial law years under the late President Ferdinand E. in a distrust as to the integrity of government itself. Marcos and the 1973 Constitution was born. resulting in a way. Section 17. The immunities herein provided shall apply to the incumbent President referred to in Article XVII of this Constitution. First. no suit whatsoever shall lie for official acts done by him or by others pursuant to his specific orders during his tenure. en Banc [Puno]) 72. Can former President Estrada still be prosecuted criminally considering that he was not convicted in the impeachment proceedings against him? Held: We reject his argument that he cannot be prosecuted for the reason that he must first be convicted in the impeachment proceedings. who argued that the after incumbency immunity granted to President Marcos violated the principle that a public office is a public trust. we extended it to shield the President not only from civil claims but also from criminal cases and other claims. a tendency to unrest and disorder. that the latter is liable when he acts in a case so plainly outside of his power and authority that he can not be said to have exercised discretion in determining whether or not he had the right to act. we enlarged its scope so that it would cover even acts of the President outside the scope of official duties. Thereafter. 83 ―Recognizing that the Impeachment Court is Functus Officio. A 49 . Considering the peculiar circumstance that the impeachment process against the petitioner has been aborted and thereafter he lost the presidency. COMELEC. G. on February 7. Xxx Indeed. G. it is untenable for petitioner to demand that he should first be impeached and then convicted before he can be prosecuted. Congress should not have allowed the COMELEC to usurp a power that constitutionally belongs to it or x x x to encroach ―on the power of Congress to canvass the votes for president and vice-president and the power to proclaim the winners for the said positions. 2. 9189 empowering the COMELEC to order the proclamation of winning candidates insofar as it affects the canvass of votes and proclamation of winning candidates for president and vice-president. although two distinct remedies. Section 4.A. No. Article VII of the Constitution which provides that the returns of every election for President and VicePresident shall be certified by the board of canvassers to Congress. The plea if granted.R. 04-003 and in urging the Supreme Court to instead take on the petitions they directly instituted before it. En Banc [Puno]) 73. Such a submission has nothing to commend itself for it will place him in a better situation than a non-sitting President who has not been subjected to impeachment proceedings and yet can be the object of a criminal prosecution. 2003. 9189 is far too sweeping that it necessarily includes the proclamation of the winning candidates for the presidency and the vice-presidency.‖ Since the Impeachment Court is now functus officio. This is in accord with our ruling in In Re: Saturnino Bermudez that ―incumbent Presidents are immune from suit or from being brought to court during the period of their incumbency and tenure‖ but not beyond. petitioner Estrada cannot demand as a condition sine qua non to his criminal prosecution before the Ombudsman that he be convicted in the impeachment proceedings.5 of R. In addition. Section 18. 2001. No. Mar.A. 9189 in relation to Section 4 of the same Act in contravention of Section 4.4 of the law x x x clashes with paragraph 4. Discuss the jurisdiction of the Supreme Court to hear and decide cases involving the election.5 of R. Election contests consist of either an election protest or a quo warranto which.A. would put a perpetual bar against his prosecution.A. Is Section 18. En Banc [Austria-Martinez]) 74. would have one objective in view. No. is unconstitutional because it violates the following provisions of paragraph 4. paragraph 7. of the 1987 Constitution in assailing the jurisdiction of the COMELEC when it took cognizance of SPA No. 146710-15. (Makalintal v. Nos. 9189 appears to be repugnant to Section 4. Section 4 of Article VII of the Constitution x x x which gives to Congress the duty to canvass the votes and proclaim the winning candidates for president and vicepresident. et al. Desierto. Article VII of the Constitution? Held: Section 4 of R. 2001.the walkout of the prosecutors and by the events that led to his loss of the presidency. No. 9189 provides that the overseas absentee voter may vote for president. vice-president. No. proclamation of winning candidates. To be sure. Xxx Petitioner claims that the provision of Section 18. Section 4.‖ The provisions of the Constitution as the fundamental law of the land should be read as part of The Overseas Absentee Voting Act of 2003 and hence. Indeed. senators and party-list representatives. July 10. the phrase. returns and qualifications of the President and Vice-President. x x x and Velez x x x invoke the provisions of Article VII.R.. No. the proper criminal and civil cases may already be filed against him x x x. Article VIII of the Constitution only insofar as said Section totally disregarded the authority given to Congress by the Constitution to proclaim the winning candidates for the positions of president and vice-president. X x x Xxx Ordinary usage would characterize a ―contest‖ in reference to a post-election scenario.5 of R. the debates in the Constitutional Commission make it clear that when impeachment proceedings have become moot due to the resignation of the President. the canvassing of the votes and the proclamation of the winning candidates for president and vice-president for the entire nation must remain in the hands of Congress. (Estrada v. i. the Court notes that Section 18. the Senate passed Senate Resolution No.5 of R. to dislodge the winning candidate from office. Held: Petitioners Tecson.A. in Section 18. 157013.e. or unlawfully holds or exercises a public office. paragraph 7. It is well-settled that only presidential appointees belonging to the first group require the confirmation by the Commission on Appointments. intrudes into. those whom the President may be authorized by law to appoint. Experience showed that when almost all presidential appointments required the consent of the Commission on Appointments. On the other hand. et al. there are four groups of officers of the government to be appointed by the President: First. Rule 13. returns and qualifications of the ―President‖ or ―Vice-President‖ of the Philippines. Article VII of the 1987 Constitution whose appointments shall require confirmation by the Commission on Appointments. Aug. and Rule 14 of the ―Rules of the Presidential Electoral Tribunal. 161634 x x x would have to be dismissed for want of jurisdiction. placing absolute power to make appointments in the President with hardly any check by the legislature. as what happened under the 1973 Constitution. of the Constitution. Aug. only a registered candidate who would have received either the second or third highest number of votes could file an election protest. 161434 x x x and G. ambassadors. No. The rules categorically speak of the jurisdiction of the tribunal over contests relating to the election. leads to abuse of such power.R. No. In such context. (Maria Jeanette Tecson. and those whose appointments shall no longer require such confirmation? Held: Conformably. officers of the armed forces from the rank of colonel or naval captain.R. under Section 16. This rule again presupposes a post-election scenario. would not include cases directly brought before it questioning the qualifications of a candidate for the presidency or vice-presidency before the elections are held. v. Fourth. 312 SCRA 239. Thus was perceived the need to establish a ―middle ground‖ between the 1935 and 1973 Constitutions. other public ministers and consuls. the heads of the executive departments. A quo warranto proceeding is generally defined as being an action against a person who usurps. Second. Who are the officers to be appointed by the President under Section 16. Sistoza. Accordingly. Mison. En Banc [Vitug]) 75. officers lower in rank whose appointments the Congress may by law vest in the President alone. as consistently interpreted and ruled in the leading case of Sarmiento III v. 312 SCRA 239. (Manalo v. In Rule 14. G. 2004. and in the subsequent cases of Bautista v. This Court touched upon the historical antecedent of the said provision in the case of Sarmiento III v. Salonga. Carale. all other officers of the Government whose appointments are not otherwise provided for by law. defined by Section 4. 161434. as was the case under the 1935 Constitution. would support this premise x x x. No. Article VII) of the Constitution has been the subject of several cases on the issue of the restrictive function of the Commission on Appointments with respect to the appointing power of the President. Held: The aforecited provision (Section 16. 1999. En Banc [Purisima]) 50 . 11. March 3. (Manalo v. and Calderon v. Sistoza. 11. Mison in which it was ratiocinated upon that Section 16 of Article VII of the 1987 Constitution requiring confirmation by the Commission on Appointments of certain appointments issued by the President contemplates a system of checks and balances between the executive and legislative branches of government. and not of ―candidates‖ for President or Vice-President. Article VII.R. En Banc [Purisima]) 76.perusal of the phraseology in Rule 12. State the reason why not all appointments made by the President under the 1987 Constitution will require confirmation by the Commission on Appointments. COMELEC. It is fair to conclude that the jurisdiction of the Supreme Court. the election contest can only contemplate a post-election scenario. of the 1987 Constitution. 1999. and other officers whose appointments are vested in him in this Constitution. The framers of the 1987 Constitution deemed it imperative to subject certain high positions in the government to the power of confirmation of the Commission on Appointments and to allow other positions within the exclusive appointing power of the President. G. Constitutional Commission. Quintos-Deles v. Third.‖ promulgated by the Supreme Court en banc on 18 April 1992. the commission became a venue of ―horse trading‖ and similar malpractices. 312 SCRA 239. Consequently. Article VII of the 1987 Constitution) require the confirmation by the Commission on Appointments. The appointments of respondent officers who are not within the first category.‖ On the other hand. the appointments of police officers whose rank is equal to that of colonel or naval captain will require confirmation by the Commission on Appointments? Held: This contention is x x x untenable. Aug. Thus. En Banc [Purisima]) 78. Discuss the nature of an ad-interim appointment. whether voluntary or compulsory. no less. The Constitution itself makes an ad interim appointment permanent in character by making it effective until disapproved by the Commission on Appointments or until the next adjournment of Congress. The Constitution. Congress cannot by law expand the power of confirmation of the Commission on Appointments and require confirmation of appointments of other government officials not mentioned in the first sentence of Section 16 of Article VII of the 1987 Constitution. need not be confirmed by the Commission on Appointments. Section 6 of the same Article of the Constitution ordains that: ―The State shall establish and maintain one police force. 11. which shall be national in scope and civilian in character to be administered and controlled by a national police commission. can be withdrawn or revoked by the President at her pleasure? Held: An ad interim appointment is a permanent appointment because it takes effect immediately and can no longer be withdrawn by the President once the appointee has qualified into office.77. En Banc [Purisima]) 79. Article VII of the Constitution provides as follows: ―The President shall have the power to make appointments during the recess of the Congress. directors and chief superintendents of the PNP x x x do not fall under the first category of presidential appointees requiring confirmation by the Commission on Appointments. Sistoza. The fact that it is subject to confirmation by the Commission on Appointments does not alter its permanent character. the police force is different from and independent of the armed forces and the ranks in the military are not similar to those in the Philippine National Police. Respondent Sistoza was appointed Director General of the PNP but he refused to submit his appointment papers to the Commission on Appointments for confirmation contending that his appointment shall no longer require confirmation despite the express provision of the law requiring such confirmation. Under Republic Act 6975 (the DILG Act of 1990). but such appointments shall be effective only until disapproval by the Commission on Appointments or until the next adjournment of the Congress. therefore. (Manalo v. Singson. The Philippine National Police is separate and distinct from the Armed Forces of the Philippines. Congress enacted Republic Act 6975 x x x. Aug. It shall keep a regular force necessary for the security of the State. Is it temporary and. therefore. The authority of local executives over the police units in their jurisdiction shall be provided by law. As held in the case of Tarrosa v. 312 SCRA 239. 11. Under Section 4 of Article XVI of the 1987 Constitution.‖ To so distinguish the police force from the armed forces. sets forth the distinction. unconstitutional are Sections 26 and 31 of Republic Act 6975 which empower the Commission on Appointments to confirm the appointments of public officials whose appointments are not required by the Constitution to be confirmed. The second paragraph of Section 16. and other top officials of the Philippine National Police (PNP) shall be appointed by the President and their appointments shall require confirmation by the Commission on Appointments. Deputy Director General. as may be provided by law. 1999. Sistoza. Thereunder. Will it be correct to argue that since the Philippine National Police is akin to the Armed Forces of the Philippines. 1999. the Director General. Should his contention be upheld? Held: It is well-settled that only presidential appointees belonging to the first group (enumerated under the first sentence of Section 16.‖ 51 . (Manalo v. ―The Armed Forces of the Philippines shall be composed of a citizen armed force which shall undergo military training and service. but such appointments shall be effective only until disapproval by the Commission on Appointments or until the next adjournment of the Congress. means a permanent appointment made by the President in the meantime that Congress is in recess. it is not indicative of whether the appointment is temporary or in an acting capacity. the President nominates.‖ Hence. all the powers pertaining to the office. it is used to denote the manner in which said appointments were made.‘‖ Petitioner cites Black‘s Law Dictionary which defines the term ―ad interim‖ to mean ―in the meantime‖ or ―for the time being. Rather. petitioner argues that an ad interim appointment is undoubtedly temporary in character. 1978) . 12. In the language of the Constitution. while the Board of Regents. The fear that the President can withdraw or revoke at any time and for any reason an ad interim appointment is utterly without basis. it is easy to see why the petitioner should experience difficulty in understanding the situation. The term is defined by Black to mean ‗in the meantime‘ or ‗for the time being‘. although not found in the text of the Constitution.‘ It is an appointment permanent in nature.‖ 52 . is unable to act. Article VII of the Constitution.‖ The Constitution imposes no condition on the effectivity of an ad interim appointment. was issued without condition nor limitation as to tenure. has acquired a definite legal meaning under Philippine jurisprudence. the appointment extended to private respondent by then MSU President Alonto.‖ Thus. the ad interim appointment remains effective until such disapproval or next adjournment. In referring to Dr. Ad interim appointments are permanent appointment but their terms are only until the Board disapproves them. we held that: ―x x x an ad interim appointment is one made in pursuance of paragraph (4). Intermediate Appellate Court. decided on October 25. and only upon the consent of the Commission on Appointments may the person thus named assume office.Thus. Said appointment is of course distinguishable from an ‗acting‘ appointment which is merely temporary. or to discharge the duties of the office during the absence or temporary incapacity of its regular incumbent (Black‘s Law Dictionary. This argument is not new and was answered by this Court in Pamantasan ng Lungsod ng Maynila v. which provides that the ‗President shall have the power to make appointments during the recess of the Congress. Section 10. Thus. An ad interim appointment is disapproved certainly for a reason other than that its provisional period has expired. Ozaeta. X x x. it is the literal translation of the word ‗ad interim‘ which creates such belief. the term is not descriptive of the nature of the appointments given to him. and the circumstance that it is subject to confirmation by the Commission on Appointments does not alter its permanent character. His title to such office is complete. this Court had already ruled that an ad interim appointment is permanent in character. which is originally vested by the University Charter with the power of appointment. good until another permanent appointment is issued. signifying that it can no longer be withdrawn or revoked by the President. 1948. as a de jure officer. In Summers v. In the instant case. More than half a century ago. the appointment is effective ‗until disapproval by the Commission on Appointments or until the next adjournment of the Congress. where we explained that: ―x x x From the arguments. It takes effect at once. But such is not the meaning nor the use intended in the context of Philippine law. Private respondent had been extended several ‗ad interim‘ appointments which petitioner mistakenly understands as appointments temporary in nature. done by the President of the Pamantasan in the meantime. Court of Appeals. The term. The individual chosen may thus qualify and perform his function without loss of time. an officer ad interim is one appointed to fill a vacancy. It does not mean a temporary appointment that can be withdrawn or revoked at any time. and thus an ad interim appointment takes effect immediately. The appointee can at once assume office and exercise. The Court had again occasion to explain the nature of an ad interim appointment in the more recent case of Marohombsar v. that is. the term ―ad interim appointment‖. Jr. It is not so with reference to ad interim appointments. that is. where the Court stated: ―We have already mentioned that an ad interim appointment is not descriptive of the nature of the appointment. Esteban‘s appointments. In Pacete v. rather it denotes the manner in which the appointment was made. Revised Fourth Edition. this Court elaborated on the nature of an ad interim appointment as follows: ―A distinction is thus made between the exercise of such presidential prerogative requiring confirmation by the Commission on Appointments when Congress is in session and when it is in recess. Perhaps. The permanent status of private respondent‘s appointment as Executive Assistant II was recognized and attested to by the Civil Service Commission Regional Office No. Petitioner‘s submission that private respondent‘s ad interim appointment is synonymous with a temporary appointment which could be validly terminated at any time is clearly untenable. Secretary of the Commission on Appointments. In the former. as used in letters of appointment signed by the President. 380 SCRA 49. (Matibag v. Benipayo. En Banc [Carpio]) 82. an ad interim appointment becomes complete and irrevocable once the appointee has qualified into office. April 2. in effect. Article IX-B of the Constitution) Thus. The first cause is the disapproval of his ad interim appointment by the Commission on Appointments. 2002. This situation. The Constitution itself sanctions this situation. The second cause is the adjournment of Congress without the Commission on Appointments acting on his appointment. however. A temporary or acting appointee does not enjoy any security of tenure. Benipayo. by the President when Congress was not in session. Art. consistent with the requirements of due process. however. Borra and Tuason are expressly allowed by the Constitution which authorizes the President. unlike Commissioner Haydee Yorac in Brillantes v. the President did in fact appoint permanent Commissioners to fill the vacancies in the COMELEC. He enjoys the constitutional protection that ―[n]o officer or employee in the civil service shall be removed or suspended except for cause provided by law. En Banc [Carpio]) 80. En Banc [Carpio]) 81. Borra and Tuason were extended permanent appointments during the recess of Congress.‖ (Section 2[3]. 380 SCRA 49.An ad interim appointee who has qualified and assumed office becomes at that moment a government employee and therefore part of the civil service. It will also run counter to the clear intent of the framers of the Constitution. This is also part of the check-and-balance under the separation of powers. The withdrawal or revocation of an ad interim appointment is possible only if it is communicated to the appointee before the moment he qualifies. respectively. No one. as a trade-off against the evil of disruptions in vital government services. after notice and hearing. How is an ad interim appointment terminated? Held: An ad interim appointment can be terminated for two causes specified in the Constitution. subject only to confirmation by the Commission on Appointments.‖ this provision should be harmonized with the President‘s power to extend ad interim appointments. 1(2). Yorac and Solicitor General Felix Bautista in Nacionalista Party v. How is an ad interim appointment distinguished from an appointment or designation in an acting or temporary capacity? Held: While an ad interim appointment is permanent and irrevocable except as provided by law. of the Commission on Elections. Benipayo. He can only be removed for cause. To hold that the independence of the COMELEC requires the Commission on Appointments to first confirm ad interim appointees before the appointees can assume office will negate the President‘s power to make ad interim appointments. an appointment or designation in a temporary or acting capacity can be withdrawn or revoked at the pleasure of the appointing power. IX-C of the Constitution that substantially provides that ―No member of the Commission (on Elections) shall be appointed in an acting or temporary capacity? Held: In the instant case. April 2. 380 SCRA 49. Once an appointee has qualified. during the recess of Congress. This is contrary to the rule on statutory construction to give meaning and effect to every provision of the law. Bautista. however. The Constitution has wisely subjected the President‘s appointing power to the checking power of the legislature. While the Constitution mandates that the COMELEC ―shall be independent. (Matibag v. can complain because it is the Constitution itself that places the Sword of Damocles over the heads of the ad interim appointees. Did their appointment violate the Sec. no matter how briefly. Benipayo. This situation. The ad interim appointments of Benipayo. Tuason and Borra were appointed Chairman and Commissioners. These two causes are resolutory conditions expressly imposed by the Constitution on all ad interim appointments. does not compromise the independence of the COMELEC as a constitutional body. (Matibag v. The vacancies in the COMELEC are precisely staggered to insure that the majority of 53 . Xxx The President‘s power to extend ad interim appointments may indeed briefly put the appointee at the mercy of both the appointing and confirming powers. as a trade-off against the evil of granting the President absolute and sole power to appoint. April 2. and any withdrawal or revocation thereafter is tantamount to removal from office . a Sword of Damocles over the heads of ad interim appointees. 2002. in only for a short period – from the time of issuance of the ad interim appointment until the Commission on Appointments gives or withholds its consent. he acquires a legal right to the office which is protected not only by statute but also by the Constitution. Benipayo. 2002. These resolutory conditions constitute. to make appointments that take effect immediately. They were not appointed or designated in a temporary or acting capacity. explained the rationale behind ad interim appointments in this manner: ―Now. Upon the other hand. Inocentes why by-passed ad interim appointees could be extended new appointments. Borra and Tuason. being a refusal by the Commission on Appointments to give its consent after deliberating on the qualifications of the appointee. they were subsequently reappointed by the President to the same positions. the framers of the Constitution thought it wise to reinstate the provisions of the 1935 Constitution on ad interim appointments. the disapproval is final and binding on the appointee as well as on the appointing power. because the incumbent can not continue holding office over the positive objection of the Commission. Article VII of the Constitution – on the nomination of officers subject to confirmation by the Commission on Appointments – did not provide for ad interim appointments. 2002. Since the Constitution does not provide for any appeal from such decision. A by-passed appointment is one that has not been finally acted upon on the merits by the Commission on Appointments at the close of the session of Congress. Article IX-C of the Constitution. April 2. Jr.‖ (Matibag v. In this instance. It 54 . and no one President will appoint all the COMELEC members. 4. Did their subsequent reappointment violate the prohibition against reappointment under Section 1(2). Article IX-A of the Constitution). the reinstatement in the present Constitution of the ad interim appointing power of the President was for the purpose of avoiding interruptions in vital government services that otherwise would result from prolonged vacancies in government offices. Benipayo. 5 and 6. However. It is well-settled in this jurisdiction that the President can renew the ad interim appointments of by-passed appointees. X x x Xxx Clearly. we rule that the ad interim appointments extended by the President to Benipayo. Hence. (Matibag v. 3. This is recognized in Section 17 of the Rules of the Commission on Appointments x x x. In fine. respectively. thus: ―In short. 380 SCRA 49. under the Rules of the Commission on Appointments. of the COMELEC were by-passed by the Commission on Appointments. a bypassed appointment can be considered again if the President renews the appointment. the evil aforementioned may easily be conjured by the issuance of other ad interim appointments or reappointments. once Congress has adjourned. if they expired before the session of Congress. but because of a final decision by the Commission on Appointments to withhold its consent to the appointment. The ad interim appointments of Benipayo. However. Inocentes. En Banc [Carpio]) 84. Benipayo. decided under the 1935 Constitution. 2002. including the three constitutional commissions. as COMELEC Chairman and Commissioners. April 2. Because the same evil would result if the appointments ceased to be effective during the session of Congress and before its adjournment. Absent such decision. the President can no longer renew the appointment not because of the constitutional prohibition on appointment. The disapproval is a decision on the merits. Held: The original draft of Section 16. Justice Roberto Concepcion. The disapproval is a final decision of the Commission on Appointments in the exercise of its checking power on the appointing authority of the President. the President is free to renew the ad interim appointment of a by-passed appointee. In his concurring opinion in Guevarra v. The original intention of the framers of the Constitution was to do away with ad interim appointments because the plan was for Congress to remain in session throughout the year except for a brief 30-day compulsory recess. respectively. En Banc [Carpio]) 83. Article IX-C of the 1987 Constitution? Held: There is no dispute that an ad interim appointee disapproved by the Commission on Appointments can no longer be extended a new appointment. why is the lifetime of ad interim appointments so limited? Because. the evil sought to be avoided – interruption in the discharge of essential functions – may take place. An ad interim appointment that is by-passed because of lack of time or failure of the Commission on Appointments to organize is another matter. Borra and Tuason as Chairman and Commissioners. x x x The special constitutional safeguards that insure the independence of the COMELEC remain in place (See Sections.its members hold confirmed appointments. Discuss the reason why the framers of the 1987 Constitution thought it wise to reinstate the 1935 Constitution provision on ad interim appointments of the President. lucidly explained in his concurring opinion in Guevarra v. because of the need to avoid disruptions in essential government services. Jr. an ad interim appointment ceases to be effective upon disapproval by the Commission. 380 SCRA 49. Justice Roberto Concepcion. There is no final decision by the Commission on Appointments to give or withhold its consent to the appointment as required by the Constitution. do not constitute temporary or acting appointments prohibited by Section 1 (2). it does not matter if the person previously appointed completes his term of office for the intention is to prohibit any reappointment of any kind. De Vera that a [r]eappointment is not prohibited when a Commissioner has held. The second phrase prohibits reappointment of any person previously appointed for a term of five or three years pursuant to the first set of appointees under the Constitution. an ad interim appointment that has lapsed by inaction of the Commission on Appointments does not constitute a term of office. The first phrase prohibits reappointment of any person previously appointed for a term of seven years. and such new appointment will not result in the appointee serving beyond the fixed term of seven years. thus clearly indicating that the reason for said termination of the ad interim appointments is not the disapproval thereof allegedly inferred from said omission of the Commission. The phrase ―without reappointment‖ applies only to one who has been appointed by the President and confirmed by the Commission on Appointments. the Commission may affect adversely the interim appointments only by action. Miraflor. then the President could no longer appoint those so by-passed by the Commission. To hold otherwise will lead to absurdities and negate the President‘s power to make ad interim appointments. Article VII of the present Constitution on ad interim appointments was lifted verbatim. However. the fact is that the President may reappoint them. The jurisprudence under the 1935 Constitution governing ad interim appointments by the President is doubtless applicable to the present Constitution. never by omission. a truncated term of five or three years. This will nullify the constitutional power of the President to make ad interim appointments. Justice Angelo Bautista. also. In the great majority of cases. interrupted only by the 1973 Constitution which did not provide for a Commission on Appointments but vested sole appointing power in the President. Article IX-C of the Constitution. Xxx Xxx In Visarra v. If the adjournment of Congress were an implied disapproval of ad interim appointments made prior thereto.ceases. This is a continuation of the wellrecognized practice under the 1935 Constitution. the President is free to make ad interim appointments or reappointments. the phrase ―without reappointment‖ appears twice in Section 1 (2). on the ad interim appointments first issued to appointees. for. office only for. A disapproved ad interim appointment cannot be revived by another ad interim appointment because the disapproval is final under Section 16. whether or not such person completes his term of office. Article IX-C of the present Constitution. the Commission on Appointments usually fails to act. under the Constitution. simply because the President may then issue new appointments – not because of implied disapproval of the Commission deduced from its intention during the session of Congress. There must be a confirmation by the Commission on Appointments of the previous appointment before the prohibition on reappointment can apply. Article VII of the Constitution. for lack of time.‖ This was the interpretation despite the express provision in the 1935 Constitution that a COMELEC member ―shall hold office for a term of nine years and may not be reappointed. To hold otherwise would mean that the President by his unilateral action could start and complete the running of a term of office in the COMELEC without the consent of the Commission on Appointments. Article IX-C of the Constitution applies neither to disapproved nor by-passed ad interim appointments. and not because a reappointment is prohibited under Section 1 (2). Xxx The framers of the Constitution made it quite clear that any person who has served any term of office as COMELEC member – whether for a full term of seven years. a power 55 . The period from the time the ad interim appointment is made to the time it lapses is neither a fixed term nor an unexpired term. the President will certainly hesitate to make ad interim appointments because most of her appointees will effectively be disapproved by mere inaction of the Commission on Appointments. or even an unexpired term for any length of time – can no longer be reappointed to the COMELEC. This interpretation renders inutile the confirming power of the Commission on Appointments. Article VII of the Constitution. say. three or six years. The prohibition on reappointment in Section 1 (2).‖ To foreclose this interpretation. The established practice under the present Constitution is that the President can renew the appointments of by-passed ad interim appointees. In either case. If such ad interim appointments can no longer be renewed. A by-passed ad interim appointment cannot be revived by a new ad interim appointment because there is no final disapproval under Section 16. quoted Nacionalista v. But. provided his term will not exceed nine years in all. but the circumstance that upon said adjournment of the Congress.‖ Guevarra was decided under the 1935 Constitution from where the second paragraph of Section 16. upon ―the next adjournment of the Congress‖. in his concurring opinion. after confirmation by the Commission on Appointments. The evils sought to be avoided by the twin prohibitions are very specific – reappointment of any kind and exceeding one‘s term in office beyond the maximum period of seven years. Such person cannot be reappointed to the COMELEC. The fourth situation is where the appointee has previously served a term of less than seven years. Article IX-C of the Constitution. 2008. Not contented with these ironclad twin prohibitions. to a vacancy arising from retirement because a reappointment will result in the appointee also serving more than seven years. This provision refers to the first appointees under the Constitution whose terms of office are less than seven years. April 2. The same ad interim appointments and renewal of appointments will also not breach the seven-year term limit because all the appointments and renewals of appointments of Benipayo. 2002. and the appointee completes the unexpired term. the framers of the Constitution tightened even further the screws on those who might wish to extend their terms of office. Article VII of the 1987 Constitution (prohibiting the President from making appointments two months before the next presidential elections and up to the end of his term) directed against? 56 . The framers of the present Constitution prohibited reappointments for two reasons. The third situation is where the appointee is confirmed to serve the unexpired term of someone who died or resigned. whether as a member or as a chairman. and a vacancy arises from death or resignation. Such person cannot be reappointed. Thus. there is no danger whatsoever that the renewal of the ad interim appointments of these three respondents will result in any of the evils intended to be exorcised by the twin prohibitions in the Constitution. the word ―designated‖ was inserted to plug any loophole that might be exploited by violators of the Constitution x x x. a reappointment of such person to serve an unexpired term is also prohibited because his situation will be similar to those appointed under the second sentence of Section 1 (2). (Matibag v. (Matibag v. Article IX-C of the 1987 Constitution which provides that ―[t]he Chairman and the Commissioners (of the COMELEC) shall be appointed x x x for a term of seven years without reappointment‖ will apply? Held: Section 1 (2). Even if it will not result in his serving more than seven years. because he will then be actually serving more than seven years. The second is to insure that the members of the three constitutional commissions do not serve beyond the fixed term of seven years. does not violate the prohibition on reappointments in Section 1 (2). 380 SCRA 49. Article IX-C of the Constitution. April 2.intended to avoid disruptions in vital government services. Consequently. The second situation is where the appointee. after confirmation. To what types of appointments is Section 15. to a vacancy arising from retirement because a reappointment will result in the appointee also serving more than seven years. This Court cannot subscribe to a proposition that will wreak havoc on vital government services. Article IX-C of the Constitution provides that ―[t]he Chairman and the Commissioners shall be appointed x x x for a term of seven years without reappointment. The first is to prevent a second appointment for those who have been previously appointed and confirmed even if they served for less than seven years. En Banc [Carpio]) 86. the prohibition on reappointment is intended to insure that there will be no reappointment of any kind. ‖ There are four situations where this provision will apply. the prohibition on temporary or acting appointments is intended to prevent any circumvention of the prohibition on reappointment that may result in an appointee‘s total term of office exceeding seven years. x x x. whether as a member or chair. serves a part of his term and then resigns before his seven-year term of office ends. 2002. The prohibition on reappointment is common to the three constitutional commissions. A reappointment presupposes a previous confirmed appointment. The continuing renewal of the ad interim appointment of these three respondents. Any delay in their confirmation will not extend the expiry date of their terms of office. but are barred from ever being reappointed under any situation. The first situation is where an ad interim appointee to the COMELEC. for so long as their terms of office expire on February 2. 2008. Xxx Plainly. Borra and Tuason are for a fixed term expiring on February 2. The ad interim appointments and subsequent renewals of appointments of Benipayo. serves his full seven-year term. What are the four situations where Section 1(2). 380 SCRA 49. Borra and Tuason do not violate the prohibition on reappointments because there were no previous appointments that were confirmed by the Commission on Appointments. En Banc [Carpio]) 85. On the other hand. Such person cannot be reappointed. Benipayo. Benipayo. whether as a member or as a chair. Abeja was a municipal mayor. Valenzuela and Hon.J. nor does he have the discretion to modify or replace them. when contrasted with control. (De Rama v. It can accordingly be inferred therefrom that the President can interfere in the exercise of discretion of officials under him or altogether ignore their recommendations. In Taule v. bureaus and offices. Expressed in another way. Before she vacated her office. Supervising officers merely see to it that the rules are followed. Is the prior recommendation of the Secretary of Justice a mandatory requirement before the President may validly appoint a provincial prosecutor? Held: This question would x x x pivot on the proper understanding of the provision of the Revised Administrative Code of 1987 (Book IV. VII of the 1987 Constitution. She ran for reelection but lost. The first refers to those appointments made within two months preceding the Presidential election and are similar to those which are declared election offenses in the Omnibus Election Code. the President has the power to assume directly the functions of an executive department.]) clarified this when it held: ―Section 15. 3rd Div. this Court has had the opportunity to distinguish the power of supervision from the power of control. 1998. and correctly so. we held that the Chief Executive wielded no more authority than that of checking whether a local government or the officers thereof perform their duties as provided by statutory enactments.Held: Section 15.‖ Control means the authority of an empowered officer to alter or modify. (Bito-Onon v. Officers in control lay down the rules in the doing of an act. Article VII is directed against two types of appointments: (1) those made for buying votes and (2) those made for partisan considerations. Title III. In truth and in fact. 9. the only reason he cited to justify his action was that these were ―midnight appointments‖ that are forbidden under Article VII. there is no law that prohibits local elective officials from making appointments during the last days of his or her tenure. the CSC ruled. Evelyn S. 15. Vallarta. it does not include any restraining authority over such body. It should be here pertinent to state that the President is the head of government whose authority includes the power of control over all ―executive departments. Feb. though. Chapter II. Placido B. The incoming mayor. therefore. Jan. while the second consists of the so-called ―midnight‖ appointments. prohibited under Sec. Should the act of the new mayor of recalling said appointments on the aforestated ground be sustained? Held: The records reveal that when the petitioner brought the matter of recalling the appointments of the fourteen (14) private respondents before the CSC. The SC in In Re: Hon.‖ 87. It may not unreasonably be deemed to contemplate not only ―midnight‖ appointments – those made obviously for partisan reasons as shown by their number and the time of their making – but also appointments presumed made for the purpose of influencing the outcome of the Presidential election. 2001. Supervisory power. However. bureau and office. Art. Nov. as well as to substitute the judgment of the latter. upon assuming office. He cannot interfere with local governments provided that the same or its officers act within the scope of their authority. it is discretionary on his part to order the act undone or redone by his subordinate or he may even decide to do it himself. 350 SCRA 732. such conferment must be understood as necessarily carrying with it an ample discretion of whom to appoint. what a subordinate officer has done in the performance of his duties. he may order the work done or redone to conform to the prescribed rules.‖ Petitioners contend that an appointment of a provincial prosecutor mandatorily requires a prior recommendation of the Secretary of Justice endorsing the intended appointment x x x. or even nullify or set aside. 31. Santos. If they are not followed. 28. Supervision does not cover such authority. Section 15 of the Constitution. Article VII has a broader scope than the Aytona ruling. 2001. Section 9) to the effect that – ―All provincial and city prosecutors and their assistants shall be appointed by the President upon the recommendation of the Secretary. that the said prohibition applies only to presidential appointments. but he himself does not lay down such rules. Fernandez. Ma. En Banc [Ynares-Santiago]) 88. 353 SCRA 94. When the Constitution or the law clothes the President with the power to appoint a subordinate officer. Held: On many occasions in the past. En Banc [Narvasa C. (298 SCRA 408. recalled said appointments contending that these were ―midnight appointments‖ and. as and when the former deems it to be appropriate. she extended permanent appointments to fourteen new employees of the municipal government. Court of Appeals. He cannot prescribe his own manner for the doing of the act. [Gonzaga-Reyes]) 89. is the power of mere oversight over an inferior body. If the rules are not observed. Mateo A. Distinguish the President‘s power of general supervision over local governments from his control power. 57 . both of which involve the curtailment and suppression of certain basic civil rights and individual freedoms. However. which embodies the powers of the President as Commander-in-Chief. which is essentially persuasive in character and not binding or obligatory upon the party to whom it is made. in the exercise of the power to suspend the privilege of the writ of habeas corpus or to impose martial law. 58 . Besides the absence of textual standards that the court may use to judge necessity. by way of proof. the evidence upon which the President might decide that there is a need to call out the armed forces may be of a nature not constituting technical proof. Executive Secretary Ruben Torres. Article VII of the Constitution. of the Revised Administrative Code. Article VII x x x. Under the foregoing provisions. exhortation or indorsement. for a period not exceeding sixty days. invasion or rebellion. Explain why the former is not subject to judicial review while the latter two are. Section 18. (2) public safety must require it.‖ The implication is that the President is given full discretion and wide latitude in the exercise of the power to call as compared to the two other powers.‖ found in Section 9. otherwise. 4. Title III. information necessary to arrive at such judgment might also prove unmanageable for the courts. 131429. In many instances. Chapter II.‖ the President may call the armed forces ―to prevent or suppress lawless violence. then this Court cannot undertake an independent investigation beyond the pleadings.R. The full discretionary power of the President to determine the factual basis for the exercise of the calling out power is also implied and further reinforced in the rest of Section 18. he cannot be said as having acted beyond the scope of his authority. when the public safety requires it. 1999. could very well disregard or do away with the action of the departments. from his power to proclaim martial and suspend the privilege of the writ of habeas corpus. In case of invasion or rebellion. two conditions must concur: (1) there must be an actual invasion or rebellion and. G. Certain pertinent information might be difficult to verify. to support the assertion that the President acted without factual basis. should be interpreted x x x to be a mere advise. No. Moreover. or place the Philippines or any part thereof under martial law. that the power to call is fully discretionary to the President. there is no such equivalent provision dealing with the revocation or review of the President's action to call out the armed forces. The only criterion is that ―whenever it becomes necessary. bureaus or offices even in the exercise of discretionary authority. Congress may revoke such proclamations (of martial law) or suspension (of the privilege of the writ of habeas corpus) and the Court may review the sufficiency of the factual basis thereof.e. (Bermudez v. the framers of the Constitution would have simply lumped together the three powers and provided for their revocation and review without any qualification. he may call out such armed forces to prevent or suppress lawless violence. 3rd Div. The distinction places the calling out power in a different category from the power to declare martial law and the power to suspend the privilege of the writ of habeas corpus. being the head of the Executive Department. [Vitug]) 90. or wholly unavailable to the courts. is extant in the deliberation of the Constitutional Commission x x x.. Distinguish the President‘s power to call out the armed forces as their Commander-in-Chief in order to prevent or suppress lawless violence.It is the considered view of the Court x x x that the phrase ―upon recommendation of the Secretary. The recommendation is here nothing really more than advisory in nature. These conditions are not required in the case of the power to call out the armed forces. and thus necessitating safeguards by Congress and review by this Court. Book IV. provides in part: The President shall be the Commander-in-Chief of all armed forces of the Philippines and whenever it becomes necessary. Aug. invasion or rebellion. The reason for the difference in the treatment of the aforementioned powers highlights the intent to grant the President the widest leeway and broadest discretion in using the power to call out because it is considered as the lesser and more benign power compared to the power to suspend the privilege of the writ of habeas corpus and the power to impose martial law. and in so opting. he may. suspend the privilege of the writ of habeas corpus. Expressio unios est exclusio alterius. X x x That the intent of the Constitution is exactly what its letter says. Article VII of the Constitution. If the petitioner fails. i. Held: There is a clear textual commitment under the Constitution to bestow on the President full discretionary power to call out the armed forces and to determine the necessity for the exercise of such power. The President. The factual necessity of calling out the armed forces is not easily quantifiable and cannot be objectively established since matters considered for satisfying the same is a combination of several factors which are not always accessible to the courts. invasion or rebellion. under Section 18. There is likewise nil a basis for the courts to effectuate the reinstatement of a conditional pardon revoked by the President in the exercise of powers undisputably 59 . and the pardonee. Is its grant or revocation by the President subject to judicial review? Held: A conditional pardon is in the nature of a contract between the sovereign power or the Chief Executive and the convicted criminal to the effect that the former will release the latter subject to the condition that if he does not comply with the terms of the pardon. cannot invoke the aid of the courts. Courts have no authority to interfere with the grant by the President of a pardon to a convicted criminal. By the pardonee‘s consent to the terms stipulated in this contract. Jan. who in the first place was the exclusive author of the conditional pardon and of its revocation.A. warrants the same. solely vested in the Chief Executive. however erroneous the findings may be upon which his recommitment was ordered. In the same vein. Article VII of the 1987 Constitution is simply the source of power of the President to grant reprieves. or suspension of sentence. For instance. The determination of the necessity for the calling out power if subjected to unfettered judicial scrutiny could be a veritable prescription for disaster as such power may be unduly straitjacketed by an injunction or a temporary restraining order every time it is exercised. among others. where the insurgency problem could spill over the other parts of the country. But by no stretch of the imagination can the exercise by Congress of its plenary power to amend laws be considered as a violation of the President‘s power to commute final sentences of conviction. Ronaldo B. it is the unclouded intent of the Constitution to vest upon the President. he will be recommitted to prison to serve the unexpired portion of the sentence or an additional one . The suspension of such a death sentence is undisputably an exercise of judicial power. No. Secretary of Justice. is the corollary prerogative to reinstate the pardon if in his own judgment. commutations. and pardons and remit fines and forfeitures after conviction by final judgment. the Supreme Court was criticized on the ground. It has been our fortified ruling that a final judicial pronouncement as to the guilt of a pardonee is not a requirement for the President to determine whether or not there has been a breach of the terms of a conditional pardon. an accused who has been convicted by final judgment still possesses collateral rights and these rights can be claimed in the appropriate courts.‖ It is now a well-entrenched rule in this jurisdiction that this exercise of presidential judgment is beyond judicial scrutiny. The effect of such an amendment is like that of commutation of sentence. En Banc [Puno]) 92. in his judgment. 7659 by reducing the penalty of death to life imprisonment. Held: Section 19. The determination of the violation of the conditional pardon rests exclusively in the sound judgment of the Chief Executive. 2000. the acquittal of the pardonee from the subsequent charges filed against him. the decision to call out the military to prevent or suppress lawless violence must be done swiftly and decisively if it were to have any effect at all. Justify the SC's act. Such a scenario is not farfetched when we consider the present situation in Mindanao. the President's exercise of judgment deserves to be accorded respect from this Court. the pardonee has thereby placed himself under the supervision of the Chief Executive or his delegate who is duty-bound to see to it that the pardonee complies with the terms and conditions of the pardon. parole. shall fail to comply with the condition. No. Zamora. In truth. Xxx Ultimately. the Legislative and the Judiciary to save the life of a death convict do not exclude each other for the simple reason that there is no higher right than the right to life. 1999. full discretion to call forth the military when in his judgment it is necessary to do so in order to prevent or suppress lawless violence. En Banc [Kapunan]) 91. Indeed. 1987 Constitution. cannot be interpreted as denying the power of courts to control the enforcement of their decisions after the finality. 19. Article VII. G. Unless the petitioner can show that the exercise of such discretion was gravely abused. 141284. invasion or rebellion. This provision. Hon. The powers of the Executive. that it encroached on the power of the President to grant reprieve under Section 19. the Chief Executive is authorized to order ―the arrest and re-incarceration of any such person who. Aug. 15. however. a death convict who becomes insane after his final conviction cannot be executed while in a state of insanity (See Article 79 of the Revised Penal Code). Thus. In the exercise of the power to call. 301 SCRA 96.On the other hand. on-the-spot decisions may be imperatively necessary in emergency situations to avert great loss of human lives and mass destruction of property. or conditions of his pardon. as Commander-inChief of the Armed Forces. having consented to place his liberty on conditional pardon upon the judgment of the power that has granted it.R. Under Section 64(i) of the Revised Administrative Code. Discuss the nature of a conditional pardon. the President as Commander-in-Chief has a vast intelligence network to gather information. It is not usurpation of the presidential power of reprieve though its effect is the same – the temporary suspension of the execution of the death convict. it cannot be denied that Congress can at any time amend R. By issuing a TRO on the date convicted rapist Leo Echegaray is to be executed by lethal injection. (Integrated Bar of the Philippines v. (Echegaray v. some of which may be classified as highly confidential or affecting the security of the state. and recognized as such by the other contracting State. or facilities shall not be allowed in the Philippines except under a treaty duly concurred in by the Senate and. actually share some common ground. Article VII will find applicability with regard to the issue and 60 . when the Congress so requires. The role of the Senate is limited only to giving or withholding its consent. (In Re: Wilfredo Sumulong Torres.R. are deemed prohibitory in mandate and character. importation and exportation of equipment. in which case. troops. foreign military bases. Who has the power to ratify a treaty? Held: In our jurisdiction. in the legislature. troops. the provisions of Section 21. the concurrence of at least two-thirds (2/3) of all the Members of the Senate is required to make the subject treaty. the power to ratify is vested in the President and not. Article XVIII. 2000. and further defines the rights of the United States and the Philippine government in the matter of criminal jurisdiction. To a certain extent and in a limited sense. (BAYAN [Bagong Alyansang Makabayan] v. En Banc [Buena]) 94. To our mind. provides: ―After the expiration in 1991 of the Agreement between the Republic of the Philippines and the United States of America concerning Military Bases. Article XVIII. Section 21 opens with the clause ―No treaty x x x. requires the concurrence of the Senate to be valid and effective. coverage. or concurrence. On the whole.‖ Section 21.‖ Additionally. the concurrence of the Senate is indispensable to render the treaty or international agreement valid and effective. which specifically deals with treaties involving foreign military bases. No. regardless of subject matter. the fact that the President referred the VFA to the Senate under Section 21. or facilities. In particular.‖ Section 25.solely and absolutely in his office. troops or facilities in the Philippines. This provision lays down the general rule on treaties or international agreements and applies to any form of treaty with a wide variety of subject matter. however. far from contradicting each other. Article VII x x x reads: ―No treaty or international agreement shall be valid and effective unless concurred in by at least two-thirds of all the Members of the Senate. such as. valid and binding on the part of the Philippines. Article XVIII further requires that ―foreign military bases. Section 21. Article XVIII is a special provision that applies to treaties which involve the presence of foreign military bases. Undoubtedly. Oct. Article VII or Section 25. as commonly believed. Section 25. Article VII. is immaterial. Under this provision. in both instances. Dec. 1995 [Hermosisima]) 93. to the ratification. For in either case. 138570. ratified by a majority of the votes cast in a national referendum held for that purpose if so required by Congress. materials and supplies. 29. These constitutional provisions both embody phrases in the negative and thus. Article VII deals with treaties or international agreements in general. the VFA is an agreement which defines the treatment of United States troops and personnel visiting the Philippines. the fundamental law is crystalline that the concurrence of the Senate is mandatory to comply with the strict constitutional requirements. Article XVIII. troops.‖ and Section 25 contains the phrase ―shall not be allowed. Section 25. ratified by a majority of the votes cast by the people in a national referendum held for that purpose. It provides for the guidelines to govern such visits of military personnel. 251 SCRA 709. It is our considered view that both constitutional provisions. and that the Senate extended its concurrence under the same provision. All treaties or international agreements entered into by the Philippines. In contrast. the concurrence of the Senate is only one of the requisites to render compliance with the constitutional requirements and to consider the agreement binding on the Philippines. Which provision of the Constitution applies with regard to the exercise by the Senate of its constitutional power to concur with the Visiting Forces Agreement (VFA)? Held: The 1987 Philippine Constitution contains two provisions requiring the concurrence of the Senate on treaties or international agreements. Executive Secretary Ronaldo Zamora. or particular designation or appellation. G. should apply in the instant case. or international agreement. whether under Section 21. extradition or tax treaties or those economic in nature. movement of vessels and aircraft. but not limited to. Section 25. and recognized as a treaty by the other contracting State. 10. or facilities‖ may be allowed in the Philippines only by virtue of a treaty duly concurred in by the Senate. Oct. Will it be correct to argue that since no foreign military bases. As to the matter of voting. It is a finely-imbedded principle in statutory construction that a special provision or law prevails over a general one. Lex specialis derogat generali. we do not subscribe to the argument that Section 25. be upheld? Held: It is specious to argue that Section 25. Article XVIII that requires foreign troops or facilities to be stationed or placed permanently in the Philippines. 2000. XVIII of the 1987 Constitution complied with when the Senate gave its concurrence to the VFA? Held: Section 25. Oct. En Banc [Buena]) 95. These military warships are actually used as substitutes for a land-home base not only of military aircraft but also of military personnel and facilities. (BAYAN [Bagong Alyansang Makabayan] v. must be concurred in by at least two-thirds of all the 61 . guided missiles as well as huge sea vessels that can stay afloat in the sea even for months and years without returning to their home country. Art. are involved in the VFA. military bases established within the territory of another state is no longer viable because of the alternatives offered by new means and weapons of warfare such as nuclear weapons. 138570 and Companion Cases. troops. Executive Secretary Ronaldo Zamora. therefore. the provision contemplates three different situations .R. when so required by Congress.R. the Court should not distinguish . Section 21. No. 10. No. Article XVIII of the Constitution is not controlling? Held: In like manner. but merely troops and facilities. Should the contention that since the VFA merely involved the temporary visits of United States personnel engaged in joint military exercises and not a basing agreement. Sec. 342 SCRA 449. G. Executive Secretary Ronaldo Zamora. Were the requirements of Sec.any of the three standing alone places it under the coverage of Section 25. but merely foreign troops and facilities. a perusal of said constitutional provision reveals that the proscription covers ―foreign military bases. or (c) foreign facilities . There is no dispute as to the presence of the first two requisites in the case of the VFA. G. 138570 and Companion Cases. the intention of the framers of the Charter x x x is consistent with this interpretation Moreover. viz: (a) it must be under a treaty. or facilities in the country. 481-492. 138570 and Companion Cases. En Banc [Buena]) 96.R.‖ Stated differently. The use of comma and disjunctive word "or" clearly signifies disassociation and independence of one thing from the others included in the enumeration. No. To this end. or facilities.a military treaty the subject of which could be either (a) foreign bases (b) foreign troops. Section 25. to be valid and effective. (b) the treaty must be duly concurred in by the Senate and. Notably. XVIII of the Constitution is inapplicable to the VFA. we find nothing in Section 25. therefore. the Constitution makes no distinction between ―transient‖ and ―permanent‖. whether under the general requirement in Section 21. this prohibition is not limited to the entry of troops and facilities without any foreign bases being established. troops. G. are involved in the VFA. Article XVIII is inapplicable to mere transient agreements for the reason that there is no permanent placing of structure for the establishment of a military base. 2000. 18 is in accordance with the provisions of the Constitution. 10. 342 SCRA 449. unless the following conditions are sufficiently met. Article VII particularly requires that a treaty or international agreement. 10. the provision in the latter article requiring ratification by a majority of the votes cast in a national referendum being unnecessary since Congress has not required it. Executive Secretary Ronaldo Zamora. ratified by a majority of the votes cast by the people in a national referendum. (BAYAN [Bagong Alyansang Makabayan] v.Ubi lex non distinguit nec nos distinguire debemos. or the specific mandate mentioned in Section 25. and (c) recognized as a treaty by the other contracting state. Art. 25. Certainly. Article XVIII is not controlling since no foreign military bases. Article XVIII disallows foreign military bases. troops. (BAYAN [Bagong Alyansang Makabayan] v. Besides. vessels are mobile as compared to a land-based military headquarters. Article XVIII. Article XVIII. It is a rudiment in legal hermeneutics that when no distinction is made by law.for the sole purpose of determining the number of votes required to obtain the valid concurrence of the Senate x x x. 25. such that. On this score. The clause does not refer to ―foreign military bases. Article VII. 481-492. En Banc [Buena]) 97. Oct. The concurrence handed by the Senate through Resolution No. x x x. 481-492. or facilities‖ collectively but treats them as separate and independent subjects. 2000. 342 SCRA 449. (BAYAN [Bagong Alyansang Makabayan] v. is that the ratification. in the instant case . and with the exchange of notes between the Philippines and the United States of America. the said agreement is to be taken equally as a treaty. While it is true that Section 25. under international law. to submit the VFA to the United States Senate for concurrence pursuant to its Constitution. Well-entrenched is the principle that the words used in the Constitution are to be given their ordinary meaning except where technical terms are employed. for brevity).members of the Senate. G.the VFA in the instant case. Section 25. Article VII. Moreover. with the concomitant duty to uphold the obligations and responsibilities embodied thereunder. Article XVIII are present. requires that the concurrence of a treaty. Executive Secretary Ronaldo Zamora. xxx Having resolved that the first two requisites prescribed in Section 25.the VFA. Are the ―Balikatan‖ exercises covered by the Visiting Forces Agreement? Held: The holding of ―Balikatan 02-1‖ must be studied in the framework of the treaty antecedents to which the Philippines bound itself. be made by a two-thirds vote of all the members of the Senate. xxx With the ratification of the VFA. there is indeed marked compliance with the mandate of the Constitution. No. it now becomes obligatory and incumbent on our part. 2000. The United States of America in this case. 342 SCRA 449. is to accord strict meaning to the phrase. among other things.be ―duly concurred in by the Senate. Article XVIII requires. The first of these is the Mutual Defense Treaty (MDT. Oct. Article XVIII must not be treated in isolation to Section 21. it is inconsequential whether the United States treats the VFA only as an executive agreement because. of the VFA and the concurrence of the Senate should be taken as a clear and unequivocal expression of our nation's consent to be bound by said treaty. the ―concurrence requirement‖ under Section 25. As noted. En Banc [Buena]) 98. as long as the VFA possesses the elements of an agreement under international law. which in more specific terms. Its language should be understood in the sense they have in common use. 138570 and Companion Cases. For as long as the United States of America accepts or acknowledges the VFA as a treaty. Article XVIII must be construed in relation to the provisions of Section 21. To be sure.‖ it is very true however that said provision must be related and viewed in light of the clear mandate embodied in Section 21. the concurrence of the Senate contemplated under Section 25. Article VII.R. Article XVIII means that at least two-thirds of all the members of the Senate favorably vote to concur with the treaty . by the President. Section 25. Worth stressing too. in which case the significance thus attached to them prevails. we shall now pass upon and delve on the requirement that the VFA should be recognized as a treaty by the United States of America. To require the other contracting state. Hubbard. Article VII. has stated that the United States government has fully committed to living up to the terms of the VFA. In a more particular language. Indeed. to be bound by the terms of the agreement. The MDT has been described as the ―core‖ of the defense relationship between the 62 . On the other hand. 10. that the treaty . and binds itself further to comply with its obligations under the treaty. under the principles of international law. xxx The records reveal that the United States Government. which is equivalent to final acceptance. 481-492. or international agreement. a two-thirds vote of all the members of the Senate is clearly required so that the concurrence contemplated by law may be validly obtained and deemed present.‖ Applying the foregoing constitutional provisions. Article XVIII simply provides that the treaty be ―duly concurred in by the Senate. xxx This Court is of the firm view that the phrase ―recognized as a treaty‖ means that the other contracting party accepts or acknowledges the agreement as a treaty. an executive agreement is as binding as a treaty . through Ambassador Thomas C. G. al. (Arthur D. The proponents insist that a faithful execution of the laws requires that the President desist from implementing the law if doing so would prejudice public interest. on an impermanent basis. by a vote of eleven to three.R. disaster relief operations. civic action projects such as the building of school houses. 2000. however. which it refers to as the context of the treaty. In our view. What is the power of impoundment of the President? What are its principal sources? Held: Impoundment refers to the refusal of the President. driving and vehicle registration. from any political activity. since the terminology employed is itself the source of the problem. al. The VFA permits United States personnel to engage. The sole encumbrance placed on its definition is couched in the negative.. It is the failure to spend or obligate budget authority of any type. It is the VFA which gives continued relevance to the MDT despite the passage of years. which is presumed to verbalize the parties‘ intentions. state x x x. it is necessary to refer to the VFA itself. As conceived. 342 SCRA 449 [2000]). in other words. Zamora. 151445. Lim and Paulino R. The VFA provides the ―regulatory mechanism‖ by which ―United States military and civilian personnel [may visit] temporarily in the Philippines in connection with activities approved by the Philippine Government.‖ the exact meaning of which was left undefined. it appeared farfetched that the ambiguity surrounding the meaning of the word ―activities‖ arose from accident. It is this treaty to which the VFA adverts and the obligations thereunder which it seeks to reaffirm. It is only logical to assume that ―Balikatan 02-1. this Court upheld the validity of the VFA (BAYAN. The Convention likewise dictates what may be used as aids to deduce the meaning of terms. as well as the duration of the agreement and its termination. Its primary goal is to facilitate the promotion of optimal cooperation between American and Philippine military forces in the event of an attack by a common foe. unfortunately. importation and exportation. Third is the Faithful Execution Clause. et. the VFA gives legitimacy to the current Balikatan exercises. April 11. are fair game. the United States. 2002. in ―activities. En Banc [De Leon]) 99. In this manner. visiting US forces may sojourn in Philippine territory for purposes other than military. Honorable Executive Secretary. until it was replaced by the Visiting Forces Agreement. Proponents of impoundment have invoked at least three principal sources of the authority of the President. The lapse of the US-Philippine Bases Agreement in 1992 and the decision not to renew it created a vacuum in US-Philippine defense relations. assisting and training exercise. We are not completely unaided. X x x The Terms of Reference rightly fall within the context of the VFA. for whatever reason. Ersando v. Foremost is the authority to impound given to him either expressly or impliedly by Congress. which contains provisos governing interpretations of international agreements. as well as other elements may be taken into account alongside the aforesaid context.‖ All other activities. v. sea search-and-destroy operations to assist vessels in distress. that is.Philippines and its traditional ally. the ―Balikatan‖ is the largest such training exercise directly supporting the MDT‘s objectives. After studied reflection. No. movement of vessels and aircraft. in that United States personnel must ―abstain from any activity inconsistent with the spirit of this agreement. it was deliberately made that way to give both parties a certain leeway in negotiation. claims. and in particular. The first question that should be addressed is whether ―Balikatan 02-1‖ is covered by the Visiting Forces Agreement. are indeed authorized.‖ It contains provisions relative to entry and departure of American personnel. Both the history and intent of the Mutual Defense Treaty and the VFA support the conclusion that combat-related activities – as opposed to combat itself – such as the one subject of the instant petition. The Vienna Convention on the Law of Treaties. The expression is ambiguous. An example given is when through 63 . criminal jurisdiction. et. Not much help can be had therefrom. Under these auspices. medical and humanitarian missions. permitting a wide scope of undertakings subject only to the approval of the Philippine government. It is clear from the foregoing that the cardinal rule of interpretation must involve an examination of the text. It should be recalled that on October 10. To resolve this. to spend funds made available by Congress. and the like. Second is the executive power drawn from the President‘s role as Commander-in-Chief. the joint exercises may include training on new techniques of patrol and surveillance to protect the nation‘s marine resources.‖ a ―mutual anti-terrorism advising. Its aim is to enhance the strategic and technological capabilities of our armed forces through joint training with its American counterparts.‖ falls under the umbrella of sanctioned or allowable activities in the context of the agreement. if Benipayo is the lawful COMELEC Chairman because he assumed office in accordance with the Constitution. et al.‖ Evidently. clothes her with the requisite locus standi to raise the constitutional issue in this petition. in Isagani Cruz v. No. 141284. respondents‘ anchor the legality of petitioner‘s reassignment on Benipayo‘s authority as Chairman of the COMELEC. the constitutionality of the ad interim appointments is not the lis mota of this case. where she was Acting Director. We are not persuaded. G. Kapunan. Clearly. Borra and Tuason unless all the four requisites are present. Even if petitioner is only an Acting director of the EID. namely: (1) the existence of an actual and appropriate case. En Banc) 2.R. (3) the plea that the function be exercised at the earliest opportunity. and (4) the constitutional question is the lis mota of the case. Neither does petitioner claim to be directly injured by the appointments of these three respondents. 6. Hon. 64 . 1994 [Quiason]) The Judicial Department 100. 2000. and (4) the necessity that the constitutional question be passed upon in order to decide the case (Separate Opinion. 235 SCRA 506. 2001 despite the fact that the ad interim appointments of Benipayo. Out of respect for the acts of the Executive department. What are the requisites before the Court can exercise the power of judicial review? Held: 1. (Integrated Bar of the Philippines v. On the other hand. Dec. it is sheer folly to expect the President to spend the entire amount budgeted in the law. Borra and Tuason were issued as early as March 22. Respondents insist that the real issue in this case is the legality of petitioner‘s reassignment from the EID to the Law Department. In such a case. Zamora. Borra and Tuason and their assumption of office. 2001. Moreover. Respondents also contend that petitioner failed to question the constitutionality of the ad interim appointments at the earliest opportunity. (3) the exercise of judicial review is pleaded at the earliest opportunity. Consequently. respondents urge this Court to refrain from reviewing the constitutionality of the ad interim appointments issued by the President to Benipayo. Respondents maintain that petitioner does not have a personal and substantial interest in the case because she has not sustained a direct injury as a result of the ad interim appointments of Benipayo. to the Law Department. When questions of constitutional significance are raised. petitioner has a personal and material stake in the resolution of the constitutionality of Benipayo‘s assumption of office.R. then petitioner‘s reassignment is legal and she has no cause to complain provided the reassignment is in accordance with the Civil Service Law. 2000. Enriquez. Respondents claim that the reassignment was ―pursuant to x x x Benipayo‘s authority as Chairman of the Commission on Elections. Aug. Petitioner‘s personal and substantial injury. The time-tested standards for the exercise of judicial review are: (1) the existence of an appropriate case. (2) a personal and substantial interest of the party raising the constitutional question. and as the Commission‘s Chief Executive Officer. What are the requisites for the proper exercise of the power of judicial review? Illustrative case. (PHILCONSA v.. if Benipayo is not the lawful COMELEC Chairman. Secretary of Environment and Natural Resources. 15. which is co-equal with this Court. third and fourth requisites are absent in this case. where she was placed on detail. Respondents point out that petitioner does not claim to be lawfully entitled to any of the positions assumed by Benipayo. Petitioner filed the petition only on August 3. an office created by the Constitution. the petition was filed after the third time that these three respondents were issued ad interim appointments. Aug. Borra or Tuason. her reassignment is without legal basis if Benipayo is not the lawful COMELEC Chairman. the Court can exercise its power of judicial review only if the following requisites are complied with. No. 9. substantial savings are made. X x x Respondents argue that the second. J. The real issue then turns on whether or not Benipayo is the lawful Chairman of the COMELEC. En Banc [Kapunan]) 101. 135385. Benipayo reassigned petitioner from the EID. Held: Respondents assert that the petition fails to satisfy all the four requisites before this Court may exercise its power of judicial review in constitutional cases.efficient and prudent management of a project. (2) an interest personal and substantial by the party raising the constitutional question. G.. Ronaldo B. Here the petitioner has complied with all the requisite technicalities. It cannot be successfully maintained that we should await the adverse consequences of the law in order to consider the controversy actual and ripe for judicial resolution. Petitioner filed the instant petition only on August 3. Zamora. the time when a constitutional issue may be passed upon . ―if it is not raised in the pleadings. Ronaldo B. Xxx In the case at bar. et al. However. and that which is not conjectural or anticipatory. A petition raising a constitutional question does not present an ―actual controversy. En Banc [Carpio]) 102.Respondents harp on petitioner‘s belated act of questioning the constitutionality of the ad interim appointments of Benipayo. the issue raised by petitioner is of paramount importance to the public. otherwise known as the Indigenous Peoples Rights Act of 1997 (IPRA). therefore. Moreover. on its face. Held: Courts can only decide actual controversies. when the first ad interim appointments were issued as early as March 22. A law has been enacted. 2002.R. which is the earliest opportunity for pleading the constitutional issue before a competent body. April 2. assailing the constitutionality of certain provisions of Republic Act No. this Court may even brush aside technicalities of procedure and resolve any constitutional issue raised . What is an ―actual case or controversy‖? Held: An ―actual case or controversy‖ means an existing case or controversy which is both ripe for resolution and susceptible of judicial determination. G. the lis mota of this case is the very constitutional issue raised by petitioner. (Integrated Bar of the Philippines v. is whether an ―appropriate case‖ exists for the exercise of judicial review in the present case. Benipayo. In keeping with this Court‘s duty to determine whether other agencies of government have remained within the limits of the Constitution and have not abused the discretion given them. Otherwise. It is precisely the contention of the petitioners that the law. there exists a live controversy involving a clash of legal rights. and the Implementing Rules and Regulations approved. 2001 national elections may be put in doubt if the constitutional issue raised by petitioner is left unresolved. constitutes an unconstitutional abdication of State ownership over lands of the public domain and other natural resources. 380 SCRA 49.‖ unless it alleges a legal right or power. The controversy must also be justiciable. the legality of petitioner‘s reassignment hinges on the constitutionality of Benipayo‘s appointment and assumption of office is resolved. Kapunan. it cannot be considered on appeal. J. this Court may determine. ad interim appointment and assumption of office. Money has been appropriated and the government agencies concerned have been directed to implement the statute. Unless the constitutionality of Benipayo‘s ad interim Moreover. Clearly. in the exercise of sound discretion. such that. Moreover. Petitioners Isagani Cruz and Cesar Europa brought a suit for prohibition and mandamus as citizens and taxpayers. it must be susceptible of judicial determination. 141284. Furthermore. G. Borra and Tuason. it is not the date of filing of the petition that determines whether the constitutional issue was raised at the earliest opportunity. Aug. 135385. The legality of the directives and decisions made by the COMELEC in the conduct of the May 14. 8371. 2001. Moreover. En Banc) 65 .‖ Petitioner questioned the constitutionality of the ad interim appointments of Benipayo. public interest requires the resolution of the constitutional issue raised by petitioner. (Separate Opinion. No. it must show that a conflict of rights exists. the legality of petitioner‘s reassignment from the EID to the Law Department cannot be determined. There is no doubt petitioner raised the constitutional issue on time. 2000. when the State machinery is set into motion to implement an alleged unconstitutional statute.. for inherent in the term ―controversy‖ is the presence of opposing views or contentions. Dec. In any event. in Isagani Cruz v. Hon. A preliminary issue resolved by the SC was whether the petition presents an actual controversy. that is.R. En Banc [Kapunan]) 103. 2001. or that which seeks to resolve hypothetical or feigned constitutional problems. Borra and Tuason when she filed her petition before this Court. No. 6.. (Matibag v. the Court will be forced to resolve issues which remain unfocused because they lack such concreteness provided when a question emerges precisely framed from a clash of adversary arguments exploring every aspect of a multi-faceted situation embracing conflicting and demanding interests. 2000. Secretary of Environment and Natural Resources. this Court possesses sufficient authority to resolve and prevent imminent injury and violation of the constitutional process. and its Implementing Rules and Regulations. The threshold issue. The earliest opportunity to raise a constitutional issue is to raise it in the pleadings before a competent court that can resolve the same. 15. not hypothetical questions or cases . 2000.‖ We shall adhere to this view and proceed to dwell on the merits of this petition. create. (University of San Agustin. Respondents‘ submission will create a dangerous precedent. the petition of the Municipality of Cainta has already been rendered moot and academic. if indeed. (Gonzales v. Subsequent events have overtaken the petition and the Court has nothing left to resolve. Should the Court still resolve the case despite that the issue has already become moot and academic? Exception. 770. the PCCR was instructed to complete its task on or before June 30. on February 19. prohibition is an inappropriate remedy since the body sought to be enjoined no longer exists. The petition which was filed by private respondents before the trial court sought the issuance of a writ of mandamus. 1999. 314 SCRA 179. as there has been fait accompli. 1994 [Nocon]) 105. COMELEC. However. Should this Court decline now to perform its duty of interpreting and indicating what the law is and should be. 70). 230 SCRA 761. petitioner asks this Court to enjoin the PCCR from acting as such. any ruling regarding the PCCR would simply be in the nature of an advisory opinion. faced by an ―angry and violent mob armed with explosives. President Macapagal-Arroyo. Clearly. a statement of the governing principle is appropriate in the resolution of dismissal for the guidance not only of the parties but of others similarly situated. firearms. divide and/or alter the boundaries of political subdivisions. Narvasa. to command petitioners to admit them for enrolment. On May 1.‖ (City of Pasig v. which extended the time frame for the completion of the commission‘s work x x x. with respect to the PCCR [Preparatory Commission on Constitutional Reform]. either brazenly or stealthily. Thus. En Banc [Gonzaga-Reyes]) 2. Sept. Court of Appeals. 1999 and was dissolved by the President on the same day. as respondents so propose is a proposition fraught with mischief. which is definitely beyond the permissible scope of judicial power. The issue raised by the Municipality of Cainta in its petition before the COMELEC against the holding of the plebiscite for the creation of Barangay Napico are still pending determination before the Antipolo Regional Trial Court.O. It is alleged by respondent that. confident that this Court will abstain from entertaining future challenges to their acts if they manage to bring about a fait accompli. 14. When is an action considered ―moot‖? May the court still resolve the case once it has become moot and academic? Held: 1.104. Held: Neither do we agree that merely because a plebiscite had already been held in the case of the proposed Barangay Napico. 70 (E. stones and other deadly weapons‖ assaulting and 66 . Commission on Elections.O. Inc. we struck down the moot and academic argument as follows – ―Considering that the legality of the plebiscite itself is challenged for non-compliance with constitutional requisites. In Tan v. deserves to be inquired into by this Tribunal so that. No. v. Dans is the authority for the view that ―even if a case were moot and academic. 1999. No. illegality attaches to its creation. 43. It had likewise spent the funds allocated to it. the case of Eastern Broadcasting Corporation (DYRE) v. An action is considered ―moot‖ when it no longer presents a justiciable controversy because the issues involved have become academic or dead. It is well-established that prohibition is a preventive remedy and does not lie to restrain an act that is already fait accompli. this might tempt again those who strut about in the corridors of power to recklessly and with ulterior motives. 337 SCRA 733. For this Court to yield to the respondents‘ urging that. 2001. Aug. then this Court should passively accept and accede to the prevailing situation is an unacceptable suggestion. this case has become moot and academic. 10. Dismissal of the instant petition. Basically. the PCCR has ceased to exist. The PCCR submitted its recommendations to the President on December 20. the case before Us cannot truly be viewed as already moot and academic. Taking into account the admission of private respondents that they have finished their Nursing course at the Lanting College of Nursing even before the promulgation of the questioned decision. clubs. bladed weapons. 1999. At this point. We agree. Continuation of the existence of this newly proclaimed province which petitioners strongly profess to have been illegally born. However. having lost its raison d‘ être. The staleness of the issue before us is made more manifest by the impossibility of granting the relief prayed for by petitioner. the commission of that error should not provide the very excuse for perpetration of such wrong. merge. the President issued Executive Order No. En Banc [Ynares-Santiago]) 106. this case has clearly been overtaken by events and should therefore be dismissed. March 7. Under E. the fact that such plebiscite had been held and a new province proclaimed and its officials appointed. . 6. Dec. President Macapagal-Arroyo ordered the lifting of the declaration of a ―state of rebellion‖ in Metro Manila. that it might cause disorder and confusion considering the randomness of selecting interviewees. Secretary of Environment and Natural Resources. he has sustained. the Secretary of Justice denies that it has issued a particular order to arrest specific persons in connection with the ―rebellion. May 10. the COMELEC issued a resolution prohibiting the conduct of exit polls on the ground. To set aside the resolution of the issue now will only postpone a task that could well crop up again in future elections. Kapunan. (Lacson v. In connection with the May 11. She likewise issued General Order No.‖ Since the fundamental freedoms of speech and of the press are being invoked here. 357 SCRA 756.R. The constitutionality of this resolution was challenged by ABS-CBN Broadcasting Corporation as violative of freedom of expression. issued Proclamation No. En Banc) 109. et al. Accordingly. Zamora. 2000) 2. its implications on the people‘s fundamental freedom of expression transcend the past election. as distinguished from mere interest in the question involved. x x x. in Isagani Cruz v. Hence. which further makes the exit polls unreliable.attempting to break into Malacanang.. Warrantless arrests of several alleged leaders and promoters of the ―rebellion‖ were thereafter effected. With this declaration.‖ (Integrated Bar of the Philippines v. 15. 141284. (Separate Opinion. Aug. the violation of which may result only in a ―generalized grievance‖. the rights asserted by petitioners as citizens and taxpayers are held in common by all the citizens. In any event. 38 declaring that there was a state of rebellion in the National Capital Region. in Salonga v. As to petitioners‘ claim that the proclamation of a ―state of rebellion‖ is being used by the authorities to justify warrantless arrests. 2001. The term ―interest‖ means a material interest. several petitions were filed before the SC assailing the declaration of State of Rebellion by President Gloria Macapagal-Arroyo and the warrantless arrests allegedly effected by virtue thereof. 2000. 28. Cruz Pano. in a sense. The Solicitor General contends that the petition has been rendered moot and academic because the May 11. there is no longer any actual controversy to be resolved. all citizen's and taxpayer's suits are efforts to air generalized grievances about the conduct of government and the allocation of power. It has the symbolic function of educating bench and bar on the extent of protection given by constitutional guarantees. 2001. The gist of the question of standing is whether a party alleges ―such personal stake in the outcome of the controversy as to assure that concrete adverseness which sharpens the presentation of issues upon which the court depends for illumination of difficult constitutional questions.R. 1998 elections. ―Legal standing‖ or locus standi has been defined as a personal and substantial interest in the case such that the party has sustained or will sustain direct injury as a result of the governmental act that is being challenged. Significantly. Yet.R. petitioners‘ apprehensions as to warrantless arrests should be laid to rest. the instant petitions have been rendered moot and academic. No. an interest in issue affected by the decree. Held: All the foregoing petitions assail the declaration of state of rebellion by President Gloria Macapagal-Arroyo and the warrantless arrests allegedly effected by virtue thereof. a person who assails the validity of a statute must have a personal and substantial interest in the case. En Banc [Panganiban]) 108. G. such that. What is the meaning of "legal standing" or locus standi? Held: 1. 38. G. 1998 election has already been held and done with and. Asserting itself as the official organization of Filipino lawyers tasked with the bounden duty to uphold the rule of law and the Constitution. J. or rules. No. Resolve. Perez. 135385. The holding of periodic elections is a basic feature of our democratic government. among others. No. COMELEC. Hon. Jan. a direct injury as a result of its enforcement. the Integrated Bar of the Philippines (IBP) filed a petition before the SC questioning the validity of the order of the President commanding the deployment and utilization of the Philippine Marines to assist the Philippine National Police (PNP) in law enforcement 67 . 1 directing the Armed Forces of the Philippines and the Philippine National Police to suppress the rebellion in the National Capital Region. whether they likewise protect the holding of exit polls and the dissemination of data derived therefrom. 1998 election. the Court had occasion to reiterate that it ―also has the duty to formulate guiding and controlling constitutional principles. therefore. we have resolved to settle. Evidently. En Banc [Melo]) 107. exit polling is tied up with elections. Held: While the assailed Resolution referred specifically to the May 11. Ronaldo B. 2000. (ABS-CBN Broadcasting Corporation v. or a mere incidental interest. G. on May 6. as having no basis both in fact and in law. precepts. for the guidance of posterity. doctrines. By its very nature. 133486. In addition to the existence of an actual case or controversy. or will sustain.‖ He states that what is extant are general instructions to law enforcement officers and military agencies to implement Proclamation No. the IBP asserts no other basis in support of its locus standi. therefore. Inc. Discuss the nature of a taxpayer‘s suit. 141284. a law is violated or some irregularity is committed. none of its members x x x has sustained any form of injury as a result of the operation of the joint visibility patrols.R. therefore. but by concerned citizens. especially when important issues are involved. because peace and order are under constant threat and lawless violence occurs in increasing tempo. the IBP primarily anchors its standing on its alleged responsibility to uphold the rule of law and the Constitution. which would make this petition a taxpayer's suit is absent. undoubtedly aggravated by the Mindanao insurgency problem. It. 2000. the Court may brush aside technicalities of procedure. is to elevate the standards of the law profession and to improve the administration of justice is alien to. two requisites must be met. novelty and weight as precedents. En Banc [Kapunan]) 110. Rule 139-A of the Rules of Court. Indeed. it does not possess the personality to assail the validity of the deployment of the Marines. Petitioner and respondents agree that to constitute a taxpayer's suit. Considering the lack of requisite standing of the IBP to file the petition questioning the validity of the order of the President to deploy and utilize the Philippine Marines to assist the PNP in law enforcement. Petitioner never referred to such purchase as an illegal disbursement of public funds but focused on the alleged fraudulent reconveyance of said property to Ortigas because the price paid was lower than the prevailing market value of neighboring lots. It will stare us in the face again. Not only is the presumed ―injury‖ not personal in character. v. the Court has adopted a liberal attitude on the locus standi of a petitioner where the petitioner is able to craft an issue of transcendental significance to the people. the legal controversy raised in the petition almost certainly will not go away. the IBP x x x has not shown any specific injury which it has suffered or may suffer by virtue of the questioned governmental act. Guingona. where the Court also reiterated its liberal stance in entertaining so-called taxpayer's suits. highly speculative and uncertain to satisfy the requirement of standing. G. when the issues raised are of paramount importance to the public. Ronaldo B. x x x Moreover. G. The first requirement. (Integrated Bar of the Philippines v. directly affected by the act complained of. No. 674. rather than later. under Section 2. Neither is it alleged that any of its members has been arrested or that their civil liberties have been violated by the deployment of the Marines. A closer examination of the facts of this case would readily demonstrate that petitioner's standing should not even be made an issue here. What the IBP projects as injurious is the supposed ―militarization‖ of law enforcement which might threaten Philippine democratic institutions and may cause more harm than good in the long run.. disbursement of public funds was only made in 1975 when the Province bought the lands from Ortigas at P110. a reading of the petition shows that the IBP has advanced constitutional issues which deserve the attention of this Court in view of their seriousness.by joining the latter in visibility patrols around the metropolis.‖ In the case at bar. Inc. v. 2000) 111. the IBP has failed to present a specific and substantial interest in the resolution of the case. Moreover. taxpayers or voters who actually sue in the public interest. Zamora. (Integrated Bar of the Philippines v. and that the petitioner is directly affected by the alleged ultra vires act . This is too general an interest which is shared by other groups and the whole citizenry. Zamora. behooves the Court to relax the rules on standing and to resolve the issue now. Since petitioner has not successfully established a direct and personal injury as a consequence of the questioned act. that public funds are disbursed by a political subdivision or instrumentality and in doing so. however. Aug. In not a few cases. In this case. Its fundamental purpose which.D. Aug. The mere invocation by the IBP of its duty to preserve the rule of law and nothing more. The same pronouncement was made in Kilosbayan. Hence the question in standing is whether such parties have 'alleged such a personal 68 . or would be. 15. while undoubtedly true.00 per square meter in line with the objectives of P. Jr. Held: In the case at bar. Hon. namely. 141284. The only remaining justification for petitioner to be allowed to pursue this action is whether it is. No. and cannot be affected by the deployment of the Marines. When may it be allowed? Held: 1. Apart from this declaration. ―since standing is a concept in constitutional law and here no constitutional question is actually involved.R. ―Standing is a special concern in constitutional law because in some cases suits are brought not by parties who have been personally injured by the operation of a law or by official action taken. Thus. Morato. Based on the standards above-stated. may the Court still properly take cognizance of the case? Held: Having stated the foregoing. is not sufficient to clothe it with standing in this case. As we stated in Kilosbayan. Hon. it is likewise too vague. 15. The Solicitor General questioned the legal standing of the IBP to file the petition? Resolve. Ronaldo B. it must be emphasized that this Court has the discretion to take cognizance of a suit which does not satisfy the requirement of legal standing when paramount interest is involved. P. Inc.‖ Coming now to the instant case. Inc. it is readily apparent that there is no exercise by Congress of its taxing or spending power.O. as amended by E. Commission on Elections. in Bugnay Construction and Development Corporation v. the Court allowed petitioner to maintain a taxpayer‘s suit assailing the constitutional soundness of Republic Act No. No. 369 U. upon the theory that the expenditure of public funds by an officer of the State for the purpose of executing an unconstitutional act constitutes a misapplication of such funds. 881 (otherwise known as the ―Omnibus Election Code‖) requiring radio and television stations to give free air time to the Commission on Elections during the campaign period (Telecommunications and Broadcast Attorneys of the Philippines. Under Section 7 of E.‘ while appropriation made by law refers to ‗the act of the legislature setting apart or assigning to a particular use a certain sum to be used in the payment of debt or dues from the State to its creditors. The Court held that petitioner TELEBAP did not have any interest as a taxpayer since the assailed law did not involve the taxing or spending power of Congress. At the instance of taxpayers. Secretary of Public Works. Meanwhile.stake in the outcome of the controversy as to assure that concrete adverseness which sharpens the presentation of issues upon which the court so largely depends for illumination of difficult constitutional questions. however.O. Inc. 2d 633 [1962])‖ Undeniably. petitioner has absolutely no cause of action. In fact. (The Anti-Graft League of the Philippines. 43. ―In a strict sense. When. a taxpayer‘s action is properly brought only when there is an exercise by Congress of its taxing or spending power. the petitioners therein were allowed to bring a taxpayer‘s suit to question several presidential decrees promulgated by then President Marcos in his legislative capacity calling for a national referendum. 260 SCRA 250. in the instant case. the amount of P3 million is ―appropriated‖ for its operational expenses ―to be sourced from the funds of the Office of the President. 3836 insofar as it provides for retirement gratuity and commutation of vacation and sick leaves to Senators and Representatives and to the elective officials of both houses of Congress (Philippine Constitution Association. with the Court explaining that – X x x [i]t is now an ancient rule that the valid source of a statute – Presidential Decrees are of such nature – may be contested by one who will sustain a direct injury as a result of its enforcement. In other words. San Juan. the lease contract entered into between petitioner and the City shows that no public funds have been or will be used in the construction of the market building. 70. 991 carries an appropriation of Five Million Pesos for the effective implementation of its purposes.000 for the construction. The PCCR was created by the President by virtue of E. the Court declared that the trial court was wrong in allowing respondent Ravanzo to bring an action for injunction in his capacity as a taxpayer in order to question the legality of the contract of lease covering the public market entered into between the City of Dagupan and petitioner. In still another case. 253-255. cannot question the transaction validly executed by and between the Province and Ortigas for the simple reason that it is not privy to said contract. Carr. 43. The Court declared that Ravanzo did not possess the requisite standing to bring such taxpayer‘s suit since ―[o]n its face. repair and improvement of feeder roads within private property. En Banc [Romero]) 2.. Inc. And in Pascual v. there was no appropriation at all. Many other rulings have premised the grant or denial of standing to taxpayers upon whether or not the case involved a disbursement of public funds by the legislature. No. v. 7 L. A taxpayer is deemed to have the standing to raise a constitutional issue when it is established that public funds have been disbursed in alleged contravention of the law or the Constitution . Aug. v. Gimenez). a nonprofit civic organization – had standing as taxpayers to question the constitutionality of Republic Act No.S. This was our ruling in a recent case wherein petitioners Telecommunications and Broadcast Attorneys of the Philippines (TELEBAP) and GMA Network. and there is no evidence to the contrary. as a taxpayer. Thus. Inc. v. The breadth of Presidential Decree No. No. Presidential Decree No. not by Congress.‘‖ The funds used for the PCCR were taken from funds intended for the Office of the President.O. appropriation has been defied ‗as nothing more than the legislative authorization prescribed by the Constitution that money may be paid out of the Treasury. in the 69 . laws providing for the disbursement of public funds may be enjoined. 1. The interest of the aforenamed petitioners as taxpayers in the lawful expenditure of these amounts of public money sufficiently clothes them with that personality to litigate the validity of the Decrees appropriating said funds x x x. 289 SCRA 337 [1998]). the Court held that petitioners – the Philippine Constitution Association. petitioner.' (Citing Baker v. Ed. 1031 appropriates the sum of Eight Million Pesos to carry out its provisions. questioned the validity of Section 92 of B. 186. 920 appropriating P85. In Sanidad v. All these cases involved the disbursement of public funds by means of a law. Laron.‖ x x x The appropriations for the PCCR were authorized by the President. and consequently no locus standi. 1996. petitioner would somehow be adversely affected by an illegal use of public money. even as a taxpayer. Blg. Commission on Elections. no such unlawful spending has been shown x x x. Suffice it to say that by the petitioners‘ own admission that the respondents are still processing and have not approved the application of the heirs of Carantes. The petitioners questioned the regulation because of the possibility that the permit might be denied them in the future. Held: From a reading of the records it appears to us that the petition was prematurely filed. there being no exercise by Congress of its taxing or spending power. the petitioners are simply speculating that they might be evicted from the premises at some future time. (Gonzales v. but rather. the petitioners alleged right to possess the land is not violated nor is in imminent danger of being violated. they are not entitled to relief. 14. Subject to certain well-defined exceptions courts will not touch an issue involving the validity of a law unless there has been a governmental act accomplished or performed that has a direct adverse effect on the legal right of the person contesting its validity . as the DENR may or may not approve Carantes‘ application. A mere apprehension that the Secretary of Education will withdraw the permit does not amount to justiciable controversy. and it would appear that the petitioners did not verify if indeed the respondent DENR or its officers authorized the attempted eviction. ―a definite and concrete dispute touching on the legal relations of parties having adverse legal interests‖ which may be resolved by a court of law through the application of a law. The questioned regulation in the PACU case may be questioned by a private school whose permit to operate has been revoked or one whose application therefore has been denied . which the DENR may or may not grant. Until such time. There is no showing that the petitioners were being evicted from the land by the heirs of Carantes under orders from the DENR. advisory opinions and in cases that has become moot. DENR. En Banc [Gonzaga-Reyes]) 112. issued. will desist from taking cognizance of speculative or hypothetical cases.‖ If indeed the heirs of Carantes are trying to enter the land and disturbing the petitioners‘ possession thereof even without prior approval by the DENR of the claim of the heirs of Carantes. The DENR is still processing the application of the heirs of Carantes for a certificate of ancestral land claim.‖ In other words. 2000. The petitioners‘ allegation that certain documents from the DENR were shown to them by the heirs of Carantes to justify eviction is vague. Courts have no judicial power to review cases involving political questions and as a rule. It is evident that the adverse legal interests involved in this case are the competing claims of the petitioners and that of the heirs of Carantes to possess a common portion of a piece of land. Narvasa. 337 SCRA 733. Aug. it must be stressed that the Court retains the power to decide whether or not it will entertain a taxpayer‘s suit. 31. In the final analysis. Secretary of Education the petition contesting the validity of a regulation issued by the Secretary of Education requiring private schools to secure a permit to operate was dismissed on the ground that all the petitioners have permits and are actually operating under the same. What is a justiciable controversy? What are political questions? 70 . A justiciable controversy has been defined as.exercise of the Chief Executive‘s power to transfer funds pursuant to Section 25 (5) of Article VI of the Constitution. 2001. This Court cannot rule on the basis of petitioners‘ speculation that the DENR will approve the application of the heirs of Carantes. We gather from the allegations of the petition and that of the petitioner‘s memorandum that the alleged application for certificate of ancestral land claim (CALC) filed by the heirs of Carantes under the assailed DENR special orders has not been granted nor the CALC applied for. As the undisputed facts stand there is no justiciable controversy between the petitioners and the respondents as there is no actual or imminent violation of the petitioners‘ asserted right to possess the land by reason of the implementation of the questioned administrative issuance. 350 SCRA 697. [Gonzaga-Reyes]) 113. ―They (the petitioners) have suffered no wrong under the terms of the law – and. Under the undisputed facts there is as yet no justiciable controversy for the court to resolve and the petition should have been dismissed by the appellate court on this ground. What is the meaning of ―justiciable controversy‖ as requisite for the proper exercise of the power of judicial review? Illustrative case. In the case at bar. Borrowing from the pronouncements of this Court in the PACU case. Nowhere in his pleadings does petitioner presume to make such a representation. he must establish that he has a ―personal and substantial interest in the case and that he has sustained or will sustain direct injury as a result of its enforcement. (Cutaran v. This Court held that there was no justiciable controversy because the petitioners suffered no wrong by the implementation of the questioned regulation and therefore. petitioner cannot be allowed to question the creation of the PCCR in his capacity as a taxpayer. Jan. petitioner must show that he is a real party in interest – that he will stand to be benefited or injured by the judgment or that he will be entitled to the avails of the suit. the case is simply one of forcible entry. 3rd Div. In the case of PACU v. There must be an actual governmental act which directly causes or will imminently cause injury to the alleged legal right of the petitioner to possess the land before the jurisdiction of this Court may be invoked. naturally need no relief in the form they now seek to obtain. therefore. As Tanada v. as where the power is exercised in an arbitrary and despotic manner by reason of passion or hostility. Ronaldo B.‖ It is familiar learning that the legitimacy of a government sired by a successful revolution by people power is beyond judicial scrutiny for that government automatically orbits out of the constitutional loop. she categorically swore to preserve and 71 . A showing that plenary power is granted either department of government may not be an obstacle to judicial inquiry. a controversy is justiciable if it refers to a matter which is appropriate for court review. or the potentiality of embarrassment from multifarious pronouncements by various departments on the one question. No less than the Freedom Constitution declared that the Aquino government was installed through a direct exercise of the power of the Filipino people ―in defiance of the provisions of the 1973 Constitution. ―[p]rominent on the surface of any case held to involve a political question is found a textually demonstrable constitutional commitment of the issue to a coordinate political department. Aquino.‖ The reason is that political questions are concerned with issues dependent upon the wisdom. 15. Is the legitimacy of the assumption to the Presidency of President Gloria Macapagal Arroyo a political question and. the courts will not normally interfere with the workings of another co-equal branch unless the case shows a clear need for the courts to step in to uphold the law and the Constitution. it may look into the question of whether such exercise has been made in grave abuse of discretion. or the impossibility of a court's undertaking independent resolution without expressing lack of the respect due coordinate branches of government. En Banc [Kapunan]) 114. Aug. the jurisdiction to delimit constitutional boundaries has been given to this Court. In checkered contrast. political questions refer ―to those questions which. When political questions are involved. the political question being a function of the separation of powers. and related cases to support their thesis that since the cases at bar involve the legitimacy of the government of respondent Arroyo. 141284. the Court cannot agree x x x that the issue involved is a political question beyond the jurisdiction of this Court to review. not the legality. Hon. we held that the government of former President Aquino was the result of a successful revolution by the sovereign people. or the impossibility of deciding without an initial policy determination of a kind clearly for nonjudicial discretion. Lozano v.R.Held: As a general proposition. Angara puts it. conditional or subject to limitations. Held: Respondents rely on the case of Lawyers League for a Better Philippines and/or Oliver A. Sec. are to be decided by the people in their sovereign capacity. the Court does not automatically assume jurisdiction over actual constitutional cases brought before it even in instances that are ripe for resolution. In the cited cases. is justiciable . Moreover. 1 of the 1987 Constitution) Under this definition. Zamora. or a lack of judicially discoverable and manageable standards for resolving it. A more cerebral reading of the cited cases will show that they are inapplicable. as amended. Nevertheless. ergo. or to act at all in contemplation of law.‖ (Article VIII. et al. Moreover. a court is without power to directly decide matters over which full discretionary authority has been delegated. or an unusual need for unquestioning adherence to a political decision already made. and to determine whether or not there has been a grave abuse of discretion amounting to lack or excess of jurisdiction on the part of any branch or instrumentality of the Government. By grave abuse of discretion is meant simply capricious or whimsical exercise of judgment that is patent and gross as to amount to an evasion of positive duty or a virtual refusal to perform a duty enjoined by law. the government of respondent Arroyo is not revolutionary in character.‖ The 1987 Constitution expands the concept of judicial review by providing that ―[T]he Judicial power shall be vested in one Supreme Court and in such lower courts as may be established by law. Judicial power includes the duty of the courts of justice to settle actual controversies involving rights which are legally demandable and enforceable. under the Constitution. One class of cases wherein the Court hesitates to rule on are ―political questions. When the grant of power is qualified.‖ Thus. G. No. In her oath. President Corazon C. if an issue is clearly identified by the text of the Constitution as matters for discretionary action by a particular branch of government or to the people themselves then it is held to be a political question.the problem being one of legality or validity. for the improvident exercise or abuse thereof may give rise to justiciable controversy . Under this definition. or in regard to which full discretionary authority has been delegated to the legislative or executive branch of government. albeit a peaceful one. In the classic formulation of Justice Brennan in Baker v. the issue of whether the prescribed qualifications or conditions have been met or the limitations respected. The oath that she took at the EDSA Shrine is the oath under the 1987 Constitution. Carr. not its wisdom . But while this Court has no power to substitute its judgment for that of Congress or of the President. not subject to judicial review? Distinguish EDSA People Power I from EDSA People Power II. 2000. (Integrated Bar of the Philippines v. they present a political question. of a particular act or measure being assailed. It pertains to issues which are inherently susceptible of being decided on grounds recognized by law. the Constitution limits the determination as to whether or not there has been a grave abuse of discretion amounting to lack or excess of jurisdiction on the part of the official whose action is being questioned. how should they act in the exercise of this jurisdiction? Held: We stress at the outset that the lower court had jurisdiction to consider the constitutionality of Section 187. However. En Banc [Kapunan]) 116. Zamora. invasion or rebellion. G. Is the President‘s power to call out the armed forces as their Commander-in-Chief in order to prevent or suppress lawless violence. Moreover. ordinance. 2001. thus. Hon. Held: When the President calls the armed forces to prevent or suppress lawless violence. to defer to the higher judgment of this Court in the consideration of its validity. 1994. G. EDSA I presented a political question. the cases at bar pose legal and not political questions. EDSA II is an exercise of people power of freedom of speech and freedom of assembly to petition the government for redress of grievances which only affected the office of the President. respondent‘s invocation of the doctrine of political question is but a foray in the dark. Nos. EDSA I involves the exercise of the people power of revolution which overthrows the whole government. this does not prevent an examination of whether such power was exercised within permissible constitutional limits or whether it was exercised in a manner constituting grave abuse of discretion. Aug. 139-140. This is clear from the intent of the framers and from the text of the Constitution itself. The principal issues for resolution require the proper interpretation of certain provisions in the 1987 Constitution. notably Section 1 of Article II.R. Ronaldo B. it is incumbent upon the petitioner to show that the President's decision is totally bereft of factual basis.‖ Thus. or regulation is in question. The present petition fails to discharge such heavy burden as there is no evidence to support the assertion that there exists no justification for calling out the armed forces. if only out of a becoming modesty. no less than on the doctrine of separation of powers. There is. BP 129 vests in the regional trial courts jurisdiction over all civil cases in which the subject of the litigation is incapable of pecuniary estimation. order. and the allocation of governmental powers under Section 11 of Article VII. bearing in mind the consequences of a declaration of unconstitutionality upon the stability of laws. international or executive agreement. Aniano Desierto. EDSA I is extra constitutional and the legitimacy of the new government that resulted from it cannot be the subject of judicial review. 4. cannot be called upon to overrule the President's wisdom or substitute its own. 235 SCRA 135. 2000. even as the accused in a criminal action has the right to question in his defense the constitutionality of a law he is charged with violating and of the proceedings taken against him. Do lower courts have jurisdiction to consider the constitutionality of a law? If so. proclamation. Estrada v. Indeed. 146710-15. March 2. Aug. she has stressed that she is discharging the powers of the presidency under the authority of the 1987 Constitution. (Integrated Bar of the Philippines v. The issues likewise call for a ruling on the scope of presidential immunity from suit. Lim. 141284. No. the doctrine has been laid down that ―it is emphatically the province and duty of the judicial department to say what the law is x x x. particularly as they contravene the Bill of Rights. but EDSA II is intra constitutional and the resignation of the sitting President that it caused and the succession of the Vice President as President are subject to judicial review. the legal distinction between EDSA People Power I and EDSA People Power II is clear. invasion or rebellion subject to judicial review. En Banc [Puno]) 115. Article VIII. or both.defend the 1987 Constitution. X x x Needless to state. In fine. Specifically. this authority being embraced in the general definition of the judicial power to determine what are the valid and binding laws by the criterion of their conformity to the fundamental law. or is it a political question? Clarify. In view of the constitutional intent to give the President full discretionary power to determine the necessity of calling out the armed forces. To doubt is to sustain. The Court. Madison. In the performance of this Court's duty of ―purposeful hesitation‖ before declaring an act of another branch as unconstitutional. 15. no evidence to support the proposition that grave abuse was committed because the power to call was exercised in such a manner as to violate the constitutional provision on civilian supremacy over the military. which is better determined after a thorough deliberation by a collegiate body and with the concurrence of the majority of those who participated in its discussion. it will be prudent for such courts. Section 5(2). EDSA II involves legal questions. In the exercise of this jurisdiction. (Joseph E. As early as the 1803 case of Marbury v. As the questioned act is usually the handiwork of the legislative or the executive departments. likewise.R. he necessarily exercises a discretionary power solely vested in his wisdom. law. and Section 8 of Article VII. of the Constitution vests in the Supreme Court appellate jurisdiction over final judgments and orders of lower courts in all cases in which the constitutionality or validity of any treaty. only where such grave abuse of discretion is clearly shown shall the Court interfere with the President's judgment. They also involve the correct calibration of the right of petitioner against prejudicial publicity. lower courts are advised to act with the utmost circumspection. En Banc [Cruz]) 72 . instruction. (Drilon v. presidential decree. the fiscal autonomy enjoyed by the Judiciary. Drilon. There. 5) Cases involving decisions. Commission on Elections. there must first be 73 . Xxx We agree with the Solicitor General that the Ombudsman committed no grave abuse of discretion warranting the writs prayed for. May the Ombudsman validly entertain criminal charges against a judge of the regional trial court in connection with his handling of cases before the court. fix rates of compensation not exceeding the highest rates authorized by law for compensation and pay plans of the government and allocate and disburse such sums as may be provided by law or prescribed by them in the course of the discharge of their functions. Held: Petitioner criticizes the jurisprudence (Maceda v. Fiscal autonomy means freedom from outside control. 8) Cases assigned to a division which in the opinion of at least three (3) members thereof merit the attention of the court en banc and are acceptable to a majority of the actual membership of the court en banc. 1992. disbarment of a lawyer. 471-472. Inc. as amended by the Resolution of November 18. 1993: X x x [t]he following are considered en banc cases: 1) Cases in which the constitutionality or validity of any treaty. and Commission on Audit. and the Ombudsman must have the independence and flexibility needed in the discharge of their constitutional duties. 208 SCRA 133. resolutions or orders of the Civil Service Commission. The Judiciary. (Firestone Ceramics. or regulation is in question. or either the suspension of any of them for a period of more than one (1) year or a fine exceeding P10. the Commission on Elections. law. What is fiscal autonomy? The fiscal autonomy clause? Held: As envisioned in the Constitution. En Banc [Gutierrez]) 119. The issues have been settled in the case of In Re: Joaquin Borromeo. Court of Appeals. June 28. 7) Cases where a doctrine or principle laid down by the court en banc or in division may be modified or reversed. The petition cannot succeed. we laid down the rule that before a civil or criminal action against a judge for a violation of Arts. What cases are to be heard by the Supreme Court en banc? Held: Under Supreme Court Circular No. 2000. and the Office of the Ombudsman contemplates a guarantee of full flexibility to allocate and utilize their resources with the wisdom and dispatch that their needs require. or presidential decree. It recognizes the power and authority to levy. order. 204 and 205 (knowingly rendering an unjust judgment or order) can be entertained. the Constitutional Commissions. 3) Cases raising novel questions of law. ordinance. executive order. Office of the Ombudsman-Mindanao. En Banc [Purisima]) 118. international or executive agreement. The imposition of restrictions and constraints on the manner the independent constitutional offices allocate and utilize the funds appropriated for their operations is anathema to fiscal autonomy and violative not only of the express mandate of the Constitution but especially as regards the Supreme Court.00 or both. 334 SCRA 465. instruction. and 9) All other cases as the court en banc by a majority of its actual membership may deem of sufficient importance to merit its attention.117. the Civil Service Commission. 221 SCRA 464 [1993] and Dolalas v. He insists that since his complaint involved a criminal charge against a judge. Vasquez. 2) Criminal cases in which the appealed decision imposes the death penalty. 2-89. 265 SCRA 818 [1996]) cited by the Office of the Ombudsman as erroneous and not applicable to his complaint. the Commission on Audit. other public ministers and consuls. v. of the independence and separation of powers upon which the entire fabric of our constitutional system is based. 6) Cases where the penalty to be imposed is the dismissal of a judge. assess and collect fees. proclamation. dated February 7. officer or employee of the judiciary. 4) Cases affecting ambassadors. April 15. it was within the authority of the Ombudsman not the Supreme Court to resolve whether a crime was committed and the judge prosecuted therefor. 1989. (Bengzon v.000. however.―a final and authoritative judicial declaration‖ that the decision or order in question is indeed ―unjust. incorporation by reference is allowed if only to avoid the cumbersome reproduction of the decision of the lower courts. and accordingly rendered a verdict and imposed the corresponding penalty. 74 . which must be contained in a statement attached to the said decision. the determination of whether a judge has maliciously delayed the disposition of the case is also an exclusive judicial function (In Re: Borromeo. Discuss the validity of ―Memorandum Decisions. 335 SCRA 281. before finally agreeing with the prosecution‘s evaluation of the case . In other words. or quoted the facts narrated in the prosecution‘s memorandum but made their own findings and assessment of evidence. This having been said. (Oil and Natural Gas Commission v. We have also declared that memorandum decisions comply with the constitutional mandate. For the incorporation by reference to be allowed. the modifying circumstances. Permskul. or portions thereof. In Francisco v. we laid the conditions for the validity of memorandum decisions. We have sustained decisions of lower courts as having substantially or sufficiently complied with the constitutional injunction notwithstanding the laconic and terse manner in which they were written and even if ―there [was left] much to be desired in terms of [their] clarity. X x x Hence. That prerogative belongs to the courts alone. Blg. the participation of the accused. practicality. The constitutional mandate that no decision shall be rendered by any court without expressing therein clearly and distinctly the facts and the law on which it is based does not preclude the validity of ―memorandum decisions‖ which adopt by reference the findings of fact and conclusions of law contained in the decisions of inferior tribunals. 2000. July 23. [Pardo]) 120.‖ (Francisco v. no other entity or official of the government.P. even in this jurisdiction. which is to say that the challenged decision is not easily and immediately available to the person reading the memorandum decision. the memorandum decision authorized under Section 40 of B. thus: The memorandum decision. 1998 [Martinez]) 2. cannot incorporate the findings of fact and the conclusions of law of the lower court only by remote reference. Permskul. as when they stated the legal qualifications of the offense constituted by the facts proved. ―To repeat. 173 SCRA 324. not any functionary thereof. B. or (b) an administrative proceeding in the Supreme Court against the judge precisely for promulgating an unjust judgment or order. has competence to review a judicial order or decision – whether final and executory or not – and pronounce it erroneous so as to lay the basis for a criminal or administrative complaint for rendering an unjust judgment or order. or discussed the facts comprising the elements of the offense that was charged in the information. 129 on the grounds of expediency. at 461). convenience and docket status of our courts. Likewise. 129 should actually embody the findings of fact and conclusions of law of the lower court in an annex attached to and made an indispensable part of the decision. not the prosecution or investigation service of any other branch.‖ Held: 1. Pelayo. Blg. coherence and comprehensibility‖ provided that they eventually set out the facts and the law on which they were based. it must provide for direct access to the facts and the law being adopted. to be valid. supra.P. (De Vera v. the penalty imposed and the civil liability. we find that the Ombudsman acted in accordance with law and jurisprudence when he referred the cases against Judge Pelayo to the Supreme Court for appropriate action. We have also sanctioned the use of memorandum decisions x x x. July 6. This is particularly true when the decision sought to be incorporated is a lengthy and thorough discussion of the facts and conclusions arrived at x x x.‖ The pronouncement may result from either: (a) an action of certiorari or prohibition in a higher court impugning the validity of the judgment. What is a Memorandum Decision? Held: A Memorandum Decision is a ―specie of succinctly written decisions by appellate courts in accordance with the provisions of Section 40. in the decisions of the higher court. 1st Div. Court of Appeals. 293 SCRA 26. 333 [1989]) 121. so he may appeal to the higher court. a safeguard against the impetuosity of the judge.It is expected that this requirement will allay the suspicion that no study was made of the decision of the lower court and that its decision was merely affirmed without a prior examination of the facts and the law on which it is based. otiosely written. We cannot even consider or affirm said RTC decision as a memorandum decision because it failed to comply with the measures of validity laid down in Francisco v. Its inadequacy speaks for itself. where we cautioned that expediency alone. fell short of the constitutional injunction. Court of Appeals come true. especially one resolving an appeal. said court deprived the appellant of due process since he was accorded a fair opportunity to be heard by a fair and responsible magistrate. should he believe that the decision should be reversed. but never at the expense of scholarly analysis. The losing party is entitled to know why he lost. Any deviation will summon the strict enforcement of Article VIII. Faithful adherence to the requirements of Section 14. we find that the RTC decision at bar miserably failed to meet them and. where not only property rights are at stake but also the liberty if not the life of a human being. Xxx Henceforth. It is. in reaching judgment. It is an additional condition for the validity of this kind of decision may be resorted to only in cases where the facts are in the main accepted by both parties and easily determinable by the judge and there are no doctrinal complications involved that will require an extended discussion of the laws involved. of course. no matter how compelling. The memorandum decision may be employed in simple litigations only. such as ordinary collection cases. Permskul. Section 14 of the Constitution and strike down the flawed judgment as a lawless disobedience. Vouchsafed neither the sword nor the purse by the Constitution but nonetheless vested with the sovereign prerogative of passing judgment on the life. but it is starkly hallow. where the appeal is obviously groundless and deserves no more than the time needed to dismiss it. Section 14 as no amount of incorporation or adoption will rectify its violation.e. it should not and cannot be substituted for substance. The proximity at least of the annexed statement should suggest that such examination has been undertaken. The Court finds necessary to emphasize that the memorandum decision should be sparingly used lest it become an additive excuse for judicial sloth. the requirement is an assurance to the parties that. vacuous in its content and trite in its form. as in this case. Tested against these standards. with an explanation of the factual and legal reasons that led to the conclusions of the court. if an appellate court failed to provide the appeal the attention it rightfully deserved. This situation becomes more ominous in criminal cases. More than that. thus. if permitted. comply with Article VIII. lest the fears expressed by Justice Feria as the ponente in Romero v. cannot excuse non-compliance with the constitutional requirements. The parties to a litigation should be informed of how it was decided. to begin with. should directly meet the issues for resolution. Court of Appeals. in that while we conceded that brevity in the writing of decisions is an admirable trait. A decision or resolution. 75 . The RTC decision is brief indeed. A decision that does not clearly and distinctly state the facts and the law on which it is based leaves the parties in the dark as to how it was reached and is precisely prejudicial to the losing party. preventing him from deciding ipse dixit. all memorandum decisions shall comply with the requirements herein set forth as to the form prescribed and the occasions when they may be rendered. Permskul.. liberty or property of his fellowmen. the judge did so through the processes of legal reasoning. i. who is unable to pinpoint the possible errors of the court for review by a higher tribunal . It merely affirmed in toto the MeTC decision without saying more. the appeal would be pointless We therefore reiterate our admonition in Nicos Industrial Corporation v. not even at a simple summation of facts which could easily be done. of justice and fair play. It is. and again in Francisco v. It is likewise demanded by the due process clause of the Constitution. therefore. The court cannot simply say that judgment is rendered in favor of X and against Y and just leave it at that without any justification whatsoever for its action. the judge must ultimately depend on the power of reason for sustained public confidence in the justness of his decision. It achieved nothing and attempted at nothing. and more significantly. otherwise. also understood that the decision being adopted should. Article VIII of the Constitution is indisputably a paramount component of due process and fair play. This is not to discourage the lower courts to write abbreviated and concise decisions. order. Oct. 333 [1989]. Article VIII of the Constitution. a decision. Thus. or ruling under review. in the decision of the higher court. Held: [A] temporary or acting appointee does not enjoy security of tenure. En Banc [Pardo]) The Constitutional Commissions 124. apply to the Sandiganbayan? Explain. The Constitution provides for many safeguards to the independence of the Commission on Elections. convicted the accused of libel but failed to cite any legal authority or principle to support conclusions that the letter in question was libelous. 344 SCRA 202. 00-8-05-SC. foremost among which is the security of tenure of its members. including the COMELEC. Thus. we nullified or deemed to have failed to comply with Section 14. Court of What are the distinctive features and purpose of a memorandum decision? Held: In Francisco v. Assuming its validity. no matter how This is the kind of appointment that the Constitution prohibits the President from making to the three independent constitutional commissions. It is doubtful if the respondent. Article VIII. In the same vein do we strike down as a nullity the RTC decision in question. Yorac. 2000. Court of Appeals. The provision refers to regular courts of lower collegiate level that in the present hierarchy applies only to the Court of Appeals. Thus. consisted merely of one (1) paragraph with mostly sweeping generalizations and failed to support its conclusion of parricide. second. the Sandiganbayan is not a regular court but a special one. Nov. the designation of the respondent as Acting Chairman of the Commission on Elections may be withdrawn by the President of the Philippines at any time and for whatever reason she sees fit. Permskul (173 SCRA 324. 24. first. A. The Sandiganbayan is a special court of the same level as the Court of Appeals and possessing all the inherent powers of a court of justice. although it is not impossible that the approval of the findings of facts by the lower court may lead to a different conclusion of law by the higher court. contained nothing more than a summary of the testimonies of the witnesses of both parties. The idea is to avoid having to repeat in the body of the latter decision the findings or conclusions of the lower court since they are being approved or adopted anyway. Does the period for decision making under Section 15. Why does the Constitution prohibit the President from appointing in an acting or temporary capacity the Chairman and Commissioners of the Constitutional Commissions? Explain. or portions thereof. the reason for allowing the incorporation by reference is evidently to avoid the cumbersome reproduction of the decision of the lower court. At any rate. resolution or order which: contained no analysis of the evidence of the parties nor reference to any legal basis in reaching its conclusions. 24. the purpose is to affirm the decision. No cause need be established to justify its revocation. (Yao v.M. three (3) pages of which were quotations from the labor arbiter‘s decision including the dispositive portion and barely a page (two [2] short paragraphs of two [2] sentences each) of its own discussion or reasonings. 344 SCRA 202. with functions of a trial court. Most likely. decisions of lower courts and even of the Court of Appeals whose careless disregard of the constitutional behest exposed their sometimes cavalier attitude not only to their magisterial responsibilities but likewise to their avowed fealty to the Constitution. (Yao v. No. will not be estopped from challenging its withdrawal. This Court ruled that: ―A designation as Acting Chairman is by its very terms essentially temporary and therefore revocable at will. 28. [Davide]) 122. briefly. 1st Div. 1987 Constitution. this Court struck down as unconstitutional the designation by then President Corazon Aquino of Associate Commissioner Haydee Yorac as Acting Chairperson of the COMELEC. 2000. it is rendered by an appellate court. [Davide]) 123. 2001. or failed to explain the factual and legal bases for the award of moral damages.Thus the Court has struck down as void. 1st Div. consisted of five (5) pages. it incorporates by reference the findings of fact or the conclusions of law contained in the decision. having accepted such designation. Appeals. Oct. That guarantee is not 76 . (Re: Problem of Delays in Cases Before the Sandiganbayan. Held: The above provision does not apply to the Sandiganbayan. the Court described ―[t]he distinctive features of a memorandum decision are. in Brillantes v. was merely based on the findings of another court sans transcript of stenographic notes. The COA contends this is similar to its sole and exclusive authority. This Section provides as follows: ―Sec. authority. audit. (1) The Commission on Audit shall have the power.‖ (Emphasis supplied) The COA vigorously asserts that under the first paragraph of Section 2. Commissioner Serafin Guingona proposed the addition of the word ―exclusive‖ in the first paragraph of Section 2. thereby granting the COA the sole and exclusive power to examine and audit all government agencies. owned and held in trust by. Thus. or uses of government funds and properties. promulgate auditing rules and regulations. including those for the prevention and disallowance of irregular. extravagant. excessive. the first paragraph of Section 2 must be read the way it appears. This word ―exclusive‖ did not appear in the counterpart provisions of Section 2 in the 1935 and 1973 Constitutions. including governmentowned or controlled corporations with original charters. 2. and not to a phrase distantly located in another paragraph or sub-section. cannot in any way intrude into this exclusive function of the COA. however. or instrumentalities. the COA has the exclusive authority to decide on disallowances of unnecessary government expenditures. shows that the COA‘s power under the first paragraph is not declared exclusive. We declared then: ―It would be more in keeping with the intent. the COA enjoys the sole and exclusive power to examine and audit all government agencies. subject to the limitations in this Article. Article IX-D of the 1987 Constitution. This Court ruled that the designation of an acting Commissioner would undermine the independence of the COMELEC and hence violate the Constitution. without the word ―exclusive. establish the techniques and methods required therefore. During the deliberations of the Constitutional Commission.available to the respondent as Acting Chairman of the Commission on Elections by designation of the President of the Philippines. Thus. including rules on the disallowance of unnecessary expenditures of government agencies. ―(2) The Commission shall have the exclusive authority. A qualifying word is intended to refer only to the phrase to which it is immediately associated. including the DBP. which did not have a provision prohibiting temporary or acting appointments to the COMELEC. Bautista. under the same paragraph of the same section. to define the scope of its audit. the Constitutional Commission placed the word ―exclusive‖ to qualify the authority of the COA under the second paragraph of the same Section 2. However. the Constitutional Commission rejected the addition of the word ―exclusive‖ in the first paragraph of Section 2 and Guingona was forced to withdraw his proposal. or pertaining to. a case decided under the 1935 Constitution. April 2. and expenditures or uses of funds and property. The bare language of Section 2. or any of its subdivisions. Benipayo. X x x Xxx In sharp contrast. the Government. the framers of the Constitution intentionally omitted the word ―exclusive‖ in the first paragraph of Section 2 precisely to allow concurrent audit by private external auditors. Other government agencies and their officials. 77 . x x x. unnecessary. purpose and aim of the framers of the Constitution to appoint a permanent Commissioner than to designate one to act temporarily. agencies. There is no dispute that the COA‘s authority under the second paragraph of Section 2 is exclusive as the language of the Constitution admits of no other meaning. as well as private auditors engaged by them. while its authority under the second paragraph is expressly declared ―exclusive. Besides. and duty to examine.‖ (Matibag v.‖ signifying that non-COA auditors can also examine and audit government agencies. and promulgate accounting and auditing rules and regulations. to define the scope of its audit and examination.‖ Earlier. 2002. The qualifying word ―exclusive‖ in the second paragraph of Section 2 cannot be applied to the first paragraph which is another sub-section of Section 2. and settle all accounts pertaining to the revenue and receipts of. En Banc [Carpio]) 125. Is the constitutional power of the COA to examine and audit government banks and agencies exclusive? Does it preclude a concurrent audit by a private external auditor? Held: The resolution of the primordial issue of whether or not the COA has the sole and exclusive power to examine and audit government banks involves an interpretation of Section 2. this Court nevertheless declared unconstitutional the designation of the Solicitor General as acting member of the COMELEC. in Nacionalista Party v.‖ There is a significant reason for this marked difference in language. 380 SCRA 49. or unconscionable expenditures. And in matters falling under the second paragraph of Section 2. Undeniably. In these instances the government enters the marketplace and competes with the rest of the world in attracting investments or loans. promulgate auditing rules and regulations. In the case of Section 2. despite the Central Bank‘s concurrent jurisdiction over government banks. The inevitable conclusion is that the COA and the Central Bank have concurrent jurisdiction. On the other hand. Article IX-D of the Constitution. remain bound by the findings and conclusions of the COA. as when there is a private investment in a government-controlled corporation. the Central Bank‘s power of ―supervision‖ includes the power to examine and audit banks. If a law needs interpretation. 2002. By design the Constitution is flexible enough to meet these exigencies. the government must abide with the reasonable business practices of the marketplace. the COA‘s audit still prevails over that of the Central Bank since the COA is the constitutionally mandated auditor of government banks. Government agencies and officials. which should be discovered within its four corners aided. There is another constitutional barrier to the COA‘s insistence of exclusive power to examine and audit all government agencies. Hence. as the banking laws have always recognized this power of the Central Bank. in the absence of a statute limiting or removing such flexibility. 373 SCRA 356. As this Court has ruled repeatedly. whether the matter falls under the first or second paragraph of Section 2. the COA‘s jurisdiction is exclusive. the intent of the framers of the Constitution is evident from the bare language of Section 2 itself. under the Constitution. and disallow unnecessary expenditures is exclusive. En Banc [Carpio]) 126. the Central Bank is devoid of authority to allow or disallow expenditures of government banks since this function belongs exclusively to the COA. Commission on Audit. Article XII of the Constitution. or in similar instances. the COA‘s authority to define the scope of its audit. however. The deliberations of the Constitutional Commission reveal eloquently the intent of Section 2. and the clear intent of its framers. as the constitutionally-mandated auditor of all government agencies. Any attempt to nullify this flexibility in the instances mentioned. To succeed. January 16. to examine and audit government banks. the express language of the Constitution. Xxx Manifestly. Otherwise no investor or creditor will do business with the government.The clear and unmistakable conclusion from a reading of the entire Section 2 is that the COA‘s power to examine and audit is non-exclusive. Between the COA‘s findings and conclusions and that of private auditors. at least insofar as government agencies and officials are concerned. point to only one indubitable conclusion – the COA does not have the exclusive power to examine and audit government agencies. if necessary. the intent of the law is the controlling factor in the interpretation of the law . frustrating government efforts to attract investments or secure loans that may be critical to stimulate moribund industries or resuscitate a badly shattered national economy as in the case at bar. The deliberations of the Constitutional Commission confirm expressly and even elucidate further this intent beyond any doubt whatsoever. Article IX-D of the Constitution. will be ultra vires. the Central Bank has been conducting periodic and special examination and audit of banks to determine the soundness of their operations and the safety of the deposits of the public. or as in the case at bar when the government borrows money from abroad. the COA‘s findings and conclusions necessarily prevail over those of private auditors. by its legislative history. The intent of the law is that which is expressed in the words of the law. which should prevail? Held: Moreover. Thus. The framers of the Constitution were fully aware of the need to allow independent private audit of certain government agencies in addition to the COA audit. (Development Bank of the Philippines v. the COA‘s power to examine and audit government banks must be reconciled with the Central Bank‘s power to supervise the same banks. The COA‘s claim clashes directly with the Central Bank‘s constitutional power of ―supervision‖ over banks under Section 20. The superiority or preponderance of the COA audit over private audit can be gleaned from the records of the Constitutional Commission x x x. May the power of the COA to examine and audit government agencies be validly taken away from it? 78 . the most dominant influence is the intent of the law. unless of course such findings and conclusions are modified or reversed by the courts. However. X x x Historically. 127. Article IX-D of the Constitution. The findings and conclusions of the private auditor may guide private investors or creditors who require such private audit. or when a government corporation is privatized or publicly listed. Feb. and the general welfare. 3. we held that petitioners‘ removal as commissioners of the National Police Commission (NAPOLCOM) and the appointment of new Commissioners in their stead were nullities and ordered the reinstatement of petitioners and the payment of full backwages to be computed from the date they were removed from office. based on our foregoing disquisition. 2001. 201 SCRA 508). from the jurisdiction of the Commission on Audit. or any investment of public funds. Adiong‘s appointment on 11 March 1998 for a term of two years. (Metropolitan Manila Development Authority v. The State. Held: Respondents are seeking a reconsideration of the Court‘s 25 January 2000 decision. It bears stressing that police power is lodged primarily in the National Legislature . When a regular government employee is illegally dismissed. Once delegated. in order to effectively accomplish and 79 . No law shall be passed exempting any entity of the Government or its subsidiary in any guise whatsoever. Define Police Power and clarify its scope. public morals. and establish all manner of wholesome and reasonable laws. En Banc [Carpio]) B. 8551 (RA 8551) to be violative of petitioners‘ constitutionally mandated right to security of tenure. however. good order or safety and general welfare of the people (Binay v. while non-exclusive. What is the effect of declaration of unconstitutionality of a law? Illustrative case. it confers no rights. 843-844. through the legislature. As a consequence of our ruling. may delegate this power to the President and administrative boards as well as the lawmaking bodies of municipal corporations or local government units. Therefore. reaching and justifying measures for public health. Police power is an inherent attribute of sovereignty. has delegated the exercise of police power to local government units. [Puno]) 2. imposes no duties. ordain. and for the subjects of the same. It has been defined as the power vested by the Constitution in the legislature to make. The National Legislature. either with penalties or without. X x x. It cannot be exercised by any group or body of individuals not possessing legislative power . morals. 15. in order to give way to the reinstatement of petitioners and respondent Adiong. 2000. Accordingly. 373 SCRA 356. Section 3. Therefore. March 27. the unavoidable consequence of the Court‘s declaration that Section 8 of RA 8551 violates the fundamental law is that all acts done pursuant to such provision shall be null and void.‖ The mere fact that private auditors may audit government agencies does not divest the COA of its power to examine and audit the same government agencies. Article IX-C of the Constitution mandates that: ―Sec. education. Domingo. Xxx An unconstitutional act is not a law. Rudimentary is the precept that there can be no valid appointment to a nonvacant position. peace. (Development Bank of the Philippines v. January 16. The power is plenary and its scope is vast and pervasive. Police power as an inherent attribute of sovereignty is the power to prescribe regulations to promote the health. pursuant to Section 8 of RA 8551.Held: The power of the COA to examine and audit government agencies. his position does not become vacant and the new appointment made in order to replace him is null and void ab initio. as agencies of the State. 1st Div. and affords no protection. the agents can exercise only such legislative powers as are conferred on them by the national lawmaking body . Commission on Audit. wherein we declared Section 8 of Republic Act No. CONSTITUTIONAL LAW 128. Inc. 328 SCRA 836. Bel-Air Village Association. 2002. statutes and ordinances. En Banc [Gonzaga-Reyes]) THE INHERENT POWERS OF THE STATE Police Power 129. including the removal of petitioners and Adiong from their positions in the NAPOLCOM and the appointment of new commissioners in their stead. there should no longer be any doubt as to the proper execution of our 25 January 2000 decision – all the Commissioners appointed under RA 8551 should be removed from office. not repugnant to the Constitution. 351 SCRA 659. (Canonizado v. Aguirre. as they shall judge to be for the good and welfare of the commonwealth. cannot be taken away from the COA.. Held: 1. public safety. is null and void. ‖ Perforce. Court of Appeals. G. Held: The regulation of rates to be charged by public utilities is founded upon the police power of the State and statutes prescribing rules for the control and regulations of public utilities are a valid exercise thereof. 3 rd Div. Discuss why rates to be charged by public utilities like MERALCO are subject to State regulation. 15. is within the ambit of this power (Procter and Gamble Phils. The fixing of just and reasonable rates involves a balancing of the investor and the consumer interests. 2002. a legislative franchise to operate jai-alai is imbued with public interest and involves an exercise of police power. The familiar rule is that laws which grant the right to exercise a part of the police power of the state are to be construed strictly and any doubt must be resolved against the grant. [Puno]) 132. the same is subject to public regulation . 141314. transportation. A statute which legalizes a gambling activity or business should be strictly construed and every reasonable doubt must be resolved to limit the powers and rights claimed under its authority. The phrase ―affected with public interest‖ means that an industry is subject to control for the public good. but as long as the use of the property is continued. is essentially in the exercise of the police power of the State within the contemplation of the general welfare clause of the Local Government Code. Police power is essentially regulatory in nature and the power to issue licenses or grant business permits. En Banc [Puno]) 131. Court of Appeals. The legislature is regarded as the guardian of society. Thus. the power to regulate rates does not give the State the right to prescribe rates which are so low as to deprive the public utility of a reasonable return on investment. 329 SCRA 314. 2000. 329 SCRA 314. communication or electricity) – the operation of which undoubtedly redounds to the benefit of the general public. Muncipality of Virac. (Acebedo Optical Company. Thus. v. Inc. 335. it ceases to be juris privati only and becomes subject to regulation. The rights granted by the 80 . 29. 2000. Manila Electric Company. v. the rates prescribed by the State must be one that yields a fair return on the public utility upon the value of the property performing the service and one that is reasonable to the public for the service rendered . peace. order. Submission to regulation may be withdrawn by the owner by discontinuing use. March 31. Does Article 263(g) of the Labor Code (vesting upon the Secretary of Labor the discretion to determine what industries are indispensable to the national interest and thereafter. Nov. comfort and convenience of the community. assume jurisdiction over disputes in said industries) violate the workers‘ constitutional right to strike? Held: Said article does not interfere with the workers‘ right to strike but merely regulates it. It is stressed that the case at bar does not involve a franchise to operate a public utility (such as water. (Republic of the Philippines v. In regulating rates charged by public utilities. it has been considered as the equivalent of ―subject to the exercise of the police power.R. Inc. The authority. Nov. if exercised for a regulatory and not revenue-raising purpose. The Muncicipality of Jagna. Constitutions are widely understood to withhold from legislatures any authority to bargain away their police power for the power to protect the public interest is beyond abnegation. However. which devolved upon local government units to issue or grant such licenses or permits. What is claimed is an alleged legislative grant of a gambling franchise – a franchise to operate jai-alai. En Banc [Purisima]) 130. The presumption is influenced by constitutional considerations. 325-326. En Banc [Purisima]) 133. Discuss the nature of the authority of local government units to issue or grant licenses or permits. 2000. let us not forget that PAGCOR is engaged in business affected with public interest. the State protects the public against arbitrary and excessive rates while maintaining the efficiency and quality of services rendered. 207 SCRA 157). Held: [T]he issuance of business licenses and permits by a municipality or city is essentially regulatory in nature. 346 SCRA 485. How should laws that grant the right to exercise a part of the police power of the State be construed? Held: Lest the idea gets lost in the shoals of our subconsciousness. This delegation of police power is embodies in the general welfare clause of the Local Government Code x x x. v. national interests will be affected. morals. (Acebedo Optical Company. March 31. safety. when in the exercise of such right. When private property is used for a public purpose and is affected with public interest.carry out the declared objects of their creation (Tatel v. The regulation is to promote the common good. PAGCOR. The scope of police power has been held to be so comprehensive as to encompass almost all matters affecting the health. No. (Del Mar v. 94 SCRA 894). courts do not assume that the legislature intended to part away with its power to regulate public morals. and therefore is not presumed to disable itself or abandon the discharge of its duty. Does a corporation or individual not himself licensed. Villalon-Pornillos. to establish his identity and his authority to act for the cause which he purports to represent. specifically on the hiring and employment of licensed optometrists by optical corporations. even though the collection be for a religious purpose. in order that the State may protect its citizens from injury. which does not involve any religious test and does not unreasonably obstruct or delay the collection of funds. have a right to hire and employ licensed optometrists? Will the employment of a qualified optometrist by a corporation go against public policy? Held: From the foregoing. Some regulation of public solicitation is. (Centeno v. The objectionable practices of unscrupulous persons are prejudicial to worthy and proper charities which naturally suffer when the confidence of the public in campaigns for the raising of money for charity is lessened or destroyed. 81 . Confesor. it is surprising how many operations are carried on by persons and associations who. of solicitation. Sept. March 12. The Secretary of Labor acts to maintain industrial peace. but somewhere should be lodged the power to determine within reasonable limits the worthy from the unworthy. or convenience. a State may protect its citizens from fraudulent solicitation by requiring a stranger in the community. It is clear that Congress left the resolution of such issue for judicial determination. Even in the United States. It does not follow. before permitting him publicly to solicit funds for any purpose. It has been said that a law advancing a legitimate governmental interest is not necessarily invalid as one interfering with the ―free exercise‖ of religion merely because it also incidentally has a detrimental effect on the adherents of one or more religion. 236 SCRA 197. Thus. The interests of both the employers and the employees are intended to be protected and not one of them is given undue preference. his certification for compulsory arbitration is not intended to impede the workers‘ right to strike but to obtain a speedy settlement of the dispute. in the public interest. Even with numerous regulative laws in existence. in the public interest. The State has authority under the exercise of its police power to determine whether or not there shall be restrictions on soliciting by unscrupulous persons or for unworthy causes or for fraudulent purposes. comfort. It is in fact amazing how profitable the fraudulent schemes and practices are to people who manipulate them. peace. (Philtread Workers Union [PTWU] v. Without doubt. the general regulation. That solicitation of contributions under the guise of charitable and benevolent purposes is grossly abused is a matter of common knowledge. 1994 [Regalado]) 135. 1. This is done for the promotion of the common good considering that a prolonged strike or lockout can be inimical to the national economy. therefore. it is thus evident that Congress has not adopted a unanimous position on the matter of prohibition of indirect practice of optometry by corporations. is not open to any constitutional objection. and it is therefore proper for this Court to resolve the issue. The assumption of jurisdiction is in the nature of police power measure. May solicitation for religious purposes be subject to proper regulation by the State in the exercise of police power? Held: Whence. Certainly the solicitation of contributions in good faith for worthy purposes should not be denied.Constitution are not absolute. Such regulation would not constitute a prohibited previous restraint on the free exercise of religion or interpose an inadmissible obstacle to its exercise. it will assume jurisdiction over the labor dispute of said industry. solicitation for religious purposes may be subject to proper regulation by the State in the exercise of police power. from the constitutional guarantees of the free exercise of religion that everything which may be so called can be tolerated . The State is likewise free to regulate the time and manner of solicitation generally. therefore. even the exercise of religion may be regulated. They are still subject to control and limitation to ensure that they are not exercised arbitrarily. Thus. To conclude. jurisprudence varies and there is a conflict of opinions among the federal courts as to the right of a corporation or individual not himself licensed. upon the determination of the Secretary of Labor that such industry is indispensable to the national interest. to hire and employ licensed optometrists (128 ALR 586). 1997) 134. The Labor Code vests upon the Secretary of Labor the discretion to determine what industries are indispensable to national interest. secreting their activities under the guise of benevolent purposes. at some slight inconvenience. in the interest of public safety. 269 SCRA 393. Thus. succeed in cheating and defrauding a generous public. and in personal attendance at the place where such articles are sold (Roschen v. State ex. 113. 113 NJL 287. 49 S Ct 336). 220 Ind. the patient‘s primary and essential safeguard lies in the optometrist‘s control of the ―treatment‖ by means of prescription and preliminary and final examination (Small and Maine Board of Registration and Examination in Optometry. The exercise of the power of eminent domain requires that due process be observed in the taking of private property. En Banc [Purisima]) 136. v. it was ruled that the employment of a qualified optometrist by a corporation is not against public policy (Georgia State Examiners v. and lenses unless a duly licensed physician or a duly qualified optometrist is in charge of. Ward. Unless prohibited by statutes. to carry out such regulation. New Jersey State Bd. Brother v. 339 Mo 427. Court of Appeals. 174 A 353). there is also a taking under the power of eminent domain. 1939. In such a case. 72 App DC 77. 276. The primary purpose of the statute regulating the practice of optometry is to insure that optometrical services are to be rendered by competent and licensed persons in order to protect the health and physical welfare of the people from the dangers engendered by unlicensed practice. But where. Inc. State Board of Examiners in Optometry. 183 Ga 669. prescribe and dispense ophthalmic lenses. 102 Ariz 175. Held: The implementation of the CARL is an exercise of the State‘s police power and the power of eminent domain. McKittrick v. March 31. and optometry as a mechanical art. 291 Mich 152. a corporation has all the contractual rights that an individual has (State ex rel. En Banc [Puno]) The Power of Eminent Domain 137. there is an exercise of police power for the regulation of private property in accordance with the Constitution . Kindy Opticians. Furthermore. Rel. v. Sears Roebuck and Co. (Acebedo Optical Company. Board of Optometry v. Friedman‘s Jewelers. 329 SCRA 314.S. 189 SE 238). Beck Jewelry Enterprises. 246 App Div 341. 288 Ala 349. And. There is indeed no valid basis for treating corporations engaged in the business of running optical shops differently. Inc. S. 73 L Ed 722. it is noteworthy that private hospitals are maintained by corporations incorporated for the purpose of furnishing medical and surgical treatment. 331-333. Dec. Lansburgh and Brother. To the extent that the CARL prescribes retention limits to the landowners. they have tended to find nothing objectionable in the making and selling of eyeglasses. Kresge Co. 128 ALR 582. insofar as the courts regartd optometry as merely a mechanical art. Flynn. The Bill of Rights provides that ―[n]o person shall be deprived of life. eye frames and optical lenses. In analogy. Xxx To accomplish the objective of the regulation. optical shops hire licensed optometrists to examine.. Such purpose may be fully accomplished although the person rendering the service is employed by a corporation (Silver v. in the same way that in the course of manufacturing and selling eyeglasses.. trading and bartering of eyeglasses and spectacles as articles of merchandise do not constitute the practice of optometry (State ex rel.Courts have distinguished between optometry as a learned profession in the category of law and medicine. 321 SCRA 106. these corporations employ physicians. 427 Pd 126). 1999. 289 NW 112. 2000. 41 NE 2d 622. 11 F2d 518. 286 NYS 225).50 Eyeglasses. In the course of providing such treatments. Court of Appeals. 279 US 337. What is required is the surrender of the title to and physical possession of the said excess and all beneficial rights accruing to the owner in favor of the farmer beneficiary. surgeons and medical practitioners. 141 ALR 876 [61 Am Jur 187]. selling. 97 SW 2d 89) and it does not become the practice of medicine or optometry because of the presence of a physician or optometrist (Dickson v. 261 So 2d 27. spectacles and lenses by corporations so long as the patient is actually examined and prescribed for by qualified practitioners (House of $8. (Roxas & Co.‖ The CARL was not intended to take away property without due process of law. Inc. What is Eminent Domain? 82 . v.. 293 A 2d 786). 61 Am Jur 2d 289).. Of Optometrists v. a state may provide by statute that corporations cannot sell eyeglasses. What powers of the State are involved in the implementation of the Comprehensive Agrarian Reform Law (CARL)? Discuss. No one has ever charged that these corporations are engaged in the practice of medicine. Inc. v. liberty or property without due process of law. The taking contemplated is not a mere limitation of the use of the land. the owners are deprived of lands they own in excess of the maximum area allowed. spectacles. State Board of Optometry. Inc. The manufacturing. 17. Gate City Optical Co. and second. City of Mandaluyong. Thus. other public entities and public utilities.. 1997) 138. that such power may be validly delegated to local government units. for greater public purposes. at most. 268 SCRA 586. the taking must be for public purpose and there must be just compensation. (Heirs of Alberto Suguitan v. however. 328 SCRA 137. a constitutionally-protected right necessary for the preservation and enhancement of personal dignity and intimately connected with the rights to life and liberty. It is an indispensable attribute of sovereignty. and the defendant asserts title or interest in the property. (Moday v. the taking must be for public use. For the taking of private property by the government to be valid. February 20. even that of police power itself. Expropriation proceedings are not adversarial in the conventional sense. is necessarily in derogation of private rights. by filing the action. Inherently possessed by the national legislature. that just compensation must be given to the private owner of the property . all separate interests of individuals in property are held under a tacit agreement or implied reservation vesting upon the sovereign the right to resume the possession of the property whenever the public interest so requires it. the due process and equal protection clauses act as additional safeguards against the arbitrary exercise of this governmental power. the need for its circumspect operation cannot be overemphasized. Court of Appeals. In City of Manila v. No species of property is held by individuals with greater tenacity. in the due exercise of the power. March 14. Fundamental to the independent existence of a State. 13 Cal. private property for public use or purpose.Held: 1. Held: The right of eminent domain is usually understood to be an ultimate right of the sovereign power to appropriate any property within its territorial sovereignty for a public purpose . or by its authorized agents. the right of eminent domain appertains to every independent government without the necessity for constitutional recognition. Important as the power may be to the government. in many respects. 306 and cases cited [73 Am. It is firmly settled. 144-146. Dec. the power is not without its limits: first. for the condemning authority is not required to assert any conflicting interest in the property. Thus. the condemnor in effect merely serves notice that it is taking title and possession of the property. the power is inherent. than the right to the freehold of inhabitants. Eminent domain is a fundamental State power that is inseparable from sovereignty. Eminent domain is the right or power of a sovereign state to appropriate private property to particular uses to promote public welfare . its scope matching that of taxation. Chinese Community of Manila we said: The exercise of the right of eminent domain. 2000. however. although the scope of this delegated legislative power is necessarily narrower than that of the delegating authority and may only be exercised in strict compliance with the terms of the delegating law. other public entities and public utilities. not to prove a right to possession. Discuss the nature of the right of eminent domain and the limitations thereof. but to prove a right to compensation for the taking. the power of eminent domain may be validly delegated to local governments. [Gonzaga-Reyes]) 2. Thus. In the hands of the legislature. Since the exercise of the power of eminent domain affects an individual‘s right to private property. Mountainlake Water Co.‖ Furthermore. It reaches to every form of property the State needs for public use and. whether directly by the State. the inviolable sanctity which all free constitutions attach to the right of property of the citizens. Obviously. our own Constitution provides that ―[p]rivate property shall not be taken for public use without just compensation. in the nature of a compulsory sale to the State. The ubiquitous character of eminent domain is manifest in the nature of the expropriation proceedings. a power grounded in the primary duty of government to serve the common need and advance the general welfare. and none is guarded by the Constitution and the laws more sedulously. It is government‘s right to appropriate. appropriates the land of ah individual without his consent.. constrains the strict observance of the substantial provisions of the law which are prescribed as modes of the exercise of the power. 576]) The statutory power of taking property from the owner without his consent is one of the most delicate exercise of governmental authority. These twin 83 . (Bensley v. The provisions found in modern constitutions of civilized countries relating to the taking of property for the public use do not by implication grant the power to the government. and to protect it from abuse x x x. as an old case so puts it. whose provisions are taken as being merely confirmatory of its presence and as being regulatory. The power of eminent domain is essentially legislative in nature. the plain meaning of the law should not be enlarged by doubt[ful] interpretation. it requires no recognition by the Constitution. It is to be watched with jealous scrutiny. but limit a power which would otherwise be without limit. When the legislature interferes with that right. 3rd Div. and the rule in that case is that the authority must be strictly construed. and.. G. Held: The City of Manila. taking cognizance of such issues as the adequacy of compensation. 23. (Moday v. the Constitution of the Philippines 523-4 [2nd Ed. 2002. expropriation is not allowable. the power of eminent domain of respondent is contained in its original charter. In this jurisdiction.‖ two approaches are utilized – the first is public employment or the actual use by the public. Presidential Decree No. the statutory and judicial trend has been summarized as follows: This Court has ruled that the taking to be valid must be for public use. 146587. the public use requirement in eminent domain has evolved into a flexible concept. Otherwise.R. Meanwhile. then the power of eminent domain comes into play . 1977]) The term ―public use‖ has acquired a more comprehensive coverage. (Filstream International Incorporated v. as in the case of streets or parks. upon the other hand. Government may not capriciously choose what private property should be taken. urban land reform and housing. The ―public use‖ requirement for a valid exercise of the power of eminent domain is a flexible and evolving concept influenced by changing conditions. Held: This Court holds that respondent (Philippine Export Processing Zone) has the legal authority to expropriate the subject Lot 1406-B and that the same was for a valid public purpose. and to laborers and low-salaried employees of the city. There was a time when it was felt that a literal meaning should be attached to such a requirement. Whatever project is undertaken must be for the public to enjoy. State some limitations on the exercise of the power of Eminent Domain. Jan. It is therefore of no moment that the land sought to be expropriated in this case is less than half a hectare only. Court of Appeals. its demands upon the individual so increases. and the second is public advantage or benefit. It is also useful to view the matter as being subject to constant growth. has ruled that the necessity of exercising eminent domain must be genuine and of a public character. has the express power to acquire private lands in the city and subdivide these lands into home lots for sale to bona fide tenants or occupants thereof. Discuss the expanded notion of ―public use‖ in eminent domain proceedings. 84 . Public use now includes the broader notion of indirect public benefit or advantage. CA. In Manosca v.‖ Respondent PEZA expropriated the subject parcel of land pursuant to Proclamation No. No. In determining ―public use. [Vitug]) 139. Court of Appeals. It is not anymore. (Heirs of Juancho Ardona v. this Court has also held that what ultimately emerged is a concept of public use which is just as broad as ―public welfare. Fernando. and each demand is a new use to which the resources of the individual may be devoted. 268 SCRA 586. As long as the purpose of the taking is public. It is simply not possible to provide all at once land and shelter for all who need them. To the literal import of the term signifying strict use or employment by the public has been added the broader notion of indirect public benefit or advantage. Corollary to the expanded notion of public use. 66 x x x.proscriptions have their origin in the recognition of the necessity for achieving balance between the State interests. The Supreme Court. 1997) 140. Reyes. necessity of the taking and the public use character or the purpose of the taking. influenced by changing conditions. 125 SCRA 220 [1983] at 234-235 quoting E. What is the meaning of ―public use‖ in eminent domain proceedings? Illustrative case. In Sumulong v. July 2. 1st Div. 284 SCRA 716. . (Republic of the Philippines v. Court of Appeals. It is accurate to state then that at present whatever may be beneficially employed for the general welfare satisfies the requirement of public use. acting through its legislative branch. Through the years. Guerrero. which is to say that as society advances. 1998 [Francisco]) 141. by effectively restraining the former and affording protection to the latter. on the one hand. compensation must be made and due process of law must be observed. Held: The limitations on the power of eminent domain are that the use must be public. 1980 x x x issued by former President Ferdinand Marcos. expropriation is not anymore confined to vast tracts of land and landed estates. The Hon. this Court has ruled that. . and private rights. February 20. That only a few could actually benefit from the expropriation of the property does not diminish its public character. including in particular. Jan.‖ The authority is broad enough to give the respondent substantial leeway in deciding for what public use the expropriated property would be utilized. In such a manner. 2nd Div. respondent leased a portion of the lot to commercial banks while the rest was made a transportation terminal. Pursuant to this broad authority. v. 2001. Court of Appeals. (City of Manila v. this Court has already ruled that: X x x [T]he Legislature may directly determine the necessity for appropriating private property for a particular improvement for public use. subject Lot 1406-B was expropriated ―for the construction . . Discuss the meaning of ―just compensation‖ in eminent domain proceedings. The purpose of creating an ECOZONE and other facilities is better served if respondent directly owns the areas subject of the expansion program. the necessity and expediency of exercising the right of eminent domain are questions essentially political and not judicial in their character. The constitutional limitation of ―just compensation‖ is considered to be the sum equivalent to the market value of the property. 16. 349 SCRA 240. legal interests accrue in order to place the owner in a position as good as (but not better than) the position he was in before the taking occurred . PEZA. In fine. In the absence of some constitutional or statutory provisions to the contrary. The expropriation of adjacent areas therefore comes as a matter of necessity to bring life to the purpose of the law. it is well-settled that the utility of the proposed improvement. 146587. the final compensation must include interests on its just value to be computed from the time the property is taken to the time when compensation is actually paid or deposited with the court. Does it include the payment of ―interest‖ and. structures and approaches thereto. In Manila Railroad Co. Petitioner cannot impose or dictate on the respondent what facilities to establish for as long as the same are for public purpose. as well as the construction of terminal facilities. suffice it to say that PEZA can vary the purpose for which a condemned lot will be devoted to. Anent this issue. and it may select the exact location of the improvement. X x x The expropriation of Lot 1406-B for the purpose of being leased to banks and for the construction of a terminal has the purpose of making banking and transportation facilities easily accessible to the persons working at the industries located in PEZA. 2002. Petitioner contends that respondent is bound by the representations of its Chief Civil Engineer when the latter testified before the trial court that the lot was to be devoted for the construction of government offices. . of terminal facilities. Without prompt payment. this Court has ruled that in the exercise of eminent domain. The term ―necessary. [De Leon]) 142. Thus. are all questions exclusively for the legislature to determine. if so. (Estate of Salud Jimenez v. fixed at the time of the actual taking by the government. the existence of the public necessity for its construction.Accordingly. a law amending respondent PEZA‘s original charter x x x. bestow respondent with authority to develop terminal facilities and banking centers. We have ruled that the concept of just compensation embraces not only the correct determination of the amount to be paid to the owners of the land. this Court will not question the respondent‘s lease of certain portions of the expropriated lot to banks. 349 [1919]) Inasmuch as both Presidential Decree No. No. between the taking of the property and the actual payment. Chinese Community of Manila. does not mean absolutely indispensable but requires only a reasonable necessity of the taking for the stated purpose. and one who desires to sell it. and the courts have no power to interfere or to substitute their own views for those of the representatives of the people. PEZA‘s goal of being a major force in the economic development of the country would be realized. only as much land can be taken as is necessary for the legitimate purpose of the condemnation. Said public purposes were even reaffirmed by Republic Act No. 40 Phil. The Hon. [Vitug]) 2. In such a case. 7916. but also the payment of the land within a reasonable time from its taking. the expediency of constructing it. broadly described to be the price fixed by the seller in open market in the usual and ordinary course of legal action and competition or the fair value of the property as between one who receives. the suitableness of the location selected. provided that the same is for public use.‖ in this connection. July 2. 7916. growth and future needs of the enterprise. Furthermore. 66 and Republic Act No. (Republic of the Philippines v. The respondent cannot attain a self-sustaining and viable ECOZONE if inevitable needs in the expansion in the surrounding areas are hampered by the mere refusal of the private landowners to part with their properties. 1st Div.R. how is it to be computed? Held: 1. if property is taken for public use before compensation is deposited with the court having jurisdiction over the case. G. Mitchel. compensation cannot be considered 85 . that it violated the due process clause and the eminent domain provision of the Constitution by taking airtime from radio and television broadcasting stations without payment of just compensation. v. F.‖ Consequently.‖ (Art.‖ Will you sustain the challenge? Held: All broadcasting. 92 of B. Petitioners claim that the primary source of revenue of radio and television stations is the sale of airtime to advertisers and that to require these stations to provide free airtime is to authorize a taking which is not ―a de minimis temporary limitation or restraint upon the use of private property. whether by radio or by television stations. April 21. v. Sec. there are responsible scholars who believe that government controls on broadcast media can constitutionally be instituted to ensure diversity of views and attention to public affairs to further the system of free expression.C. They are merely given the temporary privilege of using them. In truth. be barred from the airwaves. A franchise is thus a privilege subject. Sec. which upheld the right of a party personally attacked to reply. Art. radio and television broadcasting companies. However.‖ What better measure can be conceived for the common good than one for free airtime for the benefit not only of candidates but even more of the public. particularly the voters. among others. 16. which are given franchises. Any delay in payment must be counted from said order. is licensed by the government. provisions for Comelec Time have been made by amendment of the franchises of radio and television broadcast stations and such provisions have not been thought of as taking property without just compensation. it is said.000.―just‖ inasmuch as the property owner is made to suffer the consequences of being immediately deprived of his land while being made to wait for a decade or more before actually receiving the amount necessary to cope with his loss. It would be strange if it cannot even require the licensees to render public service by giving free airtime.380. no private property is taken by the requirement that they provide airtime to the Comelec.. 11) Indeed. it must be deliberately done by a party in order to defeat the ends of justice. so that they will be fully informed of the issues in an election? ―[I]t is the right of the viewers and listeners. ―a license permits broadcasting. not the right of the broadcasters.‖ As radio and television broadcast stations do not own the airwaves. PEZA. As held in Red Lion Broadcasting Co. 1998 [Mendoza]) 144.P. Since a franchise is a mere privilege.00 in unrealized revenue from advertising is based on the assumption that airtime is ―finished product‖ which. Inc. to amendment by Congress in accordance with the constitutional provision that ―any such franchise or right granted x x x shall be subject to amendment. Jan. Blg. by necessity. among other things. COMELEC. which is paramount. broadcast stations may be required to give free airtime to candidates in an election. In the granting of the privilege to operate broadcast stations and thereafter supervising radio and television stations. Airwave frequencies have to be allocated as there are more individuals who want to broadcast than there are frequencies to assign. XII. alteration or repeal by the Congress when the common good so requires. The claim that petitioner would be losing P52. Payment of just compensation should follow as a matter of right immediately after the order of expropriation is issued. 881 (requiring radio and television station owners and operators to give to the Comelec radio and television time free of charge) was challenged on the ground. For this purpose. 2nd Div. 289 SCRA 337. (Estate of Salud Jimenez v. There is nothing in the First Amendment which prevents the government from requiring a licensee to share his frequency with others and to conduct himself as a proxy or fiduciary with obligations to present those views and voices which are representative of his community and which would otherwise.‖ Nor indeed can there be any constitutional objection to the requirement that broadcast stations give free airtime. the delay to constitute a violation of due process must be unreasonable and inexcusable. become the property of the company. Even in the United States.C. the State spends considerable public funds in licensing and supervising such stations. [De Leon]) 143. The constitutionality of Sec. 11 of the Constitution authorizes the amendment of franchises for ―the common good. like oil produced from refining or similar natural resources after undergoing a process for their production. 349 SCRA 240. (TELEBAP. but only the temporary privilege of using them. but the licensee has no constitutional right to be the one who holds the license or to monopolize a radio frequency to the exclusion of his fellow citizens. May eminent domain be barred by "res judicata" or "law of the case"? 86 . 2001. XII. do not own the airwaves and frequencies through which they transmit broadcast signals and images. the exercise of the privilege may reasonably be burdened with the performance by the grantee of some form of public service. ―licenses to broadcast do not confer ownership of designated frequencies. Notwithstanding the grant to individuals. Garcia. 137569. No. X x x 87 . (Republic v. However. (Republic v. ownership over the property being expropriated remains with the registered owner. 2000. Do the two (2) stages in expropriation apply only to judicial. [Mendoza]) 147. it does apply to specific issues decided in a previous case. Salem Investment Corporation. al. and the two stages in expropriation. is that title to the property expropriated shall pass from the owner to the expropriator only upon full payment of the just compensation.. Inc. it cannot. v. June 23. al. et. upon the payment of just compensation to be determined as of the date of the filing of the complaint" x x x. v. Salem Investment Corporation. the State or its authorized agent cannot be forever barred from exercising said right by reason alone of previous non-compliance with any legal requirement. et. [Mendoza]) 146. al. The De la Ramas make much of the fact that ownership of the land was transferred to the government because the equitable and the beneficial title was already acquired by it in 1983. bar the State or its agent from thereafter complying with this requirement. as this Court held in Association of Small Landowners in the Phil. and not to legislative. which finds application in generally all cases and proceedings. dictates that the right to exercise the power be absolute and unfettered even by a prior judgment or res judicata. as prescribed by law. Secretary of Agrarian Reform: (Republic v. just compensation is determined by the courts. can ―reach every form of property which the State might need for public use. leaving them with only the naked title. the latter can exercise all rights pertaining to an owner. Held: Expropriation may be initiated by court action or by legislation .. G. [I]t is only upon payment of just compensation that title over the property passes to the government.‖ This is done by the court with the assistance of not more than three (3) commissioners x x x. a final judgment dismissing an expropriation suit on the ground that there was no prior offer precludes another suit raising the same issue.. It is only upon the completion of these two stages that expropriation is said to have been completed. 137569. as an inherent power of the State. reiterated in National Power Corp.R. the highest and most exact idea of property. G. As explained in Municipality of Binan v. the eminent domain. [Mendoza]) 2. 2 nd Div. 2000. No. "of condemnation declaring that the plaintiff has a lawful right to take the property sought to be condemned. Realty Corporation. or in the aggregate body of the people in their sovereign capacity. and subsequently exercising its power of eminent domain over the same property. Therefore. expropriation? The recognized rule.. The first is concerned with the determination of the authority of the plaintiff to exercise the power of eminent domain and the propriety of its exercise in the context of the facts involved in the suit. No.R. 292 SCRA 678. including the right to dispose of his property. Discuss how expropriation may be initiated. June 23. Jurisprudence on this settled principle is consistent both here and in other democratic jurisdictions. Jocson: The expropriation of lands consists of two stages. G. Consequently. 2 nd Div. It ends with an order. (Municipality of Paranaque v. The scope of eminent domain is plenary and. et. In both instances. July 20. The very nature of eminent domain. 2000. like police power. indeed. V. Salem Investment Corporation. While the principle of res judicata does not denigrate the right of the State to exercise eminent domain.Held: The principle of res judicata. if not dismissal of the action. 137569. for the public use or purpose declared in the complaint. and they have the right to resume the possession of the property whenever the public interest requires it. For example. 2nd Div. 1998 [Panganiban]) 145. cannot bar the right of the State or its agents to expropriate private property. The second phase of the eminent domain action is concerned with the determination by the court of ―the just compensation for the property sought to be taken.M. June 23. remains in the government.R. however. subject to the power of the State ultimately to acquire it through expropriation. May the owner of the property expropriated still dispose of that property before the payment of just compensation? When does title over the property expropriated pass to the expropriator? Held: 1.‖ All separate interests of individuals in property are held of the government under this tacit agreement or implied reservation. until the action for expropriation has been completed and terminated.‖ Thus. a power of eminent. and direct the Solicitor General. x x x It follows that both by virtue of the judgment. [Panganiban]) 149. Jan. utilized and. Book III of the Revised Administrative Code. long final. As pointed out by the Solicitor General the current effective law on delegated authority to exercise the power of eminent domain is found in Section 12. this delegated power of eminent domain is not. Republic where the private landowners had remained unpaid ten years after the termination of the expropriation proceedings. but only of inferior. 12. 323 SCRA 862. Said relief may be granted under plaintiffs‘ prayer for: ‗such other remedies. (Republic v. in the expropriation suit. to which the power of eminent domain is not inherent. In insisting on the return of the expropriated property. June 23. Thus. Petitioner has occupied. petitioner insists that before eminent domain may be exercised by the state. exercised dominion over the property pursuant to the judgment. By final and executory judgment in said proceedings. When may the property owner be entitled to the return of the expropriated property in eminent domain cases? Held: 1.‖ The foregoing provision does not require prior unsuccessful negotiation as a condition precedent for the exercise of eminent domain. In Iron and Steel Authority v. 7160 cannot be understood as being the pervasive and all-encompassing power vested in the legislative branch of government. Court of Appeals. Court of Appeals. (SMI Development Corporation v. The landowner was merely given the relief of recovering compensation for his property computed at its market value at the time it was taken and appropriated by the State. for all intents and purposes. which provides: ―SEC. domain or only as broad or confined as the real authority would want it to be. Is prior unsuccessful negotiation a condition precedent for the exercise of eminent domain? Held: Citing Iron and Steel Authority v. The grant of the power of eminent domain to local governments under Republic Act No. are bound. In the instant case. 2nd Div. in what amount. 137569.Held: We see no point in distinguishing between judicial and legislative expropriation as far as the two stages mentioned above are concerned. as well as their privies. Said lots have been the subject of expropriation proceedings. Pasay City where the recovery of possession of property taken for public use prayed for by the unpaid landowner was denied even while no requisite expropriation proceedings were first instituted. Both involve these stages and in both the process is not completed until payment of just compensation is made. as part of an airport. but merely delegated and of limited application. al. which may be deemed just and equitable under the premises‘. they were condemned for public use. it must. but even then. no such voluntary restriction was imposed. De Villaroya where the unpaid landowners were allowed the alternative remedy of recovery of the property there in question. 2000. No. the President chose to prescribe this condition as an additional requirement instead. and ordered sold to the government. G. plaintiffs are not entitled to recover possession of their expropriated lots – which are still devoted to the public use for which they were expropriated – but only to demand the fair market value of the same. The judgment rendered by the Bulacan RTC in 1979 on the expropriation proceedings provides not only for the payment of just compensation to herein respondents but likewise adjudges the property condemned in favor of petitioner over which parties. [Mendoza]) 148. be delegated to it by the national legislature. there must be a showing of prior unsuccessful negotiation with the owner of the property to be expropriated. respondents would exhort on the pronouncement in Provincial Government of Sorsogon v.R. For local governments to be able to wield the power.‖ The Court proceeded to reiterate its pronouncement in Alfonso v. The exercise of such rights vested to it as the condemnee indeed has amounted to at least a 88 . this Court ruled – ―The points in dispute are whether such payment can still be made and. 2000. in Valdehueza v. by enabling law. This contention is not correct. Vda. Power of Eminent Domain – The President shall determine when it is necessary or advantageous to exercise the power of eminent domain in behalf of the National Government. 3rd Div. et. Salem Investment Corporation. to institute expropriation proceedings in the proper court. whenever he deems the action advisable. strictly speaking.. as well as the annotations upon their title certificates. if so. 28. Republic. however. It might be borne in mind that the case involved the municipal government of Sorsogon. Accordingly. Debts are due to the Government in its corporate capacity. immediate return to the owners of the unpaid property is the obvious remedy. may have guided the people in ratifying the Charter. however. Under Article XIV. 16. however. [De Leon]) The Power of Taxation 150. the trial court is directed to seize any patrimonial property or cash savings of the province in the amount necessary to implement this decision. to which the remedy of rescission might perhaps apply. 1991. 2 nd Div. and exclusively for educational purposes shall be exempt from taxes and duties. this would adversely affect the government revenue system. actually. who is now a member of this Court. Oct. buildings. Section 28. those exempted from real estate taxes are lands. Justice Hilario G. directly and exclusively used for religious. July 2. paragraph 3 of the 1987 Constitution. directly and exclusively used for religious. If any taxpayer can defer the payment of taxes by raising the defense that it still has a pending claim for refund or credit. this Court declared in Sorsogon that ―the Provincial Government of Sorsogon is expected to immediately pay as directed. Such intent must be effectuated. directly. In his treatise on taxation. 28. There is a material distinction between a tax and debt. Jr. 349 SCRA 240. No. As in other cases where there was no prompt payment by the government. G. "[C]haritable institutions. adhered to the same view that the exemption created by said provision pertained only to property taxes. Court of Appeals.R. charitable or educational purposes shall be exempt from taxation. non-profit cemeteries. (Republic of the Philippines v. Aug. we cannot. Resolve.‖ However. the paramount title is in the public under a new and independent title. Should any further delay be encountered. A taxpayer cannot refuse to pay his taxes when they fall due simply because he has a claim against the government or that the collection of a tax is contingent on the result of the lawsuit it filed against the government." (Commissioner of Internal Revenue v. Held: The debates. must be considered.‖ (Estate of Salud Jimenez v. 2001. Jan. 146587. 14. 2002. condemnation acts upon the property." YMCA alleged that it "is a non-profit educational 89 . It must be noted that a distinguishing feature of a tax is that it is compulsory rather than a matter of bargain. has long become final and executory. The Hon. 1998 [Romero]) 151. PEZA. Davide. 1st Div." YMCA claims that the income earned by its building leased to private entities and that of its parking space is likewise covered by said exemption. an eminent authority on the Constitution and also a member of the Concom. De Villaroya to support its contention that it is entitled to a return of the lot where this Court ruled that ―under ordinary circumstances. charitable or educational purposes. churches and parsonages or convents appurtenant thereto. interpellations and expressions of opinion of the framers of the Constitution reveal their intent that which. stating that "[t]he tax exemption covers property taxes only. 1998 [Panganiban]) 152. condemnation proceedings provide a judicial process for securing better title against all the world than may be obtained by voluntary conveyance . Vda. Father Joaquin G. 1406-B. the said statement was not the ruling in that case. mosques. Commissioner of Internal Revenue.‖ However. thus. and all lands. paragraph 3 of the 1987 Constitution. public interest. Can taxes be subject to off-setting or compensation? Held: Taxes cannot be subject to compensation for the simple reason that the government and the taxpayer are not creditors and debtors of each other .. Vitug concurs. Under Article VI. 294 SCRA 687. by giving notice to all claimants to a disputed title. After condemnation. An in rem proceeding. Rosa E. In arguing for the return of their property on the basis of nonpayment. [Vitug]) 2. The Order of expropriation dated July 11. non-profit educational institutions used actually. thereby preempting any claim of bar by prescription on grounds of non-execution.partial compliance or satisfaction of the 1979 judgment. (Philex Mining Corporation v. this Court also stressed and declared in that case that ―in cases where land is taken for public use. grant the petitioner‘s prayer for the return of the expropriated Lot No. Justice Jose C. respondents ignore the fact that the right of the expropriatory authority is far from that of an unpaid seller in ordinary sales. Mr. Petitioner cited Provincial Government of Sorsogon v. Bernas. Hence. in turn. a tax does not depend upon the consent of the taxpayer. CA. a former constitutional commissioner. "[A]ll revenues and assets of non-stock. Though the respondent has committed a misdeed to petitioner. and improvements. while taxes are due to the Government in its sovereign capacity. stressed during the Concom debates that "x x x what is exempted is not the institution itself x x x. 298 SCRA 83. Section 4. buildings and improvements actually. Dec. it must prove with substantial evidence that (1) it falls under the classification non-stock. even non-formal education is understood to be school-based and ―private auspices such as foundations and civic-spirited organizations‖ are ruled out. 3 of the Constitution? Held: We rule that it is not.institution whose revenues and assets are used actually. More precisely.‖ The Court has examined the ―Amended Articles of Incorporation‖ and ―By-Laws‖ of the YMCA. par. Hence. It will also violate the constitutional rule that ―taxation shall be uniform and equitable . Oct. 1998 [Panganiban]) 153. [Gonzaga-Reyes]) 156. Is the YMCA an educational institution within the purview of Article XIV. even under the cover of its authority to compromise ill-gotten wealth cases. 101-102. Even granting that Congress enacts a law exempting the Marcoses from paying taxes on their properties. Oct. and (2) the income it seeks to be exempted from taxation is used actually. (Commissioner of Internal Revenue v. directly. refers to a ―x x x school seminary. directly and exclusively for educational purposes so it is exempt from taxes on its properties and income. 1999. The term ―educational institution‖ or ―institution of learning‖ has acquired a well-known technical meaning. (Commissioner of Internal Revenue v. Section 4. The purpose of these international agreements is to reconcile the national fiscal legislations of the contracting parties in order to help the taxpayer avoid simultaneous taxation in two different jurisdictions . under the Education Act of 1982. It is settled that the term ―educational institution. S. 1998 [Panganiban]) 155. Under the Education Act of 1982. the tax conventions are drafted with a view towards the elimination of international juridical double taxation x x x.‖ (84 CJS 566) Therefore.C. 298 SCRA 83.." Held: We reiterate that private respondent is exempt from the payment of property tax. 298 SCRA 83. The bare allegation alone that it is a non-stock. 1999) 90 . but not income tax on the rentals from its property. The school system is synonymous with formal education. Discuss the purpose of tax treaties? Held: The RP-US Tax Treaty is just one of a number of bilateral treaties which the Philippines has entered into for the avoidance of double taxation. but found nothing in them that even hints that it is a school or an educational institution. such law will definitely not pass the test of the equal protection clause under the Bill of Rights. May the PCGG validly commit to exempt from all forms of taxes the properties to be retained by the Marcos heirs in a Compromise Agreement between the former and the latter? Held: The power to tax and to grant exemptions is vested in the Congress and. specifically provides: ―No law granting any tax exemption shall be passed without the concurrence of a majority of all the members of the Congress.‖ when used in laws granting tax exemptions. Johnson and Son. 1998 [Panganiban]) 154. Johnson and Son. (Commissioner of Internal Revenue v.C. CA.‖ The PCGG has absolutely no power to grant tax exemptions. Article VI of the Constitution. (Commissioner of Internal Revenue v. 309 SCRA 87. the Court notes that not a scintilla of evidence was submitted by private respondent to prove that it met the said requisites. CA. in the local legislative bodies. non-profit educational institution. the private respondent cannot be deemed one of the educational institutions covered by the constitutional provision under consideration. 309 SCRA 87. 14. However. of which the members of the Constitutional Commission are deemed cognizant. 14. Inc. which ―refers to the hierarchically structured and chronologically graded learnings organized and provided by the formal school system and for which certification is required in order for the learner to progress through the grades or move to the higher levels. Section 28(4). to a certain extent. college or educational establishment x x x. 9. [L]aws allowing tax exemption are construed strictissimi juris. June 25. 299 SCRA 744. Furthermore. such term refers to schools. non-profit educational institution is insufficient to justify its exemption from the payment of income tax. 102. for the YMCA to be granted the exemption it claims under the abovecited provision. June 25.‖ (Chavez v. and exclusively for educational purposes. Any special grant of tax exemption in favor only of the Marcos heirs will constitute class legislation. S. What is ―international juridical double taxation‖? Held: It is defined as the imposition of comparable taxes in two or more states on the same taxpayer in respect of the same subject matter and for identical periods.. 3rd Div. PCGG. Inc. for other items of income or capital. process. 1997 [Romero]) 160. the other contracting state and both states impose tax on that income or capital. reasonable. not merely the procedures by which the law would be enforced. Discuss the Due Process Clause. two conditions must concur. S. 309 SCRA 87. the treaties make it incumbent upon the state of residence to allow relief in order to avoid double taxation. Inc. technology and persons between countries.‖ and ―(a)ll appointments to harbor pilot positions in all pilotage districts shall. and just. Johnson and Son. 1999) 158. 91 . although the amount of tax that may be imposed by the state of source is limited.the exemption method and the credit method.‖ (Corona v.C. whereas the credit method focuses upon the tax.. In order to fall within the aegis of this provision. 103. Inc. The second method for the elimination of double taxation applies whenever the state of source is given a full or limited right to tax together with the state of residence. although in some instances it may be taken into account in determining the rate of tax applicable to the taxpayer's remaining income or capital. They also contended that the sole and exclusive right to the exercise of harbor pilotage by pilots has become vested and can only be ―withdrawn or shortened‖ by observing the constitutional mandate of due process of law.C. an exclusive right to tax is conferred on one of the contracting states. although the income or capital which is taxed in the state of source is still taxable in the state of residence.‖ allegedly because no hearing was conducted whereby ―relevant government agencies‖ and the harbor pilots themselves could ventilate their views. be only for a term of one (1) year from date of effectivity subject to renewal or cancellation by the Philippine Ports Authority after conduct of a rigid evaluation of performance. June 25. 102- 103. June 25. In essence. the focus is on the income or capital itself.157. When one speaks of due process of law. Respondents United Harbor Pilots Association of the Philippines argue that due process was not observed in the adoption of PPA-AO No. the underlying rationale for reducing the tax rate is that the Philippines will give up a part of the tax in the expectation that the tax given up for this particular investment is not taxed by the other country. What is the rationale for reducing the tax rate in negotiating tax treaties? Held: In negotiating tax treaties. Double taxation usually takes place when a person is resident of a contracting state and derives income from. In order to eliminate double taxation. The basic difference between the two methods is that in the exemption method. (Commissioner of Internal Revenue v. both states are given the right to tax. 04-92 which provides that: ―(a)ll existing regular appointments which have been previously issued by the Bureau of Customs or the PPA shall remain valid up to 31 December 1992 only. however. What is the rationale for doing away with international juridical double taxation? What are the methods resorted to by tax treaties to eliminate double taxation? Held: The apparent rationale for doing away with double taxation is to encourage the free flow of goods and services and the movement of capital. namely. henceforth. 12. is fair. S. the income or capital which is taxable in the state of source or situs is exempted in the state of residence. a distinction must be made between matters of procedure and matters of substance. procedural due process ―refers to the method or manner by which the law is enforced. In this case. 283 SCRA 31. 1999) THE BILL OF RIGHTS The Due Process Clause 159. In the exemption method.. Dec. First. Foreign investments will only thrive in a fairly predictable and reasonable international investment climate and the protection against double taxation is crucial in creating such a climate. 309 SCRA 87.. On the other hand. in the credit method. a tax treaty resorts to several methods. that there is a deprivation and that such deprivation is done without proper observance of due process. or owns capital in. United Harbor Pilots Association of the Phils. the tax paid in the former is credited against the tax levied in the latter. (Commissioner of Internal Revenue v. it sets out the respective rights to tax of the state of source or situs and of the state of residence with regard to certain classes of income or capital. In some cases.‖ while substantive due process ―requires that the law itself. conditions deemed vital in creating robust and dynamic economies. however. Distinguish substantive due process from procedural due Held: Section 1 of the Bill of Rights lays down what is known as the ―due process clause‖ of the Constitution. There are two methods of relief . Johnson and Son. just like other professions. December 12. Their argument has thus shifted from the procedural to one of substance. In the case at bar. they have to contend with an annual cancellation of their license which can be temporary or permanent depending on the outcome of their performance evaluation. 283 SCRA 31. an administrative body need not comply with the requirements of notice and hearing. Licensure is ―the granting of license especially to practice a profession. in an administrative inquiry.. Exevea. this constitutional mandate is deemed satisfied if a person is granted an opportunity to seek reconsideration of the action or ruling complained of. X x x Their license is granted in the form of an appointment which allows them to engage in pilotage until they retire at the age of 70 years. In the performance of its executive or legislative functions. Under the new issuance. 04-92 fails to meet the condition set by the organic law. In a real sense. as the fundamental requirements of procedural due process. may be practiced only by duly licensed individuals. failed to persuade.‖ It is this pre-evaluation cancellation which primarily makes PPA-AO No. the Court has maintained a clear position in this regard. however. Lumiqued. where it declared that ―(a)s long as a party was given the opportunity to defend his interests in due course. 04-92 no less than four times before the matter was finally elevated to this Tribunal. for this opportunity to be heard is the very essence of due process. is a right afforded a suspect or an accused during custodial investigation. each followed by actual training and practice. with more reason. are essential only when an administrative body exercises its quasi-judicial function. Does the due process clause encompass the right to be assisted by counsel during an administrative inquiry? Held: The right to counsel. which cannot be waived unless the waiver is in writing and in the presence of counsel. As a general rule. Respondents aver that said right has become vested and can only be ―withdrawn or shortened‖ by observing the constitutional mandate of due process of law. Before harbor pilots can earn a license to practice their profession. be only for a term of one (1) year from date of effectivity subject to renewal or cancellation by the Authority after conduct of a rigid evaluation of performance. Fortunately. Upon the other hand. In the past. 1997 [Romero]) 161. he cannot be said to have been denied due process of law. be invoked or rejected in a criminal proceeding and.‖ and ―(a)ll appointments to harbor pilot positions in all pilotage districts shall. the use of the term ―renewal. 04-92.‖ It is also ―the system of granting licenses (as for professional practice) in accordance with established standards.‖ A license is a right or permission granted by some competent authority to carry on a business or do an act which. Xxx Neither does the fact that the pilots themselves were not consulted in any way taint the validity of the administrative order.Held: They are obviously referring to the procedural aspect of the enactment. This is a vested right. United Harbor Pilots Association of the Phils. 04-92 unduly restricts the right of harbor pilots to enjoy their profession before their compulsory retirement. was not accused of any crime in the proceedings below. would be illegal. not one but five examinations. notice and hearing. thus. it is a deprivation of property without due process of law. 04-92 unreasonable and constitutionally infirm. (Corona v. such as issuing rules and regulations. It is not an absolute right and may. unless sooner revoked by the PPA for mental or physical unfitness.‖ In the case at bar. ―[a]ll existing regular appointments which have been previously issued by the Bureau of Customs or the PPA shall remain valid up to 31 December 1992 only. it is also contended that the sole and exclusive right to the exercise of harbor pilotage by pilots is a settled issue. Their arguments on this score. they enjoyed a measure of security knowing that after passing five examinations and undergoing years of on-the-job training. Under the terms of PPA-AO No. Hon. Veteran pilots and neophytes alike are suddenly confronted with one-year terms which ipso facto expire at the end of that period. It is here where PPA-AO No. respondents questioned PPA-AO No. they literally have to pass through the proverbial eye of a needle by taking. Moreover. Hence. Pilotage. they would have a license which they could use until their retirement. a stance it has stressed in the recent case of Lumiqued v. The investigation conducted by the committee x x x was for the sole purpose of determining if he could be held administratively liable under the law for the 92 . however. henceforth. petitioners invoke the right of an accused in criminal proceedings to have competent and independent counsel of his own choice. without such license. Renewal of their license is now dependent on a ―rigid evaluation of performance‖ which is conducted only after the license has already been cancelled.‖ It is readily apparent that PPA-AO No. while no challenge may be mounted as against the second whenever directed 93 . It is to be distinguished. the nature of which he is given no fair warning. 282 SCRA 125. Lumiqued v. [Kapunan]) 2. in Coates v. the right to counsel is not imperative in administrative investigations because such inquiries are conducted merely to determine whether there are facts that merit disciplinary measures against erring public officers and employees.complaints filed against him. it may be ―saved‖ by proper construction. 24. the ordinance imposed no standard at all ―because one may never know in advance what annoys some people but does not annoy others. or to those that are apparently ambiguous yet fairly applicable to certain types of activities. Distinguish a ―perfectly vague act‖ from ―legislation couched in imprecise language.‖ or is so indefinite that ―it encourages arbitrary and erratic arrests and convictions. A statute or act may be said to be vague when it lacks comprehensible standards that men of common intelligence must necessarily guess at its meaning and differ in its application. especially the parties targeted by it. fair notice of what conduct to avoid.‖ It is repugnant to the Constitution in two respects: (1) it violates due process for failure to accord persons. x x x Thus. the U. x x x As such. but is commonly stated to the effect that a statute establishing a criminal offense must define the offense with sufficient definiteness that persons of ordinary intelligence can understand what conduct is prohibited by the statute. It can only be invoked against that specie of legislation that is utterly vague on its face. and. 1st Div. In an administrative proceeding x x x a respondent x x x has the option of engaging the services of counsel or not. and (2) it leaves law enforcers unbridled discretion in carrying out its provisions and become an arbitrary flexing of the Government muscle. We reiterated these principles in People v. and why is it repugnant to the Constitution. Dela Piedra. however.‖ Clearly. that is to say. But the doctrine does not apply as against legislations that are merely couched in imprecise language but which nonetheless specify a standard though defectively phrased. X x x (People v. The right to counsel is not indispensable to due process unless required by the Constitution or the law. We added. Exevea. A criminal statute that ―fails to give a person of ordinary intelligence fair notice that his contemplated conduct is forbidden by the statute. X x x While investigations conducted by an administrative body may at times be akin to a criminal proceeding. and no duty rests on such a body to furnish the person being investigated with counsel. 2001. Due process requires that the terms of a penal statute must be sufficiently explicit to inform those who are subject to it what conduct on their part will render them liable to its penalties. it cannot be clarified by either a saving clause or by construction. i. The constitutional vice in a vague or indefinite statute is the injustice to the accused in placing him on trial for an offense. Thus. a party in an administrative inquiry may or may not be assisted by counsel. however. Jan.‖ is void for vagueness. In such instance.. especially the parties targeted by it. City of Cincinnati. 1997 [Romero]) 162. Nazario: As a rule. Nov. irrespective of the nature of the charges and of the respondent's capacity to represent himself.‖ Coates highlights what has been referred to as a ―perfectly vague‖ act whose obscurity is evident on its face. from legislation coached in imprecise language – but which nonetheless specifies a standard though defectively phrased – in which case. the fact remains that under existing laws.‖ Held: 1. that: X x x the act must be utterly vague on its face. it leaves law enforcers unbridled discretion in carrying out its provisions and becomes an arbitrary flexing of the Government muscle. the hearing conducted by the investigating committee was not part of a criminal prosecution. fair notice of the conduct to avoid. a statute or act may be said to be vague when it lacks comprehensible standards that men ―of common intelligence must necessarily guess at its meaning and differ as to its application. Discuss the ―Void for Vagueness‖ Doctrine. 350 SCRA 163.e. The doctrine has been formulated in various ways. that which cannot be clarified either by a saving clause or by construction.S. 18. The first may be ―saved‖ by proper construction. Supreme Court struck down an ordinance that had made it illegal for ―three or more persons to assemble on any sidewalk and there conduct themselves in a manner annoying to persons passing by. the statute is repugnant to the Constitution in two (2) respects – it violated due process for failure to accord persons. with the purpose of maintaining the dignity of government service. It must be stressed. as petitioner seems to suggest. The prohibition in Adiong. therefore. Evidently. [Kapunan]) 164. Dela Piedra. Section 2 is 94 . If at all. Appellant‘s reliance on People v. Does Article 13 (b) of the Labor Code defining ―recruitment and placement‖ violate the due process clause? People v. where this Court x x x ―criticized‖ the definition of ―recruitment and placement‖ x x x. Jan. 148560. Held: In support of her submission that Article 13 (b) is void for vagueness. Panis is misplaced. appellant misapprehends concept of overbreadth. A generally worded statute. in Panis. 19. when construed to punish conduct which cannot be constitutionally punished is unconstitutionally vague to the extent that it fails to give adequate warning of the boundary between the constitutionally permissible and the constitutionally impermissible applications of the statute . appellant did not even specify what constitutionally protected freedoms are embraced by the definition of ―recruitment and placement‖ that would render the same constitutionally overbroad. Commission on Elections. The absence of such records notwithstanding.against such activities. that the ―vagueness‖ doctrine merely requires a reasonable degree of certainty for the statute to be upheld – not absolute precision or mathematical exactitude. It is not void for vagueness. Flexibility. 1st Div. X x x The Court. An act will not be held invalid merely because it might have been more explicit in its wordings or detailed in its provisions. No. The test in determining whether a criminal statute is void for uncertainty is whether the language conveys a sufficiently definite warning as to the proscribed conduct when measured by common understanding and practice. Xxx That Section 13(b) encompasses what appellant apparently considers as customary and harmless acts such as ―labor or employment referral‖ (―referring‖ an applicant. for instance. Panis. 24. 2001. because of the nature of the act. under the proviso of Article 13(b). the Court was able to arrive at a reasonable interpretation of the proviso by applying principles in criminal law and drawing from the language and intent of the law itself. a person may be convicted of illegal recruitment. We held that the challenged provisions not only deprived the owner of the vehicle the use of his property but also deprived the citizen of his right to free speech and information. the doctrine cannot be invoked where the assailed statute is clear and free from ambiguity. we struck down as void for overbreadth provisions prohibiting the posting of election propaganda in any place – including private vehicles – other than in the common poster areas sanctioned by the COMELEC. Nov. In the present case. the crime of illegal recruitment could be committed only ―whenever two or more persons are in any manner promised or offered any employment for a fee. merely bemoaned the lack of records that would help shed light on the meaning of the proviso. En Banc [Bellosillo]) 163.‖ The Court held in the negative x x x. it would be impossible to provide all the details in advance as in all other statutes. Is the Plunder Law unconstitutional for being vague? Held: As it is written. 350 SCRA 163. The issue in Panis was whether. (People v. is not a ―perfectly vague act‖ whose obscurity is evident on its face. was so broad that it covered even constitutionally guaranteed rights and. for employment to a prospective employer) does not render the law overbroad. therefore. the proviso therein is merely couched in imprecise language that was salvaged by proper construction. such as the freedom of speech or religion.R. however. Section 13(b). A statute may be said to be overbroad where it operates to inhibit the exercise of individual freedoms affirmatively guaranteed by the Constitution. With more reason. Sandiganbayan [Third Division]. (Joseph Ejercito Estrada v. the Plunder Law contains ascertainable standards and well-defined parameters which would enable the accused to determine the nature of his violation. as in this case. rather than meticulous specificity. G. however. These contentions cannot be sustained. In Blo Umpar Adiong v. hence. especially where. void for overbreadth. appellant invokes Appellant further argues that the acts that constitute ―recruitment and placement‖ suffer from overbreadth since by merely ―referring‖ a person for employment. is permissible as long as the metes and bounds of the statute are clearly delineated. 2001. Moreover. conduct and conditions required or forbidden. the counsel. The intention of the lawmakers – who are. 1. On the other hand. e. Verily. petitioner is completely informed of the accusations against him as to enable him to prepare for an intelligent defense. (d). Series – a number of things or events of the same class coming one after another in spatial and temporal succession. par. par. all of which fall under Sec. untrained philologists and lexicographers – to use statutory phraseology in such a manner is always presumed. (1). (d). the act or process of combining. Xxx Thus when the Plunder Law speaks of ―combination. These omissions. say. hence. and its inability to so define the words employed in a statute will not necessarily result in the vagueness or ambiguity of the law so long as the legislative will is clear. render the Plunder Law unconstitutional for being impermissibly vague and overbroad and deny him the right to be informed of the nature and cause of the accusation against him. can be gathered from the whole act. That Congress intended the words ―combination‖ and ―series‖ to be understood in their popular meanings is pristinely evident from the legislative deliberations on the bill which eventually became RA 7080 or the Plunder Law x x x. indicating with reasonable certainty the various elements of the offense which petitioner is alleged to have committed x x x. and Sec. bewails the failure of the law to provide for the statutory definition of the terms ―combination‖ and ―series‖ in the key phrase ―a combination or series of overt or criminal acts‖ found in Sec. to constitute a ―series‖ there must be two (2) or more overt or criminal acts falling under the same category of enumeration found in Sec. there is no positive constitutional or statutory command requiring the legislature to define each and every word in an enactment. (3). 1. in defending one charged with its violation. A statute is not rendered uncertain and void merely because general terms are used therein. and the word ―pattern‖ in Sec. plain and ordinary acceptation and signification. 1. (d). 4. much less do we have to define every word we use. 2. Besides. Although subject to proof.000. malversation and raids on the public treasury. however. It must sufficiently guide the judge in its application.sufficiently explicit in its description of the acts.‖ it is referring to at least two (2) acts falling under different categories or enumeration provided in Sec. X x x As long as the law affords some comprehensible guide or rule that would inform those who are subject to it what conduct would render them liable to its penalties. subpar. We discern nothing in the foregoing that is vague or ambiguous – as there is obviously none – that will confuse petitioner in his defense. par. Thus. unless it is evident that the legislature intended a technical or special legal meaning to those words. (d). and prescribes the elements of the crime with reasonable certainty and particularity. Indeed. The rationalization seems to us to be pure sophistry. had the legislature intended a technical or distinctive meaning for ―combination‖ and ―series. which is distinctly expressed in the Plunder Law. par.g. par. according to petitioner. (1).. 1. misappropriation. its validity would be sustained. 1. it is a well-settled principle of legal hermeneutics that words of a statute will be interpreted in their natural. subpar.00 through a series or combination of acts enumerated in Sec. Petitioner. 95 . (d). in identifying the realm of the proscribed conduct. Upon such unequivocal assertions. par. the accused. Webster‘s New Collegiate Dictionary contains the following commonly accepted definition of the words ―combination‖ and ―series. In fact. of the Plunder Law. ordinarily. the amended Information itself closely tracks the language of the law. (d). subpar. Congress is not restricted in the form of expression of its will. and fraudulent conveyance of assets belonging to the National Government under Sec. these factual assertions clearly show that the elements of the crime are easily understood and provide adequate contrast between the innocent and the prohibited acts.‖ Combination – the result or product of combining. or because of the employment of terms without defining them. it can be understood with little difficulty that what the assailed statute punishes is the act of a public officer in amassing or accumulating ill-gotten wealth of at least P50.000. raids on the public treasury in Sec. To combine is to bring into such close relationship as to obscure individual characters. 1. violative of his fundamental right to due process. par. (d).‖ it would have taken greater pains in specifically providing for it in the law. and more importantly. or at least. 1. if there is no such overall scheme or where the schemes or methods used by multiple accused vary. since the challenger must establish that no set of circumstances exists under which the Act would be valid. Ed 325. 2d 231 [1960]) A facial challenge is allowed to be made to a vague statute and to one which is overbroad because of possible ―chilling effect‖ upon protected speech. The overbreadth and vagueness doctrine then have special application only to free speech cases. 601.‖ (Gooding v. As the U. Alabama.‖ For this reason. Xxx Hence. Xxx Moreover.‖ doctrine. 612-613. pursuant to Sec. 1 (d). City Mayor. Co. and.As for ―pattern. 413 [1972] [internal quotation marks omitted]) The possible harm to society in permitting some unprotected speed to go unpunished is outweighed by the possibility that the protected speech of others may be deterred and perceived grievances left to fester because of possible inhibitory effects of overly broad statutes. The theory is that ―[w]hen statutes regulate or proscribe speech and no readily apparent construction suggests itself as a vehicle for rehabilitating the statutes in a single prosecution. it has been held that ―a facial challenge to a legislative act is the most difficult challenge to mount successfully. In the alternative. Supreme Court put it. 2 – x x x under Sec. that ―overbreadth claims. Ed. (d).S. Oklahoma (413 U. we agree with. Under the circumstances. the Court ruled that ―claims of facial overbreadth have been entertained in cases involving statutes which. petitioner‘s reliance on the ―void-forvagueness‖ doctrine is manifestly misplaced. 307. hence we adopt. 867 [1967]) The overbreadth (NAACP v. the overt or criminal acts must form part of a conspiracy to attain a common goal.S.S. 288.) As 96 . 269 U. Criminal statutes have general in terrorem effect resulting from their very existence. the pattern of overt or criminal acts is directed towards a common purpose or goal which is to enable the public officer to amass.S. in relation to Sec. a ‗pattern‘ consists of at least a combination or series of overt or criminal acts enumerated in subsections (1) to (6) of Sec. 521. on the other hand. have been curtailed when invoked against ordinary criminal laws that are sought to be applied to protected conduct.‖ In Broadwick v. 338 [1958]. 479. As commonly understood. 31 L. 840-841 [1973]). 2 L. This rationale does not apply to penal statutes. Tucker. the law cannot take chances as in the area of free speech. 70 L. decrees that ―a governmental purpose may not be achieved by means which sweep unnecessarily broadly and thereby invade the area of protected freedoms. supra. Shelton v. 385. accumulate or acquire ill-gotten wealth. Ed. They are inapt for testing the validity of penal statutes.S. Secondly.‖ we agree with the observations of the Sandiganbayan that this term is sufficiently defined in Sec.S. if facial challenge is allowed for this reason alone. In the area of criminal law. 20 SCRA 849. 37 L Ed. And thirdly. par. the transcendent value to all society of constitutionally protected expression is deemed to justify allowing attacks on overly broad statutes with no requirement that the person making the attack demonstrate that his own conduct could not be regulated by a statute drawn with narrow specificity. Salerno. ―we have not recognized an ‗overbreadth‘ doctrine outside the limited context of the First Amendment. 1. by their terms. 12. 2d 830. 328 [1926] cited in Ermita-Malate Hotel and Motel Operators Ass‘n. again. General Constr. 518. 391. 2 of the law.‖ (United States v. the observations of Mr. 405 U.. 377 U. 2d 408. 4. 364 U. 1 (d) of the law. there must either be an ‗overall unlawful scheme‘ or ‗conspiracy‘ to achieve said common goal. seek to regulate only spoken words‖ and. Ed. the State may well be prevented from enacting laws against socially harmful conduct. 5 L. it cannot plausibly be contended that the law does not give a fair warning and sufficient notice of what it seeks to penalize. Justice Vicente V. the term ‗overall unlawful scheme‘ indicates a ‗general plan of action or method‘ which the principal accused and public officer and others conniving with him follow to achieve the aforesaid common goal. if entertained at all. in an opinion by Chief Justice Rehnquist. Wilson. Mendoza during the deliberations of the Court that the allegations that the Plunder Law is vague and overbroad do not justify a facial review of its validity – The void-for-vagueness doctrine states that ―a statute which either forbids or requires the doing of an act in terms so vague that men of common intelligence must necessarily guess at its meaning and differ as to its application violates the first essential of due process of law. v.‖ (Connally v. and Sec. petitioner must be aware that the law was extensively deliberated upon by the Senate and its appropriate committees by reason of which he even registered his affirmative vote with full knowledge of its legal implications and sound constitutional anchorage. x x x ordinarily results in a kind of case that is wholly unsatisfactory for deciding constitutional questions. ―on its face‖ invalidation of statutes results in striking them down entirely on the ground that they might be applied to parties not before the Court whose activities are constitutionally protected (Id. it is said that a litigant may challenge a statute on its face only if it is vague in all its possible applications. as the U. 580 [1998]) and is generally disfavored (FW/PBS. 565-6 [1963]) statute. 19. To be sure. Electoral Commission. Harris (401 U. 27 L. 158 [1936]). En Banc [Bellosillo]) 165. where none exists.S. 2d 362. No. Finley.2d at 841. 52-53. 17.2d 603 [1990]. The combination of the relative remoteness of the controversy. 680 [1971]. 9 L. 37 L. National Endowment for the Arts v. 362 U. Indeed. Compare Angara v. Sections 1 and 5. Ed. at 1328).for the vagueness doctrine. No. the impact on the legislative process of the relief sought. 529 [1960]. Corp.‖ (United States v. Ambiguity. 32-33. Cruz v.S. 139. the established rule is that ―one to whom application of a statute is constitutional will not be heard to attack the statute on the ground that impliedly it might also be taken as applying to other persons or other situations in which its application might be unconstitutional.) [T]he task of analyzing a proposed statute. Sandiganbayan [Third Division].Ed. 524 U. (Joseph Ejercito Estrada v. 63 Phil. 455 U. In determining the constitutionality of a National Dairy Prod. 489.2d 561. ―vagueness challenges in the First Amendment context.R. Does an extraditee have the right to notice and hearing during the evaluation stage of an extradition proceeding? Held: Considering that in the case at bar. 148560. and vagueness are analytical tools developed for testing ―on their faces‖ statutes in free speech cases or. there is no basis for petitioner‘s claim that this Court review the AntiPlunder Law on its face and in its entirety. For these reasons. VIII. so tenaciously claimed and argued at length by petitioner. the nature of the right being claimed by the private respondent is nebulous and the degree of 97 .‖ to be employed ―sparingly and only as a last resort. 494-95.S.Ed. pinpointing its deficiencies.S. petitioner cannot feign ignorance of what the Plunder Law is all about. Every provision of the law should be construed in relation and with reference to every other part. G.‖ (Village of Hoffman Estates v. G. 57 l. It constitutes a departure from the case and controversy requirement of the Constitution and permits decisions to be made without concrete factual settings and in sterile abstract contexts (Constitution. 71 L Ed.Ed. 2001.S. Ed. is more imagined than real.‖ (Broadwick v. Raines. cannot be created by dissecting parts and words in the statute to furnish support to critics who cavil at the want of scientific precision in the law. They cannot be made to do service when what is involved is a criminal statute. as they are called in American law. City of Dallas. Ed. Nov. its provisions which are alleged to have been violated in a case must be examined in the light of the conduct with which the defendant is charged (United States v.S. 372 U. 4 L.. and requiring correction of these deficiencies before the statute is put into effect. Constitutional Law 1299 [2001]) Consequently. Flipside. 6 December 2000 [Mendoza. and above all the speculative and amorphous nature of the required line-by-line analysis of detailed statutes. while statutes found to be vague as a matter of due process typically are invalidated [only] ‗as applied‘ to a particular defendant. the doctrines of strict scrutiny. Being one of the Senators who voted for its passage. 2d 524. 413 U. is rarely if ever an appropriate task for the judiciary. 226 U. Sullivan. 369 [1982]) In sum. First Amendment cases. 569.. In light of the foregoing disquisition. 223.‖ (G. Supreme Court pointed out in Younger v. Jackson Vinegar Co.. J. Hoffman Estates. Oklahoma. like overbreadth challenges typically produce facial invalidation. 107 L. v. With respect to such statute. therefore. Gunther & K. Inc. Separate Opinion]). 2d 669. at 613. ―A plaintiff who engages in some conduct that is clearly proscribed cannot complain of the vagueness of the law as applied to the conduct of others. ―on its face‖ invalidation of statutes has been described as ―manifestly strong medicine. But. A fortiori.S. whichever way they might be decided. 135385. 37.S.R. Inc. others omitted. The paradigmatic case is Yazoo & Mississippi Valley RR. Secretary of Environment and Natural Resources. 217. 29. Art.. overbreadth. 193 [1912]) As has been pointed out. 21. 493 U. v.S. it will take more than nitpicking to overturn the well-entrenched presumption of constitutionality and validity of the Plunder Law. it is evident that the purported ambiguity of the Plunder Law. the extradition proceeding is only at its evaluation stage. 98 . which in turn depends on the extent to which an individual will be "condemned to suffer grievous loss. There is no denial of due process as long as fundamental fairness is assured a party. instead of facing the consequences of their actions. Will Mark Jimenez‘s detention prior to the conclusion of the extradition proceedings not amount to a violation of his right to due process? Held: Contrary to his contention. and (3) his opportunity. Hon. would it be proper and just for the government to increase the risk of violating its treaty obligations in order to accord Respondent Jimenez his personal liberty in the span of time that it takes to resolve the Petition for Extradition? His supposed immediate deprivation of liberty without due process that he had previously shunned pales against the government‘s interest in fulfilling its Extradition Treaty obligations and in cooperating with the world community in the suppression of crime. In sum. the Court stresses that it is not ruling that the private respondent has no right to due process at all throughout the length and breadth of the extradition proceedings. we accord greater weight to the interests espoused by the government thru the petitioner Secretary of Justice. Oct. in the immediate deprivation of his liberty prior to his being heard. the due process rights accorded to individuals must be carefully balanced against exigent and palpable government interest. As aforesaid. instead of taking it. (Secretary of Justice v. when it is due. a subsequent opportunity to be heard is enough . and the degree of what is due. proceedings had already been conducted in that country. 17. yet. ―[c]onstitutional liberties do not exist in a vacuum. G. it was hindered from continuing with the due processes prescribed under its laws. No less compelling at that stage of the extradition proceedings is the need to be more deferential to the judgment of a co-equal branch of the government. (2) the extradition judge‘s independent prima facie determination that his arrest will best serve the ends of justice before the issuance of a warrant for his arrest. his detention prior to the conclusion of the extradition proceedings does not amount to a violation of his right to due process. He already had that opportunity in the requesting state. he ran away. 139465. it would not be good policy to increase the risk of violating our treaty obligations if. we find no arbitrariness. No. Stated otherwise. Hence. at the same time. persons sought to be extradited are able to evade arrest or escape from our custody. point out that the doctrine does not always call for a prior opportunity to be heard. 2000. the Executive.D. cowards and weaklings who. His invocation of due process now had thus become hollow. It is also worth noting that before the US government requested the extradition of respondent. to apply for bail as an exception to the no-initial-bail rule. either. P. Lantion. a prior determination should be made as to whether procedural protections are at all due and when they are due.R. we rule that the temporary hold on private respondent's privilege of notice and hearing is a soft restraint on his right to due process which will not deprive him of fundamental fairness should he decide to resist the request for his extradition to the United States. No. respondent will be given full opportunity to be heard subsequently. The extraditee's right to know is momentarily withheld during the evaluation stage of the extradition process to accommodate the more compelling interest of the State to prevent escape of potential extraditees which can be precipitated by premature information of the basis of the request for his extradition. once he is under the court‘s custody. That his arrest and detention will not be arbitrary is sufficiently ensured by (1) the DOJ‘s filing in court the Petition with its supporting documents after a determination that the extradition request meets the requirements of the law and the relevant treaty. choose to run and hide. In this light. Hence. Contrary to the contention of Jimenez.prejudice he will allegedly suffer is weak. we cannot allow our country to be a haven for fugitives. The time for the extraditee to know the basis of the request for his extradition is merely moved to the filing in court of the formal petition for extradition. En Banc [Puno]) 166. Needless to state.‖ Too. when the extradition court hears the Petition for Extradition. there is no violation of his right to due process and fundamental fairness. which has been endowed by our Constitution with greater power over matters involving our foreign relations. In the present case. We iterate the familiar doctrine that the essence of due process is the opportunity to be heard but. through overprotection or excessively liberal treatment. Procedural due process requires a determination of what process is due. Indeed. this balance of interests is not a static but a moving balance which can be adjusted as the extradition process moves from the administrative stage to the judicial stage and to the execution stage depending on factors that will come into play. 1069 which implements the RP-US Extradition Treaty affords an extraditee sufficient opportunity to meet the evidence against him once the petition is filed in court. Ralph C. Where the circumstances – such as those present in an extradition case – call for it. X x x In tilting the balance in favor of the interests of the State. But because he left the jurisdiction of the requesting state before those proceedings could be completed." We have explained why an extraditee has no right to notice and hearing during the evaluation stage of the extradition process. In the absence of any provision – in the Constitution, the law or the treaty – expressly guaranteeing the right to bail in extradition proceedings, adopting the practice of not granting them bail, as a general rule, would be a step towards deterring fugitives from coming to the Philippines to hide from or evade their prosecutors. The denial of bail as a matter of course in extradition cases falls into place with and gives life to Article 14 (It states: ―If the person sought consents in writing to surrender to the Requesting State, the Requested State may surrender the person as expeditiously as possible without further proceedings.‖) of the Treaty, since this practice would encourage the accused to voluntarily surrender to the requesting state to cut short their detention here. Likewise, their detention pending the resolution of extradition proceedings would fall into place with the emphasis of the Extradition Law on the summary nature of extradition cases and the need for their speedy disposition. (Government of the United States of America v. Hon. Guillermo Purganan, G.R. No. 148571, Sept. 24, 2002, En Banc [Panganiban]) The Equal Protection Clause 167. Explain and discuss the equal protection of the law clause. Held: 1. The equal protection of the law is embraced in the concept of due process, as every unfair discrimination offends the requirements of justice and fair play. It has nonetheless been embodied in a separate clause in Article III, Sec. 1, of the Constitution to provide for a more specific guaranty against any form of undue favoritism or hostility from the government. Arbitrariness in general may be challenged on the basis of the due process clause. But if the particular act assailed partakes of an unwarranted partiality or prejudice, the sharper weapon to cut it down is the equal protection clause. According to a long line of decisions, equal protection simply requires that all persons or things similarly situated should be treated alike, both as to rights conferred and responsibilities imposed . Similar subjects, in other words, should not be treated differently, so as to give undue favor to some and unjustly discriminate against others. The equal protection clause does not require the universal application of the laws on all persons or things without distinction. This might in fact sometimes result in unequal protection, as where, for example, a law prohibiting mature books to all persons, regardless of age, would benefit the morals of the youth but violate the liberty of adults. What the clause requires is equality among equals as determined according to a valid classification. By classification is meant the grouping of persons or things similar to each other in certain particulars and different from all others in these same particulars . (Philippine Judges Association v. Prado, 227 SCRA 703, 711-712, Nov. 11, 1993, En Banc [Cruz]) 2. The equal protection clause exists to prevent undue favor or privilege. It is intended to eliminate discrimination and oppression based on inequality. Recognizing the existence of real difference among men, the equal protection clause does not demand absolute equality. It merely requires that all persons shall be treated alike, under like circumstances and conditions both as to the privileges conferred and liabilities enforced. Thus, the equal protection clause does not absolutely forbid classifications x x x. If the classification is based on real and substantial differences; is germane to the purpose of the law; applies to all members of the same class; and applies to current as well as future conditions, the classification may not be impugned as violating the Constitution's equal protection guarantee. A distinction based on real and reasonable considerations related to a proper legislative purpose x x x is neither unreasonable, capricious nor unfounded. (Himagan v. People, 237 SCRA 538, Oct. 7, 1994, En Banc [Kapunan]) 168. Congress enacted R.A. No. 8189 which provides, in Section 44 thereof, that "No Election Officer shall hold office in a particular city or municipality for more than four (4) years. Any election officer who, either at the time of the approval of this Act or subsequent thereto, has served for at least four (4) years in a particular city or municipality shall automatically be reassigned by the Commission to a new station outside the original congressional district." Petitioners, who are City and Municipal Election Officers, theorize that Section 44 of RA 8189 is violative of the "equal protection clause" of the 1987 Constitution because it singles out the City and Municipal Election Officers of the COMELEC as prohibited from holding office in the same city or municipality for more than four (4) years. They maintain that there is no substantial distinction between them and other COMELEC officials, and therefore, there is no valid classification to justify the objective of the provision of law under attack. Resolve. 99 Held: The Court is not persuaded by petitioners' arguments. The "equal protection clause" of the 1987 Constitution permits a valid classification under the following conditions: 1) 2) 3) 4) The The The The classification classification classification classification must must must must rest on substantial distinction; be germane to the purpose of the law; not be limited to existing conditions only; and apply equally to all members of the same class. After a careful study, the ineluctable conclusion is that the classification under Section 44 of RA 8189 satisfies the aforestated requirements. The singling out of election officers in order to "ensure the impartiality of election officials by preventing them from developing familiarity with the people of their place of assignment" does not violate the equal protection clause of the Constitution. In Lutz v. Araneta, it was held that "the legislature is not required by the Constitution to adhere to a policy of 'all or none'". This is so for underinclusiveness is not an argument against a valid classification. It may be true that all other officers of COMELEC referred to by petitioners are exposed to the same evils sought to be addressed by the statute. However, in this case, it can be discerned that the legislature thought the noble purpose of the law would be sufficiently served by breaking an important link in the chain of corruption than by breaking up each and every link thereof. Verily, under Section 3(n) of RA 8189, election officers are the highest officials or authorized representatives of the COMELEC in a city or municipality. It is safe to say that without the complicity of such officials, large-scale anomalies in the registration of voters can hardly be carried out. (Agripino A. De Guzman, Jr., et al. v. COMELEC (G.R. No. 129118, July 19, 2000, en Banc [Purisima]) 169. Are there substantial distinctions between print media and broadcast media to justify the requirement for the latter to give free airtime to be used by the Comelec to inform the public of qualifications and program of government of candidates and political parties during the campaign period? Discuss. Held: There are important differences in the characteristics of the two media which justify their differential treatment for free speech purposes. Because of the physical limitations of the broadcast spectrum, the government must, of necessity, allocate broadcast frequencies to those wishing to use them. There is no similar justification for government allocation and regulation of the print media. In the allocation of limited resources, relevant conditions may validly be imposed on the grantees or licensees. The reason for this is that the government spends public funds for the allocation and regulation of the broadcast industry, which it does not do in the case of print media. To require radio and television broadcast industry to provide free airtime for the Comelec Time is a fair exchange for what the industry gets. From another point of view, the SC has also held that because of the unique and pervasive influence of the broadcast media, ―[n]ecessarily x x x the freedom of television and radio broadcasting is somewhat lesser in scope than the freedom accorded to newspaper and print media.‖ (TELEBAP, Inc. v. COMELEC, 289 SCRA 337, April 21, 1998 [Mendoza]) 170. Does the death penalty law (R.A. No. 7659) violate the equal protection clause considering that, in effect, it punishes only people who are poor, uneducated, and jobless? Held: R.A. No. 7659 specifically provides that ―[T]he death penalty shall be imposed if the crime of rape is committed x x x when the victim is a religious or a child below seven (7) years old.‖ Apparently, the death penalty law makes no distinction. It applies to all persons and to all classes of persons – rich or poor, educated or uneducated, religious or non-religious. No particular person or classes of persons are identified by the law against whom the death penalty shall be exclusively imposed. The law punishes with death a person who shall commit rape against a child below seven years of age. Thus, the perpetration of rape against a 5-year old girl does not absolve or exempt an accused from the imposition of the death penalty by the fact that he is poor, uneducated, jobless, and lacks catechetical instruction. To hold otherwise will not eliminate but promote inequalities. In Cecilleville Realty and Service Corporation v. CA, the SC clarified that compassion for the poor is an imperative of every humane society but only when the recipient is not a rascal claiming an undeserved privilege. (People v. Jimmy Mijano y Tamora, G.R. No. 129112, July 23, 1999, En Banc [Per Curiam]) 100 171. The International School Alliance of Educators (ISAE) questioned the point-of-hire classification employed by International School, Inc. to justify distinction in salary rates between foreign-hires and local-hires, i.e., salary rates of foreign-hires are higher by 25% than their local counterparts, as discriminatory and, therefore, violates the equal protection clause. The International School contended that this is necessary in order to entice foreign-hires to leave their domicile and work here. Resolve. Held: That public policy abhors inequality and discrimination is beyond contention. Constitution and laws reflect the policy against these evils. X x x Our International law, which springs from general principles of law , likewise proscribes discrimination x x x. The Universal Declaration of Human Rights, the International Covenant on Economic, Social and Cultural Rights, the International Convention on the Elimination of All Forms of Racial Discrimination, the Convention against Discrimination in Education, the Convention (No. 111) Concerning Discrimination in Respect of Employment and Occupation - all embody the general principle against discrimination, the very antithesis of fairness and justice. The Philippines, through its Constitution, has incorporated this principle as part of its national laws. [I]t would be an affront to both the spirit and letter of these provisions if the State, in spite of its primordial obligation to promote and ensure equal employment opportunities, closes its eyes to unequal and discriminatory terms and conditions of employment x x x. Discrimination, particularly in terms of wages, is frowned upon by the Labor Code. Article 135, for example, prohibits and penalizes the payment of lesser compensation to a female employee as against a male employee for work of equal value. Article 248 declares it an unfair labor practice for an employer to discriminate in regards to wages in order to encourage or discourage membership in any labor organization. X x x The foregoing provisions impregnably institutionalize in this jurisdiction the long honored legal truism of ―Equal pay for equal work.‖ Persons who work with substantially equal qualifications, skill, effort and responsibility, under similar conditions, should be paid similar salaries. This rule applies to the School (International School, Inc.), its "international character" notwithstanding. The School contends that petitioner has not adduced evidence that local-hires perform work equal to that of foreign-hires. The Court finds this argument a little cavalier. If an employer accords employees the same position and rank, the presumption is that these employees perform equal work. This presumption is borne by logic and human experience. If the employer pays one employee less than the rest, it is not for that employee to explain why he receives less or why the others receive more. That would be adding insult to injury. The employer has discriminated against that employee; it is for the employer to explain why the employee is treated unfairly. The employer in this case failed to discharge this burden. There is no evidence here that foreignhires perform 25% more efficiently or effectively than the local-hires. Both groups have similar functions and responsibilities, which they perform under similar working conditions. The School cannot invoke the need to entice foreign-hires to leave their domicile to rationalize the distinction in salary rates without violating the principle of equal work for equal pay. Xxx While we recognize the need of the School to attract foreign-hires, salaries should not be used as an enticement to the prejudice of local-hires. The local-hires perform the same services as foreign-hires and they ought to be paid the same salaries as the latter. For the same reason, the "dislocation factor" and the foreign-hires' limited tenure also cannot serve as valid bases for the distinction in salary rates. The dislocation factor and limited tenure affecting foreign-hires are adequately compensated by certain benefits accorded them which are not enjoyed by local-hires, such as housing, transportation, shipping costs, taxes and home leave travel allowances. The Constitution enjoins the State to ―protect the rights of workers and promote their welfare‖, ―to afford labor full protection.‖ The State, therefore, has the right and duty to regulate the relations between labor and capital. These relations are not merely contractual but are so impressed with public interest that labor contracts, collective bargaining agreements included, must yield to the common good . Should such contracts contain stipulations that are contrary to public policy, courts will not hesitate to strike down these stipulations. 101 appellant concludes that the prosecution discriminated against her on grounds of regional origins. Depending on the exigency of Government that has to be addressed. Premises considered. G. does not deserve the sympathy of this Court. invokes the equal protection clause in her defense. 128845. A strict scrutiny of classifications is essential lest wittingly or otherwise. 3. There is no reasonable distinction between the services rendered by foreign-hires and local-hires. A doctor with unique skills has the duty to save the lives of those with a particular affliction. therefore. The duty of a mother to nurse her infant is most compelling under the law of nature. The practice of the School of according higher salaries to foreign-hires contravenes public policy and. [Kapunan]) 172. 2000. Xxx It can be seen from the foregoing that incarceration. Lawful arrest and confinement are germane to the purposes of the law and apply to all those belonging to the same class. From this. by its nature. we find the point-of-hire classification employed by respondent School to justify the distinction in the salary rates of foreign-hires and local-hires to be an invalid classification. Prison officials have the difficult and often thankless job of preserving the security in a potentially explosive setting. 1st Div. Necessarily. No. who was charged with Illegal Recruitment in the RTC of Zamboanga City. We. The Court cannot validate badges of inequality. on the other hand. Xxx The performance of legitimate and even essential duties by public officers has never been an excuse to free a person validly in prison. The importance of a function depends on the need for its exercise. The necessities imposed by public welfare may justify exercise of government authority to regulate even if thereby certain groups may plausibly assert that their interests are disregarded. The accused-appellant asserts that the duty to legislate ranks highest in the hierarchy of government.R. remained scot-free. Held: The argument has no merit. and the alleged crime took place in Zamboanga City. The accused-appellant is only one of 250 members of the House of Representatives. the President or the Supreme Court can also be deemed the highest for that particular duty. Never had the call of a particular duty lifted a prisoner into a different classification from those others who are validly restrained by law. as well as of attempting to provide rehabilitation that prepare inmates for re-entry into the social mainstream. Appellant. we are constrained to rule against the accused-appellant‘s claim that reelection to public office gives priority to any other right or interest. Quisumbing. Leonardo A. En Banc [Ynares-Santiago]) 173. find that election to the position of Congressman is not a reasonable classification in criminal law enforcement. Jalosjos filed a motion before the Court asking that he be allowed to fully discharge the duties of a Congressman. Accused-appellant Romeo G. Congress continues to function well in the physical absence of one or a few of its members. Alejandro. 102 . A police officer must maintain peace and order. appellant was the only one criminally charged. Jalosjos. the issue before us boils down to a question of constitutional equal protection. 324 SCRA 689. including the police power of the State. June 1. insidious discriminations are made in favor of or against groups or types of individuals. both these demands require the curtailment and elimination of certain rights. She points out that although the evidence purportedly shows that Jasmine Alejandro handed out application forms and even received Lourdes Modesto‘s payment. An elective governor has to serve provincial constituents. (International School Alliance of Educators (ISAE) v. (People v. Does being an elective official result in a substantial distinction that allows different treatment? Is being a Congressman a substantial differentiation which removes the accused-appellant as a prisoner from the same class as all persons validly confined under law? Held: In the ultimate analysis. The functions and duties of the office are not substantial distinctions which lift him from the class of prisoners interrupted in their freedom and restricted in liberty of movement. 2000. The duties imposed by the ―mandate of the people‖ are multifarious. Feb. changes an individual‘s status in society. certainly. including attendance at legislative sessions and committee meetings despite his having been convicted in the first instance of a non-bailable offense. not to mention the 24 members of the Senate.In this case. Hon. charged with the duties of legislation. Appellant is a Cebuana while Alejandro is a Zamboanguena. 24. a denial of the equal protection of the laws. or it may only be shown by extrinsic evidence showing a discriminatory design over another not to be inferred from the action itself. Following established doctrine and procedure. must determine probable cause ―personally. The remedy for unequal enforcement of the law in such instances does not lie in the exoneration of the guilty at the expense of society x x x. was charged with the commission of a crime. was not. The unlawful administration by officers of a statute fair on its face. Held: It must be stressed that the 1987 Constitution requires the judge to determine probable cause ―personally. appellant has not presented any evidence to overcome this presumption. Montgomery.‖ Appellant has failed to show that. There is also common sense practicality in sustaining appellant‘s prosecution. to excuse a defendant guilty of murder because others have murdered with impunity. resulting in its unequal application to those who are entitled to be treated alike. It would be unconscionable. is not. 1st Div. the result would be that the trial of the district attorney for nonfeasance would become an issue in the trial of many persons charged with heinous crimes and the enforcement of law would suffer a complete breakdown (State v. and this presumption can be overcome only by proof to the contrary. on the basis thereof. while a Zamboanguena. in charging appellant in court. the judge is not required to personally examine the complainant and his witnesses. for instance. is not a denial of equal protection. unless there is shown to be present in it an element of intentional or purposeful discrimination. but no person has the right to demand protection of the law in the commission of a crime (People v.‖ a requirement which does not appear in the corresponding provisions of our previous constitutions. that there was a ―clear and intentional discrimination‖ on the part of the prosecuting officials. in issuing a warrant of arrest. an erroneous or mistaken performance of the statutory duty. he shall: (1) personally evaluate the report and the supporting documents submitted by the fiscal regarding the existence of probable cause and. issue a warrant of arrest. In satisfying himself of the existence of probable cause for the issuance of a warrant of arrest. Likewise. The mere allegation that appellant.2d 437 [1941]). The discretion of who to prosecute depends on the prosecution‘s sound assessment whether the evidence before it can justify a reasonable belief that a person has committed an offense . 325 P. Hicks. a Cebuana. Makasiar.‖ Distinguish determination of probable cause by the prosecutor and determination of probable cause by the judge. although a violation of the statute. 2001. The presumption is that the prosecuting officers regularly performed their duties. This emphasis evinces the intent of the framers to place a greater degree of responsibility upon trial judges than that imposed under previous Constitutions. Dela Piedra. In Soliven v. is not without more a denial of the equal protection of the laws. Discuss the constitutional requirement that a judge. While all persons accused of crime are to be treated on a basis of equality before the law. But a discriminatory purpose is not presumed. it does not follow that they are to be protected in the commission of crime. This may appear on the face of the action taken with respect to a particular class or person.2d 794 [1958]). the guilty party in appellant‘s eyes. not by mere speculation. there must be a showing of ―clear and intentional discrimination. The Right against Unreasonable Searches and Seizures 174. or (2) if in the basis thereof he finds no probable cause. however. (People v. Indeed. Jan. is insufficient to support a conclusion that the prosecution officers denied appellant equal protection of the laws. Where the official action purports to be in conformity to the statutory classification. this Court pronounced: ―What the Constitution underscores is the exclusive and personal responsibility of the issuing judge to satisfy himself of the existence of probable cause. 117 P. he may disregard the fiscal‘s report and require the submission of supporting affidavits of witnesses to aid him in arriving at a conclusion as to the existence of probable cause.‖ 103 . Protection of the law will be extended to all persons equally in the pursuit of their lawful occupations. 350 SCRA 163. [Kapunan]) [i]f the failure of prosecutors to enforce the criminal laws as to some persons should be converted into a defense for others charged with crime. by itself.The prosecution of one guilty while others equally guilty are not prosecuted. ) summarizes existing jurisprudence on the matter as follows: First. respondent admits that he issued the questioned warrant as there was ―no reason for (him) to doubt the validity of the certification made by the Assistant Prosecutor that a preliminary investigation was conducted and that probable cause was found to exist as against those charged in the information filed. it could have been very easy to describe the residential house of private respondent with 104 . [Gonzaga-Reyes]) 175. affidavits. Hence. sworn statements of witnesses or transcript of stenographic notes. he must have supporting evidence. Although the prosecutor enjoys the legal presumption of regularity in the performance of his official duties and functions. rather. determines whether a warrant of arrest should be issued against the accused. merely indicated the address of the compound which is 516 San Jose de la Montana St. Mabolo. the application was accompanied by a sketch of the compound at 516 San Jose de la Montana St. Thus. 18. factories and warehouse. In an application for search warrant. if any) upon which to make his independent judgment or. Lastly. which in turn gives his report the presumption of accuracy. as Respondent Court did in this case. since their objectives are different. at the very least. He adopted the judgment of the prosecutor regarding the existence of probable cause as his own. even if both should base their findings on one and the same proceeding or evidence. there should be no confusion as to their distinct objectives. Parenthetically. the Constitution. 326 SCRA 1. Mabolo. wherever and whenever it is feasible. Whether there is reasonable ground to believe that the accused is guilty of the offense charged and should be held for trial is what the prosecutor passes upon. the determination of probable cause by the prosecutor is for a purpose ―Lest we be too repetitive. it must be noted that the application for a search warrant was accompanied by a sketch of the compound at 516 San Jose de la Montana St. workshops. upon which to legally sustain his own findings on the existence (or nonexistence) of probable cause to issue an arrest order. but also so much of the records and the evidence on hand as to enable the His Honor to make his personal and separate judicial finding on whether to issue a warrant of arrest. Obviously and understandably. The sketch indicated the 2-storey residential house of private respondent with a large "X" enclosed in a square. The judge..‖ The statement is an admission that respondent relied solely and completely on the certification made by the fiscal that probable cause exists as against those charged in the information and issued the challenged warrant of arrest on the sole basis of the prosecutor‘s findings and recommendations. Cebu City. workshops. Within the same compound are residences of other people. Guiani. is that the judge must have sufficient supporting documents (such as the complaint. The point is: he cannot rely solely and entirely on the prosecutor‘s recommendation. offices. Within the same compound are residences of other people. Mabolo. Article III that the place to be searched must be particularly described? Held: This Court has held that the applicant should particularly describe the place to be searched and the person or things to be seized.‖ (Citations omitted) In the case at bench.. i.Ho v. it is not required that the complete or entire records of the case during the preliminary investigation be submitted to and examined by the judge. This responsibility of determining personally and independently the existence or nonexistence of probable cause is lodged in him by no less than the most basic law of the land. factories and warehouse. Second. offices. In the present case. We do not intend to unduly burden trial courts by obliging them to examine the complete records of every case all the time simply for the purpose of ordering the arrest of an accused. the judge must decide independently. whether there is a necessity for placing him under immediate custody in order not to frustrate the ends of justice. commands the judge to personally determine probable cause in the issuance of warrants of arrest. However. the contents of the prosecutor‘s report will support his own conclusion that there is reason to charge the accused for an offense and hold him for trial. (Abdula v. Did this satisfy the constitutional requirement under Section 2. 3rd Div. What is required. as held in Inting. upon which to verify the findings of the prosecutor as to the existence of probable cause.e. People (Ibid. Cebu City. we repeat. Feb. on the other hand. however. the judge cannot rely solely on the report of the prosecutor in finding probable cause to justify the issuance of a warrant of arrest. the prosecutor could ease the burden of the judge and speed up the litigation process by forwarding to the latter not only the information and his bare resolution finding probable cause. 2000. With this sketch as the guide.. we only wish to emphasize three vital matters once more: different from that which is to be made by the judge. indicating the 2-storey residential house of private respondent with a large ―X‖ enclosed in a square. The search warrant issued. counteraffidavits. This Court has consistently held that a judge fails in his bounden duty if he relies merely on the certification or the report of the investigating officer. Cebu City.. other than the prosecutor‘s bare report. as a means of impairing a sense of urgency and swiftness in the determination of whether a warrant of arrest should be issued. for issuing an arrest warrant was already evident from the Petition itself and its supporting documents. This description of the place to be searched is too general and does not pinpoint the specific house of private respondent. be amplified or modified by the officers‘ own personal knowledge of the premises. 1998 [Narvasa]) 177. the inadequacy of the description of the residence of private respondent sought to be searched has characterized the questioned search warrant as a general warrant. Arrest subsequent to a hearing can no longer be considered ―immediate. had the holding of a hearing at that stage been intended. after having already determined therefrom that a prima facie finding did exist. Court of Appeals. Evidently. and grant to officers executing a search warrant that discretion which the Constitution has precisely removed from them. which is violative of the constitutional requirement. Xxx We stress that the prima facie existence of probable cause for hearing the petition and. 105 .sufficient particularity so as to segregate it from the other buildings or structures inside the same compound. and giving them time to prepare and present such facts and arguments. a priori. Hearing entails sending notices to the opposing parties. 400. (People v. On the Basis of the Extradition Law It is significant to note that Section 6 of PD 1069.. our Extradition Law. It also bears emphasizing at this point that extradition proceedings are summary (See Sec. respondent judge gravely abused his discretion when he set the matter for hearing upon motion of Jimenez. even if it not be that delineated in the warrant. 9. (People v. receiving facts and arguments from them. The trial court is not expected to make an exhaustive determination to ferret out the true and actual situation. It is neither fair nor licit to allow police officers to search a place different from that stated in the warrant on the claim that the place actually searched – although not that specified in the warrant – is exactly what they had in view when they applied for the warrant and had demarcated in their supporting evidence. not what applicants had in their thoughts. This qualification would be rendered nugatory by setting for hearing the issuance of the arrest warrant. the court is expected merely to get a good first impression – a prima facie finding – sufficient to make a speedy initial determination as regards the arrest and detention of the accused. PD 1069) in nature. In connection with the matter of immediate arrest. 1. Hence. [Martinez]) 176.‖ the law further conveys that accuracy is not as important as speed at such early stage. What is material in determining the validity of a search is the place stated in the warrant itself. Is a respondent in an Extradition Proceeding entitled to notice and hearing before the issuance of a warrant of arrest? Held: Both parties cite Section 6 of PD 1069 in support of their arguments. Moreover. immediately upon the filing of the petition. and only in the warrant itself. The particularization of the description of the place to be searched may properly be done only by the Judge. the law could have easily so provided. Estrada. 291 SCRA 400. Cebu City. But the search warrant merely indicated the address of the compound which is 516 San Jose de la Montana St. It would open wide the door to abuse of the search process. From the knowledge and the material then available to it. the law specifies that the court sets a hearing upon receipt of the answer or upon failure of the accused to answer after receiving the summons. on the whole. however. It would concede to police officers the power of choosing the place to be searched. Thus. X x x Does this provision sanction RTC Judge Purganan‘s act of immediately setting for hearing the issuance of a warrant of arrest? We rule in the negative. it cannot be left to the discretion of the police officers conducting the search. as set out in the warrant. June 26.‖ The law could not have intended the word as a mere superfluity but. uses the word ―immediate‖ to qualify the arrest of the accused. Mabolo. or the evidence they adduce in support of their application for the warrant? Held: Such a change is proscribed by the Constitution which requires inter alia the search warrant to particularly describe the place to be searched as well as the persons or things to be seized. By using the phrase ―if it appears. Can the place to be searched. or had represented in the proofs they submitted to the court issuing the warrant. 296 SCRA 383. the word ―hearing‖ is notably absent from the provision. what would stop him from presenting his entire plethora of defenses at this stage – if he so desires – in his effort to negate a prima facie finding? Such a procedure could convert the determination of a prima facie case into a full-blown trial of the entire proceedings and possibly make trial of the main case superfluous. the Court categorically stated that a judge was not supposed to conduct a hearing before issuing a warrant of arrest x x x. In this instance. Held: While a contemporaneous search of a person arrested may be effected to discover dangerous weapons or proofs or implements used in the commission of the crime and which search may extend to the area within his immediate control where he might gain possession of a weapon or evidence he can destroy. (b) the discovery of the evidence in plain view is inadvertent. 148571.Hence. De Leon. never was a judge required to go to the extent of conducting a hearing just for the purpose of personally determining probable cause for the issuance of a warrant of arrest. En Banc [Panganiban]) 178. People and in all the cases cited therein. whether an arrest was merely used as a pretext for conducting a search. judges merely further examine complainants and their witnesses. Chua Ho San. or at the very least. 1999. 2. The object must be open to eye and hand and its discovery inadvertent.. What is ―search incidental to a lawful arrest‖? Discuss. 283 SCRA 159. 106 . No. There is no requirement to notify and hear the accused before the issuance of warrants of arrest. In the present case. Objects falling in plain view of an officer who has a right to be in the position to have that view are subject to seizure even without a search warrant and may be introduced in evidence .]) 179. the Constitution itself requires only the examination – under oath or affirmation – of complainants and the witnesses they may produce. C. (c) it is immediately apparent to the officer that the item he observes may be evidence of a crime. for the very purpose of both would have been defeated by the escape of the accused from the requested state. On the Basis of the Constitution Even Section 2 of Article III of our Constitution x x x does not require a notice or a hearing before the issuance of a warrant of arrest. Guillermo Purganan. The ―plain view‖ doctrine applies when the following requisites concur: (a) the law enforcement officer in search of the evidence has a prior justification for an intrusion or is in a position from which he can view a particular area. contraband or otherwise subject to seizure. June 17. Jr. This scenario is also anathema to the summary nature of extraditions. All we required was that the ―judge must have sufficient supporting documents upon which to make his independent judgment. a valid arrest must precede the search. Held: 1. the law requires that there be first a lawful arrest before a search can be made – the process cannot be reversed. upon which to verify the findings of the prosecutor as to the existence of probable cause. If the accused were allowed to be heard and necessarily to present evidence during the prima facie determination for the issuance of a warrant of arrest. Sept. 308 SCRA 432. he came inadvertently across a piece of evidence incriminating the accused . e..J. What is the ―plain view‖ doctrine? What are its requisites? Discuss. 24. The process cannot be reversed. In Ho v. The law enforcement officer must lawfully make an initial intrusion or properly be in a position from which he can particularly view the area . as the precedent arrest determines the validity of the incidental search. the legality of the arrest is questioned in a large majority of these cases. Court of Appeals. the silence of the Law and the Treaty leans to the more reasonable interpretation that there is no intention to punctuate with a hearing every little step in the entire proceedings. 175 [1997]) (People v. En Banc [Davide. X x x To determine probable cause for the issuance of arrest warrants. G.g.‖ In Webb v. Hon. (Government of the United States of America v. in cases of clear insufficiency of evidence on record. Xxx Verily x x x sending to persons sought to be extradited a notice of the request for their arrest and setting it for hearing at some future date would give them ample opportunity to prepare and execute an escape. At most. 2002.R. validating the act of respondent judge and instituting the practice of hearing the accused and his witnesses at this early stage would be discordant with the rationale for the entire system. In a search incidental to a lawful arrest. (Malacat v. In the course of such lawful intrusion. Neither the Treaty nor the Law could have intended that consequence. 1999. or is attempting to commit a crime. The members of the buy-bust team were justified in running after him and entering the house without a search warrant for they were hot in the heels of a fleeing criminal. What is a ―stop-and-frisk‖ search? 107 . its transparency. the following elements must be present: a) a prior valid intrusion based on the valid warrantless arrest in which the police are legally present in the pursuit of their official duties. To conclude otherwise would not only mean swimming against the stream. and other possessions. and c) the evidence must be immediately apparent. which includes his residence. recall that PO2 Balut testified that they first located the marijuana plants before appellant was arrested without a warrant. his papers. En Banc [Quisumbing]) 3. [Quisumbing]) 180. Jan. the police officers cornered Spencer and recovered the buy-bust money from him. In sum. 129296. Clearly. The difficulty arises when the object is inside a closed container. x x x. The guarantee refers to ―the right of personal security‖ of the individual. 2nd Div. b) the evidence was inadvertently discovered by the police who have the right to be where they are. (People v. coming as it is within the purview of Section 5(a) of Rule 113 of the 1985 Rules on Criminal Procedure x x x. what is sought to be protected against the State‘s unlawful intrusion are persons. he instinctively ran towards the house of appellant.R. They also caught appellant in flagrante delicto repacking the marijuana bricks which were in full view on top of a table. then the article is deemed in plain view. the object itself is not in plain view and therefore cannot be seized without a warrant. X x x. the seized marijuana plants were not ―immediately apparent‖ and ―further search‖ was needed. the police officers were not only authorized but also duty-bound to arrest him even without a warrant. 2000. or if its contents are obvious to an observer. contraband or otherwise subject to seizure. and d) plain view justified mere seizure of evidence without further search. they first had to ―look around the area‖ before they could spot the illegal plants. cannot be made to apply. (People v. Hence. In the instant case. (People v. then the contents are in plain view and may be seized . When Spencer wrenched himself free from the grasp of PO2 Gaviola. Doria. and (2) such overt act is done in the presence or within the view of the arresting officer. the marijuana plants in question were not in ―plain view‖ or ―open to eye and hand. it would also lead to the absurd logic that for a person to be immune against unreasonable searches and seizures. 25. appellant could not invoke the protection afforded by the Charter against unreasonable searches by agents of the State. there was no valid warrantless arrest which preceded the search of appellant‘s premises. The seizure of evidence in ―plain view‖ applies only where the police officer is not searching for evidence against the accused. if the package proclaims its contents. 329 SCRA 404. within a fenced yard or a private place. 22.]) 2. The right against unreasonable searches and seizures is the immunity of one‘s person. Note further that the police team was dispatched to appellant‘s kaingin precisely to search for and uproot the prohibited flora. this case falls squarely under the plain view doctrine. March 31. Where the object seized was inside a closed package. In other words. but inadvertently comes across an incriminating object .‖ The ―plain view‖ doctrine. when appellant was seen repacking the marijuana. The Bill of Rights belongs as much to the person in the street as to the individual in the sanctuary of his bedroom. he must be in his home or office. Section 5(a) is commonly referred to as the rule on in flagrante delicto arrests. if the package is such that an experienced observer could infer from its appearance that it contains the prohibited article. is actually committing. It must be immediately apparent to the police that the items that they observe may be evidence of a crime. appellant‘s subsequent arrest was likewise lawful. Once inside the house. J. However. Patently. 2000. No. 414-415. En Banc [Puno. G. Here two elements must concur: (1) the person to be arrested must execute an overt act indicating that he has just committed. Thus. Hence. Nor can we sustain the trial court‘s conclusion that just because the marijuana plants were found in an unfenced lot. Sept. not places . Abe Valdez. Elamparo. For the doctrine to apply. whether by its distinctive configuration. thus.It is clear that an object is in plain view if the object itself is plainly exposed to sight. 301 SCRA 668. We also note the testimony of SPO2 Tipay that upon arriving at the area. Considering its factual milieu. their discovery of the cannabis plants was not inadvertent. and Terry v. where in the course of investigating this behavior he identified himself as a policeman and make reasonable inquiries. 9. In fact. abandon the rule that the police must. the US Supreme Court held that what justified the limited search was the more immediate interest of the police officer in taking steps to assure himself that the person with whom he was dealing was not armed with a weapon that could unexpectedly and fatally be used against him. (Manalili v. Finally. p. whenever practicable. A Handbook on Arrest. 911) In allowing such a search.‖ A genuine reason must exist. Ohio. Ohio. It did not. a stop-and-frisk was defined as the vernacular designation of the right of a police officer to stop a citizen on the street.‖ it nevertheless holds that mere suspicion or a hunch will not validate a ―stop-and-frisk. p. Id. which underlies the recognition that a police officer may. for purposes of investigating possible criminal behavior even though there is insufficient probable cause to make an actual arrest. For..‖ but it cannot be denied that. In the landmark case of Terry v. Those which are warranted by the exigencies of public order and are conducted in a way least intrusive to motorists are allowed. Dec. the Court noted that the ‗sole justification‘ for a stop-and-frisk was the ‗protection of the police officer and others nearby‘.‖ (Herrera. he is entitled for the protection of himself and others in the area to conduct a carefully limited search of the outer clothing of such persons in an attempt to discover weapons which might be used to assault him. thus: We merely hold today that where a police officer observes unusual conduct which leads him reasonably to conclude in light of his experience that criminal activity may be afoot and that the persons with whom he is dealing may be armed and presently dangerous. the police officer did not place his hands in their pockets nor under the outer surface of their garments until he had felt weapons. Court of Appeals. a ―stop-and-frisk‖ serves a two-fold interest: (1) the general interest of effective crime prevention and detection. Such a search is a reasonable search under the Fourth Amendment (Terry. obtain advance judicial approval of searches and seizures through the warrant procedure. and then he merely reached for and removed the guns.) Other notable points of Terry are that while probable cause is not required to conduct a ―stopand-frisk. 1997 [Panganiban]) 2. and where nothing in the initial stages of the encounter serves to dispel his reasonable fear for his own or others‘ safety. supra.‖ as laid down in Terry. where in the course of investigating this behavior he identifies himself as a policeman and makes reasonable inquiries. interrogate him. 1995 ed. in appropriate circumstances and manner. Are searches at checkpoints valid? Discuss. This did not constitute a general exploratory search. 185. he is entitled for the protection of himself or others in the area to conduct a carefully limited search of the outer clothing of such persons in an attempt to discover weapons which might be used to assault him. (Malacat v. in light of the police officer‘s experience and surrounding conditions. approach a person for purposes of investigating possible criminal behavior even without probable cause. 283 SCRA 159. In admitting in evidence two guns seized during the stop-and-frisk. Search and Seizure and Custodial Investigation. while the scope of the search conducted in the case was limited to patting down the outer clothing of petitioner and his companions. 1997 [Davide]) 181. under appropriate circumstances and in an appropriate manner. CA. at 911. 12. to a certain extent.Held: 1. and where nothing in the initial stages of the encounter serves to dispel his reasonable fear for his own or others‘ safety. to warrant the belief that the person detained has weapons concealed about him . admittedly. We now proceed to the justification for and allowable scope of a ―stop-and-frisk‖ as a ―limited protective search of outer clothing for weapons. Oct. 280 SCRA 400. and any weapon seized may properly be introduced in evidence against the person from whom they were taken. as a rule. however. Such a search is a reasonable search under the Fourth Amendment. and pat him for weapon(s): ―x x x (W)here a police officer observes an unusual conduct which leads him reasonably to conclude in light of his experience that criminal activity may be afoot and that the persons with whom he is dealing may be armed and presently dangerous. it involves only a brief detention of travelers 108 . routine checkpoints do intrude. Held: This Court has ruled that not all checkpoints are illegal. the United States Supreme Court held that the interest of effective crime prevention and detection allows a police officer to approach a person. excused only by exigent circumstances. on motorists‘ right to ―free passage without interruption. and (2) the more pressing interest of safety and self-preservation which permit the police officer to take steps to assure himself that the person with whom he deals is not armed with a deadly weapon that could unexpectedly and fatally be used against the police officer. which expectation society is prepared to recognize as reasonable. Jan. signs. Leila Johnson. Usana. said routine checks cannot be regarded as violative of an individual‘s right against unreasonable search. Xxx The checkpoint herein conducted was in pursuance of the gun ban enforced by the COMELEC. Even so. There is little question that such searches are reasonable. are even less intrusive. 2 nd Div. They also complain of its having been conducted in an arbitrary and discriminatory manner. Those who intend to bring a gun during said period would know that they only need a car to be able to easily perpetrate their malicious designs. CJ]) 183. 2000. It would also defeat the purpose for which such ban was instituted. We take judicial notice of the existence of the COMELEC resolution imposing a gun ban during the election period issued pursuant to Section 52(c) in relation to Section 26(q) of the Omnibus Election Code (Batas Pambansa Blg. without opening the car‘s doors or subjecting its passengers to a body search. Is it still necessary that checkpoints be pre-announced? Held: Accused-appellants assail the manner by which the checkpoint in question was conducted. Jan. Passengers attempting to board an aircraft routinely pass through metal detectors. (People v. The incident. In fact. With increased concern over airplane hijacking and terrorism has come increased security at the nation‘s airports. 2000. given their minimal intrusiveness. At best they would merely direct their flashlights inside the cars they would stop. which happened on 5 April 1995. 18. physical searches are conducted to determine what the objects are. the constitutional protection against unreasonable searches and seizures refers to the immunity of one's person from interference by government and it cannot be extended to acts committed by private individuals so as to bring it within the ambit of alleged unlawful intrusion. Do the ordinary rights against unreasonable searches and seizures apply to searches conducted at the airport pursuant to routine airport security procedures? Held: Persons may lose the protection of the search and seizure clause by exposure of their persons or property to the public in a manner reflecting a lack of subjective expectation of privacy. The COMELEC would be hard put to implement the ban if its deputized agents were limited to a visual search of pedestrians. if any prohibited materials or substances are found. these routine checks. 1st Div. 881). Such recognition is implicit in airport security procedures. 28. 2000. (People v. 18. Illustrative case of valid search at checkpoint conducted to implement the gun ban law during election. May the constitutional protection against unreasonable searches and seizures be extended to acts committed by private individuals? Held: As held in People v. 138881. Dec. For as long as the vehicle is neither searched nor its occupants subjected to a body search. 301 SCRA 66. it would also forewarn those who intend to violate the ban. PO3 Suba admitted that they were merely stopping cars they deemed suspicious. [Melo]) 109 . Jan. 323 SCRA 754. was well within the election period. such would be subject to seizure. They contend that the checkpoint manned by elements of the Makati Police should have been announced. 28. Mendoza. and notices in their airline tickets that they are subject to search and. such as those whose windows are heavily tinted just to see if the passengers thereof were carrying guns. 1st Div. 1st Div. CJ]) 182. and the inspection of the vehicle is limited to a visual search. The national and local elections in 1995 were held on 8 May. (People v. 1999. These announcements place passengers on notice that ordinary constitutional protections against warrantless searches and seizures do not apply to routine airport procedures.R. There is nothing discriminatory in this as this is what the situation demands. Indeed. when conducted in a fixed area. the gravity of the safety interests involved. [Davide. Marti. G. 323 SCRA 754. Not only would it be impractical. (People v. badges of legitimacy of checkpoints may still be inferred from their fixed location and the regularized manner in which they are operated . Should these procedures suggest the presence of suspicious objects. travelers are often notified through airport public address systems. Usana. the second Monday of the month. No. and the reduced privacy expectations associated with airline travel. [Mendoza]) 184. their carry-on baggage as well as checked luggage are routinely subjected to x-ray scans. [Davide.during which the vehicle‘s occupants are required to answer a brief question or two. The facts adduced do not constitute a ground for a violation of the constitutional rights of the accused against illegal search and seizure. We see no need for checkpoints to be announced x x x. The policy of the law enunciated in R. as amended. 8203 is to protect the consumers as well as the licensed businessmen. adulterated. 296-298. to render effective and efficient the collection of import and export duties due the State.R No.‖ specify the proper fora and procedure for the ventilation of any legal objections or issues raised concerning these proceedings. Court of Appeals. 124461. not only to prevent smuggling and other frauds upon Customs. If the seized 52 boxes of drugs are pharmaceutically correct but not properly documented. [Mendoza]) 110 . The Regional Trial Courts are precluded from assuming cognizance over such matters even through petitions of certiorari. To the layman. they should be promptly disposed of in the manner provided by law in order to ensure that the same do not fall into the wrong hands who might use the drugs underground. Thus.185. but more importantly.‖ 186. 1125. otherwise known as ―An Act Creating the Court of Tax Appeals. Section 12) The BFAD is the government agency vested by law to make a mandatory and authoritative determination of the true therapeutic effect of drugs because it involves technical skill which is within its special competence. whose decision. or unregistered. reiterating its rulings x x x said: There is no question that Regional Trial Courts are devoid of any competence to pass upon the validity or regularity of seizure and forfeiture proceedings conducted by the Bureau of Customs and to enjoin or otherwise interfere with these proceedings. It is likewise well-settled that the provisions of the Tariff and Customs Code and that of Republic Act No. X x x. Section 11) cannot be neglected. Spcl. No. 1987 Constitution). The constitutional mandate that "the State shall adopt an integrated and comprehensive approach to health development which shall endeavor to make essential goods. 2 nd Div. prohibition or mandamus. (Bureau of Customs v. Ogario. it became mandatory for the government to supervise and control the proliferation of drugs in the market. The health of the citizenry should never be compromised. Section 19[1]). 2nd Div. 329 SCRA 289. This is why "the State shall establish and maintain an effective food and drug regulatory system. health and other social services available to all people at affordable cost" (Article XIII. this Court. We cannot afford to take any risk. Estrada (G. March 30. in turn. Instead. Only with the proper government sanctions can medicines and drugs circulate the market. June 26. Should the seized drugs (which are pharmaceutically correct but not properly documented) subject of an illegal search (because the applicant ―failed to allege in the application for search warrant that the subject drugs for which she was applying for search warrant were either fake. is subject to the exclusive appellate jurisdiction of the Court of Tax Appeals and from there to the Court of Appeals. in order to develop a healthy and alert citizenry (Article XIV. misbranded.A. The Collector of Customs sitting in seizure and forfeiture proceedings has exclusive jurisdiction to hear and determine all questions touching on the seizure and forfeiture of dutiable goods. [Ynares-Santiago]): ―With the State's obligation to protect and promote the right to health of the people and instill health consciousness among them (Article II. Foremost among these consumers is the government itself which procures medicines and distributes them to the local communities through direct assistance to the local health centers or through outreach and charity programs. Do Regional Trial Courts have competence to pass upon the validity or regularity of seizure and forfeiture proceedings conducted by the Bureau of Customs and to enjoin or otherwise interfere with these proceedings? Held: In Jao v. they should be promptly disposed of in the manner provided by law in order to insure that the same do not fall into the wrong hands who might use the drugs underground. Section 15. 2000. which enables the government to carry out the functions it has been instituted to perform. medicine is a cure that may lead to better health. As clarified in People v. for the life and health of the citizenry are as precious as the existence of the State. 2000.: No. x x x we have said that such act does not deprive the Bureau of Customs of jurisdiction thereon. The rule that Regional Trial Courts have no review powers over such proceedings is anchored upon the policy of placing no unnecessary hindrance on the government‘s drive. Judge Estrella T.‖) be returned to the owner? Ans. Even if the seizure by the Collector of Customs were illegal. actions of the Collector of Customs are appealable to the Commissioner of Customs." (Article XIII. The gun was plainly visible. in People v.. Mar. as a recent case law adverts to. 354 SCRA 338.‘‖ In declaring the warrantless arrest therein illegal. To all appearances. Held: In the case at bar. probable cause means an actual belief or reasonable grounds of suspicion. arrest a person when. is actually committing. in People v. coupled with good faith on the part of the peace officers making the arrest. Held: In the cases at bar. he may be lawfully searched for dangerous weapons or anything which may be used as proof of the commission of an offense. As discussed in People v. i. Doria. What he was doing was descending the gangplank of the M/V Wilcon 9 and there was no outward indication that called for his arrest. This is a valid search incidental to a lawful arrest. are not sufficient to constitute probable cause that would justify an in flagrante delicto arrest. in his presence. search and seizure in the present case fall within the recognized exceptions to the warrant requirement. The arresting office. Section 12. as overt acts and circumstances sufficient to arouse suspicion and indicative of probable cause. Then. or is attempting to commit an offense. the shabu and the drug paraphernalia. Accused-appellant assails the validity of his arrest and his subsequent convictions for the two crimes. Court of Appeals. 1st Div. at the moment of his arrest. Encinada.‖ Likewise. Mengote. ―[b]y no stretch of the imagination could it have been inferred from these acts that an offense had just been committed.‖ in a crowded street at 11:30 in the morning. as provided in Rule 126. he was in effect committing a crime in the presence of the police officers. it is settled that ―reliable information‖ alone. are valid because accused-appellants were caught in flagrante delicto in possession of prohibited drugs. As such. the Court did not consider ―eyes x x x darting from side to side x x x [while] holding x x x [one‘s] abdomen. cannot be said to have been made during an illegal search. Thus. in People v. the Court ruled that no probable cause is gleanable from the act of riding a motorela while holding two plastic baby chairs. the Court held that in cases of in flagrante delicto arrests. No warrant of arrest was necessary in such a situation. Chua Ho San. not being fruits of the poisonous tree x x x the objects found at the scene of the crime. the police saw the gun tucked in appellant‘s waist when he stood up. too. In People v. Hence. Accused-appellant could not show any license for the firearm. Besides. therefore. is based on actual facts. Both the trial court and the Court of Appeals found that the arrest and subsequent seizure were legal. 2001. As applied to in flagrante delicto arrests. Go. in Malacat v. A reasonable suspicion therefore must be founded on probable cause. it has been held that drugs discovered as a result of a consented search is admissible in evidence. [Ynares-Santiago]) 188. it being one of the recognized exceptions under the Rules. such as the firearm.‖ So also. without a warrant. though in a distant place from where the illegal possession of firearm was committed. Discuss the nature of an ―in flagrante delicto‖ warrantless arrest. or was actually being committed. committing a crime nor was it shown that he was about to do so or that he had just done so. As a consequence of appellant‘s valid warrantless arrest. No search was conducted as none was necessary. (Citations omitted. Thus. it was held that ―the accused-appellant was not. the suspicion that the person to be arrested is probably guilty of committing the offense. Aminnudin. It was only when the informer pointed to him as the carrier of the marijuana that he suddenly became suspect and so subject to apprehension. The subsequent discovery in his car of drug paraphernalia and the crystalline substance. without a search warrant. the person to be arrested has committed. or was at least being attempted in [the arresting officers‘] presence. was later identified as shabu. 14. This brings us to the issue of whether or not the warrantless arrest. absent any overt act indicative of a felonious enterprise in the presence and within the view of the arresting officers. supported by circumstances sufficiently strong in themselves to create the probable cause of guilt of the person to be arrested. he was like any of the other passengers innocently disembarking from the vessel. Illustrative case. the seized items do not fall within the exclusionary clause x x x.187. a peace officer or a private person may. the Court said: 111 . personal knowledge of facts or circumstances convincingly indicative or constitutive of probable cause. in the absence of actual belief of the arresting officers. which. the court a quo anchored its judgment of conviction on a finding that the warrantless arrest of accused-appellants.) (People v. whether at the time of his arrest or thereafter. The grounds of suspicion are reasonable when. According to the Court. and the subsequent search conducted by the peace officers. can be used as evidence against appellant. must have personal knowledge of such fact or.e. the trial court concluded that petitioner was attempting to commit a crime as he was ―‘standing at the corner of Plaza Miranda and Quezon Boulevard‘ with his eyes ‗moving very fast‘ and ‗looking at every person that come (sic) nearer (sic) to them. lending at least a semblance of validity on the arrest effected by the peace officers. therefore. if there was any. While SPO1 Paguidopon claimed that he and his informer conducted a surveillance of accusedappellant Mula. two requisites must concur: (1) the person to be arrested must execute an overt act indicating that he has just committed. thus presumably dusk. Third. it is doubtful if SPO1 Paguidopon indeed recognized accused-appellant Mula. Clearly. What is more. there was nothing in petitioner‘s behavior or conduct which could have reasonably elicited even mere suspicion other than that his eyes were ―moving very fast‖ – an observation which leaves us incredulous since Yu and his teammates were nowhere near petitioner and it was already 6:60 p. the arresting officer. to constitute a valid in flagrante delicto arrest. was not even aware of accused-appellants‘ name and address prior to the arrest.‖ (People v. Nevertheless. or is attempting to commit a crime. if possible we will settle this‖ to the request of SPO1 Pamplona to open the bag. could not have been certain of accused-appellants‘ identity. any telltale bulge. more so the arresting officers themselves. before the arrest. This belies the claim of SPO1 Pamplona that he knew the name of accused-appellants even before the arrest x x x. None was visible to Yu. The aforesaid testimony of SPO1 Pamplona. SPO1 Paguidopon. is entirely baseless. is actually committing. the arresting officer in the said case knew appellant Encinada even before the arrest because of the latter‘s illegal gambling activities. and were. It is worthy to note that. was being committed or was going to be committed. and from all indications as to the distance between Yu and petitioner. reasonable or otherwise. accused-appellants could not be said to be committing. is an equivocal statement which standing alone will not constitute probable cause to effect an in flagrante delicto arrest. on the part of petitioner. indicating that a crime had just been committed.m. SPO1 Paguidopon admitted that he had never seen him before the arrest. It matters not that accused-appellant Molina responded ―Boss. holding that ―[t]he prosecution‘s evidence did not show any suspicious behavior when the appellant disembarked from the ship or while he rode the motorela. These circumstances could not have afforded SPO1 Paguidopon a closer look at accused-appellant Mula. who allegedly conducted the surveillance. accusedappellants could not be subject of any suspicion. who acted as informer of the arresting officers. pinpointed to him by his informer while they were on the side of the road. attempting to commit or have committed a crime. In holding a bag on board a trisikad. considering that the latter was then driving a motorcycle when SPO1 Paguidopon caught a glimpse of him. to believe that petitioner was armed with a deadly weapon. Note that were it not for SPO1 Marino Paguidopon (who did not participate in the arrest but merely pointed accused-appellants to the arresting officers). assuming that petitioner was indeed hiding a grenade. SPO1 Pamplona could not have learned the name of accused-appellants from SPO1 Paguidopon because Paguidopon himself. SPO1 Paguidopon.) Moreover. No act or fact demonstrating a felonious enterprise could be ascribed to appellant under such bare circumstances. probable or otherwise. Petitioner and his companions were merely standing at the corner and were not creating any commotion or trouble x x x. admitted that he only learned Mula‘s name and address after the arrest. he was able to see Mula in person only once. accused-appellants manifested no outward indication that would justify their arrest.. Compared to People v. could not have been visible to Yu. Implied acquiescence to the search. from all indications. for as he admitted. it could not be said that accused-appellants waived their right against unreasonable searches and seizure. In the case at bar. Encinada. supra. It went on to state that – Second. Encinada. and (2) such overt act is done in the presence or within the view of the arresting officer. Such response which allegedly reinforced the ―suspicion‖ of the arresting officers that accused-appellants were committing a crime. or an overt physical act. thus. Evidently. could not have been more 112 . there was at all no ground. With respect to accused-appellant Molina.Here. the alleged grenade was ―discovered‖ ―inside the front waistline‖ of petitioner. merely fishing for evidence at the time of the arrest. however. the Court declared in said case that the warrantless arrest and the consequent search were illegal. there could have been no valid in flagrante delicto … arrest preceding the search in light of the lack of personal knowledge on the part of Yu. Among the exhibits offered by private respondent were three (3) cassette tapes of alleged telephone conversations between petitioner and unidentified persons. (People v. effect. or by using any other device or arrangement. Molina. Additionally. 190. x x x.than mere passive conformity given under intimidating or coercive circumstances and is thus considered no consent at all within the purview of the constitutional guarantee . herein petitioner. to knowingly possess any tape record. 4200)? Section 1 thereof Ans. or the existence. or to communicate the contents thereof. Section 4 thereof also provides: Sec. Any communication or spoken word. Absent a clear showing that both parties to the telephone conversations allowed the recording of the same. 19. En Banc [Ynares-Santiago]) The Right to Privacy of Communications and Correspondence 189. These tape recordings were made and obtained when private respondent allowed his friends from the military to wire tap his home telephone. It shall be unlawful for any person.A. or however otherwise described: It shall also be unlawful for any person. 4200 entitled ―An Act to Prohibit and Penalize Wire Tapping and Other Related Violations of the Privacy of Communication.A. Consequently. to secretly overhear. Private respondent Rafael S. or any other such record. 2001. quasi-judicial. either verbally or in writing. or record such communication or spoken word by using a device commonly known as a dictaphone or dictagraph or detectaphone or walkie-talkie or tape-recorder. What is the Anti-Wire Tapping Act (R. wire record. Teresita Salcedo-Ortanez. not being authorized by all the parties to any private communication or spoken word. Aug. Ortanez filed with the Regional Trial Court of Quezon City a complaint for annulment of marriage with damages against his wife. 4200. or meaning of the same or any part thereof. respondent trial court and Court of Appeals failed to consider the afore-quoted provisions of the law in admitting in evidence the cassette tapes in question. 4. legislative or administrative hearing or investigation. whether complete or partial. The trial court issued the assailed order admitting all of the evidence offered by private respondent. Hence. contents. or copies thereof. Did the trial court act properly when it admitted in evidence said tape recordings? Held: Republic Act No. Court of Appeals. to any other person: Provided. criminal investigation or trial of offenses mentioned in section 3 hereof. intercept. left with no choice but to find in favor of accused-appellants. 1. the inadmissibility of the subject tapes is mandatory under Rep. and For Other Purposes‖ expressly makes such tape recordings inadmissible in evidence. the search conducted on their person was likewise illegal. including tape recordings of telephone conversations of petitioner with unidentified persons. shall not be covered by this prohibition. That the use of such record or any copies thereof as evidence in any civil. the marijuana seized by the peace officers could not be admitted as evidence against accused-appellants. 4200 is a special law prohibiting and penalizing wire-tapping. to tap any wire or cable. or to furnish transcriptions thereof. Act No. Withal. Feb. 235 SCRA 111. on grounds of lack of marriage license and/or psychological incapacity of the petitioner. or any information therein contained obtained or secured by any person in violation of the preceding sections of this Act shall not be admissible in evidence in any judicial. provides: Sec. 352 SCRA 174. substance. Clearly. it should be mentioned that the above-mentioned Republic Act in Section 2 thereof imposes a penalty of imprisonment of not less than six (6) months and up to six (6) years for violation of said Act. purport. 4. be he a participant or not in the act or acts penalized in the next preceding sentence. of any communication or spoken word secured either before or after the effective date of this Act in the manner prohibited by this law. disc record. (Salcedo-Ortanez v. the Court holds that the arrest of accused-appellants does not fall under the exceptions allowed by the rules. 1994 [Padilla]) 113 . or to replay the same for any other person or persons. and the Court is thus.: R. Fifth and Ninth Amendments x x x.‖ 114 . (Ople v. Court of Appeals. the Secrecy of Bank Deposits (R. intercepting. which prohibits wire tapping.S. Torres. Aug. Held: We prescind from the premise that the right to privacy is a fundamental right guaranteed by the Constitution.R.R. there was a heated argument between police officer Navarro and Lingan. Connecticut (381 U. No. 6. No. 2. 14 L. No. 1998 [Puno]) 193. What are the zones of privacy recognized and protected in our laws? Held: The Civil Code provides that ―[e]very person shall respect the dignity. Torres. (Morfe v.A.R. 18 [1970]). the revelation of trade and industrial secrets. G. It is debatable whether these interests are compelling enough to warrant the issuance of A. No. 479. Is the voice recording admissible in evidence in view of R. Rule 130[c]. hence.O.A. 1. No. if not totally eradicate. The answer is in the affirmative. (Felipe Navarro v. or recording of private communications 192. Other facets of the right to privacy are protected in various provisions of the Bill of Rights (viz: Secs.191. and recognizes the privacy of letters and other private communications. Lingan fell and his head hit the pavement which caused his death. Discuss why Administrative Order No. the vagueness.O.O. 121087. Since the exchange between petitioner Navarro and Lingan was not private. 308 lies in its Section 4 which provides for a Population Reference Number (PRN) as a ―common reference number to establish a linkage among concerned agencies‖ through the use of ―Biometrics Technology‖ and ―computer application designs. Revised Rules on Evidence). July 23. 4200. and 17. 308 which if implemented will put our people‘s right to privacy in clear and present danger.A. No. 8293). Fourth. 308. Two local media men in Lucena City went to the police station to report alleged indecent show in one night establishment in the City. the United States Supreme Court gave more substance to the right of privacy when it ruled that the right has a constitutional foundation. 4200. Presented in evidence to confirm his testimony was a voice recording he had made of the heated discussion at the police station between accused police officer Navarro and the deceased. 308 is justified by some compelling state interest and that it is narrowly drawn. if we extend our judicial gaze we will find that the right of privacy is recognized and enshrined in several provisions of our Constitution. privacy and peace of mind of his neighbors and other persons‖ and punishes as actionable torts several acts by a person of meddling and prying into the privacy of another. 444 [1968]. fraudulent transactions and misrepresentations by persons seeking basic services. [Mendoza]) The Right to Privacy [T]he law prohibits the overhearing.A. During the trial. 4200).O. x x x. (Ramirez v. testified. Lingan. Jalbuena. and trespass to dwelling. The Rules of Court on privileged communication likewise recognize the privacy of certain information (Section 24. 444-445). Is there a constitutional right to privacy? Held: The essence of privacy is the ―right to be let alone. G. the other media man. one of the two media men. Cortes. It held that there is a right of privacy which can be found within the penumbras of the First. A.‖ In the 1965 case of Griswold v. we adopted the Griswold ruling that there is a constitutional right to privacy x x x. 127685. which was taken without the knowledge of the two. 308 is predicated on two considerations: (1) the need to provide our citizens and foreigners with the facility to conveniently transact business with basic service and social security providers and other government instrumentalities and (2) the need to reduce. 22 SCRA 424. It is expressly recognized in Section 3(1) of the Bill of Rights x x x. But what is not arguable is the broadness. No. the overbreadth of A. Court of Appeals. The heart of A. Mutuc. 2nd Div. It also holds a public officer or employee or any private individual liable for damages for any violation of the rights and liberties of another person. 1999. 1405) and the Intellectual Property Code (R. its tape recording is not prohibited. In the 1968 case of Morfe v. 8. which led to fisticuffs. Invasion of privacy is an offense in special laws like the Anti-Wiretapping Law (R. G. No.O. At the station. 2D 510 [1965]). which prohibits wire-tapping? Held: [J]albuena‘s testimony is confirmed by the voice recording he had made. 127685. It may be asked whether the tape is admissible in view of R. personality.A. The Revised Penal Code makes a crime the violation of secrets by an officer. Third. July 23. The Constitutional Foundations of Privacy. (Ople v. 248 SCRA 590 [1995]). 308 (issued by the President prescribing for a National ID system for all citizens to facilitate business transactions with government agencies engaged in the delivery of basic services and social security provisions) should be declared unconstitutional. p. Indeed. Mutuc (22 SCRA 424. 26. 1998 [Puno]) 194. No. it is the burden of government to show that A. ed. They must satisfactorily show the presence of compelling state interest and that the law. It will not do for the authorities to invoke the presumption of regularity in the performance of official duties. In the case at bar. The ability of a sophisticated data center to generate a comprehensive cradle-to-grave dossier on an individual and transmit it over a national network is one of the most graphic threats of the computer revolution. No. No. x x x.O.O. This approach is demanded by the 1987 Constitution whose entire matrix is designed to protect human rights and to prevent authoritarianism. It is plain and we hold that A. 308 cannot be underplayed x x x. much less verify the correctness of the data encoded. if not defeated. even when the government does not act irrationally. 308 should also raise our antennas for a further look will show that it does not state whether encoding of data is limited to biological information alone for identification purposes. this Court will give the challenged law. 1998 [Puno]) 195. 308. No. No. the least we can do is to lean towards the stance that will not put in danger the rights protected by the Constitution. G. it may also enable unscrupulous persons to access confidential information and circumvent the right against self-incrimination.O. Considering the banquet of options available to the implementors of A. July 23. Once extracted. No.It is noteworthy that A. it can be extracted together with other data on the subject. [T]he Court will not be true to its role as the ultimate guardian of the people‘s liberty if it would not immediately smother the sparks that endanger their rights but would rather wait for the fire that could consume them. No. 308 gives the government the power to compile a devastating dossier against unsuspecting citizens. under what circumstances should it be allowed? Held: The issue is whether petitioner may be cited for indirect contempt for her failure to produce the documents requested by the Ombudsman. employers.O. The threats emanate from various sources – governments. In case of doubt. 127685.O. administrative order.R.O. Should in camera inspection of bank accounts be allowed? If in the affirmative. No. 308 does not state what specific biological characteristics and what particular biometrics technology shall be used to identify people who will seek its coverage. The computer is capable of producing a comprehensive dossier on individuals out of information given at different times and for varied purposes. etc. And whether the order of the Ombudsman to 115 . The data may be gathered for gainful and useful government purposes. the indefiniteness of A. x x x [W]e close with the statement that the right to privacy was not engraved in our Constitution for flattery. the fear that it threatens the right to privacy of our people is not groundless. 308 may interfere with the individual‘s liberty of abode and travel by enabling authorities to track down his movement. [A]nd we now hold that when the integrity of a fundamental right is at stake. rule. No.O. a temptation that may be too great for some of our authorities to resist. it may pave the way for ―fishing expeditions‖ by government authorities and evade the right against unreasonable searches and seizures. Given the record-keeping power of the computer. 308 pressures the people to surrender their privacy by giving information about themselves on the pretext that it will facilitate delivery of basic services. The end of privacy begins. Clearly. Nor is it enough for the authorities to prove that their act is not irrational for a basic right can be diminished. The lack of proper safeguards in this regard of A. The possibilities of abuse and misuse of the PRN. or regulation is narrowly drawn to preclude abuses. 308 falls short of assuring that personal information which will be gathered about our people will only be processed for unequivocally specified purposes. journalists. Torres. x x x. 308 can give the government the roving authority to store and retrieve information for a purpose other than the identification of the individual through his PRN. (Ople v. The potential for misuse of the data to be gathered under A. The more frequent the use of the PRN. social scientists. the better the chance of building a huge and formidable information base through the electronic linkage of the files. They threaten the very abuses that the Bill of Rights seeks to prevent.O. biometrics and computer technology are accentuated when we consider that the individual lacks control over what can be read or placed on his ID. No. Retrieval of stored data is simple.O. the information is putty in the hands of any person. only the indifferent will fail to perceive the danger that A. When information of a privileged character finds its way into the computer. the threat comes from the executive branch of government which by issuing A. rule or regulation a stricter scrutiny. but the existence of this vast reservoir of personal information constitutes a covert invitation to misuse. No. The right to privacy is one of the most threatened rights of man living in a mass society. A. These restrictions are censorial and therefore they bear a heavy presumption of constitutional invalidity. In Union Bank of the Philippines v. 6646. No. subject to the clear-and-present danger test. In short. 3) Upon written permission of the depositor. G. what the Office of the Ombudsman would wish to do is to fish for additional evidence to formally charge Amado Lagdameo. Marquez v. al. Impeachment cases. Content-neutral restrictions. the inspection limited to the subject matter of the pending case before the court of competent jurisdiction. No. they will be tested for possible overbreadth and vagueness. 508 [1965]). Aniano A. Deposit is subject of litigation. 4) In cases of impeachment. No. The order of the Ombudsman to produce for in camera inspection the subject accounts with the Union Bank of the Philippines.R.A. therefore. Held: Content-based restrictions are imposed because of the content of the speech and are. For example.A. commentators. Hon. a rule such as that involved in Sanidad v. there is yet no pending litigation before any court of competent authority. Clearly. are not concerned with the content of the speech. An examination of the secrecy of bank deposits law (R. R. 3019. 6646. We rule that before an in camera inspection may be allowed. 2) In an examination made by an independent auditor hired by the bank to conduct its regular audit provided that the examination is for audit purposes only and the results thereof shall be for the exclusive use of the bank. 135882. 5) Sec. or 6) In cases where the money deposited or invested is the subject matter of the litigation‖. or it will not pass muster under strict scrutiny.‖ and give example of each. 1405) would reveal the following exceptions: 1) 2) 3) 4) Where the depositor consents in writing. 3019. and announcers from campaigning either for or against an issue in a plebiscite must have compelling reason to support it. the account must be clearly identified. Court of Appeals .A. No. No. there must be a pending case before a court of competent jurisdiction. which prohibits the sale or donation of print space and air time to political candidates during the campaign period. 8. In the case at bar. Desierto. Gancayco (122 Phil. is based on a pending investigation at the Office of the Ombudsman against Amado Lagdameo. Further. Comelec.A. A deferential standard of review will suffice to test their validity. et. like Sec. 503. on the other hand. and such inspection may cover only the account identified in the pending case. as amended.have an in camera inspection of the questioned account is allowed as an exception to the law on secrecy of bank deposits (R. prohibiting columnists. En Banc [Pardo]) Freedom of Expression 196. 2001. The clearand-present danger rule is inappropriate as a test for determining the constitutional validity of laws. in cases of unexplained wealth as held in the case of PNB v. By court order in bribery or dereliction of duty cases against public officials. Sec. 11(b) of R. June 27. with the Sandiganbayan. 3 (e) and (g) relative to the Joint Venture Agreement between the Public Estates Authority and AMARI. Distinguish ―content-based restrictions‖ on free speech from ―content-neutral restrictions. al. et. which are not concerned with the content of political ads but only with their 116 . we held that ―Section 2 of the Law on Secrecy of Bank Deposits. like Sec. 11(b) of R. (Lourdes T. The bank personnel and the account holder must be notified to be present during the inspection. 5) Upon order of a competent court in cases of bribery or dereliction of duty of public officials. for violation of R. No.A.. What is existing is an investigation by the Office of the Ombudsman. These regulations need only a substantial governmental interest to support them.A. there was no pending case in court which would warrant the opening of the bank account for inspection. Julia Vargas Branch. No. 1405). In addition. declares bank deposits to be ‗absolutely confidential‘ except: 1) In an examination made in the course of a special or general examination of a bank that is specifically authorized by the Monetary Board after being satisfied that there is reasonable ground to believe that a bank fraud or serious irregularity has been or is being committed and that it is necessary to look into the deposit to establish such fraud or irregularity. incidents. To apply the clear-and-present danger test to such regulatory measures would be like using a sledgehammer to drive a nail when a regular hammer is all that is needed. The test for this difference in the level of justification for the restriction of speech is that contentbased restrictions distort public debate, have improper motivation, and are usually imposed because of fear of how people will react to a particular speech. No such reasons underlie content-neutral regulations, like regulation of time, place and manner of holding public assemblies under B.P. Blg. 880, the Public Assembly Act of 1985. (Osmena v. COMELEC, 288 SCRA 447, March 31, 1998 [Mendoza]) 197. What is the most influential test for distinguishing content-based from content-neutral regulations? Held: The United States Supreme Court x x x held in United States v. O‘ Brien: [A] a governmental regulation is sufficiently justified (1) if it is within the constitutional power of the government; (2) if it furthers an important or substantial governmental interest; (3) if the governmental interest is unrelated to the suppression of free expression; and (4) if the incidental restriction on alleged First Amendment freedoms (of speech, expression and press) is no greater than is essential to the furtherance of that intesrst (391 U.S. 367, 20 L. Ed. 2df 692, 680 [1968] [bracketed numbers added]) This is so far the most influential test for distinguishing content-based from content-neutral regulations and is said to have ―become canonical in the review of such laws.‖ (G. Gunther & K. Sullivan, Constitutional Law 1217 [13th ed. 1997]). It is noteworthy that the O‘ Brien test has been applied by this Court in at least two cases (Adiong v. Comelec, 207 SCRA 712 [1992]; Osmena v. Comelec, supra.). Under this test, even if a law furthers an important or substantial governmental interest, it should be invalidated if such governmental interest is ―not unrelated to the suppression of free expression.‖ Moreover, even if the purpose is unrelated to the suppression of free speech, the law should nevertheless be invalidated if the restriction on freedom of expression is greater than is necessary to achieve the governmental purpose in question. (Social Weather Stations, Inc. v. Comelec, G.R. No. 147571, May 5, 2001, En Banc [Mendoza]) 198. Does the conduct of exit poll by ABS CBN present a clear and present danger of destroying the credibility and integrity of the electoral process as it has the tendency to sow confusion considering the randomness of selecting interviewees, which further makes the exit poll highly unreliable, to justify the promulgation of a Comelec resolution prohibiting the same? Held: Such arguments are purely speculative and clearly untenable. First, by the very nature of a survey, the interviewees or participants are selected at random, so that the results will as much as possible be representative or reflective of the general sentiment or view of the community or group polled. Second, the survey result is not meant to replace or be at par with the official Comelec count. It consists merely of the opinion of the polling group as to who the electorate in general has probably voted for, based on the limited data gathered from polled individuals. Finally, not at stake are the credibility and the integrity of the elections, which are exercises that are separate and independent from the exit polls. The holding and the reporting of the results of exit polls cannot undermine those of the elections, since the former is only part of the latter. If at all, the outcome of one can only be indicative of the other. The COMELEC‘s concern with the possible noncommunicative effect of exit polls – disorder and confusion in the voting centers – does not justify a total ban on them. Undoubtedly, the assailed Comelec Resolution is too broad, since its application is without qualification as to whether the polling is disruptive or not. There is no showing, however, that exit polls or the means to interview voters cause chaos in voting centers. Neither has any evidence been presented proving that the presence of exit poll reporters near an election precinct tends to create disorder or confuse the voters. Moreover, the prohibition incidentally prevents the collection of exit poll data and their use for any purpose. The valuable information and ideas that could be derived from them, based on the voters‘ answers to the survey questions will forever remain unknown and unexplored. Unless the ban is restrained, candidates, researchers, social scientists and the electorate in general would be deprived of studies on the impact of current events and of election-day and other factors on voters‘ choices. The absolute ban imposed by the Comelec cannot, therefore, be justified. It does not leave open any alternative channel of communication to gather the type of information obtained through exit polling. 117 On the other hand, there are other valid and reasonable ways and means to achieve the Comelec end of avoiding or minimizing disorder and confusion that may be brought about by exit surveys. With foregoing premises, it is concluded that the interest of the state in reducing disruption is outweighed by the drastic abridgment of the constitutionally guaranteed rights of the media and the electorate. Quite the contrary, instead of disrupting elections, exit polls – properly conducted and publicized – can be vital tools for the holding of honest, orderly, peaceful and credible elections; and for the elimination of election-fixing, fraud and other electoral ills. (ABS-CBN Broadcasting Corporation v. COMELEC, G.R. No. 133486, Jan. 28, 2000, En Banc [Panganiban]) 199. Section 5.4 of R.A. No. 9006 (Fair Election Act) which provides: ―Surveys affecting national candidates shall not be published fifteen (15) days before an election and surveys affecting local candidates shall not be published seven (7) days before an election.‖ The Social Weather Stations, Inc. (SWS), a private non-stock, non-profit social research institution conducting surveys in various fields; and Kamahalan Publishing Corporation, publisher of the Manila Standard, a newspaper of general circulation, which features newsworthy items of information including election surveys, challenged the constitutionality of aforesaid provision as it constitutes a prior restraint on the exercise of freedom of speech without any clear and present danger to justify such restraint. Should the challenge be sustained? Held: For reason hereunder given, we hold that Section 5.4 of R.A. No. 9006 constitutes an unconstitutional abridgment of freedom of speech, expression, and the press. To be sure, Section 5.4 lays a prior restraint on freedom of speech, expression, and the press by prohibiting the publication of election survey results affecting candidates within the prescribed periods of fifteen (15) days immediately preceding a national election and seven (7) days before a local election. Because of the preferred status of the constitutional rights of speech, expression, and the press, such a measure is vitiated by a weighty presumption of invalidity. Indeed, ―any system of prior restraints of expression comes to this Court bearing a heavy presumption against its constitutional validity x x x. The Government ‗thus carries a heavy burden of showing justification for the enforcement of such restraint.‘‖ There is thus a reversal of the normal presumption of validity that inheres in every legislation. Nor may it be argued that because of Art. IX-C, Sec. 4 of the Constitution, which gives the Comelec supervisory power to regulate the enjoyment or utilization of franchise for the operation of media of communication, no presumption of invalidity attaches to a measure like Sec. 5.4. For as we have pointed out in sustaining the ban on media political advertisements, the grant of power to the Comelec under Art. IX-C, Sec. 4 is limited to ensuring ―equal opportunity, time, space, and the right to reply‖ as well as uniform and reasonable rates of charges for the use of such media facilities for ―public information campaigns and forums among candidates.‖ Xxx Nor can the ban on election surveys be justified on the ground that there are other countries x x x which similarly impose restrictions on the publication of election surveys. At best this survey is inconclusive. It is noteworthy that in the United States no restriction on the publication of election survey results exists. It cannot be argued that this is because the United States is a mature democracy. Neither are there laws imposing an embargo on survey results, even for a limited period, in other countries. x x x. What test should then be employed to determine the constitutional validity of Section 5.4? The United States Supreme Court x x x held in United States v. O‘ Brien: [A] government regulation is sufficiently justified (1) if it is within the constitutional power of the government; (2) if it furthers an important or substantial governmental interest; (3) if the governmental interest is unrelated to the suppression of free expression; and (4) if the incidental restriction on alleged First Amendment freedoms (of speech, expression and press) is no greater than is essential to the furtherance of that interest (391 U.S. 367, 20 L. Ed. 2d 692, 680 [1968] [bracketed numbers added]). This is so far the most influential test for distinguishing content-based from content-neutral regulations and is said to have ―become canonical in the review of such laws.‖ It is noteworthy that the O‘ Brien test has been applied by this Court in at least two cases (Adiong v. Comelec; Osmena v. Comelec). Under this test, even if a law furthers an important or substantial governmental interest, it should be invalidated if such governmental interest is ―not unrelated to the suppression of free expression.‖ 118 Moreover, even if the purpose is unrelated to the suppression of free speech, the law should nevertheless be invalidated if the restriction on freedom of expression is greater than is necessary to achieve the governmental purpose in question. Our inquiry should accordingly focus on these two considerations as applied to Sec. 5.4. First. Sec. 5.4 fails to meet criterion (3) of the O‘ Brien test because the causal connection of expression to the asserted governmental interest makes such interest ―not unrelated to the suppression of free expression.‖ By prohibiting the publication of election survey results because of the possibility that such publication might undermine the integrity of the election, Sec. 5.4 actually suppresses a whole class of expression, while allowing the expression of opinion concerning the same subject matter by newspaper columnists, radio and TV commentators, armchair theorists, and other opinion makers. In effect, Sec. 5.4 shows a bias for a particular subject matter, if not viewpoint, by preferring personal opinion to statistical results. The constitutional guarantee of freedom of expression means that ―the government has no power to restrict expression because of its message, its ideas, its subject matter, or its contents.‖ The inhibition of speech should be upheld only if the expression falls within one of the few unprotected categories dealt with in Chaplinsky v. New Hampshire, thus: There are certain well-defined and narrowly limited classes of speech, the prevention and punishment of which have never been thought to raise any Constitutional problem. These include the lewd and obscene, the profane, the libelous, and the insulting or ‗fighting‘ words – those which by their very utterance inflict injury or tend to incite an immediate breach of the peace. [S]uch utterances are no essential part of any exposition of ideas, and are of such slight social value as a step to truth that any benefit that may be derived from them is clearly outweighed by the social interest in order and morality. v. Minnesota, it was held: Nor is there justification for the prior restraint which Sec. 5.4 lays on protected speech. In Near [T]he protection even as to previous restraint is not absolutely unlimited. But the limitation has been recognized only in exceptional cases x x x. No one would question but that a government might prevent actual obstruction to its recruiting service or the publication of the sailing dates of transports or the number and location of troops. On similar grounds, the primary requirements of decency may be enforced against obscene publications. The security of the community life may be protected against incitements to acts of violence and the overthrow by force of orderly government x x x. Thus, x x x the prohibition imposed by Sec. 5.4 cannot be justified on the ground that it is only for a limited period and is only incidental. The prohibition may be for a limited time, but the curtailment of the right of expression is direct, absolute, and substantial. It constitutes a total suppression of a category of speech and is not made less so because it is only for a period of fifteen (15) days immediately before a national election and seven (7) days immediately before a local election. This sufficiently distinguishes Sec. 5.4 from R.A. No. 6646, Sec. 11(b), which this Court found to be valid in National Press Club v. Comelec, and Osmena v. Comelec. For the ban imposed by R.A. No. 6646, Sec. 11(b) is not only authorized by a specific constitutional provision (Art. IX-C, Sec. 4), but it also provided an alternative so that, as this Court pointed out in Osmena, there was actually no ban but only a substitution of media advertisements by the Comelec space, and Comelec hour. Second. Even if the governmental interest sought to be promoted is unrelated to the suppression of speech and the resulting restriction of free expression is only incidental, Sec. 5.4 nonetheless fails to meet criterion (4) of the O‘ Brien test, namely, that the restriction be not greater than is necessary to further the governmental interest. As already stated, Sec. 5.4. aims at the prevention of last-minute pressure on voters, the creation of bandwagon effect, ―junking‖ of weak or ―losing‖ candidates, and resort to the form of election cheating called ―dagdag-bawas.‖ Praiseworthy as these aims of the regulation might be, they cannot be attained at the sacrifice of the fundamental right of expression, when such aim can be more narrowly pursued by punishing unlawful acts, rather than speech because of apprehension that such speech creates the danger of such evils. Thus, under the Administrative Code of 1987 (Bk. V, Tit. I, Subtit. C, Ch 1, Sec. 3[1]), the Comelec is given the power: misleading or false election propaganda, after due notice and hearing. To stop any illegal activity, or confiscate, tear down, and stop any unlawful, libelous, This is surely a less restrictive means than the prohibition contained in Sec. 5.4. Pursuant to this power of the Comelec, it can confiscate bogus survey results calculated to mislead voters. Candidates can have their own surveys conducted. No right of reply can be invoked by others. No principle of 119 The question for determination in this case is the liability for libel of a citizen who denounces a barangay official for misconduct in office. The doctrine of fair comment means that while in general every discreditable imputation publicly made is deemed false. petitioner and the other residents of the Tondo Foreshore Area were not only acting in their self-interest but engaging in the performance of a civic duty to see to it that public duty is discharged faithfully and well by those on whom such duty is incumbent. (Borjal v. v. the prosecution failed to prove not only that the charges made by petitioner were false but also that petitioner made them with knowledge of their falsity or with reckless disregard of whether they were false or not. discussions of our most abiding concerns as a nation would be stifled. above all. For that matter. but be insufficient to justify such as diminishes the exercise of rights so vital to the maintenance of democratic institutions. as long as it might reasonably be inferred from the facts. unless the public official concerned proves that the statement was made with actual malice – that is. In denouncing the barangay chairman in this case.‖ Some are susceptible to the herd mentality.‖ To summarize then. (2) it is a direct and total suppression of a category of expression even though such suppression is only for a limited period. based on established facts.equality is involved. ―public discussion is a political duty‖ and the ―greatest menace to freedom is an inert people. 147571. 2001. In order that such discreditable imputation to a public official may be actionable. COMELEC. As for the purpose of the law to prevent bandwagon effects. Sullivan (376 US 254) holding that honest criticisms on the conduct of public officials and public figures are insulated from libel judgments? Held: The guarantees of freedom of speech and press prohibit a public official or public figure from recovering damages for a defamatory falsehood relating to his official conduct unless he proves that 120 . 15. which this Court has cited with approval in several of its own decisions. It would. It is a free market to which each candidate brings his ideas. The recognition of this right and duty of every citizen in a democracy is inconsistent with any requirement placing on him the burden of proving that he acted with good motives and for justifiable ends. En Banc [Mendoza]) 200..‖ Held: The decision appealed from should be reversed. 5. nevertheless. Court of Appeals.‖ (Whitney v. because every man is presumed innocent until his guilt is judicially proved. What is the ―raison d‘etre‖ for the New York Times v. Without free speech and assembly. Sept. infringe on the constitutionally guaranteed freedom of expression. Some voters want to be identified with the ―winners. If the comment is an expression of opinion.R. when the discreditable imputation is directed against a public person in his public capacity. En Banc [Mendoza]) 201. California) (Vasquez v. even if the defamatory statement is false. Jan. (Social Weather Stations. it is doubtful whether the Government can deal with this naturalenough tendency of some voters.4. with knowledge that it was false or with reckless disregard of whether it was false or not. May 5. 2nd Div. Such a rule would deter citizens from performing their duties as members of a self-governing community. and (3) the governmental interest sought to be promoted can be achieved by means other than the suppression of freedom of expression. no liability can attach if it relates to official conduct. The Regional Trial Court of Manila x x x found petitioner guilty x x x on the ground that petitioner failed to prove the truth of the charges and that he was ―motivated by vengeance in uttering the defamatory statement. and every false imputation is deemed malicious. Held: Fair commentaries on matters of public interest are privileged and constitute a valid defense in an action for libel or slander. 1999. 1999. then it is immaterial that the opinion happens to be mistaken. Inc. it must either be a false allegation of fact or a comment based on a false supposition. it is not necessarily actionable. [Bellosillo]) 202. Sullivan. 301 SCRA 1. is invalid because (1) it imposes a prior restraint on the freedom of expression. 361 of the Revised Penal Code. we hold that Sec. A rule placing on the accused the burden of showing the truth of allegations of official misconduct and/or good motives and justifiable ends for making such allegations would not only be contrary to Art.‖ In this case. This is the gist of the ruling in the landmark case of New York Times v. G. As Justice Brandies has said. 314 SCRA 460. Can these be legitimately prohibited by suppressing the publication of survey results which are a form of expression? It has been held that ―[mere] legislative preferences or beliefs respecting matters of public convenience may well support regulation directed at other personal activities. CA. Discuss the "doctrine of fair comment" as a valid defense in an action for libel or slander. No. 14. This is the rule of ―actual malice. Ltd. 301 SCRA 1. qualifiedly privileged communications containing defamatory imputations are not actionable unless found to have been made without good intention or justifiable motive. in short. Absolutely privileged communications are those which are not actionable even if the author acted in bad faith. The raison d‘etre for the New York Times doctrine was that to require critics of official conduct to guarantee the truth of all their factual assertions on pain of libel judgments would lead to self-censorship. 14. Held: [W]e deem private respondent a public figure within the purview of the New York Times ruling. a pugilist. 14. the SC reversed applying the doctrine of fair comment. war heroes and even ordinary soldiers. Jan. (Borjal v. would not qualify as a public figure. and no less a personage than the Great Exalted Ruler of the lodge. however. [Bellosillo]) 203. 2nd Div. a professional baseball player. not the participant‘s prior anonymity or notoriety.. has become a ‗public personage. 14. since would-be critics would be deterred from voicing out their criticisms even if such were believed to be true. to be included in this category are those who have achieved some degree of reputation by appearing before the public. in other words.‖ and therefore subject to public comment? Held: At any rate. The list is. An example is found in Sec. CA. v. Is the enumeration of qualifiedly privileged communications under Article 354 of the Revised Penal Code exclusive? 121 . 301 SCRA 1. fame. Columnist Art Borjal of The Philippine Star wrote in his column that petitioner (private respondent). VI. his affairs and his character. private respondent consequently assumed the status of a public figure. Upon the other hand. But even assuming ex-gratia argumenti that private respondent. X x x The FNCLT (First National Conference on Land Transportation) was an undertaking infused with public interest. we have also defined ―public figure‖ in Ayers Production Pty. the Executive Director and Spokesman of the FNCLT (First National Conference on Land Transportation). CA. anyone who has arrived at a position where the public attention is focused upon him as a person. 1999. for he could be. 2nd Div.the statement was made with actual malice. famous inventors and explorers. 11. it does not necessarily follow that he could not validly be the subject of a public comment even if he was not a public official or at least a public figure. If a matter is a subject of public or general interest. 301 SCRA 1. It was promoted as a joint project of the government and the private sector. 2nd Div. What are the types of privileged communications? Held: A privileged communication may be either absolutely privileged or qualifiedly privileged. or by adopting a profession or calling which gives the public a legitimate interest in his doings. it attracted media mileage and drew public attention not only to the conference itself but to the personalities behind as well. despite the position he occupied in the FNCLT. As its Executive Director and spokesman. 1999. (Borjal v. by his accomplishments.‖ (Borjal v. infant prodigy. and organized by top government officials and prominent businessmen. 1999. The public‘s primary interest is in the event. Jan. 2nd Div. Jan. the public focus is on the conduct of the participant and the content. mode of living. effect and significance of the conduct.‘ He is. It includes. [Bellosillo]) 206. Capulong as – X x x a person who. because of doubt whether it could be proved or because of fear of the expense of having to prove it. Obviously. as long as he was involved in a public issue. (Borjal v. a joint project of the government and the private sector to raise funds to improve the state of land transportation in the country. He was sued for damages arising from libel by petitioner (private respondent) and subsequently found liable by the trial court and made to pay damages. Who is a ―public figure. engaged in shady and anomalous deals. [Bellosillo]) 205. For this reason. CA. i. or were in fact true. 14. of the 1987 Constitution which exempts a member of Congress from liability for any speech or debate in the Congress or in any Committee thereof. CA. Jan. as in the case of an actor. a celebrity. 1999. It includes public officers. To this genre belong ―private communications‖ and ―fair and true report without any comments or remarks. Art. with knowledge that it was false or with reckless disregard of whether it was false or not. broader than this. it cannot suddenly become less so merely because a private individual is involved or because in some sense the individual did not voluntarily choose to become involved. 301 SCRA 1. [Bellosillo]) 204. or any other entertainer. On appeal..e. 354 of The Revised Penal Code for x x x they are neither private communications nor fair and true report without any comments or remarks. the Court reiterates that judicial independence and the fair and orderly administration of justice constitute paramount governmental interests that can justify the regulation of the public's right of free speech and peaceful assembly in the vicinity of courthouses. There is no antinomy between free expression and the integrity of the system of administering justice. entitled. Conventional wisdom tells us that the realities of life in a complex society preclude an absolutist interpretation of freedom of expression where it does not involve pure speech but speech plus physical actions like picketing. Rallies and Other Similar Gatherings in the Vicinity of the Supreme Court and All Other Courts. Did it originate in the nation‘s penal laws. our rulings now musty in years hold that only the narrowest time. CA. on the contrary. place and manner regulations that are specifically tailored to serve an important governmental interest may justify the application of the balancing of interests test in derogation of the people's right of free speech and expression. 2nd Div. they must all be weighed with the promotion of the general welfare of the people as the ultimate objective. Jurado. (Borjal v. Jan. Pickets. 354 is not an exclusive list of qualifiedly privileged communications since fair commentaries on matters of public interest are likewise privileged. One of these fundamental public interests is the maintenance of the integrity and orderly functioning of the administration of justice. In balancing these values. [Bellosillo]) 208. or where upon their face they are so vague. Discuss the origin of the rule on privileged communication. Hence. this does not necessarily mean that they are not privileged. this Court has accorded freedom of expression a preferred position in light of its more comparative importance. place. 253. There are other significant societal values that must be accommodated and when they clash. For the protection and maintenance of freedom of expression itself can be secured only within the context of a functioning and orderly system of dispensing justice. Jan. the enumeration under Art. is essential to free government . and time of public discussion can be constitutionally controlled. As early as 1918.Held: Indisputably. indefinite. within 122 . However. CA. in United States v. or in the Bill of Rights guaranteeing freedom of speech and of the press? Held: The rule on privileged communications had its genesis not in the nation‘s penal code but in the Bill of Rights of the Constitution guaranteeing freedom of speech and of the press. 14. or inexact as to permit punishment of the fair use of the right of free speech. No. [Bellosillo]) 207. Canete (38 Phil. 14. Prescinding from this premise. free expression. This constitutional right cannot be abolished by the mere failure of the legislature to give it express recognition in the statute punishing libel. however. 265 [1918]) . It has never been understood as the absolute right to speak whenever. sweep within their ambit other activities as to operate as an overhanging threat to free discussion. 98-7-02-SC. They submit that the Supreme Court gravely abused its discretion and/or acted without or in excess of jurisdiction in promulgating those guidelines. 301 SCRA 1. this Court ruled that publications which are privileged for reasons of public policy are protected by the constitutional guaranty of freedom of speech. and wherever one pleases. petitioner Borjal‘s questioned writings are not within the exceptions of Art.1998 in A. The Office of the Mayor of Las Pinas refused to issue permit to petitioners to hold rally a rally in front of the Justice Hall of Las Pinas on the ground that it was prohibited under Supreme Court En Banc Resolution dated July 7.M. 1999. for the manner. "Re: Guidelines on the Conduct of Demonstrations. Where said regulations do not aim particularly at the evils within the allowable areas of state control but. But freedom of speech and expression despite its indispensability has its limitations. Article III of the 1987 Constitution that ―no law shall be passed abridging‖ them. peaceful assembly and petitioning government for redress of grievances citing Sec. In the case of In Re: Emil P. To be sure. 301 SCRA 1. It is true that the safeguarding of the people's freedom of expression to the end that individuals may speak as they think on matters vital to them and that falsehoods may be exposed through the processes of education and discussion. [T]he better policy is not liberty untamed but liberty regulated by law where every freedom is exercised in accordance with law and with due regard for the rights of others. the Court pronounced in no uncertain terms that: ―x x x freedom of expression needs on occasion to be adjusted to and accommodated with the requirements of equally important public interests. 1999." Petitioners thus initiated the instant proceedings. (Borjal v. Held: We shall first dwell on the critical argument made by petitioners that the rules constitute an abridgment of the people's aggregate rights of free speech. 2nd Div. 4. such regulations are void. non-partisan proceedings before a cold and neutral judge. No. and isolate it from public hysteria. in other words.R. 1998) 209. the resolution of this Court regulating demonstrations adds specific restrictions as they involve judicial independence and the orderly administration of justice. has been ruled as valid and constitutional notwithstanding its limiting effect on the exercise by the public of their liberties. (In Re: Petition to Annul En Banc Resolution A. 880. such as streets.P. of viable independent institutions for delivery of justice which are accepted by the general community.M. however. Sept. It is the traditional conviction of every civilized society that courts must be insulated from every extraneous influence in their decisions . as public fora.M. B. B. even. Blg. They charge that this Court amended provisions of Batas Pambansa (B.‖ by converting the sidewalks and streets within a radius of two hundred (200) meters from every courthouse from a public forum place into a ―no rally‖ zone. entitled.P. Rallies and Other Similar Gatherings in the Vicinity of the Supreme Court and All Other Courts?‖ Held: Petitioners also claim that this Court committed an act of judicial legislation in promulgating the assailed resolution. among other places. Jurado. Contrary therefore to petitioners‘ impression. and leave open ample alternative channels of communication. On the other hand. are narrowly tailored to serve a significant governmental interest. and manner regulations. spoil the ideal of sober. (In Re: Petition to Annul En Banc Resolution A. the courthouses. Valmonte and Union of Lawyers and Advocates for Transparency in Government [ULAT]. 29. Blg. there will be grievances against our justice system for there can be no perfect system of justice but these grievances must be ventilated through appropriate petitions. B.P. place and manner of conducting concerted actions. Pickets. Blg. To be sure. ―Re: Guidelines on the Conduct of Demonstrations. place. Such a mode is in keeping with the respect due to the courts as vessels of justice and is necessary if judges are to dispose their business in a fair fashion. does not preclude this Court from promulgating rules regulating conduct of demonstrations in the vicinity of courts to assure our people of an impartial and orderly administration of justice as mandated by the Constitution.) Blg. but also by irascible demonstrations outside. 323-324 [1995]) It is sadly observed that judicial independence and the orderly administration of justice have been threatened not only by contemptuous acts inside. motions or other pleadings. They wittingly or unwittingly. The facts of a case should be determined upon evidence produced in court. 1998) 210. 29. G. without more. to limit the volume of loud speakers or sound system and to prescribe other appropriate restrictions on the conduct of the public assembly. it requires a written permit for the holding of public assemblies in public places subject. 98-7-02-SC . Valmonte and Union of Lawyers and Advocates for Transparency in Government [ULAT].Ricardo C. prejudice or sympathies. and manner regulations as long as the restrictions are content-neutral. a prohibition against picketing and demonstrating in or near courthouses. 98-7-02-SC. x x x‖ (In Re: Emil P. they accuse this Court of x x x violating the principle of separation of powers. it is settled jurisprudence that the government may restrict speech plus activities and enforce reasonable time. to impose a rerouting of the parade or street march.the context. sidewalks. G. 134621. it is not any law that can imbue such places with the public nature inherent in them. 243 SCRA 299. 880 and the assailed resolution complement each other. to be public fora. and parks. otherwise known as ―the Public Assembly Act. are considered. Simply put. 134621.P. Should live media coverage of court proceedings be allowed? 123 . this Court merely moved away the situs of mass actions within a 200-meter radius from every courthouse. Public places historically associated with the free exercise of expressive activities. In other words. friendly or otherwise. We so hold following the rule in legal hermeneutics that an apparent conflict between a court rule and a statutory provision should be harmonized and both should be given effect if possible . 880 did not establish streets and sidewalks. 880 imposes general restrictions to the time. But even in such public fora. There is thus no discrepancy between the two sets of regulatory measures. A close look at the law will reveal that it in fact prescribes reasonable time. To insulate the judiciary from mob pressure. Even in the United States. Thus. place. Sept.Ricardo C. No. In fine. 98-7-02-SC . and should be uninfluenced by bias.M. Blg. Did the Supreme Court commit an act of judicial legislation in promulgating En Banc Resolution A. Thus. We reject these low watts arguments. The existence of B.P. to the right of the mayor to modify the place and time of the public assembly. X x x The administration of justice must not only be fair but must also appear to be fair and it is the duty of this Court to eliminate everything that will diminish if not destroy this judicial desideratum.R. 880. ―The television camera is a powerful weapon which intentionally or inadvertently can destroy an accused and his case in the eyes of the public. Should the Court allow live media coverage of the anticipated trial of the plunder and other criminal cases filed against former President Joseph E. Also. since within the courtroom a reporter's constitutional rights are no greater than those of any other member of the public. Video footages of court hearings for news purposes shall be restricted and limited to shots of the courtroom. telecasting is a form of mental harassment and subjects him to excessive public exposure and distracts him from the effective presentation of his defense. 124 . jurisprudence tells us that the right of the accused must be preferred to win. Collaterally. and considering further that the freedom of the press and the right of the people to information may be served and satisfied by less distracting. the trial judge and the defendant. witnesses. (Supreme Court En Banc Resolution Re: Live TV and Radio Coverage of the Hearing of President Corazon C. it also raises issues on the nature of the media. and of the impact of new technologies on law. live radio and television coverage of court proceedings shall not be allowed. particularly television and its role in society. since a television or news reporter has the same privilege. Massive intrusion of representatives of the news media into the trial itself can so alter or destroy the constitutionally necessary judicial atmosphere and decorum that the requirements of impartiality imposed by due process of law are denied the defendant and a defendant in a criminal proceeding should not be forced to run a gauntlet of reporters and photographers each time he enters or leaves the courtroom . the Court through Mr. Considering the prejudice it poses to the defendant's right to due process as well as to the fair and orderly administration of justice. identified four (4) areas of potential prejudice which might arise from the impact of the cameras on the jury. degrading and prejudicial means. For the defendant. and on the other hand. Witnesses might be frightened. Estrada before the Sandiganbayan in order ―to assure the public of full transparency in the proceedings of an unprecedented case in our history‖ as requested by the Kapisanan ng mga Brodkaster ng Pilipinas? Held: The propriety of granting or denying the instant petition involve the weighing out of the constitutional guarantees of freedom of the press and the right to public information. Aquino's Libel Case. the right of the public to information and the right to public trial. the judicial officers. or become nervous. In Estes v. Texas. play to the camera. When these rights race against one another.‖ Representatives of the press have no special standing to apply for a writ of mandate to compel a court to permit them to attend a trial. and the fundamental rights of the accused. on the other hand.Held: The propriety of granting or denying permission to the media to broadcast. The decision in part pertinently stated: ―Experience likewise has established the prejudicial effect of telecasting on witnesses. dated Oct. Justice Clark. Judges are human beings also and are subject to the same psychological reactions as laymen. No video shots or photographs shall be permitted during the trial proper. as the news reporter is not permitted to bring his typewriter or printing press into the courtroom. 1991) 211. the due process rights of the defendant and the inherent and constitutional power of the courts to control their proceedings in order to permit the fair and impartial administration of justice. Xxx Courts do not discriminate against radio and television media by forbidding the broadcasting or televising of a trial while permitting the newspaper reporter access to the courtroom. record. along with the constitutional power of a court to control its proceedings in ensuring a fair and impartial trial. on the one hand. or photograph court proceedings involves weighing the constitutional guarantees of freedom of the press. the parties and their counsel taken prior to the commencement of official proceedings. 22. on the one hand. it may as well affect his own performance. Voting 5-4. They are subject to extraordinary out-of-court influences which might affect their testimony. telecasting not only increases the trial judge's responsibility to avoid actual prejudice to the defendant. the United States Supreme Court held that television coverage of judicial proceedings involves an inherent denial of due process rights of a criminal defendant. it does so in so many ways and in varying degrees. It also approves of media‘s exalted power to provide the most accurate and comprehensive means of conveying the proceedings to the public and in acquainting the public with the judicial process in action. at stake in the criminal trial is not only the life and liberty of the accused but the very credibility of the Philippine criminal justice system. courts do not express the popular will of the people in any sense which. its exercise must necessarily be subject to the maintenance of absolute fairness in the judicial process. The effect of television may escape the ordinary means of proof. it is not at all unlikely for a vote of guilt or innocence to yield to it . nonetheless. unbridled by running emotions or passions. conduct themselves with decorum and observe the trial process. in proceedings that are devoid of histrionics that might detract from its basic aim to ferret veritable facts free from improper influence. indeed. A public trial is not synonymous with publicized trial. The conscious or unconscious effect that such a coverage may have on the testimony of witnesses and the decision of judges cannot be evaluated but. it only implies that the court doors must be open to those who wish to come. Estrada. not too small as to render the openness negligible and not too large as to distract the trial participants from their proper functions. (Re: Request Radio-TV coverage of the Trial in the Sandiganbayan of 125 . able to thrive in hardy climate. it is a fact. and live television and radio coverage of the trial will not subserve the ends of justice but will only pander to the desire for publicity of a few grandstanding lawyers. It might be farcical to build around them an impregnable armor against the influence of the most powerful media of public opinion. The courts recognize the constitutionally embodied freedom of the press and the right to public information. such that the verdict of the court will be acceptable only if popular. An accused has a right to a public trial but it is a right that belongs to him.With the possibility of losing not only the precious liberty but also the very life of an accused. whether open or subtle. A trial is not a free trade of ideas. and live television and radio coverage of the trial could allow the ―hooting throng‖ to arrogate unto themselves the task of judging the guilt of the accused. where fitting dignity and calm ambiance is demanded. sit in the available seats.‖ Xxx The Integrated Bar of the Philippines x x x expressed its own concern on the live television and radio coverage of the criminal trials of Mr. instead. or pressure that media can bring to bear on them directly and through the shaping of public opinion. a verdict that would come only after the presentation of credible evidence testified to by unbiased witnesses unswayed by any kind of pressure. nevertheless. a courtroom should have enough facilities for a reasonable number of the public to observe the proceedings. to paraphrase: Live television and radio coverage can negate the rule on exclusion of witnesses during the hearings intended to assure a fair trial. To say that actual prejudice should first be present would leave to near nirvana the subtle threats to justice that a disturbance of the mind so indispensable to the calm and deliberate dispensation of justice can create. that. the overriding consideration is still the paramount right of the accused to due process which must never be allowed to suffer diminution in its constitutional proportions. but it is not farfetched for it to gradually erode our basal conception of a trial such as we know it now . who shall then be totally free to report what they have observed during the proceedings . with every reason to presume firmness of mind and resolute endurance. A public trial aims to ensure that he is fairly dealt with and would not be unjustly condemned and that his rights are not compromised in secret conclaves of long ago. Justice Clark thusly pronounced. within the courthouse. it behooves all to make absolutely certain that an accused receives a verdict solely on the basis of a just and dispassionate judgment. Nor is a competing market of thoughts the known test of truth in a courtroom. but it must also be conceded that ―television can work profound changes in the behavior of the people it focuses on. it can likewise be said. Due process guarantees the accused a presumption of innocence until the contrary is proved in a trial that is not lifted above its individual settings nor made an object of public‘s attention and where the conclusions reached are induced not by any outside force or influence but only by evidence and argument given in open court. where his life or liberty can be held critically in balance. Witnesses and judges may very well be men and women of fortitude. are tasked to only adjudicate controversies on the basis of what alone is submitted before them. In the constitutional sense. and decreed by a judge with an unprejudiced mind.‖ Even while it may be difficult to quantify the influence. Xxx Unlike other government offices. ―while a maximum freedom must be allowed the press in carrying out the important function of informing the public in a democratic society. more than anyone else. freedom to believe and freedom to act. They quietly stand at attention during the flag ceremony to show their respect for the rights of those who choose to participate in the solemn proceedings. Conduct remains subject to regulation for the protection of society. CA. at some slight inconvenience. Xxx Petitioners stress x x x that while they do not take part in the compulsory flag ceremony. is alien to the conscience of the present generation of Filipinos who cut their teeth on the Bill of Rights which guarantees their rights to free speech (The flag salute. they do not engage in ―external acts‖ or behavior that would offend their countrymen who believe in expressing their love of country through the observance of the flag ceremony. 2001. Freedom of conscience and freedom to adhere to such religious organization or form of worship as the individual may choose cannot be restricted by law. even the exercise of religion may be regulated. The idea that one may be compelled to salute the flag. The right to religious profession and worship has a two-fold aspect. that after the Gerona ruling had received legislative cachet by its incorporation in the Administrative Code of 1987. Sept. No. freedom to believe and freedom to act on one's belief. and recite the patriotic pledge.) and the free exercise of religious profession and worship. In every case. 236 SCRA 197. Since they do not engage in disruptive behavior. therefore. in the nature of things. 1. On the other hand. Discuss why the Gerona ruling (justifying the expulsion from public schools of children of Jehovah‘s Witnesses who refuse to salute the flag and sing the national anthem during flag ceremony as prescribed by the Flag Salute Law) should be abandoned. there is no warrant for their expulsion. The constitutional inhibition of legislation on the subject of religion has a double aspect. The first is absolute but. The second is subject to regulation where the belief is translated into external acts that affect the public welfare. (Iglesia Ni Cristo v. that is. 1994 [Regalado]) 213. Estrada. 1996 [Puno]) 2. public morals. Whence. 135 SCRA 514. singing the national anthem and reciting the patriotic pledge are all forms of utterances. The first is absolute as long as the belief is confined within the realm of thought. Held: Our task here is extremely difficult. for it involves the relationship of man to his Creator .M. in attaining a permissible end. (Centeno v. X x x It does not follow. It is somewhat ironic however. from the constitutional guarantees of the free exercise of religion that everything which may be so called can be tolerated . it safeguards the free exercise of the chosen form of religion. viz. Thus. En Banc [Vitug]) Freedom of Religion 212.the Plunder Cases against the former President Joseph E. as not to unduly infringe on the protected freedom. It has been said that a law advancing a legitimate governmental interest is not necessarily invalid as one interfering with the ―free exercise‖ of religion merely because it also incidentally has a detrimental effect on the adherents of one or more religion. Barangan.. in order that the State may protect its citizens from injury. July 26. during a flag ceremony on pain of being dismissed from one‘s job or of being expelled from school. the power to regulate must be so exercised. sing the national anthem. Villalon-Pornillos. ―The sole justification for a prior restraint or limitation on the exercise of religious freedom (according to the late Chief Justice Claudio Teehankee in his dissenting opinion in German v. it forestalls compulsion by law of the acceptance of any creed or the practice of any form of worship. the present Court believes that the time has come to reexamine it. Discuss the two aspects of freedom of religion. public health 126 . of a serious evil to public safety. the second cannot be. The freedom to act must have appropriate definitions to preserve the enforcement of that protection. the Constitution embraces two concepts. A. On the one hand. 517) is the existence of a grave and present danger of a character both grave and imminent. is not lightly to be trifled with. 259 SCRA 529. for the 30-year old decision of this court in Gerona upholding the flag salute law and approving the expulsion of students who refuse to obey it. 01-4-03-SC. Held: 1. Religious freedom is a fundamental right which is entitled to the highest priority and the amplest protection among human rights. June 29. ―this should not be taken to mean that school authorities are powerless to discipline them‖ if they should commit breaches of the peace by actions that offend the sensibilities. from the coverage of a closed shop agreement between their employer and a union because it would violate the teaching of their church not to join any labor group x x x. Television is a medium that reaches even the eyes and ears of children. to participate in a ceremony that violates their religious beliefs. this religious group which admittedly comprises a ―small portion of the school population‖ will shake up our part of the globe and suddenly produce a nation ―untaught and uninculcated in and unimbued with reverence for the flag. Nevertheless. admiration for national heroes. love of country and admiration for national heroes . and all because a small portion of the school population imposed its will. and moral and spiritual values (Sec. to receive free education. If they quietly stand at attention during the flag ceremony while their classmates and teachers salute the flag. XIV. we upheld the exemption of members of the Iglesia Ni Cristo. of other persons. Art. The situation that the Court directly predicted in Gerona that: ―[T]he flag ceremony will become a thing of the past or perhaps conducted with very few participants. Should this contention be upheld? Held: The Iglesia Ni Cristo's postulate that its religious freedom is per se beyond review by the MTRCB should be rejected. we shall 127 . both religious and patriotic. the democratic way of life and form of government. patriotism. and learn not only the arts. the expulsion of the petitioners from the schools is not justified. the rights and duties of citizenship. and patriotism – a pathetic. however ―bizarre‖ those beliefs may seem to others. respect for human rights. The INC protested by claiming that its religious freedom is per se beyond review by the MTRCB. what the petitioners seek only is exemption from the flag ceremony. 3[2]. demanded and was granted an exemption. Expelling or banning the petitioners from Philippine schools will bring about the very situation that this Court had feared in Gerona. After all. for it is the duty of the State to ―protect and promote the right of all citizens to quality education x x x and to make such education accessible to all‖ (Sec.or any other legitimate public interest. 1987 Constitution) as part of the curricula. In Victoriano v. 219 SCRA 256. under the 1987 Constitution. we do not see how such conduct may possibly disturb the peace. Forcing a small religious group. not exclusion from the public schools where they may study the Constitution. will hardly be conducive to love of country or respect for duly constituted authorities. We are not persuaded that by exempting the Jehovah‘s Witnesses from saluting the flag. 269- 273. 1. their right not to participate in the flag ceremony does not give them a right to disrupt such patriotic exercises. Paraphrasing the warning cited by this Court in Non v. or public welfare. Xxx We hold that a similar exemption may be accorded to the Jehovah‘s Witnesses with regard to the observance of the flag ceremony out of respect for their religious beliefs. sing the national anthem and recite the patriotic pledge. i. XIV).‖ has not come to pass.‖ (Ebralinag v. public health or any other legitimate public interest that the State has a right (and duty) to prevent. The latter classified it as ―rated X‖ because it was shown to be attacking another religion. Dames II. En Banc [Grino-Aquino]) 214. The exercise of religious freedom can be regulated by the State when it will bring about the clear and present danger of a substantive evil which the State is duty-bound to prevent.. public morals. that the State has a right (and duty) to prevent. even tragic situation. singing the national anthem and reciting the patriotic pledge. appreciation for national heroes. Elizalde Rope Workers‘ Union. Its public broadcast on TV of its religious programs brings it out of the bosom of internal belief. serious detriment to the more overriding interest of public health.e.‖ Absent such a threat to public safety. sciences. Philippine history and culture but also receive training for a vocation or profession and be taught the virtues of ―patriotism. The Division Superintendent of Schools of Cebu. March 1. through the iron hand of the law. Xxx Moreover. and the time will come when we would have citizens untaught and uninculcated in and not imbued with reverence for the flag and love of country. ―For sure. public morals. or pose ―a grave and present danger of a serious evil to public safety. A laissez faire policy on the exercise of religion can be seductive to the liberal mind but history counsels the Court against its blind adoption as religion is and continues to be a volatile area of concern in our society today. 1993. while the highest regard must be afforded their right to the free exercise of their religion. the expulsion of members of Jehovah‘s Witnesses from the schools where they are enrolled will violate their right as Philippine citizens. Art. A pre-taped TV program of the Iglesia Ni Cristo (INC) was submitted to the MTRCB for review. not of charity. In fine. therefore. administration of sacraments and other activities with attached religious significance. the State enjoys no banquet of options. the marketplace of ideas demands that speech should be met by more speech for it is the spark of opposite speech. It is for the same reason that religious rallies are exempted from the requirement of prior permit for public assemblies and other uses of public parks and streets (B. ordinations of religious ministers. Blg. Second. the heat of colliding ideas. CA. and a religious purpose is not necessarily a charitable or public welfare purpose. 1. In a State where there ought to be no difference between the appearance and the reality of freedom of religion. an ecclesiastical affair involves the relationship between the church and its members and relate to matters of faith. religious doctrines. Is solicitation for the construction of a church covered by P. 880.‖ Based on this definition. save as those solicited for money or aid may not belong to the same religion as the solicitor. therefore. 1996 [Puno]) 216. the remedy against bad theology is better theology. 1st Div. July 26. it is part of the propagation of religious faith or evangelization. Like solicitation of subscription to religious magazines. the MTRCB cannot squelch the speech of the INC simply because it attacks another religion. it is not the task of the State to favor any religion by protecting it against an attack by another religion. On the other hand. 1564 and. 1996 [Puno]) 215. Vis-à-vis religious differences. Under our constitutional scheme. Sept.continue to subject any act pinching the space for the free exercise of religion to a heightened scrutiny but we shall not leave its rational exercise to the irrationality of man. Villalon-Pornillos. Sec. and the power of excluding from such associations those deemed not worthy of membership. But in American Bible Society v. we precisely held that an ordinance requiring payment of a license fee before one may engage in business could not be applied to the appellant's sale of bibles because that would impose a condition on the exercise of a constitutional right. 124382. 1999. CA. No. examples of this so-called ecclesiastical affairs to which the State cannot meddle are proceedings for excommunication.. 259 SCRA 529. Such restraint. or the adoption and enforcement within a religious association of needful laws and regulations for the government of the membership. The bedrock of freedom of religion is freedom of thought and it is best served by encouraging the marketplace of dueling ideas. For when religion divides and its exercise destroys. may well justify requiring a permit before a church can make Sunday collections or enforce tithing. (Iglesia Ni Cristo v. worship and governance of the congregation. however unclean they may be. To require a government permit before solicitation for religious purpose may be allowed is to lay a prior restraint on the free exercise of religion. 1994) 217. Neutrality alone is its fixed and immovable stance. Solicitation of contributions for the construction of a church is not solicitation for ―charitable or public welfare purpose‖ but for a religious purpose. 3[a]). 16. NLRC. that can fan the embers of truth. What is a purely ecclesiastical affair to which the State can not meddle? Held: An ecclesiastical affair is ―one that concerns doctrine. A fund campaign for the construction or repair of a church is not like fund drives for needy families or victims of calamity or for the construction of a civic center and the like. the need for public protection against fraudulent solicitations does not exist in as great a degree as does the need for protection with respect to solicitations for charity or civic projects as to justify state regulation. if allowed. City of Manila.‖ (Iglesia Ni Cristo v. Aug.. Such solicitation does not engage the philanthropic as much as the religious fervor of the person who is solicited for contribution. the establishment clause of freedom of religion prohibits the State from leaning towards any religion. Religious dogma and beliefs are often at war and to preserve peace among their followers.P. especially the fanatics. (Concurring Opinion. Third. J.V. V. [Kapunan]) 128 . G. Such solicitation calls upon the virtue of faith. (Pastor Dionisio V. The purpose of the Decree is to protect the public against fraud in view of the proliferation of fund campaigns for charity and other civic projects. in Centeno v. creed. When the luxury of time permits. as including within its reach solicitations for religious purposes would be to construe it in a manner that it violates the Free Exercise of Religion Clause of the Constitution x x x. since religious fund drives are usually conducted among those belonging to the same religion. Mendoza. punishable if done without the necessary permit for solicitation from the DSWD? Held: First. Austria v. or form of worship of the church. No.D. 259 SCRA 529. To read the Decree. July 26. To be concrete. Did the MTRCB act correctly when it rated ―X‖ the Iglesia Ni Cristo's pre-taped TV program simply because it was found to be ―attacking‖ another religion? Held: The MTRCB may disagree with the criticisms of other religions by the Iglesia Ni Cristo but that gives it no excuse to interdict such criticisms.R. 236 SCRA 197. the State should not stand still. i. Consequently. Jr. but thoughts and ideas must be well-informed so that the public would gain a better perspective of vital issues confronting them and. Belmonte. the law mandates free public access. reasonable and effective manner. Austria v. it is a purely secular affair.. (Pastor Dionisio V. G. the Court also held that official acts of public officers done in pursuit of their official functions are public in character. serious misconduct.‖ access to which may be limited by law.. it is for the courts to determine on a case by case basis whether the matter at issue is of interest or importance. public consultations and hearing whenever appropriate x x x.e. thus. 16. 9. Certainly.‖ the Court.‖ except when ―otherwise provided by law or when required by the public interest. 299 SCRA 744. as it relates to or affects the public. 6713. Aug. it is by ensuring an unfettered and uninhibited exchange of ideas among a well-informed public that a government remains responsive to the changes desired by the people. faithfully and competently performing their functions as public servants. [and] ensure openness of information. or simply because such matters naturally arouse the interest of an ordinary citizen. In Aquino-Sarmiento v. 1999. in Legaspi v.‖ As to the meanings of the terms ―public interest‖ and ―public concern. Petitioner is a religious minister of the Seventh Day Adventist (SDA). Discuss the scope of the right to information on matters of public concern. 1998.‖ In particular. In the final analysis. and the statements of assets. Under Republic Act No. Held: In Valmonte v. In general. PCGG. What are some of the recognized restrictions to the right of the people to information on matters of public concern? 129 . hence. willful breach of trust. the records pertaining to such official acts and decisions are within the ambit of the constitutional right of access to public records. liabilities and financial disclosures of all public officials and employees. Undeniably. be able to criticize as well as participate in the affairs of the government in a responsible. at reasonable hours. ‗Public concern‘ like ‗public interest‘ is a term that eludes exact definition. Likewise did the ―public nature of the loanable funds of the GSIS and the public office held by the alleged borrowers (members of the defunct Batasang Pambansa)‖ qualify the information sought in Valmonte as matters of public interest and concern. as held in Tanada. to the annual performance reports of offices and agencies of government and government-owned or controlled corporations. the suit may not be dismissed invoking the doctrine of separation of church and the state.218.R. 1st Div. either because these directly affect their lives. worship or doctrines of the church. Dec. He filed an illegal termination case against the SDA before the labor arbiter. writings coming into the hands of public officers in connection with their official functions must be accessible to the public. the Court emphasized that the information sought must be ―matters of public concern. [Kapunan]) The Right of the People to Information on Matters of Public Concern 219. [Panganiban]) 220. public officials and employees are mandated to ―provide information on their policies and procedures in clear and understandable language. NLRC. 124382. the state policy of full public disclosure extends only to ―transactions involving public interest‖ and may also be ―subject to reasonable conditions prescribed by law. consistent with the policy of transparency of governmental affairs. This principle is aimed at affording the people an opportunity to determine whether those to whom they have entrusted the affairs of the government are honestly. the essence of democracy lies in the free-flow of thought. No. Civil Service Commission. He was dismissed because of alleged misappropriation of denominational funds. Both terms embrace a broad spectrum of subjects which the public may want to know.‖ So was the need to give the general public adequate notification of various laws that regulate and affect the actions and conduct of citizens. elucidated: ―In determining whether or not a particular information is of public concern there is no rigid test which can be applied. (Chavez v. the minister was not excommunicated or expelled from the membership of the congregation but was terminated from employment.‖ Considered a public concern in the above-mentioned case was the ―legitimate concern of citizens to ensure that government positions requiring civil service eligibility are occupied only by persons who are eligibles. Should the motion be granted? Held: Where what is involved is the relationship of the church as an employer and the minister as an employee and has no relation whatsoever with the practice of faith. Similarly. Morato. gross and habitual neglect of duties and commission of an offense against the person of his employer‘s duly authorized representative. The SDA filed a motion to dismiss invoking the doctrine of separation of Church and State. As regards the right to strike. 1998 [Panganiban]) Freedom of Association 222. as amended]). by former President Marcos. the overriding considerations of national security and the preservation of democratic institutions.A. 299 SCRA 744. Likewise. Another declared overriding consideration for the expeditious recovery of ill-gotten wealth is that it may be used for national economic recovery. 1989) further prohibits public officials and employees from using or divulging ―confidential or classified information officially known to them by reason of their office and not made available to the public. which courts may not inquire into prior to such arrest. 2) Trade or industrial secrets (pursuant to the Intellectual Property Code [R. Ferrer-Calleja. 9. there are standards for allowable limitations such as the legitimacy of the purposes of the association. or their having taken undue advantage of their public office. walkouts and other forms of mass action which will result in temporary stoppage or disruption of public service‖ (CSC Memorandum Circular No. 180 which provides guidelines for the exercise of the right of government workers to organize.‖ This is a clear manifestation that the state may. s. all government officers and employees from staging strikes. We believe the foregoing disquisition settles the question of whether petitioner has a right to respondents' disclosure of any agreement that may be arrived at concerning the Marcoses‘ purported illgotten wealth. 4) Other confidential information. the Constitution itself qualifies its exercise with the proviso ―in accordance with law. as well as the internal deliberations of the Supreme Court. 6. information on inter-government exchanges prior to the conclusion of treaties and executive agreements may be subject to reasonable safeguards for the sake of national interest. the right of civil servants to organize themselves was positively recognized in Association of Court of Appeals Employees (ACAE) v. or their use of powers. 221. As such. 8293. the assets and properties referred to supposedly originated from the government itself. Held: Specifically. by law. as follows: 1) National security matters and intelligence information. subject only to the satisfaction of positive claims of certain persons as may be adjudged by competent courts. mass leaves. 3) Criminal matters. they belong to the people. influences or relationships. therefore. diplomatic and other national security matters. Is the recovery of the alleged ill-gotten wealth of the Marcoses a matter of public concern subject to this right? Held: With such pronouncements of our government x x x there is no doubt that the recovery of the Marcoses‘ alleged ill-gotten wealth is a matter of public concern and imbued with public interest . 7[c]. The Ethical Standards Act (R. To all intents and purposes. enacted on February 20. or even deny certain sectors such right.‖ Clearly. 1998 [Panganiban]). the prosecution and the detention of criminals. the SC enumerated the recognized restrictions to the right of the people to information on matters of public concern. 1997] and other related laws) and banking transactions (pursuant to the Secrecy of Bank Deposits Act [R. ―resulting in their unjust enrichment and causing grave damage and prejudice to the Filipino people and the Republic of the Philippines. (Chavez v.‖ (Sec. detention and prosecution. as in the exercise of the rights of free expression and of assembly. implicitly endorsed an earlier CSC circular which ―enjoins under pain of administrative sanctions.Held: In Chavez v. No. regulate the use of this right. Dec. 1987. Dec. his immediate family. No. No. approved on June 6.) Other acknowledged limitations to information access include diplomatic correspondence.A. such as those relating to the apprehension. 9. upon reconveyance they will be returned to the public treasury. 6713. for instance. Does the right of civil servants to organize include their right to strike? Clarify. 1405. 130 . ibid. relatives and close associates through or as a result of their improper or illegal use of government funds or properties. closed door Cabinet meetings and executive sessions of either house of Congress. But. This jurisdiction recognizes the common law holding that there is a governmental privilege against public disclosure with respect to state secrets regarding military. We may also add that ―ill-gotten wealth‖ refers to assets and properties purportedly acquired. PCGG (299 SCRA 744. Executive Order No. PCGG. 1987) by stating that the Civil Service law and rules governing concerted activities and strikes in the government service shall be observed. demonstrations.A. dated April 21. directly or indirectly. Nov. either petition the Congress for the betterment of the terms and conditions of employment which are within the ambit of legislation or negotiate with the appropriate government agencies for the improvement of those which are not fixed by law. and regulations. changing or arranging the terms and conditions of employment. such as the removal of US bases and the repudiation of foreign debts. including any political subdivision or instrumentality thereof and government-owned and controlled corporations with original charters are governed by law and employees therein shall not strike for the purpose of securing changes [thereto]. not through collective bargaining agreements. including any political subdivision or instrumentality thereof are governed by law. do not seek to establish that they have a right to strike. therefore. In government employment. however. Subject to the minimum requirements of wage laws and other labor and welfare legislation. workers in the public sector do not enjoy the right to strike. And this is effected through statutes or administrative circulars. should not have been penalized.‘ x x x. the Court affirmed the above rule in Social Security System Employees Association (SSSEA) v.It is also settled in jurisprudence that. rules. But employees in the civil service may not resort to strikes. It cannot be denied that the mass action or assembly staged by the petitioners resulted in the non-holding of classes in several public schools during the corresponding period. Should their contention be upheld? Held: Petitioners. 1997. They denied that they engaged in ―strike‖ but claimed that they merely exercised a constitutionally guaranteed right – the right to peaceably assemble and petition the government for redress of grievances . the dispute may be referred to the Public Sector Labor-Management Council for appropriate action.‖ After delving into the intent of the framers of the Constitution. 14. regardless of whether the disputants stand in the proximate relation of employers and employees . therefore. Alliance of Concerned Government Workers v. undertaken for essentially economic reasons. means any temporary stoppage of work done by the concerted action of employees as a result of an industrial or labor dispute. as the underlying facts are allegedly not identical. Carino. ‗[t]he terms and conditions of employment in the government. If there be any unresolved grievances.and. They refused to return to work despite orders to do so and subsequently were found guilty of conduct prejudicial to the best interests of the service for having absented themselves without proper authority. 281 SCRA 657. they tenaciously insist that their absences during certain dates in September 1990 were a valid exercise of their constitutional right to engage in peaceful assembly to petition the government for a redress of grievances. the administrative heads of government which fix the terms and conditions of employment. As now provided under Sec. the terms and conditions of employment in the unionized private sector are settled through the process of collective bargaining. Relations between private employers and their employees rest on an essentially voluntary basis. which took effect after the instant dispute arose. in their mass actions. Court of Appeals. and penalized. government workers cannot use the same weapons employed by the workers in the private sector to secure concessions from their employers. They claim that their gathering was not a strike. maintaining. Minister of Labor and Employment rationalized the proscription thus: ―The general rule in the past and up to the present is that the ‗terms and conditions of employment in the Government. 4. Petitioners public school teachers walked out of their classes and engaged in mass actions during certain dates in September 1990 protesting the alleged unlawful withholding of their salaries and other economic benefits. or absence from. Rather. through their unions or associations. their participation therein did not constitute any offense. fixing. Petitioners do not 131 . With these premises. therefore. Rule III of the Rules and Regulations to Govern the Exercise of the Right of Government Employees to Self-Organization. where properly given delegated power. Since the terms and conditions of government employment are fixed by law.‖ should not principally resolve the present case. The principle behind labor unionism in private industry is that industrial peace cannot be secured through compulsion by law. like workers in the private sector. they constituted a concerted and unauthorized stoppage of. or the association or representation of persons in negotiating. work which it was the teachers‘ duty to perform. who are public schoolteachers and thus government employees. as defined by law. it is the legislature and. En Banc [Panganiban]) 223. to pressure the Government to accede to their demands. Strike. MPSTA v. from their schools during regular school days. Court of Appeals and explained: ―Government employees may. A labor dispute includes any controversy or matter concerning terms and conditions of employment. Laguio and ACT v. walkouts and other temporary work stoppages. we now evaluate the circumstances of the instant petition. in general.‘‖ (Jacinto v. in which this Court declared that ―these ‗mass actions‘ were to all intents and purposes a strike. They also raised national issues. their ―employers‖ .could have held them liable for the valid exercise of their constitutionally guaranteed rights. In Balingasan v. 243 SCRA 666. The relations between capital and labor are not merely contractual. this Court said that the fact that the conventional term ―strike‖ was not used by the participants to describe their common course of action was insignificant. En Banc [Panganiban]) The Non-Impairment Clause 224.to fully and justly implement certain laws and measures intended to benefit them materially x x x.to dramatize their grievances and to dialogue with the proper authorities within the bounds of law. Nov.essentially. they cannot be struck down on the ground that they violate the contract clause. but all such regulations must be subject to change from time to time. such contracts are subject to the special laws on labor unions. weekends or holidays . The Article on Social Justice was aptly described as the ―heart of the new Charter‖ by the President of the 1986 Constitutional Commission. as the general well-being of the community may require. rules and regulations. To hold otherwise is to alter long-established constitutional doctrine and to subordinate the police power to the contract clause. CA. And under the Civil Code. POEA. 14. morals. Their act by their nature was enjoined by the Civil Service law. Moreover. bases and the repudiation of foreign debt. The executive order creating the POEA was enacted to further implement the social justice provisions of the 1973 Constitution. for such matters cannot be placed by contract beyond the power of the State to regulate and control them. collective bargaining. the temporary stoppage of classes resulting from their activity necessarily disrupted public services. (Jacinto v. be made answerable. v. comfort. however. for which they must. which have been greatly enhanced and expanded in the 1987 Constitution by placing them under a separate Article (Article XIII).S. As it was. Had petitioners availed themselves of their free time . 1997.recess. Court of Appeals. or as the circumstances may change. even though contracts may thereby be affected. from their schools during regular school days. wages. working conditions. the very evil sought to be forestalled by the prohibition against strikes by government workers. therefore. the CSC or even this Court . (The Conference of Maritime Manning Agencies. And probably to clothe their action with permissible character. safety. 281 SCRA 657. They are so impressed with public interest that labor contracts must yield to the common good. the freedom to contract is not absolute. contracts of labor are explicitly subject to the police power of the State because they are not ordinary contracts but are impressed with public interest. Social justice is identified with the broad scope of the police power of the state and requires the extensive use of such power . was deemed controlling . Nor is there merit in the claim that the resolution and memorandum circular violate the contract clause of the Bill of Rights. Inc.O. which was enacted under the police power of the State. Rather. It does not prevent a proper exercise by the State of its police power by enacting regulations reasonably necessary to secure the health. Verily. the Civil Service Commission found them guilty of conduct prejudicial to the best interest of the service for having absented themselves without proper authority. Is the constitutional prohibition against impairing contractual obligations absolute? Held: 1. 797 (Creating the POEA). The challenged resolution and memorandum circular being valid implementations of E. retired Justice Cecilia Munoz Palma . Jr. 1995 [Davide. No. or general welfare of the community. 1700. x x x. closed shop. The constitutional prohibition against impairing contractual obligations is not absolute and is not to be read with literal exactness. hours of labor and similar subjects.not the DECS. strikes and lockouts. they also raised national issues such as the removal of the U. for which they were responsible. since the substance of the situation.]) 132 . all contracts and all rights are subject to the police power of the State and not only may regulations which affect them be established by the State. and not its appearance. in order to participate in the mass protest. Article 1700 thereof expressly provides: Art. or as experience may demonstrate the necessity. April 21. It is restricted to contracts with respect to property or some object of value and which confer rights that may be asserted in a court of justice. their absence ineluctably resulting in the non-holding of classes and in the deprivation of students of education. it has no application to statutes relating to public subjects within the domain of the general legislative powers of the State and involving the public rights and public welfare of the entire community affected by it.dispute that the grievances for which they sought redress concerned the alleged failure of public authorities . after classes. the petitioners here x x x were not penalized for the exercise of their right to assemble peacefully and to petition the government for a redress of grievances. no one . Therefore. suffice it to state that ―[n]ot all contracts entered into by the government will operate as a waiver of its non-suability. the law will step in to annul the questionable transactions. [Panganiban]) 3. Only slightly less abstract but nonetheless hypothetical is the contention of CREBA that the imposition of the VAT on the sales and leases of real estate by virtue of contracts entered prior to the effectivity of the law would violate the constitutional provision that ―No law impairing the obligation of contracts shall be passed. (Oposa v. 295 SCRA 366. because private respondents have voluntarily executed quitclaims and releases and received their separation pay. Such quitclaim and release agreements are regarded as ineffective to bar the workers from claiming the full measure of their legal rights.. the non-impairment clause cannot as yet be invoked. En Banc [Purisima]) The In-Custodial Investigation Rights of an Accused Person 133 . except (1) where there is clear proof that the waiver was wangled from an unsuspecting or gullible person. The acts involved in this case are governmental. such undertakings cannot be allowed to bar the action for illegal dismissal. even if it is to be assumed that the same are contracts. In truth. 124. Jr. And since there was thus no extra consideration for the private respondents to give up their employment. fetter the exercise of the taxing power of the State. safety and welfare.2. 685-686. 224 SCRA 792 [1993]) 5. 235 SCRA 630. 1994. processing. v. Secretary of Finance. the non-impairment clause x x x cannot be invoked. Finally. but the reservation of essential attributes of sovereign power is also read into contracts as a basic postulate of the legal order. (Blaquera v. Petitioners pray that the present action should be barred. Since they were dismissed allegedly for business losses. NLRC. X x x. Sept. the holder is not entitled to it as a matter of right. Since timber licenses are not contracts. the same cannot still be stigmatized as a violation of the non-impairment clause. Alcala. x x x. En Banc [Mendoza]) 4. Not all quitclaims are per se invalid or against public policy. granting further that a law has actually been passed mandating cancellations or modifications. For not only are existing laws read into contracts in order to fix obligations as between parties. In these cases. through the exercise of prophetic discernment. 446. Anent petitioners‘ contention that the forcible refund of incentive benefits is an unconstitutional impairment of a contractual obligation. Inc. This is because by its very nature and purpose.‖ It is enough to say that the parties to a contract cannot. Hence. save in cases of renewal. (Bogo-Medellin Sugarcane Planters Association. In the case at bar. or (2) where the terms of settlement are unconscionable on their face. 296 SCRA 108. such a law could have only been passed in the exercise of the police power of the state for the purpose of advancing the right of the people to a balanced and healthful ecology. it is difficult to imagine x x x how the non-impairment clause could apply with respect to the prayer to enjoin the respondent Secretary from receiving. the non-impairment clause must yield to the police power of the state. Factoran. Besides. (Tolentino v. renewing or approving new timber license for. x x x. Aug. the private respondents agreed to the quitclaim and release in consideration of their separation pay. no contract would have as yet existed in the other instances. Petitioners claim that the present suit is a ―grave derogation of the fundamental principle that obligations arising from a valid contract have the force of law between the parties and must be complied with in good faith. distinction must be made between its sovereign and proprietary acts. 1998. accepting. The policy of protecting contracts against impairment presupposes the maintenance of a government which retains adequate authority to secure the peace and good order of society. they are entitled to separation pay under Article 283 of the Labor Code. Jurisprudence holds that the constitutional guarantee of non-impairment of contract is subject to the police power of the state and to reasonable legislative regulations promoting health. the instant case does not involve a law or even an executive issuance declaring the cancellation or modification of existing timber licenses. promoting their health and enhancing their general welfare. with respect to renewal. Moreover. 25. In short.‖ The Court disagrees. 11. Nevertheless. morals. the Court is in agreement with the Solicitor General that the incentive pay or benefit is in the nature of a bonus which is not a demandable or enforceable obligation. the Contract Clause has never been thought as a limitation on the exercise of the State's power of taxation save only where a tax exemption has been granted for a valid consideration . and contemplates effective communication which results in the subject‘s understanding of what is conveyed. at any time. he must be warned that the waiver is void even if he insist on his waiver and chooses to speak. or investigating officer or his companions must do and observe at the time of making an arrest and again at and during the time of the custodial interrogation in accordance with the Constitution. 7438 (An Act Defining Certain Rights of Person Arrested. detaining. the former must also explain the effects of such provision in practical terms – e. 10) The person arrested must be informed that his initial waiver of his right to remain silent. Article III of the 1987 Constitution. Explain the kind of information that is required to be given by law enforcement officers to suspect during custodial investigation. the Court. the degree of explanation required will 134 . knowingly and intelligently and ensure that he understood the same. It shall be the responsibility of the officer to ensure that this is accomplished. 1. Every other warnings. Held: [I]t is settled that one‘s right to be informed of the right to remain silent and to counsel contemplates the transmission of meaningful information rather just the ceremonial and perfunctory recitation of an abstract constitutional principle. State the procedure. information or communication must be in a language known to and understood by said person. Detaining. preferably of his own choice. invited or under custodial investigation must be informed in a language known to and understood by him of the reason for the arrest and he must be shown the warrant of arrest. he must be informed that no custodial investigation in any form shall be conducted except in the presence of his counsel of after a valid waiver has been made. he must be informed that it must be done in writing and in the presence of counsel. letter or messenger . obtained in violation of any of the foregoing. inviting. detained. 3) He must be informed that he has the right to be assisted at all times and have the presence of an independent and competent lawyer. Detained or Under Custodial Investigation as well as the Duties of the Arresting.g.225. 4) He must be informed that if he has no lawyer or cannot afford the services of a lawyer. It is high-time to educate our law-enforcement agencies who neglect either by ignorance or indifference the so-called Miranda rights which had become insufficient and which the Court must update in the light of new legal developments: 1) (People v. if any. shall be admissible in evidence. 302 SCRA 455. 8) In addition. any member of his immediate family. detaining. 6) The person arrested must be informed that. or investigating officer or his companions must do and observe at the time of making an arrest and again at and during the time of the custodial interrogation. one will be provided for him. Held: Lastly. 9) That the person arrested must be informed that he may indicate in any manner at any time or stage of the process that he does not wish to be questioned with warning that once he makes such indication. inviting. The right to be informed carries with it a correlative obligation on the part of the police investigator to explain. 226. Mahinay. in whole or in part.with his lawyer (either retained or appointed). as the case may be. radio. whether inculpatory or exculpatory. 7) He must be informed that he has the right to waive any of said rights provided it is made voluntarily. as guardian of the rights of the people lays down the procedure. Since it is comprehension that is sought to be attained. priest or minister chosen by him or by any one from his immediate family or by his counsel. the police may not interrogate him if the same had not yet commenced. what the person under investigation may or may not do – and in a language the subject fairly understands. jurisprudence and Republic Act No. if the person arrested waives his right to a lawyer. considering the heavy penalty of death and in order to ensure that the evidence against an accused were obtained through lawful means. otherwise. 1999. he has the right to communicate or confer by the most expedient means . 2) He must be warned that he has a right to remain silent and that any statement he makes may be used as evidence against him.telephone. En Banc [Per Curiam]) The person arrested. or the interrogation must cease if it has already begun. guidelines and duties which the arresting. and that a lawyer may also be engaged by any person in his behalf. 11) He must also be informed that any statement or evidence. It is not enough for the interrogator to merely repeat to the person under investigation the provisions of Section 12. the right to counsel or any of his rights does not bar him from invoking it at any time during the process. guidelines and duties which the arresting. regardless of whether he may have answered some questions or volunteered some statements. 5) That whether or not the person arrested has a lawyer. or any medical doctor. and Investigating Officers and Providing Penalties for Violations Thereof).. or be visited by/confer with duly accredited national or international non-government organization. or may be appointed by the court upon petition of the person arrested or one acting on his behalf. Feb. Canoy. counsel of the police. In People v. or by the latter‘s relative or person authorized by him to engage an attorney or by the court. What the Constitution bars is the compulsory disclosure of incriminating facts or confessions. to be valid and effective. Basay (219 SCRA 404. at all stages of the interview. who. and stopping the interrogation once in a while either to give advice to the accused that he may either continue. In fact. Can a PAO lawyer be considered an independent counsel within the contemplation of Section 12. considering that the Mayor has ―operational supervision and control‖ over the local police and may arguably be deemed a law enforcement officer? Held: While it is true that a municipal mayor has ―operational supervision and control‖ over the local police and may arguably be deemed a law enforcement officer for purposes of applying Section 12(1) and (3) of Article III of the Constitution. Feb. this Court stressed that an accused‘s right to be informed of the right to remain silent and to counsel ‗contemplates the transmission of meaningful information rather than just the ceremonial and perfunctory recitation of an abstract constitutional principle. 2nd Div. 1999. choose to remain silent or terminate the interview. his uncounselled confession to him did not violate his constitutional rights.e. Bacor. 418). chooses to waive his right to counsel. The rights under Section 12 are guaranteed to preclude the slightest use of coercion by the State as would lead the 135 . 3rd Div. Oracoy and People v. Espiritu. appellant‘s confession to the mayor was not made in response to any interrogation by the latter. [Panganiban]) 228.. 306 SCRA 522. such waiver. that he is willing to fully safeguard the constitutional rights of the accused. it has been held that the constitutional procedures on custodial investigation do not apply to a spontaneous statement. public or private prosecutor. No police authority ordered appellant to talk to the mayor. March 17. are generally suspect. It was appellant himself who spontaneously. must be a lawyer. counseling or advising caution reasonably at every turn of the investigation. or a municipal attorney whose interest is admittedly adverse to that of the accused-appellant. CJ]) 227. however. 1987 Constitution? Held: In People v. i. i. If the lawyer were one furnished in the accused‘s behalf. a lawyer engaged for an individual facing custodial investigation (if the latter could not afford one) ‗should be engaged by the accused (himself). as distinguished from one who would merely be giving a routine. freely and voluntarily to the Mayor admissible in evidence. 328 SCRA 385. the SC has held that a PAO lawyer can be considered an independent counsel within the contemplation of the Constitution considering that he is not a special counsel. Bandula. 302 SCRA 533. the assistance of a PAO lawyer satisfies the constitutional requirement of a competent and independent counsel for the accused. Thus. (People v. and other relevant personal circumstances of the person undergoing investigation. the relationship between lawyers and law enforcement authorities can be symbiotic. the choice of the individual undergoing questioning. Is the confession of an accused given spontaneously.‘ Lawyers engaged by the police. In further ensuring the right to counsel.. [Davide. Article III. the mayor did not question the appellant at all. When appellant talked with the mayor as a confidant and not as a law enforcement officer. 1999. but given in an ordinary manner whereby appellant orally admitted having committed the crime. it is important that he should be competent and independent. intelligence. 1st Div. If he decides not to retain a counsel of his choice or avail of one to be provided for him and. Deniega as follows: ―x x x [T]he lawyer called to be present during such investigation should be as far as reasonably possible. [Mendoza]) 229. April 30. freely and voluntarily sought the mayor for a private meeting. 2. as in many areas. peremptory and meaningless recital of the individual‘s rights.‖ (People v. ―x x x The competent or independent lawyer so engaged should be present from the beginning to end. What is the meaning of ―competent counsel‖ under Section 12 of the Bill of Rights? Held: The meaning of ―competent counsel‖ was explained in People v.‘ ―Ideally therefore. not elicited through questioning by the authorities. it is not enough that the subject is informed of such right. 2000. (People v. Thus. therefore. must still be made with the assistance of counsel.necessarily vary and depend on the education. under prevailing jurisprudence. whatever testimonials are given as proof of their probity and supposed independence.e. The mayor did not know that appellant was going to confess his guilt to him. he should also be asked if he wants to avail of the same and should be told that he could ask for counsel if he so desired or that one could be provided him at his request. upon proper petition of the accused or person authorized by the accused to file such petition. violence. it was held that statements spontaneously made by a suspect to news reporters on a televised interview are deemed voluntary and are admissible in evidence. because of the inherent danger in the use of television as a medium for admitting one‘s guilt. intimidation. it is prudent that trial courts are reminded that extreme caution must be taken in further admitting similar confessions. which are dealt with in paragraph 2 of Section 12. March 3. would have been sympathetic with him. 20. Aside from this bare assertion. The news reporters acted as news reporters when they interviewed appellant. Indeed. Article III of the 1987 Constitution. under custody. (People v. We do not suggest that videotaped confessions given before media men by an accused with the knowledge of and in the presence of police officers are impermissible. 2001. not by the police or any other investigating officer. The interview was recorded on video and it showed accused-appellant unburdening his guilt willingly. the police. For in all probability. Accused-appellant claims that his confession was obtained by force and threat. A word of caution then to lower courts: we should never presume that all media confessions described as voluntary have been freely given. he could have easily sought succor from the newsmen who. It requires persistence and determination in separating polluted confessions from untainted ones. Held: Apropos the court a quo‘s admission of accused-appellant‘s videotaped confession. threat. They were not acting under the direction and control of the police. 231. if he had indeed been forced into confessing. Endino. Vizcarra. not to prevent him from freely and voluntarily telling the truth. Andan. and (2) those which are given without the benefit of Miranda warnings. 353 SCRA 307.accused to admit something false. are admissible. Feb. In People v. His allegation that the fact that he was made to sign the confession five times is proof that he refused to sign it. In People v. They did not force appellant to grant them an interview and reenact the commission of the crime. 269 SCRA 95. may attempt to legitimize coerced extrajudicial confessions and place them beyond the exclusionary rule by having an accused admit an offense on television. We have a sworn duty to be vigilant and protective of the rights guaranteed by the Constitution. In fact. Such a situation would be detrimental to the guaranteed rights of the accused and thus imperil our criminal justice system. the line between proper and invalid police techniques and conduct is a difficult one to draw. 1997) 230. [Bellosillo]) 232. 2nd Div. therefore. openly and publicly in the presence of newsmen. admissible in evidence. Andan. This type of confession always remains suspect and therefore should be thoroughly examined and scrutinized. Xxx 136 . they asked his permission before interviewing him. Detection of coerced confessions is admittedly a difficult and arduous task for the courts to make. which are the subject of paragraph 1 of the same Section 12. Confessions made in response to questions by news reporters. Besides. particularly in cases such as this where it is essential to make sharp judgments in determining whether a confession was given under coercive physical or psychological atmosphere. Are confessions made in response to questions by news reporters admissible in evidence? Answer: Yes. March 3. Discuss the two kinds of involuntary or coerced confessions under Section 12. Held: There are two kinds of involuntary or coerced confessions treated in this constitutional provision: (1) those which are the product of third degree methods such as torture. X x x X x x However. Illustrate how the Court should appreciate said involuntary or coerced confessions. The Supreme Court further ruled that appellant‘s verbal confessions to the newsmen are not covered by Section 12(1) and (3) of Article III of the Constitution and. with the connivance of unscrupulous media practitioners. 269 SCRA 95. and the recurrence of this phenomenon in several cases . he has shown no proof of the use of force and violence on him. where the accused. force. gave spontaneous answers to a televised interview by several press reporters in the office of the chief of the CIS. 1997. we find such admission proper. He did not seek medical treatment nor even a physical examination. it was held that appellant‘s confessions to the news reporters were given free from any undue influence from the police authorities. (People v. Discuss why lower court‘s should act with extreme caution in admitting in evidence accused‘s videotaped media confessions. Such confession does not form part of custodial investigation as it was not given to police officers but to media men in an attempt to elicit sympathy and forgiveness from the public. in all likelihood. public or private prosecutor. accused-appellant was assisted by Reyes. as PC Captain and Station Commander of the WPD. In this case. if so. In jurisprudence. has been found to be merely ceremonial and inadequate to transmit meaningful information to the suspect. 220 – 208. 332 SCRA 190. [Mendoza]) 233. or counsel of the police whose interest is admittedly adverse to the accused. inducement or trickery. cannot be considered an ―independent contemplated by the law for the reason that he was station commander of the WPD at assisted accused-appellant. But what renders the confession of accused-appellant inadmissible is the fact that accusedappellant was not given the Miranda warnings effectively. x x x. care should have been scrupulously observed by the police investigator that accused-appellant was specifically asked these questions considering that he only finished the fourth grade of the elementary school. though presumably competent. What are the requirements for an extra-judicial confession of an accused to be admissible in evidence? Held: 1. Article III. where the claim of maltreatment in the extraction of the confession is unsubstantiated and where abundant evidence exists showing that the statement was voluntarily executed. 2) Knowingly based on an effective communication to the individual under custodial investigation of his constitutional rights. 137 . municipal attorney. De los counsel‖ as the time he This is error. in several decisions of this Court. Xxx There was thus only a perfunctory reading of the Miranda rights to accused-appellant without any effort to find out from him whether he wanted to have counsel and. was part of the police force who could not be expected to have effectively and scrupulously assisted accused-appellant in the investigation. Obrero. Section 12(1) cannot be special counsel. without compulsion. May 17. There is greater reason for finding a confession to be voluntary where it is corroborated by evidence aliunde which dovetails with the essential facts contained in such confession. Section 12(1) requires that counsel assisting suspects interrogations be competent and independent. the confession contains details that only the perpetrator of the crime could have given. De los Reyes. (People v. Now. Atty. x x x Moreover. the confession is admissible against the declarant. and 3) Intelligently with full appreciation of its importance and comprehension of its consequences. To allow such a happenstance would render illusory the protection given to the suspect during custodial investigation .We discern no sign that the confession was involuntarily executed from the fact that it was signed by accused-appellant five times. (2) he must be warned that anything he says can and will be used against him. Moreover. who. It has been held that voluntariness of a confession may be inferred from its being replete with details which could possibly be supplied only by the accused. reflecting spontaneity and coherence which cannot be said of a mind on which violence and torture have been applied. Bandula. Under the Constitution. As observed in People v. Especially in this case. is presumed to be psychologically coerced. When the details narrated in an extrajudicial confession are such that they could not have been concocted by one who did not take part in the acts narrated. 2nd Div. x x x. the independent counsel required by Article III. in custodial Atty. and (3) he must be told that he has a right to counsel. such as it is called in the United States from which Article III. an uncounseled statement. 2000. under the first paragraph of this provision. the suspect really needs the guiding hand of counsel. Section 12(1) was derived. Xxx Extrajudicial confessions are presumed voluntary. such confession will be sustained. This kind of giving of warnings. a lawyer will be appointed to represent him . in the absence of conclusive evidence showing the declarant‘s consent in executing the same has been vitiated. whether he had his own counsel or he wanted the police to appoint one for him. it is required that the suspect in custodial interrogation must be given the following warnings: (1) he must be informed of his right to remain silent. Here. and. no confession can be admitted in evidence unless it is given: 1) Freely and voluntarily. Swept into an unfamiliar environment and surrounded by intimidating figures typical of the atmosphere of police interrogation. and that if he is indigent. Fabro. 2000. The mantle of protection afforded by the above-quoted constitutional provision covers the period from the time a person is taken into custody for the investigation of his possible participation in the commission of a crime or from the time he is singled out as a suspect in the commission of the offense although not yet in custody. If the rule were otherwise. the burden of proof that it was obtained by undue pressure. (People v.. (Citing I Record of the Constitutional Commission 731-734. However. While the initial choice in cases where a person under custodial investigation cannot afford the services of a lawyer is naturally lodged in the police investigators. however. the tempo of a custodial investigation will be solely in the hands of the accused who can impede. Aug. then. the rule is not intended as a deterrent to the accused from confessing guilt if he voluntarily and intelligently so desires but to protect the accused from admitting what he is coerced to admit although untrue. obstruct the progress of the interrogation by simply selecting a lawyer who for one reason or another. during the martial law period. 11. freely and deliberately confess that he is the perpetrator of a crime unless prompted by truth and conscience. rather. physical or psychological is forcefully apparent. nay. the confession must inspire credibility or be one which the normal experience of mankind can accept as being within the realm of probability. 329 SCRA 158. March 30. the lawyers made available to the detainee would be one appointed by the military and therefore beholden to the military. to be an effective counsel ―[a] lawyer need not challenge all the questions being propounded to his client. Base.‖ (People v. 1997 [Panganiban]) 2. 329 SCRA 158. it was adopted in our Constitution to preclude the slightest coercion as would lead the accused to admit something false . [Ynares-Santiago]) 234. Article 3 of the 1987 Constitution does not convey the message that the choice of a lawyer by a person under investigation is exclusive as to preclude other equally competent and independent attorneys from handling his defense. The counsel. The presence of a lawyer is not intended to stop an accused from saying anything which might incriminate him but. When all these requirements are met and the confession is admitted in evidence. 1st Div. 1st Div. 277 SCRA 19.Once admitted. The exclusionary rule is premised on the presumption that the defendant is thrust into an unfamiliar atmosphere running through menacing police interrogation procedures where the potentiality for compulsion. A lawyer provided by the investigators is deemed engaged by the accused where he never raised any objection against the former‘s appointment during the course of the investigation and the accused thereafter subscribes to the veracity of his statement before the swearing officer. The Constitution of the Republic of the Philippines. the word ―preferably‖ under Section 12(1). Numerous decisions of this Court rule that for an extrajudicial confession to be admissible. 169-171. 169-171. (People v. Verily. p. [Ynares-Santiago]) 138 . and 4) in writing. Indeed – The phrase ―competent and independent‖ and ―preferably of his own choice‖ were explicit details which were added upon the persistence of human rights lawyers in the 1986 Constitutional Commission who pointed out cases where. is not available to protect his interest. Base. threat or intimidation rests upon the accused. March 30. should never prevent an accused from freely and voluntarily telling the truth. This absurd scenario could not have been contemplated by the framers of the charter. 2) made with the assistance of competent and independent counsel. 1987 1st ed. 347) Xxx xxx xxx Withal. it must be: 1) voluntary. 2000. the accused really has the final choice as he may reject the counsel chosen for him and ask for another one. I Bernas. Is the choice of a lawyer by a person under custodial investigation who cannot afford the services of a counsel exclusive as to preclude other equally competent and independent attorneys from handling his defense? Held: It must be remembered in this regard that while the right to counsel is immutable. the option to secure the services of counsel de parte is not absolute. A confession meeting all the foregoing requisites constitutes evidence of a high order since it is supported by the strong presumption that no person of normal mind will knowingly. 3) express. 235. Should courts be allowed to distinguish between preliminary questioning and custodial investigation proper when applying the exclusionary rule? Held: The exclusionary rule sprang from a recognition that police interrogatory procedures lay fertile grounds for coercion, physical and psychological, of the suspect to admit responsibility for the crime under investigation. It was not intended as a deterrent to the accused from confessing guilt, if he voluntarily and intelligently so desires but to protect the accused from admitting what he is coerced to admit although untrue. Law enforcement agencies are required to effectively communicate the rights of a person under investigation and to insure that it is fully understood. Any measure short of this requirement is considered a denial of such right . Courts are not allowed to distinguish between preliminary questioning and custodial investigation proper when applying the exclusionary rule. Any information or admission given by a person while in custody which may appear harmless or innocuous at the time without the competent assistance of an independent counsel should be struck down as inadmissible. It has been held, however, that an admission made to news reporters or to a confidant of the accused is not covered by the exclusionary rule. The admission allegedly made by the appellant is not in the form of a written extra-judicial confession; the admission was allegedly made to the arresting officer during an ―informal talk‖ at the police station after his arrest as a prime suspect in the rape and killing of x x x. The arresting policeman testified that the appellant admitted that he was with the victim on the evening of January 12, 1994, the probable time of the commission of the crime and that he carried her on his shoulder but that he was too drunk to remember what subsequently happened. The arresting policeman admitted that he did not inform the appellant of his constitutional rights to remain silent and to counsel. We note that the alleged admission is incriminating because it places the accused in the company of the victim at the time the crime was probably committed. The exclusionary rule applies. The accused was under arrest for the rape and killing of x x x and any statement allegedly made by him pertaining to his possible complicity in the crime without prior notification of his constitutional rights is inadmissible in evidence. The policeman‘s apparent attempt to circumvent the rule by insisting that the admission was made during an ―informal talk‖ prior to custodial investigation prior is not tenable. The appellant was not invited to the police station as part of a general inquiry for any possible lead to the perpetrators of the crime under investigation. At the time the alleged admission was made the appellant was in custody and had been arrested as the prime suspect in the rape and killing of x x x. The exclusionary rule presumes that the alleged admission was coerced, the very evil the rule stands to avoid. Supportive of such presumption is the absence of a written extra-judicial confession to that effect and the appellant‘s denial in court of the alleged oral admission. The alleged admission should be struck down as inadmissible. (People v. Bravo, 318 SCRA 812, Nov. 22, 1999, En Banc [Gonzaga-Reyes]) 236. Explain the procedure for out-of-court identification of suspects and the test to determine the admissibility of such identification. Held: 1. In People v. Teehankee, Jr., the Court x x x explained the procedure for out-of-court identification and the test to determine the admissibility of such identification. It listed the following ways of identifying the suspects during custodial investigation: show-up, mug shots and line-ups. The Court there ruled: ―x x x. Out-of-court identification is conducted by the police in various ways. It is done thru show-ups where the suspect alone is brought face to face with the witness for identification. It is done thru mug shots where photographs are shown to the witness to identify the suspect. It is also done thru line ups where a witness identifies the suspect from a group of persons lined up for the purpose. Since corruption of out-of-court identification contaminates the integrity of in court identification during the trial of the case, courts have fashioned out rules to assure its fairness and its compliance with the requirements of constitutional due process. In resolving the admissibility of and relying on out-of- court identification of suspects, courts have adopted the totality of circumstances test where they consider the following factors, viz: (1) the witness‘ opportunity to view the criminal at the time of the crime; (2) the witness‘ degree of attention at that time; (3) the accuracy of any prior description given by the witness; (4) the level of certainty demonstrated by the witness at the identification; (5) the length of time between the crime and the identification; and (6) the suggestiveness of the identification procedure.‖ (People v. Timon, 281 SCRA 577, Nov. 12, 1997 [Panganiban]) 2. x x x. The totality test has been fashioned precisely to assure fairness as well as compliance with constitutional requirements of due process in regard to out-of-court identification. These cited 139 factors must be considered to prevent contamination of the integrity of in-court identifications better. (People v. Gamer, 326 SCRA 660, Feb. 29, 2000, 2nd Div. [Quisumbing]) 237. Does the prohibition for custodial investigation conducted without the assistance of counsel extend to a person in a police line-up? Consequently, is the identification by private complainant of accused who was not assisted by counsel during police line-up admissible in evidence? Held: The prohibition x x x does not extend to a person in a police line-up because that stage of an investigation is not yet a part of custodial investigation. It has been repeatedly held that custodial investigation commences when a person is taken into custody and is singled out as a suspect in the commission of the crime under investigation and the police officers begin to ask questions on the suspect‘s participation therein and which tend to elicit an admission . The stage of an investigation wherein a person is asked to stand in a police line-up has been held to be outside the mantle of protection of the right to counsel because it involves a general inquiry into an unsolved crime and is purely investigatory in nature. It has also been held that an uncounseled identification at the police lineup does not preclude the admissibility of an in-court identification. The identification made by the private complainant in the police line-up pointing to Pavillare as one of his abductors is admissible in evidence although the accused-appellant was not assisted by counsel. x x x (People v. Pavillare, 329 SCRA 684, 694-695, April 5, 2000, En Banc [Per Curiam]) 238. Petitioner in a case ―x x x posits the theory that since he had no counsel during the custodial investigation when his urine sample was taken and chemically examined, Exhibits ―L‖ and ―M,‖ x x x are also inadmissible in evidence since his urine sample was derived in effect from an uncounselled extra-judicial confession. Petitioner claims that the taking of his urine sample allegedly violates Article III, Section 2 of the Constitution x x x.‖ Should his contentions be upheld? Held: We are not persuaded. The right to counsel begins from the time a person is taken into custody and placed under investigation for the commission of a crime, i.e., when the investigating officer starts to ask questions to elicit information and/or confession or admissions from the accused. Such right is guaranteed by the Constitution and cannot be waived except in writing and in the presence of counsel. However, what the Constitution prohibits is the use of physical or moral compulsion to extort communication from the accused, but not an inclusion of his body in evidence, when it may be material. In fact, an accused may validly be compelled to be photographed or measured, or his garments or shoes removed or replaced, or to move his body to enable the foregoing things to be done, without running afoul of the proscription against testimonial compulsion . The situation in the case at bar falls within the exemption under the freedom from testimonial compulsion since what was sought to be examined came from the body of the accused. This was a mechanical act the accused was made to undergo which was not meant to unearth undisclosed facts but to ascertain physical attributes determinable by simple observation. In fact, the record shows that petitioner and his co-accused were not compelled to give samples of their urine but they in fact voluntarily gave the same when they were requested to undergo a drug test. (Gutang v. People, 335 SCRA 479, July 11, 2000, 2nd Div. [De Leon]) The Right to Bail 239. In bail application where the accused is charged with a capital offense, will it be proper for the judge to grant bail without conducting hearing if the prosecutor interposes no objection to such application? Why? Held: Jurisprudence is replete with decisions compelling judges to conduct the required hearings in bail applications, in which the accused stands charged with a capital offense. The absence of objection from the prosecution is never a basis for the grant of bail in such cases, for the judge has no right to presume that the prosecutor knows what he is doing on account of familiarity with the case. ―Said reasoning is tantamount to ceding to the prosecutor the duty of exercising judicial discretion to determine whether the guilt of the accused is strong. Judicial discretion is the domain of the judge before whom the petition for provisional liberty will be decided. The mandated duty to exercise discretion has never been reposed upon the prosecutor.‖ Imposed in Baylon v. Sison was this mandatory duty to conduct a hearing despite the prosecution's refusal to adduce evidence in opposition to the application to grant and fix bail. (Joselito V. Narciso v. Flor Marie Sta. Romana-Cruz, G.R. No. 134504, March 17, 2000, 3rd Div. [Panganiban]) 240. What are the duties of the judge in cases of bail applications where the accused is charged with capital offense? 140 Held: Basco v. Rapatalo enunciated the following duties of the trial judge in such petition for bail: 1) Notify the prosecutor of the hearing of the application for bail or require him to submit his recommendation; 2) Conduct a hearing of the application for bail regardless of whether or not the prosecution refuses to present evidence to show that the guilt of the accused is strong for the purpose of enabling the court to exercise its sound discretion; 3) Decide whether the evidence of guilt of the accused is strong based on the summary of evidence of the prosecution; 4) If the guilt of the accused is not strong, discharge the accused upon the approval of the bailbond. Otherwise, petition should be denied. The Court added: ―The above-enumerated procedure should now leave no room for doubt as to the duties of the trial judge in cases of bail applications. So basic and fundamental is it to conduct a hearing in connection with the grant of bail in the proper cases that it would amount to judicial apostasy for any member of the judiciary to disclaim knowledge or awareness thereof.‖ Additionally, the court's grant or refusal of bail must contain a summary of the evidence for the prosecution, on the basis of which should be formulated the judge's own conclusion on whether such evidence is strong enough to indicate the guilt of the accused. The summary thereof is considered an aspect of procedural due process for both the prosecution and the defense; its absence will invalidate the grant or the denial of the application for bail. (Joselito V. Narciso v. Flor Marie Sta. Romana-Cruz, G.R. No. 134504, March 17, 2000, 3rd Div. [Panganiban]) 241. Should the accused who remained at large after their conviction be allowed provisional liberty? Can the bail bond that the accused previously posted be used during the entire period of appeal? Held: Despite an order of arrest from the trial court and two warnings from the Court of Appeals, petitioners had remained at large. It is axiomatic that for one to be entitled to bail, he should be in the custody of the law, or otherwise, deprived of liberty. The purpose of bail is to secure one‘s release and it would be incongruous to grant bail to one who is free . Petitioners‘ Compliance and Motion x x x came short of an unconditional submission to respondent court‘s lawful order and to its jurisdiction. The trial court correctly denied petitioners‘ motion that they be allowed provisional liberty after their conviction, under their respective bail bonds. Apart from the fact that they were at large, Section 5, Rule 114 of the Rules of Court, as amended by Supreme Court Administrative Circular 12-94, provides that: Xxx The Court, in its discretion, may allow the accused to continue on provisional liberty under the same bail bond during the period to appeal subject to the consent of the bondsman. The bail bond that the accused previously posted can only be used during the 15-day period to appeal (Rule 122) and not during the entire period of appeal. This is consistent with Section 2(a) of Rule 114 which provides that the bail ―shall be effective upon approval and remain in force at all stages of the case, unless sooner cancelled, until the promulgation of the judgment of the Regional Trial Court, irrespective of whether the case was originally filed in or appealed to it.‖ This amendment, introduced by SC Administrative Circular 12-94 is a departure from the old rules which then provided that bail shall be effective and remain in force at all stages of the case until its full determination, and thus even during the period of appeal. Moreover, under the present rule, for the accused to continue his provisional liberty on the same bail bond during the period to appeal, consent of the bondsman is necessary. From the record, it appears that the bondsman x x x filed a motion in the trial court x x x for the cancellation of petitioners‘ bail bond for the latter‘s failure to renew the same upon its expiration. Obtaining the consent of the bondsman was, thus, foreclosed. (Maguddatu v. Court of Appeals, 326 SCRA 362, Feb. 23, 2000, 1st Div. [Kapunan]) 242. Is a condition in an application for bail that accused be first arraigned before he could be granted bail valid? Held: In requiring that petitioner be first arraigned before he could be granted bail, the trial court apprehended that if petitioner were released on bail he could, by being absent, prevent his early arraignment and thereby delay his trial until the complainants got tired and lost interest in their cases. Hence, to ensure his presence at the arraignment, approval of petitioner‘s bail bonds should be deferred until he could be arraigned. After that, even if petitioner does not appear, trial can proceed as long as he 141 the constitutional right to bail ―flows from the presumption of innocence in favor of every accused who should not be subjected to the loss of freedom as thereafter he would be entitled to acquittal. after a potential extraditee has been arrested or placed under the custody of the law. Constitution) Hence. 1(b) the presence of the accused at the arraignment is required. where the presumption of innocence is not an issue. we believe that the right to due process is broad enough to include the grant of basic fairness to extraditees. In the second place. 24. those cited by the highest court in the requesting state when it grants provisional liberty in extradition cases therein. and (2) foregoing the filing of a motion to quash so that he can be arraigned at once and thereafter be released on bail. the second sentence in the constitutional provision on bail merely emphasizes the right to bail in criminal proceedings for the aforementioned offenses. otherwise the accused may be precluded from filing a motion to quash.is notified of the date of the hearing and his failure to appear is unjustified. 2nd Div. and (2) that there exist special. the judiciary has the constitutional duty to curb grave abuse of discretion and tyranny. extradition proceedings are separate and distinct from the trial for the offenses for which he is charged.‖ Accordingly and to best serve the ends of justice. 148571. as a matter of reciprocity. we believe and so hold that. humanitarian and compelling circumstances including. adaptable to every situation calling for its application. On the other hand. bail may be applied for and granted as an exception. For if the information is quashed and the case is dismissed. there would then be no need for the arraignment of the accused. Indeed. trial in absencia is authorized. This seems to be the theory of the trial court in its x x x order conditioning the grant of bail to petitioner on his arraignment. 2(b) of the Rules on Criminal Procedure. once granted bail.‖ It follows that the constitutional provision on bail will not apply to a case like extradition. 2000. This theory is mistaken. Guillermo Purganan. Sec. He should apply for bail before the courts trying the criminal cases against him. bail should be granted before arraignment. (Lavides v. It must be noted that the suspension of the privilege of the writ of habeas corpus finds application ―only to persons judicially charged for rebellion or offenses inherent in or directly connected with invasion. Is respondent Mark Jimenez entitled to bail during the pendency of the Extradition Proceeding? Held: We agree with petitioner. These scenarios certainly undermine the accused‘s constitutional right not to be put on trial except upon valid complaint or information sufficient to charge him with a crime and his right to bail. as well as the power to promulgate rules to protect and enforce constitutional rights. Feb. 1. 142 . What is the exception to the ―No Bail‖ Rule in Extradition Proceedings? Held: The rule x x x is that bail is not a matter of right in extradition cases. applies only when a person has been arrested and detained for violation of Philippine criminal laws. unless his guilt be proved beyond reasonable doubt. Sept. G. to condition the grant of bail to an accused on his arraignment would be to place him in a position where he has to choose between (1) filing a motion to quash and thus delay his release on bail because until his motion to quash can be resolved. 14(2) of the Constitution. En Banc [Panganiban]) 244. The provision in the Constitution stating that the right to bail shall not be impaired even when the privilege of the writ of habeas corpus is suspended‖ does not detract from the rule that the constitutional right to bail is available only in criminal proceedings. such as arraignment. However. liberty or property‖ of every person.‖ while under Rule 116.R. As suggested by the use of the word ―conviction. because extradition courts do not render judgments of conviction or acquittal. No. Under Rule 114. since under Art. Sec. [Mendoza]) 243. one of the conditions of bail is that ―the accused shall appear before the proper court whenever so required by the court or these Rules.‖ (Sec. It is ―dynamic and resilient. In the first place x x x in cases where it is authorized. the right to due process extends to the ―life. That the offenses for which Jimenez is sought to be extradited are bailable in the United States is not an argument to grant him one in the present case. Moreover. not before the extradition court. only upon a clear and convincing showing (1) that. Sec. Furthermore. It does not apply to extradition proceedings. It cannot be taken to mean that the right is available even in extradition proceedings that are not criminal in nature. as well as Section 4 of Rule 114 of the Rules of Court. 18. his arraignment cannot be held. III. the applicant will not be a flight risk or a danger to the community. 2002. Hon. Article VII. To stress. (Government of the United States of America v. CA. the trial court could ensure the presence of petitioner at the arraignment precisely by granting bail and ordering his presence at any stage of the proceedings.‖ the constitutional provision on bail x x x. 324 SCRA 321. it is settled that bail may be applied for and granted by the trial court at anytime after the applicant has been taken into custody and prior to judgment. On that basis. it partakes of the nature of police assistance amongst states. We are not overruling the possibility that petitioner may. 2. so that the vital international and bilateral interests of our country will not be unreasonably impeded or compromised. Sept. The Court realizes that extradition is basically an executive. Jalosjos. Not a Flight Risk? Jimenez further claims that he is not a flight risk. intended to address issues relevant to the constitutional rights available to the accused in a criminal action. We have carefully examined these circumstances and shall now discuss them. he has not actually fled during the preliminary stages of the request for his extradition. In its barest concept. while this Court is ever protective of ―the sporting idea of fair play. which may be granted in accordance with the guidelines in this Decision. he claims that his detention will disenfranchise his Manila district of 600. precision and emphatic forcefulness. his constituents were or should have been prepared for the consequences of the extradition case against their representative. he stresses that he learned of the extradition request in June 1999. it was already of public knowledge that the United States was requesting his extradition. Guillermo Purganan. we are constrained to rule against his claim that his election to public office is by itself a compelling reason to grant him bail. That he has not yet fled from the Philippines cannot be taken to mean that he will stand his ground and still be within reach of our government if and when it matters. it would be unfair to confine him during the pendency of the case. 3. 24. Hence. In the present case.R. unduly delay the proceedings.‖ it also recognizes the limits of its own prerogatives and the need to fulfill international obligations. 143 . Are there special circumstances compelling enough for the Court to grant Mark Jimenez‘s request for provisional release on bail? Held: Along this line. the Court has already debunked the disenfranchisement argument x x x. In short. the applicant bears the burden of proving the above twotiered requirement with clarity. responsibility arising from the presidential power to conduct foreign relations. with all the more reason would the grant of bail not be justified. and since it is derived essentially from general principles of justice and fairness. G. Jimenez contends that there are special circumstances that are compelling enough for the Court to grant his request for provisional release on bail. yet. even after bail has been previously denied. any intrusion by the courts into the exercise of this power should be characterized by caution. Again we are not convinced. 1. This we cannot allow. Hon. Yet.Since this exception has no express or specific statutory basis. any further discussion of this point would be merely anticipatory and academic. Neither is it. 148571. if the delay is due to maneuverings of respondent. In any event. this fact cannot be taken to mean that he will not flee as the process moves forward to its conclusion. as he hears the footsteps of the requesting government inching closer and closer. not a judicial. No. in bad faith. Thus. Anticipated Delay Respondent Jimenez further contends that because the extradition proceedings are lengthy. En Banc [Panganiban]) 245. Giving premium to delay by considering it as a special circumstance for the grant of bail would be tantamount to giving him the power to grant bail to himself. To support this claim. We must emphasize that extradition cases are summary in nature. In People v. not to determine his guilt or innocence. the extradition court may continue hearing evidence on the application for bail. They are resorted to merely to determine whether the extradition petition and its annexes conform to the Extradition Treaty. that is. (Government of the United States of America v. Respondent Jimenez was elected as a member of the House of Representatives. It must be noted that even before private respondent ran for and won a congressional seat in Manila. This is quite another matter that is not at issue here. as a rule. Hence. Premises considered and in line with Jalosjos. which is not normally a judicial prerogative.000 residents. Alleged Disenfranchisement While his extradition was pending. True. including his detention pending the final resolution of the case. It would also encourage him to stretch out and unreasonably delay the extradition proceedings even more. he has not fled the country. 2002. upon the resolution of the Petition for Extradition. We are not persuaded. However. It matters not how conclusive and convincing the evidence of guilt may be. If the defendant is not a conscious and intelligent participant. G. and this has several reasons underlying it. the Court held in Pecho v. the dignity of the proceedings may be disrupted. it is important that the defendant knows why he is being punished. Second. 327 SCRA 771. 2002.g. for an incompetent defendant is likely to conduct himself in the courtroom in a manner which may destroy the decorum of the court. the right to effectively consult with counsel. Third. an accused cannot be held liable for more than what he is indicted for. En Banc [Panganiban]) The Right to be Informed of the Nature and Cause of Accusation against the Accused 246. An incompetent defendant may not realize the moral reprehensibility of his conduct. and 3) To inform the court of the facts alleged. and the right to confront opposing witnesses. as an incompetent defendant who cannot comprehend the proceedings may not appreciate what information is relevant to the proof of his innocence. a comprehension which is greatly dependent upon his understanding of what occurs at trial. e. Corollary to this. the right to testify in his own behalf. To convict an accused of an offense higher than that charged in the Complaint or Information on which he is tried would constitute unauthorized denial of that right. March 10. What are the objectives of the right to be informed of the nature and cause of accusations against the accused? Held: Instructive in this regard is Section 6. the fairness of the proceedings may be questioned. 1987 Constitution). Bayya. 2000. rather than of the public. as there are certain basic decisions in the course of a criminal proceeding which a defendant is expected to make for himself. To put a legally incompetent person on trial or to convict and sentence him is a violation of the constitutional rights to a fair trial. if one should be had. (People v. What is the purpose of the rule barring trial or sentence of an insane person? What are the reasons underlying it? Held: The rule barring trial or sentence of an insane person is for the protection of the accused. Section 14[2]. Fourth. his lack of comprehension fundamentally impairs the functioning of the trial process. an indictment must fully state the elements of the specific offense alleged to have been committed as it is the recital of the essentials of a crime which delineates the nature and cause of accusation against the accused. The omission is not merely formal in nature since doctrinally. 148571. Xxx In the case under scrutiny. En Banc [Purisima]) The Right to a Fair Trial 247. The purpose of the above-quoted rule is to inform the accused of the nature and cause of the accusation against him. People that the objectives of this right are: 1) To furnish the accused with such a description of the charge against him as will enable him to make the defense. Guillermo Purganan. but an accused cannot be convicted of any offense. Even if the defendant remains passive. For one. so that it may decide whether they are sufficient in law to support a conviction. Moreover. and one of these is his plea.. The societal goal of institutionalized retribution 144 . No. a right guaranteed by no less than the fundamental law of the land (Article III. He has a right to be informed of the nature of the offense with which he is charged before he is put on trial. It has been held that it is inhuman to require an accused disabled by God to make a just defense for his life or liberty. 2) To avail himself of his conviction or acquittal for protection against a further prosecution for the same cause. It is thus imperative that the Information filed with the trial court be complete – to the end that the accused may suitably prepare for his defense. which rights are safeguards for the accuracy of the trial result. the adjudication loses its character as a reasoned interaction between an individual and his community and becomes an invective against an insensible object. he is not in a position to exercise many of the rights afforded a defendant in a criminal case. Sept. not charged in the Complaint or Information on which he is tried or therein necessarily included. A criminal proceeding is essentially an adversarial proceeding.R. the information does not allege the minority of the victim x x x although the same was proven during the trial x x x. the accuracy of the proceedings may not be assured. Rule 110 of the Rules of Court x x x. 24. Elaborating on the defendant‘s right to be informed.(Government of the United States of America v. Hon. substantial probability of irreparable harm.. He submits that the respondent Ombudsman has developed bias and is all set to file the criminal cases in violation of his right to due process. Pervasive publicity is not per se prejudicial to the right of an accused to fair trial.‖ 145 . etc. Their mere exposure to publications and publicity stunts does not per se fatally infect their impartiality. Thus. Teehankee. gavel-to-gavel coverage does not by itself prove that the publicity so permeated the mind of the trial judge and impaired his impartiality. Nos. The totality of circumstances of the case does not prove that the trial judge acquired a fixed opinion as a result of prejudicial publicity which is incapable of change even by evidence presented during the trial. we laid down the doctrine that: ―We cannot sustain appellant‘s claim that he was denied the right to impartial trial due to prejudicial publicity. What are the two principal legal and philosophical schools of thought on how to deal with the rain of unrestrained publicity during the investigation and trial of high profile cases? Held: There are two (2) principal legal and philosophical schools of thought on how to deal with the rain of unrestrained publicity during the investigation and trial of high profile cases . In People v. x x x. Estrada.R. et al. 2001. For another. et al. we rule that the right of an accused to a fair trial is not incompatible to a free press. Jr. Court of Appeals. as well pointed out. i. 333 SCRA 699. In the case at bar. US courts assume a skeptical approach about the potential effect of pervasive publicity on the right of an accused to a fair trial. v. a responsible press has always been regarded as the handmaiden of effective judicial administration. we rejected this standard of possibility of prejudice and adopted the test of actual prejudice as we ruled that to warrant a finding of prejudicial publicity. En Banc [Puno]) 249.e. clear and present danger. Alejandro. prosecutors. the records do not show that the trial judge developed actual bias against appellant as a consequence of the extensive media coverage of the pre-trial and trial of his case. et al. later reiterated in the case of Larranaga v. To be sure. and judicial processes to extensive public scrutiny and criticism. (Estrada v. We have not installed the jury system whose members are overly protected from publicity lest they lose their impartiality. G.may be frustrated when the force of the state is brought to bear against one who cannot comprehend its significance. The state of the art of our communication system brings news as they happen straight to our breakfast tables and right to our bedrooms. Desierto. June 19.. Then and now. The American approach is different. especially in the criminal field x x x. The press does not simply publish information about trials but guards against the miscarriage of justice by subjecting the police. They have developed different strains of tests to resolve this issue. Appellant has the burden to prove this actual bias and he has not discharged the burden. strong likelihood. it is impossible to seal the minds of members of the bench from pre-trial and other off-court publicity of sensational criminal cases. (People v. 718-719. It is true that the print and broadcast media gave the case at bar pervasive publicity. there must be allegation and proof that the judges have been unduly influenced. our idea of a fair and impartial judge is not that of a hermit who is out of touch with the world. just like all high profile and high stake criminal trials. At best. Xxx This is not the first time the issue of trial by publicity has been raised in this Court to stop the trials or annul convictions in high profile criminal cases.. Should the Ombudsman be stopped from conducting the investigation of the cases filed against petitioner (former President) Estrada due to the barrage of prejudicial publicity on his guilt? Held: Petitioner x x x contends that the respondent Ombudsman should be stopped from conducting the investigation of the cases filed against him due to the barrage of prejudicial publicity on his guilt. En Banc [Puno]) The Right to an Impartial Trial 248. For one.. These news form part of our everyday menu of the facts and fictions of life. responsible reporting enhances an accused‘s right to a fair trial for. appellant can only conjure possibility of prejudice on the part of the trial judge due to the barrage of publicity that characterized the investigation and trial of the case. not simply that they might be. In Martelino. English courts readily stay and stop criminal trials when the right of an accused to fair trial suffers a threat . by the barrage of publicity. 2000. Our judges are learned in the law and trained to disregard off-court evidence and on-camera performances of parties to a litigation. The mere fact that the trial of appellant was given a day-to-day. March 2. 146710-15. The British approach the problem with the presumption that publicity will prejudice a jury. or decisions based on secret bias or partiality. even the principal actors in the case – the NBI. having been regarded not only as an independent right but also as a catalyst to augment the free exercise of the other First Amendment rights with which it was deliberately linked by the draftsmen. the misconduct of participants. criminal trials both here and in England had long been presumptively open. The right to attend criminal trial is implicit in the guarantees of the First Amendment: without the freedom to attend such trials.: We expounded further on this doctrine in the subsequent case of Webb v. and assembly. have been recognized as indispensable to the enjoyment of enumerated rights. expressly guaranteed by the First Amendment. it was wisely held: ‗x x x (a) The historical evidence of the evolution of the criminal trial in Anglo-American justice demonstrates conclusively that at the time this Nation‘s organic laws were adopted. in Martelino. various fundamental rights.‘ Offutt v. We find no procedural impediment to its early invocation considering the substantial risk to their liberty whole undergoing a preliminary investigation. press. Virginia. hostility.g. the First Amendment can be read as protecting the right of everyone to attend trials so as give meaning to those explicit guarantees. that the guarantees of speech and press. share a common core purpose of assuring freedom of communication on matters relating to the functioning of government. few cases can match the high volume and high velocity of publicity that attended the preliminary investigation of the case at bar. Cf. Hon. criminal trials cannot be completely closed to the press and public. which people have exercised for centuries. 99 L Ed 11. v. prohibit government from summarily closing courtroom doors which had long been open to the public at the time the First Amendment was adopted. in the context of trials. Xxx The democratic settings. and its companion cases. a community reaction of outrage and public protest often follows. Indeed. supported by reasons as valid today as in centuries past. thus giving assurance that the proceedings were conducted fairly to all concerned and discouraging perjury. (c) Even though the Constitution contains no provision which by its terms guarantees to the public the right to attend criminal trials. it must be concluded that a presumption of openness inheres in the very nature of a criminal trial under this Nation‘s system of justice. uncontradicted history. et al. In addition. In the seminal case of Richmond Newspapers. providing an outlet for community concern. its excessiveness has been aggravated by kinetic developments in the telecommunications industry. Thus. In guaranteeing freedoms such as those of speech and press. Commentators still bombard the public with views not too many of which are sober and sublime. we recognize that pervasive and prejudicial publicity under certain circumstances can deprive an accused of his due process right to fair trial. 80 S Ct 1038.etc. and thereafter the open processes of justice serve an important prophylactic purpose.. and where their presence historically has been thought to enhance the integrity and quality of what takes place. it is important that society‘s criminal process ‗satisfy the appearance of justice. by the barrage of publicity. Raul de Leon. the right of assembly is also relevant. standing alone. To work effectively.‘ Be that as it may. ―Again. media coverage of trials of sensational cases cannot be avoided and oftentimes. Our daily diet of facts and fiction about the case continues unabated even today. 75 S Ct 11. the First Amendment right to receive information and ideas means. 348 US 11. United States. viz. v. For sure. important aspects of freedom of speech and of the press could be eviscerated. (b) The freedoms of speech. their lawyers and their sympathizers – have participated in this media blitz. the respondents. United States. In the case at bar. and emotion. Levine v. which can best be provided by allowing people to observe such process. A trial courtroom is a public place where the people generally – and representatives of the media – have a right to be present. 362 US 610. et al. not simply that they might be. the significant community therapeutic value of public trials was recognized: when a shocking crime occurs. we find nothing in the records that will prove that the tone and content of the publicity that attended the investigation of petitioners fatally infected 146 . From this unbroken.. Moreover.. we held that to warrant a finding of prejudicial publicity there must be allegation and proof that the judges have been unduly influenced. not expressly guaranteed. Alejandro. 14. petitioners raise the effect of prejudicial publicity on their right to due process while undergoing preliminary investigation. 4 L Ed 2d 989. e. The possibility of media abuses and their threat to a fair trial notwithstanding. Inc. to paraphrase Justice Sanchez in the case of Chavez v. ―Compulsion does not necessarily connote the use of violence. Dec. Accused-appellant alleges that while in the custody of police officers. Article III of the 1987 Constitution which is ordinarily available only in criminal prosecutions. Ong Siu Hong. disable him from making a free and rational choice or impair his capacity for making rational judgment would be sufficient. Court of Appeals (24 SCRA 663 [1968]). Tan Teng. 2001. it may be the product of unintentional statements. Alabama. accused-appellant executed a waiver of the provisions of Article III. G. 23 Phil. En Banc [Puno]) The Right against Self-Incrimination 250. their 26-page Resolution carries no indubitable indicia of bias for it does not appear that they considered any extra-record evidence except evidence properly adduced by the parties. for what is proscribed is the use of testimonial compulsion or any evidence communicative in nature acquired from the accused under duress.‖ Needless to say. En Banc [Per Curiam]) 251. However. did petitioners seek the disqualification of any member of the DOJ Panel on the ground of bias resulting from their bombardment of prejudicial publicity.R.the fairness and impartiality of the DOJ Panel. 361 US 199) It bears emphasis. To be sure. in violation of his right against self-incrimination. Indeed. we note. 399-401. the DOJ Panel is composed of an Assistant Chief State Prosecutor and Senior State Prosecutors. March 2. Desierto. Nos.‖ Applying the above ruling. however. although accusedappellant insists that hair samples were forcibly taken from him and submitted to the NBI for forensic examination. we held that the right against self-incrimination under Section 17. Aside from executing a waiver of the provisions of Article 125 of the Revised Penal Code. the cases against the petitioner are still undergoing preliminary investigation by a special panel of prosecutors in the office of the respondent Ombudsman. Petitioners cannot just rely on the subliminal effects of publicity on the sense of fairness of the DOJ Panel. It appears. Their long experience in criminal investigation is a factor to consider in determining whether they can easily be blinded by the klieg lights of publicity. Well to note. 735 [1917]). 145 [1912]) and morphine forced out of the mouth of the accused may also be used as evidence against him (US v. for these are basically unbeknown and beyond knowing. The length of time the investigation was conducted despite it summary nature and the generosity with which they accommodated the discovery motions of petitioners speak well of their fairness. So is moral coercion tending to force testimony from the unwilling lips of the defendant.‖ (Blackburn v. 1999. At no instance. some hair strands were taken from him without his consent and submitted to the NBI for investigation. Consequently. we hold that there is not enough evidence to warrant this Court to enjoin the preliminary investigation of the petitioner by the respondent Ombudsman. Petitioner needs to offer more than hostile headlines to discharge his burden of proof. Section 12 of the Constitution regarding the rights of an accused during custodial investigation. (People v. Held: The use of evidence against the accused obtained by virtue of his testimony or admission without the assistance of counsel while under custodial investigation is proscribed under Sections 12 and 17. the above-mentioned provisions are an affirmation that ―coercion can be mental as well as physical and that the blood of the accused is not the only hallmark of an unconstitutional inquisition. (Estrada v. No allegation whatsoever has been made by the petitioner that the minds of the members of this special panel have already been infected by bias because of the pervasive prejudicial publicity against him. Pressure which operates to overbear his will. 9. Indeed. 320 SCRA 383. The aforesaid rules are set forth in the Constitution as a recognition of the fact that the psychological if not physical atmosphere of custodial investigations in the absence of procedural safeguards is inherently coercive in nature. substance emitted from the body of the accused may be received as evidence in prosecution for acts of lasciviousness (US v. the special panel has yet to come out with its findings and the Court cannot second guess whether its recommendation will be unfavorable to the petitioner. that the waivers were executed by the accused without the assistance of a counsel of his own choice. For instance. Board of Medical Examiners (28 SCRA 344 [1969]). such as an administrative investigation of a licensed physician who is charged with 147 . 36 Phil. Article III of the Constitution x x x. Does the right against self-incrimination extend to administrative proceedings? Held: In Pascual v. that under the above-quoted provisions. what is actually proscribed is the use of physical or moral compulsion to extort communication from the accused-appellant and not the inclusion of his body in evidence when it may be material. Rondero. He needs to show more than weighty social science evidence to successfully prove the impaired capacity of a judge to render a bias-free decision. extends to administrative proceedings which possess a criminal or penal aspect. however. 146710-15. the hair samples may be admitted in evidence against him. They are the transactional immunity and the use-andderivative-use immunity. citing the earlier case of Cabal v. Whereas an ordinary witness may be compelled to take the witness stand and claim the privilege as each question requiring an incriminating answer is shot at him. the interest of the Congress in demanding disclosures from an unwilling witness. an accused may altogether refuse to take the witness stand and refuse to answer any and all questions. This right construed as the right to remain completely silent may be availed of by the accused in a criminal case. The critical element is the existence of. What are the two types of immunity statutes? Which has broader scope of protection? Held: Our immunity statutes are of American origin. which could result in his loss of the privilege to practice medicine if found guilty. (Secretary of Justice v. testify and produce evidence before it. this right of the accused is extended to respondents in administrative investigations but only if they partake of the nature of a criminal proceeding or analogous to a criminal proceeding. Clearly then. Jan.‖ Moreover. Rumely. 18. May the right against self-incrimination be validly invoked during inquiry in aid of legislation? Held: [I]t has been held that ―a congressional committee‘s right to inquire is ‗subject to all relevant limitations placed by the Constitution on governmental action. En Banc [Melo]) 252. Kapunan (6 SCRA 1059 [1962]). if pursued. By its grant. The Honorable Court of Appeals. et al. (Bengzon.immorality.‖ (Watkins v. Lantion. Jr. 203 SCRA 767. 2000. however. In Galman v. En Banc [Padilla]) 253. press. pointed out that the revocation of one‘s license as a medical practitioner. In the United States. it is only because we hold that the questioned inquiry is not in aid of legislation and. is an even greater deprivation than forfeiture of property. whether a party or not. the Court reiterated the doctrine in Cabal v. ―Petitioner. Transactional immunity is broader in the scope of its protection. Nov. but it may be invoked by other witnesses only as questions are asked of them. that every congressional investigation is justified by a public need that over-balances any private rights affected. occupies a different tier of protection from an ordinary witness. 354 USS 178 citing US v. ordained by the Constitution. religion or assembly. Cabal cannot refuse to take the witness stand and testify. 345 US 41) One of the basic rights guaranteed by the Constitution to an individual is the right against selfincrimination. it is not the character of the suit involved but the nature of the proceedings that controls. thus – This distinction is enunciated by the Court in Romeo Chavez v. and that he can invoke his right against self-incrimination only when a question which tends to elicit an answer that will incriminate him is propounded to him. 322 SCRA 160. Senate Blue Ribbon Committee. Kapunan to illustrate the right of witnesses to invoke the right against self-incrimination not only in criminal proceedings but also in all other types of suit. as accused. To do so would be to abdicate the responsibility placed by the Constitution upon the judiciary to insure that the Congress does not unjustifiably encroach upon an individual‘s right to privacy nor abridge his liberty of speech. a witness is only assured that his or her particular testimony and evidence derived from it will not be used against him or her in a subsequent 148 . The privilege has consistently been held to extend to all proceedings sanctioned by law and to all cases in which punishment is sought to be visited upon a witness. US. there are two types of statutory immunity granted to a witness. would be violative of the principle of separation of powers between the legislative and the judicial departments of government. It was held that: ―We did not therein state that since he is not an accused and the case is not a criminal case.‘ including ‗the relevant limitations of the Bill of Rights‘.‖ In another case – ―x x x the mere semblance of legislative purpose would not justify an inquiry in the face of the Bill of Rights. a witness can no longer be prosecuted for any offense whatsoever arising out of the act or transaction . 20. 184. by the grant of use-and-derivative-use immunity.‖ We do not here modify these doctrines. v. Pamaran. 1991. If we presently rule that petitioners may not be compelled by the respondent Committee to appear. and the weight to be ascribed to. In contrast. We cannot simply assume. The Court. The days of inquisition brought about the most despicable abuses against human rights. Article III of the Constitution ordains that ―no person shall be twice put in jeopardy of punishment for the same offense. 1994. 231 SCRA 783. rightly. Not the least of these abuses is the expert use of coerced confessions to send to the guillotine even the guiltless. such conviction or acquittal is not indispensable to sustain the plea of double jeopardy of punishment or the same offense. Sandiganbayan. Laws that tend to erode the force of these preeminent rights must necessarily be given a liberal interpretation in favor of the individual. (3) the second jeopardy must be for the same offense. 149 .‖ The second sentence of said clause provides that ―if an act is punishable by a law and an ordinance. Jr. 231 SCRA 783. 1994. even if the offense charged are not the same. 1996) 256. as if it was gifted by the government. must be strictly construed against the accused? Held: [W]e reject respondent court‘s ruling that the grant of section 5 immunity must be strictly construed against the petitioners.prosecution. it misread the raison d‘ etre and the long pedigree of the right against self-incrimination vis-à-vis immunity statutes. Our hierarchy of values demands that the right against self-incrimination and the right to be silent should be accorded greater respect and protection. the crime charged in the other case. conviction or acquittal under either shall constitute a bar to another prosecution for the same act. the first sentence prohibits double jeopardy of punishment for the same offense whereas. the important inquiry relates to the identity of offenses charged. The Right against Double Jeopardy Held: Our Bill of Rights deals with two (2) kinds of double jeopardy. v. The second sentence applies. Jr. The first sentence of Clause 20. even if there has been neither conviction nor acquittal in either case. Elsewhere stated. April 26. Immunity statutes in varying shapes were enacted which would allow government to compel a witness to testify despite his plea of the right against self-incrimination. as such elements are set out in the respective legislative definitions of the offenses involved. 259 SCRA 191. Is the grant of immunity to an accused willing to testify for the government a special privilege and. Discuss the two kinds of double jeopardy. the following must be proven: (1) A first jeopardy must have attached prior to the second. therefore. or is an attempt to commit the same or is a frustration thereof. July 24. the right against self-incrimination was stripped of its absoluteness. To guard against the recurrence of this totalitarian method. Over the years. came the need to assist government in its task of containing crime for peace and order is a necessary matrix of public welfare. a witness is given what has come to be known as transactional or a use-derivativeuse immunity x x x. owing to the fact that one constitutes a violation of an ordinance and the other a violation of statute. Sandiganbayan. (Mapa. v. these immunity statutes are not a bonanza from government. What must be proved to substantiate a claim of double jeopardy? When may legal jeopardy attach? Held: To substantiate a claim of double jeopardy. Under the first sentence. the second contemplates double jeopardy of punishment for the same act. Section 1. (Mapa. provided that he is charged with different offenses. April 26. To insulate these statutes from the virus of unconstitutionality. Quijada. where the offense charged are penalized either by different sections of the same statute or by different statutes. however. Incidentally. If the two charges are based on one and the same act. 797-798. Quite clearly. (2) the first jeopardy must have been validly terminated. or the second offense includes or is necessarily included in the offense charged in the first information. Those given the privilege of immunity paid a high price for it – the surrender of their precious right to be silent. conviction or acquittal under either the law or the ordinance shall bar a prosecution under the other.‖ Thus. or the offense charged in one case is not included in. (People v. The constitutional protection against double jeopardy is available only where an identity is shown to exist between the earlier and the subsequent offenses charged. It simplistically characterized the grant as a special privilege. the defense may be availed of in the other case involving the same offense. The government has a right to solve crimes but it must do it. the right against self-incrimination was ensconced in the fundamental laws of all civilized countries. ex gratia. En Banc [Puno]) 254. 805- 806. So long as jeopardy has been attached under one of the informations charging said offense. En Banc [Puno]) 255. To accommodate the need. one may be twice put in jeopardy of punishment of the same act. or does not include. The question of identity or lack of identity of offenses is addressed by examining the essential elements of each of the two offenses charged. In taking this posture. Since the criminal cases have not yet been terminated. and (e) the case was dismissed or otherwise terminated without the express consent of the accused. However. (c) after arraignment. (b) before a competent court. The scheduled hearings. Hence. What are the exceptions to the rule that the dismissal of a criminal case resulting in acquittal made with the express consent of the accused or upon his own motion will not place the accused in double jeopardy? Held: In the cases at bar. a decision in such case disposes of both the criminal as well as the civil liabilities of an accused. were cancelled when the presiding judge was promoted to the Court of Appeals. CA. 1998 [Panganiban]) 257. the trial court realized that the dates of the hearings were transferred for valid grounds. 1995 of the Respondent Court's decision of June 30. trial court promulgated only the civil aspect of the case. Hence. 1998 [Panganiban]) 258. and his successor as trial judge was not immediately appointed. nor another judge detailed to his sala. in violation of the accused‘s right to speedy trial . a criminal prosecution includes a civil action for the recovery of indemnity. April 15. but not the criminal. which order the appellate court later sustained. after a closer analysis of these successive events. Here. We emphasize that grave abuse of discretion rendered the aforementioned act of the trial court void. Thus. Generally. but later the trial court reconsidered its decision and allowed the case to be reinstated as it noted that the delay in the trial was due to circumstances beyond the control of the parties and of the trial court. the dismissal of a criminal case resulting in acquittal made with the express consent of the accused or upon his own motion will not place the accused in double jeopardy. (Cuison v. CA. 1996 Order. it was also void. namely: insufficiency of evidence and denial of the right to speedy trial. particular regard must also be taken of the facts and circumstances peculiar to each case . double jeopardy cannot prosper as a defense. 1991 Decision. x x x. and his successor as trial judge was not immediately appointed. is there violation of the accused‘s right against double jeopardy? Held: Here we must inquire whether there was unreasonable delay in the conduct of the trial so that violation of the right to speedy trial of the accused x x x resulted. however. April 15. the trial judge rendered a substantially incomplete promulgation on April 4. The Court is not persuaded. For it must be recalled that in the application of the constitutional guaranty of the right to speedy disposition of cases. this rule admits of two exceptions. Will the promulgation of the criminal aspect later constitute double jeopardy? Held: Petitioner contends that "the promulgation by Judge Ramos on April 4. Double jeopardy may attach when the proceedings have been prolonged unreasonably. the Judge promulgated only the civil aspect of the case. If the criminal case was dismissed predicated on the right of the accused to speedy trial. 22. Court of Appeals. delay in the trial was due to circumstances beyond the control of the parties and of the trial court.. Both the trial court and the appellate court noted that after pre-trial of petitioner‘s case was terminated x x x continuous trial was set x x x. 2nd Div. and he repeated his mistake in his April 12. As a rule. Mar. nor another judge detailed to his sala. the first jeopardy has not yet attached. (Cuison v. It was made at the instance of the accused before the trial court. 1991 by reading its dispositive portion has effectively terminated the criminal cases against the petitioner x x x. 355 SCRA 1. [Quisumbing] 259. the trial court set aside its initial order and reinstated the cases against petitioner. We must stress that Respondent Court's questioned Decision did not modify or amend its July 30. Xxx As observed by respondent appellate court. petitioner claims that the first jeopardy attached at that point. the promulgation was not merely incomplete. In fact and in truth. but not the criminal.e. 150 . 1995. (Almario v. (d) when a valid plea has been entered. In excess of its jurisdiction. 2001. 289 SCRA 159.Legal jeopardy attaches only: (1) upon a valid indictment. [T]he promulgation of the CA Decision was not complete. Hence. the order of dismissal based on a violation of the right to speedy trial was made upon motion by counsel for petitioner before the trial court. 289 SCRA 159. It merely ordered the promulgation of the judgment of conviction and the full execution of the penalty it had earlier imposed on petitioner. and with his express consent. In its decision in a criminal case. i." In other words. the presiding judge was promoted to the Court of Appeals. Leviste. the appellate court did not modify the judgment of acquittal. This is double jeopardy. where we overturned an order of dismissal by the trial court predicated on the right to speedy trial – It is true that in an unbroken line of cases. however. Our law recognizes two kinds of acquittal. there was no second jeopardy to speak of. or the case was dismissed or otherwise terminated without the express consent of the accused – was not met. Petitioner was not charged anew in CA-G. CV No. Preliminarily. He submits that in finding him liable for indemnity and damages. Note. petitioner‘s claim that the decision of the appellate court awarding indemnity placed him in double jeopardy is misplaced. it did not state in clear and unequivocal terms that petitioner was not recklessly imprudent or negligent. Since civil liability is not extinguished in criminal cases. (Almario v. that these dismissals were predicated on the clear right of the accused to speedy trial. 066. The trial court‘s initial order of dismissal was upon motion of petitioner‘s counsel. the latter cannot again be charged with the same or identical offense. x x x There being no oppressive delay in the proceedings. Mar. For double jeopardy to exist. Nor did it order the filing of a second criminal cases against petitioner for the same offense. Both the trial court and the Court of Appeals were thus not in error when they allowed reinstatement of the cases against petitioner. the appellate court not only placed his acquittal in suspicion. and (3) the second jeopardy must be for the same offense as the first. 22. 2001.That there was no unreasonable delay of the proceedings is apparent from the chronology of the hearings with the reasons for their postponements or transfers. private respondents cannot invoke their right against double jeopardy. That being the case.‖ Private respondents contend that while the trial court found that petitioner‘s guilt had not been proven beyond reasonable doubt. For this reason. [Quisumbing] 260. These cases are not applicable to the petition at bench considering that the right of the private respondents to speedy trial has not been violated by the State. Tampal. this exception to the fifth element of double jeopardy – that the defendant was acquitted or convicted. however. Obviously. despite the reconsideration of said order. we concur with the conclusion reached by the Court of Appeals that petitioner‘s right to speedy trial had not been infringed. Is there double jeopardy when an accused was acquitted in a criminal case for reckless imprudence but the civil aspect of the case was elevated to the Court of Appeals and the latter found him liable for indemnity and damages? Held: Petitioner opines that the Court of Appeals should not have disturbed the findings of the trial court on the lack of negligence or reckless imprudence under the guise of determining his civil liability. When a person is charged with an offense and the case is terminated either by acquittal or conviction or in any manner without the consent of the accused. The judgment of acquittal became immediately final. 355 SCRA 1. and not reasonable doubt. x x x. For as petitioner‘s right to speedy trial was not transgressed. and no postponements unjustifiably sought. petitioner had once been placed in jeopardy by the filing of Criminal Case No. Where the right of the accused to speedy trial had not been violated. that what was elevated to the Court of Appeals by private respondents was the civil aspect of Criminal Case No. It follows that petitioner cannot invoke the constitutional right against double jeopardy when that order was reconsidered seasonably. 19240 with a second criminal offense identical to the first offense.R. the Court of Appeals had to review the findings of the trial court to determine if there was a basis for awarding indemnity and damages. the following elements must be established: (1) a first jeopardy must have attached prior to the second. there was no reason to support the initial order of dismissal. The records clearly show that no second criminal offense was being imputed to petitioner on appeal. with different effects on the civil liability of the accused. (2) the first jeopardy must have terminated. Court of Appeals. In the instant case. 066 and the jeopardy was terminated by his discharge. First is an acquittal on the ground that the accused is not the author of the act or omission 151 . In modifying the lower court‘s judgment. therefore. He argues that the trial court‘s finding that he was neither imprudent nor negligent was the basis for his acquittal. hence made with the express consent of petitioner. double jeopardy did not attach. reiterated in People v. we have held that the dismissal of cases on the ground of failure to prosecute is equivalent to an acquittal that would bar further prosecution of the accused for the same offense. but also put him in ―double jeopardy. Hence. As this Court had occasion to rule in People v. 2nd Div. Petitioner‘s claim of having been placed in double jeopardy is incorrect. impliedly the trial court acquitted him on reasonable doubt. if the acquittal is based on reasonable doubt. It must be stressed. En Banc [Cortes]) 262. generally. This is the situation contemplated in Rule 111 of the Rules of Court. It does not seek to inflict punishment without a judicial trial. Panga.A. specifying the qualifying circumstances that would aggravate the offense. 8249 cannot be challenged as unconstitutional. and being merely an amendatory statute it does not partake the nature of an ex post facto law. but clearly a procedural statute.A. 350 SCRA 387. civil liability ex delicto is out of the question. 1999 [Martinez]) GOD BLESS Ǖ Ǖ Ǖ 152 . is the most essential. defined a bill of attainder as a legislative act which inflicts punishment on individuals or members of a particular group without a judicial trial. even if the guilt of the accused has not been satisfactorily established. the law did not alter the rules of evidence or the mode of trial. its mode of appeal and other procedural matters. 8249 has preserved the accused‘s right to appeal to the Supreme Court to review questions of law. for a person who has been found to be not the perpetrator of any act or omission cannot and can never be held liable for such or omission. therefore. the judgment in the criminal proceeding cannot be read in evidence in the civil action to establish any fact there determined. Ferrer.A. which may be instituted must be based on grounds other than the delict complained of. even though both actions involve the same act or omission. The same contention has already been rejected by the court several times considering that the right to appeal is not a natural right but statutory in nature that can be regulated by law. No. Jan. and the civil action. different rules of evidence are applicable.e. i. This is the situation contemplated in Article 29 of the Civil Code.. Essential to a bill of attainder are a specification of certain individuals or a group of individuals. No. al. Jan. 2001. What the decree does is to define the offense and provide for the penalty that may be imposed. 8249.A. Court of Appeals. the Supreme Court still has the power of review to determine if the presumption of innocence has been convincingly overcome. R. P. (Misolas v. does not come within the prohibition. is incorrect.‖ Although the two actions have different purposes. Nowhere in the measure is there a finding of guilt and an imposition of a corresponding punishment.D. 2nd Div. where the civil action for damages is ―for the same act or omission. 1866 does not possess the elements of a bill of attainder. penal or otherwise. he is not exempt from civil liability which may be proved by preponderance of evidence only . the matters discussed in the civil case are similar to those discussed in the criminal case. Jan. Petitioner‘s and intervenors‘ contention that their right to a two-tiered appeal which they acquired under R. G. 7975.A. However. (Panfilo M. It has been ruled that adjective statutes may be made applicable to actions pending and unresolved at the time of their passage. notwithstanding herein petitioner‘s acquittal. R. 181 SCRA 648. What is an ex post facto law? Is R. Not being a penal law. There is no encroachment on the power of the court to determine after due hearing whether the prosecution has proved beyond reasonable doubt that the offense of illegal possession of firearms has been committed and that the qualifying circumstances attached to it has been established also beyond reasonable doubt as the Constitution and judicial precedents require. was not precluded from looking into the question of petitioner‘s negligence or reckless imprudence. prohibits retrospectivity of penal laws. 7975 has been diluted by the enactment of R. which amended P. No.complained of. 1606 as regards the Sandiganbayan‘s jurisdiction. Penal laws are those acts of the Legislature which prohibit certain acts and establish penalties for their violations. [Quisumbing]) The Right against Ex Post Facto Law and Bill of Attainder 261. and the lack of judicial trial. R. if any. The reason for this rule is that the parties are not the same and secondarily. has been declared by the Court as not a penal law. in People v. 1990. or those that define crimes. It does not mete out a penalty and. 8249 is not a penal law. Moreover. 128096. 20. Lacson v. 8249 pertains only to matters of procedure. The mode of procedure provided for in the statutory right of appeal is not included in the prohibition against ex post facto laws.A. This instance closes the door to civil liability. 1866 a bill of attainder? Held: [T]he Court. treat of their nature. the retroactive application of R. et. 8249 an ex post facto law? Held: Ex post facto law.A. the Court of Appeals in determining whether Article 29 applied. 29.D. Hence. (Manantan v. This last element. the imposition of a punishment. and provide for their punishment. It is a substantive law on jurisdiction which is not penal in character.D. There being no delict. one which prescribes rules of procedure by which courts applying laws of all kinds can properly administer justice. R. In this case. The second instance is an acquittal based on reasonable doubt on the guilt of the accused. 30.A.. 659-660. the total lack of court intervention in the finding of guilt and the determination of the actual penalty to be imposed. The Executive Secretary.R. What is a bill of attainder? Is P. At any rate. On the removal of the intermediate review of facts. 153 . 03  6 .3.33 3.   ' 103. 7/30 164.6 565. /. .93&7.:.93 &7.089 .:..3:6 .9  31  6454 .9 651 / ..:.51: 6 ..  65:. .9.1.6969.65 5 &0.51:0&7.51: 6 . .5 ..333.5 . 0   5 .. 4794  : ...65 .03  6 .3.33 3..3. .05.9934. /.: ... 7/30 164.3.50(#99..:: 69 . 103..5 .9. &.3  11 &7.::6.6969. &.08915. 9...51:5 .65   9.5:109:4/9.:: 6 . /.65 .9.9./.  65:85.5: 109: 4/9...5.::69.5.3 69 ..65 5 &0...05.. : 32: 6511 65 16454    16454  56.05.9651 / .65   9.4*     .69 6 9. #3775:5..05 .  65:.. . 79.51:0956.79..:.0893.3.51: /0.0631653...3. 6516960071/   .  65:.33.9 6 !%   %  !6   0   5. 3756:   50  &7.3:694736: 0 4.56.&. / /.0:  69 :.:53.03  6 .::69.991 / 79:097..&.. "7565  .9....330891/#/30"0.60945.3 653 69 3.65.3:69 4736:    6:.5 ..0891 .65  3.6.5 9   &09.50 &66.. 56.51 503.9..0891/7/3060.65   9.6773 519 &0..41     7/30 3.51:  &7.6..    '%.6906979679.9069633...5    5 :..51: 0 9 560071 .:53.773.75.6%069#9679.30. 33 .9 5645: .&.65  3. 796015: 6 .33 56.6. 65:.: 53.603.0..: .773:653.03 65:.3644::656 69 . / /.6.9.......9.3: 69 4736:  964 ....65.5:9:  :.:03.4 69 964 .('*9.69069 79679.65 9.:796:65.5.6773   964 .0:  69 :..0891 / 7/30 60.65: 69 9069 6 33.3..991 / 79:097..657961:.  31&0.. 3 0.  .5 ..65 6 65:: .5695015..51 56.6. .6 69563533.6 0945. 796:0..9:5 964 93.::   ':  .3. 15&0.3 1 60 .6..991 / 79:097.065..65   #9:15.036.65 9..3...0.65 4. / /.5.473. 65:. 51 6..51.3   %  !6    "0.1   *    &'%'%""%! !'  '60.50(.045.5:  .3.:3.313/51.3    65  5.03.1:4::.656#69:    .9 069. 65 :. .91 .6  .906: 9: 7515 /69 . 65 10. &.60.65   33.: 5 .5.956&7.::. .      5 ..6/.. 190.515 644.915. 5096. ..5:. ..93 5 9.3 769  7.5/.6949..9  31  ': : .3 9:10. 647964: 945.:.36.9.330.69545.# . 5.5.51.6 0945. 6.9.96.. .56   :9. :651 1:09.3 .49.. 069.:1:4::.36979:.4.:6.19. 4.9.69633..65 6.: 9:10.656.6 4.796:0.6.. 0..51065.:  .:.9 6 64 ..: /0.9.31 : . 101: /.9 . 650 .5131  50315 .331:76:6.: . 065.69.06473.0:09.5.3 93: 989 .0. .96.:. :56.9. /515 765 .:9.3 0.: .65. 796:0.65 6 . 5694.:.559.5.559 5 0 .15079691 5.   "5 .0. 3.:3   ': . .36:.. 69545.0.:.65:46.0891/..5.65:8.95 .3 069.6 79:5..1:7.1:76:.65 650 .1  5 . 160.:165..69545.: /5 31 /69 ..65.: 65 .3.2. '796:0. 6 0647.4.19. 9:10.331:09.6569..:.5.9636. 0945.. .5:::  4.503:. 069.9  10:65.65 .35.65 : ...5:046.61:4::.. 4.6.60.65 5/.3069.5 . 796:0.61:4:::56.69 /.9:63.5. .# .. 069. 0.0656945...1/..65  ....5 .9.9 :05. 5 .5/.1  &.:34..90     ':3.6/0..9.51:....:.1 0. 7630 .... / .   #   &%   0     (#.#.5 69. :65 . 3:3.6 / 0.65 .3:56..7.79694.51 1:09.3113.13.45.: /651.11:4::.31   .5.  : 06473.91:65015:3:3.31 653  ..: 4.7/30.6.31.5*    3.96.3.9.::..5.5. : .9916. .556.13.906::75155..6 "  &%  ..51..33:00945. 6947345.:.945... 13.65   .51 / :..656#69:    .9.65: :05.6.91.50.50:5190.9 4:.510. .506 :50.656769  31  (*5 9 0..60.. ..0904:.51:70:. .5.511.:3 : .0644.92:./3.5:.95 .: 6 794::/3 13.9:..: 69:.30.34.36. 1:4::.773.:60.7:6.:6.    :0::.: :63 769 .:. 3.:: 0..5..945.069..:.7630 4.4:.51.. . .:3  :.. .6.4 :..6.:.65 675.::.33    .58.30. . 31 '7..9. 0 ': .14609..4.:60.4.07.4:3:  47691 .5:.6 7.51 967:  50315 .3 5 .3: ..6 .4 67769.64./3.9.3:..64693.6 7. 30.90.5.: 1:51 . 6 3.65:  .4.9.6631:5156.4   .45...6/0649.:5.65. .5.9..2.9.31.4937.:::66976736.: /56350 /.51.. 5.33 511.&.07.6.156.:6.3:6.. :./33.3:. ..07.6 /5.. 5.9.95.7.3::53 /.29: .::.3365..6: 0 56 1645. .::9075.4.0.5 7.0.4. 1:.0.. 190..4:.51:.46979:5. 9.653.519979:5. 3:.9...6/0.50(#.5*    90.5.6:6 .9606:5969.65: 631 1:09..3:..:0.5" . :60...0:  5.6.45.9.3:.9:63..65.6.:36. '7. .656./69#.3763.3.3::53:631.51 46593 .::.0795073.. .1.5.51. "   % !6  5  5.3:60.5/.65. 30.46953.5:45.9.040.  .6.6: 6...51 . 69.6 .55.:.9. ..4:65:0..51 5 . 5199651  .. 79101  .4.511.5 : 76::/3 . .93  .6 .969  .4. 1:. :/4::65:    . .934/6.3::53 .556.9..   3..007. 0.6569.3:.:.::.51:.6635..6: 5 . . 4::...:::66976735567. 0.5.:.60646.5155769 . 7.67769.6/5.. 7.30.31.65 675.6516:56.51519979:5..3:67910:.33 &076:.69.4 .9.4 :  .6.1. ::.25.30.95.:/:..3:.653.. 4...50:6..30.6.631.58.506. 07.4.9..5/.3:.6965:796/...63109.0.5.65  5.9..9.. ::..50(#..37.9.49.. 6319. 5:.95. "   % !6  5  5.51.94.19647.7.9.5".55./69 #.31 .25.9...4.1659..3./35.:5..95.4 5:.165.65.4.567 .5*     9763. 51% 79.65:  31519.9:76515.. 7.556.65:.:0.31964...9.3:.65..30./1:8. 9.65 7961:.96.:4.65   9.6:6%79:5..7..0.9.44/9:6..3 7. . 9651 . 30...65:  493 65 . .3:./30..:   &0..9 763..03  6 ..1. 65:. .915.3 965.03(*6.3:. ::..51 9.:6969.9.65:.7..37.69.9./ 9:..65.65:   9.469:.37.:4.65:.5..9469 519&0.3 .9.65 763.0.91519.51:0.  ...::.4   95 . 7.9.. 65:.5 &  65:61 765.65: 5 .9...65.5.3:.07. 7..3 644::65  644  9:. 13/9.: 5 . ..1 6. .69.3:.965.:.9.5 51.65.3:6 7961: 69 .9. :0.9.37.  .3 7. 5..2.::.65  6 %  .9. / . 7.: 7.:. 69.3 7. 965.6      69 .3 7.5.  .  &0.9.9.4 4. !"  . 510 . 3. .3  965.9.65:.51:0.: 3..5: .65.65:6906.69.9.4.51 967 6 0.69.07.19: .694  795073:.9 ....37..3..9..:.9.: .93 5645. ::.5 .3:.46:.656%3.:.511. 15: 763.65  93.6579::3:.7.441.656763.5.9.6.37.4 6 9:.167.:697/3060   9.6567.:6969.763.7.0..59..:5 .9.9.: 0.51 44/9: .160.5: 6 :095 .:0.: .: 09.37..96  &0.3.9.06. .9.6.3 7.69.5 6 ..0.9469 &0..669545. .7.0.91 5.:5616/.3.51 :7769.517630:69.37.69..5 69.:..:  69.9.765..5 1636 69 73.:..306510. .3:./3 .6965:4.4.969 763..9.4   51/.37.0.07.7.5..7.9.::.9.:5. .65:   '.763.0.7.5.3:.9.9.7.07.9..:4.37.. 30. .9.0.5 69 .0.96 ..7976:6...9.9  4...9./065::.569.5.30.3 7.. 0.. 4.3:..5.4.65 69 967 69 ..:7.:6969. 16:6   ' 98:...  69 .65:16:56.5.7.9.65:4:.5 763. ::.95..4 .6/979:5.51%  5.65..51519979:5./69#..5".5*     :...50(#.9.65:.7.5.6564.:3.31.65.1165 5.549.1:0.3:. "   %  !6  5  5.9.1519.5/.69:. .6.::9.45%03:33...::./06990.9..7. 3:... :79.5.69: /.1:0..4:56.::..6.3:.03: .51519979:5..31.4.95. 3137.5169979:5..9.1 0..9.90.5.57.07. 30.65:    .3:. 6564.549.. 6 . 56.5.313...95. 3.51519979:5..9 5.03: .1 519 .69::56..9.69: 0.5 / 979:5.9. 7. 03.1:0.33 :0.31. .: . 1465:. . 3:.9::..7. ::.9....4   (*:. .::.65    ..5::700.:   ' .517..51519979:5.::9.3:.537560.6.95.1.9.4:2:. .9. 7.5. 6: 6 %79:5.:.6/30..31..65.1:0..6: /3655.33.64.69: 69..65:./309. . 4.51519979:5..9:. ::.697630 .03.31.6 .91:.:79.11:9.5.3:. 03: ..95..4..:03.4 : 56. 90.7.5169979:5.:.07...9.57.1:09.:79.9..10.6. .:65..779679.65.5.4:43.9.5:65659:669/:#...4.6:0.51064465:5:   3.3 1:7.556..5:.94..::./:5::463:.6.4.51.9.1.119::.50 .3.6#.6.09.4   511 .76:.69:.1/.1.92 '5.931:9...:6.:.703.3:.:631339:0..91:.9:./ . .95. 1:.3 769 16: 56. 50::.54569.:323...1  69 ...5.3.9:469190. : .6.4.0.:69. .964.33  763.4..9.665:/.3365769.:64 . 6.9 5490.3964.5 .934.69.52.465.33:7..92 9.95..31 569 519979:5.....:9650 .969 ..50511 .65.156941:.9 5.51519979:5.65   '9..:665:065:. :...9 4.3 769 469 .51594.31.90.51.25 . .9 065640 036. 54/9.9..0065... 5519: 763.65.9 5490.7..0. .3 34..:   .1.4. .467769.9: 6 . :5:  .456.1 ...51:473..99 .: 56/3:.:5.5.6. 7.4.1.:700065095:6.6.6 979:5.9065:..::.55769.6.190.0.51 5 .6/30.51. 3.5659:: .6535567 /.50:.9.6. &.   5 ...4.3:. 31 . 4.:.5 6 .: 9 :.  .07..4 0..9.31 . 7.95.:1 69 .1 .51 7.519:.5:..3763.9.07.  ./35 . :9 632  .556. 9/.003   0.4 6..7765. 7./3 .. 6: 6 %79:5.9..3:.5   93  ..::: .1967: 7905. 5199651 4645.5 7669  5 .3..65: '&.6.6 064 6.331645.30.0.1195.1/.51 519979:5. /5 .3:..561:. .1 5 ..3 1:..30.:. ::.94 .51 519979:5.6: 4.93 4769: .6: 5 .:.::60...4 .1.  .95.4/1:. 5. 5 67 69 55 0.51 7. 4.90.51:  .6 5 5 . .: 511 4.5.9.1:09. .: 5 .156.65.51:.65:5694.: .51 :9: .9.0.: . 0659::65. . :5 ....95 .96331  7905. 6 .6: :0.5 . 7..69: 6 5694.3:.07.9. ::.33 065.4   5 .9.. 6: 0631 7. 5 . :. .31:.30.45.90.91. 30.557905.51.3:.94.65:..7.5.64301:9.51.51..0659::65.31950 /.9."&.65:69. 3:.65:   :.7.7976:6..1 .93956.9.30. 50 6 :0..4 56919.50 .6 .6 .565.631..51 69.6: 7931 :0.3 967: .5..90.65 5 .69: . :473:.6 675 ..66757.1..51 6 .65: .6: 6 ... :04 76::/3 .969 51 .3365.:473:.96.4 .6 5.  . 0.:0476::/3 60:6:.6 .3 .30. .796:65.::.. 6: 6 %79:5.::.5.65:     93 ..5 979:5.5.6.:/56751.9:5593.4.6:6.930.5 .556.3:.:. ::. 365 1645.. 631 / 3360.:. 365 /5 .65: .596..6:60.66..0659::65..31:.6 16 :6   659:3  .69. 1.4..31.669..94. 7.55:.:519 .95.9.5169979:5. 9..50 6 .65. '7.3:.50.5. 0. 4.3:6 7910 .31 . ::.51 519979:5.5..  /.3.656.9.1 065.6.9. 653 13.4 631 56. .95.  5.5.66369 .3. (./5..9.6./69 #.511 /50.4...663.65    9.9.65 7961: .4:.   "    %  !6   5  5.50(#.03  6 .5  " ...::.5*     &0. 65:.063156.9:  5 ..5197931.7910 6 .65 .3:..6.66.9: ..5/.:.* 7.5. 7905..6..354/96979:5.9...: :.46.. 979:5.5.3:..6:519...:50315..7.33 065:.. 33:0...65:..33 0904:..9.1:.*7.33..50:  31'65:.6/3317.(.51519.:...65989.:. 6:..... 3:.3360..4..65:473:. ....33065:.:50315...6:519..46. 7905.::.6..3:.7.9..354/96979:5.5.979:5.. 3:.       .  659:: . .6 15 .03  6 . 7.65  .51 79:09/ .:3 065: .50: 6 .33 :473 4::.: :. :473 9. 65:.1 769 . 8. . 40.1 .... /96..15 6 &0.9.. .65   9. 93 ... 4 6 979:5...6.. 6 ..9..3 44/9:7 5 ...: 9:91 69 7..3 :. ::. .3:.: 165 653 . 7905.65 730..65   ' 65:. 6: 6 %79:5. 65 5.3 979:5.0.. 979:5.1 .65. 7630 ...95 ..656 7.3:.:   5 ..65.939  659:: 103.9.  659:: 5. 30.: 065:...3 7996. 796769.91 .. 90: 6 .6 79646.1 %    : :. 1:0..::.67905..4.7..3.5106...3:.65.6.6.:  69. 631 /5...55.: 0.95..9...9.4   .9 .:56919.66/.65:.. 979:5.31.9/.5.9.6 989 7./33756:/3655.4..:.69: .36. 3:3.3.65:7.69.65 . 69 141 .51519979:5.:.6. 50::.6 065.5. ..07. 9..7.456919.3:..31.6/5.6.::. :.5...9595469..&0.: 7905.9.9.:5796769..0631.5.6:6%79:5.03  6 .51.9469 56 5557. 69...65.3:.: .469..5.65  9.: 9. 4..65.5.6.. '6:..3..354/966.6.65 : 56.6965:...3:.9.:    65:195. 035 69 7.69   .650.. 493 7961: ..9:.:5.656906..6998945. 65:..33/:65....11. 9..: 5 659::  .945.65 #.3:.9..9..5/.4.#3775. :.   "    %  !6    "0.      5 .559:5.563..50 (#.61./37.5: 19.5*     .5.9:.9. .9.37.:. 65  31  '6 1. #3775. 559: 5 .30.3:.945 . .3 7.9.:. . ..9  9:..3.65.51.7905..4.51 %7/300...3360.69563.65  ./37.5.9: ':..65 . 30.3:. 65:.!6 4.9.:.. 9...337.064/5154/96. 3:.3:. 7905.33 56. 01 .   &0651 . 6 .9....5.7..0659::45:. .3 44/9:7 6 .6.1519.9:631..:  50315 . 6: 6 %79:5.67905.6: 30. 31 6.45446..9. 7.9595.: 0.:.653.67905.. .6. 69 .3 .6:.6.  :.:   '91 .. 5 .9:.3:.4 ... 6: 6 %79:5.. .6 ...34. ::.9 8.31 . ..31..: 658.65.35.3:..6.11. 796769...3979:5.51.317.913::6.:.6.54/966.:...08.51 :5.9.:   69...0. 9.4.446.65.33 6/..65.9:. 6.565..:: "0..51647.:5.9 .6.31..5*     '6 1.65.65 ..   5.5/.6 :. .50(#..9.945 . 7.5: 19.3 6.: 0.317.33 / 0647.9..3:.9.:.8.1 5 796769.   "   % !6 .65 #.5. 69 .9...6 .:0.11.3 54/9 6 6.:  .  ::. 1:8.51.4...: .4  :631 . 6.5.65065..1:8.:/110.30.9...3:.511.6564.57.1  31 '5:...310.331 69 .65:6979603.. 6.. 65:.:0..7.354/966.:9101...6.3:. 69.5.:.69..9. :6.65: 0.6.3:.30.65  .: 910. . 3313 ..65..92 9891 / 3..51 / 46.979603.7905.:559:5.: 56 /5 ..:.3.69.7.. /504.5679.4.:  50 .5.9. . 65  /  6 %  .9.3:.. #. 519 &0.65:   %0..33 .30..  0.:.69.. 45446.:0.67905.9.6: 7.9595 .36.: .6..  653 .:.6.7..9. .69..6..3:.9..: '09..36.6. 7.38:.:'61.:0.:..31...6556:.0.4.95.:.5.945. 6:6%79:5.::. : 0.3 6..310.3:.:.511.1".:/110.33169..1:8.: .9::.::.:.4 :631..6.1  16: .. 7. 69 .9... 03.6. .:.336.: 0.4 5031 653 .31 7.6: /. 69 8.3:.9. ::. 65   .651569/9.0..5:9.6. 69.. 54/9 6 6.53/3.69.0..5196 6430 0945.10. 5.51..511.69.:16:56./6 6430.1.9691: .33 .6..9:.7730. 905 .:  '6.: ./3   5.511.553/3691:8.511..691. 5.     .6 / 103.:8:.60 56. 3/3 0.3 .19:7915065..4.91 30.9054. :.556..91.511.556.9 %:63.310.511.:.103.5196!6...1    4569../6 .3:..0.9 :70.6./6..9./14130.6./065:191:.:0.0.65 .6993. 965.51  5631..65:6.. .1.95::.696..5.31./6   6430 : 7910..510311964..:.310..4./:.::1:8...511.3::0.96.:  .6 /95 :0 .51.30.:.:1:8.34 6 56.6 93.69..65: 69 360.30.53/3 5:00.96.9 5 .:9..: 1:8.31.696:31:8.0.696:34:.56..:5.69.1 65 . / 14130.06.979.511. .60697.5 .:/56.6 . 9.. 65 6 6/.5.9.: .54/966./065:191 :...69.0.6.. .:.631 565.656% '956.34656.6/95:0. 0.651 5 .65 :6 ..:5. 06509950 6 ...69..:0.: 4..9.9 30.510.1.1 5:69.4..511. 69  .  631 565..6545..::    '6965796565045..30.:  5 0 0. .96.51 5 3.515 3.6.    !6.1596 031.: 93 631 / 195.51 . 6 .4.:..0. 5.5.95.656&0..3::0.  33 .5.69.30.9.:33.0.95:: .: .. 9 54/9 6 6. 6965 796565045. 30.  69  .9 6..::47. 6/.99.65:  5.50:69./.. .511.55 ...9.53/30.  .69.3 60:..07.511..7735.6.:.. 3/3 0.:   9991 ..43   .696. .:    %  : .9595:0651 . 30.969:79..1::.::0. 3. :70...9:700.979.4/6:    56... /065.998945.:0..99.69.5.6569.330.65: 0.937961: .50.30.. 6955 .369.:656956./6.93.3.65 6 7.6.31.08:.5.. 3:..:3.659:3.6.1     ' 3.6.515..693::973.79:5.:73.3356..5196:..99.41559:1751565.5.::563:5330.7.:.5...3:.906473.. 493 69 . ..6.9 : 9664  56.54/960659::65. 6 .0:0631 /79603.6.5:.5 .7735.:0..5.336951/% &0.6..76:.6556..6:. 69 5.:  3.656..169:..91 69.5.:0.3 :.69.65 2: 569069:..15.065.656..65  /.9. : 03.7730.03.9 50  .3.95:0.06.:17515 65.6/6..9.3. 317.. 6.: .91.51&0.3.5. 3.   "5 .0./3. 8.6 &50.9.30.0..7730.5 . 5.9.9.6 .656%0. 979:5.4.6/57969 .54.9 : .: 5.9591 / ..6./6.9 .0.::.0.45.9:79.51 3.45. 6..45..3:.10.65 6 .9   &/..963353. 6949 935 .. 065.51 96.51 : .993.: 1:8....55.516...6%:631/065:..9... 6 %   .: . .:.9. ..3:.6. 967: 964 .7.: 0.36. 519. :9./.6.3:.5:..6   ':4....433910.::. 51 519..9:631 0..7905.:3 .51 / .. .31 4. 8.31 .5 / 469 .95. :61:8..1 967:  50 1:9.9.6.317.979:5.915. 5  " .3:....65 .5/.54/9 6 979:5.07.5*     &. .9..69:   65 :6 33 9.9 06509.51  3: .:33509..: 964 ..   "    %  !6   5  5.9..5.51/96.9.65 5 %       5 .: :0. 7630 103.6 .5.50(#.15.7../69 #.135:69:0955#.9.:. .95.6:0:0.4:.69:   &0651 354.#.51 519979:5. .: 065:.0.65. ...51.:06:569:323.9:. 4..69..:::.6066:.64305.9:.6 3..69  69.51 519979:5.:   31  5 .9. 69..69:   93  4.9.:.5..0. 979:5.:6.: 44/9:7:631/365. 06530.07...65:.:.656% 56.9.:692   9:.65:.15&0.: :.51.4.5 . 63365 135:  0331 964..6....51519979:5.9.9.9691: .51 :2: .3361/%.: 3.51.5. 51: .03:65069769.69 73.07.6 7.779679.694669545... 763..31 .3.95. .9..979::3..6 73.  .  :0.5.65 4:..65 . 165 .9. 6 .69763.3. 7.96.5.65 69 06.1 :0.65 . .4:.1 51..  ..0290691.1465:.6.: .9..95.31.37.1967:15.65 /3. 979:5. 4.65 .31 .3 7. 3313.65.3796:65..64. . 9:./65 0:.15..51519979:5. .9. 5.7..31.6.....9.6/30. ..936::0.. ..5:/3655.9:.65: 190.3149365.1.51 519979:5.69.956.56.56.:... 06473 .69 7630 6 5. 4.9691: 3.5 4.:  .5:.65 6 5 .91 :../979:5.95.1 .1    '91  5  6 .1:0..6: 6%79:5.95./35 37560. 103.: 56... 4:. 6/0.79::065:. .1:8.3:.... 979:5. :6  69  .694.: 6 ..3 7.31 ..936:967 . . 4:.4  .9.9 763.9651.0. ::.69:. ::.4    .3:. 7.65 69 .:0.3:.: .91  &0 (* 9.6  7901530.6957..0...6969.9.5. 1656.65.3:.511. 763.07.103.:.969   .5.5 936: 69.519.9...369. : 905 :7769.9469  .9651:691:8..:56.66/.::60.5. 7.45.65   .9190.9.569.:69.65.3   .65  69.. 9.: :  ('* 796/. 56.:.1 9..51 :0.160.79:..65: .65 7961: ..65  69.: :.96 .0.  651.30. 936: 15645.5 695 69545.5.3:.:63506953.65 .606473.5:...65 69.:1..3.3.656.65:   .65.0..5.0.51/.5.796/.6:2.5796/.6. 936: :0.67.657976::   .9.96.:.9.69.3 644::65 5 ..:5.530.9..65  ..:0.9..763.5.:. / 1:8.69 . 65:...6:..630.03(* '796/. / 9:.3.9:.63...   69.:73.:... : ..5.917.3 7.659.65:93.:609:6944/9:695190.65   . 69 15645.9:5.65 4:..3..65:69.:.936::0.95 .65 9:..:697.679 05.:..44/96.33 56.:.:6...  .37.:. '..9.5:..65 6%   0 549.65 : 65 .3 7.9..3.  695 763..34.9555..:7..:6336:    .31 519 &0. 964 .4  6.:.:.3:.: 93:6993.9..9.56..65:.51 69 936: 7976::   . 065:.5050.9.::.9..39 .:5031&0.3:.606473.:.9.653......3:.699676963.45..65: 69.5193. ..6  7901530.7.91   !6.65: ':3.656% 0:.:...65669.9. 7.19:.:631/.30.7 01:8..2567. .: 7.69 69. 56. 7.9 6 .. 7960.31 .1 :0. 511 69 . 69 69.5 5. 69545..16:56.    ..65 4:..:76304:.5. 7..51 69 . 6  69 .517..  .6/06444/9:6.1 /  .5.9.95.69: 69.:::.::.9.. 69.5 ./1:8./3 3756 0.5: /3655 . 9 5.6 5...65:.:.9.65 ..51 519979:5.9.5.06473. / .969 .6:6%79:5.3:.31   .150.4:2: .6 4.  ::.51 / 0. 7.07.51 679.5:   .5:  69.65 4:. 6 . 69 .4  . 967 6 0... .9..3:...5: .9:6 . 7. 69 69. 69545.9.3: 5 .: 60.  ' 7. / . 69545.65 6 .9.5.1 / 0. 4:. / 517515. 1:0.69:  69.7630.3:613.6.51 519979:5.655979:5..  56.: 6 .65333.31 .6 /30.9: : .  519 &0...65.96:.:56.1/./30. .5 7669 69.95.9.556.9:.: 5645: 4:.3.1   .97.. 4:.31.31.3:.: /. 32:16:6   &5..517.65:. 0...9.336 6. 653 06473 .3:6 4:.     3 3.5 .95.5:/3655.. .::  / .6.511.5. 979:5. / 3756 0.51 519979:5..4.3:.0.6:6%79:5.6 .3.95. 56.5.:6. 33.: &93 .1.511.5: 6 /365 .95.69:.515. 3./33979:5.65.51519979:5. 7.: 5645:   '6 97.5169.6. 4.1 :0..6 /.9. 653 . 98945.. 7.. ..6 4.. 6925 03.69: :6 .:   &.  '6 .  .9.64.&. 69 69.6: 6 .31 .5.51519979:5.9.9..9.6/0.65  6 %   .9 5.0.66.65.1 :0.65 4:.9 0. 5645: 4:.5.9.025 . 9/.551:.6. 65698.696. ../.9.779679.5.33/5.796:656..37930.65 .50  .6.50(#.511/651. 5645 4:.65 ':.9/.45.3 065:.99:.3:3..6943.94: . 964 .5145.69 6 .793..9.516:631.5.:5: '796:659. 69 1.0.3 796:65 9.3.556. 4730.796:65:6:.556..5. 964 .:.0.:/59.65.    .. 32: / .:3.65:.65.56.7.65:   '65:...65.5...5.45.6915./.0365. 065:.65796115.5.4...151 763.5*     :0:: ./065:.6065.65 6 4/9:6659:: ...515. "   % !6  5   5..9:.:..91  31  ' 445./69#..:.. :./3065:19.63  5..511 /5.9::964.15.5...5/.5 445./3.5.65 6 &5.94.65. :70..:. 69 1.99:.:9..5".:.90..547.51 44/9: 6 .6.65. 6: 6 %79:5.69: .. : 0659::4.:.50.1069.06.4   0.0  / 7931 964 .99:..51 4/9:6..6.532.&0  '&5.7731653.335.51/9.519.6:6%79:5.9 .65.7.. 195 ...9.0 .69:.:65  365  .51.6 03..:6.:::65:6659:: ./96.6365.51 /9.:: 07.955964.99:.330.51565.47.::. 7.00:1. .0 6 ... 4/. 659::4.1519'. 964 .3 4569 65::  . .51 . .45. 469 .5.3 61 0631 56.51. :..90.6 0655 .:.65 6 445.330.3356.19:.%:1#5..   ' 9:.6 ..5.. 659:: : 5 :::65  #673   .33  5 .33.7936445.3   '65:.5#.6.616 .33 65:: 75:...9 445.1/..99:. .933 151 7.93.9:.51565.5 0. 90:: 69 69 .49:..: 56 445.0:. 69 .33:99519.4 7..5:.515 :::65: 69 0644..9 .4/...9. 3.  .3379:65::.151.5: .5 .. :::65  6.6 .9.7733..:. .39.. .:..: 5.6/3.3 3.03   &0      4/9 6 .6/...:765.5 .61 6 .0650315769. 03. 5. 195 : .65/96.91696:0650.:. 4.99:./3 / 56.50. : 56 . 0:.. 3.: .65:97515.90...65:..9 .9::0793:.6.6.5 : ..5 #.  / 7931 964 .9: 479:6545.::::65:.: 6955.  :631 / .6 :99519 ..77.5.616:6   '79:5. :/0.65.6...979. .4 59. 964 .9::33:./3 / 469 .:.:.5:.169545..796:65 .656.656659::.    . :.50 (5..9: 479:6545.: :/0.5 : .955964.  /.   ' 98945.3:6 /5 9461   69 93.4   69 65:: 75:. 0650.. 69 69: .:6336:  9.519.9345:.99:..44/95631. .36:6:   &%   /      5 . 5..: .4.. 5..&.6*     00:1. .5 50315./ . 4. 9:.659::4..3:3.46.50. : .:25..:6.51 0644.69...3361..7733.36:6:31....5..51.6331:0.1.1 5 .9.65/69.50 6 . 565.5 /5 0650.5: 1:7.:::65: .659::4. 5:.5%646 . 6.556./3 65:    065.  %:63   31'.3.. : 930..76307696.&.65 /5 ...579::65 6 7673.59.9 33 0.5..511 .5935 57969.56.00:1.9:. / 95191 5./.3/.695.  4/9: 5 :04. .65:.::06:4. :4.. .7733. ...9:.7961   69 .65..69.559 .00:1..3.: .5.. .0 6: :.650:./:5..51:::65:: 519:0691/&0..50 6 ..6.6 1. 6 . .036.. ...     4..339 54/9 4.6 16 /:5::  /.44/96659::50.33 065:..1695 964 1..51 4....65  9.51. 8694 . 06473 .:.51519:075. . 65   ' 065545../3 / 479:6545.656&0. 44/9: .556.691/3./:5.9: : 56.91 . 493 .5.. 3..5 0.036.3651./:50 : . 659::4..51 :::65:  .65:..5. 6 ...:56.1964.6 . 6 469 ..65 9. . 679.4.65:   00:1. 094 75:..5 : ....65 '44/9:6659::0. .59. ..7733. 9.:65:631/47.65.06473.:65 69 .:065:. :1657969.  '69. 7673 .6: &% .:56. ::75:65 : 56..45.69.9609: 5 .553::7331/659::69 6.31     . 659::4.76736. 946.3.5.   6969  5 .. .1/53.5 ...:656:0.6 79.694:06510.3.0.1 . 4.3    94. 795..4 ..0..94660 .6179.77.0.630.9.5.773 . .:563: ..5.316 &. 11 .9:631/. 256316:3.3160. .::41 .1:9.145:. 30..93555.9 .. /9..36.6 60  .3 3.9:5 964 .510.6 479:6545. 569045. 946.. / .. .6:79:5.(*  0:.: ..  .. 6 0945.69..90.:1657969.9: 1:8..0.:.:93..7673   3356.946 60 '6166.4964:7910.99..: 5..9.336.3  065545.9116969. .. ..946.5: .694:06510. 7515 .5/9.336993.7/3060969. 4:.7733.5 . :.9.6.3 : 56.13:5. ..7/3060969.6:79:5.3.:63159946.0.5065. .0.16:56..51. . .369.30650.65 :7/30:3..65."59.97515.77.3/51065545.95. 0.:.00:1 4.473.655 0945.7/300&.955..0  ::.9 : 9:2 6 : ...909.:.1 .51.145:.9:   79:650.3 .91.5.9.:3 .61697976::6.3:6:9:.5.656:.. 15: &60.3 69.66.255.1&.3 '9./:06515   '..33.655.: :.59..796.:2:..6...59.00:1.65  ..6 50.:56.5. / 151 /. .0.6.155.4:.%65..90650.: :/0.. .06473..36 #3   .6919:: .094:.60:..  .51 . ..331 ..1 / 56511 .1 ..00:1.73.5.31.   .06.145.6600.99.7733.  4. 5 ..06065545.:1:93.. / :773.556.  794.:. :0.. .:.3 75:45.:6.7 5.30...5.69.9: . 33 / 90...6. 69 .6 79694 0659::65..5..973. .51:....51 ..0.7.: .33964:0633.110...7733.:5.69.93.4/616:0.91:7.:..6.00:1.99:.: ::1   31 .:56.51.6:6%79:5.5.33  5.6.: 9650.:694. .6:99519635.33 ..33 :7951 0 . :::65:.. 9.9.6 :.:.7733.: 5625 .5. 46.:1964. : 79:5.:/5065.65   ': 0.5 56.501/0..5. / 065.633. 5.4:6..4   00:1...39:.460296.3::.631 /. .9.&.:75.65.15165:... 65:.6469...96: 5:.: 964 479:6545..9: .3 600.65 ..:32:.3361794.346. 5.6 ....1 :9.63.44/96659::  4950 69 0647335 . ! 3/1 #9:65 5 5.3'9.9.5..9..:65:  .69.. %65. .5.537.. .:033... . 6 .2.6. .4769.33 79:659:  .9 .7733.9 : . 9.33: .3 6960..65 :9..933./6793:.6919:   .3 69.9 3.1. .6.3361 .2.79:65794::   '9:56:65....5:9 ..6 .3:6 0..:65:    .9703.:69765069.5.369410..50:  ...39... 1:09...4769.00:1.656. 00:1.7733..9 3365.:2::56..549505.5.6. :.5..6 .5..6556.4.24.5. 653 3.5:69  1..2339.510659::65..: .....779.5.33..7733.3:::65:.510644.:694695..793:.65 &0.33 4./99.94.00:1.5..5.6:76:.  6 .3 03.00:1.6 ..06990.7733.5. :70.: :..::  .65::. . 46029 6 .3:6 631 / . 7976::6...: ..4   ' . ..91 .. ..9: .:  . 3.. 13 !69...: 5 . : 065:.5.: /65. .9..6 / ...4/6.90. 1 44/9 6 .5.9 9: . :50  : .7733.1 . 6: 6 %79:5. 9:. ..5. :.9 60: . 06. 6 .5.51 . 3..65:9:.9 : 4. 30.3 /9.. 6 .3361. . .6 9:70. .93 09.00:1.51 .3:6 03.65: 5  : .9 1. .3 : 1.1..4769.5579:.    . : 4. 0651.: .11     5 .6 1:0.50 6 69545.90. . .. . 50::..  .9:6 : 1:...: 064731 .5.51. 6.65 16: 56.8. 06507.51.4: .3   94.90.63. ::95964.0. 25631 .5...4 4..5..0653:03:3..  0631 ..79:65.9. 46919. ..:.:00631.006473:.9:30.7733.94560  #673  .95:: 6 .50(5... 33 .9:.65 6. 11:6 ..36:6: &% /   5..033:..5.0655:679:65 '6..65: 65 : 9164 6 .945.16:62565.9:3.3 335:: .. 34.3325631.. .6 659::  .65   ' 11 :6 ...563659:9:33. ..: 31 5 .0..656065:.3 1.65  31'..9: ...46.:69.. 065:....30644..65 160.565.3.656 .53:3./3./10. .03 .6:.:2. 6..25 589:5..&..65.:93.1/.659::69.3   65..4:  '9:10.3..7730..:6.0.556.3 /651.65..5.95 5. 5:..9. 9.6 134.6*     .036.*763..65 .0.9.6. 0.51..9::. 65:. .:9644.6 10.779679.4:.1796:65/564....65     .79694519 .5.3:  ..356 .: 69.&79469.4.38:.....65   6969  ...65 4.3    65  . 0.3/651.. 9 ...65 .: /5 5 . 905.::  &0.65:.. .65:.: !7..6 .3360.7967935895.65 9.29:506510.5:16:..51:70.56/:.. 6/3.976:: .3 03..7950735.95.65:.163:3.56.   .945.3  % !6  !64/9 &%  (. .:9:10.:5679::796:655.3%//65644.65 .956 #3  ...65. .65 79::3 9065: .9: 5 79769.:067.. '69.50.. !.3:3. 6:: 6 659:: .6 06510.655:0769.9455.7976:61.: 90: : 0904:09/1 / . 69 534.:.. .1 . 769 6 /6.6.69..6/6.5.037961:    '7696/6.606510.3..163:3.1   .3.6/4731 ': &0...1 6 3:3.51..133.. 3 %//65644.6::6659::. &% !6   5...... 589: 5 ./:63..:69../:63.65   565  9    &5.. 769 6 /6.:6.6 06510.6::6659:: .50(#.969:69 .6 79.589:5..*     : .6 06510. .65:56.6.79:5.065.6569.163:3.::65015.1   31  '  65:.&5.589:5.31..9..969  ... 69 534.7696.65 9. 6:: 6 659:: . .65:.1 6 3:3.:.3.065:191.65 595. 589: 5.6 . 33/9:70...51.9.6..1/:0589: :. 5:.0....6 .. 47345.9../9:70. .65:::700..3379611695&0 6...56.95569.00691.. 65:.617960::.65 69 9..1  50315..5:.: 79611 . 99 ..336%.: 13 7/3:193:6796019.:679:65:519.3589:695:..65 5 .606510.6336:.:4:.9.9.:.5. / 5 .. &5.1 .694.:679:65:...65::3    '769..51. %3: 6 #96019 6955 589: 5 1 6 :3.50 .1 796:65 6 .6/0647331.65   &0 589: 4.77..65  ':  .1 6 3:3.95  .65 4:.86. 334.&5.45.3.65 5659::.65 '4.93:3.65 .5.. 6943.65:.1/.3:6.656.5 79676:1 3:3..5 3...9::.6. 69 5 06550.365    :315...51. 65!.51695. .65 6 ..51...956 .5 95.65 69 ...3  !6 .... 0.65 56919.9:10.1..7695.65:.5.673.*     "5&7.99..1  565 9  &5.5895.6.331 .5..:70699:63./4..6 .5589 9:69.2. &5.:145.655190:0..90:6.5../614.5&5. &% !6   5.3 793 /69 .9. 3  #3  .50(#..:.69. 589 .6 .6/4. 65 .1 63:3.9 6 79:65.:09.651%3: .4/9 .133.50644.:.96..63:3....6. 4:.5:70699:63... ..5589:79676:1.6945.0.1/.4:.19 65 .: ..4.36950::./.. 569.5#6505931391.969.989:. 3%//65644.25.9695.3:3..:70 65 .&5.44/9    519&0 6..9.656.6531/. 4.&5.65 :0.6905...9.6/.779679..690644.3669. 5.3.5.91.636625.53.510. .0. .331765.6647.6%7/300. 9.!6  .: 7.. .91667.76::/363.1 ..:765.9..&5.93.9:.5:   /%0.03..656.3.69 6 &"" 5069769..6.45.5.  3 %//65 644.16 ..9.644. &5..1/ .4.9 #...:9991/.1.51..9:/765.:.&5...   '9...77.9  .9/69.69"93...2569.915.:.644..9.0:0.#9.0..6.6565..51%0./3.91667.   "546..9.656&5.:5:..65......516997. 65 0065.. 6 #/30 "09: 3 %//65 644..36.659:.9.516 90..:.4...6.  9651 .:53..995.6:6. .. ..7/30. 10351 .95 31 65  .:..65.51.4654.:33. .   %0.659:6.&.5/. :  069769.59.9:06.:.3 9..:5890631.97.6 .5137910.659 6:  &  565  9  32: 9:1 .6 .:!6 /69.:. 65 ..0.151.6 1 7960:: .19:3 .....916 67.51.65.:9.644.31   . 5625 : 065:..:..45626%64..5.1/9:76515.6.:. : .5  #.65:/3655. . : 76:1 .6.9 69 . 9:63.. 644.51. ..51 .50 .514 65 .65.56.9.5169550.. 6915.9 069: 6 3.5.9415...51 3:3...9765  ::7511 ..51.77.5.9 4469.9 ..:..:1  .556919..1..:5:.: ..65 1.: 589 . .9.4769.65 69 796/.679:.:.51065. . 644.659: 31 .5   ' &5..5 .65 . .9   3.5 :71..:!6  /69.. 7.. ::1 .18.51. 7.&..6 :/765.5  .973.3 ::: 9.151.51 989 . &5.: 9:10. 7.51 .1 5  90.6 / 0:1 964 .... 79.6 3 ..5/...  . 79:5.51 190. 065:.99:.465 5 796015: /69 .  5 0:: 6 .93      .:53.45 .9:56.659: 73.656. 3 %//65 644.51.4.659: .  ...65 . 3 %//65 644.9 0  .3569.1 . 7.. :7/3:15.9:2565 . 31   79:..00:5 9 %0..51 56 ::..69 593 065.&5.: ' 5..636625..1 3:3.  &5.96:5:7.331765. :70 6 &5.2569. :70 6 &5.65  4930..5:    93  .. 9676647.69 593  4.5./ 67.76::/363.45.65 6 065..0.656&0 6%!6  6.473. :..6...3: .79:65&7.916.69 593 9.6.3 6 .4/9.....1 . 51 6997.7976:.6 51 6.1 / 9:76515. 3 %//65 644. 67. . . 967   '9 .1&0.9: .9 69 56.6 5.65 : . 93.: 6 . 589 . .506550.65/0.: 6 #9:15.9. 7976: 6 .6 51 6...9:.6 .1589/9:76515.03.969 .473.065. .969 565.   5 6. .93.5.:. 5:.331:.1 63.9.916 67.77.0...5..65.65: /3655 .6 / 06510..656659:: :50..  69  069769..77.1 63:3. .6/ .1.: .6. 93. #9:15..91667..856 7.:56.5113:3..31 .9 69 56. 69 9  %0.9:10. #9.656%!6  .335. .93 9 %0.....3..163.45 626 %64.1..3 6 ..0: 0...:56. .655631       .644..9.9 691:  ..46 . .3 56..:   !69 : ..65 ..:.5145..9. 69 .50   ': ..6 17.&.31 &50.91667.../35659::.:5 69:60..676:. 95 7.65:595.65  5050 69 .6  .9.945.1 :633 69 .. 4:. 0647951: 796/: 5.: .::. / 93.0..39.516997.51 10.9.65: /3655 .3 69545.45.145:.3 17..:93065011/.: 565' . .&. 50.: 151..69.0. 4...556.65.53..655..606510..6 / 5:..1 .9 50./6936621..0.51 5 9.6.9:6511..: 79507.:  .9..6941.6 5.1 .51.0891/..6959.5:.1.25: 5.5.659:  .0:0.30::7731    . .: ..  69   069769.:3 . 9:76515.106473.: 6 . 65 6..: 769 6 589  .1011.5:9: .. 659:: .1 .   .. 534..:5..5  79:. 5:. 069.1 . 79:65.:2 6659::  5:.6 76: 06997. 9 %0.9 .5 51 5 ...3:...5 5:69.65:6659:: ':..6: 5:..659: .51.69: 69 . 569045.4  .469.:79676:16976::/3511:..3567.36 ..:/579.#9.:.7976:65.. /96.5/..1   '9 : 56 59.163:3. .1&. 3:3.2 10..50 6 .11195.769:/96.7960:: '.0.45.5031::9:610.3:3. 644. 3.4.35:.3 .1 / ..:. ..59:76515..95  : . :: :6. :6:.9.3 .50647.06473.9.65 6:.: 9.9.: : .6 75: .: 6 69545.:::5:.751506.1365/5651/.331 :.659:9:70. 6 .5. 69..606510.:31   '7696659::. .9 515:/3  .9.5.5.. . 19.5/. .. 0.:33..9   :1:  .5 ... 06473.&630.3 6 .65: 06510.3 065640 69763.589:5.3..0.1/5... : 56.9.656 ..3 0.   !6 589 : .644.65: 6 ..::6 .1/531..7.0.6 .. ..:::589:0650955. .:5:.5.. 79650 6 .9:469 .77.79.5 6 ..94:6... 3:3.53.45 626 %64.. : 65 69 0 9:10.: 0..:0.3::.:!6 . . 5.659: 065..556..39.511.. 6 06530.. &.336... 653 76: .. &.644.5 6 10.9:76515.   &% !6   5.. 6 .50..51.1 .9..6/9.39. !6   ' 7.5  .6..5 5096. 3.51. 5   565  9    &5.51.9:76515.3 /.0.5. 0644.939 :.. 3 %//65 644.: 5350 /5 4.6 /.606510. 145. 069.5 145..5  631 / .6 .10.:. &.644..5/.47.5::.045.9 65 .5 631 56.656./3065.50(#.3..5/.3 9:10.6796/.1   55 69.9/5.65 : . &..1/.: /.:655:.. 76::/3.51.1 /69 .. 3:3.4.65 . 03: 164..01/69.. .5/./1:065.. %7/30 0.:145.5 . 6 .133.4:..6..969: .51..515895. '6. 76::/3.1 40 ..*     #..644.1/69..5/. 1964. ...9.  &0 .!6 4:.: 65 6... ..6 1:.6 / 065:191 .3.:06550..65  /0.....3:: '6/5.: .5 9:65 : 64..69./.:56.: . 03:3 5 .. 9:3.    &0   6 ..69:70. 065:631.:8. 6: 6 %79:5.6: %7/300. 6 .5.9/:.1   ': 4. .006915 . 490. /33:     !6   .65 6 .:. 695... : 5 ..: .6. #3775 65:.: 9891 / 9.65 7.3356.50.167...15.945..6 .3.  11 56.:56.9.519. 490..5:  ..659:765..0.51 .5.51 &  !6   5.9 /33: 5 .5 695.4  .33695. 79.365:.:.65 / .65 .: . 65:. 691 03:3 .315 .::506 !6     31':..' ./35.. 95/33 ../. /33695.:.65.5.55.5:0.:..6:4.65:.5:   ..6:6%79:5...0:9891/.647..: .6695.: 4769.: /0....5196:0.03:35. 4../.0. 0659500644. .65 ...&5.9:3...99.:4769. ...95:..:765..:.919:65/.9:3.56.656..9 ..... 6./334..33/1:0::13.63 '76::/3..6..&5.50....5....:..1:./796101 '65::.5. 6.1.3:3.653.45.0. /3305.5156.5.3...55.....7960::0345. :.5...65...5.&5.45145.515.4..615.    '065...653.9.7696.606509.606475:.663..31:5:..6..6 6::6659::. :76956..4.6.6..65.3:6..695 /33:56919.. :76959:70.: .63:3.&5.33/ .068.:.2...: /.6:/33631/.69.45145....631/.679676:.9.&5.065:..:/:.&5.634.0.3.4:.6::7969....5. .5769.. &5.5 6 . 65:.9. 653 5 ... 769 0 519 . .6:6.51.769:/50647.51 .9 0. 6: 6 %79:5.51.:769:.6:693662:. #3775:  065::.9 1.0..9195.35 9 . 5 ..9..9.65 : :.03.. .1 56. 3:3.5 7. 659:: 6 ..9/8...: '90:6...4/9 /..  ..3 .91. 969 56:.: . &5.769 '9: . 769: 6 ..90:6.90:63:3.5769:56..9... 76::::65 6 .65690647.0. :43. /.00265. 6: .. 3:3.51 6 .9 565.0.769 ..95. 65 .:: 6 . 51  65 .:    !6916:.6.5: 6  !6  .51.. 631 :4  7.9..51 ..:5.930.: 653.5 03./33:.9.769/.6/33:65.6.5.7960::63:3.90.659:473...:  .: 0  .99891/... : ..5:/:.36.9769/.9 ..5.:5.:653.65..6:    ...355..51796/34: "5..3.51.: 907.: .9 5.379:5.0.:.51/33:6360.965.6695...4:/0..::156...7796..: 31 5 .93931..0.693595 .:5.351:.451.. 79:95 !6 7.7969..16:56.9.65:4734. ..654:.56.0.0..4693:3.:.5 .6: . / :/:. 65 .....5 .9/33:.07.6:/69& !6  .:9:.5.:.6:03...:    5 ./335. ..579676:...51 .7/301/..&5.556.65:...9:3.9/4./33  !6  ..59.6.8:.6:6%79:5./61:. & &5.6:. &5...5.4796/34:964 .6.. 79.  .65/9.965.:.51 :7.65..45145.:6565..:43. .  30..3 .... :/4::65 / .9.../336. .7696. 3:3.65/.&5. 6: /33   ' 69.6:7.77./.7730. 644.5 .1 901 !6 65!64/9 . 65.: .65:.51   !6   .33..5.6/469:5:. ... '76::::656.0..9 ..65  .65 5 5.: 659:655.3.064 964 ..6.5.51 .0. 6: 6 %79:5.6..:.::1 !6  & !6 ..6/. 5..465.1.45....:.69 . :/:.6.6.656 !6 /.9&5.6:0.46356/33: .' 3..969  519:.6559:70..45145. 6. ..:6.14.31653  &9.9 964 . .656.0. &5.5.1. 6...:659:6563365..:59/5 .    511 ...6..4:/0.1/5315.33 56 1950 /.1.&5.9065:191.6: :6365..5509.56   &09.51 .65:.65796/..: 65 .2  !6    5..9. 44/9:6..1 69 .: .:/69. 644..9 .:5::.9.  .9 6& !6  69.9 .6/33:0.5..360..679676:... 999..:.1 .695.....:493.5/70.769:.&5.4..317515907..5. 1:./33 & !6  .50 &% .379:70...970..:.90  .3.9 69.69: 6.&5./33964.939 /33:   '635....&5. . /33: /33:.5.9:70..&5.659: .6.503.:7. 6.& !6 .&5...6 065:19..6...:/:.6 . 0.0.:.1/5315. &5..5. 6 .5::..4.5 &  !6     '9 : 9. &5..90.94 79::1 69 ..11. 6 .0....1. ..:6&0.(*9/337.334/9.6/0.65:.036.::1 /.    5..0:.659:::.96   31'6/0.036..65  9.50( 516.065365:/0.*     :0::.61...33/79::15.65.65:.65 ..9    '6795..:6&0.65  9.3. 7616936. :..6..64/9.65:.3: 064795:56.1 6..9   .965/7.:::.51 55..65  .9:.65  69..93::3 .7673 .369:6365...3:3.:. .9/4..36/0. ..1..33:61:9   &0.59.:/0..93..94/6115.65::05.:979:699.0..1765.5:6796:65:5/33:60.963353:3..969 / 6936621 ..3064731.5194.:.91.65.1.5..:2:.4.779:.5.4.6.993.33 4. :.51....9..5.9.33..0 .6/5.6563:3.65.51   '6...167.036..9/5065:191 56919..7.96:07/30.33...:63:3.65   '6795. 67769.796015:.65696. 56 5694.:/0.3: ..4.51 0 4.51 0..:6. 9.03.9.91  ..57696 0659500644.6 065694 .9: 56 03.65.1:9169469.59.65 . 79610: 570.: 6 69 69545...5. .#9:15..33.::165!64/9 .5 .9:. :56.:56.5.647365./51.... 4...64.51  31  &0.:.: :7.1/13/9. / 34...6 589 5.65"096.29 6 .33163..3 59....911 / ...9.5 / 5:9.5. 4.51..6::6659:: ..:.1.65    6 9..069.:31..::.650:93." .650.5.65.15..6 .  ./:506:65.65 6 6.65 3:3.:: '635. &5.5:4 69 0647964:5 1950: /.9 9 69 5 .65  6 %   !6   ' 6. 33 56.1 5... '6/:9 .6.9..9 6 5.:..5.4:56.0 ..9493796019.91 69 ..9:10.964.*     ..69.:6593:  31'0.51. 5.657960::/7961513569.4 51.. 4.. 56 065095   ' 4.9 ... .39 .65895.06503:79:47.103.5065::.: :. 54/9 6 44/9: ..65 4610...6:6659::..6 ..167.: /651 . '..03.45.31 . &7.3.. 9164 .  #37751:::60. 06510.606473.6.. #51.6%.: 9.3...:09.:50.: 8:.65 .6 ./96...9.511. 69 4507..5:. 65:.3   "   % !6  3  5.9: 4.:4:.993:.7. 7:.: 4.93.5../61:.5 .9:   5 .45..0. :/0... .:2:.0659500644.36.9::5 .:.73. /61   65:85.95  ..50 65 0 69 065:.5 .: /96.. 3:3. .51 796019 6 065.0. :55 6   33 !6   / ..9:5.. .9.91 ...#.63.03  6 .: . :79 0 .0.16 &% !6    5.5   .9.36%:'6.335/::.51 .31.:9   .50  .:96 . 93:..9 6/:9.96. 069.0. 065950 0644. 34.65 0. .9:.9 17.. 3.35796:65/.36 . /..65  . 4:.56..5 9 . 679.65609: :93.4 .51. /61  5 ..55895.9.3796:6569.6...069. 4.. 56.5:9. 6:.9.50(#9:4. .1 5. ..06... 596331/334/61:. / . .:.530.9 0.9.9:/0.. :/0.. 93:476: :.1950..3  9756 4. ..9 :%:. 98:.65.:.3313/0..90. :63..9769  9966  50.65  07961:69.. &631.:..1 .: . 06473:930..:.:15. 55..:9:10..065. 40.69..:65.9: :.6.3 . .34.: ! ' #%"! "%  !% %&'%'"!""'%& "#'!&&' ""!'!!%&'%'"! #%&%!' #%"%&'%"!'"%!'##%"#%'"!"!&'%"% &0. 6: 6 659:: ... .0..00691531035.: . 6   0 7961: 69 .6 7.6 .  ': 09:65: 6009 5 9 ..56   &09..45. ...5. .69695.50(#9:4.33 ..6516:56.:..0..65 ':::47. 3.:/5:. 600.6 ..3315.  5 5.:    9.65 #9.5.3.79::15.365950644..4. 069.0.9.0 5 :/0.4 6 69545. 065950 0644.0 6 ..656. : .969:.33.15 .5:63.64 :93 34..:   065950 0644.6 63365. /.1 5 .656 % : 56.50(9*     :0::.39.:: /6.606473.630. ......51.9. 6925 6 69 ::.45.649969 3351690. &%   ( 516.9 0.64. / 9. : ..::545...3  49 .31 69 /5 63. 33.5. 1.: .906503:6.1 69 1:9.. ...9..6.65..55 9:.51.9.9 69 ..: .3....9 550.1 065:195 .6.45..*     &0...93:6. . ' 0.50  .51.5 9 .65:.31. 73.3.656 .:0::..5065.:.:  9:3.065:.3  "   % !6  3  5.6.: 9:10.5 .*     6 069. 7.45.6960.: .167.5. 065950 /33   . 5.790:1950:/..690:.51.: 9:10.0.65   31  3 .9 : %:. : 93.1 ..::545.5963313360.9 :. 769 .569. 7.1...:/0.: 6593: 5.9 4..6.65:9. 13/9.. .3: 5":45..65:. 596331 /33 160.65.1.6.. 6.65.09.6 9:635....65:1:09/1.769.6:0790:65 .:/0..65609: .. 4.1..65 5650.9.656.5.6: 69. .5 49 064.66:: 59....5.9..6569.3 6919 9:.51.49. 6 &0..3:/0..31.9..9.1 9:3.&5.36.93.9:6/6..51.59.65:650659500644.51195. / 9334.5.91./61.55.69 56.1 5 . :/0.51 94.5.:.33.3:.9765.1 .95694:679::65 .4.630..6 .65/.. 65:.. .65 .25 / . 6:  .33 ... 10:65 5 ..  469 . 130..36.95   31  519 .:56..95  9756  4.: 5.:7.6.989659::.6. 0:.9... 9.4: .9.77961/.0.9 09..09.9.   .5 796:65: 6 .6 1.51 065:3. ..945 ..6.: 69. .945 03.9 .. 695.6 1..: 93  .. :..&5.065.1 /5 ..596331 /33 . 79:15 609: 6 /6..:69.: 9:1 . /33 .1 .:9.: /5 13 7.. 6:: .339:70. ..9:56150.3 ..656 ..::1  519 . .: .. . 79676:1 . 596331 /33:5. /5 6/.6 .: 6.:. 65:.33/.: 151 03.5 .156.: 9:1 .69.156.9:.9.33.6569..6 .9 0..1 /5 :997.65. #9:15. 9...51 796:65: 0 ../:63.5.. .6 5 36625.9.1 5 ....45145. .: 69.60.50   . 065. 13 . 511 .3.  &0   6.4: .6 7.6::6659::   '596331/33160..965.1 /33 69 9:63./33 563..65:..45145../33 56.:.65 . .1 56.45145. 065950 0644..6:3 5:9.:9:1.936150 :33.:   ': 69.069..5.1/.: .. .65 4769..: 09.636625.0.3. /33 . :.     '635.: 6.569 6 .95 .1765.51:/51565..51  /0.79:15609:6/6.:.:4. .::  .65: ... .16   5 6..3361  #37751:::5 #9....9: .56 &09.6 .:: .765. 65 .794.. .3:  9966 50.3 : 9.510. #9:15.50  .6.945 5.6.4 /...0  .1 3.51  31  ' 695.0. .5.5 7..51 0.559 :..695.. &7.: .*     5:631.6/3.:.9:9. 695.: ': 55.::9.7796. 4.3 ..: 7. 17.616. .9./3:1 .. 0.9.1 :5065694.51 517515.9 ...63365065:19.:.0.51 .65   .::1 / 659::   ' 9:70.5:..51.::1 / 659::  . 65:.#9:15... 479.007..296.45. .1 5 .3 17.3:6/5..56535./3:1936150  9966 50.:3.:06503:765.91 9:70. 65:. 46:.96....6:6%79:5....0.3.9..646996 69... 160.51 69.45.069.: 6 7/30 7630 69 9.:0. .5 5963310.4469 6511.33 31  6    3.5.3356.1 '596331/33939:.92    &         1     (*   '6 6993 .66.1.:':631/7/30 /0.55.5 .&09..: 989: ..  .9 4.../1:.6 / 906911 .::9.79616..45.55.3 .91. :762 6 .695. .:/5103. &% .1&.961..1. 17. 9:70.6. 765 . .. 5.96&. 1 . ./:50 6 150 .8:. :6345 .60.:9:..99:  65 .::1 659::  .5. &%    ( 516.0. 4.:64946.. .51.. 3:3.6 4.3: ..3/9.9 9891 / .0.../96.00691106503:0..9:.9: .69..516.65:     : .05 6 69 0.6 971. .9.95   .9: 5 .0891.3:.&5...0:..915 .50 / .5 .0.765.: #65: ..9:: .9.6 ..65:6.: 69.:..4 .65.:.99891.: 06503: .7796 .5. 065. :6..5.3/ .911.1 &.0.:631/794.911 .9. /33 56. ..6065694. 069. .:6.:: .: ..:: .6 1...65796793..6 068.6 .: . .6. . 9:70.:  0.61.: 7/30 4469.9.9.6 . . 7..33.: 56 .3: 6 .0.:  .69545. 3.: 6.#9:15. .95 56     : . . :..:65.6 .9 . .. 10.33 /33: . ..515..98:..   ( 516.9165.:65.:.  &0 (*    '.:65.99891.3 31   '.*     ./33 9.156.665.39.65 ..91.6/5.515.9:.58:.695.515.4..:. #9:15..9.5. 065:..5.65....1769 .769 .:6/0.34..   31  ' ..#9:15.6.:. 5 . 6 .65:65..33:.0./3369.:76597.:.67696.1..6 1 &0 (* ..:90: '.65./:63.65: .44/9:79:5.65:.61 1  9966 50.:.03.::5.6769:56.659.       .515.6. 1  &0 (*    '..:./33. 0. &%    ( 516.51   '#9:15.9.6...9 796:65: : .*     ..3:67961:34.65 . 7.5 90: 6 . 656.4 69 .65 51: ..65  95 69 .7:5.6.5 56.94.51 . 065.. 4.779679.90:6.65..5.51.919: /5..4:..69096::56.5 69 5... 0..551:75:.4:5. 465 ... '"&:06990.4.. 4.556.615715.336956.65 7961: .:.9.6..9.6 .01..: 69.059 6 69545. 5..69 :1:32: 5.9.9.9 /33:  .6956.:70035: 796:65: 697.969 0647331.03../33.6. 653 . .6 1:.469.9.:.779679..4..7796.: .7796.064:.4: 4./335.6 .6....5.9:   ' #9:15.::379611./3 . 7..0..55769.33  69:0.: ..6.9.69.9.9 /33 5  ..6769 .6. .6 95 .4  565 9365 &% .:65.51.:9    ' 65:.. / .9 .. 145:./3316:56.:5.6769..6.61   ' 769 .9.:51:9...77965.33.4 ..65.5.:59...5..0..:.:9.779679.32..6.7.5.6569954.63..5.65:..5 6/0./3.9/33 50315./37.. 65 6 465  56.  5 . 5.03.6.779679.5 . /33  .5 ..5:.45.65 .65.      1    103..3: .4 0 5 .65 /33 6/6:3 4.0   &       .67696.5 .6 / 7.551:/3:4 6 465 110.50.51:9. .51 79.4 .50(..   31  ' .1.0.4964.9 3:3.1 7976:  '5.6.9.6 ..4 6 ....775: .  793  5.91 ...9 0650113 195./37. 0.5: ..6 .3 796:65 6 3.9: ..#9:15.: &794 69.  0 ...9.9 6 :. : ./3399:. :64 59.779679..5.:6 .1.7.0 .. 5.:3 : .99*     :..:.5 ..51 796:65 5 /1.1:.65 /33   565   9365   &%   .1 &.: 6 565   &09.94: .796:65593. :700 .779679. . .&0.0..69   31515635.1. 4.65:690.:365/0645..3.56#9:15..65:6.9.796:65:6.:.0..3.  793     5 .#9:15.6.10:656.51 0.50 (...5..:...69.9.6.6103.99*     .6.:.3. 9:/69:699.:16999:.  .6..510.#9:15. .:69.6 769   565   9365   &%   .3..5.94660 !.9 4..69145.90: 6 .96.:. : 30....0. 6: 6 %79:5.79603.: 30.: : .: 31 . .6..99*      1:8.65 69 ....9.: 9:10..65:03:33611.6:6%79:5..69:63. 69 659::4.169:10.5 /69 .65 6 7.9:10.75151:8.6 9:63 . 0..5:." 566:.511..1.41/0.3. ..." 36:..65691:8.: 03.65 .4:..3 %'   31 (#*. "    ' 3.30.650.. 30.65 0...6 95 69 44/9 6 ..9:10. 30..65: .:56.69.659696:3065.511.50 (..75151:8...30.:6.65:56:.65 ..969  :6311:4::.65 0. 1:8.: /69 . 30. .: 065:195..:.65.: ..65 .7.69.65: .51 ..650.  793     5 .30.. 6: 6 %79:5.6569.659: 8.30.51:. 0.30..31 . 8:..51 ... .3 '9/5. .30.... .3 %'   5 ..9 .3'9/5. 410.: 5 . 65:.59:631 8:...511..0.90.65 7...65  9.95: . 30..4.5 30. :63 1 6 .6: 6 %79:5.036. 5 .5. 44/9 6 .9   #.51..9:.655. 555 0.:../6.33 065.33:.65/..::.76:..: 93." 0644.6569.: .530.33 / .6...659.5550..51 .1:  '&5..55 .50.64. 54/9 6 6.655 190.5 ...511. &0.. 4/9:   519 .30.1:.65 6 9:10.65 .5 .. .6:6%79:5..9 9:70.65 16:56.... &0651 659::65.6 .2.3'9/5.3065::./:61:09.65: 6 .69. :..33.: .51 8.3 :.6 .5579603.30:.65906555.1:96:9969.511.659.... 1:....519.. ::75:65 6 : 79603. :.4   "/. :.036.::/0.530....:..&5.69..65.....65 69 .51 8. 79603.511..3.556./579603. &5.9/06444/9:6.516. 44/96...6/.65:.6 .65:45:.95: .3  # 3 506550.95   ':  7..6:6%79:5.93 ..9.65  9.6569.6&0.&0  6 %    .33 065.65 3.3 . 69 ..65: 6 0.69.651 ..::4: 9:10.51 7.   .65. .6:6%79:5.659  .659: 065.41......4..9 .511.1..4.:56.9 .6.: 93.5.9/5.:  0.6600.30.336: ::75:65 6 79603..50: 45. ..65 .5. 6:6535.6 . 30.. 06510.25:6. 30./3:1 .: /5 :./:..656.30..:56.511.1 796:65  .5550.: 69 ...65 519 0904:.9.3 03. 65 ." :47691/&0.630.6&0.65 6 8.5596.65 51: 56 /.6 .68.: 5 .0.. 30.6 7.30. .:  856   "    &%  .5..9.65:60.331:.4645.65 593..30.6065.511.. 8:..390:964.:: 5 3.5.65:...:: 765 .30.511018:.65..69.65:93.16 ." :.  /0.69.65 6%  .64..9 . ..: ..659 56.65 69.::67.9.65 .33 065.31.65  %64...5 .65:..511..65 6 9:10. .5  *      : .: 9.6 .65: 6 44/9: 6 659:: /5: 653 . :63 1 6 .9 .:30.30. .. 8:.51 8.65:  9.30. . ...95: . 44/9 6 . : 6/6: .   5.: 765.3: :776:1 .65 69 ...: /064 . /5 . %' . 6: 6 %79:5. 6: 6 %79:5. . 0.. &7...::47.65   #. 6: 6 %79:5.:  . 30. 30..50(.:.75..: 56 9:10.6:..659:8..: 93.3'9/5..65: :0.. 44/9 6 .%': 9:10...    &0     65:.69.6 . .906:   "    &%   . 51 03: 9:10..3 '9/5..50($:4/5*     #. :.51:.0.. %'   ' 9.:93." :10:65.79603.31 69 5..41 0.  .4.95: ..::41 60 ..33 6 .:. .95:  ..6 7.30.65 6 ..51 ..33 .511..65 .4.03 &0.39.6 .3179603.9. 6: 6 %79:5.4.65: 533 .. "  : :. 93 / .511.6 .:.65065.:. .65.9 :0 533.65 69 .      5 .65 6 60 / .  .... 065.3.:.65:61.::41 60..::47. 555 0.36..5  31  3 .: 93.  &7.: 935 : :3.6599.6:6%79:5.: 769 .65:51: . 769 .79603.660.656.: :63 ..1 ..5:9. %': 65 9:10.:.31.9.4.. .9:.34..95.659   31  .65/ . 0.0.51..33 065.656 .:  " :9:10.51.:0659::4.65 9.9 ..3 065.:. .511.:. %' .51 ..5...5550.6. 6 . ::/:85.79603..51 8.511..1 .4.61:065.3 /651.51 .25:6.. ..511.50 (. 6 0..51 50. 30.0.: 79603.518.9: 56 79603.690:.65.65  9..6 8.6998945.6 ../3  5 1950 . 16: 56.51 .31. 79603.656930.51 .: . 5: .9:. 49 .9: :7.: : ..65:.4.5  *     33 .119::1 .:  ':  650 ..65:  9996   "  &% 3  5..5 30. 9 ..: %' 5199.6 9. 30..:65 69 .51 61  .6 103. "  .. 09.3:.0 .:1 .:659::4. 6: 6 %79:5..656.. .69.. 79603.6 103./5..75.%'.::4:9:10.6:.511. 9:10..6:30..76:.65653.. ..:9:. 0.::47.511. 9065.25 : 6..65/5: ': ..65: 6 44/9: 6 .::: /:.51 .511.%':659:10.65 .5 .::41:76:.    5 .. "  ..: /5 79603.006915 . 555 0.76963365..4  . 179 ..: : :.30.65 69 .69.590:59:10....41 559 .41  .:.65 6 . 44/9 6 .65. 9::1.:93... 69 6: 6 %79:5.:: 5%6/3:  %' .65..9 .90:6 .3..3.33 ..119065 6069: .5:..'9/5.:..51.3 /. 03..69.5.7.9950  .30.61: 1730.:.6535.3 964 .:6.51 .95:..5.69..145..:30.95:..:73.:6316.33065.3.3'9/5.39507.9.%'65.65  9.5  31'65:.15. .69..0.51.30.  69 ..:.::6.93  56.5:9:..:769610.9/65110.0 9:70.941 . 1 9.65..33065.1. &5.30.65 /.  9996 "  &% 3  5.16:6 69 /.:.65 9. &5.5 065:.5 ..3 .33...90:6.91 .90..:6316.6/ .. 10:65 6 .769.4.3..6.69.:69..44/9:   '69.518.3'9/5..65.:9:10.3 /61:  .5..65:644/9:6.3031:.::.51..%'..6.99:70.654..3'9/5.51.9. 7673: 4.0.69.65. :.659.:.6.3. 6 796015: .69.25/..: 30.:93./91/.6 .695..77.4   '69.6909. '7699.:.:69..50($:4/5*     : .:69..35..30..: 6 9:10.6: 6%79:5.5.65:.518.30..30.: .5.:6365.6315 .6.55.4.6:6%79:5.5...30..3:.65:.3 '9/5...69..65:6.  03..65..9:656:0..:5 69996 60.'9/5.5:.9..9.99:10.9.:...:065:.65.3 065:./:     ...36 1 7960:: 6 3.65697.9 54./:61:09.65 6 .6915..'9/5.65.65 765..945.331.9.9..15.6/.79.6950::6 ..65 69.:/.6.5 :09.3:10:65699:63.1. 9 03...:95191.  69 765 .../:61:09.0.9 #3  765.:.94169:0.51479615.:769.36.9/.:9:10.9.9. 1465:.1 9969  4.  5. 6 .9.0.0 5 . 0.51 ..5.3...    5.#9:15.1 .5.33: 69941. /9.5 5.9 ..6694 .9 .3../.9 5.: ..5 7901 9969: 6 517515.0..65  /. 7936: .9 ..0...9..66296.0.6951/. ..969:5.94:.95.6.6591156.50: 6 .65 /69  ..38:.95665 6 .5 #.65:..4:.. 16: 56.3.65:56. 69545.65. 6.::.5/9.51.:6.:.39966.:.6 . .9.9:5.   ' .:345.5.65:03. .7.5/79:: ...7.9..1.5.69:631/065:1919:51..   5. 9:5. .3 . 15..5..9    ../53..6 9:5 .6 510.5.7.3 6 1 7960::6906990..5 %' &% 0  (.9:5.::6.0.' 69.9/6518//3./ ..0..33./:61:09..0.5. / 06731 / .0.5/4731 :365.. 6 06990.563::.0.9  5#9:15.693..0.:. .*    '0.0.398945.45.5.65:69.65:.31. .:#9:15.:.    16949#9:15.30.%7/30  31%:5.65.5694..:30.5/69.. 4:.5 .369.5694.: 6 9358:45. 064: 5 653 5 .:. 4769../16/.9:5..51.9 69/.510904:.:.5.4.393.:.5   5 .:#9:15.67969 065...7.:065941/ : 3..556..0256311 .     .7.9966 65:85.9.5.9451964:..6.0.: ..::   :5.6599:51.:.:.6599:51.96956. 631....6/1.5.6590...     5:4 631.55 : 5. 6..3.7.969.95.6..0.5164::65:/69 195.0..56:..: 065..51 76:.3.3 :..5 ...3.25 6 9:76515..656. 79:: 93.45.3150/.5065..3 .9.1 ... 267..256.5.0 .5.69./3...   47.3.:3.9:9.3.51.:#9:15..6/5./6..:.65.3.:3.5156919 .3/.:.#.0.6..#.65 1156..%7/30...357960::6695.52516 5...79:150 69.6.6.9:76515.01.:1.:65..:..:3.. 0.:57.79:15.::4..54..9: .:7.5 ..9.1:.9   #.9:   79::1:9.659: 9950:.  .51 :631..1.767369..   :9.7673.65.3..:9995 . 0631 56.7. .69 :5.6 65 4 5 ..:.67769. 064 ..335 . 5.79:150 '79::93.:92 964 .65.1.65 .3 :79.::665.6.1 65 .6   %  !6:  .90.5. 6 906503.79:150. 7.6.3356.:.4 ..6:9.. :.5:  :.90. 5.77.51 :631.:.659:.:#9:15. / .9 0. .1:.310. 6 ..   9.:.335...606.53  .9.6./3.3 :79.7.16/.6:9. 4...9.::91.5.. 065:. 6 906503..65 6 ..51    0.4 :90 6 69 065.0.6..933  : 79:150: 565 . 79646.65 ...960075.511156.67769.331 65 : :7769. 50(#56*     :0::693.5196::31 .: 9:10.  ..445.06.3:.445.   31  ' 160.65 491 .9:76515.956 0. 0.:3.:669/: .'.   5 .4965 69/:  69569.5 .5 :1 7.: .0  606'.659   .06 .5:0. 0.90  5.69650. 9 9 :.6 796/. 69569.3313 065:795 .51  %  '96/91   6 #630 .3  69 1.. 6 .: 69 .0.10..51  6 .  :7.53.5.59.   5 9. &09.: 6 .4.65 . 9. .065:65 31  '79507365653.69.6 1769.915 .60 .  9:70. 4 . #3775 :3.6 .6 5.... .5 . &90 6 .25.4.: 69.3 6 .51:     ./3.:56.16:56.5.9.  065.5.5 ....5:350 "5 . .55  519 .3.1  1:.:/4.32 1.9.69.5:.  519 069 6 : 60  16 .3.79:65591/.4. 065:.59.9: ':16:56..5.9 .965 9..65 631 4.. 69569..  0631.9../96./3519. 4./.. 0.90..51   &0 .:693:3...9.: 6 79:65 .  4.5:  :473  .99506069.3 .  633 99645.51 6 79679.9  ..51 59:.. : 6 .0.:56941 /. 0.  33  54711 ..3 .4:.656 .5 . . : '10.15.506:60.51 .  103.9 : 796793 79:5.1 .0.6 .99..59. 44/9:6. 1: 6 .9 .0.0.065:850:6.3 .5 ..: .5133 5.31. 6 .6 . 069.56.: .5 .600..:69.9.3  32 . 4.79694. .:33769.9 4./79:65.33430.4.51 . 69569.15031.5.:65 :.:3. .0.. .93. 59.9  5 : 79:65.:76::/35:.5173.579:656. :. /5 5..3 69 79679.69.5161.. ':941:. ./:0.333.5 56.3.11  5 / .6979:65 694/3696 . 9..5 0 .:86.. :.: .9 065. 10.9 0. 6 .69:79679. 69569.   ' .:5.:/51791 :3/9.::91.0. 16 : 430. 3   .59. 1 7.9.9:964.#3775644::6569.50 6 : 60.79:65.5 ./3../3...065. 44/96.679694.5 469 .   !. / 79:65.: 795073 6 5653.: 0 9:3. 0 0. 4.93.4:.601. 79694.36/.6 1:.5.5119.03.650. 964 .3 1.  ..77.33:1. 56.65.. 4.33 5 1.5:03.#3775::4/3 #/30763069/1:. .5 .769.1:0::659.9.3.93.9 .:003..5 .3 "5. 03.5..6.51.33593..69569./3.4.. 0.9 16: .6.::060..669..0. :   .6 .3.65 4. 8:.536. 8:.31. .51.96956.69.026. 51.0.1..: 1. 76:.69.9455.90:11:09.796..65   5 :0 0. 10..     101 9653   : :..0..65.69./3..1.5:.59... ...65 5 1.. 79611.69.: . 56.3:65:. .:319: ...0.  /.6.4.3.691.1.1 79611 .3.51145.69.: 65 69 0 ...9.:65.196479:65.6.03.33 796... .6551...0.0.33. 65:.6 796.:16:769.: 69569..1..::673.0.3 19 69 .0..0./35.9.33:11:09.6 45  9.0.:.:56.6535.:  56...9455 ./:...33/796...5...6956.65 6 : .556.:56..945.9691: :5.9: 3.6.6 :0 45 0631 56..0.0.::673..3 .69./3 8.0.0.65:.:5.9455.6.69.319/.65 6 : ..31 69 .  &0.659:...655..5.669545..906: .5..: 6:.  65:..6 : :700 6919: 195:.3 0.3 ..:   #3      (*  7.65 0.6:31..065:850:6:..: 6 .445.65 1:9:70.3 .65 0..9 03.: 165 / 4 69 / 6.03:.#6336..773 .0.55.65:.  5.4/.51.#9:15..9  56 :.51 65 6 .0965:4.59   '9. 55191 69 .45145. . /.91   /96.1445.906:.5150.51656965 '69.065:65519:0691.3:6 964 0945.36..906: .: 069.6 .0..46:... 79:65 6 65 6 ..3: 6.9: 79:. 51 33 ' 5: 5 ' .65   5 .906: 63. 065. 5 #9:15.. 195 : .5#. ..::0 4:.15::51964   '"776:.:. 0.3/.65   5 : :0651 05.:: .3 . 59.5 65 /.. :.   9 :.51 .0.5169.765.5 .. .: .. 631 069 5 .. .5:969.51:/:..60.036..50 3/9./. 60. '50.6: 9:95964& 69.#9:15. 65 . :067 6 60.45. .964 :.4.9.506507..79.17656919:6.1.6.. " #93 :  5: '6 0.1.151 .51  . #9:15.4. #9:15. 445.9:.#9:15.: :067 :6 .91.33. . . 56&09.  65:.606007:. 4.9: 6:...5.6#9:15.: 95 79611 :.9.4: /. #.       0.69545..3 445.9 0.9.50 6 .535 3.653..5 :700 796:65 65 0.56.65396403 03.51 6.31   '  65:.6   %  !6:  ..:.. .91 .49:1156.95. 4610.445...1 . 796:65 6 . :75./:63.006...65 765 4769.:56.6 16 ...9..95 .3:66.5. 490.:.. 9:.9 5..93..915.1 964 60 / .65 .06507.&. 504/5.5:..979:65: /.511.56.45145.445..6%6436 6. 13.60. :.6503156.5/:..6 5 9.1 .51 69.   :9.7/3060:.65: 69 .59   ' 445.9:.1  ' #9:15.51 ...65      :.:09.5.:9..6  :6.3. :..4.6 :.:69 :..:3   "9  65:.4:   &0651   53.951065:3 6949.65..3:6..65:.6.. .6.:.7/30.5...9 / 53..   &506 #96::69.:   51 .:6..1:.: .. 11 56.:1 .511:69199:3...655 5. 0 0.: 065:..65 9.: /695   5   .#9:15.3 1.65 '46. . ..3:69 79.5.659:3. . 9991 .65:0.0.0. .: 5631 0.0. .97...511.65 ...662 0./:63..1.31  #9:15.445..:1 ..795073.5 4/96#. ... .33 3 69 60.:31/.9.4 . 445.:65:..95.3 .. 156501. #673#69963..65.. 445.0.:..065:850:.9504/50445.96 5. 5 9.511. #3775: ./5 /9.250. 6.:1:8:..4511 .9:519.33 .53.1 / .5.1.33 / 445 964 :..43.. 793  '6 86.5..:6.9. 9.:.367.9:..:.:.15 .47.045.9...1..0.90  5.4:./796:0.0650.9.5:.15...4  3190.045.796015: '47.945.1/   .169..50(#56*     .33/796:0.1.045..556.796015:./ 0650.9.:.:65.47...  .33065:195..659:.56949#9:15.:56.10945../69. 51.:369./3697. 796:0.3373.69: .797.6 14.6064451 . 31 .&5.: .. :50..:6319:.69..565./.::1&5.1 6317..01.51....3/.045.:5..65!6 %0655..5:.%:63. 79:150   511  65 /9.6 : 36:: 6 .69.5./47.:56...045.51 / . 9.1 '73.5/796:0.5.:796:0.5.47.47.. .9  .50650. 5. 6 .9.65.:"06 &50..:/4::65.:606 .1/690.9:.65 &0..5.7..:5650.045..326..659.. 47.796015:.6:0945.0945.#9:15.0.045./5:/0..195.9.6.:. 56.39.79:150 7. . 4   ': : 5 .6..9...556.15. 504/5.1.5:..0.14.703../064466.."4/1:4.1/.3796:0..65./0650.9.3796:0.659 .5#9:15.3644::654.2.7..936:.65:...6069..6  % !6: .6/0.9 445964:. :9.656.51.1.9504/50..51030.....565.7960::.659:.9:5.79616.65 '6/:9 .50.1/31./69.:56.51.65 :58..::4.:/5..547..03.5:.045. 1.51.:.045.47.0651..6.045..00691 . #9:15.90904:.3.5/.1.9.5..59/. 796015:  :.: 5.796015:.69964/5/96./651 65:195..6 47.51..1. 69 935 5 5 % &.65/69.9556 941 .796790945.: . .5065.40.65.. .656&0. 69:. 0. 69 79:15.6&0.65  31  &0..:.65  6 %   !6   7961: ..50(#56*     :&0.  .: . 6.9   5.656.65:.9 4.65  9.5. 6.65 6%  !6 593.9.036../:5. .69:. :5.517.79:15..9. 511." .::66.51 0.5..65 6 555 0.0..9.5179603.4.0.3:...:.796:656&0. .979:5.6565550.6 6919..4..65 6%  !6 47695.79603.:....4:.:      #.:5:6..: 69 79:15.511..65903.:. . 65:. 63365 796:65: 6 7.3 /0.: . 6.  : 5065:.9..03  6 . 63. 1..: . .: .: 69 79:15.5.65  6 9. .9..6 659:: .:: .65.7   &0.51 79603..51 0.. 555 0.4 .79:15.511.6 0.65    0 : . ...:69.511.79:150 .: 5&0.4.51.4.65 6%  !6 :..6565550.0.: 79603.5550..935031:.66:75..656..79.50::.511.9 .      511 . 79:15.79603. 510.:69.65   9.6 659:: / .511..33 1:9.77.65 .911 ..4. .6 &0.1 &0.76:.69.9 .6 / 975. 5 ... ..9: .79:150    &0.6.5.5550. 65:..: :.65.03  6 .65   6 %   !6   ..679603. 65:.65 653 5:6..65:679:15.    5.036.7 &0.11.9.03..&0..7.6569#9:15.65 ..9..69...510.65:.65 6.:.6507961:.3.56.::.65  9..95:6930.79:15.9.. " ..510.65.33/365:.6659::    659:::63156.065:.5./6.6 .65096.::9:..7696659::.:6979:15.60.6.769..1/. #9:15..33/09...:.6:97.5.3361.065..9160.69.::. .6. 51.51.51:631/9.65.:.45.65 6 .:/:5.: 69 79:15.51 50  ..769.33.9. .6'"9:.:.6. .79:15.4.1. 0.176:.3.5.: .51 0.65:.511. 6.:7..51 ..4.50. 79603.679603.6.65: '796:65:6.559:69.::5 6 . 555 0.. .50(:.55.35.5.95.2..9.69.654:..51:6659::  .94.3 "    % !6  3  5.79:15. .5*     :0::.9.511010.518.95: ...&79469.30.9:10.656.9.::5635.#9:15.6.510.65 9.30..65:6. 65   7.35 .796:65:69.662 065.   31#..65 6 . 9:10.65 5 .9.513562.03 &0.50 6 &# !6  .  65:. "  5 .659:'0:65 .. .9.3 .7   6 .::...#9:15. 9. 76:. ..1/69. 631 0.51 5 95 .. 065.6 5:.65: ... 5 9950 .35:. ..       "915.2 65 .1 .6 . 190. &794 69.0.:.9 . 7.9 :. 3.99.511.6 .5 30.:. 65 6/0.6 1:361 . 86 . 6 .96  30. 964 60      ..:.: 065::.30.5.65 065.65 :05. 5      . 941:  631 .6 1:. 69 . 555 0.65 796.6 0  .50.9 . 3 79643..95: .: 93.&79469..65  9. 79. #9:15... %3: 6 .3 '9/5. .33 :7.65: 6 ..2 6 .1/.330.3 6 .:794:     ' 93: 0.5065793 631:7769. 9:10. 30. #9:15. 69 0.:.51%3 6 .51 8.30..6 .9/5.690.:6365 %3   %3   .5 .3 69 065.5/.79:.69.65 6 .  6 0.511.:69#9:15. #3775:  ..690. 6 .#9:15.51 56. 5 ..91: 5. .5.  . . 7/30 60   5 :0 065.473.#9:15.33151.33 631: 69 90:: . 76:.:.:/5.65 .65 065. 86.99. 0.6  69 53.5 653 065..0. 79:65 6 :97:  5..6796015:59. 30.5:. 65 :05.579:776::..30.. 9:.76:.:.65796.9.54/966.901.910.91:.511.6631.96   5 %3   653 .:065169.:0631 3. ':93.530. 9.9...65:05.&79469.96    . 8.65 7..655 . /69 .79:150690.511.9.30.:. 8:. 5031 0.:: 190.0.. 151/&0.  65:.69.656.30.6065031.65:6.65  631 56.9:10...3 /96..7   6 . .9  .7765.6/1:4::169 .913 .515.  65:.3.::.::6.  .644::65657765.: .35...33.03  6 .5 .*     6 .::.: ::1 / .36 &:.51  73.6  :65 . 519 &0.:/5.6 . ::.45.7765.45.3.1 / .65:   ' 9..51  65:...1: .50.:/0.6 :.0...5  76:.:9.:4.7765.51 /.9 .7765.5.:.45..: 7901 ..:6556..015. .6..7765.  .0.45.3398906594.: .45.65 .65 / ..379.90    5.&0..945.3    "    %  !6    .6 .6  :6550.0: "5.51 . '0:65  ..0644::65/0.30./: 6 :0 769   ': .519315..... 769 6 06594..6.6 . $5.03 6.:065::.5.7751 519 .5. 769 . 09. #9:15...3.601765...03:.50(. 644::65 65 7765.. 69545. .: .51:43.90.57696.:.644::65657765.:. .: 6 09.3 . 3:3..6533 98906594..6.51 . 4113 9651 /.#9:15.1796:65 &0..45.  65:.65 6 . . #9:15.65: 5 .9.05 ...:9891..65:. ..65 9895 06594.9:. .690.945./3 ./9.519.03  6 . 644::65 65 7765...9:70.365..57696.65.:   31'.45.656.6.9.150..65/..:6&.336 6.49: 6 ..1.:519.346:.5 .45. ':69.1 796:655. 609: .7765.6 4. 479.5 .513:3.65 9..6569.5.65:.:..4.690.:/:85.5 002 / .94.6 / .65:.  65:.45.50: /.473.65/.15.65 141 .: 5 ./3: .5669:.65  3.  65:.#9:15. 6 9:10.#9:15.79:150/69.51 % !6 631.0.65  3165694.1765.644::65657765. 6:9.5.3379:15.33563659989:006594.65   .50:669545.931    0069153  % !6 .30.656:.65:.9.50(#9:4.45.: . 51 .0.:6&.:. 065. 65:.:./:63. 0.065:5.7765.6:6:.: ..979.6.5.65   9.. 7950:61..1/.9 .5..45.4 6 002: .6 :/0.2 . ..::65.644::65657765.605.7765.6:..*     &.45. &%     5.9 76:.::6. .7765.. &. #9:15... 9609:6.. 0. : 33. :.. 17.6.169:  6.65:.4.65  &0651 .::.7765.526063653695.6:.969967:6609: 6.5 . / 3.  '91 .: .45.9.6.6/.:  .31965 .52 6: ..7765.69545.65 9../.#9:15..:65:.1: 6 .  69.941690:964.3 519&0.956..#9:15. .9609:6: ..7. 659:: 4.9:79611 69/3.65 .7765.365   ..45. 5 .691/3.:.  9:.  .45..51 065:3: 609:6.9.:.9: .45.7765.51.3644::65  .7765.03 6.9:.9 7/30 45:.9.4/.30.6:64.3: 65:.65..1545.69545. #9:15..  609: 369 5 9.516.9.336.1/... 6 .50(#9:4.7765. 06594.*      ... 644::65 65 7765.45.65 / . 9:.36   &:..:...   &%        5.5.: /3655 .6.:  .31 . 653 79:15.3 . 967 989 . 9895:006594..90.45.6959.79::796:656. 644::65 65 7765...: :.45.7765.90.6760.9:1.:. :.#3775!.: 69 06594.190..  519%7/300.45.0.3 .6959.45.65 / .6:/4.6.36.9.7.3:6. .7765...7765.1/.5.:.6 ..51 6.#9:15.3..3 7.33/.3#630 #!# :.515 .65 065. : .65 &631: 065. . : 33.7765. 644::65 65 7765.651:7. &:.79:.65/731  31  .65.#!#/..6.33 989 06594.51.7765.:   %:76515.9 .33 56 3659 98906594.6959.45. 65. 0.:6 7630 609: 6: 9.65 / . 644::65 65 7765.:   : 31 5 .96331 / ... 964 .941690:.65. #3775 !.*     33. 9891/..45.50.*     :0::..9.:./3: .51 0 :795.30..69: .52: 5 .   '6:61:. #3775:  ' 65:.. .50(#9:4. 06594..956.7765.91 .65:.:9895 06594..: /3655 .3 ..969. ..45...7765.5198906594.45.515.45.65   65:85..969 .941690:6. 941 690:6 .55.36   &:.145:.5:.50 6 &0.51.6..51 03.9 .7.3.6 / . #3775:  ...3 5 :067 ...656.:5..96.53398906594.5. &5:65 659::0.6 .65/.65:.65...9.:09.511:.036.: 69.651 5.  ' &.04769.33 / 5.  65:.956. 5.76305..&..5 65 7630 690  0 :.6360..6.51. :.:.5. 9:.7765.3:56.:6'.3:6:.45.65:.1..769606594. 1:.   "5.. 93.656.25.69.. 9:. 0. .:50.3..609:6.50(#9:4.996:.6. :43..:  .3 7630 0644::65 '.: '.65..7630690:195. #3775: :.644::65 657765.. 644::65657765./3 '#3775!.941690: 659::5.3#630:.3 #630   ':  190.9:. 43.:.969050::. .6.6.. 9:..65.656.9.69679:15.7630690964.65   9.65:. 644::65 65 7765.    '9519 .9:. &%     5..45.45.964./3.6/065941  .:69:76515..0..60.. 0.5.7765...65:.52: 8.65  563:: :.65/...45.6 .5 5 0.036.6: 5 .:67/3060.7765.49.65:.5 ..7765.6.3 5065:.45.:4. 653 79:15.31 .69  51 56...:.:. 967 549.65691..#!#1656.969545.30...9&0..644::65657765.65  989 .516%7/300.7765.941 690 0:.45.5.50.3327.0. / 065941 / ..9.45. :5.03  6 ..33 /79611/3.33519.99:10.1 519 .33 :.5:.:6..1%7/300.506&0.0.:.#3775!.6569.3 ./06990.51 4.65:5.5 .6.5.51 065.644::65657765.:  31':065./79611/3.65   ' 941 690: 6 .9...6 .33519643.   &%        5.556.45.9 56.3#630::7..036. 6 063653 69 5.9.51 &0.:69.:5.9 .5.6569.9 ..51517515..65.33 / 06476:1 6 .65  519 &0..7.5.606594.51:90 .:66.65.9.36 &:. 51 0..7765.7765.19..0.169545.31 5.: 8.65.2 .::.7765.45.. : .77961 / ..659::     .9 '65:.6.9.: 69 5..6535. 5.:9   31  5 .0.: 794. :..:.1 5.9 / 4.:0.3.94.0.644::65657765.. .45.:695.:16:56.::/0.0.94.51 ..973..3 .4769.:6336:  ' #9:15.9. .45.5/.5 / .794.2:. #9:15.606594.9635..55. .: .9. 769 .65:.45.657961: .7765.. 644::65 65 7765...2: 0.5.45.9.25 ..3 /. 794.6659:: ':06517. 650 .65 9..50.3 1:. 659::  .45..:34.5 56 3659 / ..15.969 0.31:.76&0. 441.5.3 . .33 .3. 90::6 .7765. 0. 5.45.94 .169545.45.7765. .9.55.55.6 60 '.5.6 4.7765.5 699621/.65/.33/0.644::65657765.: 195 .45.9.96906473:69 /.7796...19.#9:15. /0.036.  .51 .51 .9 . . .469905.45.45.4769. 3.0.  .9 #9.15.:  0 : 695.7765.9 .5.9 6 .6. 441..1:097.7765. .9.3 4.45.:5.6 4. 644::65 65 7765.: 165/..2: 0.:0.:./:5069.7765.0. 3. 5 5.6 .. 6 .7765...7765. 650 .   5 ..0891 ..9 5&449: ".45.7765.3 06:5 4.1 / .4769.6 4.2 .:5.94 . .1 5.6 #3775 3.950.#9:15.: 69. : 5. ..1 /. 769: 79.94  .3   ' .55. 644::65 65 7765.5 .:6 .79.45.3:01:.94 ..6/.6 :0 60 : 06473.94:.0..: 69 5.:590::  5.1/..4 69 69 .:65.7765.6949 .: .5 .45..45.9:76515.:  ..651.6.9.:.7765.1 ..6.69.55 .45.. 1:097.1 65 .19.:.45..45.7765..45.': ../3.5.6.3. .7765.5 ..45..65  . 0904:. 659::    #. : 6 069: 1:.5..5 .15.510.3 .94 .50..9:76515.15. ..02: ..5 .5.7765..955.1 5..1 5...45.....#9:15.3..935.515.94 .9. ....45...:16:56. 0.65 #9.45.5 .4   : .5.659 0..: 69. 7.0.:.9..6 73...555:615 . 6 .::.5. 6.15.: . 5.169545.0..3/..5.511:9. 5. : :/0. :5.3 7961 .65 9.65 : ..55.3 .65476::560651.:  .9950.7796..  .1 .659::/4::65.911:.:.3"0 !6  #. 101165"0. / .9: 6 .5169.94.: 0.4 ' ...0.: 796:65.5.9 69.15.5. 769 6 .9 .6 1:0.45. 659::   .93. 9895 06594..:56..94 . .65  . 6 .9.5.50.9  ..945.45.94.1 .6. 9:76515.50 69.:93.7: . 5.5699621..9.9: .6.9.7765.   ' ..: 6 79.3 .7765.7765.94./69...: .64 %.4769. .5 & #9:15.69.7765.#9:15.94:65.6.0. :0 : 56.::    69 .0.6  . ..45. 769 .4..5609.7765.1579:..7765.#9:15..45.7765.: 8.45.7765.3 .313.94.9 6 .45.1 5.3695..33 . . .1..91 6 %5.33 :. .2: 0.. 5 519:.7765.1.9..5. : 56.3 .9 6 .60195.5.0. 0651.0.4769.64.5:.7765. 3.1 931 .7765.4. 973.1 5.0.: 4.9504/5.: ::1 .65 .7796.94.7765....94 .: . : .45. 0.:.96535. 0.511 5 . 9..9  %:1 69.6156.45.: 6336:   1:.94 .69962.:::1    '65:.45.4 /5   50  7..65 569 34.45.5 . : 516/..55.: 69 5. 0.: ...9.5 .5 ..7765.45.9. : 0.659 .945.65  0 7961: .51:..1 5.53 69 .5 .19.65 ./.9 ..55 .9.:56.   &09.1 5.9.511 ..5645.659:: /.3 1:.65.:94.7765.7765. #9:15..6.:4... 3..45.94./.6 06594.. 5./3 964.6 56.45.5 56 3659 / .94 .659 :631 7950 103..: 07.4  3 .515.4..39.96 .4..5. .9794.94.:.:90651.7  &0.644::65657765..::1.: 3..33 .55.94 .15.45.4769.94..156. 644::65 65 7765.6/9  31. 4.:.1...3 . 5 #. 65:.6.:. 36:: 6 . ..691.5.5 600.633.7796109.6565.     964 .5/. 90: 6 :0 79:15..41.94: 56.5.253519:.3&90644::65%65.00631/.55 519 #3775 9:79150   ' 69. 4.:::.: .55950..... : .941.  5 9995 .945.15.1.55.5.1 5.5 .:.5:0.6 .4. .5595 0:.065:5.7796. 651 5 . 5.94 ..9 5.45.45.7765..4.::4 60 .3.4.5.: .169545.4. .:.: :1 5 3.55.6.::56546:.9 794.4769.79:65.6 79.15.5:91 / ..... ..5 .4:03.13 .59.: 69. . 59:.45.94.5 69 9621 / .9   ': .15. .45.9. .7765.5:.6 :  .9664/:.56.:1:. .51.: 195. 1. 065.0. . .1..51.7.45.../3 15.. 9.: .6.7765.5 0.3.6594:. 4.4/5 ': .3 7996.1 /5.5. 0 :493 .677. :55 .. .7796:.:. .45. 9:76515.55 569 .:6.45.9  661 5.: 4.:5659:::5:::65.:794. :.: ..5 5 .39. 6 ..45. 65:...9695.6.9 0 15: ..6535.7765.. #9:15.:.:65 6.45.:6. ..31:.65/.7.:654. 60   5 #..94:151/3.9 5 0.:56..: .5.7733.9 .7765. 65:.5 .45.656.5: .0.94.19.55..5 .3 ..4.03 6 .     ':  . :51 / ..5...6.15..55.0. : 794.45./..1 . 365.7765.65 / . :.: .5067. ..4.0.4.51.51 90:  .45. 5.... . 650   ' 511.794.5.. 794.5. #9:15.7765.1  .53.65.02: . 05.94009.3: 9.51 79694 : 50.3 / .9.5:. 0.59   ' 794. 644::65 65 7765.96.7765.5.4    .16:56.145.9.: 791   &..7765.45.6 9  :. 1 9 609  ..7765. .:65 .::460 .7765..65     .#.46969.90::6.50 . 15..5:3.45.6  9  .. 5 ..4769. .7765. . .33/0.5.51653765.15.7765.644::65657765.169545.7765.::0/3 '.:  .659:::5 90:: .02.45.3. 1 5.65(* 9. 69545.690..0..4736.. (5*66096947365..9.31 .065:.51.03:90:.. .::41 60 /064: .: 8. .65.51 . 5 .33/946169::751107. 6 .  &0.65.94 .3796. 4645.6.03..7765.:79611/ 3..9697.03:90 56:. :../064:06473.0.7765.976*     57.45.65: /.6  '.156.3.60. 56 .5 ..6565.65:..6.45.945.:   ': 9:63.&0   9. 63.. 644::65 65 7765.506473.0.94.3 964 60   "50 .   57.5...:9 6 .5..569 .5/0..0.1 5.7765..5 .3  6 .9 6 /93  .5:::65 1. .: 6 1 7960::  .65:.65: 065:.7765.15.5...7765. / .50 (.&6916..  . .15. 98945.519960./..  .19.94. .5 ..9. 69 1:5.31 5.7765. ..7765.9 : ..7765.65:.565: ..50 (.3 69 960.1:6.::.50(.7765.1:6..7796.7765..1 .90.15.956. .59  56 4.5:1964.94.9..::.5 :09.5 .7765.9 69 .7765.5659::.3:6/.0.65 ': .7765.51 699.: : .5.6 . .600:796..45. 0./307.089:.6   &%   793     5 .51.45.7765.465..45.4769..:..94.39.:8.94.5 / ..644::65657765.7765.6  &% 793  5./.. 644::656530..94.65:.69 0651.94 .1 .:::7015..:  .3:  .45..5 69 9621 .655.45.:56.:  ':06510.7765.4769.:3.15.1:.5. 5 0.51 644::659:  9:70. &6916 .7765.945. /69 .45. ':.:794.1 5.6.55.6   &%   793     5 ..7765.. . 73.45.5.45..15.94 .169.  313..5 .690651.0.94.65 0..51 9960. 57.   57. ./. .19..: . 9 .45..0:.7.15.45.7765.../3 650 .653/:. .65 5 ./.976*     6:.60..1 5..  8.65 6 .5.1  315.65:.7765.6 60   ' ..9 69 .5 769    .7. 4645.976*      6:.94. .15.. 1:.65  ' 9:.5653/9461690.691:5.: !665 69 0.6 946.94 .0.#9:15.5 0.6659::.:79611/3.45.5 .99:63.4769..45.65..0. 16: 56..3 6 : .3 69 960.7765.5/.0.4603: 69 . : 064450.0. 6 ..33.:65 . 73.169545.19...95 065::.: 8. 0.65 .31 .65:79::3476:1/.4603:69. : 76::/3 653  .5.:.6..51 . 0.:.51'.65    '#9:15....45.. '.7765.55.45..4769.50.3.15.: 769 .:.9..5.  532 644::659 .69: .7765.5:.94 ..45.94.0.46::.:769.6 9:.33/517515...7765. .65 . '65:.1 5...3.49:6.1 5 ...: .69 796:656...569...3:.6  699.90.0..65.:65 9 .: :69.65.. #9:15.65  69  5 653 69 . 06594 .69065:... 5175150 6 . 90:: 6 659::  ' 9 56...9 69 .64..#. ..0.::.5. 7961964. .1 69 1:5.6 699..45..51&630..9.:65.45. 644::65 6530."  :/0...:   '6 631 .7765.0.:.45..511 794.65.95.644::65657765.653..5!." :.:796:65:631 / .441.94.5..6. :69.33/. .45.33..:..6..: .. .64. .51...511/937.64..65: :.:4.90::6659:: .45.644::65657765.. . ..2.45..50:5..65:.7765.6 .  315.7765..51 .65:.606594.94651 .3   3.15.5.7765.506.65:.65:.1. .7765.3:695065.794.0.654.633. ..7765.644::659:.#9:15.6.15..9.1 69.:657.5.7765.:/69.: '::065.6.:065:5.94 .55.15.9.650.:3:. .337961:.#9:15.2..: 195 .065..0. 195.33.7765.7.0.6.94 . .65:..51 '.:.45.: 57.0 5 933.115..3361/.. 644::65 65 7765.4769.20.5.:/:.65/..!644/96.7765.1 5. 490 6 /6.5 0.631:.5.6959. .15.7.9365:..:0.: 69. "  989: .94 .7765.03.51 065945 769:   ': :.7765.9.510.5 .9.55.51.#9:15.::460335..979::3.90..:769. 3:6 7. :90:   ': : ..9. 002.5:. 6 .6... 3 6 1:97.3 69545.65: 5 . .51. 9./..50 519 .3.1.656769: .9. :7..:. .51:63769.4.7765.91.69..#9:15..0025769 6.5....5:.65  69  16: 56.65.50:5..1. "  ..6.6 .. 065:. 0647964: .65..369.7765.65:9../:63.9   ': :.::3:/0.3/61 '..5.: . 5175150 6 ..6   ...0. '65:.6.5769." .9790:3:.#9:15.:.3:3. 65..33.3065:..51 9..3:." 94.: ..91:.0 &&0.515665#9:15.5573..65:   .7765.7765.:44/9:631065941.51751506..5:9.33..03." 44/9:  ':70.45. 6 699.: "  .:.03 .0.657.:65  ..7765.: 796/..4769.94....511/.45.51 644::659:  9:70.#9:15. 065:.15.94.6.45.7765.1 / &0.3  16 56.65    9.9 69 .51 '...5 ...65   55 93.65:.5 . . . 65 / .966. 5645. . : .: '695.73.6 06594...1 5. 6 &0.:6....5. . 9.65  65 .65796:6565.976*     :0:: ...:.65 .   57.7765.45.49:6 ..94 ..50(.45.5.65   9.94...35.656.:...616..15.65 6 609: :/0..  65:.907. 6 .:69659::. 65:. 65:.55:::65. 644::65 65 7765..65   .03  6 .15..65.94 .7765..7765..49:6. 7961 69 ./.694..   31  ' 695. .6.:65  .6 95:.69.65:.9.45.45.#9:15.:/0.6 &% 793  5..  65:. /9 . 9.3 19.:  11 56. .6155.51 /69 ..65:..5.94.65.7765.4 3 631 9:3.94.2 73. 9. .651 4.9 .9065:.6.51 644::659:  9:70..: .369545.1 5. : . :90:  .7765.7765. .: 0. 3 ..93 .9 6 ::5.7976:6.  65:.:3 / 06591 / . 6.15.9:631 9:3.61 1:97.3 50.6 95:. 796:65: 6 .:    3...611  5.15.169545.7765.6.7765.45.791/69 .7765.997.997.65: 5 ::5.65:5.:69.: .9.: .45. 1:0.976*     ' . .50(.95:./.. :::65 6 659::  .:5.79:5...: .94.46.:4. 3 :6.45.: .5 .9647963651.559  !6 :.:  . 06473:69 90::   69  /0.50:569545.9 ." 9/.: 6 57. :::65 6 659:: . .6 ..94 .65.   765 ..6 / .57696.0 %6/9. 195 .45.15.94.3 6.6 &% 793  5..45.6 / 0..:  1011 519 .:65 ....94.   5605.6 6507065 9 73.6945. .656.30644::65: 5 : 0650995 67565 5 .45..16951  .1..7765...0   0.:90:.3/51. 57. 65:.::634. 65:.99.3.10.: 6 ..3 69545.  .7765.60: 50315. #9:15.50 6 6. .94.45.65 5 . :.:699. .49: 6 ..51 '.0.65  :..6565.1 5.65.45. 51 . ::.51  650 659:: .:1 .65:  4.6  699.15..51. 3 9. 9 :/:85.644::65657765.. 9.1 / . #9:15.4 76:. .65 . 519 &0.7765.03 . 796/. :.45.: 69 .7.5:.7765.65:   1 .45. .6 .::1/. 9..45.9 :/:85. 63.7765.65    9. 310:656.7765.6 .7796.9 13/9.45.:65. .#9:15.:.45. 644::65 65 7765.7765.51/51565 .94.7765.94 .:065:5. '1:.7765.3 796/.3.30. 5.511.644::65657765.7796.. 1:. 65:. . 6 ....0.77961 / . .   &50 ..796169..69.  /. 90: 6 .5 .631.45.    5.77.65   31  '9 : 56 1:7.7765. 8.65..6.6..65 65 . 644::65 65 7765.3:.   ' 1:..7765. /0.: 5 . .50 ...3 / . 644::65 65 7765.. 49.5.:5:.5563659/.6  ... #9:15.5 .:/.45.: 065:5.5. .65: 6 .5769 5. 65:.: 6 .: .7765. /0.1:.6516:56.3:5... 56. 9:.7765. 065:...3 10:65/.7796.: 6 .5.7765.:  /5 .45.: 0025 769 65 . 10:65 65 .45.:0.7765.45.45.556365995 ... .45.3964:010:65 .5 65 .15.:33.3 : .1 5.. 7.669.469.9 /.::1/0.45..396.:.644::65 657765.5:.56.94.:63..026. ::1..:010:65 .:::656659::  '9 : 56 5.94 .7765./55.45.6 ..65 /:5.6  69 .:9891/.45. /.45.1 5.#9:15.33 . .49.176565..45.695.: 065:5..7765.:56.. 6 .036:6. . 644::65 65 7765.7765.644::65657765...3 10:65 / .0...: .45.:65.7.:/.:.65:.631 .:9.  644::65657765.: .7.65  6 .45.45./.: 50 519.%3:6.   ': : 90651 5 &0.::1 .7765.644::65657765. %3: 6 . 7765.0. 7.95:.7765.::1.:33.#9:15.    ..45.45.5.5/065:191.. 315..7765.15.595..:.0.:9:10..65.94...:6 /.#9:15.45. 7. 5605.0 %6/9.7765.6 6507065  9  3013 73.51 5 : 0650995 67565 5 .::1 .:/.:   :.99. 5.7765.::1.   .656.45.:.7.94.7765.065.0.6/0.5115.504/5.15.7796.15.0.6/0.76:.:  5:69.45.94.3/.::.:0631/.556.: . .644::65  /0.563156069.7765.7651:.644::65 . 19:3 .0.:. 644::65 1101 964 .36.45.4.0.15.:: .: 6 4731 1:.7765.:  56.6: :6 /.169545. /0.7765.5..94.45.54731 1:.3:6 765.6.6659::9. .94.5.#9:15. 65:.0.659:: :473/0.7796.3 6 .7765.0631563659.65 59/64::65 .169545. 5.65  .5.:4.#9:15.9.7765.. :::65 6 659::  69  519 .5:: 5 . . 644::65 4.45.7796..: 5..17969.:653/.65 195 .6 .. . 5 .94.: 69 9....9.7765. #9:15.0904:.7.169545...1 5...::1 / .1 . 4.7 6 &0../3 . 6 ..2 .65   9.93 510.79:5..7765.94.6565.45. :0651 7.945. .65:. 659::  .:   .644::65 /.:16/. .65:.79:5.94 .65 964 9 .036.45..:65 69 :.7765.:/.6..33135991964:.50.96.656955.:3.7765..0519.3.. 0.94 .65 ':. .4  .7765. 765 :..45.. 644::65   ..#9:15.6 4.65 6 . 9.65:.99...: 1011 519 .1 5.: 03.65:..7765.65:.45.19/.:.7730. . ..  ./3:179.4 '9:79150 519. .: 6 /.. : ..#9:15.1 .164::656.45.9..  65:.45. #9:15.79:5.15.0. .7765.0. : 9 . .: : 56.15.7796. 9.3::. 1:..5 95 . 65 6 .94 . 065.1 5.5.:   ': : . 33.::1 .7765.7.. 65659...65:.    '796/.0519.:.7765..45.6501156.1:63. 9065179.1653/.0.57695.5&0.65:.644::65657765.997.65  9.7765.65 5.03.#9:15. 796169.45..:/. .65:.773:5.65.61:.77961569/..6.9 . 7765.::1..0.03.94./0.45.77961..7765.1:.7765.3:5./ 91/.1519&0.9.65:.:796/.3519&0.65 .9.5156.:.15.94.7.45.15.45.15.03 6.45.56.94.7765./0.65  9.65 9.556.: 1:.:.7796. .65 /.6.. 65:. 03.49:6...(9*.45..9 : 56 5. ...065.79..15.::91. 9.45.45..65 1:7.7765..99.969.: .7765.0.94655 .9:  79611 : .40.: .369 :.9: 695.:.:31 6065369 :.45.55791.9:3..1.45.946:5.946:5..5.94.::1.644::659. 5.7765.979.7765.56.950.  9.336316069..:56.1..6.94669.946 60.5.0536.:" 44/9..979.65  .9 ..969: .7765..:..8.1.3 1:.:5.6.5563659/9.03  6 .3:.65."       5:.556.94.1.65 ..796/.9469.9.9.7765.51 :0 5 .3356.. . 01 55 .7765.579:656.9: 5 . /0. 5:065099567565 86.94 33 56...5.9:../91/.6...03 .654.9: ...:95/651.65:.535.9:.1    '669036:.." 44/9:.7796.65   9..514. .15. 65:.33   ': .65:. 79::796:655.65   9.77.7.3 519 &0.1!.05&0.7765./9.33.15.9:      '9. 5251    69  .:  ..65 / . 644::65 65 7765.0.773   '6 631 6.: 796/.9.2.45.5 5791 .1 ..59.644::65657765.45.:.45.6.9:   ' :0651 79. 06594..7765.979.33. 6 .45..7765.94669. 6 ...45.:: . #9:15.94 .065:. : 4.6 9 ..0.2 ...7765.6.64.#9:15.:  ': 5.4 65.:  ': 33 533 . 644::65 65 7765.: /5 .15.7765.9: 631 4.:    5.7765. 693.65   ' 9:...: 9.5..9. ..94660 5 .. 79:5.1 . 796/.4.94.1 5.7765..95556.0..6 .90.9 ..7765.69.64.5106473.45.:/0..7765. 644::65 65 7765.45.45.4.94.3.7765..3.94660 '7961964.6.45. .4.94.6.6.79:65796:3.7765.1 69 .7765.773: 653 .77961 / 49 5.9: 33 3..5 .026 .: 6 .9. #9:15.96956.:.3:.: .:16:56.94.:9:.60... 3.. "  .5 79:65796:3.94 6 60   '9 4:.45. /69 ..5 ..65.6796/..65:.45.65 9519: 5.1 5.3 / 1:.3 769 6 . 6 .2.15.. 79. / .650631:.94 6 :5 .6 4.:46:.:519 ..94569 .7765.#9:15.6.65 6 .7765. 1.: 9. 065:.5.45.94 66069.6.45.515.94.#9:15.7765.: 33 0./:91.5.::./:53.644::65657765.15.0.169.45.644::65657765.7:: : 5.94  '6 6316.7765.:: .:    ' 79.:769. 6 . .. .45.: 3. 9.7:1 / 5...9.16:56.9:79:. .106473...65:. 769   ..: 796/.: .45.: :0.:079:6506473. .::1. 65:.7765.94 .9:. 0.7765.9.::.7765..3309.1 / ....:0.0659457696.6 65 6 .5 79:65 796:3 .: .65 6 .65 5. 644::65 65 7765.65 65 9.15.7765.45. .5.15. 796: .6 .45.53:.45..7765.4. 065:5.45.556 3659/951 .51065941/. 5 251 ..94: 6 60   ':  .45.65:. 065.5. : 064465 .5 796/.5:5.94 5 60 /651 .7765.7765.49:6. ':.....6 .45.45.:651656..1.:.45.3.95.15.511.51 015 65: .65:9.1 .51 .65:.7765.94.6: 6 4.510659415.65: .9956796:.:..45. 79:776:: .44/9:6.7765. /9.45. :5.7765.0.796/..7765.6. :0651..969.30644::65: 1656.9:3.65 ......339.9:   ' 3: :6.933/569.79676:.65:5.5 09045.45.45.44 7961 6 :5 .5.369545. : .9.7765..:5:9.5195.7765.6.. .19.94660 015 :5 . 4.611:97.796/.:9169 3::.5366763..65 65 9.6.1/63.:90:    ' 796/.1.4.9 065:.7765.:5.4.45.0 .556.51 5 9.:..51:/:85.7765.656.65.94.796/.6:6.3:6.796/.:657..63. 644::65 65 7765.:9/651.9 . :09: 65 ..5 251 "5.9:   !6.:.65 . 9.6065.. 691 1:5.65659..45.6.49: 6 . 65:.65:  .69:6.7765.65659.6795.7765.5.673.6 ./5796:3.51 . ..6565..45.65.: .51 '.946:5.:/0.3 0644::65:   ' 9.7765.36.:90: ':69..0.45.45.611 / ..65796/..45.9 9 :700  9.9: ':0651:.:. 796: 065941.69.7765.6795.45..3:656.369545.1.6 / .69.2.53 .6..9065:.15./736.45.: 96503.65     '.:65: '9:.7765.:    9..9:      #3.1.5. 9 065941 / .7765.::5. .5.:33.:69.4..5 796/.511.65659.45.....9 .79:5.1 .511. 6 ....6 699.:/:09/.4769. .7765.7765.65:9. 94:6607965 /9.653356.95.: .65:5.3 6.45.94.94:660 65:85..99:76515.51.7765.65:.3 .5796/. ...:.5 6.6/690:1/.65659.59 .94 34..6  699.65  9.94 795 65 /9. ..65 '065. 5 .9..3: 6 .5.: 69:6365.: ...:.15.796/.9 06594.45.36..45..511.7765.94.9:561.:69.7765..:65 .9 69 . 1 .9      5 13.45.:339:3.3:5.51 95.03.7765..:5&0.63...:.9.33 .791.6.51 '.45.9 .6.: 6 57. /0.7765.99:76515.5595.15.9  16:56.:6.. 69:..9../.65  9.50(..976*     .65  .. 65:.65:9&0... 57.03.6 &% 793  5.6.. 94.644::659: 6.9.169.5."  :.6.65    9.33.51.(.03 .7765.7765.*.773  31  &0...9:.45.6507961: .65:...6.946 :5.33/.. 6.. 6 .15.: :9::33:5..94.7765. "  .* .:.45.:796:6533.7765.773 '9:.65 7961: .644::65657765.. (.51 .....94 6 :5 . 644::659: :. 9..65:9.906594. 65:.45...  '9 .6.65:9.5.9 69:...9: .65/.1  69 .94.5 .33 /.7765. .0.9:   ' :0651 :.94..556./9.1.44/969...0.:.33 :95 469 ...7765."  .6:.9 06594.9..65 : 9 .  .7765.5 :5 .9.:33 .9.5 /0.5 / . .:.59:5:/69::5.65 :9:.6.7.51.94.94 &0 79:650. 94:..7765.7765.06473..50.65 : 9 ..:.7765.51. 33 9:3.9: .7765./9.6 :9 . .3356.9466051: &0 79:650. 33 9:3.03.3:6 :95 469 .5::95469.9:   ' .45...945.3:6:95469.9::9641. 5791 .7765.45.7765.6:9.1 .65 : 9 .94 &079:650.9: .9:    ' 69.:: :.9..9 .506&0.556.7765.7765..699:5.6533/:43.65 5.9  .5 :5 .7765. . 5 .5 :5 .0.1 .1/0.945..: .51.9:59649.45.: .3:6796/..:.6:079:65.50.1519.94 6 :6465 6 11 69 9:51 .50.7765. .9:3. /0. :.6:..: . 44/9 69 0.:0651:5.: 796:3 :91 .44/969..55791.9.9.9 . ./9. .94 6 3:: . 9.91 :..6.6. 9.9..9:5 9649.0.6. .0..:.5:5. 5 .. . .65  9.5 :5.556.5791.. : 065941 ..0. /0.7765. 45..:.65   9.:/69..: 519 .03  6 .  65:.9644. 9:. .7765.65 796/.79:15.5 ..5.9919649/59...25.6 &% 793  5.5:5.5:.6.7765.6465.517..330.65  ': 796:65 99: ...7765. .6. 65:.9: /.94 190..1519.5:.. 65:.7765. #9:15.93::.7: 6 ...94: 660 .. 516:.65:./.65 6: .976*     '6 .: : &0.45.50(.9/..65  .6 .    .  57.1. .6: 4.65 65:: 5 .9.51   .1.:79015.7765.6:0.: 4.45. :6.91 30.03:190..65:   ' 9:.65 61 3 .6: .9. :0651 065::.#9:15.65 9.:.45.9103.6:4.6 .6465. "45/: 30.1.6.:  .1 69 7.330.5.: 6 .: .51.31&0.9:43.7765.. 99: .65.6..5:.169 /5 6.7:6.5 065:19. :..65 6.9..56.:650.45.65..65/.31 &0.3:  &% /   5..7765.: 79:414.3 .:.. 5 ..69 &9..91.:.25  /.9.6:..:. .03  .&931 .69 6 90..:5.335 :..669.6:4..9..69.: .554736:6.: 65 ..9 4.7765.6.4 6 .796/..3:964 4. .335 .45..45.1.969 796/.653415.& .16/6:369 7.7765.1... 935   .65.1 9651/::.5  79.:6:699.60.7765. .36:.:65. 6 ..69:.45.6065.7765.69 765.3 69545.45.7765.473.65   9.6 :.3:6 .45.65 69 .45.:.9. .6539. 35& /.51  31  ' 90691: 9.:65: .0.. :. 6..:9415... .45.:/69.:9415.0..33.51 .   &% !6   5.569930.511794..9:76515.9 54/9 .19 60 .  * 03.330.7976:653505.331:.19 :067 .331 415..169.9 6 90.51 ..:..:.45. .969/1155199.0646. 69.0.:6.6 79:15. 59.7765.1 .#9:15.3:6 ..515.51 65  #3.677.45..: :65 / ..9:563.25.50(5. . . 7.6   .59  %.:065. '506454..:360..5 9. 65:.330.9:. 69:. ... .65.4.659 /96.7765.65   &631 ..50(!.. 4.1796/.45.353.. 4.. 5 4.:  ' & 5 5 % 65  .515.0./3/141..65     .65:. /96.:.03 &0.5 .773:653.3.1.:195.5106990.:.55.34.7765.7765..::4560 90.4507.0.:. .7765. 56.1519&0  9.4507.016   .3.: 5. 5101..956.965:7..692165699..5165699165/::/6915.:609:.69.  .:0676.3.69..5 5969 /61 .963  5'. .3 :79:65 69 360..&. .5.: 79611 / :.9 .4:3 &79:6516:56. 7.5 . &79:5609:493 :.3 69545. &79:69769 5 065.: 964 : 065..616.5 9:.. 360.956.:    0. 67769.. 6 0025 .165:093: 56916:. 769 6 49 69:. 0. 360.6919.96 79694 .: .69 4.9. 769 6 065.5 600.63361 .. 69 .:.6...963361 /.6:   31 .9.. .3 69545..6 1:. 69 .9633. 5031 .69 5.0.:.. 5.93:.6/:91 4.: .6*     :.0.469.3 69545.9.646169973..93:5.5.1 . 065.5..9..4 ..5.0. 769 6 :79:65 964 .0..66919..69:0/61 "09:5065.69.3   &..4:316:56.5: .93:.55.165...9 1.: 69.45.65.: 769 6 59. . 16: 56.65. 311 56 469 . .:1:09. .0.556.963  : . 609: .069:0.1656.: 79611.69.5. 1:09.:. #9:15..:65: 5 ..963 769    31  "5 4.1 .99 .5: . 55969.6.79:09/193: 0.  . 165.556.6..79:09/:654.0.1656.6065694. 51 &% ."565 95..5   91 (65. ..65:6. 7969 9064451.963 69 .: .765.#9:15.659: 065.6..:1  ..45. .5 47691 609 .. 609 :0065945.05169:5..:33.930..515.:.515 6 .7765. /9..51....6/.3 796:0.96:.:50::.:/6915.694914:.7.3796:0.6.. 17.9. .995.69./519:..6 .769.55.:. ..779679.7765.:::.51 60:   65..609.65 .616 662 '.3: 519469. %:1145:.4731:09.65664 . 796:65 6 .7765.963 4.   .50.50..3 .69 4...69: . #9:15. 4.&09..9064451.79694.65 631    76..0.45. 79679 519:.45. : .69 98945..65:     .45. /69 .. .6..506:1.5160  .995.    5.0.51 0.6 .3 .9 .55. 79::15. .1 6 69545.&09.99064451.9 6 :.6:/:..9 69 461  69 5 533 69 :..3.. 769 6 065.5..:...4..9..6 :..65660.56. .79650.: .9.5.1 / ..3.330.5 . #9:15.%:*     : .7765.661.145..656. 6: .9 &0..51.769. 5031: . :631 / 9 79..  33 79650. 6 . 7969 9064451.9.:1655.7765. #9:15.: :.0069153/5991..6 .9   #. .5.::4190.:  /9. :/6915.:..#9:15.90:61:09.036...7765.65 6 .4:.6.0. .33 / .511.656..3..51 .0 .51 .5: .#9:15. 6 .313.5 .9964. 79650...9 .3.. .69  31  ': 8:. .17.65:. &09.693 989: .. 65 .9569. 796:0.69..6569.3. 0.59.45.7.*     :./515696/3.. .9644.69.656.9 '.65:9 56.%:1145:..60. &09..1 '9064451.664.: /9.9 %/5 '699:   %  !6         91 (.   941   0.65 .916916..9 '#9:15.33469.515:667.3379:.1: 69.3 662 6...9.#9:15. 69.9.9.9.65 6 .:769..45.1.6/...5 0.5:.1:6955.556. 79.. 065:191  6 ..0.:5 0..61 :631 /5.979.9..69.0.:..69765..:4.16.  06319331:9.7. 90:61:09./:.65..: 765 9064451..:6960:55.5. . 0:::5.336.7. .    .1. : ..9.:..0.1/651..9 6515&0.. &09.5.49.9. /5.17.65695169:45..:067 6 : .941690:..5156.519. 5.  769: 6 ./.03.3. .69...56919 .65.665.679603.03  6 .90:6:0 769   &0.3::6350 5.9.519.5.65:.65.3 93.6795.6.:0697: 73.336.30644.61. 65:.9.6/:..45.945.51 ::751.60.7936.65   9.9   31'9:. #9:15.6.: 644.:65699/3365 964:769.65  0 4/61: .33 1:09..44.#9:15.50::.519..51...69:779::3.9769...9...610.6949:56.:/0.9..3.941690:. 5. 33/.:.9.  '#9:15. 7961:57.644.519. 5. 3/. .90:6. #9:15.6103...4:.6476:4. 69 :779:: 3..::9.35.33.60.6573.6.:5.51   7/30:..6 0.91: / 659::..556..769.6 795..965 : .4.57. .9: .519.:.65.51 511.6.  69.96   69  . 960. .519.9:.3.:  54.65/651.0. .3 /.. 0.3./3 .0.9.3 50::.6 1 50::.9960.3::6350 5.:3.9.3 644::65   '9.769.65 50::. 13/9.90:6 .6::751.94:..101.#9:15.6.956.7936 .91:.:6 5..9 .: .9769:   .../.9. .49:6.  69 ::75:65 6.769.6 .:65 69 9/3365  5 ..9 56.335 6..6..6..3 9164:  . 4.769.655:5.9   4.:0 03 9.3: /.1:.51 .. 989: ...651769:3.#9:15.91.69:0.4..769.1.941690:6.6 795.6&0..5.6..45.: :1:.9.. /064: 50::.69964..65:4:. /064: 50::..3:: 6350  5.:65699/3365.3 3.33. .73.:0697: 6973.:: .67966 .064/5..51.7..069.5.:: .35 .6 0.30.:4733471.33.069.69 696335.5.:...0.33./6.: 6 ...5...0..51.0.1507650.9  .: ::751 .65.00::/3. .336.3.. 6 0.5:.6.. 7/30:.3/..4..9.0:..:0506...9 ..:69.33 6.9 : 56 :0 8.069.0.99569015... .65   9.9.: 50::.9.51 :779::65 6 09.   4.51.58.51 13.91..65..35..0. 0.3.65:..9 .65.3 :.941 690: : 56.... .5.4:. 793 6 .941 690:   ' 653 09.65.51..6.9769:.15: ' .65:.60.3::9.55.5.65  5 ..3356.06509  ..9: '../.1.50.1:. 796:65 1../.0.:.9./.945..3.769 5.50: .../...336.65 69 96.6 .33 .3:64731.0.55:.51/96.97696.6476:4.:0697: 6.65. /6.6 . 7961 56.:0697:69.:69.941690: '1:.7936.0. :065:191.769.941690:4....   '331:09.3:6 796 54.9.519/.519.7936.33 6.  : .65:.15. :0 .19505.9.5.9.. 69 :779:: 3. . 9891 5 ..65631.:3 8.6945. 6965 796:65:  659:: 4. 0.9.0.6.4.33.769:.:..66..0.33 : 33 1:09.5.069.. / 6/0.9:.69./0.60.99 ./103.. 59 ..:65699/3365 '4730. 5694..:   .33.965194.:5331:09.::69.656:9.941 690: . .03:6.0.9.5694.336./3.9:.:. 90: 6 .....65 9..5 :.5.9.579.6 ::751.#9:15.:6569.   6969  519 &0. 4.. #9:15./:506.65: 6 4.../3 69 . 962 :0 79603.60563..51469/557690647.195.989.#9:15..065:.69. .#3775:69.1:09.2.. 769 .3:.9:065:19169:./..6..37966     .:0647.60651.556...#3775:.09.6./3:1:504..65: .51 59 .: 9.65 79::656: :. 769 .0... ...659.60.5517515. 5 .050.65:.61.5 /.51..654.7936 .. 65:. 769 .. 65:..6.03  6 .517961169.9 56. :0 145.#9:15.:..5.345.:0697: . ':0651.03   519 .941 690: . :..94..65 6 ..6.3.65.5.6:7769...3356.6.:0697:.6::751..5.51 0.#9:15.:65 69 9/3365  5 0. 015 :.  .519. 6.9 .51  ."5 . #9:15.: 644. 5. ::1. 5..3350 5.0. 769 .6 0. .6.:09./03. ..692 .33  65..5..   5 .95694.:306515.6 ..369.65 :64604. 90: 6 .:. :.: . .. 65679679.50::.51 10:3  ..:76.33 6.. 79:5.9.02..6.99:. .765.9.1. 10:65: 4.65:.656.6 795..65691:.9.6 .5..5... .: 6 .3:: 6350 4:.9. 065.65.53:. 511 .65.50631/.6:.6 0.769:/0.01 5  065:19 .3 . 6. 10:65 .9 '1..6../379:097.::07694. / 479.3 :09.5036115.:644.4769.5 0.65 5 51.36::64.0./513:.5 550.9 7.9.. 43.514..1/...#9:15.945.33   &0 .9 5 4950:.3356.90. :05.9. :.::1:. / 165 :.:. 5:950 796/34 0631 :733 69 .:90:1   ': .519.69.9110. 69 :779:: 3.9..5569199. ..6  9 .9 .:.65:..4. . 9 .96 : 56.. .3 50::.6569.9. 5. /:1  .73..0650..96:.  65:.910:65:.9./3.556. 3 6 .11.. 5.  % !6     5.0. 0651.3. #9:15.9/73..1 .90650.. 964 ..1 35.69:779::3.569045.9:5699.5..00:16.61.60.. ..   31  &0.65: .6590. / 5.1656. .9 6 .5 :6 .3 145.: 695:.69493393.6 10.9.9 . #3775:   65  %65..: 9.6.5. .4...1./151....75..:./.659::0..361 '::75:656:0.3 65    . .:.6 :.556...6/0..3 :5.5194.69:13.:50::.679:65.50 .06.41 5 .1 / 5. 7.9:5.4656.517.7:.65 6 . 0.06.:4739..:.5.6 ...6 :9 .%:1#5.65./0. '0.5.: 155 . :..65 .9.65:.65 :..:09..6:1.30650.650.65   9.: 0..79:15.036.:69576969.. . 10.33 76::::: 0633.03 65:.9: .50 .37696 979.969.5. 0650.9..: 9.45145.5169. 769 6 069. #9:15.50:51:7. :3.5. ..:32..5.656.1. 0..0. 0.3.979.45 .0 &% .0650..19.5.: 769 .6 065.94:6..37..:.9165:...014:3519.. 1.1.963.9 6 .. 63.1/3.590:610.'%"65.0650. 6.3::6350 5.6/064:5:..:.955:145. &9..:79:656.656.: 9.7. 1:9: .9165: 065:5.63479:6545.5:.....39  310651.:65.03  6 .   ': 796:65  69  0.5   5.941690: 331:09.469.:0.3 .: .0650.6 16:656919. . .5 .96.5. 069.60644..: 69. 5791 769.51 .:65699/3365 53::.6795.: .9165 33/90644...9165:5.9:....979: 0644.. :5.:/50650. 90: 6 :0 1:09. 5. #9:15.65   ' 769: 6 .15.:56.5096.9769.65 : :473 .   5.51 .:065.006911 9:70.0.:.  .7..6.50 69 .9.6 .5.1.650.9.7.1.6.75.3.10945.:..3369.7696.9::75:656. : 90: 6 145.65.5..65 6:5.5.0165.: .69.3 145.065..451%  !6 /9105 ..& :.:735.51.5*     ::5.65 . 69 960...9... :5...65 6 .4.5.16:56. &09.6 .350.6 0644.60.9651 .6:0.65.5.65.4.5 / 03. :690 6 769 6 .779679.4769.9. .63  0. #9:15.0.65 9.....65 / .06473.  5..65.031..9.50(#56*     :0:: .:97.3 9..3 7.1 ./56:. &79469.0..50(.6 / ..979519&0.9165   : .94::..90:/659::6. :/0.9. #9:15.9:/0.0651. 5.69...: / 065:191 .0165.316 .9165.50: 6 0650.3769 .43.3.556.5.9.451 3.0..65 / 5.. 691... 7..9165   519 &0.51 0651.65    6 . %:1 145:.7.916506473:.6.99:......:..519.  0.65: 6 .6:. 61  .9..66919.94: ./651. 50.9165 7.0651.  :.33.65 690651.65:6:7. : 56 .656.50 .963 69::75:656:5...909.3.5:079:656 5:145.606473. 33. 3 :09. 7. 56 .5 065:5. 0651.3 7.9165:56.08.03:.:/5. 7..65. 4  . #9:15. 0651.9165 9:.:: 69 ...99.96956.945.91655: 65 145. : /651 10./3   .7.1935.5..6 0.3 7.. 069.61.5 .3 .#9:15.:.9165 964 .98945.  . ..9165 9621 / .  ..65.9165   '9 : 32: 53 .69.3 7. 65.: 6999656:.516.45..9: 31 .65 :.6 73..3 7.9165.73.. .37..6 5.69.696 .0651./9.65.: 9:10. :651 145.65 .: .65.1 ....6 . 562 .94: 6 .5.4   69.. 65 0651. 6 ..556. /.65 6 .9165 ..515:4.3 6 .0..5.3.:69191    3.:.  0.06 .3796565045. ..: 03:3 5 .. 0651.10945. .6 ..9:.069633. 63. .9501 93 5 .65..0..5   ' 1. :. #9:15..310. 0.1 6 . 769 .. / .4.: 9..45.1 ..:/56969.9.6.51 .: 90: 6 79:15. 0650. 6 .99 . :/:85.. 7. .:960.: ..6. 9.9165 765 .97996.: . 90: 6 769: 51:7. 6 .0 : 3/9. 145./7650:90644. 95:.  0.5...3 :633:..5:.5.3 145. 069.9165  .65 6 . 7.. . .945.15.695:. .6569.6..:695.769. 9.695.9.: &0.9.51 ....653.: 065:.469.    31'#377565:.31..316.5690:945..945.3:3. 6 .6.95.3.65 9.  31 5699:10...9 %65. 65:.6 .3769.&09.0.:.*     6..6796:65:9895..3 5 : 60   5 % 3916 &4365 '699:  &%   0    (946::4..65 ..945.  % !6  "0..:...5:.:065:5..50(5. 69 06509950 .65 .53::06509915/ ..3.65./:63.:34.1:  !6.2.:.5* 0..773: .5:.9.9.95.91 .:0644653 /31 5.33/.5 .#9:15.&5.9 '9636./. &5.15.:.769.69. &5..65.039...606509.3. 90: / .65065.51 56...65..:633 ..065099506..   5.65.6315.1653.65  !(.69.*     0 796:65 6 . .::.510.. 65 9.1/.:. 06509950 6 .6.5...76735.:: 69543.79..6.03 7961:  .%7/306.&5.:  .9.9/.399514 3169.&5. ..:..3356./.03.5.9667: 69.945.4.:6490..0650955 3..33615.7976: .6.65 9.6..9..33.1306509915/..9.5190651.519..#3775: ..5.95.    &0./.3 500.: :.....:: .3./....:.51.:695.:559.    &0.0.69.031.9.91:6.51 5 ..65.3:.65.:0.9065..5&.945.659:::6989: 9.6556. 3./.. 4/9:6.1&.9..#3775:07.9. 6.1 &...9 964 065...: .65..03.69.3 9:10.. : 653 65 6 .065:..33 :.&5.9667:.7.33.69543.33615.510.6:/0.95.915.91:5.5...#3775: ':796:65 3.9  ..    '669451 .65   9.:: . 6.:5.65  4645.3: ...945.:4.: '6.3 796:65:  .9 33...31...:4.4796:65 :44.03.65 9..79.945.3 .065099506.695 4.9 79:6553  ...773:..65 9.6 695 :0 :..:165.5. 5:.. :/0.  4769. .95. 7961: 69 .:.65.: 5635 695 43..51.51..51.65.515...65.51.#3775: &0.511.1.50:  .6065:19.9667: 69 .65 6 8745.9/..65 9.95..6.0...9   5 7. 695....9667: 69.9..:69.: 6 43.6:065640 55.9.9..:09:.69../3 .  9.45.335 .: 796:65  .9... .9/.65 9.65 .50 .:  . ..:.56946..1. 4.95.: 6 ..2 .51 .9667: 69.34.&5. 4.5.6 4.03.:. 065:....09.: /. 6 .1 :5: 69 ..:!6. . 5..9 141 796/..9 0.65065...9 989:.. . .6/ .945.9.51 79:6553 :. .945.6 9519 .65 9.3361 11.31.398945.03 ..51 . 4.33 1.91:   6 . 6.: :631.6/..015:...::  .9..3.9..13  &0. 6 ::3: . :69 065:191 .: 0. 7976:  :6 9891 / 659::  .9.1.6 9519 06473..: :0 / .51 0.39365.:.    5065.: .7735..:.9. 5.51&0.3 99514 31 69 . &5...65. 98:.913::6:/0.. 697.9519&0. 13 0650991 5 / .6569.9. .33  5 /6.: 5 .65.    .::. 4/9: 6 .9.945.03   0 :700./.9991..51 769. #3775:   519 .7733.9.9 069.3..6. 4.5. 4/61 79..59. 135: .65..3:.0.. 79:50 6 695 43.9 :0. 065:.63 ..909.:: .51.519&0. &5.5 .: .773:..51.#9:15..9..9.51 90651 .....9.9  &0.945. ../3..51 9./.0. /6.9 /..9 065.:..033351.51 .9 :64 064465 9651   ': 065:..9.#3775:653/9.65 989:.03:..4.0 6.. &5...9.0369&0..:..90...31.6..1 / .65 9. : 9891 . &5.039. 06509950 6 .65.6569.3796:65. #3775 69545.65.51    .5 .5:..65.9 6 0945... &0.796:65:6&0.9.. 6 .9 15: ..03.9.9..9.#3775: 9.0.:06509950519.3 98945.6.065099506.10.3.606473.91.65....34.:.03..3 796:65: /6.03  .:695.5:. 06509950 6 . #3775:   .7730.51/51565.:0563 .:: 5 .::..03.56. 69 5.&5.&5.6 .:695. 5 ./51565.:    "5.3356.. 9.5&..45.9.3 695.1&.0. 5.65 675: .51:773:    516/.. 5 .. : 51:75:..9.510.33 .:70. 5.54/966.:637976:61.69.31065099506.    .:..9455... &5.66/.:9891.53. 51647.3: 69 .79.3   ! (.565.3: 196.3 65    :70.:: "0.2.5:..&09.4/1117950735:.3796:65693.5*   0.9%65....90..469.. 59. 59..    &% .316.:70..5 .65 3./.  % !6 .65.69065:..  : :706: .5:5.65:4.9.69. .  493 5631 .  6.5. 43.65 .65:.7730.65   9.1/3.65:.: 79:6553 5../3:45. :50 ..9 : 56 794.945..51794.9145. &0. 6 ..5:5. 1:.:631 56.: 6 5.969  &0    9. /..50..: .55.:65 .6 .1 &.:5 ..53...6.654. 9.53   51 56.:.0:.55.55.6516973..03  : 5.9 69 . 065.5: . 989: 695 . .65:5.1 5 65..: "5..*     &631 .. .  9..9. 43..945.9 :. /731   31  .5561:..05 6 :.65   9.50..01794./3 .9 90:: ..51 56.03.50(5.9 ... 5.  .65/.4769.5 5 &0.90.3945.9/.#3775:    . :.03  .9667: 69 . .6 49 ..2:561:.35..: 69 . 73.7730..::069 . ./3.5.6 / :.  / 3 565 1:..51647..2.5* 0.469.65 3.  % !6 .565.9%65.316.59 1/46:  ! (.5 ./.5.5:. 50 56: 1:.::  "0.&09.   &% . :796/.51517515.:/0. 493 695 . 79:..03.669543.945.:06330. ./06990.6.::60..:: .5964.03.34..511:50..50(5.065..6./3  .96..99..1 065:..5.&0.::7.9 5631 5 ...1195. ..65:56..473.9 56315.03.:  . 6.549.3 ..515175150665..9667: 69..65:..:505669543.51..9667:.6.:.796:65065.493..65:56.9667: ..036.9/.:: /. .65.96335  315324.65 9.:/:09/.4..*     33.51.9.1.65.::/5:...51 .:.: ':6 0644..:.: &.:: ../3:1 '03.9:503115.65 9.9/.3 6 :.. 065.3 796:65 9. 796:097.3/.:..9667: 69..6916903.    !6.559 1656.9 /.5695/.03:56.9667: .65:.9195..969 &0.96335 :50 56 695 43.03.6.65 :0.:16:56.9.03.9...3: .::  /..: . 5.65 069: 695 43.93:5:1:...9 /.. :: /  695 .9 . 43. :/0..  695 /.9 . 6 0 0631 / ..:. .9..9667:  69 0  695 .03. 33:1..515.:51 .5.9.51 ..5:..519.. 3.9:065::. .5 .9.36573.51 .9 /...3.: :.6.656...979.6&0.51.. 0.0:.9: ...65     6969  43.765:  11 4::3:./3:1 . :. ../3 /0.03    '6. 5 69 465.5.:69.765: 6 .955.95.: 33 .36..6..:5.65 9.9 :0 ..5 .:..9.49:6.9.. .9 . : 56 3659 .. ::3: .9 ':43.9.::/:.9:..0.9.69 6 .9 :.9:7:..: 503.: 691 / 5 4.56.964065.56.:: :.069.99.5: .: 6 .5.. .51..909.653643. 3.:56.64/.03.9 46/3 ./.91 .6 .3:6643.979:6553.9..51.: 0647..: :1: ::3: . /.469.51647..18.9.   &% ..::  "0.  % !6 .5 ..5* 0./.9:   ! (.9%65.565.:1 43.&09.316.2.5:.9 .65 3. &5.5. / . / . 4. . 0.: 5 .3.0398959.65:.:56..3.:6&0  9.0. 65:.9:50659::.: 5 .00691.*     9.5... 45.03  1:...: . 6.9..9:05..6.: 0.5. 6.5  &0.9.9519..6 ..4.6 .03 ./1306509915/. 79:50 6 .&5.399514/5550::.:.9 53::.65 9.51 0  90651 .796:655.65 9. 5. 6 .  ' 06509950..   : .336: 695 43.9.9..::  ..: 6 .51..50.03..945.4. .: . ./06509915/. 6.5&0.51 5:69891/659:: 9.. . .4:.65   9.:.398945.31.9.9667:  69 .  31  &0.3 99514 .6/..9..:06509950.1/..65   9.96%:63. 69 5.98945.9.. 76735 .69.65 .796:65:6.65!6 :5.:7004.9 065.9 6 6.511/.6.6..65..69.59.4:.6 98:.65. 6.65:. .03  7.9 /...510. 4:.34..   '9 : 56 1:7.65/.3. .03.  ..65064731./519.0.03 69.:0. 065...93 989: . ..50(5. 9:.95.. / . 5..:.5:...633650651...9.9891.65.6515&0.&5. . 91:6.33..   . 44/9:6.65.65 9. 6965 065:. ...51 &0.3 796:65:  ../ 1306509915/.   7735 ..&5.6..9..03:4737961:. "5..6..9..&5.  &5..91: 6.. 06509950 065.465 6. : 03...1 / 3.51 . 6 .313 6/. ..5:  .9 .51 141 79:5.9..33 ..473. 44/9: 6 . / ....03  989:  .: .. .93 9891 :6 .. .65   9.   3 . 4. &0.9 .  5.0.5:..5.:.. 945.94...9.. 695.94: 989:. .&5./1306509915/.65 9..65.03.6.  /4.065099506...51.6.51153... :.969.95.3../93.1/.03  05469:700.4/6115&0..1..:9.1796:654:.. 33.65.6..03   : 56.9.1  .&5.03.44/9:6.473.03 5.91 5 93..6&0.... &5.:.034.15:63.065099506.65.6.03  4:../. / 065:. 06509950 98945.034:.796:65:6&0. 56.065..91:6.. .4697.. 511 &0.9..65 9. 519 &0.5:.65 9..65 9.93.6.65 9.3.1519&0.5.65   9.. 33.44/9:6 ..91:6../36.9....&5..606509..69.. .0.. 94  ..979:5....5.6 :/4.. : 6 .: .:69.     ': 69..511365.9.007..  :.....59:631.9 065. 6.. . .  ' 5... 4....: 0.9. .5.: &5.::765.33567.5 :...00691:.9:.9... 5 .1 &.:79:09/15&0.5.55.9.9.5: ..6..4.0.6..:.6.57.65 :. 5.5.65 9.1&.9. 79..6.:.: 90651 .:  ..0.5:.:   33.025631:.9065. 69 0650995079:.: 6 490.90..:65:.79.03../ . .698:.:631/90651. '6989.98945.1 &.  ..6 ...:     .:6490..945. :.9.: .379::656695.  .050......1 ...... 5.51.65.  69.515860.. 795073: 6 5.9519     ..   '6 / :9  .65 .: /515 .9 .:6490.6615.  653 ..  .:69..  76::::: ..:.94:6. .:330644.6357. 6.3 .9 47361  5 0 0.. 519 5. 345.1&..5* 0.: 6 490.69 . 4.3.9.258.9::5.&09.945.03.65..65. 0:8.     '90691:9.5 .:.51 9:765:/3.9 . 1....:365..: 4/611 .3 3.9.1&.479. /0.316.9..5.50 . ..9 : 511 4.: 6 . 56 /064: 6/3.9.565 .50 .0..945.5 .       &%   .025631:..6 7631 .  .0.6 / /651 / ...6.169'64.55 07. 065064.:3 9.964/.9.5064465:   6969  . .: .65 /. 795073 .5.65: .1 . 9 .::.5.. .007.: 365 .95..065099506 ....:.51...65: 519 .94: 6 .1.51.:69545.::  "0...5. 691: :1 5 .91  .25.01.51 .51 647. 65:..921 06473.9 6915.. : 5065:85..5 0.: /.66 :.6.&5. .1 &.9 .: //.6/.6//651/ :.. 5.9501 : . .  !(..: .::.95.65 :065:5.56 56... .: . ..945.65.469.35.5:.:.5.   %  !6   .3 ...5 . :50.:5:.:69545./.: .5.6.:3.95.. #3775: .65:.007.: 6/3..5 0....  .3.: . ..:.0.3 3.:  519 5.65   69..653...9.. . .3: .9..#9:15.50 .33.9%65.1. 65 69 7.1 &.:631/519:.. .:631/..6 06473 .9.2. 6/3.9.3 3.: .51 504/5...  .656.  519 ...945.  ..1&..945.: ...9 4..51/51:. :..6 / 5 .94: . .5 .5690:945.   31' 6315 6 .:.*     9...  5 ...32.590::0691/.32.50 (5. . 15: 93. .   ' ' .6.65:7 /. '  69 /9.:3 '9:. .015.: /5 1:09/1 .35:'9.60. / :..#3775:/651.:.4692 6 .11 5 ..   . 069 6 .::. .5. 4:.: .9.5 . 9.. 9.:.9.5: .3:7769.0.69..5065.9.51.:. ..9.60.9.9:.55.33 ../3.6/3.7.:669.056360.: .97.1&.5...65.65.94   '3... ':6/0.9.3.50..95190.:0.5..:490..4:.&.7:6.:.3 0.941690:..19.: .:2:.1.:.9665.#3775:.51.32.:.3.51.65:.5590:190.:.:. . .10:6556.::945.#3775.5.1 .04 5 &..09.695.51.  .9.0.65 4645... ..4: 4769.6 .50 .  .51 17.945.5 .65 03..9. :. .9 . : .65 /..: ..93 5 ...51 #3775 43.: 69.::. 6 .. 6.9.93..01 / ..65:  .65 .6 ..51769. #3775 69545. 6 .51 93.77961 / .9 6.. ...51 .: 33 .6940.119::1:..  .6::3:.9 6 490.3 .909..5 690: 945.65.51 03 9:...:631/.65 6 .31.331 .65   . . . .:.65 6 67.5: 796:65: 93.3 : .. 79646.8:.5 .6 5.. ' 1:7.5103.: ..3   &%  (*   '  7961:.5 79:6553  195 . 7.   .5 490. . 6 ... :  5.6 .4769.: 794..9.32.* .:43.4. 731 ..: .469..1&.6/9     / ..9  ..9:   . #3775:5 06550.  0 : 065.51.5:4/05. :631 / 90.#3775 15: 93. 065. 0644656   '9:. 19.02 / .65.39:10.579:6553(4.5.  .3 06679.65 0945. 5. 6 35 .3    .: 973.9 690: 5 . 65 "0..03.  .  !  . ..945. :.065.34.9.065..   ... 691..7796.9.:.:4.255..:03.:6.5 ..6969 7976::6..:6906. 5..9. 336.:996515.796/34 ' 794.65 ... .653.5157.4/.7.4.1&.25: :/0.1&.9 :06501 .5 690:945.90..: 3..36.9964 569..11 69 '55.9..5.55.6/6.:5. 569 .556.1:..5..06473.. .:5. 5.:.3..:..7.9.4/6:  794..6532:10.9 964.95.543.5..0.:79:65534:././:1.#377569545..0..:.9.:3.9 691: ..6..556.51 :933.50 .0015.. 5151   ' 79::65 : .556.5.4..03.956.: ..0065..5/..:79:6553..699.55 6 0 .: 56.:3 !6.979.1 ./..65..5073.:0691/.945. 1 :067 6 519.4   ..3..45.69:...0.4.559 :. :.0.6.654:.9964.95 9:690:  :.:50::..65 5.5.99.6..6.:..1065.: :.9.6....6.   . ':63504/9.655..6965.963 .65: '655.0.:: 5..9345.53..0.5&690:4.3 :50.:66955#3775.1. .3.0....77.65:65.9.: .01..5.40370.9.96:964..5:796:6:69555.5.6..3.09.94563647361:.563.0.65 :06015.:79.65: 4.365:1.915..90::4..:15.65 6..6565.6 796. 65.69/.: 33.6'9.058:6 7..5031.9.: .5..11930./:.945.55655..39365.979.91..5065::..35.   ''94:6%9509. 0:79:41. '69:63..5763.0165.:4..: 0 065.9:. 4. .:13/9...99:.33.54794.5964.65...9.0.6110..94: 0.65. .51. : .4.:70:  . .::4 ..514.:::.6 . ::3: 5 1:.0 . 0995.65 7960.65: .3 . /315 6 :0663 6:: 410.:.0.: :0 .5.32.6 .3..6 .9 93 679.5 .65:  00 ...32   519 .9::  1:..32.1:..96 679.5 90::   .9..  : 3.: .. : 653 360.54::65: .51.. . .3 .  ..5. 4. 5 .. 06503:65 ..5.65169.9969:4 .55 90: ..33:519.336.69 .51 . ./3.945.:::.3 5: '9. 064/.9..51 5..51 .0.6:.  6.6... .50. . . :..4/933. 6 .:5.065.1:5  .  :7769. .6./1..3.:. 769 19.:79507..39.:..65  .3.9:..: 644.5:.9511.1.7..&09..64..6.:64765145..5 964 .6.5.:. #9:15.69.519.99. 0.6064/. #9:15..99:.6:751696/3.93.50(65*     .:65 .#9:15..65:/0..516 6569.3:690:6..69../3/659:: .:..: 963 .1..6.6:75151: 4.5621.:6776:1. 69.69..:3:0..0..#9:15.:.9   % !6  793  5..3:690:  314765145.:...36./30... 6946:.76964765145..7   #967655.51#.9.979::36947313/659::  &0651 : .9 4.691  9..356% 9:.979507.6.5.6476515. 5. 656.30.1::.3.473 5 : 5 .96   .:5::.9:.30. 3.:   '7967655.:989:. ..   5 ..#9:15..5 ..   '91 : .653.  165 :6 631 7910 7/30 5.......964 47345. 1 5:0.. 5.517915.   #"!&   598 &%   ($.5.39  31 '.::9 633 ... :/:.590:.6 :751 .69.37.:/69. #9:15.465.6 70.45.9 . 3.0.769610. . .5.:65*    '10.6.9.1 5 .4.9..    ..3:. /1.7960.98:.5:.0.94.05.: .45.4. : &7. . #3775:   65  %65.:   5..6336598:.65...:7696 10.. 90: 6 10.9.9:. 17.3 8:.75.65.3 0.316 .3 5. 7.98:..9. 9.51   .. 90: .  0 : 06.:   .::   ".5. 0.9.65 : .96596545..3 8:.99.:1 .33.69.93:.3 ...3 8:.0. ..11 .:.. ..3 8:.506 ..779679. . 50::. 065:. 67769... 7.:.7967990:6.5.9.:. 065:..3933:.39.65 / 90:1.:   ..65.9.90:610.3%:690: .6 101 . 065:.590:... 0. 6 ..91:69.:5 ..506.. .9064731.:..: 6 .:. ..39653.65   ..59 &09.3  %  !6  0   5.51.  % !6     5. 3: 46..769610.5  5:. .9.9  .50    58:.3:50. 0.::9. .7.:/69.9 6 . 6 ..3 / ..5 .779679.0.45.... .0.:69. 065:. 6 9:70.::1 765 5 6919 .51 :/:.3 ..93:..6998:.51   . 79:65. 73..51 ..75. 79:65.65.469.9:..3 . 69 .5 5.50. 0.6:. 4.:    31%:76515. "7565  .:5 . .5...1 .43  .3 9 5 065:. 0.5*     .:.: 769 6 10.. 67769.3 9 : 73.9.9. 9.65..5.65.0.51 :/:..65   .3:.:69.1:.65.. 5. 50.5.50(.51!. .65:6065:...65 / 7. 6.: 9 ..45.5 7.:.659:9.45.6.93.656.6:56.94 ..::57..6: ...9.359 57.51'.3.6.94.3. ...3" .56.99:76515.5 .6.3591/.:.69.03...65.65.01651. :0651  .94.51 :/:.45.68:.6 / 3.3..:  %:76515.15..3 %:76515..6:.69.31 .::545.51:/:. 9:3. .65     .9. : 3.. 57..31.. "    ' 9. 98:. 5 ..3.6557.:.5 036.5 190.: 0.6.6 99..:.#9:15.6  699.   9 : . 7.: /0.... .:.15..964 ..6 9..7.: 6 57.3.45.:5::.:7.94./:5..3.7.:765.9.45..9 .3.. 1:7..3 .15.3:46.3.:9 . 79:65..1 5...99:76515.6 .9.69.5.1/ .65..6:56.:    %:76515...9.5/0.51 57.7765.5 .51.065:..50. "    5  7..7.5 6 .45.:9::1.65    "5.:65   !.657.5 .45.79:..::545..: 56.7765... .956.659: 9.: ..67769.:73. 65:85.94. ..:6553::..657.4.7.6  699..::. .:.065:.3.3.: 4. 190.91 .98:.25. ..3 .::41 / 57.65.: 69.    %:76515.:    .3 :: .65:.0.659 16: 56.65660 %:76515..3 .:..4.3::5.3:.  9:76515.::545.5 964 95.:    %:76515.7..51.6 .65660  #. 3.:657.:::1/. 5 .9.6565365:.45.9:63.3 ..45.65: . ::..5 0.94.: 0.7765.579611.   9 9. 0.50 ..65: .659 : 653 .: .93 7.3. #.3:6065.:.00691.51 : .6.11    57.1 5.5.5.94.5.:.065:.::545.65  .::47.6/190.:0.5 6 .94.6 : .6:.8.3" .659 964 ..3 5..65.7765.67.51 .15. . 65:....51'.3::5.644::656530.56009. 79:65.659:9..5 90. 9:.7765.65903...:.:6.:.: 69. .:31.67..659:79:65.9.360::.: : .94 ..::545.: ..91.94.51'..6 57.::545..."09 15.3.: .3" ..6.:. 76:.6 699.::51 7.65916:56.6:.659..: ..90    6969 .7765.3&90.::41605 ..9.3 9:76515..979:5..33.:5.: 56 0..5..514.69 6 .. 7.69  .644::65: 0.65. .3 .:65.5069..:0. .::47.65.:659::1.94.6.065:.3.065:.6 699..9..95: 65 .:..7765.33 5..51.9 16: 7.9 69 56.:03.. ..00691.51 69..6998:..6 06473..4..:79:.659. 69 '..94.: .9:. 6 .. 59 .51.. 3.3.4.3/.5 6 .93:..65931..9 .::56.. 9.77..94 ./3610..5.94 065.065:.::545.4.651 ..45.45. 6 .:..3.94..69..659   .945...65.97 699:63.51 .9065:. 065:.:1 / 7. 9:63.:9::1.51.:5 .65 6 ..65.15. 065:.9.93:..93:.065:.6   &%   793     5 .15... 4:.51.::545.96.:15.90:6:6511:09.3.659:/3...065:.. 43.65.51 10:65: 4.3:46. 0 :2: .7. 67769.65 .:65 5 : 31 9 7.3./:1.3.: 064731 .41 964 .5 .7765. 657.6 9:63 76.969:  315. 7.3 796/34:    7..45.465.4. 7.. 6 35 6 .51::07.45..65:4.31.59:631 5 275.050.: 6 796019 .51..1 5. 5: 65 .69.0.65..: ./7.556.964.659 .6565365:.   5.9:.1.969:4.:1 / 7. .::1765  '9:5616/. 989: .0. 065:.5.97657.50:669545.:1/7.659 : 6 7.:1 ..945..45.: 6 57.:0. ...5:..65:.0.:  69 595.0650.0:56.6 699.6  699.93 .659: 9.3.65 49: 790:3 9.: 9..:70..0.61..79:5.3::9. 065:..3.9469 . 1.4 ./..556.5.:.655.65. 6 ..050.6973..7765.67. .:69.65.9: .3::65. .07. 03.3 ::9.3:::..3.. :6 .65660:9:631 .1:09.5..:65 #...6599.330.50 .4 :0.0.5:: 79611 5 . 98:.19:.0. 1.65931.69769  6969  .:69.369.969: 53::.69.. 3..3 ::.0. 6 .065:.7765...33 .:1 .65..65.3   #.45....65 9.50.02 :0 06509. 6 9.655...3::4.516/..15: ./7.: .:56.7.9.:1. 3.5/9: ..15.69:63:::0 94./1.0647.1 / .3 69 51 065:..659:3./61 9.65.6: .39. .9.6.5.51 '.3.59:63.%:76515.51..:56.7765..069.:69065....65.3 :: 9.65 /69 .51'.94 ..51 9:63 .94 ..:1.: :.5:.65.5.5.5.. . 065:.976*     ..3..:1/7. 065:.::47.945 5. :: 9..6: .65..30.. 06530.::.5 5.9.6. 6 .969: : .:.33:.7.73.  0 : .3::/69.::47. 7/30   ' 3..6.9451 3...:.15.:6.5:. 190./ 065:191 65 .: 73695 9 .6516:56..65. ..:1   9 ..3 :: 9. 1. 79:5066776:5:69065..9.... 67769.38:. "  5 ..65.659 8:.45.33/6901.  . 0647.:657..6 57.65 . 4769.65. '.56.30.: .5.3065.67769.94 .0.68:.34.. 8:.:   6969  7/30 5.3.945: .6 7. .65.065:..5.50(.065:. .65: ".945.6 .5..73.65..94..659:9.93.:69.1 5.:69065.656 60  53:: .9 .:.:69065..3.659    5 .90  69 . .5 065:..65 .: 69...69  69 ...93:.65..   57. .: 6 .:..5 560:1 /0.. 06510.4    6969 ..969:0:/6..7765. 5 .15:/69. .9:.    5.. :.3  % !6   0   5.51.9.: ..37960::  &7....3.65   5.51 795.3 1.4455.5 5065:.1 .65 4/9. #3775:   65  %65.1 :. 4. :..51 .3 .65.:790:3.51 6..9. ...51 4.  :631 .3065.. .51%3.7.65.. 76./10. &..: 69.779679.0  065:.50:0650951. 69 796/.059 : :..6 9:63 ."7565 .51%3.:47345..969:4:..969:5635.  % !6     5..: '065.609.3 8:.556..9 .9.: . 5 6919 .5 . .51. . 659:7 69 3..0.65..659:.065:.::.: :.9 ....:..51.: 0. / ::07.9 6 ../3..65:./5190.9::9:631/.9 967. 065..0.969:.50     .96596545...656.0./3 6 10.. 76::::: :05.  .69.1 ...51: 6 .&.7.0.5:.5./:00::33 4.:     5.9. 5.5   5:.65 6 &.39:63..35.3 065..59..3%:690: .6 47345..6 065:19 . 90: 610.51.65 .3:6/:.4:..65: 79345.065:.!6   6..1...5.331 5065:.5 9 .0...50(.0.9: .59 &09..: 69 ./..65.3 :.5 .: 3.9:631::  .0.969:   31  69.19: 065:850: 6 .65 .5156:#673:%.0.51!.:/5 5. .5%3:.0506530.6 46.:/5.065.3..5.5...5 .39...6579:5...47345.3 065..1.65 .7. 7/30 164..9:.. /96.5*     #.79:5.51976910.469.:63.0.9 5.0. ..9.9:2565.5155..77961  65.316   .4: . 3.3 9:690:   6969  5 ....: .969  : . 69545.5114.. 0..51.945.659: :. .:.03.75..:.65: 69 0. 3.5 653 101 .969::  56.656.5163.779679..51 :.::   ' .9:.647345.:.3 95 ...  65 ..: 0.6 #% .:0.5%3: .:.75.5796:65:6%7/300. /3065..33  7.#%. .  5:.:1. /61 :6.6504/9  .: 69.6.:0. . 93 79.#9:15.41.6 / 5651 56 3659 :.5. .4 9.069.:.5.6 565 .779679..5 .476::/3..:56..#9:15.69:63    ':..65:..339:63.51.:2 65 69 /69 5      69  65 /9.:2: .6.1...140  31 .:.56365979:5. . .65 6 .:1::631/.0. #9:15.53.:.0.:./.1.::/69::4.5 5..65065:191466.51:.9     .5..331/9:76515..90.: 5:.:650. #% 964 .69. /064 .6..536:.69644::6565 65:. 06473.0..:. 0644::65:692 '#%:/4..140 69 1.69.3%694* ..:33..#%(#97. "919 !6    "  !6    0 .132::75.: :0   3.6 06473.659 .51.5.14694.:651 .. #% .1   519  "  !6    . .140 .9:70.:/064 466.25.3360.659   ..65.5..: . ::1 0.6:.9 &/:85. 7.9..:.: ..:9..0.65.1 69 / 7.0.:.:0.69. 941 :50 ..0.5:.:.1 ..51.  ': .969:/0.4 69 .:0. ::: 5631 .93  796/.:9064451.. .0.65 : .. .9    5.65.511 .51.35::6.0.:/064466.65:065:191466. 3/651..3..50(65.#%631:473/5..:765.:.915.9.3769  65.5..9.5.39.65:.795..794::/3:067610.5116:56. &%    5...69:..3:  !. .1:6967565 0:15.9 6.0..:.006473 .1.941.5.5. .796/./3:1.59359..:. :.33 9:63 ...73/:0.:6336:   65:195.1.656.. 0..69.466..6.. .95 96..9::43.79643.3: &%   .9.: 6.706 .. :.9.5. 9:63.335169565.: 1:7.:6 .90  (!6065*     &631 ..09.:.5:.: .69...3.6.66.9.51 :631 .65  59:. 7.5 644::656530.916... 0.140 .65/69.51...6.:9466..5.6 .: 6 .3.469596345..706.3'9.69.33.5 69769...93:.39.969 / 1:4::1   69  .140 '::9.63156.9:76515.9.5.:7.. 69.8:.50 56.779679.0.." ...0.9!9:5069:.3.%:*     '7.3375151..3069.4: .51.1 :........!.:/. .6.:.659:..51.:.65: :.0.51.9.9:.0..9:76515..14.25/ 5.:03.65 6 1:4::. 5 . .5 50  69.65   31!.3 69 .5.656....613365.79676:1.5:1.1 /064 466.:.65 %  .3.:0.1/5315.255..39.64.5..14::65679. .56336 !9:55/69.60644.677.:7.50 6.45.7636%65.::. . .5.19.. 653 6 .93/569.902165..0.517.945. 1.:.10.656..:31/79.:/69..369.: .39.73/:0. :: .49. '.5::..5:.1/595191466.:3:0.6&.5.5..493/0.73/:0.14007.!.65110:65 ..650. 507.   5'.6.9.65/69 . 6955 795073 : .:1/.0.:6.. 507.140.5179601.7.6.51 ..0065.:.945..:. : 69.0... .51. 0699169:6769.   5.659: :.51:631/ .39.3 :/1:65:  .5.3:. .5. &631.33/695 1:9:./6.. :0 73/:0.1 /5 31 .006473  ..51.51. 49 11.0.5169.5.9.1.9.9653796::. 5:....0.:09.9 .93/1. 79676:..6/58915.556.:.51510.506.0...:/69:0..65 9.3.55 . 065:.53 69 :..6:6:..51 .6..:..65  .9969:63156.: 09..6 . ..3:6./6.65 : .9.: 9:76515.9 /9.:/5./:.:69...3.:6317.6 31 .103556.: "  &% &7.. 7.:5379603.5 .41.1 .6/.0.:69.1466.. 5 79650 79603.. 79..5 964 5.417965007.. 9:76515.7765.6..679694.6 /95 .007.47. .:1.9:. ...6. .3  06515.:.5 5.9.3. /651.3.50.5. ..3..001. .5..596:79015.9: 6 763.50(5.979.:'9/5.:4.: :6 79676: : ..06473.007.65   :4::...: 95 ./533.65.:60..51.:..35 :.0644::656.6 . ..96946.9 0. ..7961..140 65. .69023::3. 9 0: 69 797. ..:  .6#.3 6 . 4:0   %:76515.: 69.3 98:.:  .5... 4.: :/4::65 33 09.65. 33 ..65 ..  511 33.006473 .. .656.::3.656 :0 965  69 ./3 ::.5.9.0:.335: . 5..&. .6*     "5 .   #9:15.0.7.3.. 5 .65: .01/ .59 .51 635.94:  /3.13 . 46/ .51   .5 .51 6.11 .765:  03/:  :.9 1.3.9966  .765: .941 .::. 736::  9. .9/33659 .3.3#630.25.0.5. ...941 690:6.!.99.6/9.65 6 &..65!6 103..9..::.5...5 ::1#9603..99:.65: 9 31 /69 .:.9:6...6 9/33655.47.3.!.35 .9.7.65..7.:. 103....3"919!6 190.3 7..3%65 .6 .5.3..7.65..5179646.3313.9/33655.3::.9.:6:9. 369.4. 6 %/3365 / #9:15.3 ..6:779::.#3775!.51.1   50  :9.3.9 0. & .19:.5..3%65 &32:::159.#3775:.0.95.65. 3313 0...51 ..99.0. .103.69657..3. .9.9966 .69/3365/#9:15.7.3:: .3.::.65:.369..5.656:.99:.: .1/9.96   3133..  . .5 56 /.. &50.99:.99.9966 .0.  #9:15.3 65 .5.96  .5153.5.3313 0.1 / 9.0.3:: .7.51 ..: .3. .: .5.:: /6. .: 0.  5.:.0:65 #9 &%  .0 15: .9 59..7.9.94.4.:70079:65:506550..:103. 609: . 6 . 4.:.69/3365: /5 :1 / .69.6 3. &09.. /5 95191466.99:..99.65 6 . 30.1.3..99.96 ..3::.7633:593.56 . 0..5.: 1:6919 ...9.99:. .9:63. ..50: .6.  ..: .5..4.99:..51645::6:30.65.:..3 5:." ::1.   0069153  .. 9651  .. #9603..51 065:65 065:195. ..: 9:63.0. ..65: .65!6   .4..9 6 :. ..: :631/3.465 6... 103.6..7795:65:.65 .. :.51.65.51 43.65 7.65.9: 09.9.50( 36*     506550.5..55.65796/..996669191. 6 9/3365 5 . 06510.9.6 :.6 47345./3  ' 065:. .5. 7.3351 / &.69:.. : ..65: ..: ::1.03. .140 :..656.659:03.3:: ..659:.3. 6 .90.9 .5..2:. 5:.9/3365 :.65: . 7633: 65 .96919.53.9:  .. 569045.:  .79603..67. 1.30..65 5 / 065:.140 /0.:  160. 6 796.55.33 . 7.365.069545..3 065.9:563659..::.5.9 /55621 9   .65 .: .3 9164: 6 :70 ..96335 065:.55..6:.:65. .: /5 95191 466.51: .::563365376:..51 6 .:95.765.51 165 . 7633: .9:631.! 96. .51 .    30.5 69769.1 /5 31 .50676:.45.7673:51.6 .69 59./.0. 1.65: '6:.:1. 1::45..966914609..51 /..95:  69 93:  . ..32:796.5:051..5...65.:   &50 ...1600.3 69.6 6943.: 63.65 6 1....5 /50 ..7.69.30. 191 ..: . . 6 9164 6 79::65   ' &630. .69.:.0631 3309677.65 6 10.65 9991 :700.3 .65  .9964   &.:0..: 4730...51 .31 %:63. .9.51 065.3065.17.65:65. 9#.63156...65 .3 795073:  7907..930. 51.51 .56 .9..39164679::65.0.9.65::.9:63..9 .9 65 . :4/630 50..65 '6315 67961030.3:6.9.9.969:. .:2.10... ..0.65.. .: . 15 .    30.45.65 .39.:.6/9:631 %:63   31  3 ..969 .:..65:   5.. .5.76335: . 79:: .0. 5&..656. ::.63..515.6.9:.:.5 69769..969: .&8:.9315064465/.65 6 ::: 765 0 ...656..:21 ..9: .31. ':.#3775: # 31.0644.1 / 7.59 &09.5   5.9.3 :. ..#9:15.51.5: . 49 5015.:::.:5:  .65604.25 .1.7.   %  !6            5.50: ..55.::9. 59.: :0.469.33.::9.5 . #3775:   65  %65.: 0..65.31.0..51!.   15.65:  5..79:656.9 69.50     ::9.:/50.5.51 ..65. 59 ..0..4..9 6 ..9..9:.96.31 9..9363.0.79:65.31 9.33 0.: /5 151 ..515 69 360: :.9:.7.51 6933 ::.19:5::0:. 5 . 9:3.0.: . 8:.... 69545.3  .. 190. 065:. 59 .: . 0. 79:65....065. 7. .9:.9.5:. .5*     .51:/:. 60.5 190..51 ....3#630 #!# 53.65 .9.: .4.51  31     ..5:1 964 49 5.3 .:.5563. "7565  .3..9..65..:.9:. /6515 1..3:.3%:690: ..51569360::.65 6 3756 3..35..11..06509..6.5. .9:.506.35.4.5 :: . 9.655.3351 '..975:.6 .9.. .69196.6.3.. 6 .:.5. .: ..5..3.5:.:.:3 .3:.: .3 8:.316   ..50(#.6 7631. 5 .: 1:.6535.65 /69...515:./6.51 ..65   "   % !6  .4:.945.3  %  !6  0   5.! 96.: :0 .: ::.173645..95:.51 :/:.51 .   . 5 .6.30.9:3.9:.5  5:.#3775!.5.51 69 33 ::.6. 9:3..9 59.5/.9..0..: 569045.8:.3:.5..569045.79:5.7..656:. .10.5.0646. 069.:::.51. 6 . 06510.#3775 ...65 5631  69 .33::079:65.:.65 6 103.3360.1 .65 6 769   &7.3 69.5  .9 ::.. 6.65:.9...5 :.75.969:.:69065.7. .. 1751: 69 3345. 6 69545.659: . . 109  .50   ..1 / ..9:..35.3 5.6.96596545.: ... : 56 6...9967:.:360::. 93 6 3.6 7631 ./6. 964 ..#794.5469 3516/.65/.6959.:..38:.3.9.630.0...:.55.6..65%:63   315.::9.95:/3.93 63..9 :56.51565.::.:0.5156.63.9651.59 . 65:.:165..9.51 ./655.0::..7.5069:.:1..51 '49560.65  69  .:.93.7.5155.679:9.6036.13.963:..651.65   7..#.4.3.:  '::.9 . # .3319:765:/3..51..5156.91/6..::5:7769. :.6659. ./..9/.51.. 3.:05..: 103.96763: '&630.9:..91:.#6.3:. 1 .45.#.9:. 0.35.51:/:.:...65   %3 ..: 51..9:63.:..31.679:5.3 7976: 0  519 &0.5.5.:700.:   .65 6 .. 331 3. %3: 6 69.0. .. #3775:   65  %65.: 56.02698:.0.: 79015. 3.:5. 0 16: 56. 33 :.: 7.65:.3:::01:9.501 065:./563.9.13.569045.55:6..3 .6 ..9 6 7..4:..514.:63. ..655.50 6 .69..659 ...465.. ..6 3.050.469.5.: .515 .  9  9. 7. 6 .53 33 56.99:..:56..0. 4769.8:.9.:1..: .3 :. 7..173645.#.346:...5156.0....963: !.9.9.1/.1/ .3:/1:65695:. 54.50(.51 . ..:   6969  /0.6 93.945. 98945.: 4.7....: 4:.   %  !6         5...9.#7960.3..:/5.31../4. 93: 65 :.65169545.5#377514609.51 0.:::. # .9.:596::.2 065.6965 ..659 .. 8:...9:96:5:: 563.331.04.6 1736 .65:950796/34 .5:0515.3 :50.:: .. 569045.0.6 .145:.. 6 .656.465. :00::33 :.5055. #9:15.56.6.69.:. 4..51.6.0.50 .9./3 .35..659 9 .0 .0:.1 .9 : 5 .#.1/.3.:3/9.5  50    565.469.33796793.: . :..3/9. .1 5 36:/.169.9 .0..:1 .3:: 63506009:5509.51 3.   ' :.:9/9.6.6.6 .7/3051:.36595 !6.9... ..969: 9.5   .9033/9.. .  3 :703...6 ./47. 679.65 . 9.9..350.: 0. 6 :.9 .51 .: 44/9:    ../. 190.0.3.476 516/..:1 5 .3361  31     #..65.....316   .7...:  .515 5 7.3 065. #3775 .76::::.... . 6 ..9 519 065:..556.64796. 09.3../.:44/9:...: 32: .7.9   5..::91694.79:65.515 165:6 . 7673  ':  5 .43 .95: .: ::.:65..0. :: 56  9. 065:850 6 .31.316   ...6.167.1:09. 69. .9.  .0 . 69. 7. .:.   %  !6          :0::.1.: 6 796019   5 .: .3.51.: . :..1 .95: .66 . 4.5*     65:195.51 79:65.5.50 .../3:1 ..6 .  .653:.9. . . 9:3.5 3..515   &50 7.95:  5..5 :700590..... 6 3.:69...75.: ..6 98:.3.1/.6 065:..:  31. .:5631 556...:..969  /66: .. .4 796565045.56. 7/30  .7.360::.9 6 .. 511  565 6 . .. .65 :6: . #3775:   65  %65..69. 65..79:415956.3..9 6 .51 6919 ..1.658:.659:190..51 9:76515. 9: .94.56615.51 509.3.91:6.506.   .1 .. . :/3.:1 .5 694 6 59 .  : ..1.5:.65. 7...9 : :.6 :.3 59 .: . 6919 6 . #!# 5 3.51.16:56.6 9:63 .6 . 5.656:.::. ::: 9.516..7.169:64993.9 .5.0..6 09.165.65 6 . .9.9:..659 : .:0644.. .: 469..5 :: 6 .:.1.15 6 .96..763..:776:1 43.79:65.:..51 . /9: .9..9.173645.:69.2065..3:69.:.5 .9::.3. 98945. 51. .651..9:.63.9.6563.5.51 ....3.6..3:. 5.173645..05:.91:/9:1/.796::65.95:  6969 . 38:.9  1:/9:45.65:..7.: .1 5 36:/..:/0..2 .9 4..659599991.3.:.33. ..9 /96.3319.515 :631 56.9065.0..33591/.331. 7.6"9. 06507. 5 / 4.31:/9:45.1 5  5 .9:..9: 6 .1.:0..6:0790...95631 036:9. 0. 50.. #  79 :8.659..:: :.65. ....25  /.5 :: 9  :50 :..69/60.515 : ..: 7. 79.335 4769.56.9 . 6/0.65697. 6 7/30 51: ./579:65..51956065:.969  0 631 4.: 5 :64 0.6   &.7. .: .8:.3 .:6.:..656.9: 69 6..65 .0.131465:../7.0.9:07.92.3 065095 5 065:..51: 964 "9.:369.67/3051:/. 3.79:65.179679.45.5:  .:6319.135.: 653 4. :  69 631 /  190.9 5 35 .9.7907..: :70. .506:..:6.9 ::.: ../:5.5.65.3361.515:.:.:.9..656.5.65.6/.35 4.55:.51 6   :  :..65..0. 5 065:. /... :70.. #9650 /6.: .: 6 #     #.533.. / 0650951 0. ..60:165. . 7/30 5. '65394. 98945..0.:.1 .5 50   69.0..0.  .679: .3 .. /0.65 : . .0.335631    5 .0.655:.3 6 5/695 36.3   .:::.679.:   ' 9:..659 : :.3 3.1 / .9 ::.515 : .7.33 : 5 ..9...331:0. 06473. 1/.6.:...79.29 .1 / .93 1751: 69 3345.659  5 .: .5 33.65 6 ::: 765 0 .6:.:   ' 5.38:.6.65  ...:. :6 3....9.3 56 360: :.51/.065.19:3 . 56.9.556.:56.0. 79:5...1065.:69 .51 065:85..0.7..5.975: . .65 6 103.65:  .06509.51  5 ..25.99  &   1 1(*   515.9691: 7./3  .: /5 :65     7.../:63.0646...3 :6 7/30 465   5  69  56 :0 53.:4739. 065:...19:5::0:.65.:. 069..969:.::9.51"9.0.. .8:. 0.: 6 .3130.0.5.65. 5:.65.: .9 0.659 631 :646 / ..3 :7515 .:65.3560..5.7.659.9  7.#9650.5:.  . #3775: 50  &.5.9..5 &% . 6 . ..:.79:65.65: .5. .1:/9:45.:5631..: 69 935 5 .779679.:.515 .1 / .6 9.751.4 ...50(%6496*       .0..    5.65: .....7..4 .63.9 : 141 .6915.: 796:65:   ' 5..4:. 0.3..47.65 50 .65:9.5:.3 :: 5 . ....563.65::60..68:. #3775: '#  .3 99514 .6.:..659: .7..465.6535..656. 3365 #:6: .56.67/3051: /..:......65: ...: .309!6 ...16 .  (*.65: 195 .55.31:6906.569 :7515 769   ': ..51 96.&.335 69 . 69.309: .5 #9:15.331065..3.:.73.31.959..1 644::6565 30.7.9:765.:46 .65: .: ...9: 3.906: 5 : 3:3.3361.:56.6.9:50 . : :.: 569045. ...7.6 .9..379:15.6..31.9:5.565.9:.10.65:.0.0..: .59.5.7..9: 2565 .969 56....5.31.65  ': .506. #3775:  50    644::65 65 30..309!6 0.7.10.5:.69..6 ..692  50  8:.65.. 065:.9 ...99 6..569:75157696659::   . 0.659:..41 7.0.505.5.33.656 3365#:6:69.93.651 . 905.:. 6 ..:.695..1:/9:45.#9:15. ..56.51   !.96:05.65.9::.3036.313.: 95 7.31..9935:./5651 765.7.515.659'#1156.:.09:..1.3751.9.90.3.:67/30465:05.:..659: '3064450.../065.1156.7..590:/659::6..:7961569.65:   &%(* '69.9:3...151:   5:...065:.6/95. .3 109: 79643..9:.:.6 0.779679.6  9 .: . "45/: 30.....7976:60.65 61  9895 9. 0..36:. 6 &0.99:.51 . 0.67/3051: 4.7.55065:..56096...6#9:15.69.9:.967/3051:/.3:3...190..695: 6 .656.7.::..7/3051:.794:1.659:.94.9:../3:1.96. 644::65 65 30.3. .1/65633::. ..779679. 5...5.9 5&..55...5 7961 '3064450.65.65:796793/96...69..69.51.65  6  #  3   6.695: 6 . :.3:65 :.656:051:  '/9.47345.7.5..:7976:: #9:15.. 0. .51 96./51:/9:15.#377565:.7730..:. 92.51 36745..956919.6 .1 69 .4.9:56150 . 69. 76::::.65 ..569:7515769 '#%.065:./659:: 5. 3:3..3 75:: .779679.9:.779679. 65:.:.9 6 #/30 692:  .779679. .9.33617.3.9..  5 .51 0644.::5631.:56 .91 5.65 .90...7.0.:.065:.!6   5:6.6 / :6901 964 .3. #9:15.       ' .6..7.51 . 9.. .#9:15.90.1/.9.5 #  69 .: 56. .6/95:0.6 &5.25 964 51: 5. . 465 4.5 469 .65   .:.::55 .659   ' 69.91 ...5 7.98:..65  6  "  !6    .: .91 .65 .5.5. 6 .779679.:.4511 /  "  !6     519 &0. 065:..9..:091. 99: .65 .945..92..51. %.691/..61/.0.6.65 ::60.51 %79:5.... 5..7.:9.69:  ' 51: :1 69 ..51 :02 3.065.0.. 79679.65.:0695.51 . #9:15. . / 7..9::. 3....5 ...&.:0.1: .5:6.9:5690:/659::6.9:./56933/:15.779679.65. 6 # 43365 : ..5. 33. :6: .13.67/3051:/4.0 .77.45. 51: 6 . #% 9 . 69 ..3   &09. 103. ../315   64556.3365 9:76515. 60...691:964..1 6. 7961: 69 9.95. %. 7..: 679.65.1:. .. 6:: 6 659:: #3775 65:.7. .6 /.68:.65  97..6 .5 . 56 7/30 51:.6%7/300.90.: .. 56.53  5 5..515.: ..::.    .9 :.!6   ..7.65. 6 .65  50    45   51 5 #. :5:  .0.5 79...1 / 3. "0 6 .65..63. 103.03..:0.: 965 5 .659 .51 479645..7/304./9.65 6 . 69769.5 ..:9  3 .9 .6 /95 .. :...68:.3. 65:. 5.5:..65:69.65 5 : 0.:09. 3:3.91 5.779679.9.0.64.: .:.. 796. 30..9 .#9:15.#%9.65 4..6 ...6 /.3.65 69 550.56 11 56. "0 6 . .65 79:09/1 / .7.965  .51 7.69.09. '9.. 065.9  . . 6 19 96.659.56 .90.465.7..3:6515::6%7/300.9.1:/9:45.65.:50(6*5.5 ..: /5 11 .5..: .: 065.0..3 069..33   5 .656.65 .6..9.5 .6/:15.3: 6 /6.. .511 69 .6 " !6  .   . 6 ..69: .0.6 ..515.. .5:4.065:.0069...9::..7.0.35. 69... 9.5.69.:../3:.769...5 190.9::.5.036.969 56....:6....6.65    5.:.65  69.3. &%    5 .:..979:5..55.7.9 691:  7..3: !. 65:.9.79:65.9:.: ::.9 /.9/55690:/659::6.50(65.51:/:.5:.   5 6.....68:. !695:73..169591/ . 9:3.0.0.5.:...:.656.#%5:0.33:.9 4:.69.09.7.65  65.6//5..31.:769.37.:.5.33/5.9 .51 .5.6594:.556.0.6101..:..9.659 79:4.. 6 .  ..15:16:7.9..145.9::1.:.: ..6590. 5......5:951:79:.51.51 69 33 ::.3:: ..7...90:6.35.:.... .5 69:7515769 7..9:.5.0./...3:6.64. 59 .9.2:0.3361.4:./.65.9.../:.: 569045...9.6.335.6&0.3. :656./3065.0.659: . 7063..533.65 6 .656./3 065..15.:0.65:6.::...9:6.:56315.3769..7730. 9: 6 .:/5151.. 3.9.5.069.51 03..65 . 7.47345.659:..3 69 4455.9: .65.7730.33.9.65:631.: .0..5103..9:56:.1 .5.0.1569.506:703.9:6.1:7.0.35.393.676::::.503.773169 ::1 '!%::...7.51 .65 6 .96 .6 9:63 ./59.656.9. 09.5610.4..969: 69 .656909.4  0 ... !%4.9:.56.0.7.0.0.5. 7.50:. 69..9 : ./3065.769 610. 90691:. .51.::.5.9.331.964.4:6.51:7.0.:.0. 56 :. ..35..51 .7967990:6.9. 51:7.:04.0.33 7960::5 ..9:.3 3.::..51 :.:9651    ..5..3.:519. 63..: 69 ...0.60565.::9.:    31  964 .069.8:..69.4  31/.51/9.3.5.: .77.6976.: .30.609.556:. .7730..690.1 9.:98:.5.63..7.6 76::::. .%:*     .659: .93 331::..145:.6..:.969:.. 069..6.19:3.7.651.::5635763.31!%:70.33..36919:...19:3.9.6.:../9:631/. 6 . &/0.514...1:6967565:..7733.6 : . . .694...25065.::.969:/.38:..:.51 ./3065.51.515 0.656.50:...1.5106509.:15.9.15 6 .9.....65:67..:.65.: .0.:: .51.: .:/064466.65:.9 : 56 .50   :.9964. 9. 9:76515.3933:... .659:4469.93 31  519 ...: 794. .:56./51:4::1/.969:.:. 0647.9. 7..064465769.9. 4 / .9...51 / ..: 56 :.5 .6 93    49 .659: :91 56 965 / ..5   91 (65. 7.: ..79. .9 .4  .:9156965519 .:69.39. 93 65 ..0. ... 796565045..656.7796 .3319.0190..3.::69 33 4455.. :64 . .9 .. !% 69 ..969  . 4.651/....5.9 6 10. &09..9 56.9.51:56.9651.9.7. 47345.794.5.. 9  511 .65193.53::.19:0...596/563..659 ..969: '8:.30.659: .6:09..65 065.10.: 59 .691.9:6.94:6. 93.7730.659:65.7795:65 .337960::5..4 '7.9.65 .5...: .51 /69 .3.:.: .65 ::1 / .6679..6. 794. .:6.6679. 09.659: 11 56.:: 6 7.9/5 .659:.60.5..: : :473 65 6 690/35.77.9.: 519 6919: 964 .9:./5621 '9:56:65.369545..1 ... 9: 6 ..33. ..659:9/50.3 / .:4.7.9:76515.:/5.0..1 964 .9 .1.4   69965 964 .65..79:65065.7796 .45.3 0. !% 6 .9.:.51...465.:/5962169656:. .31 ...:. 8:..: 964 .969./8:.:1:4::165. 9:10.:69..3.65   5.:06636: 794.5 16045.33679.:/5151    ': 69.1569:54455.190...006473:169796941.0.6 76:::: .5../3 065.651 93.794.3..9.655...969: /0. . 0.3 :0 .5...7730.6:.56:2.:.794.5..76::/3.65 33 .9.. 7.6.65 6 . 631 .65:069. 5..47. 03.7.19.:..6.31..0.:6.:0663:.9.95 . 6 ..651..  .6 .65 . 69 4. 9: 6 .63. 5..659: .. &09.6 :..3. . .4...5 .9:6.656.659:8:.31. ..14::65. ..65...9 :473 :703.659: 76::::65 .: . 16: 56.656.51 5...15107.9 6 10.0. 9:76515.96 5 . 7.7.9. 7969 ..5 !% &% . 3..: 609:.51 1:. 9: 6 .:  .69545.65 : .51 .5 511.56.. 0.0.51 ..9 ..65 ..65 &0.0.6 5.. 7.556.9 6 10.51.5:: 5635.659:..31.7730..65989579..: .#0. ... 3.5519.659: :703. /..4. / 0..335156935.331 3. 7. .694. 794:: ..51 .659: ..66/../151..: '94:....9. 7...:6#  &09..6 :... !% 4.3.4 6 .1 964 . 93. 56.: .676::::.77961. 3. 9: 6 .:3356. 0. !%   ' 7.:.9 ':69.9 .:.33 .31 . !% 9 :65 .9  ..5.. 7./.7730./../3 065. !% 33 . .: ' .7796.6 .3 9.5. 6 ..5.65/0.#0..65 . .. 65:    .0.9763.969:.:./3065..0.:..%:*     .38:.. 79. 31  : .0./69..:65 : .906...:3356.:::17515.:: 95 .0.65:   ' 9.50.6 93 65 .6925:6. 69.65:... 6..3::07.30.0.969: : :.555:.765.90650951.3 8:.65 69 .0.335. 59./36/5101165 9651: 90651 / 3. 99: .656.64.0.0.9595.::/96...:7.:9/5..50:.9. .03.9.9 0 : .9 763.::.6:::0.7. 065.::6 0.3065:.9 .33 ..38:.65.997699:63.: .3::  ..9..99... 16: 56.5:.31  6969 . 763.6 .:164 56.0..763.   !9.656769:  ..3 8:.65 "503. 69. :..69069.069.65/5.65  .69 4./3  .::4 9:10... 4.3.5694.56.3 79676:.3..779679. 945.3 51 69 58:.51.38:..91 .3 17.556..5./3 ./3 .1 / ..6769:8.7673.6:.:65.65 7.39.65: 9:70... 79:09/1 8.::5631:.65   '  65:.65...   5.: 4.0.02 6 .99..96:17.9. 69 .::6 . 65:.9 .38:.5 /9.65: 99 .0.65 5 .:/513. 476::/3.9.4:3:.33 / :.69.9.67631. 6 .3:3.0.:/.331465:..45.9. 251 03.6.65.65  .91: 69 9:635 ..3.65 ..93 69 56510.763.655.9: 69 1:09.658:..65 6 .::.64/. 6 .3 065.6.69545.65 69 .. 69 . 34.95169. 763.50 69 5:.65: .  10. 9:70.31 0651.369:/0./9.75.0.65:651.65 .515690...5 5.  763.50: 6 69545.6 :.3/9..65 / .69 5.65:.3 769 5031: .: . 69 .6. 7.93 15.65  .99 (7*96455.55./3.: 0..9/9.: 69.6: 8:.69.9.65 519.::45.5..96956.169.3 7630 1.6. 1.9.03 &0 6.30644..0..763.690.45.9 .::06943.3.634..  69 5 9. 7.5. 65:.51.51../3065:..945.069. 6 10.0 . 066915.0.51 5 :0 369 069.0.:15.50./3:1 / 3.763.656.65: .61.0955.03..9.3 9 / 79615 .6 ..9:10.1950.69.::6: .03.6 / 1011 / .8..: . 3.6/.5 ..5. . 69 ..310:65. : 519.9.1.65. 7.51: .65:.45..0.: 4.0.9.0....:31 . ('* 10..65.5 :: : 03. 1 066915.65.50669545. 6 .50669545..65 65 .3 8:. 06507../: 6 1:09.5.1  : :.. 6 1015 ..65: 69 0651.:9.76.5053::. ':   .656:.3314.51 4.6563. /9.96:796565045.6..25 517515.  9.465.6 . /5 4.. 6 . 79::5 3.93..7.6 3.5 5:.6.65 ...02 69 0:: 6 9:10.:./3 :.763. / :..3 769 :. 476::/3.0.1 5 65 &794 69.65: 0  519 . 069. 7673 5 .9.30.65   : '.0.02 6 10.9 .03.3 1:09.945.6 0 33 1:09.:31.14...969:: 5635 9.6. 069. . 9:63.1. .38:.65/651.3.29 ..06. .9 :695 0./3.3 .9.964 43.33 1:069...51...:/5.: 6 :.. 9 . .04.65   856  .. 6 .763.65 .3:09. 190.9/.. .3..: . ..3/.6565.5.0365 !63::..7.:6.1 0.9966:56.. :50 .95631 .9 17. 3.  .7730.916465:./...31.69545. 6.03 . .65 .65 46909/9.963..3 8:.662.:/513.:: 31.156. 769 6 .: 7.6.5 6/:.  .6:.:65 6 76:.: .: 6..3.9:10.6..:4730.& &95 : .6 .. 3. 519.. 065:.0.6 .7906:694:0./3 5.469.51  .3 .6.5169"39  6.::164  6969 . .02690::69:10.06.69.3.10.6 :7769.:6957673 .069. ..3 589  69 .9 '6.316 .0.969:  5. 65:.5*     : . .65 .::65696:.65 :65.9.0. 8:.159.:91/./:61:09.9563.96 .69545..: 5:.63.:....9 .:00::3963.65:/58:.5 .5: & #673 #69  964&#673#69   31%:76515.. . / .5..5....65:4.33 :69 .69..38:. .7. 56.: /5 .51   .7...0.65   5 9 6. 3662 5..  65:.9 69 56..4.50(.9 3.1.6 134.65..31014.950. #9:15. 479615.65/7673769:/65110. :/0.1 0.69. 6 659:: 69 6 .69:76515. 9.769:90:15..6 79694 .. 69545.. 69.559/9.9./:61:09.:6567.36:.9.9. 69545.969  56.65.:/54.65:..5.90:69... #3775: 65 %65.79:5.65.06..51 96:: . 69 .3  . 56.390:6145.::...69545.:: ..1 .3 9  :. .3 9966 .:..: ..96.. 4.9 769 : 9.6./: 6 1:09.65..: ... 369.0.465.0. 5651/3.9:090:.9:.3.: 145.9.9:3..0.5.769..0..065:.65103.690..51 93..95.4511   ..5 69 ..65 34.9:690331:09.69:76515. .:.:/55.56   #9:15..511:76.33 69/..39.945. . 1.9:.45.9#3775:.856. .6563. 763...:69.65 . 6 69545..6..473.9:. 5763.9 .60.::47.0.4.651   9.91.6 10./:.0..796/34/56563.0..75.::33:6.3 8:..:: .65 . 856 69545.  65:.. 1.3 9:.1 ../3065.3/651. #9:150 6 #9:15.3 3667   5 00291 065.4..335065.964. 69..331 . 1. 0.6 79:9 .735.:.66949#9:15.65. : .. 90: 6 .9.6 190..6 10.65 /.5. 9.6 :/:.6 .43.9966 96 . 69 .50 6 .  % !6     5. 4.:15.:69.6 .:9365.:9..6 . 519 .64. 3756 7673 5 1..656..9..0 6 .955 .:00::3963. ... 796:65: 6 .  : 0..65  . .465..: 56 769 .0. :560. :. .65 56.0.03 ..::4/3.51 .. 79:150519./.976:3. 160.4.: ': 9:76515..3769:519&0.9.:47. .65   55 .51&#673#69:03.93565.9 :/0.:6 .6 10.:1 . 065:.69.7910. 769: 6 .65:/.51&0.76737696963.3/9.   &  : .569545.3.1964..3369..763.0.0.. 69 919:: 6 9.65/.3 9   &  79:5.1:65 ./3&0.9/9  .38:.9.3445..6569.95..: #9:15.556.590:67673769691646:70.. 065:.6 10.979.31:.796795.6:....65609.0..1165..9.51 ./.65 6 .659..317.0..9..65: '79507.51 91646. .151 .5.6  %  !6: . 0.65669545.65:   !13::.03 ':::32:0.69.:067679:15.92  6:7 :.38:..65 69..65:.50..511. 69545.160. 0.65.  &:..33..956763.3..3 .1.  65:.::..51.03 .9::1 .38:. &  : 5.6369545.38:.65:..93.3:6563.79650 . .67.3.....3 9  /. ' .656 9..656.964:.1.65&563:3.1 .5156..65. 0 #9:15.1.45.51. :. ..96:.06990.90:6.3:::69 9:63.65.6.3.06.0.0.5 #9:15..6.6:. : : 1:0. #9:15.0..5:. 5. 60 6 .65989.5&#673#69.. 9:5..9:3..:/0.. .10..50: 0 653 .: :.65069.9  &563:..3..37/30..95 .56:9..656.:. :00::65 6 .763...67.5796:65:5.:/53.3360.65   511  : ..9. 50(#56*     : . .6 0. . #9:15.  .519.33 6.: .90  5.941 690: .: 769 .9 644. 5. 9451 . /61 .:9:10..6 . 6 .#9:15.: .6 ..3..5..:9:10.3356.065:19.9....50(.3 69 0.3/.556..33 #:.07.0.5.6 1.45.9:.9 .0.:65 69 . : .:5.3356.065.945.#9:15.6.6 0.3.65 6 5065:.. 6 &0.334.945 .956:7..15::164 ':: 03.4.565.795..9769:633:.:.46:.941 690: .4..3 8:.: 0644. 065:./3. .656769: :.65  6919  5:.33030....69. 103.6:6.796015:...4..1:09.0 69 .9..65.: .336%.:69.5794::/3065:. .  0 : /.90:6.69:0069..331765.3. #9:15. ..:5.65.6 ::.50::..763. .31 . 4.696 13/9.941690: .3 .:/0.: .:56:./: . ..6. .6.6.9:: .. 9:10.93:65:.3 109  79603. : 145.95.::.:.65  31   :.:  6536. .91.: 504/5.9 .8:.9145.:90:1..9.9:56150.065:. 0904:70.3 145.25.65  6915.9.:1:0::65  9365 4 &%  .: / . 065:.9 6 .. /0.9:10.3 769 .3.656 ../36705.65   5 ..3:: 6350  5.653.0... #3775:   65  %65.65 5.1 /0.65.  % !6     5. 6.65:.17.. .160.9.93.9..65:58:.65  369 069....:: ' 79:5. 5 6919 .31.69.6. 69 :779:: 3. 10.. ..655:15:.3::6350 5.:.9769. :..31. .4.15. ..9.66993....: 0..9964.7../:61:09.65 6 .316 .9.90.9 . .6.65./:61:09.65:50.65.65/69103..965.69. .: 563::. .   '6 16/..65..: .3 796:65 65 03.65 / .6.656.6.51 .7733.65:03....: 9:10.3.50. . 79694.3.56..659. ::16469:/:.#9:15.5.5.9.:3 '69.6 63.619.0..1:1 .765.35. :1.9/9. ..0.3 15.  5.: 90:1 5 :0 .9:0769.33 .9:0.33.75.61:0.506..5.9 1.... .69:779::3..5:..65 ..: 69/6.59.49:. . .: .65. .3.:65699/3365 50::.1 9:10..6 . 90: 6 .03.65.65.5.:16:56.5   5.65:. .0.  79:15.6.5. .945.65 56.559 .516919:6369069.. 59.469.:65699/3365:/0.6:67.65695.: .9.65.68:.:69.6 065:19 .065:...45..4 7.51964..&79469.:0.69.9   31  5 .65. 0633.3069. 065:850: 6 .5..516.. 065:..::50.6. 9. :10:65:.5. 43.61. 765 .6.3:.00:15.6:7769..516926.:69.6569. ./0645461:.0. #9:15.6 795.95 5 451 .965 6 .39 69:.::9.. 51.9 065694.01 5 .610.9:..0945. .60...5 .65 .65.33/7915...331:09.330.0.95.:.65.7.03 &0...33/9.:  6969  9.63.3 6539:09.65   .651.33.65.6 065:19 ...941690: '9: 32: 56150.. 09.65:.67976:3:..1 .33: ..63. 79676:.  3...: 5065:.65.:90:15.69.. 369 069.65  6.   :6  6 :631 . 0.33 .5. &700.065:..65.65  /...:. 769 ...559065:.99..::5 0. 065099506.5 :794.6 795. /5 4/9.65 .9   5 .. 3.0./915..5*     6 369 069.45.9.:5.9390::.6:7769.65690.9.7.6.51 /515 3.3:3.3.:.50 6993.     5.50(9*    . .9.6/.91/.::.50 31  519 &794 69.0.    .9 !6  .&79469.5/. 903. 31.::..5 6 .649.:: 9 . 14 6 :05.5/.50.565  . 10.::9.5015#  69/6. 069.65:58:.::51.9 .069.*63365.  0..65:696919:6.. 7961 6 469..069..0.4.50 6993.  3.6/476:1:.9649..75.9065:1915/.   .1 165 / . ::75:65 6 .:: 5 0 .. 5 /.50. 4.   .3 109  79603.3 44/9:7 4.65  6919  5:...: .   .30..:.344/9:7 6. 6 .95 69 795073 3..4 69 ..: .945.656.40: 50  69.3110:65476::.::. 6 .0.007.5.77.. 3.3.659.9. / 46116999:1   .::    .6.4.677.5.50. 160.969.97/3045:..65: .9     .9:. 065:.::9.9 0.4769.1 /9.  5.3.51644::65651.   .:.95.50 69 5 1:65 4.51065:3:   .69.90..3.: . %:63.::. 69 .169: 6. 5 /.:: . 069..51   33 6.1:6505.65   945.9  69 ...75./3.65:63.65.6.1.3&90644::65 644::65 6530..36.4511 / .0.5/.9  1:/.9.3: &% . 6 .65.4/.9  44/9:..945.:55638:.69.1 6096947366.500.3 69 0....5 . .51..3. 6919  69 79:15.  1.656!64/9    (.675656.65  9:.::50.5.1:4::.50 / .::563510:65: 9:63. ..65  6915.6. :..511:/9::0:4:. 5. :.9..3 ..: .9065:.3 ..10.. 65:. 5  5 .3.:9.3 60: . .: 6 06475:.50945.3 &90 644::65  . 506550.6 :0... 644::65 65 1.:8   &%  (* . 65:.9:.. 6.3.65..    31  #..65:   :0.3 .3135.3 644::65: ..5 065. "4/1:4. 644::65 65 30..6564561/.69.51 .51 63.9./90 6 69 065:.51:7.65 56. 79:: 4. 065:..659 09.965.779679.65 6 9:. 9:79150 .65: : .65 .1 69 . 9.69545.9.: 65 .50(..5175150.473.559 . 015 ..3360.*     .:1 065."4/1:4.51..:  31:5:6515...6979:09/1/.9 .513/3. ..069.../79611 /3. 517515.51 63.   .16...769..30.3..3.5115.90. . 653 6 .51. 6 33 3/3.5:.  ..54:.5:6..3 1.513560.3360.33.51.. 51: ..3360.01.65.9  .6564 4.9 9:690: .4 : /. .5..517. :164.:4..963   ' 10.950..1:0.9 065:.9 679.0: .65:.65.656403. 69 06475:.5.9.:   "0 6 .65. "0 6 .65 /..50(#9:4.3 ::.5. 56..3 .65: .5.45.51..511:7.::0.9."4/1:4.::::.3069.65.. .:0.96..5175150.3.51 06330..6564':0.73.&79469. :   9.9 6 ..63 ..1:0.9 .6 .:1   565   9365   &%   793    5.65.5: 9164 964 6.691 / 3.51 .. 4.91:.5.99*     .::/69.656769:7650.. 6 .069:6.: .4.:   ' 476:.. "4/1:4.65:  .0....51 065:..951:989 . :70.51 .. .6564 .  6 .0.9.:79."4/1:4.51.03690945.9 .85699646  '9 3.7730.5.1.0.1569.:001      . "4/1:4.. 51..51 .51 56.094.5..1165.. 5631 .5 56.315./5:..5/5.3.93./   ..969    '7.50644..6   &%  (*  0.169.9. &79469.5. 1  .65.1.5 . :50 : 06473.69.. ..5 .169 ':::.. ..55:.: 99656: .650.. .6959./:61:09..:0644.94:.63..&630..65 .5:.: .. .9:. 0945.5.9.1 / .6569. "0 6 .1796:0. .:  .    5::.6 : 06473..:65%6.: ../3 . "4/1:4./69.51 2565395195.556.5:..696919 0.3 0.99.3.69:63.9.145.5.. 10:65:6.31. .51 476:1 ..00:: .:.3.:. 796:0. 4469.7.1   51 .0.5..350.5106503:65:63.0.65.650. . 0..1 .10.4615 0904:.9.: 5...56...3:    50  5 5 . .00691.65 6 09. 79:65 9.. 910.:644::65  69.51 5.334/61. #3..96 5. 1 790:3 69 79643. &794 69. .....5140:65  31 469.1 .1 : .  06950 .::::45..6 .5140:65:..9/5.3064731 ..01.677.55:.5 .069. .517965650.0.: 69769.514.:.6.5.  '796565045.6995195. .65 .51410:65:0.51.510:65:/.6/. 065. 3.69 /  .10:656.7996.50 56.1 5 .9/.9 691:  .145.4.5 /5 :. 0699:76515 75. 9651: 6 7150  79.145. .69.1 5 ...31..79611.50.9.0.0650 . &794 69.3.3. 469.9...511:...3.1 .1 10:65   5 6.69 .:5 .:.:.65. 9 069.: 9:10.991.99. 7961 69 190.65: 4469.5*     .06473.00:1 .511602.. 35.167.. ..: 06479:5 .5.:706:0050.:116:56. 69 60.5.. 9 9.65 / 9950 : .0945.1 .9  ../3.065:.65: 69 ..0...3 .94: 6 (.65 6 ..:64469.5 .0..3361  ./.33 .51 ..51.51. /5 . 145.9 69 796/.96 . :.65  5069769.6 / 5069769./9950.167.79031....651.5.50 .65 69 5:.65:6.: 6 ..069.3:6103.  . . . 6 .3 . . ..:669069. 4469.  56 6./365:. 4469..369069. 5694.656. (#...5110:65:6369069.93 . 345.: 4. 0651.9/9..3.3:6:. 6 150  /69 5.6. 06503:65: 6 3.50:06 #94:23 &% (*     :0::. 69545...0.3. 4.6961:0::656.65:.. 065:..15 .::...3 550. 6 4469.696919   2:  .0.51 .5 .61 .3.515:6..691:0::1 ..5:.3.9.69..515:6.::69.38.9503.559 5 0 .65:065:.50 ..69 6919 '. 796015 5 .1..5 .51410:65:06473.95..656.33:. .551:75:... 9950  0 : ...50: .10:65:65969..5106503:65:63..0.6 ..0..365    ': .   ': : 7.   '6 97.. 1.515:6.33/95191/.51 5  .9:3.../37.6 / 1:91 5 .: .00691.033.: 3..91.65  9.65 : 511 5:.93.0.:.51 . 10:65 69 69195 8:.99656::6 ..9. 9 069.. 0..31..9 (.65:906.65 6 .656 # 3 :631..45.51!.6.9* 03.:  '4469. .50:06   #94:23  69   3..964.691 519 &0.... 5069769.:/. .6.51 .65 56.9.5069769.34.65 / 9950 . ..945.3 103. .51 .6.510.51 064795:/3.656. .:   .3.65 5%699646 :79. 796:65: 6 &0.. :.51.01 .51.:1  .65.6 .550.515.3 6 .6 469. 0.9..10:65     .3351 10:65 : 56.63. 79::5..50.:3 ..9 5..:0647.514 10:65 .5..:.0.51 9:79150 5  9991 . .3361  653 .503:10.515 .3. .51410:65: ..31 0.07.556.:. .. 10.91 5 .7733.:. 369 069.. 69 86..0. 6 . 1 #3.9.6 .369. ..9 65 515: .476:1.3: &% 3 ( .306:3 13.:  9.5:/:.39. 3.0.696956.1  0 4:.9. 5.9 .6 .50.31..51 441..514 10:65   69 . 796:0..55.0.6 / .9 5 . .1.779679.65  . 4755 .51.65    # 3   65 .::..:.  "3.30.6 &% 3  :.0./3 .145:.5610:65:.5106503:65:..03. 1:76:.796:0..6 :.34.3.916*     . 1 .6 .:.145:.75.. .:: ..:7961 .5069..    5 9. .: 0.65 5 .  065550.65. "4/1:4. 69 ..1/.369196910:65..  56.369069. 653 / 946.9.514 /..55:. 10:65 :6.3. 65: .5140:65:   31 '065:.5:.51410:65 .0069153 95191 .65:...95.:5.6 ..9: 4.51 .3:6.0. 04/9:64979610.3369:05.. 4:.514 10:65: .* 40 .650. / 065.64469. 45.5 5 9.: .6569.969 33:69.6.5.65 0.1.47..33 4469.3.7150 .9 4:9.:98945.167.33659:635.5  56.01 56.6.:::699:63..18.3.:31..7969.007.9 .50:06 #94:23 ..: 5631   ' 4469.0..:.5.:9:6..77.16.65069769.65 6.7.. 98945....95.65: 653  :0 ..50:06  #94:23  9 0.065:. 10:656.:10:65.......369769.3.: 6915..3:6 519:.514 10:65 :631 / :7. /.655!06:51:.665350.90..51.0.3:36...0:7./064.9: .923 .5 .9.606473.31..5 ../3/.51 .:.33.3550.569045.3 064730.69.51.77.65. .6.9415..::7065.514 10:65 4.956160..1 69 :/:.9.45.:251610:654.167.03 &0.6569 .4469.493...6.03 &0.69.556. 56..:70..5065:1969.65.51..33. 33 989 ...9.:65:5..92165.51.:. :  6 069:  ....::9.651 ..551:. 06473.77. .51.116559.. / 47361 5 :473 3.511 1:0::65 6 .65 6 .51:../9:69.0:565...: 0 0631 .650.677.6 4..25   .65:..1%'10:65. 3..:493.6 1:4::.: /5 519.65 '%'10:65:/9511 /./9.65   ' 69.06: 5 .514 10:65: :.65. ..: :.51.5469 10:65699:63..:56 .. .9.5.1..9 06330..:.656.:63156.600...3.941.556.: 694   .0..1 .:. 5 . 5 .6:3 9. ..: 5.: 065. 4.:4./3 .4511.31 . .:3 / 165   .: :./3.9.51 .3631/765.5:...10:65/5..9..6533 :4465.4.1145.31.65 .1465..0:6910.:1 '7964....:.11.653390.5.34.365 564.369069.:.3 :631 190.. 3065011. 69.:63.51 .510..50 . 51: 50::.45.6.65 :70..3::1:6/150    ':.4.. :0 .1.511:9:56469.9699.:.1//6...5  .65: 5631 . 4469.6 47..11.91:   51 .:3    0..51410:65/0. :631 ::.6.336  6.6.65 69.:/.5..:.556.465.1 .3:6/6:396513::....65....95.3: 5.  .33 06473 .2:69.31 .51 . / :/:.661 .30651.94:. .1.945.149.:. :473 :44.:: 9. 3. ...69479:09/1.3::   .953:13:..960647335 0.6/5.656. .: 95 :.5.5. '10:65.5./95191 51.:.4..1:631 . . %' 10:65 .9.5610:65::.56:..     5069. .9.6...  10:65 :631 / 99:1    10:65.3:::..5.3::  ...50 .14:..006911..:473:..3.617960::. : ..51..:...3. 565.. . 7.9   .069..4....6 .5:.06473.5%6496 69.5:..77.09.65   9..1950 .9.31.3.39..5/5   ..50.973.:13..5:..:32:14.51 469 :50.:.3:064.9...3..696.7.5. 65:.0.6 .65.: .5.331:91 :.9..  .1791.465.:..92..:.:6569::..:655   .3  6 :.0 .51.51:..//9. 47.9.1069. .5. 1 11 :6 ..517/30065150 5.0647655...  '::..6979679.0.65 : 51:7..3:.5.3   69 .065:. 75: 6 :063.3..625636:.   60:.65. 0.573.65.03..59 .5 .: .01. 7.9. 069. 6:5.61:069.66.. 69 9 / . .51.1.. 9 .:.1  :631  /3 .65 /.3:6.65379679.6 75765....:..:: .:951915.6 17960:::50.:.:65:.:5.3.. ...  .. 069.677...36.967769.9 73.:6957996.6 .36:57.:6969.9.. 65:.9.65:.65 '36:57. ../3 ..519:765:/34.69.3 .:.9.511/..1 7960::03.67961. 98945.:6.93 ..5.:    '::56.93.9. 9 069.7655.650.9..65. 7960::: 63.6.556...9.6.:/.06503:65:6.:0.67.6 :33645 .  5 9..5 .:5.: 6 &0.6.656. 79: / .65 '7.. 1  795.145.6 .77.::9.30.  3:.6.31. .9/5. :.. :  .510650:10:65: /.6/. 98945..::5145.:9..:.:1011  .51:790:37910.3 9. .. 76::/3 9969: 6 .96 .76969.5::6:10:65     ..513.3/9..91/./3.56..91 .7733. :691 569 ..3 ...1..0..   794. .: 956.1 5..51 .:.03  6 .. . :64.511:.4.05 145.5..3 3/9..16:56.6:.:  .2/....398945.1.65:631/5694166.3.9: 79::1/:.3175165.:5.65/064:4696456:50945..369069.7733.5 4 964 1015 7: 1.50.:.. '069.9 .0.31. 3 .065.59.4516:.3...7..65   ':   5331 69 141 .9.77961 69 .916469.&...56.3/.656969190065..65...513..:   5.06503:6563.../616.65:...9069.773:653.4:0.:06503:6567.65. 69.3..0 .: :.65/9950:15.:655:. ..4.9069..365  %#96/3463.&.65  .10:65 9:63...65:.573.5:56.39. ./6 796:65 16: 56..32:.51410:65.50:06   #94:23  &%    (*  ..3..0..3/.4469.5   ' 796:65 99: .7056.    ': ..6.: 5699950.:.16    7.3:6:0.06503:65:63.:95191/.5.. ..5/.902 165 .3:7559..6..31.9/..:  .5/.3.697.1506.* 1:.9 /5 . .:.9.5156.958:.3069.6 .5.069.769.51.9901 065::.: 61  10:65: 6 369 069. :50 .05.653.6:.: 56.   :..369069.   :...677.5176::::5.3..9.94 .6:7769.7:6.31.50.96 .0.65   9.94...5/.51/.9   7.:.%'10:6558:.65  .6(*:5.6..::59.93::1:9.5069769.50..:.323  .6 .533.515:6.595.1*     6: .51.1 .656.3069. :0651 .69795073.53.93.1:.7796.9.10:65 6919  69 935 519 9   6:.916.5:.::69..6 77.5:.9:64.51. . .9.5   31  ' .9069.61 ..  .6.6.6. (.:. 69769.9069. 7961 69 10:65 4.515.90. 10:65  .65   9. &.61.79:5.515:69 06503:65: 6 .53.': ..51 5 6 ..51.773 .65 .92165./.:/. &0.5:097.6.:6569..433.7.3/:.6(*:69.:3/36:065::. (.06503:65:.:. 5069769.7733.3.465:6.7..10:656.9.910:65.769:6.5.773 .:...69.14936 65  7.069.1:76:.5.6   69.00:163/3/.69.0  6. 369 069.7.3: &% "0..50.:70.: .5.9.5/.46:.5.:5 .10:656 .3..6 .515:6..3.5.: 69.51410:65  31  5 9. 69. '1.::69.:/.9 9:.677.51.65.:/9950.:0650.6 69.5  !6 .476::/3 . 6 77.:70.6 06473 .6 .50:..03  6 .369069.515:6.5156.65: 964 .6 93.31.673.25 519 &0.4469.6.6..569.:06503:65:065..065:.:6 0 9 86./69 .6:7769.1.5469.3365.:651:0::65699.04/9:64979610.1 56.65:. 1:09/1 (.3: &% "0.9:6.9..:.9:.9: 10:65 50315.1*     .&. .5:::6/6.3::6.69..6.69..9069.3:    '&.../ .517976:6.167.0....76:1.39:765:/3. . 65:. 69.3.61.51..03   65:.9.31.36.31 .9.3:.6 :.5.9.1...9.4...93.:493/.:44.:63690633.60.33.. 7976:: .65:6.:165.9.195.33 ..5/. 6 77.7.96 5. . . 65:. : 16/.5 .65 6 .9644.65 .94..91: . 644::65 65 30.5 : / .9 0.#9:15.596.5 . 9:76515.94: ::5.0.3 . / :. 6 ...3355.5 .65 ::45.6.5 .33 .5..5 :.:44/9: '.4769.0 ..19...9 69 .065:.65.: 5065:.50(#.7765.65796/..25..:0.  .. 69.9 .: 9 . 9:76515.:51/:.65:.31. 1:5./3:1. 644::659.5/......1 :0 1:5.19.: 0..7765.. 16: 56.9517515.5.   ..: 69.65: 6946:.4650:..:56. 5175150 6 .7765.51644::659:6..:09.:.4769..: 69.4769.007.33 !60.931.65 856 6 ::60.6 .6.3   ' 65:. 1:5. 964 ..65  33 56.65.:.5 6 . .3644::65:73.902 165 ."  ': 5933.6:.94.99..:65:::.& !6   5.0...94.59  56 4.3  .969 960.#3775:.916*    '65:."  ':69.6..0.65 / ...   1:5.0.65.65 7961: 69 4.#9:15.  .9.. 65:.65 796/.2516. #9:15. 56 :09.5.: 0....5.5169. ..51 .5 #9:15..30644::65: 50315.979:656.5 69 ..6771 964 0. 644::65 65 30.  ..:.65.:960.4.5 5 .7.. . / .5   31  (* ./3. :...3644::65:     16: ..9 6 /93   '::.169..5 ..65: 4.45.5.  "    ': 69.515063.. 931 .95.6..65.1.52:563:..6 .9:76515.9..51 .794.5.9 69 56.976*     : .7765..65 6 .3:.65:. 794691.50: 03:6:.65.6959.544/96. 7976:.56.3. "   .#3775:   . .0.3::103.656..0.65.50(..45 ..1.0650995." ...7765.#9:15.6.6.6 .631/469 5275..55.6.65 103.5. 065:..5.03 .1.65 9.#.6.65 011 56..45.915065:.6 &% 793   5.45.65.0. 59.656&0.61:5..3 :: 6 .. 51751506.5.1:5. 1:5.51 .: .69545.4769.55.69  31  ' 9:63.. .52: .. 644::659..9.46..4769.5 644::659 631 51945 .9 69 .79./..065.5 .939 5!..1.0. " ./. :63 .93  .65:.3..65 6 ... 796:65 796/.../3.:0..3..5 ..0.49:6. .:. " . /.644::656530..91.:1011519.3.51.5..: .79031..: 69.&630. 57.:..51 03:769.3 769 6 .979.65:. . .656 .65:/1:5. 69545.51.5 .94./.. . 51 751. .65:.. 69545.65 ':&0..:  50315 69545..  .45  .:6336:  &0      ' 644::65 65 1.51 :. /  69 79. 95 .51 907.: :/1:65:  ..9:.  69 .6.50:  69 5:.9: 69 :: 6 51: .: 6  .3.5 6 .55 .: 79.33 ..55 .  .51 79679.33 .6  .51 1.945. :.6 .51 31 5 .1.69.3 .0065.6 . 769  . ..  651 ..657961:. 47.65:  50315 .1.69. 9:.  79643.45 .65:  50315 93: 65 .9..9. " ..1..65   .9: 69::669545. .: 6 56 6. .65  : 03: ..7 6 &0.5 93: ...56.45 .915.50:.7 6 &0.336.6.6 .51:.7:79::3103. 65:.30..9:.. "   ' 8.. / . " .: .519. 691 03: 5 ..0065.51 03: 769 .3:  .65 :.51 93..: 9.336.6 .69103:5.1.9.6 .9.7731.51 93.. .9.9. :63 .. .969  .5..50:   69  .6 101 65 1:..51 1:. 7..6.:.556.51565./3751.: .4:0..4&0.. ": .9/ 9.1. .69./3:..51 79643.6 15 .65.65 .9. . 519 .7 : 56.76.:::43.5.03 .34. :/0.: .:06517.65 .9. 103. .6.51 .656.1..7.96331069769.9. #   ' " 065..5.51:.65 .65 .336. 65:.55   ':  .76.7.6.4 0.9.14.:6901.50:6550::. 6 &0.5103:.65:5.3644::6590.9.556..19.9.969545.656.01 .1/..9:..9 4..9: ".:0676.751.969545. .:::7731   '"696:3.960.:79. .65 ':69103:1156..33 69545.9: 6 69545..50 6 550::.:.7 6 &0.519.50:   ' /.51 .9  0::  .9 751.9:.. 691 03: .3 644::65  644::659 &9.065.9.::63.1.51.77.69. :0651 7.65. 65:..35 691 03:5 . .  .5.9. 3.   95 .5.:03:50.5 .9.5.:.:0651 7.9103: '9:.514.. 65169065.:. 795.11.921195053. 13/9.6..656 .91 03:  3 .69:5.9:/.69..6: 69 .5 93: .5165:.::9. 1:.7..33 69545.:6569 .5.3 644::65 73. .51 03: 769 .65."56:.69.:06517.796:65:6&0. :067 6 .9.97.655.65  .9.65:.5.69. 7.:50.:33. ..:4.:  47.5 .:..45.65  0.6 8.9..615. . 65:.61: 9891 .058:.5179679..9.51 .97 065.9.65: '9:561:7.9. 519.70:."519.9.65: 6 .1..: .95.51. ": 769 519 .:79676:.9  550::.  69 5065:065.76&0.9.. 79676:1 ...50 6 993.76&0.1.. 9:.9.9:      '644::65:.33.11.03:..50:  50315 .5 565.695.5.3     5 :..9.9 3.:63 .9.65   69  :6: . .3 . 6 .:. 03: . .5.1 5 .3 .65 8.511.7 69 :/.6 .1  .::60.: ..9.:0.6 0 . 79. .: 1:.3 360.9.6 .. : 441.56.699653.656&0.51 56. 79.9 7..35691:5. .69103: :55.6.77..65   ':  . .9:  . 7.565.9.. / 9.65  4:. .1 . .7 6 &0.9. 9:.:0. 1.3360650995.69545.1. .51.50: :1: ..9.9.1.. 7.69:0.69:     .5..95.655..9.69103:5.3:6./79.45.1.49:6.3364.5..".6.65.9:.65:.3.65790:3.1.76&0.. 6...5.5154:.9...51.9&0.": 769.:565.9.65:.2.1.156./306503:65964.45.'03.  .615..":...51 .49:6..79::3.50: 5 .336517515. " .45.03: "5.. 69545.65:  ..336 550::.65 .51...6 ..65933. .1.1.:067 6 .6... 79..51 93.5 93: .9.3 .6 . 6 09..5.5.9: : 03:      .69.  79643.: 5 . " 16: 56.. 69545.65:.95.49: 765.5:.9 751..50: '9.96.1.6.65 . 5:.1.11. 5 . 79.51.5 69545.45 .6 653 65 51/. .03.51 1:. .: .:9.9 : . .  ./3 06503:65  . 03: 769 .6.6.65:.1.51 . 5: '6:001 ..69545..306564.0..:5./1.9: 5.9.95:.469/5151:....5.0   ".0.6 :..9: .:50: 5.. 69545.   ' 13/9.5:.50: ...:6936..9.92. 5.0.73. 5.:. 4. 69545.5 699465:03/3./09.5:.43.9.65.0.0 ...69 33 16 /:5:: ..3 3685. 9:.. 69545.96331 069769. .64.50: 33/3.69 69 091.55:.569545.3 .0: 6 .92.:5..9:699::0.4.13:. 69315. 65:.9  1:5.../6996:465964.6././.5:.:3/3..51 0647.:.651 695:43.65   9..6533.50:45./3 /:5:: 79.3.1 69 7/3033:.3 644::65 9.. 9:..65:. 6 &0.1    5 ..:65../96.73. 4.915.65:3/356.. 069769..1 69..65  69 5 ...:./:506..45.065.95... 5.65: 6 .47.9.69.:69:0936.: .9. 6 .69545./.03.9: 56 5:...45.:...65 : 79.34. .65..4:.: 5:.0.0. 5..:.065.6.3. 0:631 /1:0691.0:79::15.:6906959:.3.656.691:6.5.46:.65 .:6&0.:.6.11 50::..:. 5.03.:69.6..3.5350:.. .65 :.5.:93197.979..65:..5.9 /.:3:3.13 .5.. '5.0.979..3.695.3.51:5.5.1645..3.6.5..96335 ..0.65  9.69 5. . :.45.65  .52:    69  1:7.9..52: 065:.9.9.6:79:.769.6 1./.6. 7/30  515.:3  ' 13/9..50603:769.: .3 7696:79:6569/.9.52:769..65.76&0.51 5301..52: 5154.9.690.52:  .9 3. ./9065031.3/.9065:.65: .. 65:../..:/506510.: 7696 .9..1.9.9:10.":..6. .5.5.45.1.45. .6 .696 69545.5.52: .65 9..6..51. 6 .52:50.":5::.65 69 69545.5.65.51 .56.: 90651 .51.69545./306503:65:..45.1.335519./.516/.33 .3 .6.33 69545. ": ...1.9.945 .52: '5.5.52:519&0. 5.1.3.51:70./.65.9.579610. :6515:: 6 .9.6.65:.52.4 03.3:69.3 ../3 .. .6 /..65 .036.065:. 5..9.65 9.5.. ": 769 . /.3 .5..65 .65: 6 .52:7696:79:655031:.65:.65    :..50:   ' ": 03.69545.9..65  519.964. 176:.:.:69    '9:../.:06517.3.65:.:5.999.52.65:../651. :.51.51.65.0650995.: 6 ..3.9 679.52: .52:4:..51 .3.51.3 644::65 06594 79::3 . 5.49:6.51.52: 0650995.6.3.03.1...525 3."..5.4/..:: 190.45. .1.. 6 &0.3.9:..1.6. 9:10.9.65. /.3 .334.: .3379.65:15.5.3.52   50  . 3  31 6969 .33669 1:.5.65.5.":9:10.33.65:03: ': .065:...52 6 .6.5106503:65:.6 ..6.:.3. .":515:..69: 0:63179.976*     .. #3775:   644::65 65 1..9    5.51..69.9: 6 69545.5. "  36745.1..52: :50 .336 751.52:1616.. /. 65:.: 50.   &%   .65 /365: 03:3 .65 .50(..679.9. .50:.1.69: .9379. 65:.696.. . " .694.50: .5160.:69545.69:06517.069..51 ..  69545.5106503:65:50::.1.25 .50: .5 / 3.1.50 6 .1. 69545. 90691: 6 ..1.51."  .369.    .6:679. " .65 53::6069::0515: .": 515:.5:.4..45 .65.79. 0. 769 6 .946116999:1/.3..:.9:.:     .50: / ..69:69091.. 964.4.9.9.5:6.5106503:65:.5/651/.51 60.69:6989:079.3: 69 94.1.51 964 .3369545.76&0. 69 79..9.9 0650951   ' :7969.179.3: ...6 .1.9.33:519... .51 06503:65:6. . 69 7976519.5106503:65:6..515:.3 644::65      ' 515: .313 .9....1. . 69545.50: 3565.31'7696.1..45.51.".6. " &0.03./.65 9.03:  0..964.556.25.. 9 : . 6 7. .69.6.: ..51.:69  69 .51 69191 .0...5.6565065:.7765.6 :09.1.5 5:.: 0644::659: 6 .6 .0..45.:  .65...52 6 ..04: 533 .9 :25 .:. :..33 4.65.1 9 533..::/:1.55.51 .69545..45.. 69.51 165 964 ..691: 56 796..: .:  .&0.33 / 533 .51 .6   %145.. 6 5 644::659: 5 .654.659: .7765.1:.:./3065:8506.9 5 .6973.. &% .:    31  %:76515.6. .61.659: 065:.69. ..1 9.4..45.633/.6 :0 796:65 :.65..6516:56. !#""  ..65 '969 .7.31 .33:. 51.65.79..99461964 60      5 5065:..51 ...65 6 ... .0.1. 6 7/30 51:  964 .50:   36745.0.1.33/7.6/0647. 565.45.9.3 3.7765. 6 7.:103..91 &0...65  6 %7/30 0.. .. 0659: 56 9.::147.1. . 6 5 0644::659: 5 . ./ 5./064.51.3..51 61 .5.9..9  5.9 69545.:  476:: 56 1.9 :.02.5.69545.33 . 7907... : 56.3 #630 644::65 !#""  . !."6..65.976*     "!&'''"!    .. !6   %   .50:16:56.69:4..: . 3. : .9.:.9 76:.5. 946.1   5 .. ..6..45.659:.4 69545. 93.5.1964..45.50(. .6 .0. 4736 : 33.. #3775:   644::65651..5. 7.656% 63.45.33 1:4::1 :76:. 6 . .3 .5 : .5 / 56 .51.9  10:65  95  103. 9065:19.51... 644::65651..6 / 63.   '49.3 .65:.45. 9:10.: 165 79:.:769 .9 0.0.59   : ..7765..45 .156919..3 6 7.0.51 . 065:850 6 69 935   31 .65: 5 ...:. . . 95:.6103.65 6 .3.  &0  !63.5.659: 946. .51 61  50315 .9 :.  .9: 79:.565.6. 79679 0. .5161  '969 /..656% :533.. 95:.7765.6 .65 . 644::659: .45. .7765. .45.5.76:.946.50(65.9069.65 6 69  ...:1656969651:8:.33 .9 :631 56 3659 / ..659:.5.: .6 .65 .5.5 16/.519:76515.16 99 &% /    5.9  10:65  .6  .6&0.67.165  .0.1 519 %  :631 / 9461 964 60  5 6919 .65 0069153 165:. .:6.:   "50 13..: 69 .51..5.6.. 36745..   3.3:3.3.3.69.: .51 9.53. 75.25 /61: 6 4507.5179.: ..5 90: 653:03:3.9  69 4./6.13. :/0.:..0.3:56..::067   31 #630769:. 7/30469..51:.33 1 ...9::5.5.6 . ...4 '769:735.769:.. 064465.3.  .9   . .59.9.2 691.51. 65:.50:  ..64.3 69545. 975.:/5151. 7/30:.%:*    '!%!'#"%&"'&''  #630#69    5#630#69.9 6 .:769.51 6915.3:3.9::.1  ..334.9.:.::067:...:33.#9:15.5 ..145:.655.9 .:.: 0.3:3..33..91:.  56.65.51 69 .9/..769 '!.51.4/.:./3:.5 ..05.7630769:3611794.9.:9:697/30. 661 .5595..3.596769/616511.1/.556.559663:64.65: 69 360.65  .76::::53:3./..9.9 .935. .65.. :.51 3.!..769 :.: 9.6 / 69 .3: ..65:.6:695.:65.96763.33. 5.5.25/61  .65.6.906599165.:.5103../3 3.:  :.5 4..4./ 90:1/.4..51:. .3 069769. 65  50   &%   . ::60.9 33.. 51   ..3 3.369545.:.3...1 . 90: 6 7630769.9/.: 13.006473:..:.6 79646.50:6...65:.679:09/93..5.. 56919.51 59.   6456  &%    ' &.5595. .96 ..  469.. .96 .3:  7.6360.9  .: . (#56*    #630769.90  :. 7673 5.  ..3.6:695.769.60.&. .:.65  6616919 69 :.0  10..  . 3:3. .99 6.:: .5 .0   &%    ': 13.3393.61   ' :067 6 7630 769 ....0.:6.6567630769:4/61:5. .:  90:169.3.6 / :6 064795: .769..3:  06469.91 6/0. .59./:5::794.  7.51 0655506 .  #630769:::5.93..9 09.60.51.6955.33.9: .5156.9. 6 9.903.0  6919  469. .95.: /5 31 .65 '.5.0..6::305::699..3. 06445.6 50647.369545.: 6 .: .33 4.346:.3   507.  :. 103.. ..69. 0.6..51.4/. &%  0/16"7.9.4/3#3:  ' 5007.677.3.3647.5.:57976: :.:769 #960.3: &% .5.6.9 .5 50   69. . : 56.: 36:.1.50(#9:4.. 7630 769 6 .: . .6 . 69.35.5:.7/30661.. . 6 . / 065:. . 1.3: 6 69 :/065:06:5::  3. &. 3:3..6065. #"%:5.7/305. .1. 90: 6 .:. 9. 5 .8.. :6.15/:5::.6 90: .0... 9.. 9...9:.  .:..:/5065:191.7/305. 7630 769   #9690  .9:.. 7.6 679.9..*     6 :631 3.91  31  :. '79. ..50: .96369.551:.9::/0. . . 6 :/0.5.0. 4.90  5. ..3. 7630 769 6 .9:.0.   ':  069.7/305. .. / 9:631 . . 7/30 469.:0 9.6 / 065:.13 91651: .6 93.65 69 30.   .9 6 .4/359. 6 .9. .590:67630769 '..::4 ..6 7.9.69.  .6/.51 ..65  064450../.65.5./3 .  ' 3:3.51563:..::..6 . : 4/1.9:.65 : 53501 / 065:. 9.5165 ..993:. ..3.9 5.9 : 9.6796....7/30./5.:03..51 ..5:769.: 769 .9  .9.6 .50:.6679..3:   ' 79:47.916:56.91.511 ..37/30 ./...: .5:.: 1.5 6:60..9763076969.:.59. 1:0..: 16 56. 7.6 90: .. 4:.3 065:19.563.3. .. .43./5.9.5.661 . .:/651.. 679.0...5.5..91 :.6 1:.41:.969 : 56..90.65    .65 6 0 516/... :0.5 16/.65:   65:.50: .5.50:... .6319643:3.9.: .9::1. 7/305.90...911 .769.:3 69 ..9:.3313:3.3 .9..:. 79:41 .5.9 .6679.... .6.6. . 3:3.65: ..9 13 519:..9. 9. 6..6 79:09/ 9. .51 0::9./3.36.65... 1:09.&.:065./9:631.9 03.:03.: ..473.:: .#3775:  .6 179 .: / .69/:5:::631/:.92 /.0.3 5..59./.65 : . 653 .3..19.9:.51 794.765.659/1:065..53.9 :6 36 ..51 .9: .30.3 065:.:. 065.65    593.79679.3: &%   ..796...39.4::/0.677.:65.5  50    69.. 90: 6 :0 9. .3..:65..6 :.:5:.5:..7/307976:.5  % !6  !6   91 (#56*     :0:: .6::699.51 /064: :/0.79679....769:.. &.9.91/7/30. &09.1 .51 :.: 6 ...03.65.6 93.5.33 93.6/0.050.55..:34.9 51:75:.5 765 . .335..3. .:79:09/1/. 305:: 69 794. 4507.65.0. .5/.: .4:.:65.9:./69 61 :. ../.9 . 5. 61   0/16 "7.151:.: 79:09/5 93: 69 ..:0 305::69794.: :::5..69.9..51:..5.5..67/3093.76307696../39.6 79646.:.31 90:.5056 .654..3. .90647.  .::6511765./3 9.3.&..6:. 9. .69.1   ' 9..69 .  ..: 9..5:.5165. .::169..91/7/30.65: 6 7/30 .5. 5.79679.7630769 6 ../316/.:9.3.9.93.  5 5 ... 7/30.50(#9:4.:65.3.693.7/30.6 93.65   31'93.65 6 .::4 9:10.9:/0.:   %7/306 .: . 0.1.3 69545..945 ../ 65.6... 6 360.:90..: 16: 56.963 . . 01631765360.9 6 .31:.7/3069.518..:9095191 '56:.9.3 3..:0.51 9.6:90:95191 69 . 51:.3 5.4:..6 / 9: 79.03 16: 56.6569.065.1 / ..6955.:563: .9  ./3 ..9 6 .99 .*     6: 9.: . 9.:6.:.96 579.: ...:.3 647.. :....:.99. .3.95655:.0.: 365.634.92  31  &. 6929: 9.65691:7.0. 59.5.3 69545.6 1.. 5..90..7/30.&.6 ..6 :: 69 9.6929:065:. ': .65  ' 93./69 . 064465661 &/4::65.: 0 .6&.:   31  ('* ::.69..: 33 / .45.0..369545. &..50(#56*     :0::9.3:.5.51 .3.: . 69 0. 60.  5. 065:495...9.9 '. 493 93.51 93.6/0.    .9: 63...51 .41519.91/7/30.50 6 /:5:: 305:: .3.9:.9...796945 .9:..5199.519...9565.7/305.91..  3 .03    6 .55:/.65 .. 5.:32 %".:. .4/35..90  5.769. : ::5.9/.9 #"% &% !6   5.90:6. 51 : ..:7630769. 4736: .: ..67967993.:64:3..065:.65 6 7/30 :630. : 6 0..9/..5 / .0. 06330. 59.16:56.5. ...5156.33 .547/303.3 . 796/..90. . .5 9.:630.90       . . 064465 661 065:195 ..5119.6 .....345.9.93   ' 5.9:. 7976::  :00150..9.65. 69065550   .. :649 :631 / 3611 .0...796.6 6/.65.9 6 064465 25631  9..5...65:65:630..:0.65 6 ./:/0.56796.&.6:.56   33... . .47.9:.6 471 .:90:   5..9.90: 6.0651506.:56465 69 0...6336 ...331 0. 7/30 5.51 .1 .1   .9. .195.5 7976:  ../3.656.65 : 56.5. . .9. :630.. 5 . 3.: 0  79769.7/305.6563156.4 '&...6065..4.36/0.. .9676307694.0 06469.596:7/30 ..4..69.65 6 51:  : 56.6. 0.7976:: '...03.559 6 :630.::69699.6.65 5.9..656065.65:. / 151  /.&.: 56.. 69545.6 69..519.:630.:.51 79679 0..6.6 .. ..65 5.9.961   &64 93.6767364./3 6/:../:1 : . 5.65.945 .65:5.. . &09.5496:93. 9 90: 6 9365 69 5.:5.9.53 . : 3::51 69 1:.6./3 ...5.   ' &.. ': 765../3. :6 /5635.61.: 765 . 0.9/.1.9.&09. .9.65..505 .0 ': :09.0.9:.:0 51:.33/9:. 7976:: : 96::3 .5065550 5 6919.5.656.:0..65 59.7/3050...65.6 93.. 675 .67/30:.: /5 :.9:.:.65.:9 '::16569.5/5:09736: 79:65:6969569./3: : 15...: 519 .92 69 36026.1. 47369: .5 / 540. 69 . 65 .1 .65/. 79646. . 964 .511 .4.06330..4 .0.9: . 65 .65: 5 661 .&..65/9895..9:.35. 065:.5 5.1.   ' 6/0.393..0. :04:.65:519.59.6 1.5.9:.6569936:7976::/:/0.9 6 ./69 . / :6 0.5.::979:564.945 ..6 / 796.65/.5: 9649..3..&. .:.573.6 :630.0..563.595 .0:.991 65 / 79:65: .963.5 065:.:5:..5 .976: ./3 34.9 5.33:95. .9.:05.. 6929: 9. 90: 6 7630 769  5.65:9.5679...51:../:63.135.6569936:7976::4..511 ..67967993.4..33:/0.6 :.365.945.6 :./69 61:.6.5179...5 .65 016:56. '..:65.5 0 4.3:6 5015...9.969 964..0.: ../69.639./3.945.0.: 6 /6.0..16929:565(#'* 65:69 &%  .9:.936:7976: &093./3 79.135. 769 .65. 90:1 .. 3. 569.65 :  ...90.9:.31 . .796.135. ..3.::47.306564 '&09.9 7910.96..   '6065031 :630.6569 06473:69 . .5 936: .15.951:75:.. 69 13./69..:6. : 32: 9 .. 7963651 :.0.65/69.65 6 065... 51: 69 .5:69.1 51:...90:693654.16/.: .35.90.:65..69. .50 .551:.:.93 5.37.9/.33.9:...:630.995 .51 4.65.9 '.51 /5635.4:55179950   ' .1 796: 9:. 50::.990:69365 .96.5:96459 . 6:630.6569:10..3.: 6 65 69 469 9365 ': .: 655. 1:09.65.90: 67630769  3150 5.9/.96956..9:51:75:./93. :71 :.1:7. 1.33 5.0..6 .::60.969  5 .65: 6  :09. 69....945..: .35.65:.9.06445.6 1.14::/3 6/:.5134. /69794.. 7.9...::49:10.  #3. 69 69.65 .956. 4..64.51 ..3 . 9.6569.5.92 /. 9 90: 6 9365 493 /0.0: 6 5:09736: 79:65: .3 0.9... 7976:: :631 56.51 16: 56.956..6 979:5./691:7. 5. 9:.9:.9697967969. : .0..#695336:   &%   &7. 6 .: .:69.546: 76:.9/069769...3 56. 4:3305:1  . 19.3069769. 7/30 7630  31  964 .65 6 .64.65 .79.51 4736 305:167. 06530.65 69511...6 9 ..3 069. .51 .: 56.: ..64.64.6565190.65 69 511. 6 . 6965  .659::3.945..65: ..3.. ::    5 5 . 5..64.69:63. 659:: .:03.65: :700.51. . / .5:.16*     6: .167.9:.9:.656:0::6910.3365. 069769.. 95.96796/.6565.6305:167.31 67. 9.51473645.3 56.9: .:.6 9 .. 9.4.0.465 . 4:3 305:1  . 6 67565: .51 4736 305:1 67. .64.:  9:79150 ..6 .1 . 473645.: %     .:/67. 9:63.9 : .9. . 8.31. 069769. 5.:  33 ..1 &.0667.     (%.. 069769..: 15. 7..069.64.50.91..33 . .0.0..91  &    1    & .0. 069: 6 79615:0 . 67..45 79:09/.5 .65156.5179:09/169/ 8.0. .3.511..0.9.: 069769.51/.06 41056967. .979:: 50 51  !1 %(49*51"7..65 &4.64.:6...:50.51579:65.0.64.51/069769.459: 914..6.9.069769.45.03:./064.5..8.65:5069769.4..3 .0 6 4105   '9 : 511 56 .64.459:5"7.9 6 . .1:.64.9:.25./:5::6955567.473645.916"7.9:.9 &.0.9   0 ! !9:&.65:5.5.: 794.916%:.9.9/0.0..95.9.1 5 . 79:65 95195 .9.9 : . 1.5:..9:.3.79..3...:9..3. 7673 964 .64.9 &.65.69.3:67:9305:1 67.5 69 .:::  :70.3:67:195.9:.:.    5 :0 .: .79..9 5..3 3.3 . 069:64. 7. 13 8.511:75:67.3 79.33./.45.5:1/..65.3:.9.9 3.: 065.: 069769.5  50  69..9:631 %6:05   . 51 5:6.1 / :.:.9. :33 3.51 .0.:  .0 6 67..: .. 67...59: 55191 / 5305:1 79.9469 ..:56.:.9   02 395.0.64. .64.64. %3 6. 02:65 355 77  !& '4.6963. & 53:: 796/.65: .9 77 1 %491     9..:. / 33 .65: 4736 7:0.451..&. 0. ...  93  0...951796::655...65 &39 .0. 7976:6 95:5 410.  93  96.5 069769..659: 6:6  3. 79.15.6...90.6. / 4..65 .64. 79.9.:931.. 40.9 ..50.0.93..79:506. 065.7961/:. .9563.45.0.9.. .64.51.40.167. .:.5196.50.03: .677. 1 ""7.335:: 67..5:  50  &...9 .:..9.6....6 5:9 ..0.51 305:1 79:65: 5 6919 ..4.51 79345.9 .03:.65::6365.65 .9 1    5.0..3 .51.5167.556..4 . 69..169 .51 7:0.:.9:.31 67.5 511.33.5..3167.36  6 &1 .0..03:  . &6 1&.9 7976: 6 .9:%6/02.7:0.64.9.  5 .:.963 6 .::: 50  &.3.95 :335 ..9:..:: 69 ...5:39: .51:3353. 069769."7.51.56/0..91 3: 5 .65:56.5.:..94.15.:::  :70.51 :90.0.9.5 . 065:.31 /.0..6.51 4105 .6 796.3  0/16"7.45..5: 6 79:097.0 6 67.51: 16 56.5:/9.3 :90: ..51:33563.:::.64.3...51 35:: 53:: .9.56.3.516 9 #1    ' 794.3 9..0   &0 7976: 4...567.: .:  ..3647. 79. 069769.73.03: 6 490.64..6 .515..45..51:70.916"7. 93..902 .6:7..069769. 13 305:1 7:0.006473:.9.3179...006473:1 .96 .65: 0. ..9...5..4:.: & & 9:6 !        '6.916.5167.64.6/0....64.655"7.....7/307630 69.3 :.9.34035:: !665.: &.5: :965:.6 / 95191 / 0647.3: &% .65 .5. :90 : 47361 / .64.16:56.5135::/069769.51 ::5.:90..51410.:  .56967. :..0..::: 9.:493.09:0./35.9..5.659:  5 .36 . 9 1791 6 3.25 065.:9891:. .9 : .51 .1 7960::63....:7630769.  .5    .6..3.6.6.0095....6534. 5 .65  . '%.. 4.5 %694... 769: 6 ...90  5.51.10::.50(#56*    '#696455.:.51.:455.656.9.164..39.  %6.6.*     .4934.79679. 769 6 455.%:..:6.473.64. 47345..511..6 0. 79679.65  .&.3 .696 . 659: ..51659: ...50 .:7961:...769 6455.5    .949/50.51: .7696455.44 ....64.3..6595..3: &% 0    5.3:6 . .5.6.9 '336%.33/50. . 93.590:6.5   ' .69 79679. 65:.. &.. 164.: 6 50  69.5 '6.65 6 .17960::63.517:0.25679.5.17960::/6/:915 ..%79:09/:9..25 519 . '90:6.:.:56. 64795: 9.(5*679:65:.3361  . ..5989:. :0 93..50(#9:4.33/179163 3/9.65 6 79.5..9.9 5631 5 ..00691.164...5 90: 6 7630 769 69 ..656.79679. 9  ..677. 65 5 0:: 6 ..2.:.9: .99 6.376::::656...:995196.6. % :0::   31'47345.1:56.   ':  69 65 65:.67.. 6 455. :..7/303.990:6.9/.9.9 .. ..551:75:. 34. 79.659. 79679.51.9:93. 1 7960:: .65   ' 796:65: 651 5 46195 065:..69...25679679.5:.....33 56.6. :.65  9.3769   &50 . 69 769 6 . 164.91: ..3 796. :695 :..50.33.6 9 517515../4730.6:695.65 03. .:69545...9469  .:.: ..51 8.6.9::.31     455.0. 34.0.03.  /.5 : .769.679646./3... .3 3.6. 164. .3: 9.6.5. 164.1.65. 69545. 9.794. 50::.25 69 7/30 : .65.6:9.. 90: 6 .3 :. (7*9. 769 0 631 6.. ..0.... 769965115.6 .779679.9. 79679.5: . 79679.3 9065. 69 065:. ..65: 6 031 065.:: . ..669545. 06475:. 9.06446551. 69545.9/.9   ':  .5 511.59.65.. / .: .5....779. .6.65 7961: .91.11. .5 .. 769 6 455.065:.9: / .5 .7/30:1656.6 79. .51 7/30 ..9..3.:   5 .6 79679.:469:136:3 .3 15.3: .935196.06473.365 &% .:63156.697696.769 .::0.3.5.6.:  6.:. .9.514.:....65679..13:3.. 5:6445.911/.6 .6 . 164.65:.779679.../53..6.563.65  5:3  65. 511.3 .69545. 9.5.9 7/30 7976::  .339065:.5.01.51 6 .9.93 5..36::09...969.50..1(4 0 *   ':..:0904:70.94:6.31  69  . .(3*5.51.53../..65.6/:9.9 7/30 5.9 5.51565:. 3:3.5. .1 .:461:6.5./:   ' 769 6 455.5 .964.: 13.  ..0.556... 6 455.65..33 3:3...679.0.::.653/90:15:.. 9.53... 130.9 190.90.: :50::.:1 5. 9.3. 9. .6/. .65 0. &...6&.6.6 .51..:  .6 .69.1  ' 90: 6 .69..556. : 065:5.50::.9.:656.4.9.51.53...6 .:..: ..5: 065:.3.63.96 ./6947..5169.91   !6 :70: 6 79679. 5..3 / .. 13.5.515..69. 3..50.0.5.. 9631 6 5. .. 769 : 50::.  .....2579679.. 4:..79:9.3.: .5  ..515.679:65.659.46:.: 943 :..691.. / .:.313 13.51.:/:.0. : 31 / 511. ...3796:65:6. / :.....6796.99: .6.:..964.796. :0 769 4.: ..  9:63/9.935..5 4769.5 4..13.3. 5 5./.979.6 360.19.506.0.5045.90:669545.0.5:.91/16/.53..9   ..2 . .:0676 .90:6.5.51  69 9.9969 .1..  69 / ..6. 6 .3 .513/9.0...:065:5. 164. 7694.3 69545../3:..3 065:.9 79:09/1.5: ::5.90.510.65:. 73.0.90.: .306550. 90   91 (65..  . 69 7/30 :69 7976:  595.6.6455.3 &.5..769:595.4. 5..7/305..5 : 4...5   ..51  ..:79:50..25    "/6:3 69 .69545.:9.:..51:6..97/305.: 6. 79679.65  5..79679.3...6: 0.: ... 5.9..9 .%:*      455...: :6 7.9.9891.9.:56.96    31'9.:/593..06473:69:..6796 . 164.67630769.151.5...9      :0::./ :.65 :.::9.9 ..677.5.0:..6/.0.5...254:.3: 5 79679.65 6:796:65:... 5.5176::::656..0...94:. 69 4731 9:9.31313.9:. 51./697/30:  .6.6569.9:.:3 54.. :695 . 164. 79. : 69545.3.19:.3 76::::1 /.6.59.33519:..661. .696.5 765 .45..51..51 :0651  ...05.9:.9.5::.6 9:4 .. / 5 ..369545. ...9 56.79679.5.9...9.  .9.69.65:.9  .9:.3:3.    ' /8.517515..3: &% /9. 76::::656.. . 0651455.6 .69.69.5.. 06475:...69 .0651456950. ./.6796.:.945...9. ': /35.79679. 9.779679.989:569065.65  61.99.164.65.3 3:3. 5 .3 :5:  69 .769 5.256 79.5 4.65/.6...3.06475:.3:695.96. 5.:. 769 .&.&.9..55. 79.6 .::9.65 4:.6455.6. 56.   .79679.: 69.69694679679. 164....: .3 5 .9 6 455..164.6.51.34. : 5:7.31 .506.::0674.51.79679.. 769 6 455..506530..676::::65 /.65:.7/307976: 51.33 :7..5 : .51: 69 7/30 : . .:/549306594..65 .779679......579679.. .45.5 631 0.5:..46:...606475:.53.51. 79679.190: 6.6.769:56...65 796015:   79679.0..:.493:9:56. :.1.9 6 .5.79679..: 6 511.3.59:70.9 31 519 .517/30.254:.   ': .25.:695 769.3695.25.:6989:.5..: .:9:. 0655.0.6/./697/307976:./3 964 :695.&. ..:.:34./..65. 69... . .65 796015: .  .6360. 659 6 .96..  4:.51.089 79.5.5179.1 .39:...3:6 :3 .25  61.7.. 511.50/../9.9:.7976:6.65.5   31  ' 34..3: &% /9. 0..7/30: 0. .79::769.: .6 :.0..6949 .0 14.164.3 :6 509. 79679.51. 5 : .:6434...0:. . .65 50::.96 .7/30 ..:.65..9:.65:65.25 065.: /5 :/0.:. :7/30473645.6 .9   69545.:.69. / 6/:91   ' &794 69.690:5455.506:0:::.:0651:7/30. 3.18. 0.51 : .5 .&.0. (.5136.6.5..:3:3.65 4:.1.4.. 164.: 69 6007.3 4.. 769 6 455.: 65.50:  .6.6.: 765.7906:3 066: .31.3 .656. 4.: :60. ./699:.6 0 .6 /65.: 14.65: 65 .677.1   %7/30 6 .54:. #3775:   ' 65  69.. 96..  .6915796.51156. .5.3:  % !6  3  :. 69.51.9 . 7/30 0. : 4:..6.  0 : .9....5.3. ..51: 765 .50::.9.50::..96. 56.64.51 6 .51 /0.9 51.5..9      :0::.55.51: 5 ...3.50 .6  . / 4.0.7696455. 9:690: 6 .53.9... / 55 .6 065:. 6 77..51.9.5.969. 79.9 .63.7796.::  .05/.6 64 36.96955.94557/30: .. / 7/30  06475:.: 3..0. 511.164.51 :/11 .65:..25 .69/5.25. :631/.51 ..51 .6.1.9.6567/30:5455..3:/. / 16.*     &.69. 1 . 5..9.51 1 7960:: 6 3. .5.0606475:.9065.:931...5796015:   31'.: 69 :... .51: 5.90:6.0.796:097.  6. . 5 455.9 .0631.9.5  (9. 53501/0..5.3.7/30 /5. 631: .0..3 .65.:. 7/30 : 98945. .65: #/30:565031:.:.164..6 .4   69633..: 631 5.6/79679.9:  .. ...:0.656.:.: .51.9696564645.511 56.0.: .:63.1 5.51:3..5567/30:5455.5796015:33:.914736:6.:.1956. .::47356. 3.4.145:. :/0. . 3/3 06507..95..5469 06551 .9. 164.45.1  &% .:    31  ': 69..65 6 7/30 :  79679.50:06*     .69.: 7/300..79679.5 ..6 .79679.::3::.0.03.9 .76::/3.653.33651.33. #3775 769.0.6 79679..   '.67961. 7.51:6. 9:76515.6503.0.33/5..9653   '96 .53.69...0../96... #960::5 65  .5..16:56.9..513.6565190.3 5069769.6 .964.969.550651.51.3.9.511:.9 9/.3.. 5031557. .519694..9.516:5  3:.1.65 : 56. 7976:6.:5.65: 5..9: 79679.31 7/30 7976:   5 &4365 9996 . 98945. .97960.336. 3.3 4..25:7/30 . .   %:   &%  (* ..51 ..01 . 0.5. 6 7/30 :   9: 6 ./3 ..:931..5:. .6 / . .:: . .7/30.4 5 . ... 769 6 455. ..: 3.53501/0..31 90:6 .7696455. 69 ..6 :0 .6 73..:.. 79:5.: .5 ..:697.550651...:6336:  ': 69.    ..31 4:. :.:.:56.254:..5469  :365.5064: 5.. .506 9165.65 .009.5110.9 :. 3/3..4 . .55 :631 / .9.:9:10.92: ".65:56. . .. 164...9 4.: 931 .33 47361 69 .../69. / /50.91. / 69 7/30 :  '9 .:/5:44.   ' 7/30 : 98945. ..656 .: 69 .:519.951. ..164..:69.9.5163506507.6 :. . 98945..69 .: .. .. 59.3 3. .3.25 ..:6:.. .516  .#3775:.656. 65:. 86..5   95. .:/96.:69473645.: 69.9..53 .3:  .94 7/30 : .:/5.9.7696455.65!6  ::1/6949#9:15.90.51 .5.915.6.6#9603.(511 *   ' ..   '6 .4...94:55:.: .164.0891 ..3 4769.30.906:  ./..7.3 491 : .3:6 31 .569:76515.90363../96.1.:/0.:695.515.1.#79679.5179:.:7/303. 3.309!6     .   69.69.67/30:0::.7/30..4.: .1.6565190. 6 77.. 469 064795: 069.    5 . 3.56:0. :065. 06507.19 56. . .9 #9:15..9    %:76515.5.111..7/30/5. 6..  0069153 :/0. : 40 3.30.9: . 7/30 : .3/.90.:1.90...5.65  16: 56.53.556.3.03.#:695. .. 9. 065145.6  .9. 6 ./:63.3 51:75:..6064490.31   #9:.45159:76515. ..69. %.9 51: 6 .5 . .9 69 .769.941 / %7/30 0.:65.51 .5...65 ../3 /.1.9  5 ..94 50::. !6    .3...4. :.65   ' .:4..065:.1 56 . 5.5./3 50::. 9:76515..1 79679.0.51 . 5 1015 69 . . 90: 6 455.: 69..: 931 .1.: /96.3 3..396.:3.. 631 / .51 0..0: .1 7/30 7976:: 9 5 9.5:769.7796. 989: 653 . 7976: 6 .945.5  653 . :/:..52:3.. : /96.5 / .36.03  .1 7976:  96..69.. 3. 5 .1 6    . 3.9. 164.:: 50::.: 06550.6  ' . 4.9:.6 .9.. .945.979:  '9:76515.3   &.169.25 69 .:79679. 79679.65   6..9    5 .656.25 ..:  :. . 9:76515.  .51 6.9 .03.9 79679.:./3""!5...7.:996515.4       ' 79679.9.51659:.4791/..79.::.5 .365:...499:.: : /.9.:   ' 7976: 6 09.55.67.51.::/0.36.5:657969.9 :91  9:76515./351:5.9.65 6 6..7..5:65 5..5 ""! .3.6. 190..9. 0.: . 7976: 6 /5 3.39.:69...90.: .9469 ... 6 .4.3 .65 6 .3 1.:..9 6 50::.559  #: 6.779679..:30. 065640 136745. ..9:360.   5 :0 .79:65:6925..9.00::/3. 4.31  9..     ('* :3.969 064: .:33.51 .7.. 7976: 6 4.: .52: .9... 3.: . 5:0.6 .25 /...15# '79679..51.3 6 /5 .656. 4.1931. 4. .6 /. 065.525 . 69 .360.6.9. 065:.: .6 /95 3 .03.03.9 631 / 9.:3 .. 190.51 69 .945.. 50::.05.51:.:1 ..945 .65 . 79679. 479645.69 690 5 .0. 7976: 6 .656.9479645.697/30: . 69 .69.1.5:769.9 4.5 79. 5.76:.3145 # &% .51.0..3 5 .: /.51: .9 65 : 69 . 3.069. 56 769 ./:50 6 :64 065:.256.3:   % !6  3  :.79647. 7. .:. #3775:   ' 65  69.4506475:.13 1:09/1 .5.59:10.25 / . 06475:. 6 77.03. 3.::.3 .65 6 :.3 069. 4.1964..5 .650.... 979:5.51 :6 6:...945.6 :33 .65 .. 5 ..76:..:.: /. : /651 / .. ..3 .:3395 675 4. 06990.3 .65:03:369.3. 06475:.3356.: /6.3:6 .65 69 . 10.: 065:. : .3.03.. 36..9.6 :/:.9.. .45.:6.6 . 659: 6 .:40 .99 69 . ..338:.:65. 55 /.65.65 : 176:.65:6...6 / .1   31     ' 065:.1 . #9:15.9:.1 /69 .6 :.  1 .:.7.6&.5031.33 763.. 7976: 69 0 ..3 69 :.:661.:.69. 3. 50::.659 065. :.53. 1.: .6/0647..:    #..656.61367.65:.51 .: ::  :0 . .. # 0.164.:.069.3 5.45..51 6915. .53.:. :.51 %7/30 0.65.3 .9 8:.0.79676:1479645. 979:5.:609.5 .. .92.6 / 7.9 0.33/16. .1 69 .65 6 69545..1. 60:   55.3:3.6. . 7150 6 065:.65 6 .9 ...9 .31.6: 6 .654:... ..6 9:76515.9  ..: .79679.65:30..79679.9.476:69 10.:33.5 . . 3.50 6 .. 6 .51 0647.5 . :.9:.00956919.1 .45.5   51 (65*     :0::..90.9.9 069: 6 3..3 34.9.65 ..4.65  .5 65 6 90:  ./..6 :. 790 1 / .6..   5: 6445...556. .   #3 (*   5. 06507..65: 6 .3 6 .37.7673   5 ./3: 69 . . (.556:.3 09 !6   .6590.*        .9 .306475:.6...9.0.65 : 065:191 ..655455.25.65.. 9:76515.25 69 7/30 : /69 06475:./3. ..6 5. 6 :. .:. 069..337.35.4 6 .  .5.6 .. 931 .. . 9:76515. . 79679.65....9:.1.25 .0.52: .52505.3. . 69 .8:.:  3 559 5 ..25 600991   %7/30 6 . .61.3 6 .1 69176:. 6 455.0: 56.06475:..: 3.  /96..: 5 /69 . 065:. .556.9.51 .69 796:65: .9 69 7/30 7976:  :.0.65.   ':   79679. .. 6 . 65 .945..3.4 .65 4/9.79679.. 7.:65.50315.. .6595.: .4964.90.45.6 / 16...945.6 / .51/./065:191   ...360..6569.5796015: 6:.6 ..4:697/30: #.92. 79679...6/0647.0.....945  ..136.. 065145136. :.6.. 065.:69..9:76515. 9. . 69545.03.1 .6 79611..65: ::5..90. .4. !6    /:. . 06475:. :4 8...: 365 ../35::6.51  /.5769.6/.9: .065:.:.51 56. 164.: . ..51 65 6 1:9: .0.90.: .51 7150 6 90:5 . 7/30 50::.. .9  . 653 .56.465.0.673. 656997.50:: 1656.65:919:..4769.59: 56./5. 6 ..79679.50::.:6 79.10.3360..4 :.. ./96./..1454:.:9:..530. .45.659::5 ...6 :.4:5:179610.3:65:./3 .65..6919679679.16.65699:.51:79:5:0:.9: :6.6...:.4.: ../065. 9.63.6&.19850: /.3 9.6 . 065...6.9..19.969.. / 59. .:..4 65:85.::47.65/.5796:656 ..6 79:5.9 .51. 6 :64 694 6 7/30 :90   5. 065:.10..:90.: 7976:  /96.441.49649.10.6/96.4 .9 9850: .5 .69. .79:65.65: 9.51 50:.65: .6067.659631/36:5#   559.  &0    511 796:65:696430'4.:165..51.65:.455..55.65   #.3:65/96.164.064465661:6989:  9.530.3. 964 9895 .:.:    .6 9.06475:. 63. .9. 793 4.9/..:.6 /96.9 .36..064465661./3 ...97936:5.:.65 . / 13/9.5  .6 / .50647.7.6:.3 .:.10.3/.10..693..514.1656.0795..765.:5.. 9850: .93.4.3:65 /96.:.9 : 9850 ..5.9.9:  / 50::.56     0731.9 .465.65: 4. 9.98945.5.:.65.5: 0.10.1 /064.:.6 065:.5:  . .95. .50 ...::5    9.9 469 511.98945.0.7.1.51.99:765:/3:063.00691..5.9:.:::5.:65.9 :690 6 95 6 9.:.:751:065:19..45145.5 /.565699 .3: 6 .33/:/0.97936:5.65:6316336.6. 305: ..16 .4 .4:.65659:. :. 7961 .25/.4 &50..6.305::.9.9.7.33 ..: 0.5.5.0 ......: .:/96..9 979:5..934.:65.9..6...50::. .6.65:.39:690:.65:::1 513.653..:.793.4 6 9 79::65  69 .:/.9.4 9 6 0....5: '9:56.6.511.  / /.6&0 6 # 3  98959.021.. 06475:.61. 6430   '#  50    "    &%   793    ( 516.:.9.::. .51:6:.6 06510.4.51 .79679.10.51 .:..:5.50:: 69.556.69.:.16.65. 13.:.:.: . 69545.:..169469/69..45145.0659659:761:5...56919.65   5.65.5:.49793 ..65 9.4769.&.16. .6 .:..:40.45145.65.51.*     .:..17960::03..9.65(*.65 :315%16596.. 6.9.51 .6.5   51 (65*     '065:. .3:65 :. 4:3 ...03. 79694.16919 69  .19.9.69: .::.6  ...3195964. .4   '03... ..3:65 /96.65::.513/54..57.991/9:10.. 6 : 06445.  &0 6.9.16 .305:794. .16 9850 .6 :.6.3:65 .455. 6/3.: 56 065:.06475:./54.0  :..16 .33/33569416...6..:.5.51 0 631 6. / 659:: 5 ..51.5160: 0 .9:79:59.41..465 6. 9651 .: ...9.:5 :/.1964:..45145.695197/30:90/59..6. .3145 #  &% . 56.65..:.45.33905...50::69. 65 6 631: .5065:.9:6/3.45.45145.9 / 9..51 :0 796:65: .6: : .:.3 796:65 .5/0650169.:.:.   !695110. 793 :/0..9.3:.5:4..65360.5.5.9 .5.3351 65 ..1/.10.9.9.1:.16.631/:..9 ../96.69:.6 / .. 9..60.9.963:65/96..513:..1&..:36:: #.94.. 4:.: '.9.50 / .6 .16 .9: 0:7.7.465..065:850:6/5441.5 326379610196495569:43.9:..51 .65.9 .50: : ..10..9.9.33/5:..6 4656763 .36/0./3 / /9151 .7960::69.3.9.79679..9..9..: .659:4.31791 6:3.4 55.65.:.0.....659: 03.25..3:65/96.9: .164.6.6679.: 5679.519850:..1.10.69545. 03:656:3360.5..10.5989..25 79679.0..69.65:. 796 69 10. 6430 9.65:... .5.51 .51.65:  .10. 0647.9.90:6.65.10...9: ..0.6 .951965..3. 9.:.335  31  33 /96.9.::.4769.65.469..6989.65:.10.6973 305::.69./96.5065:.51 679.69545.7/30 7. .6.0.9  .3:65:..3165/.. 794.:. 336::.6 7/30 .10..6.56...1.51 .65: .064465661 ..9 .16 69 / .:.  .65:  : 305:1 / ..9493 5.3:65 :.9. /5 .6:.56.65 6 1 7960:: 4:.1.679619.959.6.. .5 :0 9.79679..10..1 .4656.9 79610. 305: 69 .305:..9..991 964 ./37/3051:5305:5..   9.51./96. / 9891.9. .65: .50:699..9.6:9.5//..9.9: .9..:   : 9.6 .54696.65:1656.:.250:56.4..960..511. :6 511.51 ..69545. 7673 5 .:.09694679679.945. &.51 69 7/30 :   33:7.6 90: .:. / 699 /.5.&. 79679.31  ' 795073 6 9: 10. 7/30 5.. .  10..9 316 . &. .515 .. 5.691 .:65.0. 695.556.   ' :067 6 455.. 0.9:.9 6 455. 7969 145..9 .0.3:  .99196490:5:.    ':  . 69 .5 595. 69 .. 76::::65 6 . .6 9:4 .679679. 69 9: 10. .5:735..: . ..:...7730. 989: ...556.: . 9.: . 59 . 79679. 94. . 69 4731 9:9..33 ...5  . 9. .65   !6./:63. 455. &. 769 6 . 79.9.. 69545.0. 769 / .6 79679.91 5 / . /. 6 .. .:: .6 511...  0 51: .3:5 79679.7..51 796015:  0../616.5.51 5..5  ....9:.19.5146:. 164.65 5 59.   ' 9 5. 9..51 327630769 0. 164.0.9.5.1./9.9 :695 0.. 9..9.. .33 0. 164.59.: . 519 ..3656796:565.5:5. .4.: . 5.15!. 79679.3 ..914609. 69545.5%694  '9065193 511 :.1:.65. 06475:.65/ ..065.773 .656.945.164.79679.7..773653.369. 16: . 659 .61:76:6:79679.6 .0.06473.5:.915.6:.5.6#.::.1/069.31795073:065::.: :73.33.0...194.......6.9651.. 769 6 .5..::.73.3 .515 507.356. 8.: 6.6 .5:..&.6569/3:3...79679..369.3  % !6  5  51  ( 516.09:10....6. #3 50  &09..3 .3 .339./579679. .  9.:.50656.5 .: ./5.39.5..5.65   3179679...653.. 164.79679..315::60.16:56.7/30:69 7976:103.6:.6.0.5:991.65 .:4.069.:5679696979031:..45..5..:579679. 3.:0650951.6 ..95. 69545.3 145..79679..51:065::.9.06475:.0..6596.6:.56..469.6.51:.65 6065145.: /5 06473. 79679.65   9:79150 65 ..3. 69 .3#69697  60:65  '9:.*   .65.79679.1  659:769.:69.51..  .50: :. 0.: 79:09/1 / 3. .6 . :.9451/.2. 964 .9  0644::659:   ..65516:.6.79679.:6./6.45..69769..6/1.51.51.. &..21. . 3.65.6..6563.65:0650951.6   :.:::.345:... .:5631 5.2406...9..5*     :0::679679.455. 79679.8  %.65.6 .654. .65:  %7/30 &. 69769..25 '::165/.:   '79679..::964.473  ...3.79679..79679..69653 765 33 7.5.5659 50315.556..5.:6.0.90.3 .3.36.79679.1 .   ':06517.65 ./3 . /50.331:76:6.65 69 79679.17.:. 164.159.:5 . /5 06473.6/.: 98945...579679.5.654.. 069.65:1...3.65 .79679.79679...34 5:.06473.1:.9:.1   %7/30   &.337.590:.3  % !6  5  51 ( 516. : 653 765 . 5.45...6 :700 ::: 1011 5 ..51 .3 90:5 .90..4.65 &% 3  (#.%.5/.9.069.51 . 765.7..51 :/:85.69  31 (*. ..50. . 5   3.4 :: .6.06475:...65103./5.969.9..06473.4. 06475:...65 : :.5  .:   69 .  507.:79..9. :6.69..  .06475:..1:4::.55..65  .3 69 .96 79679.9 ..63:3.:.9.9:.3   %  !6    5    51 ( 516.. :.*    '3.479679. 06473.7.53.:6..1 .5.   '969  5..656&4.. 6..6 :..45.398945.: .3.945....6:.5156.:6537657...3 .79679.6.690:455.1..5156...1 ...65. &.  69  /.6/0651451 69.9.*     6.:90:5..65 5/6.56919 56.6  :.73.1.69769.: .6.0.: 769 6 455.69769.45.1.34 5:.:579679.5.45.7696455.: ...945..91659 65:85.4..9451...9..   3.: .5 .65    .79507369:10.690: ..:.65 6 ..164.:.9..65  %7/30   &..0891 / .6.5.6.45.. :/0.610.6569.9.. /0.3.5 69 .... 1:4::5.51659:5.089 .. .79679.:::.659:76..  .65:.65.1 .9 064735 .. 796: 0...06475:./69.39... :. 1.595.34 5:.79679069.!6 0..&630./.65.9.941690696.65796015:5. :.: .#9:15....659/5.677.90:6455..31. 06475:.79679.3.1. 796565045. 69769.3.16.:45. .6/79679.65.1 5.4507. !.65:56..5455.79679.3361.5.5/..164.5   31.179679.. 69.65.98979695:00::356.9.33./6..51634. //6955451.61 07961:  &     #69 6 455. 769 6 455.65 .5  ' #9:15. 64.65.6.0995.94:..651.565.679:09/.65.65 .51 .54.3  % !6  5  51 ( 516.5965. 69.:0651.3 69545.7696455.1 5.6 5:.69. 9:76515. 69.0651..958:..65   %7/30   &%   .5*     54./3  .90650951 6.4..369545.90.510.0.49313.:.96.. 5 #9650.:.. :765.1...5:. 6 :. 164.556.:/5..945 5 .5:56.51:.661.956.3: 7..69.677.1.0.6: ..79679. 06473..60.3.  . 57. : 50::.65 '9.3 69545.69.79679. 69769.65...11.5.33 1.164./69455...45.3.769 6455..69 59.79679.:5631.164. .515/6.    '6965796:6516:56.*     :79695:00::356.0.:519%7/300.1:.51&.3 6 .65 .6596.65 : 4..6 90: .51659:9.:.31:56765.398945.5..3.179679.65 .690:.3  59  14: ..   33.5   91 (#..3.: 69 56:0 635.::.06: ..6.1 3.164.95. .369545.6595::.06990. %:1145:. ....956.50.9 69 .7730.:65679695:00::356./90:1/.5. 164..65 . 164. .6360.51 190..5 5965.51&.: ..3 7.90:6455..563.6:.5:6515&0.7960:::56..9.:63169.5:.3: . 79.5. /.9 9:.:. &630.:.6&69:665 .::   31 55::.5.1   %7/30   &. 6 &69:665   1.79679./519:.5 5 /.:.9..0./.65.45.6959.79679.513:3.164.5:5/.69..65 6626.. 6513..5.6579015.: 476:1   &  36745.1    ':065.0651.6579015..7696 455... 9 . 6.9.1934.....5 :.51 57. 06514515..:.1 796015:  .936 90695 06475:.65796015: 5..5.33 / 4.5 . ..656.65:. . 3.. .136.7. 0647.79679.465.25 .3 3:3.../...1 '3.:.: 60071 ..653.5..51.65796015: ..79679.9: .65../5.    '145.9.31.6.: 7.956.51659.51 69191:631..5%'565.90. 0.:.9../6. 9 .1 .1 .:/.6.979679.:.92. /.:.:69. '90:6:09... .6959:76515.   .7/30: 690.::5 769 :.50647.65. .: 796565045..79679.. . 796011 .6.0. 79.:/96.   %7/30 9 ..   69 360.659.6 9.:33. :.3./3.45.7696 455.65:765.9 .6.3 69545.5:..065145511.:13.25 .5 .3316.5 .94./353..5.309. 9 0651451 69 7/30 :  .1...5.517976:: 90:116456569.9:.&. /5. 3:3.:15153 5698:.: 5 1:7..5.65365969 164.65369..979: . 5 365:6   #.51 .5.  .31.6.3  5.:.. /9.:4935.136..:79.32:..979679.1.6906976::::656.:0.:/0.95191/.6.3 .569653.779679...65 69 : 79679.11:.1 .: 4.69545. .13.5:56.9...5.556.92.51659: .17696455..9/651 #.6336:.1.: 73.631.6:.51  :6  5 .1 5 . 145.4   &.6967./9.:.1/.6/. 3655.465.45.. /141:..: .5.631./3519.969:06. .57.31.9941: 04.51659.169/. 5 :..679679.9.4:.3.769 .794::   ' 69.6596907. &.51 69.0.25697/30:79.145. 6 .9 /.931  ' 765.151973.16906551. .65796015:7961: 56..1 94. .1/.06475:.:33..9..9:0 7..36.79679...6 /    ':  5 ..69 145.69.9769..: ..510.79:.1.79679.::.:.65796015:99:.:. 9069 6 76::::65679679.:. 5./9.3:7.3.50 6 69545. /13.4 .945...79679.3.518..614.335.164.:.:. :.65659651:6565.50 69 :.4 6 /.5 .9/ 7947.5 03.9 / 79:097.9.7.  145.0...3 06473.  .65 6 . ::6565.979679.9./.65 5.65.9569.956.0.  . . 5 . 79679.51 / 635.5. 7..7: .3:  % !6  3  :..3  065145..6 .465.50   %7/30 6 .3 ..33 03.659: 79.9 .65  .51 517515.: 765 .10.773   5 5 94 796015  065145. 7/30 519 .65 796015: 7961 .. 6931 .79679.6 0 .0 .:...9 :..0.1 :339 5 6915.6.659   0.9 69 .5.*     '6 ..6 .5:.33.3:  . 79679. !6  .95 6 . 7.: 0644. . 1:7. 7.0..3 7960::69 :095 /..6. (.5 4. 941 6 9:0::65 4..6 7. / 6/.: .9..1 . 6 77..3 : 5 . 9:76515. 9..4.:569. .9 065.9 065145.   ..1 .5 57..:  / 5 56.9.1 6.3 .45... 79.9964.. #3775:   ' 65  69.. 4:11 .556. . 9:76515..69.69...65 .  69  9..  .. /.9.0.647345. .6.:69.65 1..1 #9650..: 190..659 0.91 .96.1   &631 ....::.3 7.3 69545.3069.:   : 5 6.5196915.9..:: 9 .659:6.915...:/:/0..5.5.5 9.931.51103... 6 &69:665 %6:.913. 6/6: 941   69  . 1.45..50: 441.51 0....6:..: 56..796505.1 .36. .9::1 .465.51:.. 103.3:6:. 7.91 5 &69:665 .50::. /065:191  :.69   #.9465.6.9 .31.: 69. .5: 6.:190.95.45.6:7769.3 ..6 441..9:. 6 &69:665 : 70.50...:065.: 365 /064 5.:.90904:.   ' "919 6 79679.. .1 79679. 9.:69.1:.. 935 5 . #9650.::93...65    ..65.9. 69 4:.3145 # &% .1 3     . 0. : .690...9.379679.6&.956. 69545.9 0.5   51 ( 65*    '#696'..57.25697/30: 7/305.: 56 79647.66.:5. 33.:10:65 69 .1..57..3 69545.../5065.:.  69545. 69545.65  (*.:5 .96..: .556.9:65.. 69545.9..: 5 .9 0. 16: 56.9:3.:690655./56..7.. .517. ... 5 .3.65/.. 6 .. 6 .:  . 4.06330.6 7.50..7  6 .06.55...:0. :.9    .9 1 .3%5 &%    (%6496*     519 9..5:5..9 1 . 69545.69545.5:.: 069769.7.:065.5.779.6.65 644::65965.33 .9.:.5 . .511/.95.9 0.19:3 .9.9 '9:. 7515 03..091.606475:. 03.0.7..: 06473:69 9.65  31'.0. 1751 765 .7....: ..  65:.5 ..:.56906475:. .0../3 5:...69:..5 19 . 9: .4:. .1.:4739.: ..69.45.: .65:  090:.556..9.   /... 69545.5.51 .5   50  .:..9. 95 ::.  . 0...65   7.69:6..33 1:473 /0. 15: .31..9 ..5.. .4./:/0..7...7..1:....31:.9.6 .::6950..:.6 46:8: 565.4 69 951 69 091.4 ..: 631 ..  3 ..03   &0.: / 9.656.6569.4    .65.9.:65. 065:5.51 1/.6 ..  #3 5569769. .9 6 /.. 7.956.. .. : .5:..5.  65064 1/..5. 44/9 6 .: .9733.3 7976:: :.:. 65064  . .: .3 :...0.95 4.51 .* . .    .51..65.9.65 (*3395:..3. 65:. 964 . 65 ./36910...96.. 5064 .5.47.(.06509:  :......37976:: ...0:32:0691/:.5 455.767359.65 .1796:6579......9: .65 .65.: /3153.0.51653.1  9  .65 7../0.4: .5103:3:169936: 0.65:.679679.9.: 5. 44/9 6 .65:.6.9 &05....9. ...5 .. : 47.3 0644::659  6 : 56 . 47.03 &0...:65 .147.11..    (#.65 9.96   ..1 : 56..65:.51479645.9..51: /315: .:   5 : .04. 4:..191 .6: 47.1 964 9.33 190..65 069: 79679.65  %:63   31'1/.51.85  95.. ..925:7.65     03.06:   ..5179::65:6675656.33 190.95.5..9.: 653  644::659 6 5....0 5..../369 10.3..51.1/:.76.5.3 %5      &%   "0.49:6.65.9 3.3..:  .3:6 .9::1 195 ..::..33 / 47.: .. 6949 065:...9.:1.: .5/.  :.5103:3:169936: 0..6 .5.1   0069153  :.4. .:7. :.796.51:  /315: .5*     5199.69.679...95.: 69.51 479645..:6 565..6509...:3    .951 / .. 5:.33 3..9.65  9  :.0 3. :.602  565. 51 03:3 69 10.331....65.65...3  ..65: :1 . 10.33  190.:.:...0.964.3 7976:::.3 5:.796.:  ..511.565.33/47. 796.65.3   .10.. 365..3 7976:::6..9 :1 ..65 6: 95: .: ..33.515064   319.3:964..95.9.51 03:3 69 10.65. 5064...45.65..51 .. '/..33  190.964.:..:65.964..:47..:79679.56.:79679.5:.7.:47..565.::.79..3 .9:76515. /..:..9..679679.0.65. :.602 565. : .796.6 / 9..::0.1. 47...5.  .33:519.65565.03.65.5.47..65.:/:.65:5:05./60..45.3150 ..   ..90.::4 9:   50  69 ..65064.7...4:..:47. 10.   (*..65 .6:.65964.35:.91 :..3365 .9 065:.03.1796:65 ..4:519.796. 602 565.:. 7969..03 &0.:  644::65965.198:....33.4..   (#.65695:..5103:36910.5064.:..9 6.::/4.35:..796.1964....65::1.65.:56...6563.. 65:.65  3193.35:...37976::  69 .5*     :...0891 .9410.3 .65.65 7.56.33 190.:.9:76515.95.56...510...: ..5.6150. :2:.10.51  .65..35:.0....1/79.5/.6796 .69.65.3%5  &% "0.65.955..6/47.65.5. 33..:05.. '.. . 4 .959.51.51 79611 / .4: :56546: .050.0.03:65069769.51 69 0 09.3..91 .65.55  6 0 .51 .451..11 3.65  0 99: .2565 .90.67969::.51 09656360.9.3 :0663 ::.65 : 98915691969.6 :0.3 644::65 .9499:... 519.955: 69..933: '69. 65:. 694.10.6 .96.650.3 4.45119. .6.33 :.65.3 10.900.9 141065..6:0663: ':0663::.5.:.1:6946. 9.33 9. 694. 44/9: 6 . 55..:066369...4.65 0.510...:6.55 .10.6  5565..9469  519 .65156...65   9.65...:.:.35:.  /. 3 10.661 .694.6 / :0663..65: 519:. .5100.65:.5179..:1./..:70::0.:651. :9.:.6 9..65:./3:45.33:3.35:.647964:945.5.3796:65 519 065:19.51 .65:....::1 .3:.95.. 6 .3.6. ::..:.5..360.3130644.964.5 . 44/9:6.5./..10. .65:0691/.51  .647.65: 5519....337961: !6 3.6 . :0663:45.556.65.. 659:: . 47.65: : :.5 .5.:. 5..79.65.33 .5.65 :. 06509950 6 .69.33 / 7./141656.65 :700.1 5 ..#.69.9410.51/ ..9  31  ' 769 .51. 47.5.31.6... 0696..5.65.99316.65  9..6/9.3 %5      &%   "0.33694:6.65..5./:63.65   644::659 6 5..906:9:5.5 ... .47.:79...0. .356769. 4...65 5:153. 09. 9.. 9:76515.65: 99:.6949..5*     .5/.:.5./61: &0.659:: '#.  & '969 ..6 .60647964:33.     (#.35:. .036..69..9 06336910...79679.169.065:.47.. 3315...356. .::   5 9. .: .6.5..65...:.655..6503.: :03. .33 ../3   .9.33 / 5694 .3.336%..5.5 . 065:..906:9:33065:..7.51 8.3796.5/.696536...65 :..5*     :0::.3:6 63.:  31'%#..::.9 79679.. 3..47.6.6.5 .::3:3.5 . 47.5. 659:: 5.8.: 65 .0.0.   #  &% 0  (#...65 .5.3 93 .:519...03.0.7976:6.39..906:: 964 7.:  5:70. . 65.65 6 5.95..1.:5.&'.95.95..33:3.9..9.. ...65:.57.5..9:10.669..0655.7..  .655....3..0.3%5  &   65:65 .::.91: .9..:: ..3 16/3 ..65:  69790:3 .65.3 910.61 :43.65.#3775: .6..50616/3.'9.919.:5..6195.54/96/3..3.637.:56919..6906503.656.945..9.:0..065.51&65  50   &%   .3:0.. 345.65      644::659 6 5.9.915.65:6.65 '7976:6.61..56:. .  5  91  (65. :.3910.:5 .: 65 .:../3..6.65..316/3.0.51&65 50 &%  5       ..9.516915.: 151..669 469 :...65  31  .476:.65 6 0647..37961:  644::65965..4.95.959:70.4:/0.7.4 ..95.%:*     .9.:5.3 %5 &  65:65.: .. .9.316/3..5115....516.5119: 5064964 6965:0.9.05.....6..9:  0651.6.6 09..95.:.6:9.:.6..9.9...95.9.....:.506469 0..0.:.9.51 79:65: /.:..:06599165656../476:1/..:.6.6169.:.773:59.35:.7.5:..0.6:69069:..: .1.:33653. 4645..65  31'..7.9......16/3..:.65..796.77.937910.51 :90: .503:9.5:.6.9..3 5:640.:...9.65:090..   6/3.6345.16/3 .4006564: 6955:.6/.65069.3509.51.....:::65064690.503.0. 6 0.465.16/3.33...4:65064690.034..3 56919...65:.:0.6.61:9:69.65.4.. 4.65.3.5.16/3.9065.3  .61:693.......065..3 /6..  ...65..6.:69 696.596/:.69:150 5./35.:.. 504/5...35 .519..95.65... .369165.9.65.95.9..6.3509.  ..:476:..5:./3.69:150..6345.936 6 661: ...7..9 5.65..3369356919.3910.69:150..0.....336934..9:69..6...2:73.1   ':06514.61: 9:.6.:.6:690:34..:6..7.51/6.7...:... 065.45..034.9.2.91.:.9.5:.6:690: 5.05636 ..65:..6 4...:..9.9.5:0.6.. .6116/3.5 065.6...19.:65..65616/3.65.45.65 '9.34..369165..345.65:141...:: .9:70.79:65:9:15....65 .51 ...765.:.:4. 9.:.:.9 .654.47.7.6949: 091.../3. 4...6./.9 '/.61 ...../35..6....7.3 0:.5:.4..:3 9...9455.7../35.50:..3%5 &  65:65.4. 47..6160::765 .51.69:150 ..61  ...15.61..64.:.7730.95..6:690::..255..15..15.6 .6 5:645:.5.3...3  "5 .315.51.61 5.7.4.5064690. 091.7...61::. 6...1.091.55 5064 69 0.47.6.61 ..60::65.33.0065...69:150 .  644::65965.5064 69 0.3.3.:01950/..654.5..9 : 94.51  5.7.51&65 50 &% .3.30:.6:69069:.654.091.:.:..::47.5064 690. 03.656.1 .6..:0179.167.656.9  644::65965.5.5.9.....179.:3.3 1 7960::   31&0.:2565. 5 7 69 ..:165..50.65.9:6:/:..65.317960::99:.:.5.65:165.9:6796019..1..3%5 &  65:65 .796019:/ 0 .   6965.6.6..43 .5.:5.:796:65 .4..9.17960::989:.   5. ...1:.559/0.51. #3775: 33  7 .:3 56..61694.9.6... ..3.50 5::50 796019.654:.#3: &% 0  (%6496*     %:76515.1/.9 5:.3699105.. 631 / 56901  : . 6 .. 5 .60651.:.519359. 3.50617960:: 565 :7..9 :.:::60.:6.9.17960::03.65 6 ##.656..  5      ..#3775:..2:617960::63.9065.65:4:.:    :0:: .5: :/:..493.: ::60..9/69#36.:56..: ..  #960:: 3.9/69 #36.54.... 3..17960::.33.:6.65 ..1/..65   56919.:56901 3:/:. 69 .9  9.95.9...65..556..9. ..:   :..796796/:9... 6/:91 5 . 7.06509 5. 70.: 7.51&65 50 &%  5     '"%'&  '#960::3..51 4..9.51 :. 1 7960:: 964 796019..../3  ..3699105.../4. 65:. .:  31556..336%..:56.:65.5.9...45..:.. " !6  . .69.5 .. 6 0.90..6 95. 91 .9 .. .51.317.:0.569:69.3394..:06510. 90: 6 .45.94 6 65   ..1 .6.: /064 :.69545.617960::63.65:5..  / 653 69 ..9/69736..51 03: 9.5033.9 06510.: . 0 7961: .51 .50: ..65 / ..:56./5796:3::1/.51/6/:95.3 69 0.19/93..9: '. .50 .33  5069. #3775 #69. 33 :..3:6065.5 653 / ..5 93.511.: .9 964 1. 6 .3313/0.9/69736.65 6 79694.3. .:63.065:.51.51 0.7765.##:.33736. / 736.    . .7765..6:.76:.9/69 736.:.9.  . 33.95.34.64:69.6 .: :.604/9653 .:.19.5.4:3:06315. :/0... 1:..65..3.45. 0.9::1 5 .6.5.9:./. 69. 69.6 :2 9065:19. 9 ::50 6 1 7960::   6969  .50 .1. ..5...6 151:5. : 365 ..:8:. 0.3...: .796019.: :.:51069: 0.  . 67769.5 67769.: 5 ..91 .516   5. .: 67769.: 065:.9 76:..0. .: 9.96/6:39995. 31'..:1  .6.0.5.65 69 935 06473.5. .3 . : 141 :.91 : .65 5 ./:.6 / .3 4. 03.: 6 481   65  .:70./515117960::63.: 4.6. . 7.651##.51 .91  .51. 905.45.65.5.5. 103.. ..5. 69.9. :.9 9:76515.1 .65 6 .556.:. . 79:65 : 9.  9 . "!6 . 145:.679:..:5..9.51...31..3 653 5 .15.1    !..:..569. 4..5 .145:.0.31.065:3.4:3:956. 6..9.1.4:/69.9 ::5.:.5.333.0..736.45.3 '9.916:.59.9.:6 796019. /61 90:: .6.:'9/5.945..393 56..::069 69 ..51.: 8.95 ..5.398945.6919 :.3 1 7960::  .563::..:65...:.. .511 .145:.50 .3 50.617960::63.:656.945.3:6 065.65: ..:99##.: ::5 93: .50 6 .1..6 .10.9. 90: 6 ...19..1964.51.9/69736.:.98945.51 93.../736.34. 6./615156.95   765 .31:: %:76515. 50.65:  :0 .6656:/:.:.0.9 . '9 .06473..5. 69 3:3.. : .51 03: 9.569:69.....51/6/:95.510.:/064:.65   5 .51  .::.51.3. :63 .65.9.065:. 79694...::.: 0.:.5 653/. .:. .19.796019. "!6 . 0.0.5. 519.9.9. 3.0.469.95 .45.379.5.9.64.0651.0 .9796::65: 4.55.5.:..0. 5.3:.65/.33 .7765.65: .60.9/69 736.0.::.65:..:: .513/.5305:: .9.503. :.5179.5.7969/..50.96.9: '::.51.94:6##.0..796::65 . .69....0.0.:.91: 305::. .   #36.5./3:1:.1/:640647.45..9.0   '9305::9..3   69 .19...56305::70.5736..5.65.336:.6 79.9965. 305: ..0.3.0 .15.69794::65 9.6.0 5.6.:69796::65../79.063361/.3:6..33.6 7.9.6946.326.5./:5::6916.3: 05:9 :.679.25 56.36.4.01653/13305:1511.: 0./.:.5 .3 .:0305:  631/33.69..9 796::65  .00691. "!6 . 604/9653 .3690.9/69736.5. /65369.593.95..7765.6.33 5069.9.99641..65:5 .65679694.:9.906510.. 33..77.7765.  (.6.33736.51 .3394.##.13.5033.6:..1:....317.60.94665  .45./5796:3::1/.91.:/0.6 95.76:.64:69..:.3.69.45.50   . ##:..*33:..:0.90..65/.9.::.. "!6 . 9 06473:69 9.:.65: .   5 . 561 .::5  .656 . 2565 .  ..9/69736.90. 4.9 7.:.9.5139:.:9 6 :09.45. 7.6....945.:.9: 6 65. .51 51965 .9 796::65 /69 . .. 3 697:0.9 79694.064 6 . 631 .5736.1.3.6/ .:..0631:5.5 / .50 .99.55  .65 .. 17515 65 ...50 .32.##6945.9 69 794.9 305: 0 0.305:0.5.30..51567.55. 53:::66599621/.9..35.945. 6.9.65.9:1153065965.6065.3.51.55.:.4769.656.5033.5:: 519..5::. 9305::5617515.500:06510.1653.0.91.:6 ...36.:.94:07:6 .65 679694.1/50.. .: 79.. : .9.516...39..94 95.7961 %5.3.679.50331 50 .305:.65.3   .9. 5033..2: ##.93 4.65 0.3...65 0 794. " !6  . 15.:56.0945.5 .33.: ::60.796015:/36 '5:.   6965.69.: /56216990.:69.5.5 .145:./:63. .3796015..9.4699. 6 .65 6 .: 56.6..1 / 065:3 195 .6 / .5. 9.9:59..514.6065:3 00.: 50647.9/69 #36.0644..153::.. /. 0647..51 517515..50945.5.5..3 :5:  .   5.:::..145:.9.1/.3 796015: .65 6 79679. 1 7960:: 6 3. 179. #3:   &%   04/9 (%6496*     6: .00:16.35:..9.33 594   5 .9.: . 79:506065:3 :.:637976:61..69. .00:1 5 0945.:: .:65.. 9.61.6506510.00:11950:.5 .51 065:..3.65 .9.515./3519./3 .. 065:3 6 : 65 060  481  69  . 589   5 .556.6 .:65  5 .1 . : ..9  7..51 .589  31'9.145:.6911.   . 9.659: 562 .. .94550631/31.. 1 7960:: 03.65. 59.::70.. 0. ./. .. .145:. 5:. 9..6 5694..519:...61. ..5..6  .79:65:6 6915.94.: 3.6 . 69::6515.:  .9:76515.51.33/0.16:56.. : . &%  . 4:3  ..55.06473.1 / .1/065:3 99:70.65.33.55 ....1/. 065:3   5 ..5.69545. 79:65 /5 5:.....9.0..5:.:  0945. 65:...9.. : ...51 ..01 5 4790: 3.6 1 7960:: 53:: 9891 / ... 479.5.6 1.../.65:.6:6/:09.6 . .....6 10..7.93::.1.949306015 4790: 3.0 .65 5 0.65 5:05:.:..5:.9./:.65  #673 3..6 10..365:4:. 0945.6.9.:654. .    .3: ...503.5::   ' 065:..94:6.9506510.   31 7960::989:..4: / .7..... .. 56.91. 69 515.50  .9.9:. 45 6 0644655..:796/.00:1 5 73.91: .790...5199.15.53.:1  5 0 0.0.5./:.0.0 ..503.655. & &794 69.:.::4/3 65 ..609.9 56.0.95. 975.99:.       ':  .5..33500.9.33.3 69 .50..:.9643:3..145:.3:3. 5:.95.00. 3.96  : .6:.:/56943.7976:64.. 65 ... 65:.65.9..55.02: 064795:/3 :.3 796015 .6..9..90./3.. .1.6.1 1 7960:: 69 .9:   6.6 065:3 : 56.3 79.:975.. :.:796:65:.:.: .93.1/79679065:.6.. 49..75...1 .4./3.1.96.91 . /61 ..: /.: 65 :0 .:::...1 /.5:: 60.4:03   ..:. 4.0   ..6.:795073:5#673 !.65  3 56 0..5.0.. 796015    .. 65:..65   3 5:....65060154790:3.9 06510.9 6 0  : 5 56 ...5.5.6 065:3: 56.514736: ./:05./03. .5566. 5.0.7:6.4:3: 5 .1 ...93::..9 56. .6 ..90.556.9. 5 ..39 .:/59991.6 . 9:76515.9956.: ..1 . 481 ..50644.. :.. .56909:5/91311:09.7730.1 06510.6: .51:75:.902165.65.669545. .. 3.1 .96946979:65:.773.3.6 ..65     :.5894. :.50::.0069179:65: :70..: ...:  ..6569 .65:. 63.06..9 5.: .5 65:  .  /.90..5 . .51195.9.15..:.: .1   .516.3.9.51...  .. 4..93 .93.93...1/.05 4 65 ..97. 4. 69. 69 .96:..9356.1.9.51   . 7.:.0 . /61 4.4:03 .06510.5069.5519:. 69545.9356.91:.3:.0   .:31.6915.9..9.:0644653:.9 /.9.7..9957/30609:.00691 79:65:  :70.. . 67. 3./3:5.65: 06510.5   :.14..:.0.43..3.5 .3.65:  : 61 69 ..3..1:0735. / :../3 .:.6.694..5:.5565 .:69/115/.9:/0.15.: 56.06510..94..0.3 :.5.25 .. 61 69 .61 .556.: 15.: :0 589: .: ..7730.3730.6.9 .50::..51 56 1.  ... 93  .6. /.:.3.0. 0 565...1 493 . : 975.65: /0...:90   ' 9.5:.::5 /  3.9. .51.91 .3796:0..9.9 .5653/5621 . .:05.65 5 .9/..: 796:65: ...0.955    9.91/ .65 06.0.90..4:.3 79.473.592565..5..91..6 / .. 79:65 6 6915.6550.9 ..:9:. .51.9/.:4..75.:.33504:.65 6 5. 5 . 06510..5::.. .63..9516:56..95.1 / 79679 065:... .0.:.5:..65  :.65 ': 56..: .:17960:: 69 .:4.4 ::0 ..56915. :..9 06510.145:.77.335 4.51 0650.32 .145:..4:.3350 .:.559 ./.4560644655.5:.: 3..50476:156:. /:.:.5.: .4/6:. .33504:.:69/065:...111 69 .65.6 / 1:.06510.51/064:.6 79:65: 7.. &% !6  (%6496*     :0:: . :90: 6 065:3 69 56..7.339519.945 .6.5:1  69  964 3:3.3:: :70: .51  : ..956.5.6 .3 0 5 . 0 565. :.: '9:.65:.:3.6 / .9.:75...51 . ..369...50.5 .. / 465.9 .69:70.6.: .0945. :0.9.. 5 .39 .6   9:70.790.5*     '160.995 6.6:6.95  .:./03..0. (.0.5.3:: :70 .6 .7730.51.5..4.51 /064.0945.. :.556::647673/.5119. :0651 59 190.65 .#19.:..02: 064795:/3 :.. 56909: 5/9131 1:09.51.5 :1.6 979:5.65055. : 065.1/. 9:.160.9/.15.:7063:3. ..65.33 .:69/065:.5:.: .6 .5:.:1 69 ..:.6 95: ..0 6 . : :.1.5:..51 73..0  50315 79.90.9.:.:.5   :./651...9 . 796:70.1  #673 3. 796:669.3065:19:..5 .5. .: .:09..   31  : ...91: .. 796:09/1 06510.33 6. 999.0.7730.:65.5*     :.:0:.556...943:: ..50.1 .9/.: .3 ..969  : 56.:..4/.65 / 50647....01.    :.9 ..5. 4:. .. .. .:.5065:.950.:.6./5469730. 9.:69.1 / .5. 5.  .6 7961 ./3.:    '..5 5 .56:9.. 4.7733.5179...9 5 .0631/0644.0.7733.9: . "     31 .6 / 69/96. 47369  16: 56.:..45.796:6 '. :794::/3. ..:91 / 064465 519:..   &.31:. ..51   7733.9.51 73.6.: 5 ../:.:50/4939995.:796:65: :70.3 .. 3.02690691:..00.::6/96.9 6 .45./::.669469 79:65: .5. 065: . 790.515. 064465 76:.: 03. 796:6 / .1156.19.9455.03  /  6 .9.5 6 30.5.9::1 69 .5 964 ..: .556. . .5. 6: 6/:09.65. .5:: .65 6 . 4.1.#19.69.54. .0.94/9.:7.1.:.03.0.5.9 :...33796...5::     '.: 6 .94.: 61 69 69/9.315./065:..51 50 616969/9. 3/3.6 . 15.:.   ':065. ..   7733. .3:.65:0.0.659:4:.5: 493/46..95...562: #673 #.65:6.6::..45.. :05.:15.903...:.045.03 / :6169. :9 964 69/9. 1.65..19164:..95:493060154790:3.6 ...5 473645./69 69 473645.515 .3351796:65:56.:3   &0. ./..4.515694..045..&0.. 065:.0.796:6.5./3 5.1 .556.6169.5 ('91 :65*   %  !6    !6   5.0.:0...9556.95493989: ..9 .50(336:336*     6: 9...64.979.4 065:.::.9..5  ...45.:.0.5... 63.7733.3.:65.#3519..51.  69 473645. 5.65.69:70.334794::/3. 9164 6 :70 69 9365    59.5. 6 .: 69 . 5065:.33794::/3.3 9164: ....5.31493 /0.3 3. 679.65.33 69/96. .65:  69 5:.7730.4.0945.559 7964:1 69 691 .   5 36 47.1 9 .77.3 ...63137:13..3375:1: 5065:.6 :..65..655165 .045..51 33./319609.5 .1.93135. .5:... .556. 76:.51..675:06510.5.01 '::5#.955 .0 .03 / . 5 .. 065. &% .0.. ..09. 796:65: 796/.33 .3: 5 .. 5.9.4.4699.51.656909.15..06915065:..5173.. 69/96.1 5.0 .:.51 19.::.5 4.79:654.6/73156.036::700.3..1633.9.5: 9. 631 9519 .65 ./0650./:506:090691:56..51 73. 3.5.5. / :. 0./3 :.65./:63.9. 9519 .3:61791 .. 9.6596...45.3.0.50   :. #3519 .045..01/ .065:.7735 795073: 5 0945.65.651 / ..065:.9 165   644::65 65 30.6531791.4:.5..336911:. 31 5 .3909.:..6 5/..:6915:691.9. 5 4.51.65.77951: 06507. 5#.:.65 '796/..3356..51.:6169509.5 73....5/...3  .91. . 0..65 6 909..79:5. 69 ....    ' 69. :.3:.: 3...:65 . .51 5. 909. : 9.33 . 1 7960::03.:::.65 7967.094633.18.:93..75.5... 9.3 15.5: ..50 ..: .1.5::160./56219.51/651:6..: 365.5.6. 90: 6 511. 6 69/9.969 .5:70.69.:  6:7 90. .. 3.519964.5.3 909. 03:  6. (..5..:0.:6:79679.7733.5065#673 #..339  /0.   '69.:5.3 9995 . .51.9.902 165 .99.. 631 / 476::/3 ..1   15..51.5::4:73..9. 65:.79:6569473645.1/79679065:.1 .69:/4::65.65  :0 .165359... 065:.5.1.45./315.:56...: :0 .160.9..:  315:7769..369/5.65.33.0.9 519.: .65  /  ./69 61 155 909.33..65...790:65694. 65   &0..9: 0 631 5.9.00:1 . .151 7.945 .4.65  :   ./3 .6 1.9 6 : 63.. 5.  ..31..5.    :365.9..5.91 . 065:3  5 1515 65 0.. 7/30 609 5 ..6319519.. .:6.:75.:  06510..3 730.06510..03..::. .6315694. .9 :/0.6.: 1:097.9. . 5 .6.65: 9891 69 69/115  .65 6 .65 .631/::.:05.7730.. 75:: : ..9.514694769.5 / 519:.43. 3...31 :.:.: 63.5 33...3 103.00:1 515. 6 .::5 69 . . .3 .3 1 .51 .691::64064795:/316993.9.796:09/106510.6:6.:.65.3...0.661 . 1 5 .: .094.51 0651. 4:.346.0.. 0.:65.51 79:09/:.5. :05.517./3.: . 511  .3...0043. ../309.4.345. 51.65.6 / 79 :67:.9..69 0644. 33: 03.0.15....955&0  ':64::65: .96 .    6969  .3.796/.0644..65.6. 3:3.9:..0..5../.:6.9  1 6..4 40 3:: 16  ..6. .6 : ...51 :9: 5 .50::.0 .45.656.691:473615.55.939:3.50..6 15 9 691  :   :1:  .: 6 .661. 33...18.65.6/569416.517961..659  9519 .5 .9    :.:6.#3519.. 659:::56...:33 .:.0..../34. 3:3.:3 036:3 .6 15 .6 7966  .3: .#3519..9  1 .94: .5 / .94: 064/5.094.  ..: ...:. : . 6 .65. 63 .9.65.5.5 .. . 55605.65: 03.659  69  /.0.4/6:...6797.::9.:.4 50 63.9  69 .45.345.96:345.6917. 6 .5.51 9895 .4511 5694./3..659:06473.51 69/96. .6 7.::9...: 6515&0  7.3 .9 :1 .51 61 493 /0.. #3519 .  0 : 1:.: 59. .1 . 064/5. : 56.    5 . 3.694679::656.6 :/0./309.... 5065:.. 6 .9..:65.006915.: ...6.65 .3 ...969.65 .9:6/6:3565 . 9.65 6 .91 964 ...510...:.. 2 79... 6:51.:.67961 69 . 3.  510.00:.617960::    ' 9.4.1. .65:07...93 :6.69.659:.5.065.5 ..3 69 /5 4794::/3 .65 :4: . 3.3.  0. :9: 69 064/5.00:..51&0  . 95191 509.3 69 :.3 79::15.69 15.: .65: 7. 3.90. 3.3.9 : 56 76:..:549.:5.1     1:09556. .65 69 :9: 6 69.5:. 69 0945..3 .5:. 155 .:6 365 ..6:615 .94: .331.65:.4/. :.51.15&0  7.5::69..5.3356.6965. 33 065: 7..3.:3519:. #    .6.15:    #.02: .. .95  69 /0.51 154..: 765:05860...5 5.0.39.0.9..51.9.:.9 ..396 .55.659 5 : 15:   3.:.:6.51 9 691 5 .335.3569416 .0. 473645. 065:.. 30.69 6 549.9    .513069.:7.3.3..56.9065.0.65.65 53::.7/30.259.65 .6/519:.13/9..115. 691: 6 . 4..9:!633..3.4.29: 6.65 651 5 &0    7.#3519..633650644653 .:9:.1 .369:70..3:3.9 5.65.65  4.9.691:064/5.31 795073 6 3..6979..9   93  .:: 0645 65 . 631 .66/:09511.97673.51:9:.007..6/955.9  1  :/7..559:.4.: .335 519 .6.5:5:700.9      "5.51.6.5.691:064/5.79:.519.65.659::5./. 54/9 6 .9  1 :/7.6064/55.9..:15.007.:6365:0.6  .511 .65:7.55.656.95.6615.51:9:   64/5.3 69545..39:..:.:9995./3305..  4:.9 . 33 / 5.5: 69 5..0.. :..6979610.9 5 :7.65:65.050.       ':5.6::.55::79:.5315..9.:..6..65 .511.:..979..6:0036:93.0..3.3:00::65    '.4 0.6 .1:65..9 6915.9.6:691: '5.6  6946969..0.516915..50 6 .135.690945.511.369 1:...3. :.335 519195..9  1 :/7.9.65.9  1   9. !.. 3:3..94:.69:69549..779679.#3519.2:6064/5.3 73.:9 ..0: .9... 065.5173636:.51 ..4% 69...337961569.9.6065:...519&0  7.    .9:   &9:  ..3360.964.55 69 064/5.5:.51:50.50.:.33.65796115&0  7.::.656.:.65.34.4769. 3:3.:9 5 &0    7..0.33/0.1:65..7/30..97.4 03.0.697960::6064/55 '6 064/5:.51 :9: .9  1  :. :.3 945..050.15.:79:41 ': /:.9:3.33519&0  7.95. .65..: 6 ..5..: /3655 .93 5..5...9 4.3.51 9.65. .. 7.3 0 : .  .65: 6 .51&0   519&0  1 6. . .9  1 ..91: .6 ..31515&0  593.5/.: 549.: .:.6   6 &0   1  &06513  79:.95065::. 3. 7/30 609 .95   .1 5 :/:0.9 ..51..4.6.089 33.5 .65.::  .6 5.  : 69 7. 69 0945.0......064/5.3..5.6569 :9: 6 69. .7..95 6 69.3 . 6/:9.94 : :05.6 &0   6 .1 . .65:   . &.3 .:6.. 69 .: : 190./3 .0043.0..3.6&0  7.. 69 0945. 064465 7976: 69 6.  .0 6 .0 .5 69.51:05. ..0644656.69.:/3/065.065:79.0 :.1 064465 6.5.3      50 .46336. 59.6 .95.9.0...6947..3   5 .556.3.955. 0904:.3 73.:4:.94 69. 79507.516. .00:1....5 .9:06555.3   519 ..913  .3:04 69 065:79.0 .33 53.9 .6. 56.65 69 4. :04:69 4. :2: .3.61::1/43.517/30609.3 :04 510. .3.0.661  .50 65 .73.3...73.6.0....50:  7.0.16:56.33 :0469 9 .5 6 .: .61 0 .00:1.3  : 0644653 519:.6.  .9.6 75.9 : 56 :0 69.   51 .33 53. 61.511.1064465 6..69:.9 4:.6.9 / ..3.690945.659: 93. 69. 95:4.#3519.9.65:6 9 :.01      6969 .5169/96.:.. 195.:.  516.34:73..33.5::160.. ..167.005.5:...:.116 56.  ' 61....65:.396.6/:9....31.. 50.0. .65:6..69.13/9. 69. ..33 59.5.0..95 :.5:: 160..5.94::6.3 6 1 7960:: 6 3.33504:. 0 .1656... ::5.365:.65 63..1594.50::...6 . .. 9:.9 6  &    1 (*0.93::.: . .: ..: .  655.4560644655.: .: 4.51 19 .55 . :.....7730.9 69/1: 69 989: . .3 7976: 4.5..6 / 4. .5 3. .51 .51  109: .: 93. 6.179::65:141.6065:..65.69:70   ' 69/9.51 ...567565/:.0.. 5.  &     1 (*&3.:   ': 9.. .1 .1:.9 5.. 33 / 795.1*   ' 76::/3.  5 96.1 /..65 ::.  &  &794 69. ..76::/3. 6..6 9 :70 0.3365.3 0..93/96.:5. ..5179019.1/.0. 56.51 6.: .6 75.1:.:6693/96.9. 9:... &. :.79:654.991..365  .99694 0..3 16: 56.0... 765 796.66575:1:6... 9:3..1... .01 / 4.9.:5.   &    .56.90.7730.:676::/35/.33:60.9:4.021465:. 69 796:09/ :70 .0.3.25.: ./1..5 .19.13.5794.3361 ..50: 3.063156.0. . ... 4..5 964 ..:65 06510.:.50:..1:71.3 "79..6796.  6615 3:65    &         1  1    (* (5..3 .3361 69 .3 ..95 ..90651.5:.335 : ... :70..996:700.335 : . .3 :.03699...31.6 .50  ..:.3 5 ..5698945.3.943 06510.65 '029  &    1 1(*    .4.95..9/0.3 :.569/9..:.:.64.796./3.0.1 :70   ' . (*5 :...9 .. .33 .69 : ...6 .1.69   &%    (*   ' 69/9.33.95....6 .6:60.77.65 4.5:0515..5:: 160..: 9.5:0:7550::.6:.7. 160.19164:  !# 3.6:.92: 64.1 964 5..51 56 9.3 86.:   945...::   ' .34..1 /0.. 065:..9.5..13 .0.51   .:.0.:53796:0. :60...:3.69: ::5    ...0%58:.102 "23...1 065..65 653 .33796.5.94..7.5:645796. 45145.0.51.1.:1 .20.3 0. 6 75.6 65 0 : 69/96.9/5. 69 ..95 6./93.3 :.5 .9 9 :. 59. / ..3.. 69545. 5.60945./.:   : .: 6 76::/3 0335 0.9..65 .95  65 .:65 ..556..02:65693 /96...773 .: ..1..1:7066.65.9.690. 160.     1  1   . .0.30.5 ./3: .33 . (*  ...335...3 69/9.69/9. 0.51 5 0.:: 5635 :...1..631/..6915.:/531.4: 5.6/..1&..33.9.:5190.51 .9..9:6..7731.5:.46:...:.. /5 5.6 465.3359 4:.03..3:3.90945.51. 931 . 6 0904:.335 ...6 93. 69.:65 .1 06510.6.50::.31  5.0.3155621.4: 6 ..653:7625691:.:. 56 :..:9.0. 03. 0...: 0  / .. 69..0.. :   . 103.6796..1.... :./5 09.: &.3956 :79.. :00::33  :50 .0.. .9 .94:  :2 . : 5 :64 06510...9:  33.: 50  &  .335. 69...:.5 6 . .5:.95 .93 796:09/1 0.4.7730.65..: 37:1 64.. 3.5:: 6 . .: ..: 76::/3 .0653.. : 03..::.5..5::160..5:. 5 ... 06473. 6 6.3.33 .7731 .65:    73. 6 5.:.5.1.6 .664. 06510.: ..0.. .556.  : 5631:.95:6:.0...7735.65 6 ..14..9 ...: 69. / 4.... 0.65./5065:. 0...:. (*   65:85..3:6/.   5...9151. : 065:.. 9:....02:65 5.5::..51.331 5 490.9:...5:: 0.5 3... 45145.0.65.9 0.3  5.160.5 65:.9:70. 065.33..3.556.3:...65..:.335: .6:0:.. :.....1 6.5 65 . 1 1  (*   5:4 .5.... 9 .: 5 9 :70 0..3.65 3:./3:193:.9617960::.6.70.9 6    &     3  1   (*   : ....70.663: 13671 69 . 45145. 65 .1 .1&.9.7731. .0945.: .9.1. .:651.6 16 :90 5 . 5.:.02 .:.335: 5 .66  ::::77 .654.3.::   ' 0.3  .7.33 79610 . :.:09. / ..95....65.4.33 %%    .9  &33.: %. .  9:..25.6/.1 (653*.03.659: 03.0 0.6 .9 : 56 /.1.:: 69  .5..5:  &     1  1    (*   ' 7..90. 9651.66.:: 69 7..7730.4.5 69/9. .31.:.. .3 33 56.  .65:5 0..6 64 .31.:...: : ..3 5.47313.9..7730.979:65:696. 32 69/9.0: :.91 .4 .: /5 765. 0.9365.65:  .515.0.. . 9651 . 69.925.9964. / .1 . 5 659   .. 765.:9:3.93 .: .:.:5:..  .65..0. .69.51 794..:.: 10:65: .9 5.  &  &794 69. . 6: .. 6 ...6 7. 06509.5: ..9 065:.656:.:.65  9...65..6.31.41655.9.51    647. 65:.    &0.1 1 .   30.969: 98945.9.0..17.:.: 65:.9..51065.9.7731 ..0.99:   &    .:5..3 :.65 .1 6.05..51 5 :. /69 . 4.   511 65../:.6 / 4.. #3519.: .0.065:.9.3 644::65   #3     (*   .: 56. 065.33796. :7..51.5. 9891 35.93.3:3.51 989506990.47. :703.:1050:/69.5. 79676:1 :./6.065.1   ('* .93:6.33 ..5::6 .:269.46976: 5.: 1050:  . .10.779679..65.  75765.51 ..35 ...9 '064/5.. ...946.7960::6.9: 64.5.5 .9 6 ..0.93 9..:2 6 .60. :9.     1  1    (* 6..656.....656.:.969: . /. : 6915.: 796:65: 0 ./1011   69...51 !..    &  . /5 63..38:. 151.. 3.451 5 ..:5.31 :. 0.9#961 697  &  . / .:. : 0..939:3.5.. 6 ..65.. 6 .1 5 .3:..9. "7565*   5 1.65. 06510.:   53    &     (*  ...     1 1 . :...:4. .  96.3..65.65: 09.:   !..969  . 9.:65: 65. 0 .965 4105  .5. 3.9.656:.:6335:...953 .331 ..3. .51 : 59..9455 .5:.1 &..33 1:.:9.3:: 6 1.6 / 47361 :7.9 6 596545.33.: .:/51:09/1.31.4..102   "23.:..:  &    1 1  (* 9   &09..3 %:690:   %  !6     04/9  ( 516.    &7..  .:. 6 .691 #&  50    .3 51645. 065:.91 5.. 69 . 9:69.69691015 065:.51 653 .:.05.:.64..25160.9 .6 .: 4:.35 .65.  !.0..   1 1 .  6 .97.. .35. 6 :05.57.06:303.4/. 79769.55.1/1::0.33..51..  9565:.65  .   9 796:65 6 ..51.:. '6/:9 . #3519 ...6 69.95 .5.3 4/.659 :4694..91593..7025 . :6.41.0: 6 0./7..696.. 6 . .9950. 6965 1:8:.3 .6 09.0 790:65 5 .91.. 33. .:. . 3.556.51./09.65. 3.1 .695::7769..: 0.9...2469. : 15.(*    5 3..9.59.5.51691:5. :631 / 065:. 50(336:336*     6: .95 195 ...3.1..5069. .9501 79:47.  &.5.3.. : 5/36: .6590.. 9:76515.51:651065:. ..9.:/9. 19 6   .65 :.6 56.: .65 :.3. 33256316.5/. .5('91:65*  % !6  !6   5. /5 03..1.9  .779679.9.34730.65 796015 : 653 ..65:.94.5 .69 7.9.556.6. 7.:..  6:790.51.    69. 9. .9...51..#3519. 9.:.1.::.51 .:7. &5.. 5 .1765/./. #3519 ..:3.6:.6594:../6..33.65796015  31  65:195 .: .. . 6 .31.69: 66.169...51 .  .3.65.41 / .:656059:.9 6 .3.:.5569.0 .. .9. /...65 6 065:.91:.65..506. 6 . .. 5. 0.0644.5 .&5.....1.5:313/9.51 .... 79.. 5656.. .1..7910  33 .1...9. 5.: :76:1 / ..0.:569.9 1  0 5 ...96:.0.935 . .7960:::1 5.6 .73..1 .6.505.60.:....65796015: #96019.966. 19 6 .9:.:569.1.%#.51.2   . 7969 1.:6.51/9.69:.95195.5..1 #  !6 047345.1. .659&09.945.656.6:996:36:: .3..95 1751: 65 ..:56..9:.: .. : 1   &..65:.333/0651451.....9.9 ..35..696.5511.33.3.3313 :9 : .51.617960::.. 69545. ..:..51 5 .6 .33 1 ..5.&. .9:  .5.51 .657960:: : ..0   5.79.1 6.7.9..1. .00691 9.9. : 1 .3 796. ..6.9 .9:::.3.9:76515./.9 796019.65: .69.65 :631 / 4.5 .656..945.317960::989:... .9.65 !63::0647335.69:..::6.:05...5..6 4.6 256 : 4645.. 694..7676..65   ' ..1.6 / 469 195. 06.65796015::.3.:: 6.9.657960::.:0.1/794. 7..65:.9.65:493461...3 7. 145. ..65 69 .3.1.:00.6.5.6 .1. . .31 195 .98:.1.:.9.93 .4 69 .1. : 9..5/7907.1.65 : 31 5 069.9.0064461.6 795.. 4 650 .9.691:.&.98:..9:.9.5:...1..1.6. 51 ./.5.   ' ./.6256.....9.. .6.6. 150 .3 .&.656.1. 6 . 67769.46906473355. .9.6.9..6..1.95694.69:.355069..65'9. 65  : 1.1.. ..50 6 .::50617960:::.9 ... .91:56 5.5.43.37   .: 0..656:9.9/.. .5..6598:.0 /69 .65.10.617960::  31  65..  650  : 519 .1. 69 : .6.3.330645.5 16045..01.: '9:5615..465..9160.0   65  %. 160.65.9.45.56.. .63.:.0.::91..507....:0:. 179..51..9...9  5 ..9. 796967769.51:6 :.617960::0 3356.9.3 /9.1&..6 ..9.  0 .. .:365.. 0904:. .65   %  !6    "0.: .5.65 069.1..656:3/9. . 9.91 :/:85.5..656:9.673..65   50  .3..:9.3:.   ..3617960::.1. .99.50 6 .: :7769.657969.. ::. 1.65:9.6..7969.....1.9: .3369.9.  ..1.91   9 .:6.65 6 45   51 56 .5.9 .: .9245:1.3.6:79:5. 065. 79:5.511. #.95:::.67769.6.5.6 1 7960:: ..5065.. 06503:65 6 .8.4769. :793656.4 765..3 5:91 /   ./.9..9. 0.9 769 69 4. 069...69:.3.0/. ": 35 5 069. .:/:85.69:.9.5..9:76515.9.1.99:..51.61 ..5.:.5.9.9..95 16: 56..69::.9631 6579.6.6.6 / .9..51.9: :05.50 (#56*     33 ..3 5 .1:.. 5:4 93.5.6 / .51 51.5..95:::631101.6/.6/.99:.63..77369/.:.  &09..5..:.6:.33 69 ..33/:..45.945.65:..9/..:51.45.9: 5635 69 695 93. 0.9. #.617960:: .98945.65: !13::.91/.945...5.9:.1..50:  :0 .65796015:56.145:.6.650...::56.65 796015:16:56. 0.1751565.:.1794651.653356.:.9:.3.4..5. .1.5/ .7.. 33 / 5 33 67769. .6:/5....:.65 69 ...65 7969 . .51   : 67769..95::  .9.9 : 56 63..9 .0.65..1. 441.91 '.: /5 5161 / 69 65:.65.      5 .465/. ...:  9:76515.9 6 :. .9..95:. 69545.6.5.:.98:.:/..5.93.9.6 .9.4.5.657960::46:964.651:517515.6 : 065.6.:0.794.65 6 : 9.65 .99:.3..465.95 .06503:656.3.67769.3..6 .5000.95::    65.:6.65 . .5.3. 5 69 ..006911.98:.:3.065:850:6..6569:76515. 79:65::6.5 .. 065... 6931 06445..: :560.1 . 17960:::79:09/1519./ 0.96 69796. 5:...:1656.9.1 /5 06510..6563/9. /0...:.5.. .1...45:79:65.9.  796015: . :779::65 6 094   511  (0*65:.511 50 ./..9   ..0. 6/3.33/9.65.:5.6/.. 631.5:..501.. 9.05.#.65: 066:.9:2663.9.1. .3:4:.933/.33/9.:.51 .9.37.9.91: .235: 6  5:..9 .51 5 06679.67769.:  3.55.65: .69545.5.25. 5 ..6..65 6 .9.3.9:./3.. 9:2 6 63./661 7630 ...6511..6 / .69:0..:3..9.65617960::56.1.56.3... .    '66   0.65 69 0::3 3/9..4.1 7960::.695.5:.99:.16.:.0.:.6569..6/3.61    ./69.53335.5.00691%:76515.2:.1.51:.6:796015:0631/06473./79679.1.6..1.517..&69545.39.69..7964690:.5.6509.5.56.9:..: .:  06.:.98:.1 .: .1796:3:5517.179.336 69 065.65::776:1441.5.1.3:669..65:   .:7.:/0646336  .5.1.5:..39.    5.5.6.65:56919.9.556./69.5:.5.69:63.65 '9....69545.1.45.3:. . 6/3.6 509.393    ..17960::9.. . 9:10. 98:.:5191964065.....1 5 .63156./369545.5 69 ..16.04..1.9. 5&.8.2: 6 .656..: .0 631 5069.91:1.65 03.69 .6196469.03..3.6.3#96.03   &0    6 .51 195.32 /6.519:765:/3. 1:.:   ..03.5:..9 6 .673.9:  #3775 1: ::60.3 796.0 . 03..5.3796.995.59.%8:.. :7.65   #9..6.33:5.0.656. /.7730.65 9   2:  .5. 3/9.5.31 7.6 .99..93:.:.65 796015: 631 .65:473989:.0.1:631/.6/..0.: 9  69 .6.511:0::.9 1.3796..: 79.9.69545.0.3379:65:69.3 .9.0. .0..9: 5 ..9.697910 ..989. .765.. 03.:: 6 .:476:1 &43.6.:.:4696:. 5..0.31:0945.9.59:.5:.3.:59.9.:065991.      5 . 65:.  .69.9.:.:4/9.9:   '8.65   69545. . :44.   65  33946 #9.1.36/.0656.65   ': 4.:71.653.91:0945.9 691:  :631 56.3796. 9.6503.5.9 :/0.6.03.:76::/3.5: :43...1.9.3::/54/611 5 . ..9.13.4: 9:3.3.6 7961 69 .35.3.33 5.165:. :69.: 6 490. '9.::..5*    '8.5.69:    '15.6:99519.5.5 5.50 (#.006915.65 . 6..:.34.65:4.93 :99519 . .:   006915.4/.65 0.3. 5 58.167.65:. :64..65 . 469 :700 .0.6 0.3379:65: 69 . 6 .393  631/. 7.79.3379:65: 9.9796015:  6 . 65 .5 :. 631/5.3.:..1.9 :71 1:76:.5/..1.3. .5 . %8:..4.:6:.:: .1.9.973..3:6 ..6:3.. .36535610:65: 8..69..9 796:0..: 989: : 8.16   &%   .1... 47.:   .51:3.33 6.3.8.4.9 5.54.650.465 8.1 &.5: :43.95::559.65651:.5 7.5..9451.::0.00:1 .1&.3  :6 .: 5 9. 51 69 ..9 . 964 ..6 9.0.9.5. .3351 65 .96069:5.9.3.0 6.65 6 .. 9/..  :50 .:..6.60. .9 .6 635.1.469.:: 6 ..: 1.06507.03 .015.5: .6 . ..17.5796:655.::.9 1.9.: :.9.9675679:65:69.#3775:.9/662:.51.:.5.796/.9. /.9.51 .617960:: .5.5.9 5 09.65 7515 .:9 5. 6 .6  51 ..:99519..50. 1 7960:: 03..5.065:5.03.913::6.79::3..6 73.65  .5..51 5:.6.6.6.9.979.9. .3.473 .3: .5.:   31 '8.:16:56.79:65:6.5.0..0..5   %  !6    &7..6503..0.::0.69. / . 98:. .79:65.:    73. .4 7..9..3103.:565..:65.:  5 6.964.:9640645.65796015: . 9:63..6 :64 .9: .0.3796.656.5694651..65 .7./ 0.9.1 195. 63./:506.98945.3796. 3:  ..3 796..3.0.::9169.569  .65 4.3 30.0..3.793:065991 .69469.694507..02  %:63     ..51 507.:16:56.33. 65:.3103.:16558.:796/.0.5.6503.6/0.03.:165 9.6.9.69  .9:56.656%:63.::0.31950 ..65:  .6:..6 .33/.5 .51..65:/6.694507./3 065:19.0.65 /.369/103. 03.1 . %0655.50..3 0659::65.:.69:/:85.51 9..19646315605..9 .::0.3.8.51 .65 03.6.  !6      5 . 79679 3:3.90.:.5.    5./:63..3....6 345.6 795.50(.:.3.1...: .65..4 ...:: .33/9..65/0....7906:56956511  4.40..   #..6..659:  6 . .65 : 8..5 .3..493989:...65.0..796:6563.33 79:65::.&0.9:5.:56901 ': .8.5069..32 519320904:.9.65:  .    1:.569  .4 03.65:/.3.3.:1 .:.9 : 56 :/:.31950::94.:. . 7976:  : 5./3 0.7976:6..::0.65: 93.9: 530.: ...5*     659::5.969 .69469.51 6.8.51:/:.. 5 :.9.65 6.9.33631605.....:16:56.50 (9*      ' 8..510651.959..644::65.516779::65/.773: .:.65609 6 .03. / 4751 .51 .7796.96 .65 /.773: .46.14.3 796.519.3 1:.6 .75.3.1%  !6 07961: 5&0.8.3796.9: ' 4.:6 .7....65"09:6.6.9 "  60..5 .50:.!630.511 .65"09: .6503..65:.9.694507.5.:0.3 1:.6503.6 0995.69.65 .. .0.9 0651.65"09 :.50.51 507.513.33 44/9: 6 .1:0945./3.9..6.:1659.:53:6.36.46545 .: 63.6 .5.3796.90.: :..7.:65."  .64.65. :.::0...330.3796.5 #673 &% "0.::51/.5.:65. ./:63.03.69 69 793   ..: 33 . 695. 51 .90. .0. : 5. 56. .1.. ..3.945.656% :..3796..65:/:..:31.6503.6.6/7.5:.: ..:56.6:...65 60.6:..659: .::0.65   '03.::0.4 964 13675 .69:. 6 30./306503:65:.43.65794.119::1/...33 69 565    ': : :6 69 519503:5:: : 56.56.:. 63.6.65..56/37976:6....9:56.::   .530. 3525..694507.5 ..:6 .6.9609:6" 9991. 7673 6 ..633650651.31:.1 .6/..6. .65 609: 5 6919 .: '8.0..::0..65:..6503. ..9891/.6 5:9 .19 .79:.. 6 30.:...0 6 . .11/7..519352.50..:0.6 .7738.945..257.65 .0.9.96 93 519&0.65:. .631/:05.654:.93:.9.:6.5.6:060..::.60.3:91//9. 0.976:1.31'69... 9.31 03.336..0.:.659:.9...65519&0..9.33....4.654:.0..65. 47.654:.. 69 5.   '03....5606997.3.54769.9..403..:   ' :535 6.5.65 5  6% 30.6 ...3:69..::0.. 16: 56..65:653.::0.::0.0...:.51   '03.43::6. ..0...::.691979:5.5 .3: / 795.3: 3.65:.7976:6./94. .5/1:0951. .9:.9.::545.064730.5.50651../.3.03.8.9 73.3." 5. 3:3.::0.65609:.6..25.5.3344/9:6.65:    '03.654:. 7630 6 .198945.65   5.3796.5//9..:.65519.9./34.3:3.3103.:6. :0.33 03.1 69 510.93.: .91646.9.516/3::  31%  !6 :700. 41. 58 . '69899..9 765.51:. .9 :/:.41.47.337961:.3.51:.10.6503.65 .33 79:65: . %  !6  63.9916.10.16:56. 69 9 :70 7976::   0.3 34.2: 56 1:.5./96.:065:195.7 : 0644.6: :5 .. .94  .: 6 .10. (5*0::.4 6 69545.9.51 79.6430'4:.: 195 . 69545.:.9:.9: 631  77.1  936: 69 565.656.:   964 ...: : .75.::: 6 79:65:  90 69 7669  10..('*1. 3.0:. 7:0.51 93.0.6 . 4.65.3:65. 69 .30. 98945.97669 510. 7/30 6 8.10.9. 4:.9164. 6430 .5.3.  6 50::..9.: 6 .1950:5.4 .0.9.95.  '# 50   "  &% 793 ( 516.3  .:.. :751: 7/30 51: 69 .6 ...  9756 4.  .51 ..5.10.65634. /96.33/476:1.795.9:.3796.65:4..:..10.50(#9:4.:65 69 . .569.51:.913/0.: 69 305::   ' 9. 9850: .6  9 .5 9 .51 763.:.3360.65:7.1.3::95:067.9... :70.564.0.3 1:.5: :64..0.51795..5.:.9 .51 /96.6 : .165.. 41.41.3..:.79..679619..094 6 9..9:. 031 /36 :5   . 0.1 .3  "   % !6  3  5.94769..50.6566..65   .. 69545. .9:0.0:6.41.. /96.656. 3.3360./96.45..6 :.3360.. 1.0..5 795.6 . .9 0.8..90.5. 6 0.65: /.4 '9:56:43..3 7. /0.641.. 6   .9.5 7961:0::   31'9..65: .65: 6 . 75..75::65376736. .16/96.9.56. .5193.: .9 195.3:65/96.50. 936: 69 .. .75.*     9 .3:6 31 . & .6 / :1 / .: 53506.51.10..3 .5..0.469.9...4 : .1    5 . 0.313/476:165.:6795..3..41.656969545.16 ..773: .6 5694 ..:.0651..511..3..  50.519.19:690: 93.   5.3360.51 7969.3:5 .:.006911..*     6:. 9:6.75...5:.9...6569.936:   !6 7.031/36:5..330644.1/.9.03..  ': .5:..1.9 79:65 69 03.3..1.9.33/03:3476:1  '3.5:.64.:::679:65:..75::..3..:.7.797.7.915.79:656:. . 9.65 '66316.58.51&9069769. :7669 510.5 51:91793  #673 44 .75.3 03..:   503333%. 476:.00:1964.4*      ... /.345.7669 : .02:0./:636947.56'.1..656. 6 9 4.0.45 ..50(#99.91.  % !6  3  5 .5 479. 653 5 .79646.:0.3 5:.. .../.0.3.3./..469.5 :60.96319316:56.3..9:3356.5..0.&03.65  ..1 6/3:: .513.90...::6569. : 56.0647.. 9075. 651 .3 &0663 33.65.  ' 5.95.69: &  8:.. 765.50 610.. 6. 65 47361/5.6:.95..3&0663 50 .50.65...1:.3.:/.99.9 03.5695.::0.655:. 51 360.3.9:. .: 6 695.9 9.9:     :.3. 65 03. 8.:  .65.9 9 /   .9.3 796.969  63...0.9 360.: .51  .0695..::50::..97.9: .95.3 065.69 .: 1:0945...5 .65.3 &0663 065..:   ' 5.956919.511. 345..65065640 &60.65.:930.95..63.516929  %:63   31  '..65.916403. 655..3655.65 !6    650955 :0945.9:...65  '59:.5.33.65   "9 65:. 655...365..0.5%.5.95.51 3.65.65 5 10.6565..3./69: 58.95.3%. 0:795:96459. .3795073:63.9..6564.65 : /651 065.:3:   5.. .: . :0945...5..65 5 %:70.. 7/30 7630 ..51 "007.303..65.3. 32:796:09/:1:0945.5. 6 473645.65 .51 1:0945.5:.65 .3:0945.513..: .7630.9.5:.65  .65633694:6%.65 .. 513.65.30.6/6./6969.473  796/.95.631/. 7...65.5.1101 150 . 4. 69 1:069. 69 8.9:. 59.9. 9 .3 8.:5.:: 6 .. ..65.65 7..4...: .65 .45.:796:65:.:5.1 .0   ' #3775:  .956. 1:0945.3473645.3 5 .0.6.3 9.65.94:6.: ..519:765:/3.65:  :233  69.9: ':93.69.:./6961 9../6979.91: . ..965..515   ' &0663 065.: 5 6919 .67769.33...6 .3 .510651.33 8..5 3.5.3 4736 ..5 47369 .65  .95:: .95.6.94:.33 4/61 ..347366969268.5.3&0663 50 . 7.51: . &0663 5.55.659 .51 75.: 036::.3. 6 3::9 06475:.......: 65:.3....6..65: :631/7.3 692   #9:65: 6 692 .9:4 6 8. 360.9.935. 44/9:7 5 . 5 9.5.1:43.96.&.96 .511:0945.: 794691.:7.: 56..30.3: .3 795073 .6 1:0945. 519:43..3 7..658.069.   :0945.: 795073.65  .65   ' 6965 796:65: 4795.93.9:.6 5069.5:.03103.0..:   (*. 365 65691 3.679646.773:.: 5069769.65:6473645..: :9651765/.3 ..90651.:79.03  69 ./3 5:.03.3. :/:.65.: 9:10.5:.5.515:98..36/3.. 5:7.::..6 .9...51 :...65 .  6 695..6 .3 ..9: 79694 692 8. 9:90469 '.69.:6547363::.....:0..3 692  ':79:47.3 0.9.. :.5 47369 . 47369.514.39    .115 5:3.:/915 '9:561509.1 . . ..... .. : 69 .61:0.: 1:0945..65 ...5:. 3.9:  ' 69.65:/695/360.65 : .6.4 76:.695..673.93   '473695.5 . 631 / .5.: 4736: 79694 8.57950 .:56. 4736 .9:. .673.:.51 9. 79:47.4736:.9.6 59   ' 47369 .15.4736.: . .590:3::69.473697.945.52  .00691: 4736: . 51: .31. 9:79694 46905..3.3.5.360.3690. 519:765:/3.6 5.:43.0 695.950. 51 .65: .65:   ' &0663 0.: 0.967:. 562 .9: 6.969250651.79694519:43.556. 6.99.3.6 9.5.     39065.. .3692698.:.0.65.79507368.695.37.&0663.9 16403 .655:..516..50.1:.9: .6.3 .6 3.9..63.. .9: :.6.3.55.045.3.79106360.9::63156.: ./:1. 9: '360.3. :695.9:79694.4:90:.:. 6.:.49.:.69 .1.:.51.9 69.4:..51.9:.3...3.65.0.9: .:65 .695.6/7.1:360.. 59.69.556.50..3:60.31/.0.:.5695.::69.655:.1:.59.65..0.3.1.9: 34.:  '1:360.99.1.:9.5134. .306475:.006911.5 /5.9.18.561/360.40.9 56.3.9:...:.1/09. 65: /..510.9.969  .9.9./69065./69 33 796.691 3../..7/30 5..6 .956..0.: 06330...3/..65 :775 06:.65565:... .9.: ..3.6 93.:50311 4:.0..5179646.: .: 33 56.9..336..6 :. .55.9.65:      ...:66929:.65  ' &.945. ..9:6479::1. ..9 .0. .51643.6.: 065...73.6796.:. 93.51 1.73.65:.5:769.9  .  .9: :0.53...5 :.064465661  &631 :0 065.6 7/30 7630  069. :.&.9.92165.3 ':93.93.493065.0..9 065.9....65: ...::.:6:5 .0.9:.7./69. 9.50:   '65:.31..3. 5.:0.765.: 51. 6. 903.:6695.. 1:.99.3..::0.655.:.50.6:..&0663.6547361/9:76515. 3.51360.9:. 6/.55.::0..:90:95191/695.65/.50.5.3103.9:.:65./31:.65 '9:56 9. 51360.3.9:.  &0663 6 .3.06.006915 9 :.9: '79.9: .0.6 695. 9..:47.53  16: 56.  5.:69.69: &  65  65.50610.9: 065. (.51  09.6..75.5*     00:1.5: 7/30 7630 .3&066333. 1:9.916 $:4/5  % !6  5  :..95..65. 3361. 1.51 0644.:25.50 .5  50315 ....15.%646 . 3:3.5.5:./.51.565.36:6:31.5/50650.9 . 659::4.6 33 1:0.4.46. :::65: ...: 6 .5:1:7.65/69.69.506..7733....:..9:. .:/:./365:  6: /5 .659::4. 5 .9.3195.3...50.5.336: 195...3 9:3.5.650946:.. 60..31:.5 30....45.65 . :/:.5.00:1.. : /5./. . 50 6 3.:.9 43.:...4. 8:.00:1. 3..5. 7673 .:..65 6 065:. :: /69 : /63: 165 .3379:65:.6 . 6 .: 476:1 / ..51 5 ::5.96: ' .65.65      ' 79694..313065515193.4.0.: 59 /5 .51..5.7733.  31  5 .6 9 . 79:65 .5 0: .79:659 964.403.3 1.::.3796..: / 7/30 609: .. 4. .3::  .313 5 79:65   ' 1. .3 8. .::9. 9. '.: .63:3..906 69545.1. .7733. 5 ...52: :.00:1.. 9. .5. 90.33 736: :.9..3.0.3.65::::5.6965.03..5:56.: .65 33 5 .6.1.1:658..0.9:6.0.5.119::1  .3 .: 56.9 1.65  659:: 065.::0.47.6..: 63:3./:50 6 65 69 .7:6511......7976::6.6.4.3: .91 .:.911    .::     .:.. '50. ...  6 . .:..969 51. :09.5:.9.3:6...3:6 / 141 .6:.5:.69.. :.9/09. 76...53696.:6.511..:/:.195.:476:1/7/303.6 / .5/:5964.403.9 9164 .95..15.6 50.9 54.6:.6:/3655 ..6.9: 5:16:1:0945.5...:65365644/9:6.9.659:95.65 50945.:/3..   :..5. &5.33:.31:. 03.50..03. 797.3065:..5.69669.1 5 ..1.516919 !9.51 6.6:6.::9.5169.5 . 5 ..3.79:6595. 4 964 .5967:4.65: .65:0 3.3.7.9:.7.46./.51/3.5..03.506.6..: 33 .: 6 ...9.31. 1.: 76306094:..57..  . .3:   '69.967:69.65.:65. '50::.6:6%79:5.5 .3::..773.99:.65 530.5 .   #9:65 60.90.. .9.:5. '4769.5..693.:46:.9 160.65 /.  0.956.: 44/9:   7515 65 .91:9.30. 7:0.65964..5  . . 103.30.33./303. #9:15.90:669545.7733.997...50.76:.65.0..994.:: 6 79:659: 5.94...6.909..:.03.5511....51065545.91.: 5 :60.1.5603.51.656659::4.60..556.65:.523:: 6/ 6 79:95 .9./3..9 0.94.:90: ' 1.51 9:.5.  44/9:6 . 50 6 69545.65 .651751:65..7.69.9.6 45. 6 4645.1 5 3/9.0647335519. :.73.:09.3.5: ..65 ..50.69. &794 69.569045. 0.33..6:9 79650..: 69 9..9..5.. . 69 .58 :233:.::0..3139:.336.69569.6 7961 9.::0.  09..51: 989 .00:1..5..3 4. .9 5.6 93 .51 345.9 065:.9.9. . :60.345.:   #94:: 065:191   ..51 .4   !0::.5:.59.6 .5:.93  /6.: 14. .65609. . .4 . 9.5..: 03..7733. 6 ..50(5.  #673 .  50315 . &. 69 5. . 30.65 . 7630 769 6 .6 7/30 60 : 7969.9 9.5 6.9:..9:.36:6: &% /   5. .4/6.: .:453.91.515901691: 61:.:0.5196  65 .6.51  94.45... 6.%'6.5196.6*     7733.3%09. 8.13:6: .7733. .15079769.:5915: &765.5..5.7730..5116.&.3796.....33 0.6:7.45.3. 653 65 0945.65694:.  ..33.9 .91   3. 562:.5.51 :06.0.6503.:6.5. 6..5. /..0 5.5.:..33...5.7733..331094.1.5:.651:0945..   31'.4/6.55.796:0.66273. ..945.     ..5.9659651:6965.5196:.3 695: 7733..065031:.: .51.:5649.4/6.9  964.5. 45 ...5 99656: 69 4:.656796/.5./:. 6.06.31:.:0.6.:3 .506796/.9.3569045.:     .99:...31 .691:569.506.659. 1:. 6 ..3697976:31:0945.15./30.368. '9416958.5.656.5...396796:0..0.:3 .506796/.5 345.094 .6.6/79:5.506796/..98945. 1.0699:76515796:65:669796: 065:.:7769.33.0.506796/. / .656...1 69 :56.: 79:65. 3. 569045.9.9: .9.656.796:0.1:9.91969:765:/3..5./3 0.65.697976: :56./30.60.33 .:65.9./49:703. 49191.15.99.99.  .1751:65.3796.: 0644.516045..796:0.9769.75:6:60.1./.33 79:65: .51989.91.  4:.32 :56. 765.36.796:0.3796.65::651. #96. 1:0945.6.7733.9.5.9::1.77.6:.03..469. 1  5 ::5 . .1:0945.9.65:. :..36007..65.5..9:0.69  5. /.65: /.: '53...6.965..5 65:   ' 79:47... :.:79:65.50 6 .95.15.1 .650.6 .3 ..1 /.9 56. :5:05./3  69 5:.: 4.5.3: 9769..::.0.51 79:65.: 56.695 69 565.2.7733..::/4.3.03....5. 79:65 .796:0.631/.9.33  79:65. /69 .945.3796.::50:.: / .9 .8.00:1 6 094 .65...6 / . /9.3..79:41 .6 69064 ..5 :: 5 ..:.3...25. 631 / 5065:065.:.4:.7730.:65603. .65: ':47.:.69 03.65:.6.656..3:6064465:5:79.6.6 .:0.  ...:./..:.3.65.5069.:: 6 8.5.95 .0 9:3.7. .0.2165 &..5.51.51 796019 :.145..91 . 50.65 .:..06503:65.65 6.945 796/.5. :56. (.15 .3 98945.:6.3 6 4.33. ..65 53::.65   ' 49 .5 79:65: 0.649  # 1(*   2:   (*..31:0945.515.094 #673  65. /.956.9 .65 7733.5.0.65 .:..5.5609:93.9.6 / 796./30.9.39.. 796:0.65.: .560.3./30.94:.36.8..61..: ...673.9 5.31.5.956.0945..65690.31:0945..3796.50  ..65 519:069: : ...1 65 ..796:0..99.56..511.5*    '%.6:6.6. 631 :9 .6/5991964.:56.:. 6 3.1:56..5.99:.. . .69:.5.9::65.1..33 .96 ::.91.1. 3.8.3 9:765:/3.6796:0.9 03...9.995. 06473.9:8.65/609:6.:.3.0.:   '9:. 0644::65 6 094  ..79:5.65609:151...6 .5:. 6 ..95.33   :..6.5679:65.3.: 5:..:69.:58. 9. .:4:36.....0.065.7733.15.3.: 0.91.6/5065694. 65:.:.51&9:    :0:: .5: 1.3.....6566..5   :..3..51 : .0.9.9.5:::....945.333.33.6.7733.5. 0.6../.0.6579769. .93.::.614.6 79:65.475. . 653 / :65 / .965.50:16: 56.3.515...#19.5.:54:36.0644::656./3 0.6/...55.51796.69. . .:79:47.75.:: .6:6479:65::631/ 0659../3:1 160.36.6:6..9 .65 511 . '796:0..0.476:1519796:65:. .. &% . .656.:/..51.33 /..5 150 .094 3.99:.51 65.5:05:. 06473..: 6.9.:65.::::45. 06503:65. .65...65. 065:.9...90. 7.915.6:7769.3:   '1:09.5.6.   02: # 1(*  #673 3. 150 /69 .145:..5.5.9.::..: 79:47.7733..:.656653.:.... 9:70.6.90:.5:::   63365 :..1 ..65989:.:./3 /3 . 56: 094: . 151.511.79:.91.6565.:69. . 03: . :.945796/./3 0.0./30.:: 69 79:65  69 .:/4::656:7769. 6 4919 /0... 63.9891..5.5 :. .77...::0.3.016:56. .4/6. 7.36.6 0: .35.93796941.69 1..0. 4.1   31./3&.25 79694...3.3.9.15:696.: 9.55.50 631 /064 .335.3379:65:8.55.0.5.5/ 69064653/7966.79656501  .9. . 6336 .65.:796:0.5::...51 .9:3.  . ::51./30. 16: 56.49:.:.65 6 796/.9651:56796/..95:0 150 :65 .7733.65 ':4.65:   5&635  ..656.:.59..65   3 .0644::656.5. .00:1:3.06473.05 4 519 441.33  .653.6 ..65:0.:3155. .95.. 6 #673 /1 :44. .6 513/915. 0. 796:0.4.33..:   &0651 :50.0 ': 5/6.. 0 : .9.5.:/6/35../3 . 1    16 56.5. .479601569150 .9. 46:.5:.3.: .695515796/.65515796/. : 6569 ..6 4..1 / ..   653 : . /. .6 .51.91:.657960::/69.99./3 0..14:.6. . 10..5.:7769.9195.. 79345.915.33 ::.:631/.9 :.91.656796/.5 .. 9891 .556.615691956. 6 .69:69.99:63.51. 5.:.9.5.6 . 101 517515. :.:::765 '1 65.. :7769.5 150  6.00:1 .51 :7../30..6::.3 .0..697.51 .796:0.9 ..5 .9.065.6 47..33:7769.3   50   4:.:.:65.69: /. ./3 9651 .. 6 1.9455 79:65.51 ..6.6.451 / .4.: :0. 69 5.:65.:.506796/.1 4:.60.5 ..99.9:9. 6919   ': 9:765:/3.6 / 4.9:631/56065:65.96/0.51517515...10..9. "/6:3 .51 1.99:../3 .3.9.: 195 ..50 69 565:.65.. . /9156...00:169.6 /3.51   #...5 : 65 515: 65 .6/0..9 9769.: 065.6. .:. 3.99:.06473. 06473.9769.50  6 796/.945:.:9891 9.: .9515:6565.506. .: .6.3 4.6..:0 3..516045.   ..1..3  .6 3.51:/.565:.6 :: .90691:690.9 .5. 69 73.796:0. 796:0...9: 650 469 9:./30.  765 0 .51 .:/.956. 150 65 .:6336:  :.4:47369.90691:.00:1     .65 / :/4.51:6:.9.9 90691: 6 .5 ..99:. : 56.69 0631 .6:.9 : . 964 .:.51631469.:05./30. :631 / ::1 .5069565:.: .7976:669195..3:.59:7915065.9::.51519:.51:631/3169.: /.3 51565.9 .5..51:717.:..3069.45. .99:.51 ..2 : 79:65...1 .3.1.99:. 5694.. 50::.9..33. 1   .:.:6506503:65.9: 9.:.1.0:.3 69 .69:9769.6 5.99.9.6.9.::361154/ 56 3:: . 796:0.7976: 195.796:0.: .3:6:6406.93:63365.::.:6..69:.6..9 5:.  / .945.9.66 97.50../30.9 . 679:65.9.3..14::65 .5 .6 :.6:.95 : : 9769.65.09.:.5:::69.3.915...51::1. 0644. .0.: .9.: '765.69 .99.6:0.93.9769.5.6 . 09.45.506796/.6..651.:. 11 5 .9 5:.796/..:   3.   ': 69..51 06473.99:. .   93: 49365. 931 :633 .5:.6956:.69: 9064451..::1.::.365.: 6 .:.::.167.. 1 .91 5 . ...506.14.5.65 .51:....1   5.99.65   97./3 0.6569.2:517515.1.65 31   ' :./3 0. :::.519064451.:0.: 0.93:633.6.699.379:47. .:6.3: 5 : /6515 1..1 / .:63/.8:.331..6 16/.5.0.0. 79345.99.: .65:64..796:0.3 31 ..145..0.:5.515:6.:56 9. 6 . .: 651 . 9:76515. 79:47.: 065::...51 . : . 796:0..945796/.0.515. 7650.: :695:....5 &% /   91 (65.::6.009.:.556.31.9.506796/. 796:0. 65:.7056. ..6: 0.:.69.6..69..5609  .: %:76515.1/.:.5./50 9:76515.45. .. 796:0.:...:65 69 4  .../30.5.:.:0..: .:.::65  /13...796:0.65 6 ./30.64. . .5..915 .569.51 50.99:...31.:./30..1 . 5694.65..65 4.506:60.69 ..1. 69.3351.69:515: .654.0  .65: .5:..: 06510..:.93 65 .65693. 796/.5:097.5 7650 .5694. #96:0.09.5:.145.79694.65  . .6.65:  0 5 ..1. ./636 /.56:13.00647.5.7730. 65. .%:*     5 ...5 .5.99. :2.&..5.65 69 :.90 .7730.&.0 6 ..65 .  . 0647651.: .51 / ..  . 510. :  .. . 1.::1  69 493510../636  / .9 .  &.398945. .: 31 .0.03. .:/3  5 ..519&0.0 6 .51 / . 73. . / 56.::. ..901 .. :2.6/:.  .9.93 1:09/ ..00647. 0647651 .5 6: 1 3.  . .:.5. :631 7.119::6.65 9.5: .&. 4:. 79:65 69 .96: ':..40647651.: .9:76515.9 9:150:66.97673 692:67: 60: .:8. ...9014:.03.. : . 65... :..7730.. 73.3 6:679. 65.9.5.5.99.1.6 / :1  99 .1 .931:09/1  31  ': 69.5..99.0 510..065:.65 69 ...0 .51 .. 0.95036:15.69:.56:13.   ' :2.03.51../7.1 .06476510:&.7730.:.699:15../636 /.5.0. &.65.5. 79:5.:.90.90 .6 / :.51 59.3. 9:76515.9   . .9:15.61:09/.    . :8.. 9:76515. 3..3 6: 6 79..0631../59.99:150:66.5 .0 .69 9:15.0..:.97673 692:67: 60: .96: . :.:..:.1 . .9  5036:1 5 .4 0647651.36:679.::2.51.69:. 03.51 .0.95.0.15.6..90.656. 49 :793.6.. ....7730.3.9.9.0:.556.945.31.51 79:5. .794. 9:76515. .:. :.70...4731694611/.655:.2.03.1. :.9:76515.: 964 ...: .64. :.:4.5..6619:.7. /.99.3256316..9.945.656 .77.5.00:1 ...79:65:69...95:56.99.945.51.:.:.99.0.479::65..0.6 797..:7006:679. .: . ::.516045. 7630 609: 06510.:/4.1 5 ..6/:.699.90.39 6 .773169.5:.1 9.796:65 15.935.0.5..3. 1:09.6.00:1   ': 8.931:09/..:70...95 .9964.4. .5.656.6 .0.93:.0515:05.65 6 .:5 .5:9 69 765 .6.99.99.1.4 . .964.   ..3.5.9.7969  69::5.3069..99.3.5..64.0. 0631 56. ./3.91:..65 0 . 63106501.99:.3::51556.03.:.5*     ..:69. 6 .:.:3 .9014.. :5:6950..6 / :.1493..0631.15.73.31.79. ..069.656. .114.:56.90..3: &% 5 (!.9. 691 ..9.1.:  50 ...9576546.95.6 ... . 6 .  #673 69.9.0.3367630609:. .5::5.: 0.65.7966:...0.9.73..6.99..51.#.9.65.   .398945..901:. :70: .     "5.9.65 ..65 .5. 7.5.65..56: 13.99:.511.  #673 :.969.. .03.99.351.0:.5 150 ..9: 5:1 .90 .31.:.95 .0.3 :. &0.56.: .99:.9: ..65 0 989: 5..51 653 5 .0...: 99:.:0.6.:0.5563659/065:191441.4.5.... 69 .5.9.. .:7015.65 631 / 95191 5.15.&0..0..::6..51:./:1:1:09.9.950.73..3.51 5 ..6.5.9065:..69 / :.115.9.677.964.0.901 ..1. ': .:4769.4  ..  &%  ( .506...39.6659..99:...506 .90   .6/:.:.65. 5..0.6.65..9 .: 765. .90 .99:..656..95 765 907.9..6.06476510:&.5.5.5 69 .  65 .99. :.5.6:.6.9 6 441..51 9.3..1.:  905 . :6.9. 0:63..6/:. .51 16:56. 964 .25631..6/:..:.511 ...9: 47.:5...73. .::7769..901 5.3 .9     .5..5.65 796015: .06.9.:... .. .99.:3.: . 631 675 1 .59.155.7696066:5.99:.945...99.5. . 691 441...794..91 .5.6776:5 7.*     :. 4.5. ./:5.:.2.67..4.90 .794:: 69...67630609:. ..51 ./636 /..65. .691.5. 1  . /..95.9.00:1       :.9.95/69. 1:09.9451.0:56. 3.506..356.5 . : :50. ::5..5.5.99.65 964.5. 069.119::6.901.65 441..5.:.73.::.5. 0. 0.656#5:7769.7.9::./30./: 6 .99:.99.656.3..3..5:9 .6.50.6.6 :9..1:097.5.609: 6579:65.1./3. 65:. :0 ...18.1979:5.065:..5. . 6.1.65 69.9.:715.99.65645    6969  .3.. / 3.6 8.11.9. :6 .73..069.9:7769.5.:.. 65:.1:097..75765.3:6/..9 .955.1.&..5: .964.5:6 47.5.51..51.7.9..:3:679611 . :.. . 8:.6 609: 0.30.65  6 #   69 .901.::. 63156. .15..: 796:65 :. 6.65 .: .90 7960::  .15. 135.945..4 .6.5.945.65..9.4 0647651 .5.::.. 6 441..99. .6.: :71...../55.150.9.051511:.493510.:/:85.9:76515.99.51.1.3765.6/:1 .3.99.6/:.511 .5.1.. 4. '3. :4465:   5 06550.796793/165653/ .  31  &0 .5.9.:    6: .:33.9..656.73.5.:631/::1    :5.9 905 .1105:7769.9.90:.6 .901 .5 : 796:09/1 / .7730.:05./.9455. 9:76515.: 69.4 '7.656.:.5.90.3 ..0:.: 790:3 9461 964 .65#960155..  :: .. '.6 56.  .....  316.65 %' 1 #9.33:..6.39.1..73.99.506796/.:9.99...569.9 :44. 1669 .3:.9 & &0    #   5 5.:3  56..9 /315: 69 :. 9:150 6 79.. ':1:097. 63  .. .51 ..009.956930.651 :.99:.5.794.: .0195. 65. .5.. 5:5. 1 .6.5.5.03  6 69 65:..69.6..79601906310659..:69.79610  '9:5698945.5.6569..5.1...6. 0.69.. .5.:.19891.506.5.0.:05.3.95 59.56...5.6.94.516045../30.51.9593.: .     "5.1...794..0515&0.69..:   5// 65 .:.:776:1.99:.. 7650.33:.1.   56 #673.47367769...69. 989 ..::....5 .. .:  .99.506.0.:765 0.. 606510.0.93:.:65.4.:: 6 03...93 .506796/..5:::.99..::.65. 56..1.5.00:1 9 .65..4.:6.'9..33..:3989: 653.99:.945.5.91.9.9. .98:..5.67496479:5.::0..631/1:0691.: .5.655196..3:...631.656.1.51 50::.1/.99:.9..65  5 &0./51.3361 .6.9455796/..   ..::.. .0631.69.631.. ../30.:.93.75..5.606510... 796:0. 5.:..6/.95.5.6 79:5.794.99:.   '61.::6...:61:9:5:69.0..1:.7 !..:64...6797..0 69 ..69.65:.91 .9796015:       93:515.06.06.65..5.506...69.5.  5 0.794.51.9 5:050 6 150 65 90691  1: 493 9.9 .. 79:5..150195./35.95:..945796/.:0.95.631 :..:.98:.6 5.4    .:.79..14:.6.515:6.31.6 / .:7769..99:.65.51 .. 46:.99:.50 .2:517515.99...66.6.99:.45 06473.97976:6 /6.00.510.:69..95/69::5.569.::.5..656.01.690.69.65:.511.00:1964.99.'9.64.65606473.00:1.506.9.9:56 5.5:::   5 ..5.9 . .:.5 :0..51:.3 69 ..51.065:850 69.00:1/69.945..45.7976:679:65..51 5:..979.5./30.95 /69.::.65    16: 56.4699.65  6 9.5:..973.331.6750.9 ::.5. ...9.:3506.506 .679:65::6. 9.. 69...615::..515. 339891.. 6 9:76515..76.33.51 :.1.::.99..5:::.6.145.:56.: 5..99. 6/5.::    .96:.56: :.9..0.3: &% (*  #673 .19.906.677.:0::   31  3 ....90 0.6 1:069 1.79015.: 06509 .5  160./5.096::..5176::/34.5 / 4.9796015:.2.4:.:.76569150 0. / 0....5.9.76:.1 .  .50.99:.     . 63365 98:..6906510..51.6.:.31.9 / 9:..:.5*     .094 065.9 .79..2...65:  69545.35.6.5.95.90 .99:..:44.6596400.6.3:6.5.5015.6.596:.:1:0695./365.609.9.9:/0..46/:9:4.065.6.:.5 .905015.4.4.5.:98:.      5 .19.50 (#.9:6569:5. 065.556.5.335573.765:697966:6947345.510:.. 3. 7..3.961015150 ' 73.99.0.55.6./675.5.51696..36.94.93..945:.773: 5 ...:. 6.:5:..:..3:.3.576::::656.69.9:6569796793/5. 79:65 .19.5   %  !6    &7..   65  33946 #9.556.5  : 5.334.51:./99:1   5.094.6596400.1 . 69.6.4 5.: 6 490.95.7969:.56.0644::656. 5....3.90 5.1.57.3 .3.7901.3.70615050945.904.5.9.3.99:.6. 3.1&.6.96394. 3..99:..9:65 0.3.6&.5 &% 5  5.5/.. :.95 .9. 7960:: 0. .51.5:441.::15. .73.:493:1.99:.::. 609 5 :...35.56096.99:.76:.51.:0.99:.36.5. 150 5 73.:8:..6.. 0  .77. 569045.9.5160.3.9..6515../.03.90 '7960::0.99:..55.03. .1 4. .::7936: ': :05.. 1:069 6 .:.150..5..9. : 441.069:6:03...65.96. 76:.1.3..95..905015.:.96 .4:.569045.9./1506.6094:.9: :/0.50  ./99:1  .:.31.5.514.9.5.:.9.6:9 '3.1 9   *     .90.6.90 6 .00:1 '6/0.3 .50(.6:95.5.51 .4769..3. /69 .3.9. 989: .6569.::0::   31 "/0.3.. /  . 9::5.5.631.:.51.5.960.5:445. .9.02.9 79:65:  56.5.0. .69.5.:.     5..9651.../441.. ..773:6539.9.9 .9 3.. . .51 9..5 .7733. 5 73.. .0.9 :..50..51 '73.:9651 5 .. 7630 . 511.99./441. 99: .. .9. .. : 5 73.9:65 /. .:.. 3.6. / :1 .7733. 065.:1 65 .99.6.5:.65956.5.950  69.995..51 1  73. '9.6. 6/:9 4.51 0  .:.7733.1.3 .  .773 .514.:.5.5.: /  . .79:655.: 56 .: ..9 76::::65:   ' .5.096::.5.:1:.03:141573.7733.53 76:1 ..99.5795016/:99063159964..31 . 73.5.56/:99 .77. : 03.5.50.3064:.036:1 7.9.9.76306.3.:06503:65. 6/0..00:1 /. .4: .6/9.79:  .9.:.9: 631 56. 6 .9.5.19.93 .: 2.1 .73.95   5&750995014:39964.5.6911 / .0659..93519.5. :..1 .   ' 103.5. 6/0.9573.  ./3:.556.9./0.50.9.1 ./.90 6 .:0.065.33 79:5. 501./ 4.1504:.3. 7969 .: 794::   !6.51 6.: .569675.99.5:7.:4:.550945.95 .5:.6. ./3:.9. .0 '336%.56/0.:3 : 56.150. 5:. 5 0 .:65.0.59..9 .3       .5.7.0631:76.77. 094  065. 6: 6 ..65 .9:..5:.6 / 796.: 56.1.51 7966.55 790:3 .6.   50  ..0..5   5.. 796.31 5.5.6 .9. 6 65: 79:65  0 5031: : 9:150  : 7. .5.33::8.4 ...90569150.  ..9. 0 79011 .. 445.3 :09.::0.9.9.51 69 6.5  .01 . 5./:1 5 6.:6.. 796/.6.3 .6/0. / 5 : 64 69 60  .5.1 .:... 441. / 150 6 . 59.: 9 56. 6 79:65.: 0.90: .036:1065.90 ./69. 9:. 5:.9 6 : /19664   #673   / .63.03 .0:  '6 0650316..:14.5.1..6534.6:9  #673 69.149:96150.:5.  69 ./79:5.5.4:..5160.. . 562 ..95..51 :9:   4:.343 .: 1:7. &.90:/.5160.79.9169.90:.3069..63662. 0.5:. . 73..50($:4/5*    65:195.:065. : :6.5.5.:.. #" .:.99:.: 5 8:..90 69 .:  90.50(#56  *    69.99:..:.. 360.90     5 .5::.6.31:0691/.:1. 59.:65..5. 0631 56...5.   ' :9 6 150 5 73.:065.19. 7630 .7630609:56.96/6:.65 ..6 ..: #.73.9.3:63. /.3:: .4:.: 511   5 :4  . :.:5:1.02.:65.969 0..77.99:.5.9.4 .:/365:. 73.9..1 369..:5:1.5.6 / 445 .&.6..39.5. .59 9..79603.5.6 ..31   %  !6    &7.:..79:65.: 53.6/0. .31 .:.91: .5.:..3.5 6/0.77.95....5   .9 .160.9 ..5.5.4656&#"'7.51 . 7630 ..9..73.3 5 .91:0696.7733.373.:76#".5:...95.9: :/0. 511.. .773   !690.95. .02. .9:65 .33 .5.19. 9.31.33.:40.5 .9.79:. 4.1 .6.9.796/.556.573.5. .63365345.. ./3 :.3:: .9/.6.   ' 44/9: 6 .5.50.6 :.5../:73.5:..5 5501 36. 4.765.:3 : 73./:91360.6 :.5:. 9.6.5:.55. .7.3:656. .6.73.:.5..9691: .4.9:.: /69 .51:9:: . &% .  .:.3 5.. 96.3 "50 5:1 .:.6./:. 6:  .51 5.4 9 :. /.51 90691 .3:6.90.95. .1 5 9555 .5. 7630 609: 069591 &7509 ...5.350945.6:.9 4 ..99.69.  06509   ...5 69:.: 4:.3  0645 .5./:..: 32:3.9.6 / .47.7630609:956.1 :..5.9. .3#96019   &0.  : 0644653 9991 .5 .:   9 .60644. 6%36.5.3:61.99:...3:60..025..: :/:85..5.330644.5.7733.676./3    50  .79:5069.65  ./902:0953365 .: .5609 ': 5.0.5.:....7733..6 . .. 0.6 . . 796 &0.9..0. 130.691/.5.53.9.5 69.025.510..0644.6.. 93 65 5 3.094.99:. 79:65 .65 .%3:65945.::597.. 4..:165 5. .5.5.653.4.5.0.7733.99:.6 345.697.: .99:.: :.130. 465 964 4  '.  .51  :069.14:. . 6..99.47.45.5.   #673   3.6.96   &%   ./651.99:. . :.90     51   ($:4/5*     .  .:.67.. 51.. 90     .9:2:. 514.67.920.:6'99 "6 .3.:.31 5. .51. 65 1 7 ..65.76304.5.796793/5..6.66.90  ..65::1195.::.0.35 4.56:079:65:5.:65.0904:..69  15.:.61:069 .941 .51 79:5.0. 5.9036.03.. 6 : 7950 .93334.51 ..:.65.517.55150.5 . 5.1:.0.961015150.565.5..:.765 :     9.0..9: 5 ..35:.51956.3 . / ..469 . / .   : 5.0.6./3.65 6 4:3 69 6...79:65 5.. 069: 6 5:. 6 0. 79:65: ..03.5 76::/3 0945. .51 4....:65. ..9. 79:6596464./3:.47.5./:1.51.55.511.1 &. 094795. 796/.9.9.7796. 4. ..76306096/:9:.: &794 69.64./3 .90.3 1.5.906.9:2. .5 ..6 06510.3 /.. 0945..7630609..779679.:65. 69.765:14.51'99 "6 :79.:151.3:.559  69 7976:: 6 5:.336:.2. .90519.6:.9:. 31 .4 &0.9: :.596:  9 5 .51 :.31 69 .:.6..3365 :0 ..61.6...1 4:3 .3.0. &. 64  : 1. .: .50: .9 : 5:05.9:9:.306510..5065.9 1:5.6 065031 5 3.51&9.7630609.45145.969 : 65 69 6.25  999.69 5 .3. .95. 7    5 .0.67..:.996.9.. 5.99:.4 .55:.6 . :..:6.14.:65./3589:  .90:.67..1:49. 796.61:73:9.: /.765:04..656.51/6626599:.9.   5.51 4./30.5:.2 9. 51.. :56./3:0676...90: .0..765.1.51 :9: .51.469441.99..5.13.:.  (#..93.:1.:.90.5.5*    5679601.6569.7: .5....35..50:  .6.64..6..67..6..0631 570.3 ... 796019  0:1 653 / 5.0.5.76304:.&&79469.79:65. 0904:.34.:..33  &% "0...51..336./.::94:3.3 6 :.1156.76306095.9:. 69 .5/..4   .96 ..0.:..1:. 5979.25:.31 ./3 6/.. .9:2 .1.941.50 10.5.7796.5165.5:.33/:1. 51.. 76304.:.9036.03.6:7950.31 69 ...9066.9036.0.34.569. 64  : 1.90 : .:65..:6.69.:.765: .3.51 6.:65.514.35 4..3:.9 69 : 65 696. .61.6 1:069 ..:3.93334.0.906..941 .3..9:2.5.66.1:.6.6915.606503153.:4:3. 796.9: :../3 .9: 5 .6 ..45145. .3 1.:65..65 6 4:3 .1: 49.9.596:  9 5 . 069:65:. 9.67.76306096/:9:5:.5.9:9:.. .9.   : 5..55.47. :.. / :1 .56:079:65:5. / ../3 :.5...:65..1 796..765: 0 4.69.56.0.:/.11655'99 .51956. '99 ..90 519.51.1..0./3 589: ./3..0./.::..51 79:5..5065.:63:.2:9. 5..306510.0945.5.606510.4.:.:  493631.:.0.. 4   &0 .61:73:9. 79:65: .65 69. .51. :6'99.:.9/3..15.67.656.0../30.59.:067 6.69:.0169..9036.0.:..3./3765. 73. 7630 609 11 56..565:  .3796/.9:9.567.5: ':1156.659 .:34..9.67.9:5.13.51 9461..:.796..9.51. 6.945.54939.90 1   ".51 : 0647.6.::56.5165.9:2.51: 5 .956..7630609.9891.065:.765: .9 7602.606510..516.:5.: 569 519 ..9006510.:.06.37369.0 : .1.. .51. 67. 49 ::7065 69 . 59. .3:: 631: .. :.31.. .. 50 33 56.9:2  . 51.. 51 :996515 0651.  5 3.67.:65 4:..51.765:0650...79:651.65:  ..:./6.99./3.:. 6 .31.33 . 7630 609: 7950 .4 5. :.9:2    55 9.5..6 . .51. 9:2 :9: . .6.  469 79::5 5. 519.. 796/...631 5. 59..3 5.5.559  .. 6 0.51 1.779679./3 0.9:. 094 795. 6 :.515.69 5 .51 :3.65  0 5193:.0.0 .65 . .3 /.65. 4. 79:65 69 7976:: 6 5:.779679.   .: .9065.6.50:..9:.9:.0904:..76306094.51   .5 76::/3 0945..7796.. 2 :..656.65 /.6 .:931.: 16 5. 7630 609   .5..5.1/ . 79:65 ./91./151.90:. 64  1.:.69:.5 002765.677.50:67/306919.9:.0.6 .9. . .. .002765.   69.5.6. .::. ..6 9 7.646.5...1*     9:.51 .33 / :1 .79:9.941 ..  65 46.. .69:.: 9.33002765.:.56..563:653.39:    ..3361 69  .3. 0631 570.:..93 .. 1..91  .13  96. 7630609 .:.7: ..::9 4:3 .3:: 56.65 0 794.. 09.765 .:.15..9..51. .14.0.13 .6 .3: &% 0  (.13 .997.3.5 .906510...3 '6:0.5..31:0::    31':69.99.. .9.556.933.5:. 90 :..911. 0.5.: 96.:365.03:6007. &% .9:  #673 :..002765.31 :.9.: 6 .5:70. 1 . .90 .1 *     33:. 06510.6.:.51.:65.  . (. 59.9:.03 :34. 195 30.. 002765.9.1 ..556.1 5 .5:9. 03:5.:.1.:/79.5.9 5 3::5.90 .5:.6 69./61:.. .5002:0.65 :./3:./98:.  .196.:6007. 5 /.:.5 3.:..:63.::/0.6.6.3:.5.90  5 .1.901569.99891.6569.3350::.5 002:  5 06510.6 47345.0.3:9.9.1950.5511.5   :.6.656./9. 556501  3100:1.. .7751 65  793   .: #.65 8 6. 065:.3 9..:106509569.6 / .7976:690:0/.:3 797.631:.9: 1669: 69 :/0. 6755 .6. 796019: .656 .:..559 5 0 .65: 5  9 31 65  . (.556501      !6. :..9:.3 30..06002765.9 .:934.65476:5.96.9 1 360.6. "   '" 631/.9.67  .:.025. 30.:06510." 9:63.694.5/506510...7./3:.:1:09.6 .5:.026:/0.: 33 . .5:...5.90:. 653 631 . .356.559    ..90   '9 : 56..:...0..:797.663.9769.1...51.9.99.3..210.3  ..0 0 70.5 1:0945.91. #630 :631 .6 . ...5.773.::.6:65.58:.90 .964.63..: 6 .:3.: .65.4..00:1.6:.631493190.65 61 ..51 .99. 0.9/. /5 .1 *     6. 95 06510..94.9969:4.47.: 6 ..:.4. 6 .:.67750.9955: .1. / 479.:.91 4.90:06510.511:0945..3:606473.::59: . :0651 651.9.3 . 36: . 465.69: .551 / 345..   ' 5015./3 &09065.::59:.5:. 002765./:.:65.909.4/.70.:.0.30.51.5 56901 / .0.796019:  31  #9:65: 4.9:.5.1 7961 631 256 .7/305.6915.5195 .5..93.6 / .6.65679..  0 .1  #673 :.33. 141::706: :0.5/.2.65..5.647345..14.5.56.31.3:61.1 '6:6 5.1101 16 56.3.65:60.6&0.4.. :.9 79:65:6979679.0.696.559/0. 5 /.: #.3.7733. 93.657961::179:.65:.9..65./3 ... &% ..5.631.9493:.5.3 :.9769.002765..  .51:9 #"&/. 065:.6:6:516:.::.5..06.5. .65.. 653 51 .15.. /61 :..:064509.9.51 .96 90.5.1:63.: 5 79:.5 5:6 /. .65 0 593.65 .5.9769. .5.5: ."45/:30.90.506.6/6.3:.:65../..:9.1  ' 065.5 .3:669.50 6 .5. .5 .51 :9 03.6&0.559930.95.: 7....: 4./.695. 0.:5:1.6:...6 /95 . 796.306:1:5:    ' .:09..: .556501 '.:.. 631. 3     ' 5.::..9769. 9651 69 .65 6 ..973.657961       ' 002765.65:4730.0.1..90671:.::59:.5   :.59.509. 5 195 :...6514.917.79:.: / 76:9 6 ..1 .1:.:09.1 ....537. 4.33 / 5991 964 .51:     : 56 51 69 002765..:17.51:9:.9 679.91.:.:5:..5.0.69065.5.90.51 .51 360. :00213.53:/0...6.996.1.65/.:33.. 9.:65.9 651  :0 631 / :/0...945 .:65..1/79.0.0.0.9 9.3796.... .65 .1. ...6 96..65..51. &% .6.5   :../3  5 ..: 796019: ::..0.3:  31  : 31 5 #673   .0.90: .. :.7/30.3:69 :/:..3:::.3::6.556.5156.5:.511..90:.6 :9   ': .02. 9.:  7:0.65..9.1 .0 7.9:5::  .:0644./3 :.6915.6:.773..5565045.96.9:65  #673  516.51 .51 ..4:  :5: ./ . 065:.3 796.50: . ( 36*      .445. 59.1.5796/.3 :.:.5:.. 6/0.9..9 454.065:.59.*     .. .9.5: &631.65..::59: 65 56.51:9:99:.90.511.5. 6 .: 5631  .935.6.6/95.6. 65:65   %  !6   0     51   ( 516.65 .65.556.9:. 79:50 6 ::706: 6/0.33153.0 70..511.90: .4/. 9101 79.:0.0. :0 :.65: .9:/0.5:.: ..: 73..::60.796019:  #673   3. 5...935.0.5.9065:..:65.3796.1/79.3 5.6..  .665 :79:659645.:.9.99...51:9:1656.9950/69545..9..65:.3 511 ./3:.511.3 8:.90: .39:.3 5.14.9769.: 0644..0:5...5.9 06510.51:9:/..9769.6 1.119::::.96.90:.9   '9 : 3. . . .90 .13. 19: 5199651   : 03. :/0.:631/79647.65 69 :.4.. .. :.: .1.55979611/3. 965 .33 5.1 .33 5 .9 7.7730.99.  %!6  5  &703 51  (5. .33 06990. 796793 16045. 19: 69 0 : .94.659  5: !6 5:.7730.5 33.6 .915#673 1:...56919 .9:. 9 .' :.9 .90 /0.: .4 16 56.2  4:/9. : .0.91  /9.3 :..6 .1  :/0. 56. . 6 .5.511 .9.933.9.31:76:165. :1 19: 0 .  &631 .99.90 .6 5:9 . ..31 .5.951.1 6959:. /.7735 69 :.0.6.5. .51: 6 4. 3..79639.065:49:.1  ./3.3. 4.5 .31.3:2330:. 661 . &.6:9.9.3.51 064795: . 0.33 51.6 4.6 69.4.9/.9. .65. . 9.:03:9:10.2 .5..691.99..51 4./3661: '%65..9.50..:::.6565696.511..3 69.9 .9796015:06510.4 9. ..4:   "53 ..9.696:...51..92.: 065:49: : .:.1  '9 : 56 8:.64: :.55:9.6/.0.50 69 :0 4.9: 5 .:.:.51 6.::765.3.     6%65.1 '::  .5.::45 065.54105:.51 19 93.:.94. 0 :.65619:5.4. :6/3.65 (*  .: ...50 .50 .65  5 6919 ..3 661:  . .. 79679 69545.69 ::.51065. 0. 065:06:5:: ..51 .:796015: '6330.9.335.::70.6796.6.1/..3 1:76:1 6 5 .6..:935::... 6 19: /0.3'9.2 . 796793 16045.:796015:  315.51 69.:33.9:5.4 .65:.963. :.:32:33.03   &0.6.30647.6993. 69545..&. :631 / 79647.9.965.. .5119:0903.3.506.9796015:06510.50 :..0647.3.19:5199651   '76306.050.3.15%  !6 :.33 .65:0.96 7.9 790311 964 .9.59:631 59/0647964:1 '6.337673.51:64.. &.96 190...51 ...6565696.59:2 69.0. /0. /..51 1:.6 :./3: .59 9..33 8:.: ..51 5:.4 4.59.65: 6 09.65.961.: . 563:.5179646..    ./530.3: .03   &0.9.96 6.69 .3 06445.69 69 .6796. 69545.6:79:.6 5:9.9:5.3'9.65    '  : ..65 0.1 / 3. . 360..51.6 ..5169.&.0 .60565. 9..65    65:.677.:3 0 79609: 4105: .99.3. .9: 69 . 1...3. . .51./306:.5169.3.1..:9.65694.3.39.65.6..3.465 . 305:1 /:5::45   6946:.3.6. .69.6*   .9 7.&.:7906:.65 6 .3 .7673.5.0..465 ..4  9.9..50.51. . 360.67.0. 4.5.6:9.0.945.. . . :1  /6: 6 19: ..9.0 . 69545..3..51.3 4.5 0647.09.5 5.5 4105:. :.9.7796..: .9796015:.3 :90: . 7969.1/.2 ::5.1 .33 06990.5 0.64:.:..33 :.6 1367 .51.0.556.7.6 ..51 0. '065:.50 '.64:.69 .6 7..:.3 '9.65 ...4. 5 6919 .03   &0.167.0 0.6 ...4:   .6..9.41656.:.33 .6.92...9  796/.945.69..550.369. 56.9 :60.51.03 &0.5169.369. %65.691.6..556.4.:: 765. 136745.9.6 4..6:. .:69.559 79611 / 3.6993.:.9 161 6 ..31. 05.  6330. 69.5.05 56 550::.'.50.656. 77.3 .996..3 69.179.965  9.9 519..1.5 3.77. :435 ..31.6:.64:69:10.65...9.0.3:   ' 93 ..:..9.:0 . 56 9 769: 69 :0 796015: : ./3:.6 %7/30 0.:9/.6'.9:10. !6    .1: 765 :.64: .51769.: 796015:   ':  ..6..51:. 05..677.60.51 796019 69 .51. 69545.679694   5.6330.96 &% .0.65: 69 ::: 9.1. 653 .:1 0650955 .03: .51 6. 469 4769.6.796:65:6. 5.69 6 :..65:.65: 6 . .51964.06330.64:  /.3  . 7630 6 73.77.&.. 6 '.64:933.6.3 '9.9: 2565 .64:61.65 6 .5105.5 .: 19  56.4511  6..3.: .50 65 .6564769. 79679 69..:/55:.6 795.1.9.:1..7733...3 6/0.69. %65. 69545..: 5 0.644::6596:.50691 765 .6 9519 0.16:56.696:.6:...69.9 9.64: 6:10:65 5.: .95 ::/0. 9.3:..3./3.3:  :70 .64: "....9 ..:. 90  51 ( 516.*      .  .   00:1. 0215.677.. .. 6 : .7733.5.::..::.6 094: 6.65: 69 .0..:. .9 00:1.5.6617  '5.99:.:06510.:73. ....90.:565.3:651..31..53:/3 !6:.5.51.51 : :/:85.69.99:.7630:../.3: .1..7733. 0650.:9 93. .:.3069.3   315.51:/:85.:50::.9 .9.5:. 3.5.1:0695:0./5656.901 69 1./ .99:.5.9.90 ::0 ./:1.5.065:5..9 ': .7733.765:69.0644.5..3:. :056.96197. .:50::.795.7733.:./59.3.6.596:.9.5.33.79:506..1  0.19:1:0691.9.94 .33 :..94 .1.094 :0..9 15.::.335:/:.5:.65 '::..  ':/:85. 0.90: .5..6...9..31:.:.65 .:1..6.5.1195./.99..7630609: !6.:150 .:.4:1656.: 50 56.69.:651.5/:1.903...0631 56.  (5.:6.51.795.376::::6569. 065:850 6 .99:.09:..3..3.9.76:656:.. :1: .5.:.:79666.0644::656. :.33.556.533.9.99:.565: .3..46:../:.1  #673 6 &%  ..03:65.:0644.99.9:3..94.31 ..50 0 .905015.:6.90.65.1:.6.9   :.99:.7733.73.6/0.99.%3:    : .   4.51.1:..3:: .9:.0945.6.:796115%3 &0.197.65:64.5.1..9065107..:3.14::/35150  .95 :0.09649. .:/531.504.5 .: . / 3..:5 0./54.9...5305:69.65:519. 99:.99:..5.515.&.553.99.00:1.0.60650.50691.5.6../.5..99.5..:145.6565.:    315. 33:.9 .069...86..130.0.3::.5.6. ..9.9.:.6*     :0::.3::.96. 7733.51.:.7.1/.:/:85.31/0..9006510.9.: .00:1.0609:  .5.:. : 6 3.969 4:.5.    :.5.79:65.50: 0655053 510.330644..7733.7.631:.79:655 5:79:50 .119:  ':/95::.6 /..::.9..79:50.4:3: ./3 0.5. / 6511 65 796/.98945.5:.93.:65.6.510.99:.3656:5.31..7.:65.0.6065:.. ..6576::::656796/.: 69 0904:.5..796/. 79:65 .0. 65:  : /. 6 &.    5 #673   .130./:506.51. :.5..99./3 0.1    9. 796/.99:. ...33..3256316:0./30..:.65.6 79:65.::6./:5...0.47.6.:: 65 3. ': 5 #673   45515  .6.3/36.51:95.60644.365 . .:  .5 60 .99:.79.565: '.69 .979:5.:0644. .1 : 796/..99:.99:.3 :. 6 796/./35694.:     :7769.:.: 31 .6 .0609:4.99:.9065107. 130.99:. 7.:3./3 3. 6 ./39651:6::7065  '9651:6::7065...99:..6 09..6 / .1.25. 31 . 6 0644.:.3/3699./30.99:.5.3 ./35 5.6.0.. .0..0.130.0 60969.79:5.3::..90.:1 65 .:4.0.5.1 / 0904:.0.7731.905.5609:  .661.6.9.65../30.96956.5  .5609:  .5.... 0.0.: 06731....5 69:.:   : 1:0::15#673 69.99:..00:1./3 ::7065 .:65.99.79:654..99.956...99.. 69.:90.... ::7065 .19..6. 79:65 ..5..9. .130..9. .9.3 25631 6 .5 0.5.653..53.65:.99:...5.:05.5 ..965 5 .6... 69 065:.6..: .. 796/.5 69.5.6 / .99:.553.50: :05..969 4:.1 :.79:65.:.. 33 .094569.69.3.3.5 .573.:   006915 .9533.9 7.91510.165:6 .650631.51:6:/0.46.925 964 ./55991964.1 :.3 1:4/...:1:0515..5. 6.50:   .0.: 32 .6:1 (3*6315(65:*.:165..9:    '5 ... .51.979:65..::59: 55605.33/50644.99 :0 .9..:.95 .673.065:19:1...:566.3::.5136625.51 510.5 609:*79:50 &6..0.1 69.:1153/0.:.77.3: ..69.4::70.91.7733.15(..99:.1 69.50: :05. 56949 765..99..515.:.1156.3069. .999 6 . ::3   .069596#3.1:.4..616:669../3 0.677.931..99:.6 ..:.3 ..47.526.96: ::7065 .: 56.565:../0. (/*56:.65..56796/.69..1.659.5...9.9./5. 6 : .....0.5.::3.: ...: .4 5103.9...47./1645 5../30..6 4 .:65./50644.5 6 . 4.60644.331 69 : .:....99:.5.:.9. ...:..51 0904:.0.. .0.. 9.6.6915.::./6. ..36315.1    ....6 .1 .51.:.3:6 5#673 505.5. 0. 3065..51$6563. 6 796/. 0.66 5 .693.5964:1.6.0650311. 69..7795:65    2: 5#673  56.. 4645.46955 ..:.:. .: 653 5 .5..  .:.:69..   '6 .094....5. 69..: :4659.06...9.:./3 964.064 :0 5..09611:.99:. .7.0/.9.  0644. 5.9.93 ..659 510..506446.65 0 3.50 /..: .1 5 49::70656. .:.3153.5..51 : .1 063156.510.33.99:.659  ..99:./.:../50644..6/3.51  :069.47...60644.: 9 569 5.1 ./3 1:2  #.51 964 .1   ...9: .659 . 965.659: /.659.565: 9493:.95.02 6 79:65..51 ..:...69 69 06510.9 .3 /3  .39.6065:.7.905 3.33195.:.90631.5. : : 9 4659 .3153.14.698:.5 69.: : 509136: :50  .5.3.. .: 79:4.5 5 7.65: .1 . .9. .:. 9.515.765 !65.6.5 69 :.659 .5  .9 .0.9.99:.1 :.3 25631 65 .. 6 ..:1655. 1:.6569..99:..1  7 4  ../3 30. 0 0631 .569.33569651 796/..0.0.96/3   '91 ..7...6.0..5 609  69 .::/3..169. 3..5 ..094..6   3.::.:65.06509  .5.5 .9.:.:/5 0644.5.79:5069..51 : 0647...1:.09.:65..35 6 7.659. .99:.  &0651  ./5:/3.. 7..130.1.941..65.094.. ..659  .: .6/.06959...13.0..0.9 ..44.6..5.../3696...33 510.5609   5...9 .: 1:0691 5:1 ...130..14:..79015.6/0644. 7..00:1.79:65 .. 65.6 ..5 6/:9..6:.5.67.51 7. 7:0.9.0644.9 7..: 511 15.1 ...6 69.: .330644.: 56./556.::45 .51956. 6   .:4:. 9 .65..7733.1  .   5 6315 .91510.5:.00:1. /. .99:..5...1566.631:..91 ..:4. 65 /6.9:2. .. 094   . . 0644.5  . 4..00:1.5.: 0631 56.5 . 69 ..1 . . / :..6 / 0644.7733...9: 56.47...6 0644.1 . 98:.6.7733.::70656......3313956901. 9:765116:: 76::/333:.6675.5./. 635.99:.:..5609:. &0 9:765:0..47365.3.6&#"#.00:1. .515.:9 0644. .094 :.956 #..5 5 3.7733.5.9.0:.16765 6 11 56.3653356.5. .5.....1 ..07. 56.99:.6 .9.065:. 7.6 0. 130.   !6..00:1.3:. . 493 765.. 9 . /.5860.99:./30.796/.45..:. 5 . 69 &#" .  .5.00:1.7733..99:.6 .: .5 609:  . .1.00:1.:933.5::7065 9./:/0.51:5694906510.506..41.6.:65.7733.9:   3&#"#./3696.:063156.1676503..5. .  .99:..951 3.119:: . : 469 ..  653 3.1 .51 .14. .: 5.16765  69  .  &#" #.16765 511 90651 . 3..7733.4 .00:1.  ..3 &#" #.: 16/..5.9 . 64/:569493.5.   . .1.6 : 3..00:1.965.7733.6911&#"#...50:0631 56...  /69 .6 56. 5 79:65 653 650  75765.1 ':0904:.96.: .036:93662./3 . . 3.16765.. : 69.. ..:16.   .99:. .3.: ..7733. .69003 5 &#" #. 3.. 065:195.00:1. 9:70. 46..6 .5 195 .. 347: 6 4   .9 .5. ..16765 0.. 1...00:1..159:54/69.99:. 03. &#"#.   ': /3: .14. 635.  25 ..16765.7733.4 6 . 5....5.47365. .4 6 &#" #.. 7733.:3::   &#" #. 3..951 .969  : 5. .  .: 5 /69.47365.1 . 5. 0631 56.69:.5.:.99:.465 6 &#" #.93 /.47365..00:1.4 6 .   ' . :56.16765 /0.00:1.96.5.1..5..7733.50 .331306510.:933.: 964 &#" #.: #.16765 4:3 6. 51 ./509...1 .16765  6 .119::7969.:5.56.7733.5609:. .99:.. .4:3: 063156.99:.99:.0.5.3  &#" #..   15.5 609:  469 :6 .00:1.6.4.: 56949 6 .  .90904:.69.6#673 505.* 796:0.99..915 :. .0.46.7.90 9 33.46.1:4/.:.9: 33.56095.979:0631/.505.3:: .0. (. !6.: 6 .. .519:0/.31.695.99:.3656:5.:25. 5 /69 . :.3:: ..0.1..519 964 .69..99:.   647. .5::706:/.. .1.5.: .65: 493:569150.33510.91.:  3515 ...99:.1 0.65.5..3 . :7693961..   6969 .921964.:.. 3. :4/3.6..1/.0609: !9..5.5.1465:. 0631 56.103.7733.693.00:1.50:  #673 505..65:1501156.. .0.1 .5..9..99:.10. / :.4/35 . 065:85. 3.7733..7733...5. .506. :79.51 .:15..:09/1 .1. /0.99:.:6.:  ..3  6315 .7733. . 5./3 :.:..:.5:.5 063156.1 .6.90 .. .:65.9 9.. 59.9.08:050.51:9 4731.7733.: .90:./5469   . .33.5....55195.69.065:..6..796.65.56906900904:..5..::065694....51:..99:..9.631:.:065:191 56065:5.41.5497.3 .00:1..3.   .50:. .33519.1..5..65: .9.:16:56.3  ./.0609:063156.:.3361/.00:1.93: 50 .14..:32:33.07.4.:150.5.979:65.5:.165.7733.:1/..7.9006510.3 65:85. :.51...696.56060/.00:1.:  .5.6515..: 3.7733.69. 50(5.:  #673  635.  &% /   5.5..9:.7733. &....5.5.:.6*    '%.65:..51699:765150    .0664450.6#9..  796/.35 9.7750.3 3.9'. :70.5 .51 75. %     5:   %    : . 33.5 79...76569..9.7691.59690.064450..32..7656910.691/.65  .907.6569:7625691/ :5..369..775   &0.:.33/53.99.6../3 69/:5..6569:7625691 .6. . 6990691:0064450.10.91069 .7.579:65 56.1006446532565..0.369.:.9 5.:./5.96 7961:  &0  .9.6:09.545.7.56. 7....32 69. 65    &0.33 .9..469.5.:3...:.9:1:09/1  .:0. 69 56.:  :/:.07.469.:.579:65563.62565376::::.906919 69696.79015:0.0.95065.4.0. :.3665::45.5 79:65  /  .65 69 :7625 691  69 .5:097.979:656979:65:69..96.5.9.5 064450.:1505.9/6969.96.50  065.96 69.6596 :.5..3356.:0.96  .556..315.96  6 .65:.14::/3 5 150 5 .3:67961:  &0     5 064450.3  8.51 6/./0691 /.5 6.65.65 69 :7625 691 :091.3356..9.5.6569.979:65#9611 '.:796/.5.5067:.5 10. .5169:091/.96 .50 ..50  79769.: 75.. 694.656.6973.5.3 69 .695:.559796/.:6:09069169..1....1/.0.3:6 / 53.65:6.9.336959..99/. 7.:. 5 .57. 69 .79015:5..697.3 .56.56.:.5 69. 69.55694.6515:0.6...6 . 0.35:.790691 990691 1:0 90691  69 ..:..9 :0 90691  69 067: .5 03 0945. :.9.65.9.6064450./ .906473.065. .. &. 69 .95 69 5:.9.. 1... 9:76515. .65     #9.145:. 06473.. :   95 7.3016. .. .. %65..3 &  "9. %.5 31 .5:.4.55345. . 69 .. 6 $65 . 6 4.659  '9:.: .99.10.3 69.3 '9.3  3:3.5. ..3361 : 951: 964 .3765 1.51 5 79.14::/35150    3.9..5 7./.02 6 4.9:76515.9 ..:691/79.5..179:65: ':.3150.6065:19.3765 0659:... 9:76515.. 6 .150691/79.7906915:94.7:6.677.163.3069.51#5.9 %3.::..9..7967935.5169 7:06360.7 906915:5.15150:..69.65:6. 7..65:67. ::1 .5169".9  0..3:.9.0.14.6#96/..51 6/..6 9 .7906915:  31%7/300......3 069.3069. ."9.331 .336.9:76515..7906915: 6. 43.0664450..1..65: /.5169..7 : 64 .51 515....::.3 50.51".5  65 9651: 6 3.#9.9#976::79::34.!6 5.37650659:.659 465.0.316919. 50315.1 79:65:   ' .. 305: ...775.659.515.31..659 .1 .7.9.93 9:76515..2::0..39'.14.65 .99. . . 906915 6 . :/0. 3...65. :.14.. .9.: . 0.65: ../6.5 5 150 .3361 .7: 5 8:.33 . 7. .. 6 .6 .65   /:5.51.69519%7 0. .::..7::4.:631/45. 03. /6...86.3765 0659:.9 :65 ... 5..!6    11.14::/3.1796:65: 6 .. 5 .651.4  . 656.:.651%7/300...3::.  &.5:  465.517.656 :.5&0.6:  .9:6963.10.6479:6545.96 476::.45.65.75.3016.3.    .65..5 69.: .6.. .69769. 5 .."9.1 .5.7630:.945.65  .:65 65 5.3315105..*     '6360. .45505./3:45. :.9 .133.677.... :..5 7630   .341.3: &%   (#.. /.. .  7.3  .5 33 .996 .51 5.14. .  . . 60 906915 .5  .65 /. 10.16..609 !.6 :..1 1:0::65 .51 .1 #9:5.5  65 6 .:9.:1 : 1.:.60906915.6 41. . 45  0 31 .1 .5 .3/5. 4..   95 .:.465.15150.996 .45.6   : .:1  5..9.00:1 7630 609 !. 25631 6 ..:.25 . . ..51 : ...606594:.14::/3 5 15056%   0796/.5  0 . 7630 :.6.: .0:   5.9 41.. . 0 0. 6. 65 . 69.7906915:56. 9.. 5.:.. . .:56....679.65...65: %.:.515.:.: . .9:.365   5 .6.3651.0 : .677..6#9.60906915.9. 469 :/:..679...931.1 ..3:   &%  (*   &50 .14::/356%  !6  0796/.49   69. ..679.45145.9.. 796/.: 5.9.: &794 69.94.79.4.0    :.3:   % !6     51 ( 516.*    '%.:9..5..65.659 !.:.   &       1    (*  .0..31.0.065:..3/5.:6.754/9.9.5/651.9:. ... ./..5:9:5 .14.000.5.5  69 906915 6 79.     ('* 3.5 7..5 /.39.996 .:6 69  .7:.1  37!.796/. 0..: 6 9:631   6550...05...775  31(*.6 / 3.  0..065:.50.:21 .465:065941/.775 '.9.. 6 77.95  5.0  31  ' ::50 6 79.51 !5..907.996 69. '91 69..0 &% . 064450.1 &. 65.336 %.1..(*979:65:.3 .56.3..:  '65:.5175::.517..065:.167. 336%..65: '%:1#5.2: .0.979:65:.579.15..0.65:6#9..3651.0   511    ..513/9..51. 6 79.96:796:65:6.9.51 69 10.::9.:6.0.519065:...9:..6.93.:69./3691.0 : 90651 ..0.5609 .9 .:.:6.7/3060969473669 .61335 5.65.:/.3.516.9796.:: .56. 9.511.39. 79:65..: &0:     ..  33 51 .064516:5/69:. 79:65641135.679.5151:.656.:..0:.:79::3906515&0.. .65  69  ...33.69.4.:65679..063.565:5:70..63.064450.51  "73 '699:  % !6  3 (#56*     .3:09.517955. .9:7.: ..65:679.79./3.656.65.51796.0.656:09.3796:65:66965:.1.679..094.65  6.9.0 &% (*69.339:70.9:.516..9.9.15..0.0 .51 5:9515:9.3:6631:.9. 79.563.79.:  31'3617961:..15693.5.06...33.979.:/.3614.. 9:631935.: ".0 7 (* .:32.090651.9.979:65  . 9.9:./:5::. .657931064450.9.3#9679.6 :.79619:.. .65.1 /. 1396/. 6 " !6 047345..6.1/:64 0647335 :.5: .5:..6.1337.796:65: :631/103..9.4 69 .651:5:     ...61 %   '%3:6 69.679.55694...:56.50: .65: .6:6.5..5    "   !6   : 7910.:5.:0:90:.:.65:   .9./96.90:'05636.59   '..330.1 5 ./3: .#9:15..9.6 065:19.. .1 65 ./3.: %   .6 910   56./3:.51   ..3:09. .679..51 6959: .. %    . 5.6 .1.:0:90. " !6 ::...65: / 79:65::25/.51:60. ..79. :664."919!6  ::1/. : 5.0:.697673:9. .45.65:. /:5:: .6 " !6 3:5.5:: .69/9.65 50 .775.0.39. 69545..65%950!4/9 #%!  .96 . .33 9..3:09.79:09/569.3 .9.::.510647.33 0.506 " !6  .9963 19.0503..03. .5: .0.915065:.6 06555..5:.3.03..9..&0906./915669545..9.9.65 &0.5. .:0:90: .:&0..9.: .:1/./.!.9:...969545.65..3 ::.064465 9950 54/9 ..794:.516..135....99...9.465 0650951 ..1.51.  9.50: 5.5:.6 7961 69 0.65 %3 (0* %:1%3:65150  "73 '699:  % !6  3 (#56*     :0::145:.:.3   3179:051964.7730.51 4:979:5.9064733556.5179:5.5:.51.10.0609.#673.9. 352.0.6507961:69...5.: . 51 .15:: ..6532:9065.51 ..65: .51:60.945.5276:... 51 . . ..3..53..35694.:..6/.91519 " !6 0... 515.0.58.656.6 .3 5694.65 .06697673:56..511.9..00:: 06515....3:65./6.3..69.7976:6.:73../35:09736:79:65: .9.05636:. 069.64:: .3:6 9...6.   .:..9.5.69 .55...667.:.65.69.51 9.5.:069.3: '1./3.6..0:.: 69 .30.5..9:969679:65.35694.1 .679. /.4.0...6 /6360. .0..5..51 6941..9/64../3 5694.9 .6 :.50 6 /315 ...3 5694..9.. :6 .33::69. .1    ' 469 985. ...9.69.3:3/9.916 " !6 4./61 ...79:65.:. 7..9 50615 6 1.02165:4645..:.95694..  .51 :369545.65:. .506.. .:700/6360.93  ...5.7976::/.96513::    "  !6   :631 .5  .3/5. ./35..6.: 69 ..69:6 " !6  .511.69:64669. " !6 16:56.::95.:.650 33/./....91:5.6.../51973.5.:56. 9.69::. 9.9169.6. :3.90:.91.669..6569.30.96:#%!   '76.1. : 34.365 69 15.65 7976::       3.. " !6 ... 965 . 16: 56.5:.:.9.02 679679:.65065:..47345.:.9..15.69767333653/7960::1695860.33:7017976:: '3.99.65/.:9.5:: 6  "  !6   0.51.556.51631. #%!  ..615.. 0....9:.65...3694::6.. .7673633:2.4.4.9 3662 33 :6 .69.9...65 .51 09045.47.33/:1. .51.5:..03./. 65:195 .9650352./.96.:. 69545. . .336%.15065:19..96369.0165: 403::9..0./:: . .5::6...05636..::2:.0./3. 9.9.9.9 .1.. 59.1 .50611 '.50945.: 6 .:65.169 73.05.51 0647.064795:09.....90: .69....5/9. .65 .. 69 :5 71.5:.005.6. #%!  /64.90: ./: .9.33.5.51 4:: 6 .659.9.13.6795.11..:67:.51 .. 7.: ..   '.65: / 69545..511..51 :9:   ' 76::/3. 4.06990.02:065./3 :. . .6. 5   .9.50.0647.656:5..579:65 ' 51679.0.: .0..5..5.96::690:69545.33   ' 4:.4. 441..69.65.7673:3/9./3 6 796105 .9.9:.:616/.51.65. 93 6993...692:656.0..3.0/5:   ('*69.2 ...: 47369: :60...0.59.3: 6.0 / 5 5694.679031 .964. . :.. /..45.65 ..91: .33. .36:.0. 4.9.:0 9.56.9.3.:..6796.:64.:6..16::965 .50669545.:...70.91. 064795: 16::9 65 511.46:. .3356.:::60.9 ..5511.9 . 16 69 .0647.3:05...1.0.35. 79:5060647335:. :.3.9 :09.631 56.0:656.33 ..79694.:.46:..5.65.655.996319. .69...5.5:4 50..:796.65:. 7673 .65:7.94.5:4.  .6 796 .656.65 .1 .3:.:.0.9.6.69..:/0.. 51.6..9.9.79:47.6 :99519 .519.65693.:..6.951:... 0.:963.0.9.....195..0. : 56.5. 79.5.69.:4.511/. 99.:.0/::5 " !6  79::9: .9 .39. ..1  5 5 ..5.9..65:.0:14..:.693 :6 ..::473  55694.6795..0.. 1...1396/.7...51.679. 69 ./:: ':.59..: .3356.6562..9.9.:.65   '9. 33 56. 0..6.9.145:... 6919  93 69 93.3 :46.5169..03...9.6.917976::  %.0 5..:.90.31.65:5.5 / 145:1   56.90691.6911.4. .064:964./.9 .9..516:.9.3 69 .79310.: !69:. .9 9.9.5694.9 ./6.5/.:69.65.. 0.50660.7.4:. 0631065:4. 99. :7.5... 631 9. "50.5.3.92: ..6.1/.7796.56 69 .51../9..9 : 0.65   ' 0647..51..9 79. 6 5694.:.9 963.. 16: 56.9.63..4   (*5156631.. ' .3351 3.9:1:51.5.5355.9 .59 ..91.0.4:3: 65 . 9 .:..51./. 69545.: /.51:6.: 695.:0:90: 5.. 49...../.:.33.6 79610 . " !6 :.656.9  "73 '699:  % !6  3  (#56*     &631 5 0.9 7.. ..3.1 69 5190.3361  31  ' :: : ..5 ..507..6 79..3361   5 .9 ..65693.:56.156965:.45.: %  !6     .3361.6 .: 98:.:.5::70..8:.9 653..:...65:090 6/.6790.5:.0065.5:70.69545.47.2757696..49.50.5   51 . 0904:."4/1:4.52 .65 6 /..52176:.. 69 9 . 6919 6 .65.3. / 0..0. 1.:. 5:70.651..0647.59..0065.769.39 .  519 . 065.94. "4/1:4.550.516::9. 5195.606473. 16045.1.1 / .5: (*036:.59.: / .9...659 4.50::631.6. .:315.3.51 :0 5:70.1 .65:    9. &.5: 065.45.:.65.6 / 79:5.69.306515. 6319 4:.:165.5631:. 6 0647.98 65 5.51 ."06 ..  .. 56 7515 3. "4/1:4.91 .5794::656.069.67/3060.:..691 / .:0906/.75155:.5 : .51.65 6 %   !6    &0     . / . 65 &0906.654.5:.5..654..0065.9 4:.::   069.:. /../.916*    9164679::65    :.0.51 %    93 .3361  .4511 103.15.46 . .3:  69   50./.:59.51   93.: .1 .65   &0  %  !6  50..69195/9/9691930.69065:5.49.50(#.. 565.:/0.. 6 77.9 /5 :.:   5 565 .4.5276:. "4/1:4.46  .52176:.069.6561.51.:/0. / 56.9:10.45.33/ 69. .::.9 6.5   47.15.. 195 .1695:.3.#3775: 3.69    50. .069653.5 5:.52.069:6..5..1 69:96: 993.5.  .::6573.50. 67556...63365 07...45.5 069.52../3::09..9.1.:6#! .1.7/3060.6."4/1:4.:.6.526..5:70.79611.3150.::9.1.65 / .:3.636625.6694.9.9 . . / .5.176:..:/69.5   3.9 .33 .: .: 56 7515 0.3.465176:..."0 6.6  %  !6  5  5.:.6 /3 .0.#/30:..60647..52 6 .69.5.5 .65.45.52 9.6561.. .65 9.65 /69 ..656.50.5.:9.65:69.: :700.65   5 ....656.:65..::6/9/9691930.45.. 5:70.  .6:69.51.1 ..:1 ..50 :/.606510.0065.75150.99.1/..4:...65 34.9:10.94. .9 . .045.65. 0 631 .045.:/0.:50::.52    7659..307.1.15.5517515. : :.069..:70.52176:. 65.65  6 ..0065.176:.65 4.5.9  ..1...5.  .0.3:   76:.6/. %  !6  6319./.330.3:   31 ..56 :9.. 5:70. ...49..3  69 63.. 4.16 .654../3 9651 .51.6.52.0065.5...11. &0.::647. /.: 5 069.96..9945..695:70..9.1:. "06 .5.9 : ..6796106950.: /569 : /5 0644.5  5:69./:63.. 0. .5.06 #3  (*   '69196. /69 .16. 5.96:.9.5:. 7515 0.5 5 0..    5.    76569196.    5.63.6:.36959.9:3.3  ./...52 79:6553 ..65 '/.::/0.:93.65  691:'  .9:/../.75150.03::6.93  .96..5:70.616:.0065.3... "4/1:4../03.4.176:.65.7976::653. #3775:   69.65  ...: /69.1.9315.169993.5/.60647.6991/.9 : 9.0647. 65: 65 9 :70 964 065.:1 9:.90./.5. 9.0   31  65.90.51 .65:  .4736.5.3 9:.5.  03.51 . 065.6 .: 6 .9.9 476:1 /0./.65: . :70 . .5. 6 .969  :/0.:1 9:.90.9  . .51. :.65 .06473359.5.5   ': 9:.9 .: 06445.1 6430 796/.5:.51.16976::/369/9..:65.59 .65.51 .969 69.5::   65. 1.33/.506345:.3356..11.7.79:5. 5.473  ..3 . 93 :0 ..9 519 :...4:.51. 79:47.:.69: .65: .556509:9640. :09.6:7769.31..73/:0.55.: .5::5.47. .9 05:69. 5631 5 &.90. /.::4:.. 69.   69 .1.35..65 6 065:.969 .90...51. 5.47.:..5..31.. '03.9.:.30.51 32&0  / 6%  !6  0796/.9:.3 5.51.0...4.:70 ':93.9 56.9...51..:195.90.0..511.3:.369545.6.4 195.369165.6763..9.065.0650951.65:51653.916933:0.57961 .6:7769.65: 65..9.9.:7.39:.: .5..:/:.6.656795.0.6. 51. . : 32 &0  / 6%  !6  0.6763.:...653.63.: '6..0.03.9 5015.9..65.773.:.956.779679.065:.9455..3.79:5.691..5.5993:5.31.1:/.0650951.065.1.3.. .51. .:511    .449:.79:5..:9:631/32:5.9... :31.33.93.694.5.619.6:093.:.1.35.59.449. 0. '.656:70:.69.336:.:19505..:.6569..90..9:.5.065. 9 :70   !6 :0 9.51.9.69.6 .33476:1/0.7/301/.:19:.0.65:1:.:6 .03.47967946..:65: 5193 065.5.. . 7.. . /.90.9:.9 6 6 7673 33 9.65 . *     .::4/3:519 # 3   .5:5 065.. #/30 ::4/3 0. 535.90    ( 516..3 .5.9. 46:.65: 3293.:.4 73. : .   "    &%   ..3 93.514. 69 1:. 6    ":45.559663157/30..5.0.656. 5./.:1 964 065. 1.90.154/9:.393.5:::5. 46:..:&79469..065:.3 5...65:  31'5.02.  &   1 1 (* (/9.9:.5... 9.9 .3 9:.5 4769.51  . 535.5..:.69545.37696 .1  ..1&.9.3:.3 . 69545..5.6.:9:.9.3319:.6565.9.9: .506.5..35.. 69 1:.: "95  (*.5.5015.45145.:593.3 93..:779::656979::65.9:.. 69545..    .9164: 6:70 79::65.    .. 69 :/:..:....3.111*   ': : :6 .5.5179:: :569.6..65..5:5 065.65::05.9 .3 69545.3155..1&. :1 964 065./.5. 9 .... 69 967 76331 &0651 ..4.79:506.1:65.1  :0 69545.3 /.37960::.3.3.:.5.51517515.51. 9.17965... .96 .65: 33 699 94. :9  .559.03.0631/191964.0.9. 796/.76::/3565064450. .65.7731/...65.11. ..064 6 65 0.1.9:. 7633: '6315..30.30. 06445.6.9 ...95.:56.9196476331511.6. .2: .33 56.3631/17916 :.:.65:  :50 .51:.96:03. 69 930.7633:0. 59.5961:.6 :... 6949 : 653 7..6 .37976:58:.65  &60.5:9: .:.96.69.:99:3.9..9.   519.51 573691   53:: .:: 165 6430 &%(*":45. 9. '9:56:65 69 .:165.65.6:56...9.: 33 . 6../0640.13  .5150/579:5.:/5.54769.36430065.30..3 :5.9.964.6.45.9793:703..909: :60.60..1  .9:.1:691969065:.9 :30.:779::6569:70 .57976: '. :9 8:.79643.965..5  65:.7633:1:6919.64309:63.656.1 :50.3 93..5150..6579050.6.4 /.4..5 52565 .60995.9.:63159.7633/&!79:5.:.(.51 0.1 * . .5../36.9: .6.69.: 9:. 3.1 69 /.515.5..9769..2...5..:.3 5.33 795.51::. 6 .6.66.9.5103. 5.: 69....: 76::/3 / 979:5.069/.31.. :779::65 6 9 79::65  6969 5.5 65 .1..9:5.65796/.530.5.9.9:3.69:/:.:631 / 5.3.9:   6969  .7633:69.65:.9   '" :065095.:796/.:56. 6.556.1 .35.. 6.:.5 653 / 510.5 5.9.:.:.66/96.65 6 ..35.9.935.9: 69 7.5..6973.9 4.. 06330.:...6.:   5...0.6 .9  &33.:.7.6 .5 05. 6 .*     6:..4   516/.:4936.9&.65.:..3: 5.3.5./3  .: 0.65:.5.: 40 .609.31.6..69.7633 9769.9:.9:0.. 593.6:6...8..1. .65.505.51 065:65 5 .4  31&0...30.30.9    .511.: .5. 7633 3 593.06510..1 .31 6430%:63.90.51645:: 6 :30.. /.9 : 69 .5:.3.5 : 50::.5179:5.76335967..51630.65 65 9164 6 79::65 : 9. .3:05..0 .5...9:  0 9.34../3 9:.76335: 1:97..6 .3.47. .51945..6..369545.559..9: !.5 : 9:.7976::593...:.:165.9.07. 69545..65 5015. /.60..9:7.:. 6430 :79.5650.::.3. 7633 1.091/3.. :..9. 091/3. 9:..:. 065::.:.33 ..7730.51 .: .51.56./35694...6.:6.9:  16: 56.9.5164  :6 . : 56.69..:...6.6956.65: 0.675656.51.30.. 9:3. 69  6 ."95.990::. ..945.65: 50  6430  % !6   .511.3:: / 5.. 5.  5.:.9.50( 516..65:.6:6065:65065:195 . :..9..556. ./../:63.64300.0./.76335  "5.96.69:656..:65..4..9:060:   '..65.:9:     ./3.516..31.519.3.5.5476:1/.6156945451:6919.96.675 .95.1..969 /:.5536064450.51 .0.6..9.765694.6.//96./6.3.6.51065:65.5:.1 .514.9.9.16:56.9.51.656/.6430516 .0. :....65:.5 30.0 .. . 69193 7..51 7/3010.345.1 / .6315665:. 5 9105 1:97. . 6 . 065. 6965 794::  .51091/330..663:69.9  5:.03. 5.1 6 1:97. 19.... 41.1 . 7633:  796793 06510. 30.3. 6 .: 0650311 . ./9145.65:  .1 9.65.9.5/.65 : 6. 065:.69..9:.5169 . :.   $.5.: 6 .51 .65.65630..9.33 . 1.930.516.5 9.333:  &.69. 33 56.0. 565.9&. 79.65:  50  &&  .0.50(#.65.::../7/3:1:5  1.5   5.511.5*     &0.65 '&60.  0 7961: &9: .5 5.3 0.: /69 .530.5 30.3356.569769.: :.5..5 360.5   1.9 30..65  "   % !6  .!96.. / 7/3:1 .65 0...511.:.65   6 %   !6   ..3 0.5/..65 .10.51 :9: .3.:/69. 602  565.:. .: /69 .90: 6 9164 6 :70 .:.  '9:.6 :.6.. .53.51.6.51 :5   1.65 :9:  0..3 9:.656. 79:47.31. 5..:.:.99:.0. &0. .96: 31: . :0 9:.5:9: 5. &.9. /0.335/::.79991:.65  0. 5:7..65 3.51.3.91  .79:: :0.:065:.59:593:3..65...79 6 59..79699:..65 50315 30.065:.39..9.:65 9519 5   631 .: 441../. / ....69:.9.59 ..5. 1.:6:70 79::65 .1796:65.51.31.51 79:5.   &631 ..65   6 %   !6   065:.:.. :60.9.65.4: 6 5694.99:.:.  ...65 ./9145.9: 5:69....3.51.9 .5694.796...65630.379:47.51  31  69 9. 0..5..:.5   1.:6.3 0903..6591646:70 79::65 .: .65  7/3:9 6 .65.5.3.65 6 5.79699:.65  0 ...5:..6.79:09/17961:6 .0.:9 : .6:09:.9.3 30.3351.4.:69.1 / .5 5065:.36.7/30.5 #/3:5 69769. 360.065:.511..5.6569.4 6 7969 9:.50.65.5 03..691646:70 79::65 ..95..90 5:.   511  ...3 79015 .65.65:99:3. ' 69545.65.65 06510.79:47...79::   '6/:9 &0.79::/ 796/.65   !69 4.:0.3.5.: 6 9.5 ::.5./9156:65:.:6..065:. .51 .569045. .3 30. 4..31.5.. .91 .65.: 6 79::65064:.. 6 93. 4.. 69 . .0: . 64305199. 679.50: 69 .31. 9..55 . 6 769 .65 6 5.:  . ..6 .5 65 41. 65:.:45.1 6.65 6 41. . 6430 :79:69 769 ..0..  &0   6 . 5 ::.5. 6 064450..6 .3.:  . 5645.65  56 79:47. 765... /. 763.19.3 ..:9 32 &0      69 ..65 6 9.65  0 : . 65:958.367769...47..:631..506.::/0.650.::. 69 1:.9: .5:..5.3 .5:5 065.1&.. 7/30. /:. 69 :/:.4650..51694:.:.3 69545. &0 :34.6.:33.9 .9164: 6:70 79::65.1  . 69545.9065.65..5..:./..111*   ': : :6 .5:::5...154/9:.9:  0 :43.65 ' 5..9:     .9:.9.3155.5. 535..0 ..9:.3.:65.945..90.:60.165.065:..6&0./39.3 9:..9651.:.5015.9.31.35.:5694.9665:99:3..519.02.4 :7.65 6 30.65:9:/:. 69545.34.5.    .:569:.1&..    ..65::05.17961 56.65: 65 . 9.5179:: :569.9.:697/30 5694.90. 46:..54/.37696.93 476: 9:.:..3..1.:6:041.5. .4.: "95  (*69545..91.0 !.65.96.5...7/30.51  .03.9 .93...5..:593..56530.69..::.9 .914609.:     !690..:779::656979::65.9:69.556.61.3:.5/47361.: 569.:476:5..:&79469..6565../..51..065:.9:.  &   1 1 (* (/9.90.5.:56..5.3 5.1&.6.6565..1&.65:9 9:3..1.9..45145.6 973.93.511.9065.: :9 : 506503: .65 :9:   .: ...9.5 4769..5.0.65630.3319:. . /.:1 964 065.5. 7731 / .: 69.3 5.5.1  .65 65 9164 6 79::65 : 9. 5.31.3.:: 165   6430 ":45.:.:63159...1.:. 593...6.1  :0 69545.5. ..0 .96:03..:.37976:58:.:56. 5 ..54769..31.6 ...   6430   519.69:/:..65    .3..6 .65:.3:: / 5.7976::593.:631 / 5. 9:.1./0640. :779::65 6 9 79::65  6969 5.6.90. " 95 .. .:.5650. 3.: .:779::6569:70 .9:.5 : 50::. ..3 93.69.6 0. 69545.9 . : 56.9.51::.9:.: /5 .1 .9 .5.9..369545.9:...35..35. 5: .909.::9.33:.65 &0  .3:.69:  . 76::/3. &0  .69:.: 69 .5656.65 :9 9:3.656 79::65... 69545.940.9:..533.90.5.:4::. /...30. 79::65 6 67565 0650955 .:56769.9 67565 4.. 7.9 69 .006915360:65.654.4.16 . 6 9164 6 79::65 4.169545. 765..:  .5.:.03..9   56..  &0    :6: ..:.965  6. .:  '9.: ..10.735:2 !.3 .79::65/0.:6.3.9.: 6 ... 4.: /0.47:9 ."95.0.0.69:.:  9..:1.65.69:1.5..6065:19.  / 79995 79:65.9.3 67565 .:   ' 065:. .. 7/30.51 ' 06445../0.: '5/.9 / 5:7.56.:.593.2::05.65 6 30.79::65.51945..656:70:631/731653. 5796.9 :/0.5.7731.0.6 :.6.9.3365 ..29:   5 0.33:779:::..51 6.:.4 :/0.:07/30. ..:.35.9 . 4.65:. :.3 9:3..5 .:065..79 06345:.63 03.64.9:.:.306550.6.4..0..::/0.1.:: 6 79::65  3 . "9589:631.6..6&0     9:.:779::65 6 9 79::65    796/..09. .654./:.:60.5:.34.351..65..3534.51.9667: "5:43.34.0.6569. 796.51:5  1... / 191 964 .691/.51":45..: .69545.5.3 .956::5. .651.65.5 65:.34..4 /.79699:. .794.9   55:6..51.9.655. ...656.3/69.51469.:65796. .65.:: !6656318:.5:9069.3   .5 /5.5 69 .90.795.65.576:.36/:.65. 3 4..35..6..:::6:70 .1/.9 9 .151..556..30. 6 0 .6.: 441.37.65   '::05.656.0. 7./69.3 :779::65 6 .31:. 6 .65.:.3/36:  .:69.1.6796:9:.:66350.996334.65/..54/9.5...9.1 .165.0..6: 0 / .:6 .65..5 691:  .315!.69 6 :70 . 59 69 ..998945.5.3796:65 9.: 6 1050 4.: ..9:.17961.:69.0.4 : 03.45. 59 /5 .0.5:769.51 . 4.96/690669193 69545./796..6.: /590651653507.1:70 5!.  . 5:3.796/. 31 .795.. : 653 69 .9:...0.6561.:909.93 6.51 75:45.9651.96:0:3.69.56919.103.7 .3 '796/.5476:1/%  !6   &0  / :56. 06445.6 50. 7961 6 .. 9.:700065:.3 .51 .09..5:..9./.9.65476:1/&0  0.5   1.5::&0  964%  !6  &0  / 0.65:   ' :09..:.653./:63.51 :/:.3 796/34   ': 5031 .51360..330.6 /.516/:05  ...360..0&0  3.6. 0.   ': ..65.:. ./:63.99651: . :.: ..50 530..0 (&*0.50.5.5 441..:653 69.6 .: . 6 79::65 : 190.3#9::3/ 6430 .51 : 56. 065:.330. .5  ..51:6535015.6 9. 4.7/30.:31  ('*796.50:...1.9.0 6 . 6430 69.:56.. :60.: . 6/:05 7/30.9. 4.3.65..9... /9.3/69.515.  .:441.6.. ..1 3:: :6 /0.   !69:.. / 56901 . 6 . .345. 3  &0    565.9:..1:.4:.9:..35.6 / 79646. &0  /.:45.795..:.3356/.0 .09.16. .3.39. 9:3.:/. :779::65 6 :70 . .9.:.653.90.3:.:6..95.51 .65/56.9.9:...965  6.9.5 :50::.51643069   &0651   5  .9...:69.6430:7.656 3....5.19.64.765..:.69545.5 9:. .0.3::.65641.... 5..5/. :.90....9.69.3:6 79611.3 5.1 &0   ."95.5":45.65 6 9 79::65 : 653 5015. 69545. :/:...6 .1 .1 : 593.43 .. :6.:. 6.9:  .519:69.. 79::9 65 6..50.2 69 36:5 0.65 0.: .65 6 /. 09...  525 6 ..3311.51.511.694630.650.1.45. 515&0   #9:.51.6795.5:.3. 6 79::65 5:0....40.65065.45.0..0.3::9:.9..3:30.3.3 .3.5 065:0.996379:1/75:553. /6::9 9:3. 51.92. :.60.5 33.90.1 6.9 65 :9: 06510..:5.65 4.5 53.657967./.:.. 145:...3. :0 :70 09.   &0 (* .51.51.. 0..: .51 .  .0.:16/. ..4.5.:.: 769 6..650.94..69545....: ..0.9..: 0. .67 .: 9..:69.5 / 5621 / 6..  69 065:0.00./.: 6 .5.5 ..9./95::1.5:70 /0.7976:6 .616 2  '.9156.1 .6430:5.796/.1569. 6 973 0. .6 4:3.303.9:   !6 795073 6 8. 93.9.556...3.51 :.511.:5631 .3  3/36:  4:3.: :69.090 6 .51..95   ':::93..769  '6 :.4: 6 . 0.7795:65 ...5/4695. /  .511..59 6 :0 3:   ':  519 .  &/.3 9.: . / .6.. 6430 .1   !6 9.3.: 0..67 .9:   . 1.: #9.9 165  . 9   . . 559:   &64 .9:   &64 6.1 .5.9: .5150 6 :64 6. . 56 .6 / 15. 4..:/531..655.79699:.6 :.65190.: 6 .4.9..653 .5.6796.: 0.77.51. 4. 0....9:.5... /. .9651.0.51   .::07.01 / 4.6.1 7961  .3..:  /.65./3.9.560 '%65.1.65:   '6:44.693/36..53.5.659.6517.69:.505.9:..5.. . 0.1 0.6. :779::65 6 9164 6 79::65   &60.31964:631/99:1   5 156505 .94. .. / 5:05.33:7769.7.5 5 .3'9.9 ..3 .476::... :6..45.. 190.6 / 79646.5615650:.5 .79950:69 /3:9:70..3 :779::65 6 .65 .1.656 :99:3.31/0..0.9... /.6 :0 :779::65 : 653 69 .3796/..69 6 79::65 5 ./3.31.50( 516...  5...9..*     '8:.50614609.   31'10:65.95.::.3 .:  7.51 . .&0   :5. 90: 6 9.: :6 ..659 .:  .9145.5.60.7/30.(49*3:3.3 5.369.9&.... 34. 69545..: 145:: .5: 6...1/5.:0.54.5.9:3. 9164 6 79::65    .: 46.5 631.93..51 .9:67/300655504.5 5 .6 ..6.979:65.:0.9.3.. 9 56.1/:779::5.6 .945. '6516 69:69 9. .3 .. 6.:/3.694679::65.3694:06510.659 3.. :0 .65: 50  "   % !6   .05:. 0.9.0.9 9:15.6.65691.5 / . : . 0.33.1.. .6..936.  53:: .5..: .659 9 ....51 33 / .0:5065::.. 065:....33 . :3.690.:0..:9.4.3 0650951 796: .: 44/9: 6 .30 5. .3:6 .91 6 ..7796. :.9 0..25 :0 ..69:..1..9.1 / 7.3 4:06510.65 ..9 1.   6 .511.1 9164 6 79::65   &0 .3:6956..25631.  .6 60. . 7/30 60.1 .3: 563. 0.9: 4.4.656.3: /.5169 661 46. .9 .05 65 ./3.55 .4 ...9:. 653 . 00 1.3:.5 0.45.3 06510..33  595 65 .5.5 98945.9.9.51 :.65: 631 56.34. .33 . 79694.:6510:65: '::..6 796 56.: 504/5. %:1 #5. '9065. 93 631 1.920.:. 6 .514.3..0.6 9./6 .: . 73.. .6 : .51 69 :... .9 .: .:69..65: 6 60..14609.    93 73. 661 46...55..36.: . 9 .9..31 ..:. 631  .: 0. .9355.. .35:9.0.  '::. ./3 51:   69. 796:0. /915 6 7965 .3:6956.3 61   .6 . /915 6 :65 .0..0.6:65 64 :0 1.:6 !692'4: &33. 653 / 065.50 6 ..5..9 ..00:1 .3:69.: 4. /.916...1 .5: 964 796945 . : 1:0.1 .:./3 51: 69 4.:.91 .05 65 4 .. 69 ... 25631 6 .93.9 5. 5.. 7. 9023:: 1:9.659 4.. 7/30 1.9023::1:9.:.3 4.: .45.5.30.33.65.. 0.: .9 7931 .. : 56.0.5 .:.916.65.0.   5.: .50( 516. : 10. .:  69.!692'4: &33..  ..0...306:  59.. /..9 / .. 25631 .3115:5....3 4.: .51 9 . 67565 .. :.906445.9: 6 7/30 5.1. 06445.4. .5    51 (336:336*     .3 .6.:8 69. 69 .6 9164 : .3: 69 .. .:65.: :.906445. 5.969. 9 :70 .5179::796/.: 4.65 ..:.  .519 '160.65.9./3:1 .3: 3.3697/309 964906951. / .3:66193.65693/369:3..5 79::656 67565  /. .3559. . ..65 : 141 4.51 .:165 :.3 4. :0 1:091. 7673  .956. ..3: &% &7.5   . 763.0.:.4.0:4:65.1 .5.:69.3: :776:.5 & 6315. ..657/3034. 9.4.9..51 ..3  &% . 50::.93 ....0 . 9023:: 1:9.5.3:6956./15 065095: .4. .3: 47.*     :0::.:. : 44. .9...06510.3 1.65    . .53::796:.      .51: ..5:79:4155605.:94.5..33.65 : 190.775: . .7/3060.:651.5:.31   : :.0../3 47....0..:1 65 .30     .160.677.9: 65 4.0.5..956..:691646:70./3/5991964.9 06445. 45.0 9..1 ./347..::4/3  1:0::65: 6 69 46:.:.517/309:...:365.9. . 4:...3: .45.5:.0.65693/369:3./3   5 6919 .6 . 1:091.6 / 4:.5 59. 7/30 60.1:141. 06445.3: /0.9:./3  .25 .0. 7/30 79:65 5 : 7/30 0.65 631 / :.7.19643/3145.67/3060.65:.51 065:...69. 09.519   31  .33 7961  ..65 6 .306510.95:3.:  31'.3:. ../3 47.31 15:5.39 1:091.1  7/30 1:0::65 : .9.5. .6955 06445..3::  5 .3695...6:60. : .   . :6313....'9.0.9..65:657.0:660.9.:651.!692'4:160.:.33.95.6.969.::9..6 .563/3145.6:3.306510..5.1.3.698909..9. 05:69:7  :50631. .15:. 43.!.51 69.255:1.   31(*1479.9:..:0.3  &% ./3/.9. 69545.1 .51.9 6995.!' 9:. :0.96.3 6 ' #3775 &. & 99:1 .9:59643/3/7.:3/.:51:.5:769..065.5.36595065..77. 6 .. .36513.9:76515.3  .1.9..6.5.. .9 06445.51 .:33  :.69......5 79.::1 691.659 79.64796 .141.56.. 9. 7960.51'9.9.65..:0. 7/30 5.  69.653..4.9:76515.7/30 9   .659 79.51.   ..51:762:4.6796.79:65. .0:631/1..69  .51:/:85.:/51..7/309. 1.6 7.0631/796169/0. 9:76515.3:.5519.9 5.0:4:5:09/31.65. .: 79646. 5 : 06345 .:.!.:6.90..79...:9.:616/.51 '9.51&762:4..6.75: 6.3. 7.0. 79.51 4.3 069..6769545.9.:65 .796./:5::45 69..3.90.909.9 96..:.51../09..9 /0.:..9919646056.51796455.:6.51/.5.5:769.::41.4.::45.5   51 (336:336*     6345:.36:1.:.65.3: ..69.65 . .6/ ..1 .!692'4: 935   '!' 9:.0. 69.36595065.65 .065:85. 65.63.5:769.6556.5.. .69545.7960.: .655. 160.6.51197/30.60.7735 .95 6 ..564.69...69.:   "5 ...065950.0.9:76515. :6994.65:...65:.7961 .6/0645631 '7/30:794...:56.7:67931064450.659.9..3350:..65:569.506445.5654.551.95..65969.:/0..5. 661 5.7/3060. 56.: .9 : .5   51 (336:336*     : .: 365 ..51.9 .5:.65:065./3 7.: 5.16:56.:..6:6.79:65  69.0....73:.0.:7969.4.3  &% .36169.9:.4.:  ..   .065./35.969:/0. %:1 #5.9.06510.5    51 (336:336*     .9.3:6 151 7/30 9 5 9: #9610.69...5..    6 ..51 8.9:.:5 :64:5:..5 9.361 .9:.0...79.936336.69 .7/3079:65./.5 .9.3..9.:6.51:50.99769..07.4 461635 69/. 4.65:.1    .51.5.65.956.9:6319: 5. 1:7.69..46:55..07..  69.06510....65 69 :. :/0.6.5.7/3060::65.7. 6 7/30 69 59.006473:45.9:76515.. : 56..9..99769.313 /.96 765.7/309 . ..063156.. 5031: 7/30 609:  .3   &% .65 0 47.5.:5.51:0.8..95.6:0.064450.945.:.50::.5:165: :..659::695.: 0.6/503115.3  &% .9.7/3006445..!' 63156..:  .: 5631 5 .65:60:17654.   .51. 69.9..77.95/69..07..5.65..:.6519.59 ' 3:.  .996:.9.5.  31  ../3 53:: 651 . "/6:3 .167.65:69.56.5.5156915. 6.556..5656.7/30. ..7/309 .635. .9.03/9.:/.69: .991..6.5.51 73699:  .3.65/.15/.01:6419697..1 .  6'%:1#5.: 59 /365 79...9691: ..65 #..7/30  .3:8:.:   .  65:.796::6569 0... 549.0.95. .51563::.92:  69.0.7/309 69 0631 /  .76:.:  69  /96..5.. /5 4.31156.79.5:70691/..37931698.0..1%396.5    51 (336:336*     6:...6956.65:  317931064450. ./:63.7..313 7931 064450.511.9. 44/9 6 659::9643.9:.369..79:656 /:.473 : 651 5 &0    9. 7/30 ::    .3.36103:  3151:7.1 .3373.654./3.93066:.19 ..064450.79:65.3137931064450.3.3 5.37931064450.9..3137931  /:63.9 .50 6.51.65: 519 9. 7/30..6..: /064..796::65.5.5.9...3:563169/0.5..:1153/0643:::6493/0.   '6 .5.979.956.:. 0.:.956.76:..7365.5031: 5:69.69.511..03  6 ..51.: .69..6.5644.3/.65 6 8.67/3006445.6 .6560071 5.0./3 46.6947.   5 . 9.:#5.65 1.9796...3 3.   : 56.313 7931 064450./63:1 / .3   &% ..93 .5 03: 3:.65:75.: 695...6. 75:5 3/3  69.5.065:.. 79:: 9065....51 6 .5.5 ..9.9. 5 ...65..6... 3:3....556.1/.65:75..: .50::.65 .336. 549.3 9..:65:67/307630.3.. 33 6 %.696.:16:56.5.506445.65::50.1519&79469.591646:70.79::  31'93657931064450..336%.691646:70  ': 065:. 65:...9651.65 519 9..6::794.:5::56.:.65:.5..5.5      51   (336:336*     '"06.33. 0..695.:6 .:65.59164 6 :70 .:#5.39 6 .65 5 .651.9.:69..9:654.93657931064450.516.659:.67. 49 .5.65.1.9:.. 79::   : .66319. :.13 5 !6 .92: 69 .5.:9:1.5.9:67/305.:796/.:.:6994...3061/.50%:63..932:7931  69. / ...906445.6956.0.0.956..9 .7/30.934.9 7931699..:  55.1&. 6 8.6  . .5   51 (336:336*     :0::..335 965.3      &%   .931. .5..65:0.. #3  (* .7931  '6 / :9  . . . .6 / .4..51.6..7/30 :9. 6465:.33..3 ./:63..:  .4.5169193::.69:70.33 / 1 ..145:.1:.5..4   .41 /.65: #02....50.:69:70 979::65 7. 5..:.6.33 4769.3.5.9 6 .5196. 5..5.9:.51 69193 .52 65 4..4/./3.3 . ./3 517515. / 76:1 ... ..9.3 5.7730. 5.:. 9.::4/35.   .3../9145.9156.9:.5.:69. 0.9.5.. 9.:7073:7:0..51".33 065.65.51 .9.69:70 :093. 6 .65 6 .656.1 .00691. &794 69.: 51.51 1:0::65  : ::5.9../:1 .999164:90:15.:51:75:.9&43.5179::65 9:.65 ..65 .35..5 .5 .6..9.:. 7.9.     91646 79::65 51: 65 600.:.79991 76:...461:75:5:.51 .0.93..5.:: 69.65: .9.9:.691:0::65 699765.::1.9. 59.: 794:  .5.9169.:. 9.96..655 3.145:.: .9:./.16 .193...9 69.05..3691.3 . 7673 .3.96 ..3:661: 4. 3/9.:59/5519:.4:.:.47..94:.09.9.6 .9 .9.5. 69 5 0:: 6 9:10.9:6.: 469 0647.659:. 4:. 695.: %.465.3:...75:45. 4.6069.9: .50: 0..336.65.3: 4. . 653.51 79::651:7.5.655 6 .65 5 79643.661./3.259 69 .5169 .54769.65.65:32702.5.0.6..65 6 :..1 ..19.0064461....1/3.51..511..16   &%   . 93.06473:60.79656501556509.0. 796015:   ' :/4.1 / ..0064461... 59..65 6 :.: .16:56.:60.: .5.51 .. 563.33:.51 69193 50..9 7630 : 56.655.9..5.. 79646../915..0  '9 : 56 .5.95:..& 5.:635.5..4 6 .9 :7.6/0.:..517.9.65:1656.6:: 5.6510.50 6 .6794. ::.065.      5 % 43 #  9. 93. .33::.3 7/30 5.&79469.9.: .3.6679.6... :7.. 7673 :9.:34. 65.979..69:70..03.6:135:   31:..3: . ...569..9:6.514.6 9 69545.::4/3. 69.4.655 69545.:   "5 6 . 9164 6 :70 . 4769.:30.65:..656.7673 :.:65%43# 9. 065.  515. .514.6.656.505. 5:.9 691:  6 .51.9.5 :....5 &0    9.0.:.3.0 065:.4 73...03.. 5.9.996:.. .1 ..9.:. .:.559 73.5 '9.3 3.6569164679::659...:   #. 3.3 5175150 . 69545.5.007.9.9: 631 .51 33 ".4 ..:.0.51 . 0.1/7.. .65: 69 139 6 :.3 06445.5 9 79::65 . .55.96331   ('* /..0 .065.9 ....9....9.: : ..961   #9:0515 964 . . 5:.. 9. 6 .: 5.5 / 065:.659: .: 1:09.:65 . 10....9:. 69 919:: 6 9.9:   655.796.2 .0 0 ..56379:70/..31 %135:65.3:164.5069164679::65.50.69.9:50. .1 .6 .6.0 .065.0.0.51 7..339:. 7960::: 6 10. 7/30 5.....5.79031.6:9.03  6 .0 69. 69545.9.5.:.7673 :9164679::65.3 .:.6:7.145:.9:700.65: .9.50  50  69 935: 56 4:.03.. 03.95:5. 98945.: 6 8.5/ :091653..1..51996573..../.0..467/30 1:0::65 0.93:065:.3../:63...515 .  65:.05. .:6:.  5 6.0069119164679::65.:66..963/..564 /.945.50565.13365.45.65 .:.. 3/9.55993.4...33/7. 5/.3: .: .  (*    . 653/065.65:6.5156.. 10.:5:1 /. 69193 .36 :6/9  565...53  :763 ..:1 .9. /5.51 .3:6/9.:0/31465:.9.65 6 :.6::  ' .3 5175150 .6:. .145:.0.9.0 .47..13 6/:91 ..5369 5.. : :.1. 069.. 65 .5:.61:76:.1.51.6.51065:.9.6..7/306.93/9.::631 / 1..:69.9/:5::5.96.6/.9.6569031:60./5:3.: .469.51 :631 / 553501 / /.9069. 90:/..:34.:.50.9.96.:65 . ...9451 765 150 796101 5 069.50:4:.7..7.4:.31.31:19..3:6.65.515..5 796015: /69 .65.356...77..:.0.04:.:  5 % #.90.4 '6/ :9 . 461:5275.:  7910 69 :47..55 695.:9.9..1..33145:56.:.6 553 5 .:::3:6:.:10.  .9:70.50:.069.46:.653/..51:50::.65..656:.5.50 %:63.30650.9..973.511465:.56:53505....31 55.9.145:.65.5/56790./5. 069..9/.:.702.515.779679.9.1..9 .19649.65: 46.15: &0.56.1&.65   ..65:696.910:65: '.5.5..1:..::.:.91:..796/.9. 0631.6:: .5:.9.6345.0::.933/9.0.:.:/5931.:4:.0/..3.69:.0.:6.. 1.:   '.9. . . & . 0644..55.9:.51160..65579643..:69'9.3465.610.       1.950569545.515656.&79469.916   . %0.65   .0.5.5:7..33:3.50%:63..('*  %  !6  &7. . . 0.: :1.5.::.65 6 769:   90. 6 465:..00: .6.469 .:365.65 5 79643..319:79150. . 5 .5 .65:.. .65:  #02.5. ..5 3..9.514. 795073 6 :7.:.9 065.: 69.:65.9 .33: .9.9.90.9.   5 6. 6    63.3 3:3..32: .7/3069473. 6 10.5 .9.6 .9 691:  .4 73.5:...9:..55993.90.9 595.:...:36..6:964. . .::60....659: ...:  %.92: .1.5 .33 65   ':  .: #/3073. ../3.  #  3    6.0 5.690.95:5..5133".#/30 ::4/3 0.:  31  #.0 .:.33. :1.3:6 03.65:.990: 679::.. : 56.: :0.0.9.:.5 .0: .4/.05. 0.1:6.5156909.65191  4.31  % 135: 65 .9:..&79469.9&43. 56 9..: 69...9065:191 .  / 0659.::.51 ".31 9:63.6.945. 6510.9: 2565 .::..1 . .69545..51:.: . 7/30 5.:70 73:.9:9649069. 0644.&  5.969..4 .55:07/3069. .0.4511 796:65: 6 .4.: #..5 4/ :0 73.517.32: ...: 69. .4 ..65   ' 0.0::.6/7/30 69. 9.: 69.65: 65 .: . 493 461 .6:9.50 6 #  3   69  16: 56.3.9.. 65:. :.3.1 69 :..73. 036:3662. 06510.::4/3  .675. 7/30 .5 47.6 34.9.145:.6. 4.0. 05.9.9.79:09/: 9.6 79:09/ 6.0 ...513..95.5 794.5 93: 93.65   65.6461.65   '6 5:3.0.51.5.9:.: .4./3: :.9 . 634 6 361 :7..9 .0.5.65 6 :. ..69.35..779679. 9:.5. 6 069.: 4. 6 .5 06510. 989: .9.473 . :.3.65:. 5 .659: 479::65   #  3   11 56.553:6064450.9..0  .65:5 ..9. 9.51 :63. 6 1465:.:50.51.  .339.:64./3 .0: ..::4/3:57/3073. 964 79643.. . 996.. .. 10. 79031 .6 7.32: ..559 93.90.::4/3   ' :.6 .51 4.. 7...3691.3 .29: 69 :651 ::.9 964 46/ 79::9  9513 69 6.9..9963.90  ..4  73.1 / .69545.6 476: .0.4656.::9 69 7673 6 .::...9:  ... 69 . 964 7/30 :. . 7/30 .5.51 .3 .65:   ':  .969 ..5 6 . 6315 6 7/30.6.51 69193 .973. ..0::/0.:7/3069.9.:65.46 .4 .: 69.51 :1.95. 5.0:.65: "5. 9:63. .0.50/./.6: 55  # 3 476:: 59. :631 / 5 0.6 553 5 .3945.99..69 4.51 /6.0 6.506509..514.51 .   #  3   .65 0647345..9.0 '9:.559606510.51 .:9:   &473 7.11: :700 9:..50 %:63.65: .6. 93.3 5175150.39:.51  .94651 .5 1465:.6 :..31 9:63.: .5..65 6 .:561:097. . :. 563 10.65:.51.::.9 :663163365.9.6...77.4 73.69 796:65 :631 / ..69193.: 69.90...  76::/3   5 % #.: 6 93.5..06530.0.65: .65   .145:.65 .069.90.656:.9353.93 .9.1:9649069..1.95.4. . . & . 9:..3465.10.69.  90691  69 76.916   . ..3361  31  ' 79679.3.65. 6 9.796015:/.950569545.9.065:..6 /96.7069.6069.5:7.        &631341.:. %0.('*  % !6  &7.6 . .5 69 155 794::65 .:691646.:69'9.51160.51 565 6 .5.79::    . 41.5.796015:563:5.069. 65 .. 41.9 .:6.5:7.../96.799769..3:65 069. .9.: 6 .10.... 0945.5 .9664   5 :.  .: .51  .9 .51065:.03.0 633.  6..6 065.151.1 .:1656.4 793  .05636:653.9 6 . .969 795.3:65 .51.3:56.:  .6 7/30 .595.:   '..00::.3:6569 5: 9769.51 .9.33794..6794.16.9664 :50.569 .145:.51 ..963..9.65. 9.:: ::: 65 .5147.: &794 69. 5: 9769. 31 . 794. 6 . 6 10.579::5. 15..9.33 ./69/115. 5.3. 65 .1:0945.:.069.6 5694.3:6 9..9.5.17960::9. 6..6.1 &.51 6 .93 .0. .9.37696. 47.3:6541..  7.51..9 : 56..: .5. 9.9.51 .069.5.79.069.3 151. 5..6 /95 : .6. 7/30 .51 65 .9.: 963 5 :60.3  65 .65.5595.3 6 1 7960:: 9.. :.9...3 796015:563: .9796015:56919..     69.:.5:.656:.  0.51  73.49./3:1 .. .92 15..6 . 47.9.96 9 :.9.6915.5 65 .:.  . 7910.5. 69.30.9 6.9.5..  ' 10:65 5 7.. . . 151.169  .5:::  . 0.  69 /064 596:   ' .1  7950 32: .3 0.: 65. 79. / 9.3:.5::: 4. 9 .9 :/0.03.: :.49..55. 6 ..9. 6 .51 .3791004.5:::  .9:964.:676.6 .0.3 1 . 6.  : 65 79694.6../79991..9.5.65 4.4..       &631 .5: 41.:63365.5: 39.9 675 69 :/.5 :6 ./.:647.::5..95#375. 6 .65 65.963.:.151. 7693 .6..9. .10.9:656979:5.257969.379679  &79469.51..45 69.4 7:06360.19.6..69.5 251 6 79::9  ..65.765 0 5.33 / 9:.3361 1666... .6.0..0.::9.9151.3.45.6794..33.91646.33 69 5.3.145:.:3.9.::6.9.5579015.9 .:5.6069..6.:3 0. 0945.796015:6.7/30633.3  5   ..51 6.9.5:.5.910. 6 .69..1/.5169193.69.51..96.956 9.5.6 ..65   . 73519 .51..65.9.79::.:691646.69.2..65.3:6 .3356. .6.3 .:796015:55:95. 6 9769.9 .511:7.5...6.07.:.5 /5: . ..:1 / 3:: 1:.5..511:.69...9612.145..56.9...3 6 .9664   65:195.56:70.1./.7/30   %79:5.15 .77369. :9.::45.7:.6:6.65 %'..65./69.3:65 0.5.556919.00:1 .::31.33 50::.95: 69 5: 7976:::.:6.694645..9664 .9.51065:1959.9:693.3:.:6 069..309.51 10694.:569:.3..50%:63.51%.5147.3.9. .9 :065:.6 95 .5:7.3 .9.50   1: .:.0.:.30.79::.9 4.151.51....:4964.0 .:33..:6.3 :50.6 5694.9065:3.39...9.:9.30..6:6...61.0.51 :.936...9664..796015::.656:.6949#9:15.465   3:6  .98945..3:6.51.7.76::/3..1 ..51.. .6.1195. 653509.00:190:.::65.:.:  31'79679.0.51:/0.3   5./:63.9.5.60::7/3076:9.910..069.3:65069.5. 151.64.:.5:.51.3 0.4.9.. .51  .00:1 4:..:1 .51 . .0...476:1/17960::63.515.1 .65563.1 .5 56.9.06445045.39.65.:.6 .:9.46:79 ..5.3 4.65:.944/96.9.. / :91 .65 6:15:   ' .:.5::: 5:.0.&.56.6.16.:..3.33/794.065:..9769.5:..7::.660.5.79:5.151.9. 5 .65. .336 3 41.9.51 .65.:  1. 069.: 6.96 .9 10.:6976.90..5.5.1669.00:1 65.5./.9 0945.6:7 :.51 76.0..9.51..069.:6.9.9505...51:0.5.56..6.:6.....51.:. . 065:. ....065:.3796015: !616 :6.7910..0...5.969 1:.: 33 .554. 069..51 34.37910. 10.9.9.6 / 5/.6.7906:3/9.365.653.1"0.617960::.33::.::.64.6065. 7673 .5.9 9:79150..39.631064653.9: .9.5  19.5.151.76::..45. .3 609: . 069.67/305694..3 796015 :631 56.00:1..:4. : .5..7.069..79:5. :.5:.9..569155. 069.606473.79::.9.79:.5/. 69.5.3..65 856 :/3.9 :/0.7/30  .51 7910. 5350: 0 4.5 1:. / 6901 .1.37696.6 .636:556.1 / .30.9.5.656091/3150 .956#9:15.: 98:. /66:.51 .5.31 :9:765:/3.49. ./6.1  .63.50  /.5.3.3.6.5.9 5 ..9.0.065.1.67/3067565   '6:.. .3.769341..511091/.3.. 1:.0..54795.96.3. 06503:65: 9./79:5.:...:1 690 69 5350  /.4. 4.3:65 4.6 :.50:14.5. / ..:0..79:47.1.  . 9 33 / 45 . 9.65:697.959.5. .3:6 / 065011.6 . 0./3 . 4.53506. . 9 9.4 190.519. 964 . .6565560505.9 5101 56.16::65:6 4.   ' 0. 653 / 150 ..46:.556.:.1.:56../.7 .01 .91 034.515.55675069..532:/:... : 56. .0.0 0.65..3.7567/3067565 .: /. 6915..9/.5: 6 7966  /.67/30:./...37910:6319:.9 161 6 :..6 /.9. 451 :6 51:75:..34 ..4. .515.3...:65 . .:.. .60::65 5 3 . 511 .. 65.1. .5 09.9651 ..65 6 :. 4:.9:79615.:0..9./. .69556050..51.0..::65:   7960::.6 99.6 79:4 945:: 6 451 ./3 .10:6561:0.5 6.7673. 0...5:5.6313.9519: '065:06:695065:06:0.34. 9.5 /95 .796015: . 41.: 99644796795350 ...9..56927966510.. / 103. 5350  69 79::9 .1/.. . 519..9.51 .6 8./3.:.631.:/.:511.1  .510. 6 .4.:.51 9:63..9 65 .6. 565. 1:75:..511   .3:..6 .56/0.00:1.5::: .5579101451 5/9131/9555 46.945.696.9469. :0.4/..6/31..: .50 6 .5:.5.9..4.0./3 .3:: .33532369...9650: .5:5694.900..3:650..:56.65.4656.5:::.069.0 .51 1: 4.9 4.51 645 6 69.51.51 13/9.  6 .3:65.936503:656.3:6..9.5 5..513.9 ....  9.3.5.00:1  :0 .65: :.069./.:256.3.:2 6 15 ..6. 79:: .65. 6.3/.51:..069.:.3361.6 .. ..9.5:::195.9. .:.9. 7673. 069.6 ..93/:/0.95:5.965 .5/.9.. 10694 .51. 1669: 4:. 3.6: 6 : .0 /.3:.511.3.9 091/3.16.065..:.66/:9.79::5 0. 33 / .::6.0::. . 59 / .996. 065:../3 :.51 3/9.65 5 .6 .65 . 33 653 7.3:: .995 6.4 ..9.9 69545.9..2565.9.07.:.3.:69.65: 6 :. . .25 .0 :60..9664  %%8:.4491644:.069.0. ..9 56.6559..3 0651451 .. #3775 0945.0.519 . .7/30.6 1 7960:: 0 4:.9.1769.79.3 796769.9.6 :9 145.. 1:9 69 7/30.51 3 .0.3.92.9. 0647964:1 5 :09.335/.7/30.:..6.03.:21./.66.6.:.3361 .9.08.69519.6.3.6: .93 1.1 /69.:..0647.5/3109.465..5. .5.65 6 56945 .65..79::... / 675 .339.97967950..5.6 .110.16069...9.306507.67961.:.653473:.161..0945.6/:91195.67...3:65 ... 6 .:6365.:.7..9 33 6 .:964.51 6/:9 .51 9.54.. 653 . 51: 6 :. .37960:: 5. 6755::53/3.5156.: 16 56./354/96. 4769.336..6.33..16069... : 9..:  06510.9.6.6.66:4.365::/4. 60:  069.3 7960::   5 .1 .95::5.51 631 56.:9065.796015: 56.6.0.3.0169..: !69:.565 3: 9:3693/9.3396169/. 06503.3 : 56.5. . 56 .3.00:1..9.:379656501 3..6 .69915065:19. :/:9 .9664 :631 .9..3:56. 5 .7/30.3 .1.3:6 9 :..00:1 .:.5.64 469.6 7/30.51 .4:3: .9..0./3 653  7673.51064795: 4.969::65..3:65.7/301.065:.:. 910..5 :5:0 5:.5.6653.3 :5:  .33.3 33 56.9. 14609.6.796015:   '069.3.65.9.5.656.%.::9.334/61191646..7/30.16069..00:1 /. 0945.37.9. 6 .7796:641. 5. 50::.515.33.519.65::.10.69.9. 7/30 5 ...5:60655.9.9.: 90: 4:.506.9.03. .10...9.16 069.96.:65./365:.5..009.515 3.796015:.9.50 7/30.. .3:65.007./:63.: .37960::     '5..56   5.4:3: .65:9. 7673 5 ..1.4:.6 064  :.6 .61:.1.9. / 5:..69769. 50.663. 6 .: 065:.9.9:     532 6.4 ..9.. 6 .5.:56546:.#3775:79::1.3:0.67/30 5694..51 9.46:.. .92.:6506509565.3:56. . 069.519.4..  .9..3.4.30631. 9.5:..35.&.56 .6949#9:15.6:7 :.51.  !6 .6..#3519..5/.'069.::.1.'9. . . *    91646%365   .50(.& 5   5. .6936:796::65..6..:.5169:7.:6916469365  31 '9.    :0::.:70.6. 796.6 06507. : .: .: .569045.0.796.51 .3..4:.6 /3. ...1 9164   50 5..6 /3 .65.066:0.9 6 .0.1/3.0.16:56.6565 :/3 '9:.. 65:.693. .3.33 ..69:.5 .5:96459   .65 590..6569.19 .556. 65 .../93.969 964.. 3.5::/0.769.:56......:.31 .693. 5.65. /.9. 50::.1 .65 69 694 6 69:7.: 655. :0651 0.95.6./ 6510.506./:63..15.5694 6 69:7   9164 6 065:050 .6.779679.55.   ' 9:. "5. 9 90: 6 9365 493 /0. 69545.06.796.!9:..794::/351 .556./9:.6.990:69365 ..91:. "5 .: . 9 90: 6 . :  9164 .69365. 06:5 694 6 9365   ':  .65 4/9./:63.195.:.5:  ...6  &% 3 (#56*    '065:.5.65.:.:70.9./:690:1 5..365. 1.16/3.509169.6336 .7/303.:  .5 / ..0.:70. .6565.51 ..: .0.4.6 :0 936: 69.6 .519164.659.. 3.35/.9.65:.5065550 56919 .94.51 9164 ..631.  9164..9.79.346.:.35.995 .  5 ..6563:3.1. .6 93.6 .90.656:60.9  3:.4. 9..796...5...0.90:693654.33:06473:65/3... '9164.. ':0651::/0..651359565.0.6..639.0.331 0..5 0 4. .: 6 65 69 469 9365  5.:64:3.....:365.511.:.5:3.505 ./3:06551..1 .0.679:9.. .56 33..3 0.0: .:6.: /5 :..15.0.93 5.9:. .4:. / :6 0.007.3:6 5015.34.51 9164 ..:/0.6.6../3:.065:.&..6.945..1   ..:0.: 56. :29:..&..3.3.#695336: &% &7.:79:09/1/...5.3.16*     :0::  .:.3.6:.943103./..5:::69:.3. :631/. 935 :.51651   31 "9.5.3.41953. 73:65 964 7/30 :0663: 6 03195 6 6.09465 . 965..65.  (%.51:5. 69.5 . .51.69.9.3 .69.3 .990:6936: 796::65.09465 ..69.9..39..5.3 ..1   ':.563:. /:5.065:0506.67/30:...5.006915 .9.45 .506.::64. 3.5..5190.69     #. 3.9/6.659::..45.:.3.  :.9.:...3.37910...5.9.9:56.50: .596. 90: 6 936: 9164 .659:964.09465 '8..: 5069769.9..:.73:656.9.6563756:60.99:70..0731 195.9.77965.06473:693.6 :.. 65 4.1656./.9:...0..51:97.9366065.&.3..3..0 3.9.9.4  .5..93.0731..0.1.:6345 796015: &50.15965.965069 .: 69.5179:5.5 ..6/.73:656:.. ..0..616 ..33694:6.5.. 65.9.6:6066:.9.1656.   .9456/3579::5 .4654..:069. 69. .5..19013:3.9...5965..56.:6.96.. 5..59.79:5. 7/30469.07.56/51:4::196465:6/696/5 7331964:0663 :.:.16 56.27.6 9.5.4.935..    .79:5.5.655.52 5 : 1::5.65 65 ...963110:656.:69/.473:..67/30:.9.69.69.99.3.3.7.:0.9.4.5 &%  :.09465657./3:.9::..631651.0.. 796./.4.7/305.5.336%.0.9....16 '. :0. 0..3.   ' 1.51 90..65 69 . 6.6:9.5.51.9.:69:.99.67.95.51.511.15.3:.5 67565 5 94.6.65.. .9065.. 7.3.5.6 ..09465.  :5 .69 ...66/...3.51..145:.96.:.6:6..69:70 '3. .96.65.0:5.35..506. 7969 9:.:96:3.:.3.6/:9.:064. .:0663::56. 76315.5169:7   %36:9164:.190.65195 .. :55..3.3.965..59... ..5.:..514455..65:764.. 69 34. / 0647331 .93.7969.5..5.31.. :56.7.0.6795.65..51.6.:.3: 7/30.931.973:65   ' :63 :.65.. 1.656.55 69 ..   6905 ..9: !9.909056.96.14.27697.:07.656.::631 56.665.99.0731 1656.6  964 . .515.. 10.35. 6 ..    5 0..51 03.63.51 54/1 .0 / ..056....0... .513.65 5.903.09465. :0663 .5.69 .69...6. :5:/3.3:.: 69.25 . .653. 965 .: .69.  7/30469..7.63163...33 0.6:/3: 4.6 4..51.9:.9 7693:: .6.3: &0 (* 9..31 %67 6929: 565   731 .7.3....:.:4.65.9..51 79646. 6 .5 .51 :1153 79610 .990:6.0.914:...6.1...96.:4 366065.:5.51 7.51..517.547.96..:67.:.: :050:  #3775 :.8.79:.9 . 4..396:  .4:  3.. .:0663 7673.:6:006510.5:::964:.1/..6366065..  /9. 036:1 :67 .9.6533:.63.56..14609..6 796.51 .65..:      6969  ..: . 5.6./69967      631.51 55030. :4..99. 7.5:::.5.4330645631. 3.0 5..0: 6 .51469.65 69 796::65 ...69.9 596331 33 63.:.. .596.56   3.555 .. 5..6 7..9/.9.3. 3.5.: .64.79.51.945.9.15 ...5!65 .  65:.69..3.51.33 .6. 069. 1.96 .:: .69.9 /.:6 0.947. .  :55 .1306479::. 6/:9.36/.09: :.511..9.0:.3.65   ..5:7 .91 5 965. .4  ..9 47369 .. &.3 .: .3.506.67. 936: .:4 9:70.65195.9.... ...659::2653: 47.6.0  6 6.96.33 936: 967  . /.00::/3./47.65  .5:  519 .: 936: 967 0 ..6.:4.93.65. 6 .654.  7.  ....6913065:.96.51:79.51. /9.:7. 9..0 6976:. .9. 9950 69 . / .9.4. .65: .07..094656.65 . :.5 .5179:5..9..064.7.9.96.51 6 ...090:: #.9.9 79:65: ..6 8. ..694.0.::4.5 . 09465  56.659: 964 #3775 :0663: 33 /95. : .:9. 3.9..69:70.0946516:56...51366065. 60..91.:56.3.5155030. .:.3.5.6911. :5.1. 9 :..4 .3.65..3:6 90 .65.  7335 69 /. . 4. 9.56.65 .65695. 03:65 964 .4 ..7.6&0663:6/ &% .9556.5190.006911.5156.65 ..56.9 7../.3:: .1 5 .51 90.3: 7/30.6979.66.33769./6.51...53.3.65 .61:97.. 3.2:010.67.5::: 964 .4/1..65695.1 .:.: #3775 0. 7..59.('*3. 7. 73:65 6 44/9: 6 6.995069.9.0..3.0 731  .:..6 795.51. 651 .5..6.9550.65476:1.:43...11./.511.9 9.33/0.5: .956.4.094653..76::/31:..7/305.3.65 6 44/9: 6 . 65:.07.:..3.9... 7/30 :0663: 9 .:....67/30:.7:06510. .09903.:4.51694669545.9.65695...07.  65:.1 .:  /6.. ! 9:. :0663: 9 .33769.9936:/3: 33.33 &0  9..65  69 .7790.:69.:.99.913/06510.396: .6 90 9 10.51 / ..:06637673...&.96.396: ..0946533/064. ':65&795.9.:4. 3:.149.9.:.65 964 .99365 .149.7..5.5 ..5.:96:3.511.9936:/3: 69/.65..:33 14. :631 0644.9 699:70..1.5. 47.5. 565 /0.5:5. .65.6 1:0735 . 50(956.90  5.   . 856*     79. .5.: 7/30 /96.9. 65 ' 6 .5179:5...5.::1..9.::/4...5/93.:/:.&. '%699  '3.:. 6 ..: 065.:1.10.0:5.:9... '% :631 / 90.5.3:..1/..: 936: 7969.596.1/03. /6:6465. 6..3..33/95.6 ! .1.::65..1   ...6 :76:.03..95..:936:9164:79:/6519/.9:603195 ' 90:6936:91640.46.:936:9164:79:/6519/.025.!9:...:.4: /95: ....1/0.3/3 '3:65:.:.45.6.56... '% &631./6.51.&.:.99365 '! 796.6/.903.. 1...30.!9:.71'7969.65/731  31'3:..414. 51. ..61.: 9365 : .:9.3451/.51:09.6.99.: 90:1:.::.3.167.5: .:.0.3.6  &% 3  (#56*     .69.9.   :96:1.5/:10.65./651.390:.5.6 / . 6 065095 5 69 :60.6.990:69365..90: 693650..:...96: ...3356.945.6795.. 63..3.3 .5.64..:..65.&.33 065.67/30.   69 :9   :.51:.51 065.6.:7..6.9 3.65..5:.3: 697/303.5 /..5 695936511:.9763065.!9:.3/9.:63156.469 699155.9:.69065:3:. 7/30469.:/351.33  3:.6:/0.7505.069.   1.35.9.06990.!9:.6 :79. '%.0.3:.1.. . 9643.. ...69 ../3:45. &. : 56.0:4:  69 503. :0 09.5 9365 / 796. .6/...59365 :.025..71'7969. .:2 6 .33.0.6.910.99365 %36:164..:651.5 .91:.5:.:..02/..3:.99365  31' '%4.99365:/..679:97.0...4:473 /0.09..6 5.963369: :70.65.6 ..96.5 ... .51.55.6/.0:4:66.:..:.51/3:... .03.56.0: .. 56 0: . .!9:.:6916469365796/..5. 4.3 :04  .1:.465 .9..5. : ..56..9. /   519 69 065:.5 .&. . .6:...&  31  9:.39.02:. 936: 7976: : 56.9670 .365./.: 5. 631 476:.:65.51 .5 .49365.:936:1950: ..6.:14.0651../69. 69 . !:473/0.:. ..9... '%0.656065.56:56/.065:..5:.:.53.65.6 796.5 ..5098957.92: . 9.: . 090 0691 / #   !6   ...:91/5069.65:  !.   3:.656997...4:60.6569 0.51.396./365... 33 :.56.9/..465 .656.9. 79639. 936: 969 6 . .32 2:630.51: ..36 //3: /0...090:56.7731 .92.16946569.636 : /.90.65   650995 "7565  516../3165.5.:  "5 .65 16: 56. /..93.061.794. .6 .1 .51 .9.4.73. :..5.65 .9..:830.... 941 .569.50 .69007960.667.  56.6 3.656 936: .5:.51  :50 936: 51 19: .65.33 06510..3361: .:70:631/4. / .98945.: 40 . 5.:7..4794...67969794.0.65:695690.5 .9. 69 ..9:70. 9 90: 6 9365   &0 9:. 4/9: 6 .325119:6951.9 7976: /.065:.47.90:6. 5 .3. 51 69 7/30 796.69:630.:/:. :.6063315 1.99365 5.6: /3655 .636 '/196026916469365:91646..65   &0 :630.936:9. ::... 65 .  :.56   33.9. 50::.9.:..5/3 &60.33: 765 .65:69.*   '6 9. . 7/30 ./69:630.77.. 5 /:5:: 0631 56.3.33: .9.6.4.:.5: .: . 19 .:630. 065:.5: 69 0.5490. 51 69 796.6 .559.:.: 50315 .90. . 79:65 6 : :630.65   &0651   ' 7976: 6 . 9..135.:70 .6969..0.96.947.73.:706. 65:.00 05..065:.0 :630./469:7069... .3.794.90.9.6 0.:.. 090:56.. ! 9:..51 :.6:630.65:69936:7976::631/. 0. 936: 7976:  .65: 16: 56.9 ..9.4.: ..9/.51446...06330.5 .5.53.:.6 .9..3361  4.:7.: . .6569936:7976:4.9794.54.58.5 .1 5  6 .96.69545. &630.5:.0900.&..6..&.::4/3:.93..9::67/307.065:.14.51  .:630.65 0.969  .69 &0:630.969  75:.90. 790:331.65964./3697/303.... 9895.061351.9 :.656 51 0.5. 9./3 69 7/30 3. 09 : ..9.       5 5.5.51 6...6::630..1 .990:6 %365 3.56..9266776:.51.656:/:097.5 . .9..92.63.556.7733.9.56915.45.4 9365  ..: 16: .697/30.6 . .0.6/561950/.4..65 69 .2&51.50 55 .1 .516. :630.65..6565.0.97976: 510.: 9.6...65 6 .....:1. .365:.9. 73.9 ..6065:.:  .305:/69 65 4./3:..6      &%   3    (#56*     : :630.:.1964.43:690.6936:4.65:690. 5.656.50::.: 6 .: 9. 7969 9:.47.7967..6.   . 09  .65   '91 '6989. 9.:.900 7960.:69.49. 6 9164 6 9365  .1 69 065. 6.:  #  3    &0  (.. 556..::60.9 : 65 ..0659.5 003:.:...95:  69:7 .60.: 44/9: ..473:6. 090 ..:.51 .0..:1 65 .: 15.167. 44/9:7  .9 563: ..9: 6 ..&..6 4..936:..51 695.  936: 160. 065095: 160.51 .5 .6: 141 56.0.65:69 .6565133.793003:.65:7 /. 090 69.506. 6 44/9:7   .5..9.::60. 769 6 0315 964:0 .0..65: .65 '6/06509.::6.51569045. .3 .       .3 .#695336: &% &7.:..:..4113  31  5 003:... 69. 6 .:. .51 93.5193. 69545.0. 93.3.95  091  69 694 6 69:7 6 ..65  .65. . (.&/69.01936::50.... & .533.556..65615645...65..6965:6  :.691979:5.:.516.5:.5*     #..06.145:. 6 1.09.945.650.:1:4::1/0.0. !%  % !6     :.659:. 79:65 6 : 47369: 13.65  6915....5:.3 530. ..3314:..351: 333/9./.5 65: .: 6.51 0644::65 6 .3.3. :96:4:06510. 31..51 .936:45:.45.9..15..779679. &.50  #..3.656:./69   .:.: .:.9:  .9. 96:: .65: 6 936: 45:.:..:.331003:.60 .96..&5..0.:.9:.9:.. 0.75.9 796015: 69 064450. 4113 .9.0.. 00:: ..5/.90.65   6. / 1:4::15625. / 34.94: 4/9..65694.945.:..9.:56.:. ...65694.:.#673.5:.96:.69.51: 653 ./30651. :.:. . ..5./ 4.96956..51.. 4.9:..46.:65.47.45:.:69. 3465.0.9/.6.44/9:76. .65690.9.1  319.547369..517/30 065095 ......069..9 3: 69 :473 /0.:/.0.75.9.9:.5*    '%.9.5 5.0. .160..9 '&31.3. 793:03.9.: . .3465.5..: .7/30   65:191.0.  &631.6.3 .94 6 :/0..: .61./6. /96.160.93.9.:0676.95694.1697331964.9: 5..0659.956:7.:1... 56..4.9:.1 964 473645.6 256  ..00. .3:6 / :/0. 79.945 65.4:.55:6.9..65:6.956:7.7/300650955..::.:69 ..9..:. 090   .9 /0.65654.6 69. :.: .5. .03.6 9..9 .61:4::5625...65:76.50 .  69:769 160.9:56 91.0 .33 .51&.090.0.  #.45:..5 4736 .5694.9: 6 7/30 065095  ..5.:5631:...65/9.6965:6 :.9:.:::65.. 7/30 4.0 6 .9:67/30065095   315.95:6 .5.4.31:.  !%  % !6     :.51 .9 ..9:6#/3065095    :0::. (.9455.9   65:85.65.46.3  ..7.65:79:09/1/3..9.65:67/30065095.064450.:.0.51.93.. 5.1 / 3...65: 5635 7/30 5.6 .: 190.7731 #/30065095327/305. 7630 6 33 7/30 1:036:9 ..:. 9 .6 0 4.1  51.6565 .93  .3:: .: 56 93..1 :70. :.56915.694769..:.9. .51 4.65 ..6.69.:7 3&90644::65 301.:. .9:. 15.   &43.: 0 .656090..94.:/0..: :0 4.65 /.94:7/305. 59.5: .0.. .9:.51.:. 36..37/30...45. .150.0./351:6..: 4.5. .3 .5: . 5 .5:.6.656.:..60.96.76:.5 #..3465.9: 6 7/30 5.4.65 :6.3.0.7/306031/.7/305.65:989503:903/3..4/.&&.5106510. ..51 065095 5856...06509560.69545..51.96:3..6510.:315'.65:.960071653/79:65:6. 5694.:.56.9 3/3: &6.5..:.51.  8.93.65:9.1.331/69969: 44/9:6.:.....18. 2:11. 3.6.67/3090691:   519 %7/30 0.6:060.9 7630: .: 6 60: .3:631..&.3: ..00::  .97/3050.9 ../3 69:  .5..6 7961 5694.51./3 3.3.9 . !6    7/30 60.:9 7/30 .03.51 .00::..3 79694.6  69.50: 6 69545.065:.6.:65.945..  (.90691:79.5110:65:.3..7/305..4/.:67/30609:1655 79:.4.9.51 69545.51 519:.6.65  7/30 065:3..9:..9:79611/3.55. 57.: .95 59 .65 65 .51.51* 5:9 6755:: 6 5694.     07.51 796019: 5 03.0.39.0.69.9.55.0. 9.1 .65.350.779679.6 ..60.51 4736: .51 . .65: . 5 6.5.950 .. .9 4.9.50 9769..65:.6959891/.51.6 ...960... 6 64 .. 7630 6 .9 65:.3 796945.5 67769. . . .515.3  .96331 069769.9..9: 6 . .65 .9:.: 515.337/3060.65: .51 0647. 69545.3  9.514736:   5 59. .65:.6915 .. 7673 . ..5.51 . . 5.3 50.45.3 .: 6.65: 4:.9 . 7/30  065::.51: 6 7/30 609: 5 06550.5. .945 .31:036:9:6.6 1.5.:  3.00::/3 ..::50614609..03:5./3 .9 60.:7/30:9.:.5.950 6 69545.33 ..: . / .::.50.6: .9:   ': 795073 : .3:./3.6 .1 . .651 69 065..5:7.9.950.41 .5: 0645 5.6 . 6.: 4:.51 1. / 33.36 6 .: . /. .6. 4.. 5 .5 .1 0.: .5 .465 ..56941 :6 . /.07.5 6 1. 6 .6 09.: 7.5.53 .:/5:95 ../3.91 ..: 33 .9.69545..510. 33.0 ..5 5.9 79:70.559 9.9:765:/3 9.51 55/. .:  / ..51  .4 . 7/30 631 .9:6./3 ..3 ::: 065965. .:65. 1 .9..5..   #  &%   0     (#.90.65:. 7673 .0.:6336:    .5.65 65 4.5/.5:9:765:...6.. 6 .5*  .. 9.906519:.5/.6 . 94.9: 6 7/30 065095 .5:1:91/.9:646. 90651 9:.56941 7/30 .6 5694.. # &% 0    (#.6..65: . & 549.9: 67/30065095  31  5 .90.65654.. .6.65694.7673...5*     .7673  . 69545. 9 5.: 9.65   ': 9:10..9  17364. 69545.3 793 .  !.5:. 4... 7/30 1:036:9 ..3 :09..9 : . 064465 3.0 .9: ..9. . 4..915 43.. 9:70.51 5..65655.3 :09... 6315 ..65 9065: .. :09. .3350 5694..51 6.9:  2: 5694...6 :.65.65. 5.65   ' .5: .332565..9.993.51/.65 ...5:. .65: .65: 79:. :::65:6 .5.0.     :.::1 5694.1 . .9 6: 6 659::  .9:.. 4./5.9:  :0 .   '9. &0906.6 .91: 69 ..91: 0.9 .33133.9./3 . 1./3 :.3 :09.51.65 .3 69 03.0.: 33 ..5.4511*    945.6:0.:65 6 .9 06515.330.. 796:0..: .6.3 5694.7795:65  .3 13/9..67969.51 4736: 964 :5 69 135 06515.90696. :.: 4...6: 93.00:: 5031 17364.945.:65.: 7/30 60.(%  !6  ...65: 6 . .6.56.9 796/..0256311 34.77961655 *.3 &.:.:4.6 9.5.0.5.6 . %   !6    5.6560.:.2 6 5. / :/0.99:.: . .6 .5276:.6560945.65.3: 0069..69545. 4..3 5.3 #9679.5895.65   ".51 0. 5. 61 (%   !6    .: 79:.1 65 /9.4 / 9.6 5694.5 .9.6 .65.0.9 60.1 69 51:.5:7969.06503:656.51 796:0. &79469.... 7/30  &0  (0*  /1   ".9     9.516..: .51 56.525.3 4.51 0.13.:0.3: .3.95. 1.51 ..0 0699:765150  036:1 1669 . .:0796565045.906::.906::.. . ..5.90696 .:66969545.9:5616/.6.3.:9....  31.33133.4..6..967/30065095:/0.6. . .3:6 .:..3.5.4..6..967/30065095.  4.11 .7/305. . 33..514/1.9:.  71.3 69 5190.9 7/30 60 69 .: .6.94796796933..7673 ::0 7659065. .51036:.4. / .3..:.:.2551 .6.91 69915 065:19.335. .::60.5.5.6 .37567673.:.50 .:59.517910.5 79:65: .517976:: .9.5 .:9 :/0.9.3:669545.951.: .51 79679.:   56.4:609./365.0891  190.: 4.: 79769.9669.. 99: .51 79679. .906: :441.43 93.9 103.9 : 6 769:  5350: 69 93.5.6.33/9.6.59045.%7/306.1 964 .:3 '6.::.510.65:7:  9:3.969 .653.65 69 . 69545.111 / 0647. 6 ...93  .6 :776:13 695..6:9069633.1.03...51.9 5:.9:3.:69.13 ..5.51:6979679..3 /6949#9:15.:..7/30. #3775:   3.::.5 5 .6.: 9991 ..1. 069.:.6.0.65676:..: .. : 1:036:96.9 7.65 :..133.659 ./.906::79769. 8:..: ..991.3.4.5.6 9:76515.5.6.945..:.65... 9..65 6 . ...30656409069    /3 ...3: ./:1695. .4.0650955. 6965 1:8:. .669.5/.5.603:9.923.9.5.669..3906515 ::60.99.4:3:..5.65669.5*    91646::60.5..3:4736:   999.9.:.:76:.603:9.677.6:. 6.65    6:. # &% 0  (#.33 .:.  .5.9   31&700.3.55031. 559..9.0.0. 0.50.7976::6.65:65.5 .516.796:097.33 69545. .5.9.92   33.. 796:6 5 .::60.:.::3.5 695:.5.56..9 694: 6 4.9.65:  ...51.90:6. .65:   : 9. 609: ..:5..65.65 0 33 9:3. 9..691656..69915065:19.51 93: 6955 06509.: 9..691:97. 93. 90:6.3. . :. 9.00691.: 90: .03.514903.:.6929:.05:..94.06 .9 0 565: 519 7..51 4736: 964 :.3.9!6  :   1.. .3: .33/6/:91   .51 473645. 6951509.: 6979::65.656 14609..5:0.6 :.50 4730.65..4.65.:.: .669..7/30:0. 9.92:  1465:..50.31.92  .92:5.:6.91:69. 65:..33.4769./69 ..35169:1.69::09..145:.69545...5 6 . 6929:   5:.5:. 3 &90 3..4..326..1 ..9.6 :.:.65 .. '::.3:6:.669545./334.677. .:  .1 793    /:..:3 8.9.3159:79150."919!6 07961:135:69.:     .65:  4.6567/30:90 & 469.51..65::0.336..3:09.:90:.5 :.65 .65. ..9:.79:9.::4/3 ...9 6 ..939& 0903.516. /3... .50 6 650951 69545..51 :.91: ..3 6929:5.:: .9:.. 510651.. :0. %3. .65:6473645..513..5.969  .  &50.945.9/.94:.&:.65.6 :09 0650::65: 964 .9 .3:/1:65695:.4.510651.5763. 5651 79.51  69545.4/.   ..96 .'59.44736:::60.94:.69.47369:.65:669545.0903./:091.93.779679.516. ..1..6929:0..65:6 473645.::0.0. 5 .51:..6.96.:.:69.5193....6.65:  . 4736: 4. :0./.5 .9. 473645..91/ 3...9 :..5173..1:669545./69 565:4 5 79.37..50:  .:   .65: /.65 .51:.51 0651.65:6473645.51 0651..96 .3935..55.9606330.  69  .9.94736:9:.65: 6 473645.3.94:...145:.6: 0 .579./69..556.9606473:65/3..9 6951/3.  .:.00.91355.. .65 .556.::60..55   5 69545.69 .50: 69 .9 ..6. 479645..93:3.5 59:631 9. 50315.3&09..9. 1 /3.510651.941.94:. 1:7.31 .:6..65.7.:.9.69 .9 / .65: 56..94:.9..51  9 796793513.5./9991.65 &&&  69..677. 6 3:3.6 .765:47361/.51.. 6 .659::69.9 565: 69 .454498945...6.33635.9 7.65 69 56.145:..0.:.945. 69545..0.9./69.9 47369:   ' 795073 /51 3.. 7960:: 6 06330.69..9 56.. . : . 51.49:6. 69545.4.6929:5.79:5.3. ./693 5&60./. /..9: 93:  .1769 ..65:.:: &/0.6..69545.9:...#/30&0.5.3:.96:.517.79..473645.945.5::5.:.. 3:3. 4736:5.650369. 90:6.0.6 .6695...326. 69545.6:.9..001 . 79.6&3.65:.9:69.9692 :.6 79::9 .%..56.4736:.677.5.69  .669545.9 14.65  .:  32 6929: 5 .516.03:904. .45.51: :5679611519&0  %36.51%3.. .:.779679.92: .4769.%3:.6 . :0. 5 763.5.* . 5:.. .96:  (....96 ..3.0.51 69545. .  50315 .9.945."9.3 :/1:65 69 5:. 5 .65: 6 473645.6620.5. 69545.65 0.51 0651. 1:7.94: . : 5 &7.: './:..:::..5*     #.9 :0663: 195 93..50:.51 6.69. 493 90:1 .3216.65 .5:(.65: '9:1.5.0..3:69.:..96951/3.50(#.:90 69 ...:.0.30.65:.3.9:..: 5.9.5 1.946.69919::69.69.36&/.95:. 9.154.  964 . .33 .3 .9: .51:/:85.6 16:6.515.659:7/30:0663.6.4/9  796./3 .96331 069769.5 .6 7.6315 6 . . 79679 .9269 . 03. 5.65.1 9..9:.5/.3: &% !6   5.5.677.51 7.1 .51.4:3: .  .:6.6566951/.5 .94.514736:.6.65:195 09.65.7976:6:0950..::.9 :.9..:.:  .::.1 5 :.31   ' 151 .6*  ..92 /.3356.. 971.5.903.::.695.09:.651 .606510.66921:7.6919:.0.:15./:5.7910..51 065. 065:.05..6.9.6 69.396513.::4/3 ..9065640/5. 69545.331 53.95.41 .3.51 75. .3::: :0. .9 :0663 1. 03 .33 9:63 .9...92 . ..7..31 &631....25 69 ::5...51'  .5.91.92 .51 5.57../691:7..:..65:6473645..:65:  :631 56.1 .:. . 065:.6 79694  519.1./151.0.5.5:..92 ....5.51 0651../691:7.9 .93.:.335..551:.3.:4.51.677.1/..1 . .67..99.515.556./:50 964  692 0 ..  9.. .:5&7.94:. . 79507.1:7.9..5  5  4.6569979:5.15..9 .6.4.517976::..331356..969:69 4.06509.::..:56.0.:69.:.9:.6.:69545.5031:. ..103.79:5.33065640 9.97/30:0663.6569.65   . 9.514736: ..94: .55 69 .3190:6... .65..969 :63156.5:.65/731  31#..39..969 .6.4769...::.3693.65647369:.50:6.9.09:.5.0./:50: 195 09.: .5.9 065. ...::4/3....691 :.919::69.65 64736:.913::6.: . .69.:.::60.659: 6.65:9..0.5.9.7..0. 3... 06509..15.65.55 .:..0..6 :.9:3.65: 6 473645.::4/3:.51.3   &. ./3: .0..95. .69545.510651.50: '03. 565. 6  69 .55  0.5. .65.9065:..65679:65: 5 56.4736: 1656.7964....065:. .9.09: 1.:794:: 56.951156.51 .956 50.07..06:35::.4.65.97.6692165/.6 :.5065.677.:2 .565:  #&' . 4.:151/3.0904:.::.51935.90650955.92   %.4/99...69./575.659:9:3. 69.331 . :6. 919:: 0650951 . 0699:76515 7961   #.6315 6 03.659: 16 56.39 6 7/30 . 9.50: 69 0 .::: 5 :9. 1:7. .... ..: .3 7/30 :0663: 195 . . .9 47369: . ::5.33  .  946. 09.3 6 .:1 5.9.65.6036.511.794::/30.44.3 47345.51 .65 6 695   . 971.: . . .  &  /.3 ::: :0 .51 4.:9: 5..51 :..33 51796/.3:6 9.9.9  ..0..65.: .6 33 ..6/5.0./3.5 3.9.:: .. 659:9956...677. 5.. 6 . 3 &90 644::65 651 .:141065.. 69545.: .90:6.07.7.  :50.3 .:.77. 4.5.9:0663:19593.9 064465 069: 6 .:69..5.5 69. 565..9:06631.65 .92 ..6 . 7910.35.6.4 3.:.65 .5 .9. 919:: 6 9.033 .65.0.5156.9. 5.07.: 5:50...5 ./:5.3169../:50 530.. /:.1.9.75.99.: 56919. :1 / ..6 1:09/ .:: 796. 7. 964..506..79679.0.0655..9.. 69 . 6 06510.:.6 7..94 :.50 . :90 69 ../3 9:3.1.::4/3 7.:.50:   %.: 56.9:.96335   6969 .:.51 .65 .69.:/:.4:3:..55 .3: 69 .3.6 7.9  .  .1/.6. .1 7.: 6 10..4.51 5 .63156 03. 179. 9 9:765:/3   ..65 6 :.::: .659: .15.99.4:3:6.31 ..65  69 0 . 903.:..90:: .::: 251:69631. /651:63.:.79679.5..99.4.51.619..69. 5665..50: .61...36. : 69. .56.& ..&69 5 . 65.9 065:.. 50::.6 / 69:.5/.3&903.4769.0. .1./.31 90: 6 . 796/./3  . 6 03.5:.5193.::: 9:3.. 0631 .9 :. 9 3 :6.5.9.5*    '!65.05.6  &% !6   5.5:9.:5651/. . .5 964 .:  ..4:.9 .95.:   : ..0..5.. 31. .1 7/30 :90:  .9./3 69 .331 / .65 .93 1:97. .33 .4 3. :.969 /4. 6929: '9..1 9.677. 93:.50 (#.92: / 69545.65: 690 . 6 .4   93  .: .:65.1 69:04. 6959.5.9.0.&. 6 .  65:.5140903.906445. .::9. 5 .33 065.0../3 50::.949.5:..69:646/0.33 9.  31 !69:..: .65:4:. 469... :7.3 1:09/1 .1:0676..3796/.51 .65 / 73.3 :.03 9..517/303.. .:.65.9 :/0. &.0 796:65: 6 .9..0.5964.. 6 ./96.6 9.36/3.7730.9:70.3.  65:.95065.36/3...0.03    ' 9..3.15.65.6065./651. :60...4.6 065.9 ..&.6.. : 56.7.65.9.51006599. 76307696..003.33:3.5:.03 65 &60.963.1/. .5..65./73.96.1.65.56.556.9.03.0:15../:63.9 47345.0 ..:56.3: 06469..5 93. .. 76307696.9.65:0.3.693..7696...0.59.511 5 . &60.6534.96.05 .065:.333.60.164.4..01/065.: 5.6:. 065. 5...0...9.769:6... 6 . #" ./:/0.0.1.795.5.1 .3.963./.51 065../:63..34. 56#. .6:.51 7.16:56.06445.: 93.65::56.65:.51989:..6065.7/309.33:093....65  0 .65.336%....: 7630 769 / 5.: .4 519 ..515635 .3 644::65 9.069.0..5156. .:.47.0.4.95065. .6 .&.3 :.  65:.47.  /.9...0./3:1/./:63.. 9164 .:    :.3 5.945.:6. /5 9.6/9.93.5:: .:.501 .0..3796/.&.5.5::6:0769    '065:.:4.9.79679 90: / ..6 :09 .:.91:. #9:15.9.:   ' 0.1.0.9:63.. .4 .0.59...47.: .3:.03....9 / .:.:9:.514469..90.9. :.:......4/:.33...5 .65: 9.9.. 6919 09...51:56..3.0.9/ /.9..0.9:0. 9.67/30:/0..679679.65. /69565: 06330.65: 69:63. 51519. 989  69 .: 69250651.3.9 :6 479::1 .9:/0.9730...76307696./..51:43.0.9.33519:63..6 .9./69.65:6 " !6  9.:70.5136026.3147345.6 .. 06445.  '93.03..: 036:1:67 . 1465:. 4.0.3 '.9:/0.92:.956.493065..:..65.1519./0. .513. 31 . 0904:.&.:5.33.9/5.9.902165 65 ./56..7..6.55  :. 065.:63.0.9:.9 365.5140903.:   '0.: 7950 4..65:/.0.956. .76307696./69. 064465 661  '969 :0065.6915.96 79::37961:  9. &../69.5  69 .361 065.0.9..7/305..9065.:.9.9.0. 50::.9:. .. 0. 03. 63..:/.556. 3. 9.9479::1./69 065. 9651 .9.514469./:.50: 4.50.3:/0.6.0..#" 0...:653. . 7/30 5.5.: 4:..9: : ..:   '6 631 6.9. .0.: .3. ..659:79.4 .:..55550: 50  #" &% 793 (.9:76515.   ..9.065.6:/6915..657..4.79:5.659:03.03.51901.1 9 *    #..0.4:.9.. 635.:79..51.5193...930.3160.7630769....9.0.95.:..18.::..65..65:631//.9:7..:  '659506 . #.6./3:1 065:.03.991 /0. ..5.1:. :.31 065... 9.45../064731./..65: ..69063..7. 196. : .5661.65 6 .6565.3.9: 9:79150631:. 51....   '69.:.65.79:5.514:. 6/3.9..9:5 964 .0.065:. .3 795073 .9.9..5. 9 .9 473645.95065:065.3..6 .9. .03.9.65:   &0 8...6569 ..6 .:.:65.4:.65 69 33.9 5. 9:76515..334.: .9 9...75 . 9 1:4::1 . 8:.3   66.55::70.94:6:.065:19.: .4..:.: ....036..5193.345.657.3361 .. .4 . .93. / .  9.:: .9.:0..0. ::/0.7/307630 07..945.0./3 .6 065..6929:96403. 469.9.51 93.6.65 7.9:03.:   5.:56.:./.553 .  :0 519.911 ....03.65: 79646.56933/379:65 69  9.9.0.338.9 :7.. :.9 !6.0 5.   &50 .33:.3169..6  7 .39.556.3. .6 9.945.9.9 .9.65./33:3.6 :7.9. 93.9. 79.91.5:.51 .3313 69 /:5:: 36:::  ...3: :.6 /.5 ./6961 51:50.25: 0.45.:5065:19.8.97966.9:76515.0.47..:.:96.31 . 7630 7696 .: 50.513./365..5199.3 1:4::.6/.79.979:5.03..531964.5:.65 6 . .6 065.5.: .5. .:.653.9./9.0.90:6./:..5..:/59.:  /..69545.7..18.01:09545..3.&. .0..90.0.::.:59/5.::1 .5.9/..65   '635..5.65:.33/7. 1335 &.56   &09.0 9.:5.65: . 3.3:: 76.: /.917969. .9: ::60..9.6 065.. .:.9.65....34.:.506.. 6956.. 565.:0 76:. /.5:.65  50    !%   &%  (#. 9:9...6..3796:65 ..3 .31065:19. 0.. 065.5065.5:.5. 47.3: .. 6 .5*     "53 :3..: 9.0.5 7.5166169196:60.0.47.3:6 9. : 7696.9..65 6 065..0.5/.6065. 476:.169.1 5. 065:... 6/3.556. /.6:09.1 5.3:..90:6.50   &%   .. 6 ..065.: 5 6919 ...6565.0.9.:. !6 3.5 #3.&.945.656 %..79:776::.6.7.0.0.: 6 :695 769 : .96...: .3..9..9 6 5.90: 67967.6...9.:.65.9.0.6 6/3..:56.0..33:: .9:.3..9..   5.65 6 ::5.5 3..6539.9.513.95 .65.'65. .57696.::69..3:.69. 631 63. .4.9...6:.656.3 6919   ' 76306796.9.0.47.         5 .*    &50.9.50 ( 516.065.565.: .956.4/9305::.0. 945.0.9.5033.: .656:.:16:56.::41.563..03.5.65694610.::..95.556.5.4/9305::  50 .0.4.50.6/.:0.47.:.695.0.9065. 3..50103./5621    5.5:..565.:... 3..:0...0..::14.:.33/:.4.63.65:694610.65: .1 .:.9.:.33/57.:..5033..51.40.556./5621 !9.. .03.3:: 9.59.656.565.945.:.556.50...47..5. 33.69.505 . 0631.515.9 59.9.:95.501.9.51.90:6..::15.5.505../.9..565.6.47.6.3.: '::/0.7976:6.945.30636 79646.76307696. .3.517976: :0.7673.9    5:69.653/57.1.3.03.3..:/.:. :4:.64.47..:..:103.31.565.03.76307696..33 .945.   5.6.56. ::695.: : ..9...9 964 905  .9:70.:5.:0631..065..347.:..69545.0.945. 7.0.0....9. .6.:565.33679.36/3..:.03..695.659: 065.779655.631.4. .5.6... 79. 6..4/9305:69 :.6 565 .5 9 &%(*      55.33065..6 .9 .95:.96.3 .. 9:76515.6319:56.9. /5.65 ...15.945.91 5..69..50.  "76:..007....9:70.6/.5 5065:.6:.:.0.5.3 56065..:.65.773.31.50:  6969 .969. &09.5  7960::5  955 69 ..65 :0.(5*6.47. 690/3 951 6 505.   &%     &7..65   3.969545.:5 . 14.1/..69/5.: '.6959.96../4./65:0: 56.*    '5.     5.3 :1: .0..50(#9:4.69.3.: :695.5.:.:5.:0.   30. .505.&630.5179679.51.654:.:56315.7.50.89.3..1:.0./3 69 5690.:.5./3 6/3...945.../3.9.  796019  135: .65    .. .61.51 6/:9 .99:..35:.:6.51 .55  5.99:..46.35. 16 .5  1.565: 4:.5  69 5:.51195.51 1..25 .0:.500:1#9:65    &..996.65%..: 0 ...5 .. .:.5. ..5 609 69 : 0647..61.. .4 6 4. .. 5.61...  ..5%.3.65:.:6.1 .516/:9.5 .65.46.5 6 .5  1. 69...996.50.515:....5:  .464.65:9.. 796019  135: .51 .565:4:.3 065:195.5 .:.. .5.559. 31.3 5:.560969:0647.00:1 9 6/.61..35.65:'96 .: 165 .61...3 4..25. .55  5. 7673 3.16.99:..3... 9.: 6 ...5"09:. ...:33.655.150 .:..5156919.5  69 5:.:6963.: 0 ..!6  50.99:.5169519:...51195..: ..:6#9:6599:.96 3.65 9:79150.5:.:.51.75..51 %7/300...0:.51#9615 #5.51 1.99:.91.55 .00691.. 693...4..610. 569045. ..50:6530.506951950.:6.9/569. .7765.:150.565: /.51  4:..9 65 33/79611694.5:35.3:6/5..69 0659/.99:.:...7657...71..9.:    '79:65.9690.33/06510..4   4:.579:655:/. 3.14:..5694:..3. .9 799.2: 4.   .955:  5694.:.65 69 064450.:0.79:50 6..96956..45..99:.3.94.951./56941.1/..5   9 6..3.33. .1./56941...16965...510 ..99:.4 ./:1.694.691.51.:563...715.35:.2565.61.:65 69 .3./56941.61.99:...51 5.6/.1/.99.107.5:.9.179:65   4:...655.6064450.69.1/0645:05.510647.9.5517515.4.51. .9 .1..9..661 / 4 6 . 9.. 6 ./36:65060   4:.:/54.:::..3  694.0.9 4:.79:65.5:...653..9.65 4:../.5.6.5..4.1695190:.5.3   '.5.6 .:90:6..79:65..51..9./56941.661/:.:..560:.99:.331 9.654:.4:.79:506:065:36.51.069../569415.:.46:./.31.1   '79:65..51 519:..656.4:.1 1.5.3 5:. ./5...556...3.. / :65 .99:. 2565 .5.3136745. .51519:.5:.0..5.3. ..9694::59.16 3.3765 9. 5169.544/96:441.3 565.6945:.4369/:065:3  69 / :.:3..3 69 5.7765.0091.1 5..43 69.1 /0659 .565964:441..65..906:5/469/...1 .95..65.5410..3160.99. 13 .9 .69  79:. 95:95.519:.996.79:65.656.5..:519:..96:9.9.6 ./ 6/. :/0. .:5... 19 6 73..9 5037.65 6 .69545.9: 4:...36.56.516...1/5   '79:65.694. 0644501 69...6065:3 065..5 .79:65.4 . 796:65: 6 &0.::9.:.: .:/0.03  6 .473.94:  . 6 ..65 .3:6/.3:6/56941....51 5 ...14::/35150  #673  ..:.6509:3. :/0.694.5..7960::.2   '.5:91:648:. 09465..33/.5..335.064450. 69.0.1 56. 7630 5:..654:.51066::.:569416:09.35:./16559.996.51...559.. 251 6 5694....9...996.6 / . .6 / 5 / 3.69 90.5.:.54.65:9..3350 .:631..515.16:56..69  5 63 69 5 7.0. 569045.65:69635..949645625.6 73....650 4....9..6065:3 .51 7950.51:   ' 9...../56941.::. 5.65 .515.6/8:.45. 56.69 69 037..51175165./56941.65:9.6065:369.5. 79:65 519 5:.93 519:... : 9891 ...5:.:16:56.99:.51 065..33/.65 59...69150 .65..:56. 7630 4...65 4.. 3..6.5.:21... 609: .4*     73. . .6. .609...4. 4:..91:64 :.11.19.. 16  ..6 / 56941 0.5.3.:0.45.:56. ..93 25653..14:. : 064795:65 ..473. ..99:. 69494:.993.6:7..5  . 4  ..5.65  .2: :0 510.99:.4   5.:.006473:1   4:.10.69 .. 9.9:765:/3.79:65. ..0.50:6 .4./56941..:. : :6./.30904:. .5:35.. 56.3.99: .3...5..6/569416.0.6..515:9.:...9.1.955.65 9.6.553 5694.:   4:.1950:.5669.3:673.0.9...1 635. .5. &% /   5.9. 7960:: 9.51  .61.65 .  :....94.3795073 .4 69:.5 6 .9 :.56:.5..4195..:4.3.9:6155::. 69 4.50(#99.:4.0..79:65519655:..65   31(*.5:4::65 6 4.65:.9.65 9891 33 50::. 7..65 .:0651  &50 .661./:.9.93.. :.  65:.:6:0796:65579...56:9.515 63...913::6.5:35./.510./56941.65   9..31.39.79:506065:3 6..51:631/   .3 ..065:.:.6.. 06993.5156. .:5.649397.651.:79611.9..5..5..69.:.:.65 5.951.. 6/3..::.9 4:.. 6965  ..65 .79:65519 5:. .65 65 .14:.6 ::70..5... 5.5.515#673 5.:79666...91.:..3.5...945.331 .5/. 96. 5...359:79150 4:.5:4::656 4.:4769.1 / .39..9: 93.5.4507.65 065:195 ..: .3.1...  4.5.9  #673 :79. 3 ..3.. ... 964 .3:..633:.9/065:191..5/:4/6.5560647.655 ..6/..6 9. 3.:73.93... ...00:16979:65..6065:3065.:.473. 5.691/.694. & . : .691 65  :631 / 5.6990.169.:70..65  9.09465.473..00:1 . .7630 .9 :6 5..5:.4..659.65 650 5 .3065:3 7/306979.51 #673 .9.656.65.5:35.69..065:3.#"3.. .9  #673 ..0. 3.00:1.9796/.996.959.5*     .51.5.6 . 9 . &% /   91 (#.511.796:0.69 065:36.    .691/4.65.6 .0631. ...: .: 9.19: ./376::/3 .65  31  5 #673  "9.9 .:1:.5517515.0606..6 / 79:5.03 65:.9 065.065:3.650631/796114.1 :631 / 79:5.00:1 4:3  69 / ..1:50.6956: 5.33:.:.:::.33/4.631.5560647..9965 95:15.1/ .65 6.33.5 / 065:191 .969  066::. 195 :0 5:. 065:3 6 : 060 69 . #" 3.651   .10 ..35:.9.90  :. 65:.95 6 .9.6979:65..5511.5 .14.3...6  . .5  066: .3 6 .  101: 56.5 :35.....659.506065:3 6 51979. /555..33::70.. 6 ..:335.473....51 4.:269065:3:61:9169.00:1 ..5 .069.69569/.:54. 69 ...5..51517515..95.9..517950.. (. .:65. 065..656&0.56  &%  .9 065:3569..:69....065.31. 5:.065:3519&0.:..65  .1 *     .5  7947.9. .7630 69..06 . 4:.:6336:     ('* 3..:6.13 .: 5#673 .53.9 .3:9.:.65.. 065:3 ....513.:9.00:1:/..9:.6.510. 69 517515...6 94.65 .969 .465..6/569416.9::1.: 31 .51  .9 0.5..:631/0647.6065:3 :0.9 0..65 :631 / .65 6 ..050:.:  31'4.  ..3 6 65 .5:1 964 65 6 631 493 / 5 .:98:.65 . 765796797./..9 .6775 ./3.5. . .5...9.  : 56.9 0631 56. 065:.3.656.3.513. &%  .:6..3519658:.0      ' 0647...6 / 79611 69 4 ..065:..:65..3 .5. 3..:0.336%.69 .95.00:1: 9.5 517515.5.65:7/. 511.63:07.51 :..9:5.569045..3795073   1./:. ....00:1.61.5535694.9:.5.:.553:: 90.51:776:15175150  . : .*     : . 360.5.. .7735 &0.33   !6 7630 . 609  : 5065:331 065::65 .69.1 .6 .37630.6.51065.159:765:.65.51635.1.00:1 . 4:3 6 :765. .. &. :3.069 &% 793  51 ( 516.5 0644.: 065::65 .3.159:765:.6969..514.0.65  69  ..65.. ..9. .../3/141. 0647.: 519 &0.6 795...14.93 .9. : 6 069065 / .. 4 964 93 . . .: 631 3..559 9/ .3:79:65.69.65 /.: ': .6 . .1 . 69.69..7733.14... 06473:69 1:036:9 6 50945.51 517515.4507.93 .96369.4. 094   . 609 69 7976:: 6 ..7733.:/531.51   6 9.56:3 93.4.56::. 0651.14::/35150 065:195.. 69191 .996.51 635. 3.: 69 065::65:   ' 9.6 4 11 56. .5 . 6 ..5 ...9 4. 65:.:.3796019:650:.5.5 .5.7733..35:.6 ..... 4. ..: 65 ..65 1656....9.03  6 ...69 .5...1 .256 . 56.5. 065:3 69 .5. .45.. 4.691156.1 .51065.9.6 065:: : 3.6 .: 56. 63.69 .5. . ..96369.. #" 3.691156.321 .6 79031 .:679.56:3  93 .61.79. 4.3...609  313.655/.569045..8:. 569045..65/.....  #673 51. 360.5..335 .3 7630 .51 635.69.65:/5:9769. .. .: ..7733...50 6 .90      9065::65:4.: /.3:  56.:: .30.69 .33 .7733. :64. 569045. 4.51 4./3 / 141 .32 .4. ..65.9: : .55.3 98945.:::.7733.39.6 . 065:.7733.5 &%  .9 :.0.. .:679.65.6.55 .968:. 3.773.14::/35150   .9.6 4   5 .65  .34. 065::65 6 ..65.00:1  #673 .7733. 65:..5. .5 6915.:765. : 065:. ..5.5.65    .65.69   .68:..:.   ':  .9 .51 56. .. .9:..93:6.00:1 5 :765. .065:..9 5.: .5 '4. .3:79:65. 7733.14::/35150     :0::  369 069..159:765:.3:15.0.:065::65:.5. .6.190.:5:9769..9:959964 .61 .094 5..5.065::65:   31  79676: ..94 0. 5.4.6.65:9769. .5.5.0.00:1: 16.56:34. ..9/:9.:.14::65 6 ..9 5:.5.65  .:21 : 794::65 /69 5..69.. ..5 5 150 .5:45..95 4   ' &794 69.65:. 06.90  .91.51 .51.0644::656./..9:65.65:/5:9769.5:9769.7630 69 ....9:.9.::70.0691/&0.7733.0.45. 9.65 .65.1.51  69.3 065::65:.:765.: '5:9769.14::/35150 5#673 51.5519.1/.5195..  9 .: 9/..9 .9.5 6.7733.9.14::/3   5 #673   0.55.9636...7630. '956.9:5.51065.3:15.5:9:.9141635.68:. 5:9: 65::65:4..379::9769.65 5 . . 690...606..14..00:1  519 0:.9:5.5 609  .: :631 .7141.. 86: .:31.00:1.5 &%   .69.0. .7630 '1156.0.6. .::765..969 . 069.5515350964..& ..036.9 931 ...956.9.9: 56.:31.99.56:.7733. 14::65 79679   ' 5.7733.. :61 .9 .: 16.51 .71 065::65   51 :0 ..5.: 906911 65 16 .00:1. .517/3035..4    69 /0.: .:0069964.9 .00:1.0.9 .6/3.  7.14.: .51 .: 065:.3697:06360.565:65.33323661 631.9 .: .: 6.65   00:1.00:1.96906901065::65:519&0.51   ...65  #673 5156 &% /   51  (336:336*     :0::.3:.: . 065::65 . .3 .7 6 065::65 . 6.1 5 ..5 65: 3.969:631/. 065::65: 1:09/1 ..::.9.1 5635. 909950 6 .16.  .9 .3..47. ..3 065::65: .9 94511 .5 .:/0.:.:6951.3. .:.6 4..14.6251:65635.65631/1..5 .9  690  6350  .058: .6 369 069.9.14.25 5 9.9.1.9455 . .51796.515.9.. /5 93 5   ': ..969 06901 065::65: .33 41..: 5 1..630.:6..0.9.:3:6. : 7915.51 ./5 :47...33 796/.: .9 .579679 ..: 56.6 19.6.165: . .51 5.51 73..57633..65 9.0. 65 .3. 5/9155:3. / . :631 .955: 0. 256316.3 5:.65 .50 . 989: 79::.41.5.9.916: .00:1.6963.45/. 7630  .94 0.65  0 .46:79    691 6 0.. 5 7..9..3.7  6 &0...7/30 :1: .: 5 519 06907:0.65   33:.. 79610...65:.51 .3353 6753.9.7733.51 06510.10.9 1. 069.0::.0.3 0.69.61: :0 .. . 103.5 :43.945.  65:..:.6 065::5 0631. ..9 93 / .. 5 .65 5 :7.5:456 5./5. 694 7.65   .7790... .6 251: 6 5635.5.31 7630 .2   .51 1./3.:47.945.41469.3:65..4&0. 103..: 635.9 69 06901 065::65:   31  '9 ..4 06901 .. 03:65..35/.61.4 /651 .5.6 9... 0655.4   1656.9.5.03.:2 69 .. 6 0:.6 3.47. ...93 5 0.91 19 4.9.:  :631 59 79:4 .6: 0 .79:5065:45 &0065::6516: 56.. 79.97 145...9. .0 .5 .. :.67.3 069.6: 0 .9 5 . .:4793690945.451.3:65 &0..9.51 .65. 6 .51..6 7630 609: /..:6. 45 5 .5.19.. 69. : ::5. 6 .9.50 6 5:09736: 41.51. : .::  .595.  .1065::65:9645.  5..9.: 9 .659:  4.9.....036.51695::964.1511/569015.:: :0 ..1/.9.6.9 065::65:   69 5 .13 .5 ..71065::65:5/6941.595.6 41.76.51:09.6 4.14..94794::/3 511 .79:5067630609:.65 ..5..: 7564565 5 :9.3 796:65   .65606901065::65::...5: ::70.:.2 :.: 94. .65 4:. .516350654 1156..4::7966...65 .00:1.9. 065::65 .9.: :51/.45..51.::655679666. 065::65. . .65 .6:5.. . . ....7733.4:. . . :1964.51/690.93 0.9:1..33.:6/.3 .:2410..9.. ....: 5635.: 4.::9.6 :5 . 7:0.  .03.:6690.. 569 5 .45.1 964 .:065::65.: /..0..3 .1 ..5.      1:095 56 :5 .0.65   : . 4:     ..7733.5.. 2 7.351016.3.3:5.065::654./5065060.: /5 .5.1/150.1 .1.5 ../51.:/5973.556..3:0063176::/3/:7731653/.51069500 0.51 9 .5:..9.69.063156.9:0..9. 065::65..16.: 065:5./5. 5 0.. 03..5:765.9. .65 6 .10.797.5.3:.9.. 065::65 : 5:/:.4 6 4. 103.10..0...7731 5..4516506350. .515:0065::65   ..:06996/69.... 103.99.51  5 .95::6.:/531.1  :0 065::65 33 / ::.3065::65./5991964.99. 5 .5.93 0.3 065::65: ..14::/3 .   '9 : 9.: :65 ..::5. 5 .9.. 1.. 9519: ....45.00:1.51.0.5:1.5   . :.5. :.51   6969 ../:.9.9.065::65065.9  .: 635. 065::65 : .:65 69 515 ..653. 065::65 6 ..0.9 9..00:1 930.1  .3:. . 150 :.9.696. .4 .1/6561156...3 ..1  9 .9. .:065..45..99.5...0940631... .6/635. .  ...635...../:50 6 06503: 150 :65 .9 79:41 635.15 .5....: 5.  5. . .. .0..14::/3 : ..7733.5.00:1. 6 / 7:06360. 7950.955:  4:.51.51.51.5.69 9.: 56.6 .:. .41. ::70..4     '9 .694.6 . .. 7. 5 0:.70. 9.51  4:./.:96409./569416:9./5.3   519 .:0.. 9. 5. 9:..3315...: .: 796:65  .15 6 ..:191 : 79:41 .03 &0..9 596545.65  ..7765.61.9. 9.3.5:.:515..516065:3   !6  519 .46:79676305.3 5...5:35.45. . .3351:.5 5065:31 :.5:.36.9. 5 .5133/:1.. 65:...::70.1&.7733.955: 0..9.:0. ./.654:.65  . :0.  4:.65 ..5..1.6065:3 .9..996.4.5 5.15 .59:.:.: 653 .00:1.996.5..51 :996511 / 5.933/.951.5. : 9891 .6979:5...43.63365.33 06901   &7.. .7 6 ..: .631 . .::70.553 5694.569. :70.. . 7630 .51 :6 . 65 69 4   ': 251 6 5 6 .1..7765.955:  5 :9. .5.:0.65.:69.6.6 .335..4.9:631..6.065:3.5.6516.9.6.: 0.9./5:09736:36/:91/.1:65 065:3 69  .00:1.:/5651. 76305:.3. ..9644.3 10:65:6..5.1 .9.6/49309465..69.6.515.5:4.18..7733.. 5.653 5:1..65: / 0647. .:::.   9  ..345.3 5.00:1..65:065:195.69.16.:8:. 065:3 .: 5 0:..7733.9:0663   6969  9..33.996..5 ::70.61.::700.65    989: .03   &0..51 517515.5..9.:21. : :.5 517515.:::.556...5..00:1.:::.473. 69 .:65 .1 / ..5./3 0647. 9. .65 0644.: 065.4  .1. .  .7733.1 / . 3.: .  0... / 065:191 . .   36: %:  6  . # .519 6 .6 79:4.. 065:3 .. 76306906063156.65  0..513.3065:3 7/306979.6 .6. .13.6.9. 36:%: .065:39891/9..: .3 .69 4507.5196..../70./:70.796:0.00:1 5.00:1...5.:0.1.19:.65644.7.1 .14.517515.7733.3.# ..556.:..51 :09736:3 .51&.76306:5..:# . 0.:7.695 69065:36 .9:.5.03  &0.:::.    '::9969 :6/:915#673 . 50 631 9519 33:69 .9 ..9.  51 ( 516..7733.61. 5:. 195 0:. ::70... . 98945.0..: 69 .65 5 . 5 ..*     .3 5:.336 :0 ..65   '6 .65  #673 "/996 &%   .5.5 .775:. 796. .6 . 33.:1 65 .52510.656:065:...9029   5653 /.3.14::/3 5 150  31 59:79150 56065::650.6 .3 065::65 6 .656. 064450.5/.007.5.346796/.39...065::654:.9.61..5694.51064795:656.1515053::.:065:850:   "50.93 .00:1 ..5..7790.:5    93.5:79091/3.6. 511.14.14.69/650..335.51   5.65.51635..69.06473:65 51045.3 5:.1 .65 .6 / .3 519 0:./3.5 0.:/5.5 ..379506 4.:4769..50.     .10.:. 9 4.6/.5   '4..00:1   #673   .: 98945. 094 53:: 79647.: :7769..3065::65.9.5./96 &%   (#.1 / ...5.14::/3 .36796.5/..51 .9.10.65 .9 4.5060647..6911/.51 065:050  5 .5.965 79:47. ./6.0.41./ 635..5...065:3 79:: .51 / 51 79::9  .9. 69 5.5.51 13/9.:::. :.65 9:.:065:. 56 79:65 6 5694..14.: . .  : ...33.5*    !496:10:65:6. .:1506.69 6 .3 451 33 25653  93 ....9. . 065::65 : . /915 6 7966 .696598:.1 / .: 765 ..93.1.1 5 150  .51 59.: 6/.:69.69..65.6919:50.065::654. 797. .....3 065:: . 4:.51517515.33 .. ..   635.65: .95.9:.6906473:65 7:0.77.3:69033.656.:   &%   .0.515.. 0644::656.995. 5:.09469964.4 . 5. .65 6 : 76::/3 7...6 5...14.3796:65069:.:06901..9   #673   .5. . .335.93.511 ..:.79:47. 79:65 : .65.5 5.993:794:165.05 7630 5. 1.151.6 ...96 45.1065:.25 5.   69  .76.5.3:61:9:/.: ..3.65.3.46:79 9555 .65 5 .6 ...3697:06360.6 0:.00:1 964 065::5 3.: .9.50:..::70.00:1964.6 .86.7961 964 . 5.61 69 . 93 : 56.61 '03:65.9 .996.07.43.5...3.5...4::5316.6796..65 796019: 9 . ..14.0644::656.    (5.90     :.  .9:. 6*     :. 1..3.9: 5 . . 065:3 03: .:90:6065:317..1 / ..5  %0691 6 .6 79031 6.65.: 3. 1../944/915.3 3. 7961  .6 . 43.61.  65:.: .6560.. 511  ' 79.3 644::65 .79:655190:.&.5.:90: 6 .5 631 / 65 .35:. 65:...5.5 9..111 765 .9.3. 4./:63.9 8.556.6:09.691.33 0647./3 6 : 65 060 9 730.0606.3: 0 9 . .969 /6315 ..9.9.:56..5135:15:  31 ...65.51 .7765..:9. 43.9 .. .51 517515. 67.50 6 4.51 799..5.1 ..1 6.3.9  ..: 0647.:: 9  195 .9/.51 517515.4:..6065:3:44.695: 964 ./3 . 79::./3 ... . 0.6 .65.91..9: 4. 3.3 644::65 6 765. 65/:473:30.90.49:6. .5471 5.:.5 0.7969::6.45.6::.511 .00:1.996.4766.98.695:964...650..51:6 .3336115...9 65    3.67 .35:.: ./3519&0..* 3.00:1 964 93 .6 ..56.5:.61. 9.:1 .51 635. .335 ..6 : 035..9 79611 / ..0.9 51 56.0. 6949:. 065:3 (.00:19..5 ..5.5 . 065:3 06:5 69 4 .65 . :64..33 .65 ..: 631 3.. 5:./3 .9  .3 ..9.691799.1 5 69 65:.6790316... :3. . 6/:./69.79:655190:.14.93...195.65: /5 79676511 .5 .65:.6 79031 ..65  9.9 6 69 65 9. 0..691.0:.0606.95609   93  .9396. ..065.556.00:1 964 :.65:03: .00:1 9  59 9../:91 :05.5:. 069065 .   #673  ...3.6.  9.9. 5. 3.9: .335 .9.#3775: :.00:1 .:65 69 .9   3..:2 69 .6 / ..6533/:6335.9  : 56. :90:6.65..5 0 4.:   &%  .5.473.330647.9.1 / ..:  4.0.33.76305:.61.69: : 141 5.3. .5135:15:  .7765....5.79:655195:.%7/306.3:   ' 065:3  69 :631 59 795..3.  95.069:6.5...9:/:09/:. 50945.6516: 56.. . . 3..5..:.51517515.4::.69: .51..3060 .5 .5 .. 4 /./5065.:.35:.5 .56.   ': .. 90.45.5 6/0.9 : 56. 8:..:  '65:.6 :.9..306050.. : 5.656.9:5.036.::9.9:.51 .167.00:160.1 .1 7        .:..96063156.9/.   ' 79:50 6 . .6 796.1/.. 90  :.  .9:. (5. 51 7:06360.   .6 1:. .993  31'03:65.6 .3 ..7735..993:79.76305..69796019:3. 9:765:/3. 69 . 9..6*     &631 069.5: /.61.51 0:..3 9651: 69 069065  7:0.65.5.9 8:..: / .14.655 .03:65.3  6 .9065.65796795. ::70.3361 .3 5:.5 79345.5964.&.996. 00:1964.3:61:9:/.65 79679 5 .14::65 .5..4 . .0691/. : 33 519:..1. .993   ' ..6.335.995.00:1:56...5..6 1:.... 03:65.65. .77.0945195:.6796..6 .:9 :69.. .65 .50:.6569.9 56.5.. 065:3 :631 / :..9 93   5 5694.14::654.  635. .0651.5: /.9.3064450.655 . 79:65 519 5:.:.902 165 .511..6 .3313 4...:06901..9:69.6.661   5 4.   69.93. 9.9.65:9769.:6 .79:65350:.1 / ..14.50 6 .1. 15.9 .5 .3361 .60.51 ..5 517515.99891. : 065:191 .00:1964065::53.5.:/531 69 . 6 . 5 .03:65.9.: 5..61. 0647..5 79345.14::/3 ...7733..5..943::6955606:. .6104.51 0:.:::.0.: 98945.3 5:.3 6 :0 9..14.3.6.65 ...14::655/.: .515.7735 ..9 8:... .:56.6 5:9 .569045. : 56. 694 6 .5. .: .5 .1 .773:   '...51.331.1.5.65.3 .00:1.5 5694.. 9..35:.576::/33.4 .::5..99:.1..0..1.47.51 2335 6      ' 03:65..5 6 . .7.657969:56. 7969 56.14::65 : 50945.1195.14.69:6.:.9  . .46.1..69./30644.7733.6.6:079:47.5:.10. 797.7751   ' ...5 .:4.5123356..6:065:.:796/.99379:4:. 5694...094.55 . 73..7.. 7630:.95...6./3.:.14::65..6.77.5 609 195 . .609045. 0647.51 .9.1 / 4 79.5.794::70.99:.:...465. 5 .576304..6 944/9 ...00:1 5 .51.65:.:7.:519.4.: .6.93/5::.99:...: 5 0:.993.14. 094 .:56.61 .. .7630:. 5 .5.  11 56.66 1952 ...45..65 6 : 065:..331 .3313 4.61  &7769...65.14::65 .14::/35150 '76304. 796/. 794 ::70.5 .556.: .14::65.1 /5 .1 .1   '03:65....3 065::65 .4 .5.:.5.51:...331...0: .65..32 .0945195:.991965::6319/.7733..5:35.5.65 .6 : 76::/3 064730./3  '.3..6 ..7 .. .6065:3 56. 0...1.327969..3 ...60:.39..6.5. .5..0...5 /0..5..61..5:..9.:.93..7733.:06901 .9.... ..358969.99:./:506.9:..51..... 094..5123356 '.9.65.99:.5 76304.:4.3 9.14::65.0.694..33134.9.9..59.93:.7733.1 .99:.... :/:85.55694.0644::656. 7733.: 5..0.14::65:631/:..10.6 &% !6   5.3065::65.5.. ..6.:15.14::/3  #673 9..3.6.33169.35069.902165.14::65 '.51.331.50(65.  796019 69 6.%:*     73.5 . 6. . .51.796019696.6:015.52 9 .:.0.14::/3.73.. .65   31 5#673 '. 15.6 1.51 .0.069.: .65 6 ::70.69.945 . . 6. 1 .0. 15.0. 6 :0 15.: 195 0:.::70. .65 .:6 15..069.65   . 63365 .14::/3.:.65:6..61.. .5 .3 5:. 3:.945 .6 1. ..51 . 51 35.: .7  4:6. 9931   "..7:  ' 69. 6. 15.069.0.76305.9:6..:165 .65:06510.1/.96:.: . ::70.964.0.94:6.9357:9.6.615.::70.7:9....69.5::15.65  .:976.0.5::.7976: &5006997.7:.0.6566.5::6915..967679:65:351 769.:..9:65.365:/96.  ..::70.:..:165..6.3:6165. 6. 3 6 .9.65..6 .317960:: 59:635. 6 .::9 . .5.9.45.15..51 935 65 6.95::. .:6065:.:  069.65 069.:.50.. 93: . 0.65065.: .14::/3.: .0.069..98945.:651 6. ...0..51. 15.65 195 .:06473. 6. 1/.6 6.009...3 98945.. 6 0904:..094  .06.5*     '...69:     .46.0945.0. 1465:.0.094.:.5::  ..6.91 . 065:. 9 .5:: 6 .65.655/.. 63365 ..5::196..5.5::...0.:...65.6.35..: 6 1 7960:: 5 9. 15. 15.0.. 065:19 .579691:097...65 .3.15.:651790:3.50: .51 .167.0.5.6. .65 796019  #673   '465 &% !6  (#.5:: 67769.. ..::9.5.50 . .: .33609.65 6 ::70.. .:/5..:33.:  069.6. ::.5. 069.6.9.4/.4  . 15.5...:06473..3.5/..65  .3...95::.51   .1 .. 6. 9. 065.65   ': 0. 6 5...0.069. 5.0.1 . / 065:191 .65 6 .. 15.45.6 795.69: 4:. 6..61.6.50 6 065:3 .65 69 0:.65: /.51.0.:::.65 06510.. 15. .763035..49 &% /   51 ($:4/5*     6: .. 796/.069.9  #673 . .3 5:..1 .79:655. 5.5.765:85.6 .:::.0..00:16.06473.3 :.1/065:3195763035..65/79.15.:56.. 7.14::/35150   .  31'796/.6..763035.79:655.6516:56.51. 51 : :531 6.65 ..3 5:.. 7630 35.61 . 5 . ..51 0 .6 0:.5 5:.51 .:21 .5 5:.6 .. 0644::65 6 . ::70..95 .. .35:. 094 519 5:.. ::70.65 ..:. 6 .65  .6 :..: /5 97. 79:65 : .:.13 31 .9..6 0:.. ..61.6 . .65 : 56.7/0.9.61...51 .:2 8:.14::65   ' :. . 7630 609: /5 .51 5 .: 7.65: 65 .... 0:.: .6 30.07. 7.5 ...65 064450: 5 . 79:65 : .65 95 ..25 5. 763035..: .0.65.6 .6 065:3 /0. 4. 563: .51 : 7935:.:.55065:3115. .3 6 796..7 .3:6/531...0..6 / 6..5 5:631 094 . 9.5..6955.65 6 .:1 ....: /5 31 . 59.9 .3 589 5. 79031.14::/3..55.6. 716:56. 5.763035..0. 06473.1/.5.654.069.79.15.5.0...65 '15. :656:./10.33.14::/35150 .9.5.7765..6#.00:1.69::.6.3. 9 &%  .:56.33..:::..7733..5.1/065:3  #673 #. 33...9.14::/35150:50:95:.9...659 5 .:.61.50(#99. 0.:19150.25.473.473.55065:331 .4*     #. .:50  .51  .: . 0:.1 56 065:3 195 .. 793  5.51040.655:95:.964.3 5:.69 .451 /.3:65.:..:   76:. 3 . 47.14::65:964.306473:65 ':.9/.9.: 56.11 '9.6../3 / :473 6/:9.3 06473:65 .65519.107.:.6065:3/5:964.0.65 796/.5169065::6569.659 03.9164964..6 / 165  .3 69 469.0644::656.:.::6. .40..015195:.65   5 .9..:09.:.5609 :...796:097.3 065::65   #... 064450.65.71694.59. 7.61.5173.6/.79:506065:3  69  .9.56. . 90691 :6: .6 .3  5.: /. 4.630.6569.6 46 : /61 .4: .5:.33:.6. :.636..6 5.69.515.306473:65:50.: : . 51:036:1 .: 9.510.65:../4. 9555 .465...01  69 ..9.00:1 ':.:.465./3.5.0..3313 63..945..313/0647331. .6/76. 6965 .4.4.956.4 964..65964..79:.945.651960..6 ..:28:..69..51 : 06.5 7:0.10...65.656.:4../.:.0.0.65:.5.655.1.255.5.. ...9.: 1...473 .5..79:65:.03 &0.  .5503:656:/615150 5..556.5:..5694.00:1 &09.5.00:1 /..5.:91 69:.094   5.0.65:/731  31. : 6 7:0.25 6 : 95 :...00:1..00:14./616.1/.50..4510.5: . 65:.:69:6: 9461 69 973./...65.3.659 .6 0:. . .6 5...65 &631:065...65:.. 10:65:06473351:.65 :9:.51:0.65 .0.51..35:00..65 964.690:510.90:6   369 .1.. ..3.: .: 4.7.:.3.965 10. 796:0..9.995/.:655:.6/../7967969. 796:0.9.33/9.6   !..51/.796:0..6..65  319:79150:973.9891.5 .31:09.00:1::.11015056776:..9.6 9.65196.9 &.7730.93. /. . .65.:59 /5976:1765.365: 33... 6 .:569....164..11.56.61.65.00:1 9 56.5.3 .65:. 06510...45. 0.6 06510..:   &.659.635.365: '.7.3    5/.6  :.00:1:.31:09.1.69.  %64..::69.606510.00:1:0.43.1 9.945 .0065.6.95  ...976:: 56 6/0.65..3.5.6.998:. . 796:0.7730.65: 50.6015.:.33/1011 '4. 0647331 .5.6 79:4 .365   &:65 ..7730.69. 1 ... ./:5066/0....69 5.91..6..91.5.  .0.51.1.95: 5/.7730.5.69 1.. .690:1:09.. 19.5.69 256: .3. .5 #673 &% 3  51 (65*    '%.:: 69.6 :0 .69   476:1 5 .0.  : 165 65 .65.796:0.6.465./.95 1:7.796:0.65:59.3  6:3.6.7.6569796:65.473:6.1/6964 .. 956.00:1:0. 63365 1.6:/4.:6 .3    !6.5*     .7.. .:06 %.365:  31  .7730.::6/..696.: 9064451.36 550.90     91   (#.9   %  !6    .65 69 /.5/.5..6569/.7730...3 15 :0 7.. 0..65    .150.1.....9..1.9..7.3.796:0.3699894..91.65:9.:6.  .6. /. .965  1:0.9 6 1506. 3. :...65 9::.. .00:1 : :. :44.690:.00:1 : 56.::6511:09..796:0.9 .679:5. 6 .913:: 6 . .965 /.00:1 765 .111'./6..65:631/151   '69.7730.3 6 .65   01 .95 6 . 796:0..069.  6510.3/651 ". 150 6 3.65 69 /.3 9.150.7796.7976:6 5./35. 6 .96569. .:1 65 .6:6.9: 7. ..65    .3.00:1::. . .9 ..9 69 56. . :9.:44.15069.699:.35....65..3 :.069.10..6 06510.6569/.6796019.. /.:: 6 0 :631 / 6943.95::.465...: 69..: 6 .51.6510.1796019:631563. 796:0.6/.610...9.3 1 5 0.3 .96.90:6 369 .9.5696646916/. 1..1 . .065..76:.6 !.7730.65.96   11.3.65..5 .9&.. 9.:70../:50335.796:0. .51 51.96:065:191.::.5.45..3. 1 : 65 06503:65 65 .. %64.15.5.5.65  65 .9.:.33 .00:1 ':44.549.9 :0 150::. .69.:.96556.6.9.36.3  6:3.317960::69/6.:0 .42563169..34:.65:   &6 /.631.95506550.544/96.61:03.796790.5... .6 .36/.5.31.:: 6 /.15:.7730.  145:.5.51.6/5.00:1796:376:.:.31.616 .3361796:65. .659:6473...3.65  %3 6 ..:9:10.6/.3: .5.3 069.  69 6.0..5.6.51.. 6 77.9.9..00:1694.33/9.31517.9 &0.6656:9 #.90  91 (#.3361796:65. 964 .3/651: 7.: .3. %3: 6 69./.0..51.9.99:.69:76515.33/9.9././.90650.964. ..9..6 .631/506596:.5..9..65 0.955: 964 .9..90650.550651.51.65/.. 0:.64.69.6 :09 65: 93. 3.9 .3 :631 / 5 ..5/.06990.3.3.77.6./.65...3:/4::65.  ' 7976: 6 /.3069.3: 7.9.3/651. .659:46.194.4511 / &794 69.51 .99:70.069.6965.50.:3..1/:1195..3  31  :7.65 519. .....65   '. 69.:.5 6919 6 ...659:.979616.9 .36919.65..5*     &631..4:69.51 6. 903. .9:  17916 3/9.9   % !6   .. .519.:1:09.77.3/651195..5   '/..6.5653/:1195.. 5.33/9.00:1.10.565796:65.00:1796:376:./651:4.3/651...6065. :.065:5.336.3:/0..    '69.6.  7961: ..6.7961.4/.65 4... . 79643.:. 6 .3 '::065::.:695. 145.3.:6.77.5156..  99:70.6.9.33:.3 %3 ./.50331  5.5.9.3 .7961.979616.&0. 5.0.9.96101/ &145:. 0.195.3 69.5..31.77..1. ':.3331569.765. 6%3 07961:.3:.3 '9..77..903...:  53:: :6659 0.45145.55690.6.65 6 .33/0.7796. %65.5194.6 .65 .. 3: &% /    :./:5.5136:... ...00:1/9:....    &0    6.9.6065..7961.9.3  6969 519.. 57... 1.99.5033...5..55690.:5195.33/ 0.0.4/.33/9.9:.77.5.3.55 ..77. 6 .9.655...3069.9: ..39 .3/651195...5.:365.3 069.9 : 5:. 7.: 69036:1  .90.:6.33:.67.. 795.691 '::4:..945.77.0651.579601.677. :.545.1 /.../:50.65:9:79:50..65   "/.6.9..51/690631/9.:.93 ..99.655.3:.3.9 964.65 .:331....779511.3.6569/./.39 .9/13. 3..659:/.:.:69190651..545.1 /.9.30631/.95 .79:5.6 95 .90691  .579611.1 6 .75.655.. 79616.: : 56./651:4.65 . /651:4.00:1.91. ..9.99.93 69.46.65916:56.659993.5194. 065:5.7..6 ...51 .6/.77.30....51 : .7730.:.3 065:5.99.31  31  5 9895 .4 765 ..51.5.:165/.69.:..9 .5./651:4..5:50::.3/651::631/1991 5.5 ..531.3069.30631 //5. .::  50 .5:796:65.9.65:.63193:0.:. 6 .6567.9. (.3 /651 69 .65 .5*     :.7796.17..9.: .696. 69.3.77.35.06473.5.35.5.3  .51.9964.5.6.....65965:.9:.367.51 /69  0631 / 9. .3.: 79.6/.:5.:6.0.659 / 9:.545.1  :50 519 9.659: /.99.11.     .5.99. . :0651 73.33 . 9../79031196435..65#96015  31   .00:165:..5..3796:6565/.3 069.9.545.9 6 907960.. 0945.     5 .:.51 .9.6 /.69:.6.5 / .3 .60.5/9:631 :. /.306550.1/69.6.33...:&0. 069.3    6969  .33/9.6.95 .:.  69545.9. : .5 . 0.9631/5. 065:.3796:65 65/.:5679::69:700:. 69 .9. .556.9.5694.68.:796:65.9.0.659.9.5   %  !6  &7.:1656.65:./.. 6 9 79:65   .1/.609/9.773.3796:6565/.9   '. 065:.6/:.69 6 9./.93:.33 96459.:...1.: 0697: 51: .9 6 9.65 .5:. 93 ..6/..51 9:35..36::69164.9.51:50.65.5.9. 79679 069.  ..::   69  ..3.0 /3.556.5.307.1. 793 6 .. 69 79679.3..651 65::   .065:.9. 6 .:191::5.: 9.5.1..5::   '796:655.3 9.545.65../31 .65. 9.51:6631.:/5.9.6569.39.1 / .0  .!6.65 9.545.9.65  50 .. 6 ..:0651:5.6 .:65/.964.79:47.4655..65:8..99:. 4.: 0697: : ::7511 16: 56./3.6/7.9..:: 56.956.1.:65  &0  9.1 . / .355.9.../:/0.751506.1.50: 50315  .3  4...51 ..617960:::/96. .065:..0. 9.51 .39.691 /.1..3 '::05.9 ..5.3 : .65...55 .:.51:9.6/.6: 0. :70.:32.1.656556050:56.679643.5.6 .07.03 65:.9631.616.0:.35 0945.045.65069.3:46.773 .3:.65 6 556050 5 . 6 ..773:653 5.*     :9:76515. 5 .9.545..99..5. .65. 1.1:  &% /   51 ( 516.5/565169..50.569190.9 :.769 .51: .65.79:65.65.9..3:631/ 9.9.. :.76..1&..3 : 56. .95::.0.6 1 7960:: . 3.00:1:065:.65  ..9. 519%3 &0  / 6.3195.5  35.: 6 490.5../355.:.91 69 9/33656965::595.95    &50.51065505:65  .33569.: .5:.5/.4:.33 56. 065:.::75:65 6 . 5 ..3 53:::3.99..76:.5.545.7730.9:.:65.91 :631.99.5 .3795073:6:. 691 0650. 6509./.765.6/.65 653 .9.00:16:63156...:9891   "5.. 0631 5:9 .9:.1.00:1   5 . 3  3/9.65.656#37750945. .. .:/5.::.00:14.46.1. 10.65796015:..0.65031. 6 ./7961/6519. 4.65::69045::6.51:6:.. .65 653765..505.9469 /3. .%3:669.7730.::.:0.1 &.5.9519145. 79:47.:0.5.:. 35 6 ..9.1. .  ../365350945.3.9 / 93.00:1 :.03..51.1.659   : ::.65.1..6.9.7730.0945.6/.9./47.6 73.511:.69.. .:1..5.545.9:./:61:09..50(#..:33.69:4:.5351945.51.9..1..0.369...:07.. /69 .511.51   .51.65796015: /0.99..:..6 8.60651.94..:9..:: .6945...51.../56. .08.   65  33946#9. .5..65::690:0..7730.6.650.:33..3/0..:. 069..1.99. 98:.... .:. 59 :6 9891 / .:5.1..6 79:65: 10.9:: .:165/.9.51 0647335 0904:.3 796015: 69 ..65 .6/./91567965.9.:60650..65.6/.349347./316/. 650 .: :6 .6 .. ....00:1..59 . / . 46.51   6965 .65.65.9..094.5.0651. 4  56.33356.96:09. 793 6 .: .79:506.3 36: 964 ..25 .3106473.9..695694.16973..659 .156.65796015:.796015: :0.1. .65./.5169195:79:50.6..3:.3/69..773169.3.65:6/...01519.: .6....65:.6 4. 069./6..46. 6. 06445.. : 15.:13.. 1.77369/.656%36.56..3 .99:.95:: ..650.515690065:...3 .08.5.:: .77.1.3%35.065:.65 0..16:56.6336:..5*     . : 6 .31.6.::1:4::1 .33 0. /. .0.::9.9....9 /69 .99. 79:50 6 7...516963..6/..1.: '6:.5:.790:3/9.3#96019 656.0.73.3.5.40 .65:05.68..9.65#96015:  31  ' 93    : .545.507.6066:/.945.519.3.69/.1 / .6.5.  0.3796015: ..5...5..9:7.99.631/.65.99.5.. .65 069. .65.7..34.5.6796.79:5./35...9155 . . 7.25 5.: 69 .51.92455.%3:65945.9..1.: .: 93..68.6/..9/.':...: 511 .3 0.3  . 964 .5/.: %3: 3519%3 &0  / .31.3356.9... 5..9.65    0069153.. .:.9. 9:2 69 .31..050.39.5.. .:.2:6.9.7..50..9. 56.33.: 0:   ..606510.65: 5.03. 790:65. ...79:15.3 9:765:/3.10.9198945.1.65: /..06507.5147.695 93.069035:: '69.9:5964...:/.9:.3769..3:..50..9.5..465:.:::. ..967630.:0.9. 1 &..1.: 56.5 69 .69.33.5:. . 56.3 .5. 47.6 9..659 4.: 06503:65  .: 65 7996...50:.   %:76515.5.1.67/3060:/ .. 69 : .4   :.:: .5...5./59. 98:.0.11 5#673 . .30904:..6 493 .4     331:59.1.945... 0.5 .5069.3069.656.:65..693.65'9.33 .9.93 5.9245:98:.511.  513 13..5..39.1.: 765.5/.9 /.9..65.3  0 4.140    69 . 7960:: 46: 69.07.. .4: .65:   69545. 66...3. 69.6 0:.6. 79:5.393.1. 31 ./3..5.. 6 .:65/.  .25 .51.:5..9. 90: 6 . .:69.336     !6.0.9   '9   . 98:.91:0::656.9 03.. .5./:..9:63.00:1    . 50315:1.6  9:15. .51.65     .. 631/5..25 5.5:.9..44/96.:0904:.:6 69 065..1.53. 69935 .5.45.:.519. 4:.  5 .30904:.:03.5.. . #3775: 0..65.9.5*   '%.:: 03.5159. 3.9.. 9.51535. 0.6 4:3   .650.5 5.36:6: .9.9 3356.1..065501 4:.5.5 69545. 6 .51 ..65    5.9.6 9. .50:.945 .:600:.6 4.39:63. 5 /69 79..6.3 631 / .: 765.6 4.9: ..5.5 1...3   .465.65331:59. 1:59.5.61 .951 6 .9 065.3 ./34711690647964:1 5:69.5. .5.06473359.50:45.0.56. ..656....9:65 / .6 . 9:2   '6 :7769.956.59.4:..9.9:70.65.55:065694.7: 6 .9.751506.451 . / .1.  5 /..0.: 56. 3 .    .: .:969:631.... 65 33946#9./. 769 .: .51.10.13.6 5 4 .50 . 51 .:.00:15.: /5 796:3 151   5 .91 / 0. 7...: 03.. 31 964 ..5..33561:0::.9.53.... .9.:56. 79345.:.::..1:.  .0.3.5/...5.35.6.:70.9065:..3:6 9065: ..51:.55.51 7969 ./5797.3 93...!.. 505 036:9 .65..:65 /.5.69.64.65 98:.5 .9. 5 5  . 135: 5 .4/.6/.50(#.5.69556050 !.:6490.613.: "5.: 769 :631 / 0. .:7515 %:76515./.61.50:45./065:195.1 ..6:.50:: .9:. .9:. 69 796:65...5  % !6  &7.:3..657515. 69796:65.51.6: 6%79:5.69.07. .9.:  .3 6/3.95.30904:..: .1.3  31365.356.3 7996.9. ..: #94::065:191 .556..1..:30.65 69 /.119:::::93. ..3:6 5069.:35 45065.1 57944... 9..4.657. . 9:76515.: ...   5../.31.3     5.:9796..:.   65  33946 #9..5.4699...50(#.:. .606554195.56.9.: 6 490.51 036:9   '..  .4..1 5 . 069.. .556.. 796015: '::8.0.51 : 9651 .25 .6569..1.6 1.7730.6.5..90647335 56 69 .1&..9.. 34. ..0.:  .956.5*     9. / 9.  33 56.5.5 .595:69:76515. 45 9.9 56.00691.65  :6 .5. 5694... / 56.1/.556..  .51 :.36:6: ..51: .33 / .39.65 50  :065:.631/493.9 5 5./313.50:06473355669..:30. 0659::65.933 ..: .1 .9979:5..5.5:.9. 4.3.9:.6.   5.: /5 ..13.9.51 65 ..94. /0.3:..65   .:.7730. ..9 73.167/3025631.:65.9 9:69.34.0.90.065:850:6..5.69...: .. 065..65 796015: .9   ' .9169.9:70. :769.95.9::: .. : 98:..:1.  33 :.1 .51 /3. 9.95 150 65 .:65631.::9 ': .4 .6 333 5..   ..:98:.9 35.65 0. / ..1.. 631 .5 9.6 .6 145.9.9 ...9.945: 3..3 5.65. 0.: .  : 56.  3.65796015:5469 ': 0.3 ..: .:.79:. . /.   50  .65.: 6 .:.065:. 5..: 0:65  69545. 065.33 31 195 .0 6 69 69545.773169. ..:1.5 ..1&.. 6 /.65 069.91 .9 :44.%:2   45 9.#.: 56.65.: 6 .9... .69.:.39...:65.5:.6/569416. 76::/3.9 :.11/521..4.515.6. 3.51:.1..5  % !6   &7.0945.   3:.06.. .:9..: 6 .%3:669.. 6/0...: 6 ..5.90.65 %36.91:&0.. .65: .00:.6 / 56941 6 ./6.5:.00:1  315:.9 . 5. 9.  .   '7976:6.9 . .51 0. 15.6.9.656.3 17515..00:.:9.5.5 65: 56.4.51 /064: .79666:556050  6969 :56.3347.69115.00:1  9.1  69 .5.. ..  #673 .655 6 .151.15:69:3693/9.65(* 65:.65:93. : ...606510.33 .5.:..  31'93/.37960150.15:   '6..3    .  .60.:...656..009.909.0647951..56. 50./3797. 064795:65 0 : 9.355.5 5:5:/3 6/0.369..101..:...9.3..65 65:631/.9:05. : 4769.9.069. ..069. /  /. 151.:6.9. ..065:06:..91/69:7.3 '64::65:56.9350311  ..:.5::: 09../34 .1/563::.9.0.0.: 519 :09.4.995..:65:51935.3.9.6./06473. 5... 065:.5:.565:9.796015:4..:9.: : : 73.3 '60650.3350647.. 7976: 6 .. .110..39:3.0650..51 65 6 .51. 6 .6065965.9.39795:/3.5 .0.9.. / ..00:10.796.00:.5..5.:.493694.56.1   .51 9.6569.5:.9.5.069..9169. / 1:97.5:.556.169 .4:35..9.5 50647..91631065:.651 .6 ..65694.9796:0.5.33 .6.9:56.4    .5 .369796..796015:4.65.:.565.33:.00:16.5.9..3:6.5.:56...5 79:65  .5./69.00:1. 15. . ..:/531.6 06503: .9.9. 4569.33. &%  .:.3 ..9..6.5.6 ...9.5.9.9..36.:.:0 10:65:5.36.:65: 51935 . 5694....96.. 345..9.5.9/.9.5. .3 .19:. .0.3..00:1 0./31/61.0..1..4.9.315 #06 #673.5.:.515 6 . .7790.6/0.4..40.5.1:097.: :9.00:16.5 5:.9....:6..95.0. 150 6 3.:323.:..65 3.4./313. .2694:3 .556.193:.:.9:.:70065:../50644..51 :5.7/30 .: .5.:479.: .51 065505 .3 9.5.069:6..64..9:.5.95::6.579:65:69.: ..5694.9.9.   '91  ..3069.. 0940135.0945..335.::5..5.6911..0 6 .695694.:7965195.5: 65/.5. &% .365:..6 4..96645.:70..50(#9:4.55:.510.51 . .36.0:0.06 .5. ..009...9..:510.5.5 50647.33 .:.4 . 6 .6.695694.6560.79:6565.6473. 63. 765 : 519:.331../8:.07. 4.. 6009: ..86.0945.6776:5.:.6536::.0647951. 256:   : /5 75:1  .151. 4..51 : 06445.6.65.1.3 796015 : ::5.34:36:0650.51   '65694.3065:3.9 .65. .51.6/56941 .60650.5. 151.065:3 ...:.5..1. .: :50...9..5  .:. 9..6..7.91:69.9.369:5.9..65 4.9.56..6565 0:..:54...30..2..9    '695:.4:.96.5:.9.:0.2. &0651 .9. 151..51.5.319.. ':60...65.3 7960::    0945.50 4 : .5.90.6:7769.6906..:335.510.5:.026064795:6551.995 . 56.6473.03  &0.5 .6569 .6/569416.56.50  #673 :.5..3 .6. 0.76:.5.5/... 93 /.506.90  5.3   550647..60.65.5:7.6/.5:.: ..6989.9..45.94. 796015: 4./369469.9..08.1 6 .0.9550::.106946 .65:..5510..45.969:15: 69633.   69.6:06510..5 511. 0. 151...690:4./9:.64..5 50.   69 65  .469. 151.5..:/96.50 6 ..556.00:11:.65 6 .9.45..:0.:   ..55904.65 .1:.9..*    '%.556....65.:6.9.9:50 160.9'9.:651 5. .915.151..3 69 :5.0.3 796015    .65650:. '67. 151.:..65 16: 56.5694.:: :3.3.915.69.9.00:1     5 .5 3...:.515.::91  .65 /..6531.:6. / 0650.3 9.331 :6.9 .00:14.5. 796015: 4.5. 51. 5  5..517/30.9 .3:0663:6 .5659:. 65 6 ..3 3...3679630..9.9.6 1.547.51 736:670.6 79507. . 9. .6.3    .50(#56*  '%.3 .::   .3'9.9.5:. .65.3 .6.195. 9 . .3 0. 9. 6 997..9. 9.51736:670.5. 13671 195.   :9.::4 .::   ' 9.: .   & 069.9.: ::     :/:.0.5 . 6 .: ./3. .9.3 6  7963 0.5 . 9   ':  53: 069.7796.   ' 490.1..7796.7796. .65 ./6.9 . :. 5:.. .6..00:1 .13 :./3 .9. 33 7910 .:.65 .9 ..00:1 .6 . :27.0  :.9.5 6 59:. 7/30.5.3:0663:6.6 9:63 .51 .6 .0 .9.0 : 195.5: 6 . 31'9.656.6   %  !6:  .3   ' . 6 79.33.: .3 . .. .3: 5 ..9.0 .3 :9: .3.5 . 1. 76.61. 6 . 796/34 .9.3 796/.: 9.51 79:5.3.51 :. 65 . 9.9.59  .3 .965 323661  03.9. 195 . 79:47.94  :..67 0945.: 7/30.6  79507.51 7/30.. 69:  .65:579630945.0.656:9.::563.15 ..5/:..3:69.5.10..79:7910..3 ' 49. :...:5..659 6949#9:15.5. "4/1:4."4/1:4.3: '5.3 : 56.5.:56.6.979.5:. 9:76515.6 .9.6 ..:03.95.5:.0..4.5:7/30:09..1.9.5 :631 / :..  0.1..677. "4/1:4.67910. 6 :.99.5.6:..:.9.90     5 .5.3 3.7733. 7630  796:0.:/59.:/59.: .5.9. 0945.617960::       '::56.0 / :/0.9.::31. 9. ....5.:33765.00:1 .5:..514.556. 6 .1165.3.65:3.5.:.6.5 ..51:.369 .32.331 '79::16:56.65 : 3. :.: .9./. .7733.5:.0. 9 79::   '6 / :9  9:765:/3 9769.0.99.3.5109...5.5 .:6.:69.20945.9:765:/379:: .6..9.::.5:..6 3 . .4.6.00:1.99. .::31.335.160..:.31..656.: 7/30.6771 964 06510..9.16.51 10. 69./3 ./.0. 9:76515.52 9 3.:: 5#673 '.50 (#56*    &631.3:/.9.: 13671 /. .337963.9 ..10..65...37/30.9..647./6.91:.51: .::6.9.1. .6.6.9.  .30..3/7/30.9. 7.659    065.6 7910.3 7960::: .911....99....37/30..51 : ..41. 50647. .:7/30.6 .656.9.5 .  31  #.145:.3...55..:151..    :/4. .5156 93.99.9.30. 4:0.9.00:1:9...795..5530650.37/30.36.:.41..51/96./..9:.9.:15.3: .:....1560...3.67910.65 :70.677196406510.67...0:4   #9.9..3.33 :. 0945.50:..:4737/3:5694. 6.. 1. .3. 6.. 3.51 47.. ./ .3069. 6965 ..1 . :6 794.9.3. . 451 6 .91 : 47.6:../5096479. 7/30...9.3 1 .:3 796 .:476::/3.451:644/9:6.16:56. 9 6.9.3 .51 6.. 31:56..65::. 6 .510.775:.3 0.9..51.6:6.9.069. .93.9.0.60.19647/30.3..::   ' :.56.9.61:9.  "91:.9693796. 6 :5:. 6 69 064450. 7/30.:.:../3:.456.91 6.46:44/9:..3 0945. 6931 .5:.66991.51.9::.9..6...:.9..9 691..6.6 69/19664: ':5:6947...331..3..2..9.6.94.0..4/95:5:.5147..669/9..65:63 69 ..65. 3:.56.519..36:.9515.947.. 069.150.5165. 7.:.0.0.6.5.5:.7733.91 6 76::/3. 069.0.51.49./..65. .:   5 . 0..6.1.0.. .5653065976::/3..90691:1656. . 5:.167...6 .356  .../ /.9.3 /.51 ..   ..9..6 7/30..517966.79.. .:. .5: 41.79:.. 0..6. /.3 1 13671 . 065:850 6 .65 '94976:9.947..3350.:473..3   90.:. 5..99.94:..33.37910.79694.517/30.9. 6 7910 .37/30.: .5.5196  . ./.0.5./551353501 56.  .50:67.51567910.67/30.9. .65:..99..9.99.3 6 .4.3.1:. .6.: ..:16:56.9..91 ..: :.51 .3    3.5.0.65 ..31 1 .9 ...:931....9. 6 7/30..6.9.3.:6./:.1 ./.7733.6791065.:. .  79345.36:0.3 7/30.9 9.659: 9.3 1 .3 47145. 0. 9:3..:. 1 67565 .::    .6 .0891 .3.: .9.3 7/30..: 160. 0.65    51 56 796019..0.9 5:.6 1 7960:: 3 51965 79345.:.5.: 6 //   65  %.51.. .3 9:2 .9.6 .9.6. .5650.51.../36 0. 6 7910..50:6.3.5..95 5 .:16:56.0. 796 .51. 6 7910.9 65 .: ..: . :/:85.:0647.1:0.5  7.7. /915 .: .:56.3/.1 195 .. .93 560.0 .9 3/9.60904:. 65 . 0 : 50.5 5 / 150 79:5. :/:./915     76511 9. 63 51965 ..3 1 65  .65    .9.3  7733. .: '.9 5:...91..65 065:195 .6 796. . . .3: 0.9: . .: /5 . : 56.:.9 :6/9 .07.:.9.9.51 ./6.1 / 25. 6 41. /3..6 .51 7/30   5 . !  ..0.3 . 7.33 /64/.3 56.: . ...65.9. 9:76515.51 ..79: 50  95.65:51:.:  .0.690.556. 79:: .61. ..9:  ./:: .51 6.51 :/34   511  5 . 7/30 .: 065.556./. .91 .3.9 "91..:6%04651!:7.65 ..5: 41.634..9 3.5: 5..: 41.69: :.63.611 .9 . / 06473.::0.511..30.0945.95:..31.0:..51360. .0 136745.5 6 0 .656. :45.0.9. .: 5 .::0.66 4.0.3:6:5:. .3 036:1 .   6445. 79507... 0.54./.9 :47.. 0.   '14609.. ':..3 ..1 5 .5.6 .: 0::5:: .1 5 ...:  .79345.30.35536.515  0945.069../.6.9 69:9 0.   ' 76::/3.69: 5 .. 67/30.9 .9.656.31506.3064450.6.::331     .9.9.51 0..4:  . .:.5799  .1 365 /5675. 0.6 / 79:5. .: &   1    & .:45    .3 7/30.:6:70. 064465 069 7976: 6 . .5../.9.3: /6.11..0.5 :.:.5196  . 9:..1:.:165:09.37960:::. 65:.3 0.33796..5/59..51 .  . 93 65 .65... . ..7/30.:.9.5.: ..511. .5: 5.516. / 79611 / .6..9 79:50:.67....5.653..9..50.490.....65: 69.5. 5.7.0. 23 3.9..51 0945.6....4.51 7/30 796...65 : .61. 6 549. .33..5.6 6/:9 :0 7960::  964. .9. ..: 6 7/30.3 5:. 45145. 79673.. 6. #96:0... 0.55.1.9 6 ..6567.5.5 .7/30.91156.45.:65:..9.9.53 69.0 ".93 036:5 069. .65 65 4.9...1/9..:  56.   / '9164:6:70 79:: .: !..5..51 5 53.65.:.51 7910..: &  11 &.: .3519.3:6.:5:6.5/9. 511 .33  . :60.5 /:. 0945.6 065:19 5 1.5.3. 964 :44. 5.9...5 6336:  .:906515.9 .9.::4/3 79::3..:691646:70. . 9 90: 6 .6.9.3 3521 / ..506:. .1  .515 ..6.3365 7673 .0..90:16905.3. 79:47.51....9664 : .69:   '9 365 7950 5 0945.   0 0. .3 . 5 . .. 519 09. .51 46.: .:..0   0  5 .9:. 79::3 .659: 0.5 6.5:. .9.9.3 ': 5 .: 06503:3 .6.6.9. 9.3:  .4 .: 13/9. :50.:3 / /3511 / ..: 6 .4..:473.690.90691:.51065. . " #.5179:: .93.9.9. 4.::95 9164 6 064450.: 9 . : 4769.:796. 5.:0.1    ./.9..5.9../  /. 69 06445.6965. .6.9455 . 4:.:./. 9 .167. 5645....9.330650951. .6.5. 0 .9164.1 365 /5 79:47.1.6 .9.51 9 .  .: ..9. 9.5517515.1/. .9.: 5 .3    3. ....: .: . 9...9 4:.6 .517966.59164::0. 065095  6:.0.3: . /5 90651 ..2:73. / 0650311 .50:0...:505. .3.3   31 .67/30..3 9. 45145.9 5156.5 :::.51 &569 &.3:693..9. .51:0.9.65 6 6.. .:70.9: 7.96: 51.5 0904:. 41..53 : 06476:1 6 . 9.3. / .9:93.5.5.9:. 79.9..3:.335/2565 .:697.0.6 . 515 6 7910..::9.:.9.  .0 :9 .65 .65   '6 692 0...: 0945.45145.99.:51:75:.69 :7769.6 .659:..:!.46:.. .6 ..79::0631/:09...9. 9:.65 065.51 47.1 . 7/30 .0 9 .3 7/30.0./551353501 56....  .77.3 .65..6 ..:   ' 9./. .:/5. 7/30 73.9: 4769.167.511...: 6 .655669545..99..5.3: :6 .::4/3:.. 9 5.95::6.51 ..07.6 .4:06510..51 /651 2565   '6 / :9  .5.9.51 0945.: 6910:65:/.:6025094 6009:  ..6.5.9.6. 6 .5..:67/30..0    5 5.:5/9625 5065.9.55 . 9:.3350. " #... 06445.356 .1  0945.9..5179::  :.796015:906510. :.45.1:...065.9 .. .06445.1  6969 .:4.53   #.69 ..6 90 5694.95:: . 45145.:..0.6 .. 9.518.50 3.:. 675 7960::: 6 :.5.94: .5: 56 796:65 0 / .5...3 675  .9/.51 979:5.3  .9.1&.9 .  &. 6.6: 730. 7673 59.65 6 6755:: 59: 5 . 45145..65.50.511:069.9.6 .50.9 9:.:  4.3:.367/30./3 .5.1 9.365  796/. 5.5179.10..1&. .0 7976:  79615 .9664 1669: 0 .3.6:6 :70.45145... 69545.5 4769.556.3.5.:.31. .51 ..33..   9065 ..65.6.3 .65:::. 19.  :.3 069.0 1465:. :/345."#.3 : 4730.69 .00:16:17960::9. #96:0.3: 07673. 650..6/.5.77. .99:5651/.065:191.%:63.16:56.7.9.9....:69./3510. 1101/.9069115007.   .150796793. 5 . 1:069 46.67910.9 : /915 6 7966    51: ..565:..7...544/96 .   7735.5:.65951:..0064461. 117..30.91 0.656.6 :00::33 796 ."4/1:4.9145..9.656.669469 .56150.5..50 56. 596:.5 6:.65: 6 7.37/30.4.:44..9:76515.6 9519 ..6.9/64/.:9:3.95:..9..: '35.65.6565. ."#.5964. .7.. 47..2 33 6 .3 :050 150 .659/. 1 .5 #..0.. 0 ..9:56.96516/.659: :7.5365.1 ../6935 631.659::2.. 6 .95:: .:06510.6 :6 469 .95. :60.9... 79345.7.135: .51.6.11:7.1:8. /.6 1:0.:69.99.3 .:. 659...::.:515:.1.:70.0...:9064451.7.."4/1:4.9.5:.9:.659..:.6064 6..44/9:6.5:.: /5 4..1/.:69 .659  :.65/.37.37/30.1/550.0..7...:7910..1/ /..::70.69: 5 .53..53 6 796:0..556../3 .5   !6..4 511 ...3 7.9.:70. :9.95:. 9:76515.69.51.37.53. .451:6.:/0.33.39.79.6  % !6: .:0651::.65 .6.656.9 10:65  33.7. 60 6 .69.6533/5.:6.335196579345. 50(#56*    '%.&3.90  5..  ..5:. 50945.65    00:1.. . ..:3.33: .65  5 63.6 ..9..61 6 7630 609:  :64 ..25 964 4 ..9 :.5. : 065:5.65 6 : 9.1 ..51: 9 . ! 69 5:.7733.51 :/4.6.5:... . 3 5 . 0:.. 796:65:69... %:1#5.96.361 .00:1.5.65 :19640.50945.036. 50::.. .6 7.0.3 0690 5 5.. 69.: :.65..036.61..3 :.5:.0. 9...65 6 .61.6.3 :.:..9:90.65 .. !13::..65:796:09/1519&0...3 060 69 47./6.00:1 195 0:.6 69/.:   #9::9 0 679.. . 7:0. 0.91: : 595...3 145.151..77. .3069065. 6 55.69:.46:79 6 0:.9 : 0..51/9.:6 6350 . &6:469.9306556....506. .: 6 ... 5 . 9065..6:..631/:05.:::..9   69  ./:50 6 796019.25 ..: 6 .3 5:..0..1.51  9.5.65    ' .46569.:.9 :.515.50 5 .: ..5 ..65:. .677. 9 .45.6.96.00:1.0 &.25 9. 69.79.. 65:.9.3: &%(* 6473:6516:56..65 6 .: . .65:.465964..03 &0.65.533537:6 .65: 5 ..6690./3 4 964 4.3  56.7.3 5:. 79610.65   . 69 4.9:  69 ..5.51 9..14::65 ..:::.5.3 .00:16/.9 : 33  1:.915 .. 7:06360. 65:...6:.65.506065:335190:.796:65:69.61.065:36: 65060   31':6150.1 93: . / . 4.65 9.35:.7733...1/. :: 69 ...3 .65.: 33 .45. /3661 6 ././..5 5065:.: 7:0. &    ..0690650.4..65..65  3.334.9.519. .00:1 : 56.358:.9:47.5../6.92 6 ... .. 653 ..02/95 3.3 .651796:65:. .94.51 .5 / 45. 00:1.1796:65: .064450.:..369469..86.69..33796:09/1 :.306473:65.:67:0..6.0.65964. 4.6.3.46..5:. / 901 .3  .3:6/:1...00:14.00:1 4./4.6.9. /61 6 . ..: 6 3.7733.00:1... 4 &   "5 & 65   #3   (*   65:85..514697569016.. .3 695:.: 150 5 796:0.503:656:/6151505.1964.504..65 69 .: 150 .0.6 .5.5 '5 #3 (* .5156.:06:5:: &   '.50 :/:. 1 .5 150 064450.... ./. 5 5.4 69.5:.:.9:.. .: 6 ..9 :.0891 964 .465.473: 9 690/3 .51 :/4..00:1 519 19::  #673 %65196 &% . ! 69 695:0 . 5::....7733.15150.9 .473:4.65 .45.6 .5.. ..25 964 4 .14.3 06473:65 69 .: .:796:09/1:. :3. 0   5..50(#99..9.4*     6:.5:.  .459: &%(* 31.:0..9.6.... :3.5:.65..51..3 6.796015:  315#.3.50945.9.145:.916 10. 0945.. 5:..65 519 &0.  :0 ..93 .9..3.3..96:.0.65 6 .3 .679.5:.6.1 6.041056513.9 6 79679..65 ..65 &%  .5 6 : 0.75.793.65: .5   5.0. .9.5 . .50945. 4469.9.0 ...  65:..5:36::6..  0. 305:1 7:0.369 75.3 79.65 0 : 6915.796015:076::::.91 .  &09./.: 6 .65 6 65: 305: ..5 5 9.5 ..3796:0.:3.9 179.3   .:70.145:.5  &%  (*  765.5. '69.65   9. 960.145:.03  6 ..50( 36*     .51:.: .. 410.. 006319:3...659  : .: ./365350945.939 0.5 69.. . 65/..163:3.50945.65    .31356211955895.. : /5 31 ..1/.. .6/. 5.35:.5::    0.33 93.90659::65.515 1:036:9: 964 . 69.65:.::4 69 . 659:: 5 14. 31  (*.6  .90. .0 6.:.:.0....6 . .:    5.34.:.0.55895...93.01/..5 5335 .3.5.65 50315..6 589 : :/0..556. 0659::65... .336%.5..49:4/3.506 .5063:3.34.. :473.336%.:  .. 6 .65:73.656569545..9:.65: 6...7/3051.3345.56.51.: 9...3 0644.7976:63156..65::.:09/1.: '09. 25: & &&0.9.65765.679..65:.:.659::16:56.:3.3.9. 9:765:/3..:09..65..5& %43 &    "56. 73.3:9.3:.65:./91:3/9.50: ./10..5511.6:70 79:: 936569 .1   '6 16 :6 631 / .0. .1/...65:9..5:...::4/3  .5:.0569.9./.5 79.01/../.6..0765.: .6 .10. 9.5511..5../3 5096. 3 :35.9 .: .56915..0. 69..677.6.00:1 4.96.6 94.5   #.50. .00:1 6007:. 4.65: .2.511 . 9..:. / .5.1 .5::: 653 .5:.9.5 .5 . / 5621 / 6.: 9. .5103..5.: 8:.6. .31 6 / .00:1 : .: . 4.. ..0945.653./.75.:.6.: /.9.7.65:550.6 9:76515.65 9895 .3 0.5%646.: 5 .5 06473..5:9.2.5..4    ':1:. 9.4..9..4.5 .0 8:.3.5:::.3796015 5 .3:  .5:::.91 .9.366:.65:    6969  . :3.95 5 .659 .34.26.195.5:9 : :6.51. .96796.:21 6 ..1/.69. .9. 50945. 9.5::: .00:1 5 .9. '6569.793 ..379601569. 6 .5  .9 . .6...51.5::4. 160. 065:.65   ': 9.5::  9.65964.5 50945. 0945.145:./369..3   ..6.. 9. 5:.65: /./0647331.519:.6 33:. .0945.56915. 6 ..51..338:. 4  .9.3 .9 9:...6 562 .:  #.. : . .30. . .../.5:.....0..65 56..:31..3 0.51.51..0. :3.5562:9. 653 5 0945.5.00:1.51.9..51.:50:56.2.:..6.0945.    ...33 6..::56.9:..556.95:.7:6:.5:::.3 796015: /...50945.3:6 5 .  1156. 51 .5.93. 644.  &% !6   5. .9 6 .9..::5075:45.9  .9.6.9. 10.50.6 30.4:79676511...:.:.... .51 79610 150 /69 . : 653 /0. .3 /5 31 .5::   ' .96.6956. :.163:3. 3:3.65 0 .9.0.  . 6 .1&. .5 . 8:.: 160.69 445..9.65  565 9  &5. 56...651/3.6.963:   ' 793 . 795073 6 :7.3 445. .. 9.:0. 5.     16 56.5.:631.5 .5.7:6445.. ..0. 065.9 ... 33 50945.9.133. .....9.5:9 .1 .0.51/.*    .6 .77.6.::6..: 065::..:/96.51 79:1 631/63.6. 9:76515.... 691.659: 4..65:..330.1765.:..7.9.6 .5 ..64 3.6/ :.: 6 69545.5631 /.:.8:.7: 6 :.95:     79:5.45.50945.5.5:..65  31"9445.65.33796015::. ...19:0676796.:56.65 6 769: /..3 17. 9 461 .0.5695 5.: .51. / 0647331 / ...3%//65644..51...51 .:.651589:56..50(#..51: .65 653 5 .6 .9.65. 7.5:: ..96490..3 93 . 796015: . .51. .. 19. 5::0.5:.:/96.:69. .5.65 .:445.5.169.195.0.9:56.565:.0.6.5563659/796:0.:796.0.3445. 6 :. .. 9.:9.0..65.:.65 5 065.:0676.  / .69.9.9.5:. '9. 51.. 19... 5  &%  .5:: : 653 .  .51 150 191 964 . .51.7.:..9..  9    &.03. / :1 .5:.: 445.9 . 4 69 9 5 . :/:85..465 .65   . 33 56.::91 .5/. : 69 9 7.. 796:0. .3065:.00:1  31(*90.659: .069.91..5:. ./:.0.51 .969 4:.91.9:76515..   .65445..6 . .5.6 445.9.90..5..90.:70.. 793     5 .:70.0. 69 .3 065:.330.6 .00:1 335 .9./:.:.:.6:0.5 .5:.3 793 .9.7.:. 69545.3793 ..5.50 (#56*     : ..91..:935.:473:.4:. 9. :651.:3...:.25.9... 5. 9.69545.51.4:9..1/.9.365 7196..:76:.1.5:.9 .9. 65:.50945.. .  :3..:445./::..3::  '6 .6..5 4./3.5:. .:.3.6.65/96..:658:.55.5:.336. 909950 6 .:.5:.4.91 .: ./:::.3.61  ./6.:   '1.. ....1:70. 3. 9.:..59.46:.9.: !6.6:51..:606901065::65:.79.6.. 33.51.9 6 7/30 3..:6.6 .9: "9...:::.51.33031065.9.0064461. 69545.:3.55 094 69 7.45.9: 69 0.51 6919 : . 50::.4.51 .:2 6 065.0 ...9 4.50945.9 '6.: ... 5 ..: 5:065015...5:.65 . 606473.:5..10631.:73.5:.33669545.5::  445./:63..97716.9..6..7:95.95:.6 ..1:7.5::.:...50945..0.65.:3.:.::.:. .::..65 '65:3.65.6/2565.9..369..5:::5..:064.:.:.50945....0..3.65..:964.9:6 5065:.5:.. . .19.. 9.:3...79069.5.7936445. '6: 5.7..:.  "99.51:.93 .906../65.5:.6/:35.3:14...956. :445... $.1..97906:9.96469545.03..:995196..:.:445. 51..9.50::. .9 9:70.16. 9. 9.655.3  .:631 / ..6 961 .511. .: 79455.5 &% ..:4:...51 796. 9  &...51 . 690 6 ..93/5.50945.3 '69545.696.0.5/.979. 9.65.51..4:..65   .35..:.: .6/:35.3/9.006911 9.7.6:63094:/. 9796:0..:0651065. .. 79611..65: 6 .915..939.5:..5.60.91675:45.56.0..473./36539.65:.: .99.91   :0::.50  0650..35.6 .4... .9: . 69 / 195.01519656.3165.31 .6::...9 5015..90.:16/367.1653. .465: 69./3. .:70.51.179643.91 '8:.1  9 .:5./7965     9:. .3 67..1 03.03.50 . 9:.0..0  31'6:/:. :. :0651   .69.:. .3.  ..:/:85.:.::0 345.50(#56*  '%.  793  5.6.3796..91.465: 5.4 6 16/3 67.914:..51.9:.5:.69.9/.65.65:0.:5.: .65:0.5.:. 4769.....:.91 4:. /.6569.91675:45.02615.:   3:9 :..665::0. 15.5.91.65 6 .6.1 .3.9 .07. :5.91 4:.367...0.91   31  "9 33 6 %..6. 67.91 5 .6/367. ': . .0940.50 654.95:.31 73.4.:.5 ...15: 4.5.. 3.5 6915.633654:.3:56.:10:655...6:.. 3:3.65:: 69..51 .3519 .65: 6 .90. / /69.:16/367.6 .:/55. 9:70.503115 6916:56..31510..3.5*     5.06.50 6 :. .33/.0945.616/367.91   ' 9:.65615..6569.5 .119::1/.33 :00650..4  65 .6915.165: ./.: ..91 6 75:45.9 . / 7961 .56.9..30.00:1   :65      &%   793   (#. :.50796/. 1  5 .3 33.. &% 3      .::56..  519.0.4616/367.1  .91675:45.9 / 195.. 67.65::0. :. 63.91 .::65.: ':0651:5.. .:70.91:..3345.65..:/5.69. 3.65 9.:... 0650.1.::5.6 . 0.65 69 .91 .567.455. /5.51 .4 65:   ' :0651 :5.. 69. :.45. 9:..90650.6.0647.6 :/:.:065167..693.:  ./3 / .08.50 6 3.6 .. 4:.0:653  765.5 6915.6 0644.5679:65:. 5 ...5:.5/..: .: 1:4::1 69 6.....: /. 0 .90.03.91 .9 ..79643.51   . 6 65:: 0.6 65:::....9 :.. 6. 79:: 065:5.. : 75:.: /5 5.195. .91  5 4.5.9065:. 63..:.313.. 5694..9:..73.50.:16565..91675:45.773: 5.567.:.65:0. 65: 0. 6 ..545. 589 93.:065165: 5031: 69 : 50::.5.465:  &6365...6.16/367.16/367.33/.6   251: 6 16/3 67.0./69. . .036.9.:67. 65: 0.65. 03. .. /...: . ./. ..:6.9 75.08..515.91..9:. 6.3 519 .945.91 .9 :..6251:616/367.51:75:.: &0.96   .9.4 :.65  69 : .91  '065:.796:0..91    ..069.4.5694.65 569.0...08. 69 .469:. .9.656:.15.. 65 065:. .0.3: . :0.:.91 .465:9.33 065:.656.47.: 1.65519..5..0..9 56..945..93 50311 5 . :0.08.07..6569 .6.4.91 .9:..50:.0945.3..01 7969 . 5031 .: 7961: ..0945.915650.:5635.: . 65:: 5631   #673   $.65691. 6..5 5 .9.3 6 .963.3.::. .5 . 6 ..3 069.69..3 4.506 .    .9.5... 56..7. 73..:79646.65: 5. 9:...9.1. ..51 .%:76515..3   ('*79643.3:6 .0.91 69 .75. 6 ..36.31..:59.: .3 069.0945..:70. 07.6.00:15 16/367.65 6 0.:1:76::6/6.:4..6:71.6 / 95:.5./.65 / 1 %. 0.91   3194:.69.656.65..7.5*     ....6510:651156.9.06510.51.1 569.:.3069.589.176546.03 3..6595191.1 ..:.6::.:33.6:71.:.516.60650.31  #..:  6.:..3  34.9.:3  0:65 .6 .56.656... . : 56.9. :/:.::.00:1   9  .9.069.6 :71 1:76:.36.00:1/69.:9:3.6 ..653356..330.79643.. .3:6 / . .3.0.: 9:3.1.9.1:4:. 59..:93.51 0904:.91 .9.31..9.:65.51::00::69.3361 . 677. %:76515.911.659  :65    &% 793   (#.9....9.63.435:050 6150.9.51: .3069.65 69 .. 03 .30.1. 906965145..06473.9.65/065:3697... 93  ./:61:09.:1:76:.9.. ../90.:1.1 ..34. 79643.79::065:5...656. :10:6565 /9.6.15.1. 03 ..30..9 .65 . 5..3 0.6.16/367.9 ..3 0.653356..65 6.08.: 56... 67.:0.945./3.00:1:9.3: &%  .914..:4.451.33 .63.3069.6.9.46: 65 793    6 . .:: ..4:.9.3 .00:169765:6546.56..7730.: 10:65 ...5.:1:4::17910.6:71.51.9.6945.607.08.5115..79643.0.: .. .3...01.1. 0945.6.../313.50:/651.3.00:1: 9. 13.9.765.:65.979. 50 ..00:1.96 69. 0945...0945.. .0945. ..50: 703..1.33506473.656.3/. 1:4::..5.3.5:.556.3 /.5.9. 56.73.0.::  7.1  . . 493 506473.769.. ...3..:.515.656.9::. .05.1 653 . 9065:191 ..00:1516/367.065.65  .0.3 0..656.0.36. ..3:6 61   5 0:: 6 .:165.00:19:3.659/69. .165..3: ... :.49369191. 0945.14.00:169765:6546.3 069.65..910.01  50 16/367....7733.:56.79:151..945.145..691961:4::..25:793  "919 47. .9.5:. 065:.0:65. :8:.3 6/367. .: 6 .51 .9.....:6.55.3069.65: .5. .9..9.3 1 95191 ..6:71. . .331.659 56..441.7765.46169.65903.03. 61   &50 .../3 563.9 . 0..659 065.9:.3 796:0.91 .5:..5/.9   51 ($:4/5*    ..69.:  /. ..1 69.. 56.65 5031: .1:4::.: 56.: 56..796:79.9 9..3 . . /5 .651.4:.3.6565793  ./5796365159..9 .91 4:.9691: 7.9.: 9:10.:.9636.677.0945.1 . 93 .9.256 .0..5197.60904:.65.. 0945.. .65 6 .91   315.: .79643.67...796015: .. 79643..0 0.939476:1657.10:655:00.79::065:5.9. 9.:79::065:5.: ...11   : .15:   4:. 79:.6:71.   ' 69.:. /563..31.:1 .036:9.9.3. 7...06503:659.3      ./3:6.5:9:    '9/5566779::13..  ': .69.:. 13.3:  ./313..9.3069.: 5.1.945.3 1 ..95:.9.3 .503315. :...60904:...  ':0131.156..6::.5.9 1 1.95: 69 90.3.5:99169.:65:69.76545.9..7.976:.:56 9. 0.:5:.31...7733.:6. .9.6.796015::.3.3::6.3.9..:65.367.3069.677.56...6:7769. 6 77.964.9..3691961:4::.6:71.:65.09656366.51 : :00::69 .56:.5.:69.:.9::.5.5..7765.065.3:.79:151..659:0.:5659..9.796015: .1065.76545.3 069.1.9.069.3.516.9. .95:9.319651: 50  ./55951 9.:1. 69.: 56.: .3 6919 ...1 .9.6.::00:: 5. . 441.515676:.: .::.1 .659  0 6919 .51 95:.069.:: .6:71.7733.3..6.50:/651 .9..1  569 .      :6/:91/9:76515.. .77.9.659:9..5:.9636.7..1 56..00:1...01/.51    '...9.:79646..9..9.:.95...9..9.  06509. .65 69 5 .91.:.08.5/0.:.551543.5:..51.3 9.4441.659..: 07.3/0..6./50065:195.945.659.51.91 .659  34...4.3  .... . ..659:3. .:1659.6.. 6 7.00:1 .. 3.5:.1.3 .:.65.3.. 3.1:4::.65.515:6. 69 535.08..9...91    6.7733..:65.3 069.3 0...5 4..4.: 56..08..46915. 069.9 47915.6...:. 065:5.1650/573.16/367.01567..5:.:3.5156...: .9..3069.:1:4::1696..5 .911156..  &50033.6. .559 ..:: . ..73..:.51.9..::.7.4:.659: 9.:: . 79:: 065:5..6...9.../316/.9...01  5..1.556. 56. 9:.7.9.08.:.../3695145.914:.9.6935#673 '.069.4. 6 77.6.:..:56.0 :.3 .1. .   50 47313 .3 069.:065167..914:...../3 16/..:70.55/96253560.511. :/4. 0945.9155145.9..: 56.616/367.08.5.3: 9 .6:71.39..556./3.91..::.3069. . 5.6..:  1:7. 515: 6 ..1 / : 1:0.3691961:4::.0.79::065:5.69.1690650... 4 5 16/3 67.151..:..9.7.1.0.: ..00:1.3361 95:. 69 . 69.91 6916/3 67.: 9065:191 :. . 67.:: 69 .3: &%  .1 ..1156.. 63365 345.:.: 7....956. 0.79../316/.6945.: 4:..945..:65 79.9/1. 9065:19.:3. : 6 1.:56.51  . .:.9: . .365: '::16/367..: 5..677. .6 9 .6.3:6 7.91:4:73. ..:   #9345.9155145..55 .96516.1 5 .3. .6 :.:8... / :.9::1 69 ..1 6919  16/3 67...91.6:71 .1 465 9.9:76515.9 16/3 67.7910..:0.. /5 ..562.659. 67.15#673  :..: !6   ..31.7.45.91 .0.35.:5.3.6.    ./3:1   .5.9 . ..3.1:...91 5 .6:71.9    51 ($:4/5*    : .9:.:: .:065.79:65:0.:56.5:150945.565:.47..9:76515.3 ... .. .:  31#.3.03..69.9.: 69 9023:: 479150/...165.9.3:.91 4:.3069....99.345.9.0.7733...: .9651 43..631/.94:.65 6 :./316/.93 7.6796:0...3 069.:./5 7965/6519. 6 77.3 5 ::7065  /.:56.9.:69.51 1...91   #9.677..  '.360..: .1 .9.7..:65..659:065:3 504.&.3 069.65 6.::69:. 5 9969 5 .:56.02 6 5350 69 9023:: 479150 519 .08.:65.4.562./3. .01 : .96 69. 16/367.9./3695145.67.659   '.39.9:76515.:0651  .6 1.....9:.3 ':0... 6919 .:65./ 69.0..6:71 .5691961:4::.6336:.  ..9   ' 145.:./3.3:9 7910.3:/ 79.659675:.945.. .:70. 969.99.1.9:.: 7.: 515 .511.01 4516/367..91... 569 535. 9023::347915.0.515860..90.51 . .156.9. 69.9796:0.30.3:/..3 069.. 69.9... ./:.::65 .6.915..03.165..69.35.3::63156.3: .9455 : 03 3.51 .00:1.3/.08..465: .9.6.7730...5:9::1  ...503..:1:4::.465:.:65.:9. . 653 73.5:..:76546.:!6  #.08. /.6590.1/..:. .. .00:169. /. /563.91/.659:03.3.4.3 69 0650.677.5..  .1 69.6 ..6....10:656.:  .556.9 / .951.065:. .1600.945  . 6 . 6 .../3   69 .01 7969.00:1  .3 !6.08.65. 0..5.. 35 6 945.9.3...9:76515.651..6567.03.99..:65. 65 .945.677. /3 16/.5/573.08..67.461.00:1 : 56../ 5:.797..659:03.036..:145..03. 69.7.9:..:/547.:0..0.91.361 9.:/5651.%3:669. :. 9065: ./3:1 :56.:65...1156.559/313.65965.6:7.696. 195.:1 65 9651: 6. / /.964033..7733.130.5.3/.556. % !6 ..3..3 54615.. .069. .50036::.77..9 . 033.65691.:.65 065.50:.0.6.65: '90691:03. 06473. .3 !6911. .65.6.00:1 5.3  . 9651 .9.::.145..:.   "9 3. ..93 :6.79:656.365:15.6..6/56.:1659.04.5:.9...:06510945.26 #.56:06510945./3.0.:06510945.15%36./3.. .969 ...693 :.1 4:.473.: 65 .506150653  '::.6:6..00:1.465: "/6:3 ./3.91:506990./7961/7976519.0.65 .6 251: 6 . 0. : .5 04.6.:56.47.6964::650.65065.369 069.51 6   ': : ... 130.30..0..4.473./369:06964::65 '9 /556130..5.5 ..: 5.51. 69 64::65 06473.5 .35 6..08.. ':06515:.516 ':5:.6..::69   .01516/367.1669.8:./5:.365 .. 6 .365:./3. ...3..03.6033.3.159.00:1   9:.08.:56 :065167. .510.1..08.6919.46.0.65 .:.69 6.65969. 03 3. .4.:6569.01.0.9 1 .. ::5.6 1..93  195.6.7731 .8:.:56.99.9967..9455.0.4...76::::. 75.5.0./3   50  56....5.3.03.9:1:0::15 .9..4 . &%  .: #.3 ::5.:.945 .51..9 1.: 7961 /651 9..51.51336.0./3:1.6530..3 ':3.045.6.9. 93: 6 150 .65511.6964::65 '9.956.:.:.519    ...6.65 .:.515 95 7..0.145...519  31('*69. 769 6 ....: 75:45.65 6 ..656.3 796015 0.9: .9. 476:..:.3../3: ..6 15 ..3.9 ..379015.1 5 150 5 . 796:0.   .3696.9.:6.94: ... ..7730..:65.08.1..10.5   51 ($:4/5*   '%.65:../336 .350904:.3.:989  :63. 65: .03.945.7.6:1:0::15./3 16/.6.:.. 5#673 999 151. 0699:76515 75:45..3..: /50644.51563. 109 16: : ...3: &% .6567./336..10..195./336.65:.30.. 65: 6 33.5 511.5 ..4..030..65 6 ..3:3.95 ... .51.345.6964::65 3. 967 6 511.0945.3 76::::65 6 9. / 9.65 6 09.3..6.3 #  !6 16:56..9451  5 .6.519 .5.3:6/651 9../336.3 . .5110.69...4..519 .6/6.:/5:.0..350904:..3 !695. .677. 75:45..33.65:  '9 : 56 5096.3:  ..0.0530.50:.51563. 75:45.:2.:93:..95.51 7961 69 .026069.655. 069.3:6944/9:6....9:43.5.: 69 .51.65 .9 ..9 .6 :..5476:. :700. 03 ./316/.#6:.5:. .790311 964366255../336.9./476:1 :705. .519:#  .3:51... 75. ...:...7.4.0.9.8.0.50:.65..:65.3: 69 .6.51 :0651.:9:.6..375. :.:. 65 .631..6 77.3.:...345.8..556..659:.1.6.16:56..659:5350699023::479150  . 5.0945.6.6/476:1 :..65:563.7976:: ..46:.02610.9.5 69.519..9. :69.9..576:.: /5103.. .9:.0 !6.0..4511#   ..65.3.09..  31   76:.03.65 .75.0. 6 75...556.33..:%  !6 .93.:.51..0.51796169.63.6.0.96. 75. ..9 #5.:.91/.656%  0. %   0.650:56.5796793..5   5.33.../3:75.5.65:69.9.3351.0.7730.9: .90796/.:6.:56..:  %   : 56.   65079:09/: 93:6796019/0069..6./5.5.3..9569:065.3   #.77.95. /.77353.:.33  796/.33.659:.5:9:10.65. ....&.:9.:...6:.69.50(69. .63.9796019.9. 15094: .516.:/:.:*     .796019.9..:6.6: ..576:.6 3.6.34.91:..3.0.:4616.. .75./0.  59..5/.: 9.33.975:45.33.5.0.:.145:.:3..350.51:. 75..96:70.99.33251:0.963..:5065:.515..3:..659:10.9 .  :. ':. /..::.: .65..:/5931 . .5 9.77.: . 064 .9.5     ( ..1/590.6 1.6%   :506990.9.9:6796019 . 75.36.63.796/. 0.941. "5..: /5 0655053 69064  #..6 ... 769 6 9 .3 ..45.. :.5:..16:56.0.6 ..5. .5..33.96.4:065:195..... 79:47.0.:9.93:615069.3.30.0891 519%  .969  16: 56.65: 7515 .503115.0:65   ' 0.39.7730.1 / 3.5*   "&&                  .9.5 ..9 7.3   %  !6    .. .&79469.946.  %    ..1/.3 ...77.   ' 461 6 796019 79611 69 5 .51/5493. 4.77.1156.5: 653.3.5.3.65.9. .39. ..3. 6. :.9 .46 ...51 59:631 .65 6 556050 .... ..00:1: 9.5. / 4.65  6969 .10.7.451.3:56./3 ..91.9.69:..   .6 98:..: 79:91 ... ..5. .51  .6.:/513.945  ...536  .  .699.5. &794 69..1/.4616. 16: 56.65:63.76:..9  . .: 4..0.   .3:56.069.0.1 .96 .: .2.9. .6.4065..69 5 5. &09.64.: %  79.5 / 93. 796/.6 3.6.5  76:.:.77.0..
Copyright © 2024 DOKUMEN.SITE Inc.